1028

CLP-1 Differential Calculus

  • Upload
    others

  • View
    32

  • Download
    1

Embed Size (px)

Citation preview

Page 1: CLP-1 Differential Calculus
Page 2: CLP-1 Differential Calculus

CLP-1 Differential Calculus

i

Page 3: CLP-1 Differential Calculus

ii

Page 4: CLP-1 Differential Calculus

CLP-1 Differential Calculus

Joel FeldmanUniversity of British Columbia

Andrew RechnitzerUniversity of British Columbia

Elyse YeagerUniversity of British Columbia

September 2, 2021

iii

Page 5: CLP-1 Differential Calculus

Cover Design: Nick Loewen — licensed under the CC-BY-NC-SA 4.0 License.Source files: A link to the source files for this document can be found at the CLPtextbook website. The sources are licensed under the CC-BY-NC-SA 4.0 License.

Edition: CLP-1 Differential Calculus: May 2021

Website: CLP-1

© 2015 – 2021 Joel Feldman, Andrew Rechnitzer, Elyse Yeager

This work is licensed under the Creative Commons Attribution-NonCommercial-ShareAlike 4.0 International License. You can view a copy of the license here.

iv

Page 6: CLP-1 Differential Calculus

PREFACE

This text is a merger of the CLP Differential Calculus textbook and problembook. Itis, at the time that we write this, still a work in progress; some bits and pieces aroundthe edges still need polish. Consequently we recommend to the student that they stillconsult text webpage for links to the errata — especially if they think there might be atypo or error. We also request that you send us an email at [email protected]

Additionally, if you are not a student at UBC and using these texts please sendus an email — we’d love to hear from you.

Joel Feldman, Andrew Rechnitzer and Elyse Yeager

v

Page 7: CLP-1 Differential Calculus

To our students.

And to the many generations of scholars who have freely sharedall this knowledge with us.

vi

Page 8: CLP-1 Differential Calculus

vii

Page 9: CLP-1 Differential Calculus

ACKNOWLEDGEMENTS

Elyse would like to thank her husband Seçkin Demirbas for his endless patience,tireless support, and insightful feedback.

Andrew would like to thank Zoe for her patience and support while he hackedand also Locke for his insightful subediting. He should also thank the various coau-thors for not being too impatient while other manuscripts were neglected.

Joel would like to thank way too many people to name for their support andencouragement over many years.

Thanks to

• Rob Beezer and David Farmer for their help converting this book from LATEX tothis online PreTeXt format.

• Nick Loewen for designing the cover art, help with figures, colours, spellingand many discussions.

• The many people who have collaborated over the last couple of decades mak-ing exams and tests for first year calculus courses at UBC Mathematics. A greatmany of the exercises in the text come from questions in those tests and exams.

Finally, we’d like to thank those students who reported typos and errors theyfound in the text. Many of these students did so through our “bug bounty” programwhich was supported by the Department of Mathematics, Skylight and the LoafeCafe all at UBC.

viii

Page 10: CLP-1 Differential Calculus

USING THE EXERCISES IN THIS BOOK

Each problem in this book is split into four parts: Question, Hint, Answer, and So-lution. As you are working problems, resist the temptation to prematurely peek atthe hint or to click through to the answers and solutions in the appendix! It’s im-portant to allow yourself to struggle for a time with the material. Even professionalmathematicians don’t always know right away how to solve a problem. The art is ingathering your thoughts and figuring out a strategy to use what you know to findout what you don’t.

If you find yourself at a real impasse, go ahead and look at the linked hint. Thinkabout it for a while, and don’t be afraid to read back in the notes to look for a keyidea that will help you proceed. If you still can’t solve the problem, well, we in-cluded the Solutions section for a reason! As you’re reading the solutions, try hardto understand why we took the steps we did, instead of memorizing step-by-stephow to solve that one particular problem.

If you struggled with a question quite a lot, it’s probably a good idea to returnto it in a few days. That might have been enough time for you to internalize thenecessary ideas, and you might find it easily conquerable. Pat yourself on the back— sometimes math makes you feel good! If you’re still having troubles, read overthe solution again, with an emphasis on understanding why each step makes sense.

One of the reasons so many students are required to study calculus is the hopethat it will improve their problem-solving skills. In this class, you will learn lots ofconcepts, and be asked to apply them in a variety of situations. Often, this will in-volve answering one really big problem by breaking it up into manageable chunks,solving those chunks, then putting the pieces back together. When you see a partic-ularly long question, remain calm and look for a way to break it into pieces you canhandle.

• Working with Friends:

Study buddies are fantastic! If you don’t already have friends in your class,you can ask your neighbours in lecture to form a group. Often, a questionthat you might bang your head against for an hour can be easily cleared upby a friend who sees what you’ve missed. Regular study times make sure you

ix

Page 11: CLP-1 Differential Calculus

-1.0

don’t procrastinate too much, and friends help you maintain a positive attitudewhen you might otherwise succumb to frustration. Struggle in mathematics isdesirable, but suffering is not.

When working in a group, make sure you try out problems on your own be-fore coming together to discuss with others. Learning is a process, and gettinganswers to questions that you haven’t considered on your own can rob you ofthe practice you need to master skills and concepts, and the tenacity you needto develop to become a competent problem-solver.

• Types of Questions:

Questions outlined by a blue box make up the “RQS” representative questionset. This set of questions is intended to cover the most essential ideas in eachsection. These questions are usually highly typical of what you’d see on anexam, although some of them are atypical but carry an important moral. Ifyou find yourself unconfident with the idea behind one of these, it’s probablya good idea to practice similar questions.

This representative question set is our suggestion for a minimal selection ofquestions to work on. You are highly encouraged to work on more.

In addition to original problems, this book contains problems pulled fromquizzes and exams given at UBC for Math 100 and 180 (first-semester calculus)and Math 120 (honours first-semester calculus). These problems are markedby “(*)”. The authors would like to acknowledge the contributions of the manypeople who collaborated to produce these exams over the years.

Finally, the questions are organized into three types: Stage 1, Stage 2 andStage 3.

Exercises — Stage 1The first category is meant to test and improve your understanding ofbasic underlying concepts. These often do not involve much calculation.They range in difficulty from very basic reviews of definitions to ques-tions that require you to be thoughtful about the concepts covered in thesection.

Exercises — Stage 2Questions in this category are for practicing skills. It’s not enough to un-derstand the philosophical grounding of an idea: you have to be able toapply it in appropriate situations. This takes practice!

Exercises — Stage 3The last questions in each section go a little farther than “Exercises —Stage 2”. Often they will combine more than one idea, incorporate reviewmaterial, or ask you to apply your understanding of a concept to a newsituation.

In exams, as in life, you will encounter questions of varying difficulty. A goodskill to practice is recognizing the level of difficulty a problem poses. Exams

x

Page 12: CLP-1 Differential Calculus

-1.0

will have some easy questions, some standard questions, and some harderquestions.

xi

Page 13: CLP-1 Differential Calculus

FEEDBACK ABOUT THE TEXT

This combined edition of the CLP differential calculus text is still undergoing testingand changes. Because of this we request that if you find a problem or error in thetext then:

1. Please check the errata list that can be found at the text webpage.

2. Is the problem in the online version or the PDF version or both?

3. Note the URL of the online version and the page number in the PDF

4. Send an email to [email protected]. Please be sure to include

• a description of the error

• the URL of the page, if found in the online edition

• and if the problem also exists in the PDF, then the page number in the PDFand the compile date on the front page of PDF.

xii

Page 14: CLP-1 Differential Calculus

CONTENTS

Preface v

Acknowledgements viii

Using the exercises in this book ix

Feedback about the text xii

0 The basics 10.1 Numbers . . . . . . . . . . . . . . . . . . . . . . . . . . . . . . . . . . . . 10.2 Sets . . . . . . . . . . . . . . . . . . . . . . . . . . . . . . . . . . . . . . . 50.3 Other Important Sets . . . . . . . . . . . . . . . . . . . . . . . . . . . . . 80.4 Functions . . . . . . . . . . . . . . . . . . . . . . . . . . . . . . . . . . . . 120.5 Parsing Formulas . . . . . . . . . . . . . . . . . . . . . . . . . . . . . . . 160.6 Inverse Functions . . . . . . . . . . . . . . . . . . . . . . . . . . . . . . . 21

1 Limits 281.1 Drawing Tangents and a First Limit . . . . . . . . . . . . . . . . . . . . 281.2 Another Limit and Computing Velocity . . . . . . . . . . . . . . . . . . 361.3 The Limit of a Function . . . . . . . . . . . . . . . . . . . . . . . . . . . . 411.4 Calculating Limits with Limit Laws . . . . . . . . . . . . . . . . . . . . 541.5 Limits at Infinity . . . . . . . . . . . . . . . . . . . . . . . . . . . . . . . . 761.6 Continuity . . . . . . . . . . . . . . . . . . . . . . . . . . . . . . . . . . . 871.7 (Optional) — Making the Informal a Little More Formal . . . . . . . . . 1041.8 (Optional) — Making Infinite Limits a Little More Formal . . . . . . . . 1101.9 (Optional) — Proving the Arithmetic of Limits . . . . . . . . . . . . . . 112

2 Derivatives 1192.1 Revisiting Tangent Lines . . . . . . . . . . . . . . . . . . . . . . . . . . . 1202.2 Definition of the Derivative . . . . . . . . . . . . . . . . . . . . . . . . . 1272.3 Interpretations of the Derivative . . . . . . . . . . . . . . . . . . . . . . 146

xiii

Page 15: CLP-1 Differential Calculus

CONTENTS

2.4 Arithmetic of Derivatives - a Differentiation Toolbox . . . . . . . . . . . 1532.5 Proofs of the Arithmetic of Derivatives . . . . . . . . . . . . . . . . . . . 1572.6 Using the Arithmetic of Derivatives – Examples . . . . . . . . . . . . . 1602.7 Derivatives of Exponential Functions . . . . . . . . . . . . . . . . . . . . 1722.8 Derivatives of Trigonometric Functions . . . . . . . . . . . . . . . . . . 1822.9 One More Tool – the Chain Rule . . . . . . . . . . . . . . . . . . . . . . . 1942.10 The Natural Logarithm . . . . . . . . . . . . . . . . . . . . . . . . . . . . 2092.11 Implicit Differentiation . . . . . . . . . . . . . . . . . . . . . . . . . . . . 2192.12 Inverse Trigonometric Functions . . . . . . . . . . . . . . . . . . . . . . 2312.13 The Mean Value Theorem . . . . . . . . . . . . . . . . . . . . . . . . . . 2422.14 Higher Order Derivatives . . . . . . . . . . . . . . . . . . . . . . . . . . 2562.15 (Optional) — Is limx→c f ′(x) Equal to f ′(c)? . . . . . . . . . . . . . . . . 263

3 Applications of derivatives 2673.1 Velocity and Acceleration . . . . . . . . . . . . . . . . . . . . . . . . . . 2683.2 Related Rates . . . . . . . . . . . . . . . . . . . . . . . . . . . . . . . . . 2763.3 Exponential Growth and Decay . . . . . . . . . . . . . . . . . . . . . . . 2963.4 Taylor Polynomials . . . . . . . . . . . . . . . . . . . . . . . . . . . . . . 3173.5 Optimisation . . . . . . . . . . . . . . . . . . . . . . . . . . . . . . . . . . 3713.6 Sketching Graphs . . . . . . . . . . . . . . . . . . . . . . . . . . . . . . . 4113.7 L’Hôpital’s Rule and Indeterminate Forms . . . . . . . . . . . . . . . . 453

4 Towards Integral Calculus 4794.1 Introduction to Antiderivatives . . . . . . . . . . . . . . . . . . . . . . . 479

A High School Material 491A.1 Similar Triangles . . . . . . . . . . . . . . . . . . . . . . . . . . . . . . . 491A.2 Pythagoras . . . . . . . . . . . . . . . . . . . . . . . . . . . . . . . . . . . 492A.3 Trigonometry — Definitions . . . . . . . . . . . . . . . . . . . . . . . . . 492A.4 Radians, Arcs and Sectors . . . . . . . . . . . . . . . . . . . . . . . . . . 493A.5 Trigonometry — Graphs . . . . . . . . . . . . . . . . . . . . . . . . . . . 493A.6 Trigonometry — Special Triangles . . . . . . . . . . . . . . . . . . . . . 494A.7 Trigonometry — Simple Identities . . . . . . . . . . . . . . . . . . . . . 494A.8 Trigonometry — Add and Subtract Angles . . . . . . . . . . . . . . . . 495A.9 Inverse Trig Functions . . . . . . . . . . . . . . . . . . . . . . . . . . . . 495A.10 Areas . . . . . . . . . . . . . . . . . . . . . . . . . . . . . . . . . . . . . . 496A.11 Volumes . . . . . . . . . . . . . . . . . . . . . . . . . . . . . . . . . . . . 497A.12 Powers . . . . . . . . . . . . . . . . . . . . . . . . . . . . . . . . . . . . . 498A.13 Logarithms . . . . . . . . . . . . . . . . . . . . . . . . . . . . . . . . . . . 498A.14 You Should be Able to Derive . . . . . . . . . . . . . . . . . . . . . . . . 499

B Origin of Trig, Area and Volume Formulas 501B.1 Theorems about Triangles . . . . . . . . . . . . . . . . . . . . . . . . . . 501B.2 Trigonometry . . . . . . . . . . . . . . . . . . . . . . . . . . . . . . . . . 503B.3 Inverse Trigonometric Functions . . . . . . . . . . . . . . . . . . . . . . 512

xiv

Page 16: CLP-1 Differential Calculus

CONTENTS

B.4 Cosine and Sine Laws . . . . . . . . . . . . . . . . . . . . . . . . . . . . 514B.5 Circles, cones and spheres . . . . . . . . . . . . . . . . . . . . . . . . . . 516

C Root Finding 526C.1 Newton’s Method . . . . . . . . . . . . . . . . . . . . . . . . . . . . . . . 528C.2 The Error Behaviour of Newton’s Method . . . . . . . . . . . . . . . . . 535C.3 The false position (regula falsi) method . . . . . . . . . . . . . . . . . . 538C.4 The secant method . . . . . . . . . . . . . . . . . . . . . . . . . . . . . . 539C.5 The Error Behaviour of the Secant Method . . . . . . . . . . . . . . . . . 542

D Hints for Exercises 545

E Answers to Exercises 591

F Solutions to Exercises 672

xv

Page 17: CLP-1 Differential Calculus

THE BASICS

Chapter 0

We won’t make this section of the text too long — all we really want to do here isto take a short memory-jogging excursion through little bits and pieces you shouldremember about sets and numbers. The material in this chapter will not be (directly)examined.

0.1q Numbers

Before we do anything else, it is very important that we agree on the definitions andnames of some important collections of numbers.

• Natural numbers — These are the “whole numbers” 1,2,3,. . . that we learn firstat about the same time as we learn the alphabet. We will denote this collectionof numbers by the symbol “N”. The symbol N is written in a type of bold-face font that we call “black-board bold” (and is definitely not the same symbolas N ). You should become used to writing a few letters in this way since it istypically used to denote collections of important numbers. Unfortunately thereis often some confusion as to whether or not zero should be included 1. In thistext the natural numbers does not include zero.

Notice that the set of natural numbers is closed under addition and multiplica-tion. This means that if you take any two natural numbers and add them youget another natural number. Similarly if you take any two natural numbersand multiply them you get another natural number. However the set is not

1 This lack of agreement comes from some debate over how “natural” zero is — “how can nothingbe something?” It was certainly not used by the ancient Greeks who really first looked at proofand number. If you are a mathematician then generally 0 is not a natural number. If you are acomputer scientist then 0 generally is.

1

Page 18: CLP-1 Differential Calculus

THE BASICS 0.1 NUMBERS

closed under subtraction or division; we need negative numbers and fractionsto make collections of numbers closed under subtraction and division.

Two important subsets of natural numbers are:

Prime numbers — a natural number is prime when the only natural num-bers that divide it exactly are 1 and itself. Equivalently it cannot be writtenas the product of two natural numbers neither of which are 1. Note that 1is not a prime number 2.

Composite numbers — a natural number is a composite number when itis not prime.

Hence the number 7 is prime, but 6 = 3× 2 is composite.

• Integers — all positive and negative numbers together with the number zero.We denote the collection of all integers by the symbol “Z”. Again, note that thisis not the same symbol as “Z”, and we must write it in the same black-boardbold font. The Z stands for the German Zahlen meaning numbers 3. Note thatZ is closed under addition, subtraction and multiplication, but not division.

Two important subsets of integers are:

Even numbers — an integer is even if it is exactly divisible by 2, or equiv-alently if it can be written as the product of 2 and another integer. Thismeans that −14, 6 and 0 are all even.

Odd numbers — an integer is odd when it is not even. Equivalently it canbe written as 2k + 1 where k is another integer. Thus 11 = 2 × 5 + 1 and−7 = 2× (−4) + 1 are both odd.

• Rational numbers — this is all numbers that can be written as the ratio of twointegers. That is, any rational number r can be written as p/q where p, q areintegers. We denote this collection by Q standing for quoziente which is Italianfor quotient or ratio. Now we finally have a set of numbers which is closedunder addition, subtraction, multiplication and division (of course you stillneed to be careful not to divide by zero).

• Real numbers — generally we think of these numbers as numbers that can bewritten as decimal expansions and we denote it by R. It is beyond the scope ofthis text to go into the details of how to give a precise definition of real num-bers, and the notion that a real number can be written as a decimal expansionwill be sufficient.

It took mathematicians quite a long time to realise that there were numbers

2 If you let 1 be a prime number then you have to treat 1×2×3 and 2×3 as different factorisationsof the number 6. This causes headaches for mathematicians, so they don’t let 1 be prime.

3 Some schools (and even some provinces!!) may use “I” for integers, but this is extremely non-standard and they really should use correct notation.

2

Page 19: CLP-1 Differential Calculus

THE BASICS 0.1 NUMBERS

that could not be written as ratios of integers 4. The first numbers that wereshown to be not-rational are square-roots of prime numbers, like

√2. Other

well known examples are π and e. Usually the fact that some numbers cannotbe represented as ratios of integers is harmless because those numbers can beapproximated by rational numbers to any desired precision.

The reason that we can approximate real numbers in this way is the surprisingfact that between any two real numbers, one can always find a rational number.So if we are interested in a particular real number we can always find a rationalnumber that is extremely close. Mathematicians refer to this property by sayingthat Q is dense in R.

So to summariseDefinition 0.1.1 Sets of numbers.

This is not really a definition, but you should know these symbols

• N = the natural numbers,

• Z = the integers,

• Q = the rationals, and

• R = the reals.

0.1.0.1tttMore on Real Numbers

In the preceding paragraphs we have talked about the decimal expansions of realnumbers and there is just one more point that we wish to touch on. The decimalexpansions of rational numbers are always periodic, that is the expansion eventuallystarts to repeat itself. For example

2

15= 0.133333333 . . .

5

17= 0.294117647058823529411764705882352941176470588235294117647058823 . . .

4 The existence of such numbers caused mathematicians (particularly the ancient Greeks) all sortsof philosophical problems. They thought that the natural numbers were somehow fundamentaland beautiful and “natural”. The rational numbers you can get very easily by taking “ratios” — aprocess that is still somehow quite sensible. There were quite influential philosophers (in Greeceat least) called Pythagoreans (disciples of Pythagoras originally) who saw numbers as almostmystical objects explaining all the phenomena in the universe, including beauty — famouslythey found fractions in musical notes etc and “numbers constitute the entire heavens”. Theybelieved that everything could be explained by whole numbers and their ratios. But soon afterPythagoras’ theorem was discovered, so were numbers that are not rational. The first proof of theexistence of irrational numbers is sometimes attributed to Hippasus in around 400BCE (not reallyknown). It seems that his philosopher “friends” were not very happy about this and essentiallyexiled him. Some accounts suggest that he was drowned by them.

3

Page 20: CLP-1 Differential Calculus

THE BASICS 0.1 NUMBERS

where we have underlined some of the last example to make the period clearer. Onthe other hand, irrational numbers, such as

√2 and π, have expansions that never

repeat.If we want to think of real numbers as their decimal expansions, then we need

those expansions to be unique. That is, we don’t want to be able to write down twodifferent expansions, each giving the same real number. Unfortunately there are aninfinite set of numbers that do not have unique expansions. Consider the number 1.We usually just write “1”, but as a decimal expansion it is

1.00000000000 . . .

that is, a single 1 followed by an infinite string of 0’s. Now consider the followingnumber

0.99999999999 . . .

This second decimal expansions actually represents the same number — the number1. Let’s prove this. First call the real number this represents q, then

q = 0.99999999999 . . .

Let’s use a little trick to get rid of the long string of trailing 9’s. Consider 10q:

q = 0.99999999999 . . .

10q = 9.99999999999 . . .

If we now subtract one from the other we get

9q = 9.0000000000 . . .

and so we are left with q = 1.0000000 . . .. So both expansions represent the same realnumber.

Thankfully this sort of thing only happens with rational numbers of a particularform — those whose denominators are products of 2s and 5s. For example

3

25= 1.200000 · · · = 1.19999999 . . .

− 7

32= −0.2187500000 · · · = −0.2187499999 . . .

9

20= 0.45000000 · · · = 0.4499999 . . .

We can formalise this result in the following theorem (which we haven’t proved ingeneral, but it’s beyond the scope of the text to do so):

4

Page 21: CLP-1 Differential Calculus

THE BASICS 0.2 SETS

Theorem 0.1.2

Let x be a real number. Then x must fall into one of the following two cate-gories,

• x has a unique decimal expansion, or

• x is a rational number of the form a2k5`

where a ∈ Z and k, l are non-negative integers.

In the second case, x has exactly two expansions, one that ends in an infinitestring of 9’s and the other ending in an infinite string of 0’s.

When we do have a choice of two expansions, it is usual to avoid the one thatends in an infinite string of 9’s and write the other instead (omitting the infinitetrailing string of 0’s).

0.2q Sets

All of you will have done some basic bits of set-theory in school. Sets, intersection,unions, Venn diagrams etc etc. Set theory now appears so thoroughly throughoutmathematics that it is difficult to imagine how Mathematics could have existed with-out it. It is really quite surprising that set theory is a much newer part of mathematicsthan calculus. Mathematically rigorous set theory was really only developed in the19th Century — primarily by Georg Cantor 1. Mathematicians were using sets beforethen (of course), however they were doing so without defining things too rigorouslyand formally.

In mathematics (and elsewhere, including “real life”) we are used to dealing withcollections of things. For example

• a family is a collection of relatives.

• hockey team is a collection of hockey players.

• shopping list is a collection of items we need to buy.

Generally when we give mathematical definitions we try to make them very for-mal and rigorous so that they are as clear as possible. We need to do this so that

1 An extremely interesting mathematician who is responsible for much of our understanding ofinfinity. Arguably his most famous results are that there are more real numbers than integers,and that there are an infinite number of different infinities. His work, though now consideredto be extremely important, was not accepted by his peers, and he was labelled “a corrupter ofyouth” for teaching it. For some reason we know that he spent much of his honeymoon talkingand doing mathematics with Richard Dedekind.

5

Page 22: CLP-1 Differential Calculus

THE BASICS 0.2 SETS

when we come across a mathematical object we can decide with complete certaintywhether or not it satisfies the definition.

Unfortunately, it is the case that giving a completely rigorous definition of “set”would take up far more of our time than we would really like 2.

Definition 0.2.1 A not-so-formal definition of set.

A “set” is a collection of distinct objects. The objects are referred to as “ele-ments” or “members” of the set.

Now — just a moment to describe some conventions. There are many of these inmathematics. These are not firm mathematical rules, but just traditions. It makes itmuch easier for people reading your work to understand what you are trying to say.

• Use capital letters to denote sets, A,B,C,X, Y etc.

• Use lower case letters to denote elements of the sets a, b, c, x, y.

So when you are writing up homework, or just describing what you are doing, thenif you stick with these conventions people reading your work (including the personmarking your exams) will know — “Oh A is that set they are talking about” and “ais an element of that set.”. On the other hand, if you use any old letter or symbol it iscorrect, but confusing for the reader. Think of it as being a bit like spelling — if youdon’t spell words correctly people can usually still understand what you mean, butit is much easier if you spell words the same way as everyone else.

We will encounter more of these conventions as we go — another good one is

• The letters i, j, k, l,m, n usually denote integers (like 1, 2, 3,−5, 18, · · · >).

• The letters x, y, z, w usually denote real numbers (like 1.4323, π,√

2, 6.0221415×1023, . . . and so forth).

So now that we have defined sets, what can we do with them? There is only thingwe can ask of a set

“Is this object in the set?”

and the set will answer

“yes” or “no”

For example, if A is the set of even numbers we can ask “Is 4 in A?” We get back theanswer “yes”. We write this as

4 ∈ A

2 The interested reader is invited to google (or whichever search engine you prefer — Duck-DuckGo?) “Russell’s paradox”, “Axiomatic set theory” and “Zermelo-Fraenkel set theory” for amore complete and far more detailed discussion of the basics of sets and why, when you dig intothem a little, they are not so basic.

6

Page 23: CLP-1 Differential Calculus

THE BASICS 0.2 SETS

While if we ask “Is 3 inA?”, we get back the answer “no”. Mathematically we wouldwrite this as

3 /∈ A

So this symbol “∈” is mathematical shorthand for “is an element of”, while the samesymbol with a stroke through it “/∈” is shorthand for “is not an element of”.

Notice that both of these statements, though they are written down as shortstrings of three symbols, are really complete sentences. That is, when we read themout we have

“4 ∈ A” is read as “Four is an element of A.”“3 /∈ A” is read as “Three is not an element of A.”

The mathematical symbols like “+”, “=” and “∈” are shorthand 3 and mathematicalstatements like “4 + 3 = 7” are complete sentences.

This is an important point — mathematical writing is just like any other sort ofwriting. It is very easy to put a bunch of symbols or words down on the page, butif we would like it to be easy to read and understand, then we have to work a bitharder. When you write mathematics you should keep in mind that someone elseshould be able to read it and understand it.

Easy reading is damn hard writing.

— Nathaniel Hawthorne, but possibly also a few others like Richard Sheri-dan.

We will come across quite a few different sets when doing mathematics. It must becompletely clear from the definition how to answer the question “Is this object in theset or not?”

• “Let A be the set of even integers between 1 and 13.” — nice and clear.

• “Let B be the set of tall people in this class room.” — not clear.

More generally if there are only a small number of elements in the set we just listthem all out

• “Let C = 1, 2, 3.”

When we write out the list we put the elements inside braces “·”. Note that theorder we write things in doesn’t matter

C = 1, 2, 3 = 2, 1, 3 = 3, 2, 1

3 Precise definitions aside, by “shorthand” we mean a collection of accepted symbols and abbre-viations to allow us to write more quickly and hopefully more clearly. People have been usingvarious systems of shorthand as long as people have been writing. Many of these are used andunderstood only by the individual, but if you want people to be able to understand what youhave written, then you need to use shorthand that is commonly understood.

7

Page 24: CLP-1 Differential Calculus

THE BASICS 0.3 OTHER IMPORTANT SETS

because the only thing we can ask is “Is this object an element of C?” We cannotask more complex questions like “What is the third element of C?” — we requiremore sophisticated mathematical objects to ask such questions 4. Similarly, it doesn’tmatter how many times we write the same object in the list

C = 1, 1, 1, 2, 3, 3, 3, 3, 1, 2, 1, 2, 1, 3 = 1, 2, 3

because all we ask is “Is 1 ∈ C?”. Not “how many times is 1 in C?”.Now — if the set is a bit bigger then we might write something like this

• C = 1, 2, 3, . . . , 40 the set of all integers between 1 and 40 (inclusive).

• A = 1, 4, 9, 16, . . . the set of all perfect squares 5

The “. . .” is again shorthand for the missing entries. You have to be careful with thisas you can easily confuse the reader

• B = 3, 5, 7, . . . — is this all odd primes, or all odd numbers bigger than 1 or?? What is written is not sufficient for us to have a firm idea of what the writerintended.

Only use this where it is completely clear by context. A few extra words can save thereader (and yourself) a lot of confusion.

Always think about the reader.

0.3q Other Important Sets

We have seen a few important sets above — namely N,Z,Q and R. However, ar-guably the most important set in mathematics is the empty set.

Definition 0.3.1 Empty set.

The empty set (or null set or void set) is the set which contains no elements. Itis denoted ∅. For any object x, we always have x /∈ ∅; hence ∅ = .

Note that it is important to realise that the empty set is not nothing; think of it asan empty bag. Also note that with quite a bit of hard work you can actually definethe natural numbers in terms of the empty set. Doing so is very formal and wellbeyond the scope of this text.

When a set does not contain too many elements it is fine to specify it by listingout its elements. But for infinite sets or even just big sets we can’t do this and instead

4 The interested reader is invited to look at “lists”, “multisets”, “totally ordered sets” and “partiallyordered sets” amongst many other mathematical objects that generalise the basic idea of sets.

5 i.e. integers that can be written as the square of another integer.

8

Page 25: CLP-1 Differential Calculus

THE BASICS 0.3 OTHER IMPORTANT SETS

we have to give the defining rule. For example the set of all perfect square numberswe write as

S = x s.t. x = k2 where k ∈ ZNotice we have used another piece of shorthand here, namely s.t. , which stands for“such that” or “so that”. We read the above statement as “S is the set of elementsx such that x equals k-squared where k is an integer”. This is the standard way ofwriting a set defined by a rule, though there are several shorthands for “such that”.We shall use two them:

P = p s.t. p is prime = p | p is primeOther people also use “:” is shorthand for “such that”. You should recognise all threeof these shorthands.

Example 0.3.2 examples of sets.

Even more examples. . .

• Let A = 2, 3, 5, 7, 11, 13, 17, 19 and let

B = a ∈ A|a < 8 = 2, 3, 5, 7the set of elements of A that are strictly less than 8.

• Even and odd integers

E = n|n is an even integer= n|n = 2k for some k ∈ Z = 2n|n ∈ Z,

and similarly

O = n|n is an odd integer= 2n+ 1|n ∈ Z.

• Square integers

S = n2|n ∈ Z.The set a S ′ = n2|n ∈ N is not the same as S because S ′ does not contain thenumber 0, which is definitely a square integer and 0 is in S. We could also writeS = n2|n ∈ Z, n ≥ 0 and S = n2|n = 0, 1, 2, . . . .

a Notice here we are using another common piece of mathematical short-hand. Very often in math-ematics we will be talking or writing about some object, like the set S above, and then we willcreate a closely related object. Rather than calling this new object by a new symbol (we couldhave used T or R or. . . ), we instead use the same symbol but with some sort of accent — such asthe little single quote mark we added to the symbol S to make S′ (read “S prime”). The point ofthis is to let the reader know that this new object is related to the original one, but not the same.You might also see S, S, S, S and others.

9

Page 26: CLP-1 Differential Calculus

THE BASICS 0.3 OTHER IMPORTANT SETS

The sets A and B in the above example illustrate an important point. Every ele-ment in B is an element in A, and so we say that B is a subset of A

Definition 0.3.3

Let A and B be sets. We say “A is a subset of B” if every element of A is alsoan element of B. We denote this A ⊆ B (or B ⊇ A). If A is a subset of B and Aand B are not the same , so that there is some element of B that is not in A thenwe say that A is a proper subset of B. We denote this by A ⊂ B (or B ⊃ A).

Two things to note about subsets:

• Let A be a set. It is always the case that ∅ ⊆ A.

• If A is not a subset of B then we write A 6⊆ B. This is the same as saying thatthere is some element of A that is not in B. That is, there is some a ∈ A suchthat a /∈ B.

Example 0.3.4 subsets.

Let S = 1, 2. What are all the subsets of S? Well — each element of S can eitherbe in the subset or not (independent of the other elements of the set). So we have2 × 2 = 4 possibilities: neither 1 nor 2 is in the subset, 1 is but 2 is not, 2 is but 1 isnot, and both 1 and 2 are. That is

∅, 1, 2, 1, 2 ⊆ S

This argument can be generalised with a little work to show that a set that containsexactly n elements has exactly 2n subsets.

In much of our work with functions later in the text we will need to work withsubsets of real numbers, particularly segments of the “real line”. A convenient andstandard way of representing such subsets is with interval notation.

Definition 0.3.5 Open and closed intervals of R.

Let a, b ∈ R such that a < b. We name the subset of all numbers between a andb in different ways depending on whether or not the ends of the interval (a andb) are elements of the subset.

• The closed interval [a, b] = x ∈ R : a ≤ x ≤ b — both end points areincluded.

• The open interval (a, b) = x ∈ R : a < x < b — neither end point isincluded.

We also define half-open a intervals which contain one end point but not theother:

(a, b] = x ∈ R : a < x ≤ b [a, b) = x ∈ R : a ≤ x < b

10

Page 27: CLP-1 Differential Calculus

THE BASICS 0.3 OTHER IMPORTANT SETS

We sometimes also need unbounded intervals

[a,∞) = x ∈ R : a ≤ x (a,∞) = x ∈ R : a < x(−∞, b] = x ∈ R : x ≤ b (−∞, b) = x ∈ R : x < b

These unbounded intervals do not include “±∞”, so that end of the interval isalways open b.

a Also called “half-closed”. The preference for one term over the other may be related towhether a 500ml glass containing 250ml of water is half-full or half-empty.

b Infinity is not a real number. As mentioned in an earlier footnote, Cantor proved thatthere are an infinite number of different infinities and so it is incorrect to think of ∞ asbeing a single number. As such it cannot be an element in an interval of the real line. Wesuggest that the reader that wants to learn more about how mathematics handles infinitylook up transfinite numbers and transfinite arithmetic. Needless to say these topics arebeyond the scope of this text.

0.3.1 ttMore on Sets

So we now know how to say that one set is contained within another. We will nowdefine some other operations on sets. Let us also start to be a bit more precise withour definitions and set them out carefully as we get deeper into the text.

Definition 0.3.6

Let A and B be sets. We define the union of A and B, denoted A ∪B, to be theset of all elements that are in at least one of A or B.

A ∪B = x|x ∈ A or x ∈ B

It is important to realise that we are using the word “or” in a careful mathematicalsense. We mean that x belongs to A or x belongs to B or both. Whereas in normalevery-day English “or” is often used to be “exclusive or” — A or B but not both 1.

We also start the definition by announcing “Definition” so that the reader knows“We are about to define something important”. We should also make sure that ev-erything is (reasonably) self-contained — we are not assuming the reader alreadyknows A and B are sets.

It is vital that we make our definitions clear otherwise anything we do with thedefinitions will be very difficult to follow. As writers we must try to be nice to our

1 When you are asked for your dining preferences on a long flight you are usually asked somethinglike “Chicken or beef?” — you get one or the other, but not both. Unless you are way at the backnear the toilets in which case you will be presented with which ever meal was less popular.Probably fish.

11

Page 28: CLP-1 Differential Calculus

THE BASICS 0.4 FUNCTIONS

readers 2.Definition 0.3.7

Let A and B be sets. We define the intersection of A and B, denoted A ∩ B, tobe the set of elements that belong to both A and B.

A ∩B = x | x ∈ A and x ∈ B

Again note that we are using the word “and” in a careful mathematical sense(which is pretty close to the usual use in English).

Example 0.3.8 Union and intersection.

Let A = 1, 2, 3, 4, B = p : p is prime, C = 5, 7, 9 and D =even positive integers. Then

A ∩B = 2, 3B ∩D = 2A ∪ C = 1, 2, 3, 4, 5, 7, 9A ∩ C = ∅

In this last case we see that the two sets have no elements in common — they aresaid to be disjoint.

0.4q Functions

Now that we have reviewed basic ideas about sets we can start doing more interest-ing things with them — functions.

When we are introduced to functions in mathematics, it is almost always as for-mulas. We take a number x and do some things to it to get a new number y. Forexample,

y = f(x) = 3x− 7

Here, we take a number x, multiply it by 3 and then subtract seven to get the result.This view of functions — a function is a formula — was how mathematicians

defined them up until the 19th century. As basic ideas of sets became better defined,people revised ideas surrounding functions. The more modern definition of a func-tion between two sets is that it is a rule which assigns to each element of the first set

2 If you are finding this text difficult to follow then please complain to us authors and we will doour best to improve it.

12

Page 29: CLP-1 Differential Calculus

THE BASICS 0.4 FUNCTIONS

a unique element of the second set.Consider the set of days of the week, and the set containing the alphabet

A = Sunday, Monday,Tuesday, Wednesday,Thursday, Friday, Saturday, SundayB = a,b,c,d,e, . . . , x,y,z

We can define a function f that takes a day (that is, an element of A) and turns it intothe first letter of that day (that is, an element of B). This is a valid function, thoughthere is no formula. We can draw a picture of the function as

Clearly such pictures will work for small sets, but will get very messy for bigones. When we shift back to talking about functions on real numbers, then we willswitch to using graphs of functions on the Cartesian plane.

This example is pretty simple, but this serves to illustrate some important points.If our function gives us a rule for taking elements in A and turning them into ele-ments from B then

• the function must be defined for all elements of A — that is, no matter whichelement of A we choose, the function must be able to give us an answer. Everyfunction must have this property.

• on the other hand, we don’t have to “hit” every element from B. In the aboveexample, we miss almost all the letters in B. A function that does reach everyelement of B is said to be “surjective” or “onto”.

• a given element of B may be reached by more than one element of A. In theabove example, the days “Tuesday” and “Thursday” both map to the letter Tand similarly the letters S is mapped to by both “Sunday” and “Saturday”. Afunction which does not do this, that is, every element in A maps to a differentelement in B is called “injective” or “one-to-one” — again we will come backto this later when we discuss inverse function in Section 0.6.

Summarising this more formally, we have

Definition 0.4.1

Let A,B be non-empty sets. A function f from A to B, is a rule or formula thattakes elements of A as inputs and returns elements of B as outputs. We write

13

Page 30: CLP-1 Differential Calculus

THE BASICS 0.4 FUNCTIONS

this as

f : A→ B

and if f takes a ∈ A as an input and returns b ∈ B then we write this asf(a) = b. Every function must satisfy the following two conditions

• The function must be defined on every possible input from the set A.That is, no matter which element a ∈ A we choose, the function mustreturn an element b ∈ B so that f(a) = b.

• The function is only allowed to return one result for each input a. So ifwe find that f(a) = b1 and f(a) = b2 then the only way that f can be afunction is if b1 is exactly the same as b2.

a You may have learned this in the context of plotting functions on the Cartesian plane, as“the vertical line test”. If the graph intersects a vertical line twice, then the same x-valuewill give two y-values and so the graph does not represent a function.

We must include the input and output sets A and B in the definition of the func-tion. This is one of the reasons that we should not think of functions as just formulas.The input and output sets have proper mathematical names, which we give below:

Definition 0.4.2

Let f : A→ B be a function. Then

• the set A of inputs to our function is the “domain” of f ,

• the set B which contains all the results is called the codomain,

• We read “f(a) = b” as “f of a is b”, but sometimes we might say “f mapsa to b” or “b is the image of a”.

• The codomain must contain all the possible results of the function, but itmight also contain a few other elements. The subset of B that is exactlythe outputs of A is called the “range” of f . We define it more formally by

range of f = b ∈ B | there is some a ∈ A so that f(a) = b= f(a) ∈ B | a ∈ A

The only elements allowed in that set are those elements of B that are theimages of elements in A.

14

Page 31: CLP-1 Differential Calculus

THE BASICS 0.4 FUNCTIONS

Example 0.4.3 domains and ranges.

Let us go back to the “days of the week” function example that we worked on above,we can define the domain, codomain and range:

• The domain, A, is the set of days of the week.

• The codomain, B, is the 26 letters of the alphabet.

• The range is the set F,M, T, S,W — no other elements of B are images ofinputs from A.

Example 0.4.4 more domains and ranges.

A more numerical example — let g : R → R be defined by the formula g(x) = x2.Then

• the domain and codomain are both the set of all real numbers, but

• the range is the set [0,∞).

Now — let h : [0,∞)→ [0,∞) be defined by the formula h(x) =√x. Then

• the domain and codomain are both the set [0,∞), that is all non-negative realnumbers, and

• in this case the range is equal to the codomain, namely [0,∞).

Example 0.4.5 piece-wise function.

Yet another numerical example.

V : [−1, 1]→ R defined by V (t) =

0 if − 1 ≤ t < 0

120 if 0 ≤ t ≤ 1

This is an example of a “piece-wise” function — that is, one that is not defined by asingle formula, but instead defined piece-by-piece. This function has domain [−1, 1]and its range is 0, 120. We could interpret this function as measuring the voltageacross a switch that is flipped on at time t = 0.

Almost all the functions we look at from here on will be formulas. However itis important to note, that we have to include the domain and codomain when wedescribe the function. If the domain and codomain are not stated explicitly then we

15

Page 32: CLP-1 Differential Calculus

THE BASICS 0.5 PARSING FORMULAS

should assume that both are R.

0.5q Parsing Formulas

Consider the formula

f(x) =1 + x

1 + 2x− x2

This is an example of a simple rational function — that is, the ratio of two polyno-mials. When we start to examine these functions later in the text, it is important thatwe are able to understand how to evaluate such functions at different values of x.For example

f(5) =1 + 5

1 + 10− 25=

6

−14= −3

7

More important, however, is that we understand how we decompose this functioninto simpler pieces. Since much of your calculus course will involve creating andstudying complicated functions by building them up from simple pieces, it is impor-tant that you really understand this point.

Now to get there we will take a small excursion into what are called parse-trees.You already implicitly use these when you evaluate the function at a particular valueof x, but our aim here is to formalise this process a little more.

We can express the steps used to evaluate the above formula as a tree-like dia-gram 1. We can decompose this formula as the following tree-like diagram

Figure 0.5.1: A parse tree of the function 1+x1+2x−x2 .

1 Such trees appear in many areas of mathematics and computer science. The reason for the nameis that they look rather like trees — starting from their base they grow and branch out towardstheir many leaves. For some reason, which remains mysterious, they are usually drawn upsidedown.

16

Page 33: CLP-1 Differential Calculus

THE BASICS 0.5 PARSING FORMULAS

Let us explain the pieces here.

• The picture consists of boxes and arrows which are called “nodes” and “edges”respectively.

• There are two types of boxes, those containing numbers and the variable x, andthose containing arithmetic operations “+”,“−”, “×” and “/”.

• If we wish to represent the formula 3+5, then we can draw this as the followingcherry-like configuration

which tells us to take the numbers “3” and “5” and add them together to get 8.

evaluates to

• By stringing such little “cherries” together we can describe more complicatedformulas. For example, if we compute “(3 + 5)× 2”, we first compute “(3 + 5)”and then multiply the result by 2. The corresponding diagrams are

evaluates to evaluates to

The tree we drew in Figure 0.5.1 above representing our formula has x in someof the boxes, and so when we want to compute the function at a particular value ofx — say at x = 5 — then we replace those “x”s in the tree by that value and thencompute back up the tree. See the example below

17

Page 34: CLP-1 Differential Calculus

THE BASICS 0.5 PARSING FORMULAS

Start 7→

7→ 7→

7→ and we are done.

This is not the only parse tree associated with the formula for f(x); we could alsodecompose it as

We are able to do this because when we compute the denominator 1 + 2x − x2,

18

Page 35: CLP-1 Differential Calculus

THE BASICS 0.5 PARSING FORMULAS

we can compute it as

1 + 2x− x2 = either (1 + 2x)− x2 or = 1 + (2x− x2).

Both 2 are correct because addition is “associative”. Namely

a+ b+ c = (a+ b) + c = a+ (b+ c).

Multiplication is also associative:

a× b× c = (a× b)× c = a× (b× c).

Example 0.5.2 parsing a formula.

Consider the formula

g(t) =

(t+ π

t− π

)· sin

(t+ π

2

).

This introduces a new idea — we have to evaluate t+π2

and then compute the sine ofthat number. The corresponding tree can be written as

If we want to evaluate this at t = π/2 then we get the following. . .

2 We could also use, for example, 1 + 2x− x2 = (1− x2) + 2x.

19

Page 36: CLP-1 Differential Calculus

THE BASICS 0.5 PARSING FORMULAS

Example 0.5.2

Start 7→

7→ 7→

7→ and we are done.

It is highly unlikely that you will ever need to explicitly construct such a tree forany problem in the remainder of the text. The main point of introducing these objectsand working through a few examples is to realise that all the functions that we willexamine are constructed from simpler pieces. In particular we have constructed allthe above examples from simple “building blocks”

• constants — fixed numbers like 1, π and so forth

• variables — usually x or t, but sometimes other symbols

• standard functions — like trigonometric functions (sine, cosine and tangent),exponentials and logarithms.

These simple building blocks are combined using arithmetic

20

Page 37: CLP-1 Differential Calculus

THE BASICS 0.6 INVERSE FUNCTIONS

• addition and subtraction — a+ b and a− b

• multiplication and division — a · b and ab

• raising to a power — an

• composition — given two functions f(x) and g(x) we form a new functionf(g(x)) by evaluating y = g(x) and then evaluating f(y) = f(g(x)).

During the rest of the course when we learn how to compute limits and derivatives,our computations require us to understand the way we construct functions as wehave just described.

That is, in order to compute the derivative 3 of a function we have to see how toconstruct the function from these building blocks (i.e. the constants, variables andstandard functions) using arithmetic operations. We will then construct the deriva-tive by following these same steps. There will be simple rules for finding the deriva-tives of the simpler pieces and then rules for putting them together following thearithmetic used to construct the function.

0.6q Inverse Functions

There is one last thing that we should review before we get into the main materialof the course and that is inverse functions. As we have seen above functions arereally just rules for taking an input (almost always a number), processing it some-how (usually by a formula) and then returning an output (again, almost always anumber).

input number x 7→ f does “stuff” to x 7→ return number y

In many situations it will turn out to be very useful if we can undo whatever it isthat our function has done. ie

take output y 7→ do “stuff” to y 7→ return the original x

When it exists, the function “which undoes” the function f(x) is found by solvingy = f(x) for x as a function of y and is called the inverse function of f . It turns outthat it is not always possible to solve y = f(x) for x as a function of y. Even when itis possible, it can be really hard to do 1.

For example — a particle’s position, s, at time t is given by the formula s(t) = 7t(sketched below). Given a calculator, and any particular number t, you can quicklywork out the corresponding positions s. However, if you are asked the question

3 We get to this in Chapter 2 — don’t worry about exactly what it is just now.1 Indeed much of encryption exploits the fact that you can find functions that are very quick to

do, but very hard to undo. For example — it is very fast to multiply two large prime numberstogether, but very hard to take that result and factor it back into the original two primes. Theinterested reader should look up trapdoor functions.

21

Page 38: CLP-1 Differential Calculus

THE BASICS 0.6 INVERSE FUNCTIONS

“When does the particle reach s = 4?” then to answer it we need to be able to“undo” s(t) = 4 to isolate t. In this case, because s(t) is always increasing, we canalways undo s(t) to get a unique answer:

s(t) = 7t = 4 if and only if t =4

7.

However, this question is not always so easy. Consider the sketch of y = sin(x)below; when is y = 1

2? That is, for which values x is sin(x) = 1

2? To rephrase it again,

at which values of x does the curve y = sinx (which is sketched in the right half ofFigure 0.6.1) cross the horizontal straight line y = 1

2(which is also sketched in the

same figure)?

Figure 0.6.1

We can see that there are going to be an infinite number of x-values that givey = sin(x) = 1

2; there is no unique answer.

Recall (from Definition 0.4.1) that for any given input, a function must give aunique output. So if we want to find a function that undoes s(t), then things are good— because each s-value corresponds to a unique t-value. On the other hand, thesituation with y = sinx is problematic — any given y-value is mapped to by manydifferent x-values. So when we look for an unique answer to the question “When issinx = 1

2?” we cannot answer it.

This “uniqueness” condition can be made more precise:

Definition 0.6.2

A function f is one-to-one (injective) when it never takes the same y valuemore than once. That is

if x1 6= x2 then f(x1) 6= f(x2)

There is an easy way to test this when you have a plot of the function — thehorizontal line test.

22

Page 39: CLP-1 Differential Calculus

THE BASICS 0.6 INVERSE FUNCTIONS

Definition 0.6.3 Horizontal line test.

A function is one-to-one if and only if no horizontal line y = c intersects thegraph y = f(x) more than once.

i.e. every horizontal line intersects the graph either zero or one times. Nevertwice or more. This test tell us that y = x3 is one-to-one, but y = x2 is not. Howevernote that if we restrict the domain of y = x2 to x ≥ 0 then the horizontal line test ispassed. This is one of the reasons we have to be careful to consider the domain ofthe function.

When a function is one-to-one then it has an inverse function.Definition 0.6.4

Let f be a one-to-one function with domain A and range B. Then its inversefunction is denoted f−1 and has domain B and range A. It is defined by

f−1(y) = x whenever f(x) = y

for any y ∈ B.

So if f maps x to y, then f−1 maps y back to x. That is f−1 “undoes” f . Becauseof this we have

f−1(f(x)) = x for any x ∈ Af(f−1(y)) = y for any y ∈ B

We have to be careful not to confuse f−1(x) with1

f(x). The “−1” is not an exponent.

23

Page 40: CLP-1 Differential Calculus

THE BASICS 0.6 INVERSE FUNCTIONS

Example 0.6.5 Inverse of x5 + 3.

Let f(x) = x5 + 3 on domain R. To find its inverse we do the following

• Write y = f(x); that is y = x5 + 3.

• Solve for x in terms of y (this is not always easy) — x5 = y−3, so x = (y−3)1/5.

• The solution is f−1(y) = (y − 3)1/5.

• Recall that the “y” in f−1(y) is a dummy variable. That is, f−1(y) = (y − 3)1/5

means that if you feed the number y into the function f−1 it outputs the number(y − 3)1/5. You may call the input variable anything you like. So if you wishto call the input variable “x” instead of “y” then just replace every y in f−1(y)with an x.

• That is f−1(x) = (x− 3)1/5.

Example 0.6.6 Inverse of√x− 1.

Let g(x) =√x− 1 on the domain x ≥ 1. We can find the inverse in the same way:

y =√x− 1

y2 = x− 1

x = y2 + 1 = f−1(y) or, writing input variable as “x”:f−1(x) = x2 + 1.

Let us now turn to finding the inverse of sin(x) — it is a little more tricky and wehave to think carefully about domains.

Example 0.6.7 Inverse of sin(x).

We have seen (back in Figure 0.6.1) that sin(x) takes each value y between−1 and +1for infinitely many different values of x (see the left-hand graph in the figure below).Consequently sin(x), with domain −∞ < x <∞ does not have an inverse function.

24

Page 41: CLP-1 Differential Calculus

THE BASICS 0.6 INVERSE FUNCTIONS

Example 0.6.7

But notice that as x runs from −π2

to +π2, sin(x) increases from −1 to +1. (See the

middle graph in the figure above.) In particular, sin(x) takes each value −1 ≤ y ≤ 1for exactly one−π

2≤ x ≤ π

2. So if we restrict sinx to have domain−π

2≤ x ≤ π

2, it does

have an inverse function, which is traditionally called arcsine (see Appendix A.9).That is, by definition, for each −1 ≤ y ≤ 1, arcsin(y) is the unique −π

2≤ x ≤ π

2obey-

ing sin(x) = y. Equivalently, exchanging the dummy variables x and y throughoutthe last sentence gives that for each −1 ≤ x ≤ 1, arcsin(x) is the unique −π

2≤ y ≤ π

2

obeying sin(y) = x.

It is an easy matter to construct the graph of an inverse function from the graphof the original function. We just need to remember that

Y = f−1(X) ⇐⇒ f(Y ) = X

which is y = f(x) with x renamed to Y and y renamed to X .Start by drawing the graph of f , labelling the x– and y–axes and labelling the

curve y = f(x).

Now replace each x by Y and each y by X and replace the resulting label X =f(Y ) on the curve by the equivalent Y = f−1(X).

25

Page 42: CLP-1 Differential Calculus

THE BASICS 0.6 INVERSE FUNCTIONS

Finally we just need to redraw the sketch with the Y axis running vertically (withY increasing upwards) and the X axis running horizontally (with X increasing tothe right). To do so, pretend that the sketch is on a transparency or on a very thinpiece of paper that you can see through. Lift the sketch up and flip it over so thatthe Y axis runs vertically and the X axis runs horizontally. If you want, you can alsoconvert the upper case X into a lower case x and the upper case Y into a lower casey.

Another way to say “flip the sketch over so as to exchange the x– and y–axes” is“reflect in the line y = x”. In the figure below the blue “horizontal” elliptical diskthat is centred on (a, b) has been reflected in the line y = x to give the red “vertical”elliptical disk centred on (b, a).

26

Page 43: CLP-1 Differential Calculus

THE BASICS 0.6 INVERSE FUNCTIONS

Example 0.6.8 Sketching inverse of y = x2.

As an example, let f(x) = x2 with domain 0 ≤ x <∞.

• When x = 0, f(x) = 02 = 0.

• As x increases, x2 gets bigger and bigger.

• When x is very large and positive, x2 is also very large and positive. (For ex-ample, think x = 100.)

The graph of y = f(x) = x2 is the blue curve below. By definition, Y = f−1(X) ifX = f(Y ) = Y 2. That is, if Y =

√X . (Remember that, to be in the domain of f , we

must have Y ≥ 0.) So the inverse function of “square” is “square root”. The graphof f−1 is the red curve below. The red curve is the reflection of the blue curve in theline y = x.

27

Page 44: CLP-1 Differential Calculus

LIMITS

Chapter 1

So very roughly speaking, “Differential Calculus” is the study of how a functionchanges as its input changes. The mathematical object we use to describe this is the“derivative” of a function. To properly describe what this thing is we need somemachinery; in particular we need to define what we mean by “tangent” and “limit”.We’ll get back to defining the derivative in Chapter 2.

1.1q Drawing Tangents and a First Limit

1.1.1 tt Drawing Tangents and a First Limit

Our motivation for developing “limit” — being the title and subject of this chapter —is going to be two related problems of drawing tangent lines and computing velocity.

Now — our treatment of limits is not going to be completely mathematicallyrigorous, so we won’t have too many formal definitions. There will be a few math-ematically precise definitions and theorems as we go, but we’ll make sure there isplenty of explanation around them.

Let us start with the “tangent line” problem. Of course, we need to define “tan-gent”, but we won’t do this formally. Instead let us draw some pictures.

28

Page 45: CLP-1 Differential Calculus

LIMITS 1.1 DRAWING TANGENTS AND A FIRST LIMIT

Here we have drawn two very rough sketches of the curve y = x2 for x ≥ 0.These are not very good sketches for a couple of reasons

• The curve in the figure does not pass through (0, 0), even though (0, 0) lies ony = x2.

• The top-right end of the curve doubles back on itself and so fails the verticalline test that all functions must satisfy 1 — for each x-value there is exactly oney-value for which (x, y) lies on the curve y = x2.

So let’s draw those more carefully.

Figure 1.1.1: Sketches of the curve y = x2. (left) shows a tangent line, while(right) shows a line that is not a tangent.

These are better. In both cases we have drawn y = x2 (carefully) and then pickeda point on the curve — call it P . Let us zoom in on the “good” example:

1 Take a moment to go back and reread Definition 0.4.1.

29

Page 46: CLP-1 Differential Calculus

LIMITS 1.1 DRAWING TANGENTS AND A FIRST LIMIT

Figure 1.1.2: We see that, the more we zoom in on the point P , the more thegraph of the function (drawn in black) looks like a straight line — that line is thetangent line (drawn in blue).

We see that as we zoom in on the point P , the graph of the function looks moreand more like a straight line. If we kept on zooming in on P then the graph of thefunction would be indistinguishable from a straight line. That line is the tangent line(which we have drawn in blue). A little more precisely, the blue line is “the tangentline to the function at P”. We have to be a little careful, because if we zoom in at adifferent point, then we will find a different tangent line.

Now let’s zoom in on the “bad” example we see that the blue line looks verydifferent from the function; because of this, the blue line is not the tangent line at P .

Figure 1.1.3: Zooming in on P we see that the function (drawn in black) looksmore and more like a straight line — however it is not the same line as thatdrawn in blue. Because of this the blue line is not the tangent line.

Here are a couple more examples of tangent lines

30

Page 47: CLP-1 Differential Calculus

LIMITS 1.1 DRAWING TANGENTS AND A FIRST LIMIT

Figure 1.1.4: More examples of tangent lines.

The one on the left is very similar to the good example on y = x2 that we sawabove, while the one on the right is different — it looks a little like the “bad” example,in that it crosses our function the curve at some distant point. Why is the line inFigure 1.1.4(right) a tangent while the line in Figure 1.1.1(right) not a tangent? To seewhy, we should again zoom in close to the point where we are trying to draw thetangent.

As we saw above in Figure 1.1.3, when we zoom in around our example of “nota tangent line” we see that the straight line looks very different from the curve at the“point of tangency” — i.e. where we are trying to draw the tangent. The line drawnin Figure 1.1.4(right) looks more and more like the function as we zoom in.

This example raises an important point — when we are trying to draw a tangentline, we don’t care what the function does a long way from the point; the tangentline to the curve at a particular point P , depends only on what the function lookslike close to that point P .

To illustrate this consider the sketch of the function y = sin(x) and its tangent lineat (x, y) = (0, 0):

31

Page 48: CLP-1 Differential Calculus

LIMITS 1.1 DRAWING TANGENTS AND A FIRST LIMIT

As we zoom in, the graph of sin(x) looks more and more like a straight line — infact it looks more and more like the line y = x. We have also sketched this tangentline. What makes this example a little odd is that the tangent line crosses the func-tion. In the examples above, our tangent lines just “kissed” the curve and did notcross it (or at least did not cross it nearby).

Using this idea of zooming in at a particular point, drawing a tangent line is nottoo hard. However, finding the equation of the tangent line presents us with a fewchallenges. Rather than leaping into the general theory, let us do a specific example.Let us find the the equation of the tangent line to the curve y = x2 at the point P withcoordinates 2 (x, y) = (1, 1).

To find the equation of a line we either need

• the slope of the line and a point on the line, or

• two points on the line, from which we can compute the slope via the formula

m =y2 − y1

x2 − x1

and then write down the equation for the line via a formula such as

y = m · (x− x1) + y1.

We cannot use the first method because we do not know what the slope of thetangent line should be. To work out the slope we need calculus — so we’ll be able touse this method once we get to the next chapter on “differentiation”.

It is not immediately obvious how we can use the second method, since we onlyhave one point on the curve, namely (1, 1). However we can use it to “sneak up”on the answer. Let’s approximate the tangent line, by drawing a line that passesthrough (1, 1) and some nearby point — call it Q. Here is our recipe:

• We are given the point P = (1, 1) and we are told

Find the tangent line to the curve y = x2 that passes through P =(1, 1).

2 Note that the coordinates (x, y) is an ordered pair of two numbers x and y. Traditionally the firstnumber is called the abscissa while the second is the ordinate, but these terms are a little archaic.It is now much more common to hear people refer to the first number as the x-coordinate and thesecond as y-coordinate.

32

Page 49: CLP-1 Differential Calculus

LIMITS 1.1 DRAWING TANGENTS AND A FIRST LIMIT

• We don’t quite know how to find a line given just 1 point, however we doknow how to find a line passing through 2 points. So pick another point on thecurves whose coordinates are very close to P . Now rather than picking someactual numbers, I am going to write our second point as Q = (1 + h, (1 + h)2).That is, a point Q whose x-coordinate is equal to that of P plus a little bit —where the little bit is some small number h. And since this point lies on thecurve y = x2, and Q’s x-coordinate is 1 + h, Q’s y-coordinate must be (1 + h)2.

If having h as an variable rather than a number bothers you, start by thinkingof h as 0.1.

• A picture of the situation will help.

• This line that passes through the curve in two places P andQ is called a “secantline”.

• The slope of the line is then

m =y2 − y1

x2 − x1

=(1 + h)2 − 1

(1 + h)− 1=

1 + 2h+ h2 − 1

h=

2h+ h2

h= 2 + h

where we have expanded (1 + h)2 = 1 + 2h+ h2 and then cleaned up a bit.

Now this isn’t our tangent line because it passes through 2 nearby points on thecurve — however it is a reasonable approximation of it. Now we can make thatapproximation better and so “sneak up” on the tangent line by considering whathappens when we move this point Q closer and closer to P . i.e. make the number hcloser and closer to zero.

33

Page 50: CLP-1 Differential Calculus

LIMITS 1.1 DRAWING TANGENTS AND A FIRST LIMIT

First look at the picture. The original choice of Q is on the left, while on the rightwe have drawn what happens if we choose h′ to be some number a little smaller thanh, so that our point Q becomes a new point Q′ that is a little closer to P . The newapproximation is better than the first.

So as we make h smaller and smaller, we bring Q closer and closer to P , andmake our secant line a better and better approximation of the tangent line. We canobserve what happens to the slope of the line as we make h smaller by pluggingsome numbers into our formula m = 2 + h:

h = 0.1 m = 2.1

h = 0.01 m = 2.01

h = 0.001 m = 2.001.

So again we see that as this difference in x becomes smaller and smaller, the slopeappears to be getting closer and closer to 2. We can write this more mathematicallyas

limh→0

(1 + h)2 − 1

h= 2

This is read as

The limit, as h approaches 0, of (1+h)2−1h

is 2.

This is our first limit! Notice that we can see this a little more clearly with a quick bitof algebra:

(1 + h)2 − 1

h=

(1 + 2h+ h2)− 1

h

=2h+ h2

h(2 + h)

So it is not unreasonable to expect that

limh→0

(1 + h)2 − 1

h= lim

h→0(2 + h) = 2.

Our tangent line can be thought of as the end of this process — namely as webring Q closer and closer to P , the slope of the secant line comes closer and closerto that of the tangent line we want. Since we have worked out what the slope is —that is the limit we saw just above — we now know the slope of the tangent line is 2.Given this, we can work out the equation for the tangent line.

34

Page 51: CLP-1 Differential Calculus

LIMITS 1.1 DRAWING TANGENTS AND A FIRST LIMIT

• The equation for the line is y = mx+ c. We have 2 unknowns m and c — so weneed 2 pieces of information to find them.

• Since the line is tangent to P = (1, 1) we know the line must pass through (1, 1).From the limit we computed above, we also know that the line has slope 2.

• Since the slope is 2 we know that m = 2. Thus the equation of the line isy = 2x+ c.

• We know that the line passes through (1, 1), so that y = 2x+ c must be 1 whenx = 1. So 1 = 2 · 1 + c, which forces c = −1.

So our tangent line is y = 2x− 1.

1.1.2 tt Exercises

Exercises — Stage 1

1. On the graph below, draw:

a The tangent line to y = f(x) at P ,

b the tangent line to y = f(x) at Q, and

c the secant line to y = f(x) through P and Q.

x

yy = f(x)

P

Q

2. Suppose a curve y = f(x) has tangent line y = 2x+ 3 at the point x = 2.

a True or False: f(2) = 7

b True or False: f(3) = 9

3. Let L be the tangent line to a curve y = f(x) at some point P . How manytimes will L intersect the curve y = f(x)?

35

Page 52: CLP-1 Differential Calculus

LIMITS 1.2 ANOTHER LIMIT AND COMPUTING VELOCITY

1.2q Another Limit and Computing Velocity

1.2.1 tt Another Limit and Computing Velocity

Computing tangent lines is all very well, but what does this have to do with appli-cations or the “Real World”? Well - at least initially our use of limits (and indeedof calculus) is going to be a little removed from real world applications. Howeveras we go further and learn more about limits and derivatives we will be able to getcloser to real problems and their solutions.

So stepping just a little closer to the real world, consider the following problem.You drop a ball from the top of a very very tall building. Let t be elapsed timemeasured in seconds, and s(t) be the distance the ball has fallen in metres. So s(0) =0.

Quick aside: there is quite a bit going on in the statement of this problem. We havedescribed the general picture — tall building, ball, falling — but we have also intro-duced notation, variables and units. These will be common first steps in applicationsand are necessary in order to translate a real world problem into mathematics in aclear and consistent way.

Galileo 1 worked out that s(t) is a quadratic function:

s(t) = 4.9t2.

The question that is posed is

How fast is the ball falling after 1 second?

Now before we get to answering this question, we should first be a little more pre-cise. The wording of this question is pretty sloppy for a couple of reasons:

• What we do mean by “after 1 second”? We know the ball will move faster andfaster as time passes, so after 1 second it does not fall at one fixed speed.

• As it stands a reasonable answer to the question would be just “really fast”. Ifthe person asking the question wants a numerical answer it would be better toask “At what speed” or “With what velocity”.

1 Perhaps one of the most famous experiments in all of physics is Galileo’s leaning tower of Pisaexperiment, in which he dropped two balls of different masses from the top of the tower andobserved that the time taken to reach the ground was independent of their mass. This disprovedAristotle’s assertion that heavier objects fall faster. It is quite likely that Galileo did not actu-ally perform this experiment. Rather it was a thought-experiment. However a quick glance atWikipedia will turn up some wonderful footage from the Apollo 15 mission showing a hammerand feather being dropped from equal height hitting the moon’s surface at the same time. Fi-nally, Galileo determined that the speed of falling objects increases at a constant rate, which isequivalent to the formula stated here, but it is unlikely that he wrote down an equation exactlyas it is here.

36

Page 53: CLP-1 Differential Calculus

LIMITS 1.2 ANOTHER LIMIT AND COMPUTING VELOCITY

We should also be careful using the words “speed” and “velocity” — they are notinterchangeable.

• Speed means the distance travelled per unit time and is always a non-negativenumber. An unmoving object has speed 0, while a moving object has positivespeed.

• Velocity, on the other hand, also specifies the direction of motion. In this textwe will almost exclusively deal with objects moving along straight lines. Be-cause of this velocities will be positive or negative numbers indicating whichdirection the object is moving along the line. We will be more precise aboutthis later 2.

A better question is

What is the velocity of the ball precisely 1 second after it is dropped?

or even better:

What is the velocity of the ball at the 1 second mark?

This makes it very clear that we want to know what is happening at exactly 1 secondafter the ball is dropped.

There is something a little subtle going on in this question. In particular, what dowe mean by the velocity at t = 1?. Surely if we freeze time at t = 1 second, then theobject is not moving at all? This is definitely not what we mean.

If an object is moving at a constant velocity 3 in the positive direction, then thatvelocity is just the distance travelled divided by the time taken. That is

v =distance moved

time takenAn object moving at constant velocity that moves 27 metres in 3 seconds has velocity

v =27m

3s= 9m/s.

When velocity is constant everything is easy.However, in our falling object example, the object is being acted on by gravity

and its speed is definitely not constant. Instead of asking for THE velocity, let usexamine the “average velocity” of the object over a certain window of time. In thiscase the formula is very similar

average velocity =distance moved

time takenBut now I want to be more precise, instead write

average velocity =difference in distance

difference in time

2 Getting the sign of velocity wrong is a very common error — you should be careful with it.3 Newton’s first law of motion states that an object in motion moves with constant velocity unless

a force acts on it — for example gravity or friction.

37

Page 54: CLP-1 Differential Calculus

LIMITS 1.2 ANOTHER LIMIT AND COMPUTING VELOCITY

Now in spoken English we haven’t really changed much — the distance moved isthe difference in position, and the time taken is just the difference in time — butthe latter is more mathematically precise, and is easy to translate into the followingequation

average velocity =s(t2)− s(t1)

t2 − t1.

This is the formula for the average velocity of our object between time t1 and t2.The denominator is just the difference between these times and the numerator is thedifference in position — i.e. position at time t1 is just s(t1) and position at time t2 isjust s(t2).

So what is the average velocity of the falling ball between 1 and 1.1 seconds? Allwe need to do now is plug some numbers into our formula

average velocity =difference in position

difference in time

=s(1.1)− s(1)

1.1− 1

=4.9(1.1)2 − 4.9(1)

0.1=

4.9× 0.21

0.1= 10.29m/s

And we have our average velocity. However there is something we should noticeabout this formula and it is easier to see if we sketch a graph of the function s(t)

So on the left I have drawn the graph and noted the times t = 1 and t = 1.1. Thecorresponding positions on the axes and the two points on the curve. On the right Ihave added a few more details. In particular I have noted the differences in positionand time, and the line joining the two points. Notice that the slope of this line is

slope =change in ychange in x

=difference in sdifference in t

which is precisely our expression for the average velocity.Let us examine what happens to the average velocity as we look over smaller and

smaller time-windows.

time window average velocity1 ≤ t ≤ 1.1 10.29

38

Page 55: CLP-1 Differential Calculus

LIMITS 1.2 ANOTHER LIMIT AND COMPUTING VELOCITY

1 ≤ t ≤ 1.01 9.849

1 ≤ t ≤ 1.001 9.8049

1 ≤ t ≤ 1.0001 9.80049

As we make the time interval smaller and smaller we find that the average velocity isgetting closer and closer to 9.8. We can be a little more precise by finding the averagevelocity between t = 1 and t = 1+h— this is very similar to what we did for tangentlines.

average velocity =s(1 + h)− s(1)

(1 + h)− 1

=4.9(1 + h)2 − 4.9

h

=9.8h+ 4.9h2

h= 9.8 + 4.9h

Now as we squeeze this window between t = 1 and t = 1 + h down towards zero,the average velocity becomes the “instantaneous velocity” — just as the slope of thesecant line becomes the slope of the tangent line. This is our second limit

v(1) = limh→0

s(1 + h)− s(1)

h= 9.8

More generally we define the instantaneous velocity at time t = a to be the limit

v(a) = limh→0

s(a+ h)− s(a)

h

We read this as

The velocity at time a is equal to the limit as h goes to zero of s(a+h)−s(a)h

.

While we have solved the problem stated at the start of this section, it is clear that ifwe wish to solve similar problems that we will need to understand limits in a moregeneral and systematic way.

1.2.2 tt Exercises

Exercises — Stage 1

1. As they are used in this section, what is the difference between speed andvelocity?

2. Speed can never be negative; can it be zero?

39

Page 56: CLP-1 Differential Calculus

LIMITS 1.2 ANOTHER LIMIT AND COMPUTING VELOCITY

3. Suppose you wake up in the morning in your room, then you walk twokilometres to school, walk another two kilometres to lunch, walk fourkilometres to a coffee shop to study, then return to your room until thenext morning. In the 24 hours from morning to morning, what wasyour average velocity? (In CLP-1, we are considering functions of onevariable. So, at this stage, think of our whole world as being containedin the x-axis.)

4. Suppose you drop an object, and it falls for a few seconds. Which is larger:its speed at the one second mark, or its average speed from the zero secondmark to the one second mark?

5. The position of an object at time t is given by s(t). Then its average

velocity over the time interval t = a to t = b is given bys(b)− s(a)

b− a .

Explain why this fraction also gives the slope of the secant line of thecurve y = s(t) from the point t = a to the point t = b.

6. Below is the graph of the position of an object at time t. For what periodsof time is the object’s velocity positive?

t

y

1 2 3 4 5 6 7

y = s(t)

Exercises — Stage 2

7. Suppose the position of a body at time t (measured in seconds) is given bys(t) = 3t2 + 5.

a What is the average velocity of the object from 3 seconds to 5 seconds?

b What is the velocity of the object at time t = 1?

8. Suppose the position of a body at time t (measured in seconds) is givenby s(t) =

√t.

a What is the average velocity of the object from t = 1 second tot = 9 seconds?

40

Page 57: CLP-1 Differential Calculus

LIMITS 1.3 THE LIMIT OF A FUNCTION

b What is the velocity of the object at time t = 1?

c What is the velocity of the object at time t = 9?

1.3q The Limit of a Function

1.3.1 tt The Limit of a Function

Before we come to definitions, let us start with a little notation for limits.

Definition 1.3.1

We will often write

limx→a

f(x) = L

which should be read as

The limit of f(x) as x approaches a is L.

The notation is just shorthand — we don’t want to have to write out long sen-tences as we do our mathematics. Whenever you see these symbols you shouldthink of that sentence.

This shorthand also has the benefit of being mathematically precise (we’ll see thislater), and (almost) independent of the language in which the author is writing. Amathematician who does not speak English can read the above formula and under-stand exactly what it means.

In mathematics, like most languages, there is usually more than one way of writ-ing things and we can also write the above limit as

f(x)→ L as x→ a

This can also be read as above, but also as

f(x) goes to L as x goes to a

They mean exactly the same thing in mathematics, even though they might be writ-ten, read and said a little differently.

To arrive at the definition of limit, we want to start 1 with a very simple example.

1 Well, we had two limits in the previous sections, so perhaps we really want to “restart” with avery simple example.

41

Page 58: CLP-1 Differential Calculus

LIMITS 1.3 THE LIMIT OF A FUNCTION

Example 1.3.2 A simple limit.

Consider the following function.

f(x) =

2x x < 3

9 x = 3

2x x > 3

This is an example of a piece-wise function a. That is, a function defined in severalpieces, rather than as a single formula. We evaluate the function at a particular valueof x on a case-by-case basis. Here is a sketch of it

Notice the two circles in the plot. One is open, and the other is closed •.• A filled circle has quite a precise meaning — a filled circle at (x, y) means that

the function takes the value f(x) = y.

• An open circle is a little harder — an open circle at (3, 6) means that the point(3, 6) is not on the graph of y = f(x), i.e. f(3) 6= 6. We should only use the opencircle where it is absolutely necessary in order to avoid confusion.

This function is quite contrived, but it is a very good example to start working withlimits more systematically. Consider what the function does close to x = 3. Wealready know what happens exactly at 3 — f(x) = 9 — but I want to look at how thefunction behaves very close to x = 3. That is, what does the function do as we lookat a point x that gets closer and closer to x = 3.If we plug in some numbers very close to 3 (but not exactly 3) into the function wesee the following:

x 2.9 2.99 2.999 3.001 3.01 3.1f(x) 5.8 5.98 5.998 6.002 6.02 6.2

So as xmoves closer and closer to 3, without being exactly 3, we see that the functionmoves closer and closer to 6. We can write this as

limx→3

f(x) = 6

That is

42

Page 59: CLP-1 Differential Calculus

LIMITS 1.3 THE LIMIT OF A FUNCTION

Example 1.3.2

The limit as x approaches 3 of f(x) is 6.

So for x very close to 3, without being exactly 3, the function is very close to 6 —which is a long way from the value of the function exactly at 3, f(3) = 9. Note wellthat the behaviour of the function as x gets very close to 3 does not depend on thevalue of the function at 3.

a We saw another piecewise function back in Example 0.4.5.

We now have enough to make an informal definition of a limit, which is actuallysufficient for most of what we will do in this text.

Definition 1.3.3 Informal definition of limit.

We write

limx→a

f(x) = L

if the value of the function f(x) is sure to be arbitrarily close to Lwhenever thevalue of x is close enough to a, without a being exactly a.

a You may find the condition “without being exactly a” a little strange, but there is a goodreason for it. One very important application of limits, indeed the main reason we teachthe topic, is in the definition of derivatives (see Definition 2.2.1 in the next chapter). In

that definition we need to compute the limit limx→a

f(x)− f(a)

x− a . In this case the function

whose limit is being taken, namely f(x)−f(a)x−a , is not defined at all at x = a.

In order to make this definition more mathematically correct, we need to makethe idea of “closer and closer” more precise — we do this in Section 1.7. It should beemphasised that the formal definition and the contents of that section are optionalmaterial.

For now, let us use the above definition to examine a more substantial example.

Example 1.3.4 limx→2

x− 2

x2 + x− 6.

Let f(x) = x−2x2+x−6

and consider its limit as x→ 2.

• We are really being asked

limx→2

x− 2

x2 + x− 6= what?

43

Page 60: CLP-1 Differential Calculus

LIMITS 1.3 THE LIMIT OF A FUNCTION

Example 1.3.4

• Now if we try to compute f(2) we get 0/0 which is undefined. The function isnot defined at that point — this is a good example of why we need limits. Wehave to sneak up on these places where a function is not defined (or is badlybehaved).

• Very important point: the fraction 00

is not ∞ and it is not 1, it is not defined.We cannot ever divide by zero in normal arithmetic and obtain a consistent andmathematically sensible answer. If you learned otherwise in high-school, youshould quickly unlearn it.

• Again, we can plug in some numbers close to 2 and see what we find

x 1.9 1.99 1.999 2.001 2.01 2.1f(x) 0.20408 0.20040 0.20004 0.19996 0.19960 0.19608

• So it is reasonable to suppose that

limx→2

x− 2

x2 + x− 6= 0.2

The previous two examples are nicely behaved in that the limits we tried to com-pute actually exist. We now turn to two nastier examples 2 in which the limits weare interested in do not exist.

Example 1.3.5 A bad example.

Consider the following function f(x) = sin(π/x). Find the limit as x→ 0 of f(x).We should see something interesting happening close to x = 0 because f(x) is unde-fined there. Using your favourite graph-plotting software you can see that the graphlooks roughly like

2 Actually, they are good examples, but the functions in them are nastier.

44

Page 61: CLP-1 Differential Calculus

LIMITS 1.3 THE LIMIT OF A FUNCTION

Example 1.3.5

How to explain this? As x gets closer and closer to zero, π/x becomes larger andlarger (remember what the plot of y = 1/x looks like). So when you take sine of thatnumber, it oscillates faster and faster the closer you get to zero. Since the functiondoes not approach a single number as we bring x closer and closer to zero, the limitdoes not exist.We write this as

limx→0

sin(πx

)does not exist

It’s not very inventive notation, however it is clear. We frequently abbreviate “doesnot exist” to “DNE” and rewrite the above as

limx→0

sin(πx

)= DNE

In the following example, the limit we are interested in does not exist. Howeverthe way in which things go wrong is quite different from what we just saw.

Example 1.3.6 A non-existant limit.

Consider the function

f(x) =

x x < 2

−1 x = 2

x+ 3 x > 2

• The plot of this function looks like this

45

Page 62: CLP-1 Differential Calculus

LIMITS 1.3 THE LIMIT OF A FUNCTION

Example 1.3.6

• So let us plug in numbers close to 2.

x 1.9 1.99 1.999 2.001 2.01 2.1f(x) 1.9 1.99 1.999 5.001 5.01 5.1

• This isn’t like before. Now when we approach from below, we seem to begetting closer to 2, but when we approach from above we seem to be gettingcloser to 5. Since we are not approaching the same number the limit does notexist.

limx→2

f(x) = DNE

While the limit in the previous example does not exist, the example serves tointroduce the idea of “one-sided limits”. For example, we can say that

As x moves closer and closer to two from below the function approaches 2.

and similarly

As x moves closer and closer to two from above the function approaches 5.

Definition 1.3.7 Informal definition of one-sided limits.

We write

limx→a−

f(x) = K

when the value of f(x) gets closer and closer to K when x < a and x movescloser and closer to a. Since the x-values are always less than a, we say that xapproaches a from below. This is also often called the left-hand limit since the

46

Page 63: CLP-1 Differential Calculus

LIMITS 1.3 THE LIMIT OF A FUNCTION

x-values lie to the left of a on a sketch of the graph.We similarly write

limx→a+

f(x) = L

when the value of f(x) gets closer and closer to L when x > a and x movescloser and closer to a. For similar reasons we say that x approaches a fromabove, and sometimes refer to this as the right-hand limit.

Note — be careful to include the superscript + and−when writing these limits. Youmight also see the following notations:

limx→a+

f(x) = limx→a+

f(x) = limx↓a

f(x) = limxa

f(x) = L right-hand limit

limx→a−

f(x) = limx→a−

f(x) = limx↑a

f(x) = limxa

f(x) = L left-hand limit

but please use with the notation in Definition 1.3.7 above.Given these two similar notions of limits, when are they the same? The following

theorem tell us.

Theorem 1.3.8 Limits and one sided limits.

limx→a

f(x) = L if and only if limx→a−

f(x) = L and limx→a+

f(x) = L

Notice that this is really two separate statements because of the “if and only if”

• If the limit of f(x) as x approaches a exists and is equal to L, then both theleft-hand and right-hand limits exist and are equal to L. AND,

• If the left-hand and right-hand limits as x approaches a exist and are equal,then the limit as x approaches a exists and is equal to the one-sided limits.

That is — the limit of f(x) as x approaches a will only exist if it doesn’t matter whichway we approach a (either from left or right) AND if we get the same one-sidedlimits when we approach from left and right, then the limit exists.

We can rephrase the above by writing the contrapositives 3 of the above state-ments.

3 Given a statement of the form “If A then B”, the contrapositive is “If not B then not A”. They arelogically equivalent — if one is true then so is the other. We must take care not to confuse thecontrapositive with the converse. Given “If A then B”, the converse is “If B then A”. These aredefinitely not the same. To see this consider the statement “If he is Shakespeare then he is dead.”The converse is “If he is dead then he is Shakespeare” — clearly garbage since there are plentyof dead people who are not Shakespeare. The contrapositive is “If he is not dead then he is notShakespeare” — which makes much more sense.

47

Page 64: CLP-1 Differential Calculus

LIMITS 1.3 THE LIMIT OF A FUNCTION

• If either of the left-hand and right-hand limits as x approaches a fail to exist,or if they both exist but are different, then the limit as x approaches a does notexist. AND,

• If the limit as x approaches a does not exist, then the left-hand and right-handlimits are either different or at least one of them does not exist.

Here is another limit example

Example 1.3.9 Left- and right-handed limits.

Consider the following two functions and compute their limits and one-sided limitsas x approaches 1:

These are a little different from our previous examples, in that we do not have for-mulas, only the sketch. But we can still compute the limits.

• Function on the left — f(x):

limx→1−

f(x) = 2 limx→1+

f(x) = 2

so by the previous theorem

limx→1

f(x) = 2

• Function on the right — g(t):

limt→1−

g(t) = 2 and limt→1+

g(t) = −2

so by the previous theorem

limt→1

g(t) = DNE

We have seen 2 ways in which a limit does not exist — in one case the functionoscillated wildly, and in the other there was some sort of “jump” in the function, sothat the left-hand and right-hand limits were different.

There is a third way that we must also consider. To describe this, consider the

48

Page 65: CLP-1 Differential Calculus

LIMITS 1.3 THE LIMIT OF A FUNCTION

following four functions:

Figure 1.3.10

None of these functions are defined at x = a, nor do the limits as x approaches aexist. However we can say more than just “the limits do not exist”.

Notice that the value of function 1 can be made bigger and bigger as we bringx closer and closer to a. Similarly the value of the second function can be madearbitrarily large and negative (i.e. make it as big a negative number as we want) bybringing x closer and closer to a. Based on this observation we have the followingdefinition.

Definition 1.3.11

We write

limx→a

f(x) = +∞

when the value of the function f(x) becomes arbitrarily large and positive asx gets closer and closer to a, without being exactly a.Similarly, we write

limx→a

f(x) = −∞

when the value of the function f(x) becomes arbitrarily large and negative asx gets closer and closer to a, without being exactly a.

A good examples of the above is

limx→0

1

x2= +∞ lim

x→0− 1

x2= −∞

IMPORTANT POINT: Please do not think of “+∞” and “−∞” in these statementsas numbers. You should think of lim

x→af(x) = +∞ and lim

x→af(x) = −∞ as special cases

of limx→a

f(x) = DNE. The statement

limx→a

f(x) = +∞

49

Page 66: CLP-1 Differential Calculus

LIMITS 1.3 THE LIMIT OF A FUNCTION

does not say “the limit of f(x) as x approaches a is positive infinity”. It says “thefunction f(x) becomes arbitrarily large as x approaches a”. These are different state-ments; remember that∞ is not a number 4.

Now consider functions 3 and 4 in Figure 1.3.10. Here we can make the value ofthe function as big and positive as we want (for function 3) or as big and negative aswe want (for function 4) but only when x approaches a from one side. With this inmind we can construct similar notation and a similar definition:

Definition 1.3.12

We write

limx→a+

f(x) = +∞

when the value of the function f(x) becomes arbitrarily large and positive as xgets closer and closer to a from above (equivalently — from the right), withoutbeing exactly a.Similarly, we write

limx→a+

f(x) = −∞

when the value of the function f(x) becomes arbitrarily large and negative as xgets closer and closer to a from above (equivalently — from the right), withoutbeing exactly a.The notation

limx→a−

f(x) = +∞ limx→a−

f(x) = −∞

has a similar meaning except that limits are approached from below / fromthe left.

So for function 3 we have

limx→a−

f(x) = +∞ limx→a+

f(x) = some positive number

and for function 4

limx→a−

f(x) = some positive number limx→a+

f(x) = −∞

More examples:

4 One needs to be very careful making statements about infinity. At some point in our lives we getaround to asking ourselves “what is the biggest number”, and we realise there isn’t one. Thatis, we can go on counting integer after integer, for ever and not stop. Indeed the set of integersis the first infinite thing we really encounter. It is an example of a countably infinite set. The setof real-numbers is actually much bigger and is uncountably infinite. In fact there are an infinitenumber of different sorts of infinity! Much of the theory of infinite sets was developed by GeorgCantor; we mentioned him back in Section 0.2 and he is well worth googling.

50

Page 67: CLP-1 Differential Calculus

LIMITS 1.3 THE LIMIT OF A FUNCTION

Example 1.3.13 limx→π

1

sin(x).

Consider the function

g(x) =1

sin(x)

Find the one-sided limits of this function as x→ π.Probably the easiest way to do this is to first plot the graph of sin(x) and 1/x andthen think carefully about the one-sided limits:

• As x → π from the left, sin(x) is a small positive number that is getting closerand closer to zero. That is, as x→ π−, we have that sin(x)→ 0 through positivenumbers (i.e. from above). Now look at the graph of 1/x, and think whathappens as we move x → 0+, the function is positive and becomes larger andlarger.

So as x→ π from the left, sin(x)→ 0 from above, and so 1/ sin(x)→ +∞.

• By very similar reasoning, as x → π from the right, sin(x) is a small negativenumber that gets closer and closer to zero. So as x→ π from the right, sin(x)→0 through negative numbers (i.e. from below) and so 1/ sin(x) to −∞.

Thus

limx→π−

1

sin(x)= +∞ lim

x→π+

1

sin(x)= −∞

Again, we can make Definitions 1.3.11 and 1.3.12 into mathematically preciseformal definitions using techniques very similar to those in the optional Section 1.7.This is not strictly necessary for this course.

Up to this point we explored limits by sketching graphs or plugging values intoa calculator. This was done to help build intuition, but it is not really the basis ofa systematic method for computing limits. We have also avoided more formal ap-

51

Page 68: CLP-1 Differential Calculus

LIMITS 1.3 THE LIMIT OF A FUNCTION

proaches 5 since we do not have time in the course to go into that level of detail and(arguably) we don’t need that detail to achieve the aims of the course. Thankfullywe can develop a more systematic approach based on the idea of building up com-plicated limits from simpler ones by examining how limits interact with the basicoperations of arithmetic.

1.3.2 tt Exercises

Exercises — Stage 1

1. Given the function shown below, evaluate the following:

a limx→−2

f(x)

b limx→0

f(x)

c limx→2

f(x)

x

y

y = f(x)

−2 2

−2

1

2

2. Given the function shown below, evaluate limx→0

f(x).

x

y y = f(x)

−1

1

5 The formal approaches are typically referred to as “epsilon-delta limits” or “epsilon-delta proofs”since the symbols ε and δ are traditionally used throughout. Take a peek at Section 1.7 to see.

52

Page 69: CLP-1 Differential Calculus

LIMITS 1.3 THE LIMIT OF A FUNCTION

3. Given the function shown below, evaluate:

a limx→−1−

f(x)

b limx→−1+

f(x)

c limx→−1

f(x)

d limx→−2+

f(x)

e limx→2−

f(x)

x

y

y = f(x)

−2 −1 1 2

−2

2

4. Draw a curve y = f(x) with limx→3

f(x) = f(3) = 10.

5. Draw a curve y = f(x) with limx→3

f(x) = 10 and f(3) = 0.

6. Suppose limx→3

f(x) = 10. True or false: f(3) = 10.

7. Suppose f(3) = 10. True or false: limx→3

f(x) = 10.

8. Suppose f(x) is a function defined on all real numbers, and limx→−2

f(x) =

16. What is limx→−2−

f(x)?

9. Suppose f(x) is a function defined on all real numbers, and limx→−2−

f(x) =

16. What is limx→−2

f(x)?

Exercises — Stage 2 In Questions 1.3.2.10 through 1.3.2.17, evaluate the given limits.

53

Page 70: CLP-1 Differential Calculus

LIMITS 1.4 CALCULATING LIMITS WITH LIMIT LAWS

If you aren’t sure where to begin, it’s nice to start by drawing the function.

10. limt→0

sin t

11. limx→0+

log x

12. limy→3

y2

13. limx→0−

1

x

14. limx→0

1

x

15. limx→0

1

x2

16. limx→3

1

10

17. limx→3

f(x), where f(x) =

sinx x ≤ 2.9

x2 x > 2.9.

1.4q Calculating Limits with Limit Laws

1.4.1 tt Calculating Limits with Limit Laws

Think back to the functions you know and the sorts of things you have been askedto draw, factor and so on. Then they are all constructed from simple pieces, such as

• constants — c

• monomials — xn

• trigonometric functions — sin(x), cos(x) and tan(x)

These are the building blocks from which we construct functions. Soon we will adda few more functions to this list, especially the exponential function and variousinverse functions.

We then take these building blocks and piece them together using arithmetic

54

Page 71: CLP-1 Differential Calculus

LIMITS 1.4 CALCULATING LIMITS WITH LIMIT LAWS

• addition and subtraction — f(x) = g(x) + h(x) and f(x) = g(x)− h(x)

• multiplication — f(x) = g(x) · h(x)

• division — f(x) = g(x)h(x)

• substitution — f(x) = g(h(x)) — this is also called the composition of g with h.

The idea of building up complicated functions from simpler pieces was discussed inSection 0.5.

What we will learn in this section is how to compute the limits of the basic build-ing blocks and then how we can compute limits of sums, products and so forth us-ing “limit laws”. This process allows us to compute limits of complicated functions,using very simple tools and without having to resort to “plugging in numbers” or“closer and closer” or “ε− δ arguments”.

In the examples we saw above, almost all the interesting limits happened at pointswhere the underlying function was badly behaved — where it jumped, was not de-fined or blew up to infinity. In those cases we had to be careful and think about whatwas happening. Thankfully most functions we will see do not have too many pointsat which these sorts of things happen.

For example, polynomials do not have any nasty jumps and are defined every-where and do not “blow up”. If you plot them, they look smooth 1. Polynomialsand limits behave very nicely together, and for any polynomial P (x) and any realnumber a we have that

limx→a

P (x) = P (a)

That is — to evaluate the limit we just plug in the number. We will build up to thisresult over the next few pages.

Let us start with the two easiest limits 2

Theorem 1.4.1 Easiest limits.

Let a, c ∈ R. The following two limits hold

limx→a

c = c and limx→a

x = a.

Since we have not seen too many theorems yet, let us examine it carefully pieceby piece.

• Let a, c ∈ R — just as was the case for definitions, we start a theorem by defin-ing terms and setting the scene. There is not too much scene to set: the symbolsa and c are real numbers.

1 We have used this term in an imprecise way, but it does have a precise mathematical meaning.2 Though it lies outside the scope of the course, you can find the formal ε-δ proof of this result at

the end of Section 1.7.

55

Page 72: CLP-1 Differential Calculus

LIMITS 1.4 CALCULATING LIMITS WITH LIMIT LAWS

• The following two limits hold — this doesn’t really contribute much to thestatement of the theorem, it just makes it easier to read.

• limx→a

c = c — when we take the limit of a constant function (for example thinkof c = 3), the limit is (unsurprisingly) just that same constant.

• limx→a

x = a — as we noted above for general polynomials, the limit of the func-

tion f(x) = x as x approaches a given point a, is just a. This says somethingquite obvious — as x approaches a, x approaches a (if you are not convincedthen sketch the graph).

Armed with only these two limits, we cannot do very much. But combining theselimits with some arithmetic we can do quite a lot. For a moment, take a step backfrom limits for a moment and think about how we construct functions. To make thediscussion a little more precise think about how we might construct the function

h(x) =2x− 3

x2 + 5x− 6

If we want to compute the value of the function at x = 2, then we would

• compute the numerator at x = 2

• compute the denominator at x = 2

• compute the ratio

Now to compute the numerator we

• take x and multiply it by 2

• subtract 3 to the result

While for the denominator

• multiply x by x

• multiply x by 5

• add these two numbers and subtract 6

This sequence of operations can be represented pictorially as the tree shown in Fig-ure 1.4.2 below.

56

Page 73: CLP-1 Differential Calculus

LIMITS 1.4 CALCULATING LIMITS WITH LIMIT LAWS

Figure 1.4.2

Such trees were discussed in Section 0.5 (now is not a bad time to quickly reviewthat section before proceeding). The point here is that in order to compute the valueof the function we just repeatedly add, subtract, multiply and divide constants andx.

To compute the limit of the above function at x = 2 we can do something verysimilar. From the previous theorem we know how to compute

limx→2

c = c and limx→2

x = 2

and the next theorem will tell us how to stitch together these two limits using thearithmetic we used to construct the function.

Theorem 1.4.3 Arithmetic of limits.

Let a, c ∈ R, let f(x) and g(x) be defined for all x’s that lie in some intervalabout a (but f, g need not be defined exactly at a).

limx→a

f(x) = F limx→a

g(x) = G

exist with F,G ∈ R. Then the following limits hold

• limx→a

(f(x) + g(x)) = F +G — limit of the sum is the sum of the limits.

• limx→a

(f(x)− g(x)) = F −G — limit of the difference is the difference of thelimits.

• limx→a

cf(x) = cF .

• limx→a

(f(x) · g(x)) = F ·G — limit of the product is the product of limits.

57

Page 74: CLP-1 Differential Calculus

LIMITS 1.4 CALCULATING LIMITS WITH LIMIT LAWS

• If G 6= 0 then limx→a

f(x)

g(x)=F

G

Note — be careful with this last one — the denominator cannot be zero.

The above theorem shows that limits interact very simply with arithmetic. If youare asked to find the limit of a sum then the answer is just the sum of the limits.Similarly the limit of a product is just the product of the limits.

How do we apply the above theorem to the rational function h(x) we definedabove? Here is a warm-up example:

Example 1.4.4 Using limit laws.

You are given two functions f, g (not explicitly) which have the following limits as xapproaches 1:

limx→1

f(x) = 3 and limx→1

g(x) = 2

Using the above theorem we can compute

limx→1

3f(x) = 3× 3 = 9

limx→1

3f(x)− g(x) = 3× 3− 2 = 7

limx→1

f(x)g(x) = 3× 2 = 6

limx→1

f(x)

f(x)− g(x)=

3

3− 2= 3

Another simple example

Example 1.4.5 More using limit laws.

Find limx→3

4x2 − 1

We use the arithmetic of limits:

limx→3

4x2 − 1 =(

limx→3

4x2)− lim

x→31 difference of limits

=(

limx→3

4 · limx→3

x2)− lim

x→31 product of limits

= 4 ·(

limx→3

x2)− 1 limit of constant

= 4 ·(

limx→3

x)·(

limx→3

x)− 1 product of limits

= 4 · 3 · 3− 1 limit of x

58

Page 75: CLP-1 Differential Calculus

LIMITS 1.4 CALCULATING LIMITS WITH LIMIT LAWS

Example 1.4.5

= 36− 1

= 35

This is an excruciating level of detail, but when you first use this theorem andtry some examples it is a good idea to do things step by step by step until you arecomfortable with it.

Example 1.4.6 Yet more using limit laws.

Yet another limit — compute limx→2

x

x− 1.

To apply the arithmetic of limits, we need to examine numerator and denominatorseparately and make sure the limit of the denominator is non-zero. Numerator first:

limx→2

x = 2 limit of x

and now the denominator:

limx→2

x− 1 =(

limx→2

x)−(

limx→2

1)

difference of limits

= 2− 1 limit of x and limit of constant = 1

Since the limit of the denominator is non-zero we can put it back together to get

limx→2

x

x− 1=

limx→2

x

limx→2

(x− 1)

=2

1= 2

In the next example we show that many different things can happen if the limitof the denominator is zero.

Example 1.4.7 Be careful with limits of ratios.

We must be careful when computing the limit of a ratio — it is the ratio of the limitsexcept when the limit of the denominator is zero. When the limit of the denominatoris zero Theorem 1.4.3 does not apply and a few interesting things can happen

• If the limit of the numerator is non-zero then the limit of the ratio does not exist

limx→a

f(x)

g(x)= DNE when lim

x→af(x) 6= 0 and lim

x→ag(x) = 0

59

Page 76: CLP-1 Differential Calculus

LIMITS 1.4 CALCULATING LIMITS WITH LIMIT LAWS

Example 1.4.7

For example, limx→0

1

x2= DNE.

• If the limit of the numerator is zero then the above theorem does not give usenough information to decide whether or not the limit exists. It is possible that

the limit does not exist, eg. limx→0

x

x2= lim

x→0

1

x= DNE

the limit is ±∞, eg. limx→0

x2

x4= lim

x→0

1

x2= +∞ or lim

x→0

−x2

x4= lim

x→0

−1

x2= −∞.

the limit is zero, eg. limx→0

x2

x= 0

the limit exists and is non-zero, eg. limx→0

x

x= 1

Now while the above examples are very simple and a little contrived they serve toillustrate the point we are trying to make — be careful if the limit of the denominatoris zero.

We now have enough theory to return to our rational function and compute itslimit as x approaches 2.

Example 1.4.8 More on limits of ratios.

Let h(x) =2x− 3

x2 + 5x− 6and find its limit as x approaches 2.

Since this is the limit of a ratio, we compute the limit of the numerator and denomi-nator separately. Numerator first:

limx→2

2x− 3 =(

limx→2

2x)−(

limx→2

3)

difference of limits

= 2 ·(

limx→2

x)− 3 product of limits and limit of constant

= 2 · 2− 3 limits of x= 1

Denominator next:

limx→2

x2 + 5x− 6 =(

limx→2

x2)

+(

limx→2

5x)−(

limx→2

6)

sum of limits

=(

limx→2

x)·(

limx→2

x)

+ 5 ·(

limx→2

x)− 6

product of limits and limit of constant= 2 · 2 + 5 · 2− 6 limits of x= 8

60

Page 77: CLP-1 Differential Calculus

LIMITS 1.4 CALCULATING LIMITS WITH LIMIT LAWS

Example 1.4.8

Since the limit of the denominator is non-zero, we can obtain our result by taking theratio of the separate limits.

limx→2

2x− 3

x2 + 5x− 6=

limx→2

2x− 3

limx→2

x2 + 5x− 6=

1

8

The above works out quite simply. However, if we were to take the limit as x → 1then things are a bit harder. The limit of the numerator is:

limx→1

2x− 3 = 2 · 1− 3 = −1

(we have not listed all the steps). And the limit of the denominator is

limx→1

x2 + 5x− 6 = 1 · 1 + 5− 6 = 0

Since the limit of the numerator is non-zero, while the limit of the denominator iszero, the limit of the ratio does not exist.

limx→1

2x− 3

x2 + 5x− 6= DNE

It is IMPORTANT TO NOTE that it is not correct to write

limx→1

2x− 3

x2 + 5x− 6=−1

0= DNE

Because we can only write

limx→a

f(x)

g(x)=

limx→a

f(x)

limx→a

g(x)= something

when the limit of the denominator is non-zero (see Example 1.4.7 above).With a little care you can use the arithmetic of limits to obtain the following rules

for limits of powers of functions and limits of roots of functions:

Theorem 1.4.9 More arithmetic of limits — powers and roots.

Let n be a positive integer, let a ∈ R and let f be a function so that

limx→a

f(x) = F

for some real number F . Then the following holds

limx→a

(f(x))n =(

limx→a

f(x))n

= F n

61

Page 78: CLP-1 Differential Calculus

LIMITS 1.4 CALCULATING LIMITS WITH LIMIT LAWS

so that the limit of a power is the power of the limit. Similarly, if

• n is an even number and F > 0, or

• n is an odd number and F is any real number

then

limx→a

(f(x))1/n =(

limx→a

f(x))1/n

= F 1/n

More generallya, if F > 0 and p is any real number,

limx→a

(f(x))p =(

limx→a

f(x))p

= F p

a You may not know the definition of the power bp when p is not a rational number, so hereit is. If b > 0 and p is any real number, then bp is the limit of br as r approaches p throughrational numbers. We won’t do so here, but it is possible to prove that the limit exists.

Notice that we have to be careful when taking roots of limits that might be nega-tive numbers. To see why, consider the case n = 2, the limit

limx→4

x1/2 = 41/2 = 2

limx→4

(−x)1/2 = (−4)1/2 = not a real number

In order to evaluate such limits properly we need to use complex numbers whichare beyond the scope of this text.

Also note that the notation x1/2 refers to the positive square root of x. While 2 and(−2) are both square-roots of 4, the notation 41/2 means 2. This is something we mustbe careful of 3.

So again — let us do a few examples and carefully note what we are doing.

Example 1.4.10 limx→2

(4x2 − 3)1/3.

limx→2

(4x2 − 3)1/3 =(

(limx→2

4x2)− (limx→2

3))1/3

=(4 · 22 − 3

)1/3

= (16− 3)1/3

= 131/3

3 Like ending sentences in prepositions — “This is something up with which we will not put.”This quote is attributed to Churchill though there is some dispute as to whether or not he reallysaid it.

62

Page 79: CLP-1 Differential Calculus

LIMITS 1.4 CALCULATING LIMITS WITH LIMIT LAWS

By combining the last few theorems we can make the evaluation of limits of poly-nomials and rational functions much easier:

Theorem 1.4.11 Limits of polynomials and rational functions.

Let a ∈ R, let P (x) be a polynomial and let R(x) be a rational function. Then

limx→a

P (x) = P (a)

and provided R(x) is defined at x = a then

limx→a

R(x) = R(a)

If R(x) is not defined at x = a then we are not able to apply this result.

So the previous examples are now much easier to compute:

limx→2

2x− 3

x2 + 5x− 6=

4− 3

4 + 10− 6=

1

8

limx→2

(4x2 − 1) = 16− 1 = 15

limx→2

x

x− 1=

2

2− 1= 2

It is clear that limits of polynomials are very easy, while those of rational func-tions are easy except when the denominator might go to zero. We have seen exam-ples where the resulting limit does not exist, and some where it does. We now workto explain this more systematically. The following example demonstrates that it issometimes possible to take the limit of a rational function to a point at which thedenominator is zero. Indeed we must be able to do exactly this in order to be able todefine derivatives in the next chapter.

Example 1.4.12 Numerator and denominator both go to 0.

Consider the limit

limx→1

x3 − x2

x− 1.

If we try to apply the arithmetic of limits then we compute the limits of the numera-tor and denominator separately

limx→1

x3 − x2 = 1− 1 = 0

limx→1

x− 1 = 1− 1 = 0

Since the denominator is zero, we cannot apply our theorem and we are, for themoment, stuck. However, there is more that we can do here — the hint is that the

63

Page 80: CLP-1 Differential Calculus

LIMITS 1.4 CALCULATING LIMITS WITH LIMIT LAWS

Example 1.4.12

numerator and denominator both approach zero as x approaches 1. This means thatthere might be something we can cancel.So let us play with the expression a little more before we take the limit:

x3 − x2

x− 1=x2(x− 1)

x− 1= x2 provided x 6= 1.

So what we really have here is the following function

x3 − x2

x− 1=

x2 x 6= 1

undefined x = 1

If we plot the above function the graph looks exactly the same as y = x2 except thatthe function is not defined at x = 1 (since at x = 1 both numerator and denominatorare zero).

When we compute a limit as x → a, the value of the function exactly at x = a isirrelevant. We only care what happens to the function as we bring x very close to a.So for the above problem we can write

x3 − x2

x− 1= x2 when x is close to 1 but not at x = 1

So the limit as x→ 1 of the function is the same as the limit limx→1

x2 since the functionsare the same except exactly at x = 1. By this reasoning we get

limx→1

x3 − x2

x− 1= lim

x→1x2 = 1

The reasoning in the above example can be made more general:

64

Page 81: CLP-1 Differential Calculus

LIMITS 1.4 CALCULATING LIMITS WITH LIMIT LAWS

Theorem 1.4.13

If f(x) = g(x) except when x = a then limx→a

f(x) = limx→a

g(x) provided the limitof g exists.

How do we know when to use this theorem? The big clue is that when we try tocompute the limit in a naive way, we end up with 0

0. We know that 0

0does not make

sense, but it is an indication that there might be a common factor between numeratorand denominator that can be cancelled. In the previous example, this common factorwas (x− 1).

Example 1.4.14 Another numerator and denominator both go to 0 limit.

Using this idea compute

limh→0

(1 + h)2 − 1

h

• First we should check that we cannot just substitute h = 0 into this — clearlywe cannot because the denominator would be 0.

• But we should also check the numerator to see if we have 00, and we see that

the numerator gives us 1− 1 = 0.

• Thus we have a hint that there is a common factor that we might be able tocancel. So now we look for the common factor and try to cancel it.

(1 + h)2 − 1

h=

1 + 2h+ h2 − 1

hexpand

=2h+ h2

h=h(2 + h)

hfactor and then cancel

= 2 + h

• Thus we really have that

(1 + h)2 − 1

h=

2 + h h 6= 0

undefined h = 0

and because of this

limh→0

(1 + h)2 − 1

h= lim

h→02 + h

= 2

65

Page 82: CLP-1 Differential Calculus

LIMITS 1.4 CALCULATING LIMITS WITH LIMIT LAWS

Of course — we have written everything out in great detail here and that is waymore than is required for a solution to such a problem. Let us do it again a little moresuccinctly.

Example 1.4.15 limh→0

(1 + h)2 − 1

h.

Compute the following limit:

limh→0

(1 + h)2 − 1

h

If we try to use the arithmetic of limits, then we see that the limit of the numeratorand the limit of the denominator are both zero. Hence we should try to factor themand cancel any common factor. This gives

limh→0

(1 + h)2 − 1

h= lim

h→0

1 + 2h+ h2 − 1

h= lim

h→02 + h

= 2

Notice that even though we did this example carefully above, we have still writ-ten some text in our working explaining what we have done. You should alwaysthink about the reader and if in doubt, put in more explanation rather than less. Wecould make the above example even more terse

Example 1.4.16 Redoing previous example with fewer words.

Compute the following limit:

limh→0

(1 + h)2 − 1

h

Numerator and denominator both go to zero as h→ 0. So factor and simplify:

limh→0

(1 + h)2 − 1

h= lim

h→0

1 + 2h+ h2 − 1

h= lim

h→02 + h = 2

A slightly harder one now

66

Page 83: CLP-1 Differential Calculus

LIMITS 1.4 CALCULATING LIMITS WITH LIMIT LAWS

Example 1.4.17 A harder limit with cancellations.

Compute the limit

limx→0

x√1 + x− 1

If we try to use the arithmetic of limits we get

limx→0

x = 0

limx→0

√1 + x− 1 =

√limx→0

1 + x− 1 = 1− 1 = 0

So doing the naive thing we’d get 0/0. This suggests a common factor that can becancelled. Since the numerator and denominator are not polynomials we have to tryother tricks a. We can simplify the denominator

√1 + x − 1 a lot, and in particular

eliminate the square root, by multiplying it by its conjugate√

1 + x+ 1.

x√1 + x− 1

=x√

1 + x− 1×√

1 + x+ 1√1 + x+ 1

multiply byconjugateconjugate

= 1

=x(√

1 + x+ 1)(√

1 + x− 1) (√

1 + x+ 1) bring things together

=x(√

1 + x+ 1)(√

1 + x)2 − 1 · 1

since (a−b)(a+b) = a2−b2

=x(√

1 + x+ 1)

1 + x− 1clean up a little

=x(√

1 + x+ 1)

x

=√

1 + x+ 1 cancel the x

So now we have

limx→0

x√1 + x− 1

= limx→0

√1 + x+ 1

=√

1 + 0 + 1 = 2

a While these tricks are useful (and even cuteb — this footnote is better in the online edition), Taylorpolynomials (see Section 3.4) give us a more systematic way of approaching this problem.

How did we know what to multiply by? Our function was of the forma√b− c

so, to eliminate the square root from the denominator, we employ a trick — we mul-

67

Page 84: CLP-1 Differential Calculus

LIMITS 1.4 CALCULATING LIMITS WITH LIMIT LAWS

tiply by 1. Of course, multiplying by 1 doesn’t do anything. But if you multiply by1 carefully you can leave the value the same, but change the form of the expression.More precisely

a√b− c

=a√b− c

· 1

=a√b− c

·√b+ c√b+ c︸ ︷︷ ︸=1

=a(√

b+ c)

(√b− c

)(√b+ c

) expand denominator carefully

=a(√

b+ c)

√b ·√b− c

√b+ c

√b− c · c

do some cancellation

=a(√

b+ c)

b− c2

Now the numerator contains roots, but the denominator is just a polynomial.Before we move on to limits at infinity, there is one more theorem to see. While

the scope of its application is quite limited, it can be extremely useful. It is called asandwich theorem or a squeeze theorem for reasons that will become apparent.

Sometimes one is presented with an unpleasant ugly function such as

f(x) = x2 sin(π/x)

It is a fact of life, that not all the functions that are encountered in mathematics willbe elegant and simple; this is especially true when the mathematics gets applied toreal world problems. One just has to work with what one gets. So how can wecompute

limx→0

x2 sin(π/x)?

Since it is the product of two functions, we might try

limx→0

x2 sin(π/x) =(

limx→0

x2)·(

limx→0

sin(π/x))

= 0 ·(

limx→0

sin(π/x))

= 0

But we just cheated — we cannot use the arithmetic of limits theorem here, becausethe limit

limx→0

sin(π/x) = DNE

does not exist. Now we did see the function sin(π/x) before (in Example 1.3.5), soyou should go back and look at it again. Unfortunately the theorem “the limit of a

68

Page 85: CLP-1 Differential Calculus

LIMITS 1.4 CALCULATING LIMITS WITH LIMIT LAWS

product is the product of the limits” only holds when the limits you are trying tomultiply together actually exist. So we cannot use it.

However, we do see that the function naturally decomposes into the product oftwo pieces — the functions x2 and sin(π/x). We have sketched the two functions inthe figure on the left below.

While x2 is a very well behaved function and we know quite a lot about it, thefunction sin(π/x) is quite ugly. One of the few things we can say about it is thefollowing

−1 ≤ sin(π/x) ≤ 1 provided x 6= 0

But if we multiply this expression by x2 we get (because x2 ≥ 0)

−x2 ≤ x2 sin(π/x) ≤ x2 provided x 6= 0

and we have sketched the result in the figure above (on the right). So the functionwe are interested in is squeezed or sandwiched between the functions x2 and −x2.

If we focus in on the picture close to x = 0 we see that x approaches 0, the func-tions x2 and −x2 both approach 0. Further, because x2 sin(π/x) is sandwiched be-tween them, it seems that it also approaches 0.

The following theorem tells us that this is indeed the case:

Theorem 1.4.18 Squeeze theorem (or sandwich theorem or pinch theorem).

Let a ∈ R and let f, g, h be three functions so that

f(x) ≤ g(x) ≤ h(x)

for all x in an interval around a, except possibly exactly at x = a. Then if

limx→a

f(x) = limx→a

h(x) = L

then it is also the case that

limx→a

g(x) = L

69

Page 86: CLP-1 Differential Calculus

LIMITS 1.4 CALCULATING LIMITS WITH LIMIT LAWS

Using the above theorem we can compute the limit we want and write it upnicely.

Example 1.4.19 limx→0

x2 sin(π/x).

Compute the limitlimx→0

x2 sin(π/x)

Since −1 ≤ sin(θ) ≤ 1 for all real numbers θ, we have

−1 ≤ sin(π/x) ≤ 1 for all x 6= 0

Multiplying the above by x2 we see that

−x2 ≤ x2 sin(π/x) ≤ x2 for all x 6= 0

Since limx→0

x2 = limx→0

(−x2) = 0 by the sandwich (or squeeze or pinch) theorem we have

limx→0

x2 sin(π/x) = 0

Notice how we have used “words”. We have remarked on this several timesalready in the text, but we will keep mentioning it. It is okay to use words in youranswers to maths problems — and you should do so! These let the reader knowwhat you are doing and help you understand what you are doing.

Example 1.4.20 Another sandwich theorem example.

Let f(x) be a function such that 1 ≤ f(x) ≤ x2 − 2x+ 2. What is limx→1

f(x)?We are already supplied with an inequality, so it is likely that it is going to help us.We should examine the limits of each side to see if they are the same:

limx→1

1 = 1

limx→1

x2 − 2x+ 2 = 1− 2 + 2 = 1

So we see that the function f(x) is trapped between two functions that both approach1 as x→ 1. Hence by the sandwich / pinch / squeeze theorem, we know that

limx→1

f(x) = 1

To get some intuition as to why the squeeze theorem is true, consider when x isvery very close to a. In particular, consider when x is sufficiently close to a that weknow h(x) is within 10−6 of L and that f(x) is also within 10−6 of L. That is

|h(x)− L| < 10−6 and |f(x)− L| < 10−6.

70

Page 87: CLP-1 Differential Calculus

LIMITS 1.4 CALCULATING LIMITS WITH LIMIT LAWS

This means that

L− 10−6 < f(x) ≤ h(x) < L+ 10−6

since we know that f(x) ≤ h(x).But now by the hypothesis of the squeeze theorem we know that f(x) ≤ g(x) ≤

h(x) and so we have

L− 10−6 < f(x) ≤ g(x) ≤ h(x) < L+ 10−6

And thus we know that

L− 10−6 ≤ g(x) ≤ L+ 10−6

That is g(x) is also within 10−6 of L.In this argument our choice of 10−6 was arbitrary, so we can really replace 10−6

with any small number we like. Hence we know that we can force g(x) as close to Las we like, by bringing x sufficiently close to a. We give a more formal and rigorousversion of this argument at the end of Section 1.9.

1.4.2 tt Exercises

Exercises — Stage 1

1. Suppose limx→a

f(x) = 0 and limx→a

g(x) = 0. Which of the following limitscan you compute, given this information?

a limx→a

f(x)

2

b limx→a

2

f(x)

c limx→a

f(x)

g(x)

d limx→a

f(x)g(x)

2. Give two functions f(x) and g(x) that satisfy limx→3

f(x) = limx→3

g(x) = 0

and limx→3

f(x)

g(x)= 10.

71

Page 88: CLP-1 Differential Calculus

LIMITS 1.4 CALCULATING LIMITS WITH LIMIT LAWS

3. Give two functions f(x) and g(x) that satisfy limx→3

f(x) = limx→3

g(x) = 0

and limx→3

f(x)

g(x)= 0.

4. Give two functions f(x) and g(x) that satisfy limx→3

f(x) = limx→3

g(x) = 0 and

limx→3

f(x)

g(x)=∞.

5. Suppose limx→a

f(x) = limx→a

g(x) = 0. What are the possible values of limx→a

f(x)

g(x)?

Exercises — Stage 2 For Questions 1.4.2.6 through 1.4.2.41, evaluate the given limits.

6. limt→10

2(t− 10)2

t

7. limy→0

(y + 1)(y + 2)(y + 3)

cos y

8. limx→3

(4x− 2

x+ 2

)4

9. *. limt→−3

(1− tcos(t)

)

10. *. limh→0

(2 + h)2 − 4

2h

11. *. limt→−2

(t− 5

t+ 4

)12. *. lim

x→1

√5x3 + 4

13. *. limt→−1

(t− 2

t+ 3

)14. *. lim

x→1

log(1 + x)− xx2

15. *. limx→2

(x− 2

x2 − 4

)

16. *. limx→4

x2 − 4x

x2 − 16

72

Page 89: CLP-1 Differential Calculus

LIMITS 1.4 CALCULATING LIMITS WITH LIMIT LAWS

17. *. limx→2

x2 + x− 6

x− 2

18. *. limx→−3

x2 − 9

x+ 3

19. limt→2

1

2t4 − 3t3 + t

20. *. limx→−1

√x2 + 8− 3

x+ 1.

21. *. limx→2

√x+ 7−

√11− x

2x− 4.

22. *. limx→1

√x+ 2−

√4− x

x− 1

23. *. limx→3

√x− 2−

√4− x

x− 3.

24. *. limt→1

3t− 3

2−√

5− t .

25. limx→0−x2 cos

(3

x

)

26. limx→0

x4 sin(

1x

)+ 5x2 cos

(1x

)+ 2

(x− 2)2

27. *. limx→0

x sin2

(1

x

)28. lim

w→5

2w2 − 50

(w − 5)(w − 1)

29. limr→−5

r

r2 + 10r + 25

30. limx→−1

√x3 + x2 + x+ 1

3x+ 3

31. limx→0

x2 + 2x+ 1

3x5 − 5x3

73

Page 90: CLP-1 Differential Calculus

LIMITS 1.4 CALCULATING LIMITS WITH LIMIT LAWS

32. limt→7

t2x2 + 2tx+ 1

t2 − 14t+ 49, where x is a positive constant

33. limd→0

x5 − 32x+ 15, where x is a constant

34. limx→1

(x− 1)2 sin

[(x2 − 3x+ 2

x2 − 2x+ 1

)2

+ 15

]35. *. Evaluate lim

x→0x1/101 sin

(x−100

)or explain why this limit does not exist.

36. *. limx→2

x2 − 4

x2 − 2x

37. limx→5

(x− 5)2

x+ 5

38. Evaluate limt→ 1

2

13t2

+ 1t2−1

2t− 1.

39. Evaluate limx→0

(3 +|x|x

).

40. Evaluate limd→−4

|3d+ 12|d+ 4

41. Evaluate limx→0

5x− 9

|x|+ 2.

42. Suppose limx→−1

f(x) = −1. Evaluate limx→−1

xf(x) + 3

2f(x) + 1.

43. *. Find the value of the constant a for which limx→−2

x2 + ax+ 3

x2 + x− 2exists.

44. Suppose f(x) = 2x and g(x) = 1x. Evaluate the following limits.

a limx→0

f(x)

b limx→0

g(x)

c limx→0

f(x)g(x)

d limx→0

f(x)

g(x)

e limx→2

[f(x) + g(x)]

74

Page 91: CLP-1 Differential Calculus

LIMITS 1.4 CALCULATING LIMITS WITH LIMIT LAWS

f limx→0

f(x) + 1

g(x+ 1)

Exercises — Stage 3

45. The curve y = f(x) is shown in the graph below. Sketch the graph of

y =1

f(x).

x

y

1

1y = f(x)

46. The graphs of functions f(x) and g(x) are shown in the graphs below. Use

these to sketch the graph off(x)

g(x).

x

y

1

1y = f(x)

x

y

1

1 y = g(x)

47. Suppose the position of a white ball, at time t, is given by s(t), and theposition of a red ball is given by 2s(t). Using the definition from Section 1.2of the velocity of a particle, and the limit laws from this section, answer thefollowing question: if the white ball has velocity 5 at time t = 1, what is thevelocity of the red ball?

75

Page 92: CLP-1 Differential Calculus

LIMITS 1.5 LIMITS AT INFINITY

48. Let f(x) = 1x

and g(x) = −1x

.

a Evaluate limx→0

f(x) and limx→0

g(x).

b Evaluate limx→0

[f(x) + g(x)]

c Is it always true that limx→a

[f(x) + g(x)] = limx→a

f(x) + limx→a

g(x)?

49. Suppose

f(x) =

x2 + 3 if x > 0

0 if x = 0

x2 − 3 if x < 0

a Evaluate limx→0−

f(x).

b Evaluate limx→0+

f(x).

c Evaluate limx→0

f(x).

50. Suppose

f(x) =

x2 + 8x+ 16

x2 + 30x− 4if x > −4

x3 + 8x2 + 16x if x ≤ −4

a Evaluate limx→−4−

f(x).

b Evaluate limx→−4+

f(x).

c Evaluate limx→−4

f(x).

1.5q Limits at Infinity

1.5.1 tt Limits at Infinity

Up until this point we have discussed what happens to a function as we move itsinput x closer and closer to a particular point a. For a great many applications oflimits we need to understand what happens to a function when its input becomes

76

Page 93: CLP-1 Differential Calculus

LIMITS 1.5 LIMITS AT INFINITY

extremely large — for example what happens to a population at a time far in thefuture.

The definition of a limit at infinity has a similar flavour to the definition of limitsat finite points that we saw above, but the details are a little different. We also needto distinguish between positive and negative infinity. As x becomes very large andpositive it moves off towards +∞ but when it becomes very large and negative itmoves off towards −∞.

Again we give an informal definition; the full formal definition can be found in(the optional) Section 1.8 near the end of this chapter.

Definition 1.5.1 Limits at infinity — informal.

We write

limx→∞

f(x) = L

when the value of the function f(x) gets closer and closer to L as we make xlarger and larger and positive.Similarly we write

limx→−∞

f(x) = L

when the value of the function f(x) gets closer and closer to L as we make xlarger and larger and negative.

Example 1.5.2 Limits to +∞ and −∞.

Consider the two functions depicted below

The dotted horizontal lines indicate the behaviour as x becomes very large. Thefunction on the left has limits as x → ∞ and as x → −∞ since the function “settlesdown” to a particular value. On the other hand, the function on the right does nothave a limit as x→ −∞ since the function just keeps getting bigger and bigger.

Just as was the case for limits as x→ awe will start with two very simple buildingblocks and build other limits from those.

77

Page 94: CLP-1 Differential Calculus

LIMITS 1.5 LIMITS AT INFINITY

Theorem 1.5.3

Let c ∈ R then the following limits hold

limx→∞

c = c limx→−∞

c = c

limx→∞

1

x= 0 lim

x→−∞1

x= 0

Again, these limits interact nicely with standard arithmetic:

Theorem 1.5.4 Arithmetic of limits at infinity.

Let f(x), g(x) be two functions for which the limits

limx→∞

f(x) = F limx→∞

g(x) = G

exist. Then the following limits hold

limx→∞

f(x)± g(x) = F ±Glimx→∞

f(x)g(x) = FG

limx→∞

f(x)

g(x)=F

Gprovided G 6= 0

and for real numbers p

limx→∞

f(x)p = F p provided F p and f(x)p are defined for all x

The analogous results hold for limits to −∞.

Note that, as was the case in Theorem 1.4.9, we need a little extra care with pow-ers of functions. We must avoid taking square roots of negative numbers, or indeedany even root of a negative number 1.

Hence we have for all rational r > 0

limx→∞

1

xr= 0

but we have to be careful with

limx→−∞

1

xr= 0

1 To be more precise, there is no real number x so that xeven power is a negative number. Hence wecannot take the even-root of a negative number and express it as a real number. This is preciselywhat complex numbers allow us to do, but alas there is not space in the course for us to explorethem.

78

Page 95: CLP-1 Differential Calculus

LIMITS 1.5 LIMITS AT INFINITY

This is only true if the denominator of r is not an even number 2.For example

• limx→∞

1

x1/2= 0, but lim

x→−∞1

x1/2does not exist, because x1/2 is not defined for x < 0.

• On the other hand, x4/3 is defined for negative values of x and limx→−∞

1

x4/3= 0.

Our first application of limits at infinity will be to examine the behaviour of arational function for very large x. To do this we use a “trick”.

Example 1.5.5 limx→∞

x2 − 3x+ 4

3x2 + 8x+ 1.

Compute the following limit:

limx→∞

x2 − 3x+ 4

3x2 + 8x+ 1

As x becomes very large, it is the x2 term that will dominate in both the numeratorand denominator and the other bits become irrelevant. That is, for very large x, x2 ismuch much larger than x or any constant. So we pull out these dominant parts

x2 − 3x+ 4

3x2 + 8x+ 1=x2(1− 3

x+ 4

x2

)x2(3 + 8

x+ 1

x2

)=

1− 3x

+ 4x2

3 + 8x

+ 1x2

remove the common factors

limx→∞

x2 − 3x+ 4

3x2 + 8x+ 1= lim

x→∞

1− 3x

+ 4x2

3 + 8x

+ 1x2

=

limx→∞

(1− 3

x+

4

x2

)limx→∞

(3 +

8

x+

1

x2

) arithmetic of limits

=limx→∞

1− limx→∞

3

x+ lim

x→∞4

x2

limx→∞

3 + limx→∞

8

x+ lim

x→∞1

x2

more arithmetic of limits

=1 + 0 + 0

3 + 0 + 0=

1

3

The following one gets a little harder

2 where we write r = pq with p, q integers with no common factors. For example, r = 6

14 should bewritten as r = 3

7 when considering this rule.

79

Page 96: CLP-1 Differential Calculus

LIMITS 1.5 LIMITS AT INFINITY

Example 1.5.6 Be careful of limits involving roots.

Find the limit as x→∞ of√

4x2+15x−1

We use the same trick — try to work out what is the biggest term in the numeratorand denominator and pull it to one side.

• The denominator is dominated by 5x.

• The biggest contribution to the numerator comes from the 4x2 inside thesquare-root. When we pull x2 outside the square-root it becomes x, so thenumerator is dominated by x ·

√4 = 2x

• To see this more explicitly rewrite the numerator√

4x2 + 1 =√x2(4 + 1/x2) =

√x2√

4 + 1/x2 = x√

4 + 1/x2.

• Thus the limit as x→∞ is

limx→∞

√4x2 + 1

5x− 1= lim

x→∞x√

4 + 1/x2

x(5− 1/x)

= limx→∞

√4 + 1/x2

5− 1/x

=2

5

Now let us also think about the limit of the same function,√

4x2+15x−1

, as x → −∞.There is something subtle going on because of the square-root. First consider thefunction 3

h(t) =√t2

Evaluating this at t = 7 gives

h(7) =√

72 =√

49 = 7

We’ll get much the same thing for any t ≥ 0. For any t ≥ 0, h(t) =√t2 returns exactly

t. However now consider the function at t = −3

h(−3) =√

(−3)2 =√

9 = 3 = −(−3)

that is the function is returning −1 times the input.This is because when we defined √ , we defined it to be the positive square-root.

i.e. the function√t can never return a negative number. So being more careful

h(t) =√t2 = |t|

3 Just to change things up let’s use t and h(t) instead of the ubiquitous x and f(x).

80

Page 97: CLP-1 Differential Calculus

LIMITS 1.5 LIMITS AT INFINITY

Where the |t| is the absolute value of t. You are perhaps used to thinking of absolutevalue as “remove the minus sign”, but this is not quite correct. Let’s sketch thefunction

It is a piecewise function defined by

|x| =x x ≥ 0

−x x < 0

Hence our function h(t) is really

h(t) =√t2 =

t t ≥ 0

−t t < 0

So that when we evaluate h(−7) it is

h(−7) =√

(−7)2 =√

49 = 7 = −(−7)

We are now ready to examine the limit as x→ −∞ in our previous example. Mostlyit is copy and paste from above.

Example 1.5.7 Be careful of limits involving roots — continued.

Find the limit as x→ −∞ of√

4x2+15x−1

We use the same trick — try to work out what is the biggest term in the numeratorand denominator and pull it to one side. Since we are taking the limit as x → −∞we should think of x as a large negative number.

• The denominator is dominated by 5x.

• The biggest contribution to the numerator comes from the 4x2 inside thesquare-root. When we pull the x2 outside a square-root it becomes |x| = −x(since we are taking the limit as x → −∞), so the numerator is dominated by−x ·

√4 = −2x

• To see this more explicitly rewrite the numerator√

4x2 + 1 =√x2(4 + 1/x2) =

√x2√

4 + 1/x2

= |x|√

4 + 1/x2 and since x < 0 we have

81

Page 98: CLP-1 Differential Calculus

LIMITS 1.5 LIMITS AT INFINITY

Example 1.5.7

= −x√

4 + 1/x2

• Thus the limit as x→ −∞ is

limx→−∞

√4x2 + 1

5x− 1= lim

x→−∞−x√

4 + 1/x2

x(5− 1/x)

= limx→−∞

−√

4 + 1/x2

5− 1/x

= −2

5

So the limit as x → −∞ is almost the same but we gain a minus sign. This isdefinitely not the case in general — you have to think about each example separately.

Here is a sketch of the function in question.

Example 1.5.8 Do not try to add and subtract infinity.

Compute the following limit:

limx→∞

(x7/5 − x

)In this case we cannot use the arithmetic of limits to write this as

limx→∞

(x7/5 − x

)=(

limx→∞

x7/5)−(

limx→∞

x)

=∞−∞because the limits do not exist. We can only use the limit laws when the limits exist.So we should go back and think some more.When x is very large, x7/5 = x · x2/5 will be much larger than x, so the x7/5 term willdominate the x term. So factor out x7/5 and rewrite it as

x7/5 − x = x7/5

(1− 1

x2/5

)

82

Page 99: CLP-1 Differential Calculus

LIMITS 1.5 LIMITS AT INFINITY

Example 1.5.8

Consider what happens to each of the factors as x→∞

• For large x, x7/5 > x (this is actually true for any x > 1). In the limit as x→ +∞,x becomes arbitrarily large and positive, and x7/5 must be bigger still, so itfollows that

limx→∞

x7/5 = +∞.

• On the other hand, (1− x−2/5) becomes closer and closer to 1 — we can use thearithmetic of limits to write this as

limx→∞

(1− x−2/5) = limx→∞

1− limx→∞

x−2/5 = 1− 0 = 1

So the product of these two factors will be come larger and larger (and positive) as xmoves off to infinity. Hence we have

limx→∞

x7/5(1− 1/x2/5

)= +∞

But remember +∞ and −∞ are not numbers; the last equation in the example isshorthand for “the function becomes arbitrarily large”.

In the previous section we saw that finite limits and arithmetic interact verynicely (see Theorems 1.4.3 and 1.4.9). This enabled us to compute the limits of morecomplicated function in terms of simpler ones. When limits of functions go to plusor minus infinity we are quite a bit more restricted in what we can deduce. The nexttheorem states some results concerning the sum, difference, ratio and product of in-finite limits — unfortunately in many cases we cannot make general statements andthe results will depend on the details of the problem at hand.

Theorem 1.5.9 Arithmetic of infinite limits.

Let a, c,H ∈ R and let f, g, h be functions defined in an interval around a (butthey need not be defined at x = a), so that

limx→a

f(x) = +∞ limx→a

g(x) = +∞ limx→a

h(x) = H

• limx→a

(f(x) + g(x)) = +∞

• limx→a

(f(x) + h(x)) = +∞

• limx→a

(f(x)− g(x)) undetermined

• limx→a

(f(x)− h(x)) = +∞

83

Page 100: CLP-1 Differential Calculus

LIMITS 1.5 LIMITS AT INFINITY

• limx→a

cf(x) =

+∞ c > 0

0 c = 0

−∞ c < 0

• limx→a

(f(x) · g(x)) = +∞.

• limx→a

f(x)h(x) =

+∞ H > 0

−∞ H < 0

undetermined H = 0

• limx→a

f(x)

g(x)undetermined

• limx→a

f(x)

h(x)=

+∞ H > 0

−∞ H < 0

undetermined H = 0

• limx→a

h(x)

f(x)= 0

• limx→a

f(x)p =

+∞ p > 0

0 p < 0

1 p = 0

Note that by “undetermined” we mean that the limit may or may not exist, butcannot be determined from the information given in the theorem. See Example 1.4.7for an example of what we mean by “undetermined”. Additionally consider thefollowing example.

Example 1.5.10 Be careful with the arithmetic of infinite limits.

Consider the following 3 functions:

f(x) = x−2 g(x) = 2x−2 h(x) = x−2 − 1.

Their limits as x→ 0 are:

limx→0

f(x) = +∞ limx→0

g(x) = +∞ limx→0

h(x) = +∞.

Say we want to compute the limit of the difference of two of the above functions asx→ 0. Then the previous theorem cannot help us. This is not because it is too weak,

84

Page 101: CLP-1 Differential Calculus

LIMITS 1.5 LIMITS AT INFINITY

Example 1.5.10

rather it is because the difference of two infinite limits can be, either plus infinity,minus infinity or some finite number depending on the details of the problem. Forexample,

limx→0

(f(x)− g(x)) = limx→0−x−2 = −∞

limx→0

(f(x)− h(x)) = limx→0

1 = 1

limx→0

(g(x)− h(x)) = limx→0

x−2 + 1 = +∞

1.5.2 tt Exercises

Exercises — Stage 1

1. Give a polynomial f(x) with the property that both limx→∞

f(x) and

limx→−∞

f(x) are (finite) real numbers.

2. Give a polynomial f(x) that satisfies limx→∞

f(x) 6= limx→−∞

f(x).

Exercises — Stage 2

3. Evaluate limx→∞

2−x

4. Evaluate limx→∞

2x

5. Evaluate limx→−∞

2x

6. Evaluate limx→−∞

cosx

7. Evaluate limx→∞

x− 3x5 + 100x2.

8. Evaluate limx→∞

√3x8 + 7x4 + 10

x4 − 2x2 + 1.

9. *. limx→∞

[√x2 + 5x−

√x2 − x

]

85

Page 102: CLP-1 Differential Calculus

LIMITS 1.5 LIMITS AT INFINITY

10. *. Evaluate limx→−∞

3x√4x2 + x− 2x

.

11. *. Evaluate limx→−∞

1− x− x2

2x2 − 7.

12. *. Evaluate limx→∞

(√x2 + x− x

)13. *. Evaluate lim

x→+∞5x2 − 3x+ 1

3x2 + x+ 7.

14. *. Evaluate limx→+∞

√4x+ 2

3x+ 4.

15. *. Evaluate limx→+∞

4x3 + x

7x3 + x2 − 2.

16. Evaluate limx→−∞

3√x2 + x− 4

√x4 + 5

x+ 1

17. *. Evaluate limx→+∞

5x2 + 10

3x3 + 2x2 + x.

18. Evaluate limx→−∞

x+ 1√x2

.

19. Evaluate limx→∞

x+ 1√x2

20. *. Find the limit limx→−∞

sin

2

|x|x

)+

1

x.

21. *. Evaluate limx→−∞

3x+ 5√x2 + 5− x

.

22. *. Evaluate limx→−∞

5x+ 7√4x2 + 15− x

23. Evaluate limx→−∞

3x7 + x5 − 15

4x2 + 32x.

24. *. Evaluate limn→∞

(√n2 + 5n− n

).

25. Evaluate lima→0+

a2 − 1a

a− 1.

86

Page 103: CLP-1 Differential Calculus

LIMITS 1.6 CONTINUITY

26. Evaluate limx→3

2x+ 81

x−3+ 1

x2−9

.

Exercises — Stage 3

27. Give a rational function f(x) with the properties that limx→∞

f(x) 6=lim

x→−∞f(x), and both limits are (finite) real numbers.

28. Suppose the concentration of a substance in your body t hours after injec-tion is given by some formula c(t), and lim

t→∞c(t) 6= 0. What kind of substance

might have been injected?

1.6q Continuity

1.6.1 tt Continuity

We have seen that computing the limits some functions — polynomials and rationalfunctions — is very easy because

limx→a

f(x) = f(a).

That is, the the limit as x approaches a is just f(a). Roughly speaking, the reason wecan compute the limit this way is that these functions do not have any abrupt jumpsnear a.

Many other functions have this property, sin(x) for example. A function with thisproperty is called “continuous” and there is a precise mathematical definition for it.If you do not recall interval notation, then now is a good time to take a quick lookback at Definition 0.3.5.

Definition 1.6.1

A function f(x) is continuous at a if

limx→a

f(x) = f(a)

If a function is not continuous at a then it is said to be discontinuous at a.When we write that f is continuous without specifying a point, then typicallythis means that f is continuous at a for all a ∈ R.When we write that f(x) is continuous on the open interval (a, b) then thefunction is continuous at every point c satisfying a < c < b.

So if a function is continuous at x = a we immediately know that

87

Page 104: CLP-1 Differential Calculus

LIMITS 1.6 CONTINUITY

• f(a) exists

• limx→a−

exists and is equal to f(a), and

• limx→a+

exists and is equal to f(a).

1.6.2 tt Quick Aside — One-sided Continuity

Notice in the above definition of continuity on an interval (a, b) we have carefullyavoided saying anything about whether or not the function is continuous at the end-points of the interval — i.e. is f(x) continuous at x = a or x = b. This is becausetalking of continuity at the endpoints of an interval can be a little delicate.

In many situations we will be given a function f(x) defined on a closed interval[a, b]. For example, we might have:

f(x) =x+ 1

x+ 2for x ∈ [0, 1].

For any 0 ≤ x ≤ 1 we know the value of f(x). However for x < 0 or x > 1 we knownothing about the function — indeed it has not been defined.

So now, consider what it means for f(x) to be continuous at x = 0. We need tohave

limx→0

f(x) = f(0),

however this implies that the one-sided limits

limx→0+

f(x) = f(0) and limx→0−

f(x) = f(0)

Now the first of these one-sided limits involves examining the behaviour of f(x)for x > 0. Since this involves looking at points for which f(x) is defined, this issomething we can do. On the other hand the second one-sided limit requires us tounderstand the behaviour of f(x) for x < 0. This we cannot do because the functionhasn’t been defined for x < 0.

One way around this problem is to generalise the idea of continuity to one-sidedcontinuity, just as we generalised limits to get one-sided limits.

Definition 1.6.2

A function f(x) is continuous from the right at a if

limx→a+

f(x) = f(a).

Similarly a function f(x) is continuous from the left at a if

limx→a−

f(x) = f(a)

Using the definition of one-sided continuity we can now define what it means fora function to be continuous on a closed interval.

88

Page 105: CLP-1 Differential Calculus

LIMITS 1.6 CONTINUITY

Definition 1.6.3

A function f(x) is continuous on the closed interval [a, b] when

• f(x) is continuous on (a, b),

• f(x) is continuous from the right at a, and

• f(x) is continuous from the left at b.

Note that the last two condition are equivalent to

limx→a+

f(x) = f(a) and limx→b−

f(x) = f(b).

1.6.3 tt Back to the Main Text

We already know from our work above that polynomials are continuous, and thatrational functions are continuous at all points in their domains — i.e. where theirdenominators are non-zero. As we did for limits, we will see that continuity interacts“nicely” with arithmetic. This will allow us to construct complicated continuousfunctions from simpler continuous building blocks (like polynomials).

But first, a few examples. . .

Example 1.6.4 Simple continuous and discontinuous functions.

Consider the functions drawn below

These are

f(x) =

x x < 1

x+ 2 x ≥ 1

g(x) =

1/x2 x 6= 0

0 x = 0

89

Page 106: CLP-1 Differential Calculus

LIMITS 1.6 CONTINUITY

Example 1.6.4

h(x) =

x3−x2x−1

x 6= 1

0 x = 1

Determine where they are continuous and discontinuous:

• When x < 1 then f(x) is a straight line (and so a polynomial) and so it iscontinuous at every point x < 1. Similarly when x > 1 the function is a straightline and so it is continuous at every point x > 1. The only point which mightbe a discontinuity is at x = 1. We see that the one sided limits are different.Hence the limit at x = 1 does not exist and so the function is discontinuous atx = 1.

But note that that f(x) is continuous from one side — which?

• The middle case is much like the previous one. When x 6= 0 the g(x) is arational function and so is continuous everywhere on its domain (which is allreals except x = 0). Thus the only point where g(x) might be discontinuous isat x = 0. We see that neither of the one-sided limits exist at x = 0, so the limitdoes not exist at x = 0. Hence the function is discontinuous at x = 0.

• We have seen the function h(x) before. By the same reasoning as above, weknow it is continuous except at x = 1 which we must check separately.

By definition of h(x), h(1) = 0. We must compare this to the limit as x→ 1. Wedid this before.

x3 − x2

x− 1=x2(x− 1)

x− 1= x2

So limx→1x3−x2x−1

= limx→1 x2 = 1 6= h(1). Hence h is discontinuous at x = 1.

This example illustrates different sorts of discontinuities:

• The function f(x) has a “jump discontinuity” because the function “jumps”from one finite value on the left to another value on the right.

• The second function, g(x), has an “infinite discontinuity” since lim f(x) = +∞.

• The third function, h(x), has a “removable discontinuity” because we couldmake the function continuous at that point by redefining the function at thatpoint. i.e. setting h(1) = 1. That is

new function h(x) =

x3−x2x−1

x 6= 1

1 x = 1

Showing a function is continuous can be a pain, but just as the limit laws help uscompute complicated limits in terms of simpler limits, we can use them to show thatcomplicated functions are continuous by breaking them into simpler pieces.

90

Page 107: CLP-1 Differential Calculus

LIMITS 1.6 CONTINUITY

Theorem 1.6.5 Arithmetic of continuity.

Let a, c ∈ R and let f(x) and g(x) be functions that are continuous at a. Thenthe following functions are also continuous at x = a:

• f(x) + g(x) and f(x)− g(x),

• cf(x) and f(x)g(x), and

• f(x)g(x)

provided g(a) 6= 0.

Above we stated that polynomials and rational functions are continuous (beingcareful about domains of rational functions — we must avoid the denominators be-ing zero) without making it a formal statement. This is easily fixed. . .

Lemma 1.6.6

Let c ∈ R. The functions

f(x) = x g(x) = c

are continuous everywhere on the real line

This isn’t quite the result we wanted (that’s a couple of lines below) but it is asmall result that we can combine with the arithmetic of limits to get the result wewant. Such small helpful results are called “lemmas” and they will arise more as wego along.

Now since we can obtain any polynomial and any rational function by carefullyadding, subtracting, multiplying and dividing the functions f(x) = x and g(x) = c,the above lemma combines with the “arithmetic of continuity” theorem to give usthe result we want:

Theorem 1.6.7 Continuity of polynomials and rational functions.

Every polynomial is continuous everywhere. Similarly every rational functionis continuous except where its denominator is zero (i.e. on all its domain).

With some more work this result can be extended to wider families of functions:

91

Page 108: CLP-1 Differential Calculus

LIMITS 1.6 CONTINUITY

Theorem 1.6.8

The following functions are continuous everywhere in their domains

• polynomials, rational functions

• roots and powers

• trig functions and their inverses

• exponential and the logarithm

We haven’t encountered inverse trigonometric functions, nor exponential func-tions or logarithms, but we will see them in the next chapter. For the moment, justfile the information away.

Using a combination of the above results you can show that many complicatedfunctions are continuous except at a few points (usually where a denominator isequal to zero).

Example 1.6.9 Continuity of sin(x)2+cos(x)

.

Where is the function f(x) = sin(x)2+cos(x)

continuous?We just break things down into pieces and then put them back together keeping trackof where things might go wrong.

• The function is a ratio of two pieces — so check if the numerator is continuous,the denominator is continuous, and if the denominator might be zero.

• The numerator is sin(x) which is “continuous on its domain” according to oneof the above theorems. Its domain is all real numbers a, so it is continuouseverywhere. No problems here.

• The denominator is the sum of 2 and cos(x). Since 2 is a constant it is continu-ous everywhere. Similarly (we just checked things for the previous point) weknow that cos(x) is continuous everywhere. Hence the denominator is contin-uous.

• So we just need to check if the denominator is zero. One of the facts that weshould know b is that

−1 ≤ cos(x) ≤ 1

and so by adding 2 we get

1 ≤ 2 + cos(x) ≤ 3

Thus no matter what value of x, 2 + cos(x) ≥ 1 and so cannot be zero.

92

Page 109: CLP-1 Differential Calculus

LIMITS 1.6 CONTINUITY

Example 1.6.9

• So the numerator is continuous, the denominator is continuous and nowherezero, so the function is continuous everywhere.

If the function were changed tosin(x)

x2 − 5x+ 6much of the same reasoning can be used.

Being a little terse we could answer with:

• Numerator and denominator are continuous.

• Since x2 − 5x+ 6 = (x− 2)(x− 3) the denominator is zero when x = 2, 3.

• So the function is continuous everywhere except possibly at x = 2, 3. In orderto verify that the function really is discontinuous at those points, it suffices toverify that the numerator is non-zero at x = 2, 3. Indeed we know that sin(x) iszero only when x = nπ (for any integer n). Hence sin(2), sin(3) 6= 0. Thus thenumerator is non-zero, while the denominator is zero and hence x = 2, 3 reallyare points of discontinuity.

Note that this example raises a subtle point about checking continuity when nu-merator and denominator are simultaneously zero. There are quite a few possibleoutcomes in this case and we need more sophisticated tools to adequately analysethe behaviour of functions near such points. We will return to this question later inthe text after we have developed Taylor expansions (see Section 3.4).

a Remember that sin and cos are defined on all real numbers, so tan(x) = sin(x)/ cos(x) is contin-uous everywhere except where cos(x) = 0. This happens when x = π

2 + nπ for any integer n.If you cannot remember where tan(x) “blows up” or sin(x) = 0 or cos(x) = 0 then you shoulddefinitely revise trigonometric functions. Come to think of it — just revise them anyway.

b If you do not know this fact then you should revise trigonometric functions. See the previousfootnote.

So we know what happens when we add subtract multiply and divide, whatabout when we compose functions? Well - limits and compositions work nicelywhen things are continuous.

Theorem 1.6.10 Compositions and continuity.

If f is continuous at b and limx→a

g(x) = b then limx→a

f(g(x)) = f(b). I.e.

limx→a

f (g(x)) = f(

limx→a

g(x))

Hence if g is continuous at a and f is continuous at g(a) then the compositefunction (f g)(x) = f(g(x)) is continuous at a.

93

Page 110: CLP-1 Differential Calculus

LIMITS 1.6 CONTINUITY

So when we compose two continuous functions we get a new continuous func-tion.

We can put this to use

Example 1.6.11 Continuity of composed functions.

Where are the following functions continuous?

f(x) = sin(x2 + cos(x)

)g(x) =

√sin(x)

Our first step should be to break the functions down into pieces and study them.When we put them back together we should be careful of dividing by zero, or fallingoutside the domain.

• The function f(x) is the composition of sin(x) with x2 + cos(x).

• These pieces, sin(x), x2, cos(x) are continuous everywhere.

• So the sum x2 + cos(x) is continuous everywhere

• And hence the composition of sin(x) and x2 +cos(x) is continuous everywhere.

The second function is a little trickier.

• The function g(x) is the composition of√x with sin(x).

•√x is continuous on its domain x ≥ 0.

• sin(x) is continuous everywhere, but it is negative in many places.

• In order for g(x) to be defined and continuous we must restrict x so thatsin(x) ≥ 0.

• Recall the graph of sin(x):

Hence sin(x) ≥ 0 when x ∈ [0, π] or x ∈ [2π, 3π] or x ∈ [−2π,−π] or. . . . To bemore precise sin(x) is positive when x ∈ [2nπ, (2n+ 1)π] for any integer n.

• Hence g(x) is continuous when x ∈ [2nπ, (2n+ 1)π] for any integer n.

94

Page 111: CLP-1 Differential Calculus

LIMITS 1.6 CONTINUITY

Continuous functions are very nice (mathematically speaking). Functions fromthe “real world” tend to be continuous (though not always). The key aspect thatmakes them nice is the fact that they don’t jump about.

The absence of such jumps leads to the following theorem which, while it canbe quite confusing on first glance, actually says something very natural — obviouseven. It says, roughly speaking, that, as you draw the graph y = f(x) starting atx = a and ending at x = b, y changes continuously from y = f(a) to y = f(b), withno jumps, and consequently y must take every value between f(a) and f(b) at leastonce. We’ll start by just giving the precise statement and then we’ll explain it indetail.

Theorem 1.6.12 Intermediate value theorem (IVT).

Let a < b and let f be a function that is continuous at all points a ≤ x ≤ b. If Yis any number between f(a) and f(b) then there exists some number c ∈ [a, b]so that f(c) = Y .

Like the ε−δ definition of limits 1, we should break this theorem down into pieces.Before we do that, keep the following pictures in mind.

Now the break-down

• Let a < b and let f be a function that is continuous at all points a ≤ x ≤ b. —This is setting the scene. We have a, b with a < b (we can safely assume theseto be real numbers). Our function must be continuous at all points between aand b.

• if Y is any number between f(a) and f(b) — Now we need another numberY and the only restriction on it is that it lies between f(a) and f(b). That is, iff(a) ≤ f(b) then f(a) ≤ Y ≤ f(b). Or if f(a) ≥ f(b) then f(a) ≥ Y ≥ f(b).So notice that Y could be equal to f(a) or f(b) — if we wanted to avoid thatpossibility, then we would normally explicitly say Y 6= f(a), f(b) or we wouldwrite that Y is strictly between f(a) and f(b).

• there exists some number c ∈ [a, b] so that f(c) = Y — so if we satisfy all of theabove conditions, then there has to be some real number c lying between a andb so that when we evaluate f(c) it is Y .

1 The interested student is invited to take a look at the optional Section 1.7

95

Page 112: CLP-1 Differential Calculus

LIMITS 1.6 CONTINUITY

So that breaks down the proof statement by statement, but what does it actuallymean?

• Draw any continuous function you like between a and b — it must be continu-ous.

• The function takes the value f(a) at x = a and f(b) at x = b — see the left-handfigure above.

• Now we can pick any Y that lies between f(a) and f(b) — see the middlefigure above. The IVT 2 tells us that there must be some x-value that whenplugged into the function gives us Y . That is, there is some c between a and bso that f(c) = Y . We can also interpret this graphically; the IVT tells us that thehorizontal straight line y = Y must intersect the graph y = f(x) at some point(c, Y ) with a ≤ c ≤ b.

• Notice that the IVT does not tell us how many such c-values there are, just thatthere is at least one of them. See the right-hand figure above. For that particularchoice of Y there are three different c values so that f(c1) = f(c2) = f(c3) = Y .

This theorem says that if f(x) is a continuous function on all of the interval a ≤ x ≤ bthen as x moves from a to b, f(x) takes every value between f(a) and f(b) at leastonce. To put this slightly differently, if f were to avoid a value between f(a) and f(b)then f cannot be continuous on [a, b].

It is not hard to convince yourself that the continuity of f is crucial to the IVT.Without it one can quickly construct examples of functions that contradict the theo-rem. See the figure below for a few non-continuous examples:

In the left-hand example we see that a discontinuous function can “jump” overthe Y -value we have chosen, so there is no x-value that makes f(x) = Y . The right-hand example demonstrates why we need to be be careful with the ends of the in-terval. In particular, a function must be continuous over the whole interval [a, b]including the end-points of the interval. If we only required the function to be con-tinuous on (a, b) (so strictly between a and b) then the function could “jump” overthe Y -value at a or b.

If you are still confused then here is a “real-world” example

2 Often with big important useful theorems like this one, writing out the full name again and againbecomes tedious, so we abbreviate it. Such abbreviations are okay provided the reader knowsthis is what you are doing, so the first time you use an abbreviation you should let the readerknow. Much like we are doing here in this footnote: : “IVT” stands for “intermediate valuetheorem”, which is Theorem 1.6.12.

96

Page 113: CLP-1 Differential Calculus

LIMITS 1.6 CONTINUITY

Example 1.6.13 The IVT in the “real world”.

You are climbing the Grouse-grind a with a friend — call him Bob. Bob was eagerand started at 9am. Bob, while very eager, is also very clumsy; he sprained his anklesomewhere along the path and has stopped moving at 9:21am and is just sitting b

enjoying the view. You get there late and start climbing at 10am and being quite fityou get to the top at 11am. The IVT implies that at some time between 10am and11am you meet up with Bob.You can translate this situation into the form of the IVT as follows. Let t be timeand let a = 10am and b = 11am. Let g(t) be your distance along the trail. Hence c

g(a) = 0 and g(b) = 2.9km. Since you are a mortal, your position along the trail is acontinuous function — no helicopters or teleportation or. . . We have no idea whereBob is sitting, except that he is somewhere between g(a) and g(b), call this point Y .The IVT guarantees that there is some time c between a and b (so between 10am and11am) with g(c) = Y (and your position will be the same as Bob’s).

a If you don’t know it then google it.b Hopefully he remembered to carry something warm.c It’s amazing what facts you can find on Wikipedia.

Aside from finding Bob sitting by the side of the trail, one of the most importantapplications of the IVT is determining where a function is zero. For quadratics weknow (or should know) that

ax2 + bx+ c = 0 when x =−b±

√b2 − 4ac

2a

While the Babylonians could (mostly, but not quite) do the above, the correspondingformula for solving a cubic is uglier and that for a quartic is uglier still. One of themost famous results in mathematics demonstrates that no such formula exists forquintics or higher degree polynomials 3.

So even for polynomials we cannot, in general, write down explicit formulaefor their zeros and have to make do with numerical approximations — i.e. writedown the root as a decimal expansion to whatever precision we desire. For morecomplicated functions we have no choice — there is no reason that the zeros shouldbe expressible as nice neat little formulas. At the same time, finding the zeros of a

3 The similar (but uglier) formula for solving cubics took until the 15th century and the work ofdel Ferro and Cardano (and Cardano’s student Ferrari). A similar (but even uglier) formulafor quartics was also found by Ferrari. The extremely famous Abel-Ruffini Theorem (nearly byRuffini in the late 18th century and completely by Abel in early 19th century) demonstrates thata similar formula for the zeros of a quintic does not exist. Note that the theorem does not say thatquintics do not have zeros; rather it says that the zeros cannot in general be expressed using afinite combination of addition, multiplication, division, powers and roots. The interested studentshould also look up Évariste Galois and his contributions to this area.

97

Page 114: CLP-1 Differential Calculus

LIMITS 1.6 CONTINUITY

function:

f(x) = 0

or solving equations of the form 4

g(x) = h(x)

can be a crucial step in many mathematical proofs and applications.For this reason there is a considerable body of mathematics which focuses just on

finding the zeros of functions. The IVT provides a very simple way to “locate” thezeros of a function. In particular, if we know a continuous function is negative at apoint x = a and positive at another point x = b, then there must (by the IVT) be apoint x = c between a and b where f(c) = 0.

Consider the leftmost of the above figures. It depicts a continuous function thatis negative at x = a and positive at x = b. So choose Y = 0 and apply the IVT —there must be some c with a ≤ c ≤ b so that f(c) = Y = 0. While this doesn’t tell us cexactly, it does give us bounds on the possible positions of at least one zero — theremust be at least one c obeying a ≤ c ≤ b.

See middle figure. To get better bounds we could test a point half-way betweena and b. So set a′ = a+b

2. In this example we see that f(a′) is negative. Applying the

IVT again tells us there is some c between a′ and b so that f(c) = 0. Again — wedon’t have c exactly, but we have halved the range of values it could take.

Look at the rightmost figure and do it again — test the point half-way betweena′ and b. In this example we see that f(b′) is positive. Applying the IVT tells us thatthere is some c between a′ and b′ so that f(c) = 0. This new range is a quarter ofthe length of the original. If we keep doing this process the range will halve eachtime until we know that the zero is inside some tiny range of possible values. Thisprocess is called the bisection method.

Consider the following zero-finding example

Example 1.6.14 Show that f(x) = x− 1 + sin(πx/2) has a zero.

Show that the function f(x) = x− 1 + sin(πx/2) has a zero in 0 ≤ x ≤ 1.This question has been set up nicely to lead us towards using the IVT; we are alreadygiven a nice interval on which to look. In general we might have to test a few points

4 In fact both of these are the same because we can write f(x) = g(x)− h(x) and then the zeros off(x) are exactly when g(x) = h(x).

98

Page 115: CLP-1 Differential Calculus

LIMITS 1.6 CONTINUITY

Example 1.6.14

and experiment a bit with a calculator before we can start narrowing down a range.Let us start by testing the endpoints of the interval we are given

f(0) = 0− 1 + sin(0) = −1 < 0

f(1) = 1− 1 + sin(π/2) = 1 > 0

So we know a point where f is positive and one where it is negative. So by the IVTthere is a point in between where it is zero.BUT in order to apply the IVT we have to show that the function is continuous, andwe cannot simply write

it is continuous

We need to explain to the reader why it is continuous. That is — we have to prove it.So to write up our answer we can put something like the following — keeping inmind we need to tell the reader what we are doing so they can follow along easily.

• We will use the IVT to prove that there is a zero in [0, 1].

• First we must show that the function is continuous.

Since x− 1 is a polynomial it is continuous everywhere.

The function sin(πx/2) is a trigonometric function and is also continuouseverywhere.

The sum of two continuous functions is also continuous, so f(x) is contin-uous everywhere.

• Let a = 0, b = 1, then

f(0) = 0− 1 + sin(0) = −1 < 0

f(1) = 1− 1 + sin(π/2) = 1 > 0

• The function is negative at x = 0 and positive at x = 1. Since the function iscontinuous we know there is a point c ∈ [0, 1] so that f(c) = 0.

Notice that though we have not used full sentences in our explanation here, we arestill using words. Your mathematics, unless it is very straight-forward computation,should contain words as well as symbols.The zero of this function is actually located at about x = 0.4053883559.

The bisection method is really just the idea that we can keep repeating the abovereasoning (with a calculator handy). Each iteration will tell us the location of thezero more precisely. The following example illustrates this.

99

Page 116: CLP-1 Differential Calculus

LIMITS 1.6 CONTINUITY

Example 1.6.15 Using the bisection method.

Use the bisection method to find a zero of

f(x) = x− 1 + sin(πx/2)

that lies between 0 and 1.So we start with the two points we worked out above:

• a = 0, b = 1 and

f(0) = −1

f(1) = 1

• Test the point in the middle x = 0+12

= 0.5

f(0.5) = 0.2071067813 > 0

• So our new interval will be [0, 0.5] since the function is negative at x = 0 andpositive at x = 0.5

Repeat

• a = 0, b = 0.5 where f(0) < 0 and f(0.5) > 0.

• Test the point in the middle x = 0+0.52

= 0.25

f(0.25) = −0.3673165675 < 0

• So our new interval will be [0.25, 0.5] since the function is negative at x = 0.25and positive at x = 0.5

Repeat

• a = 0.25, b = 0.5 where f(0.25) < 0 and f(0.5) > 0.

• Test the point in the middle x = 0.25+0.52

= 0.375

f(0.375) = −0.0694297669 < 0

• So our new interval will be [0.375, 0.5] since the function is negative at x = 0.375and positive at x = 0.5

Below is an illustration of what we have observed so far together with a plot of theactual function.

100

Page 117: CLP-1 Differential Calculus

LIMITS 1.6 CONTINUITY

Example 1.6.15

And one final iteration:

• a = 0.375, b = 0.5 where f(0.375) < 0 and f(0.5) > 0.

• Test the point in the middle x = 0.375+0.52

= 0.4375

f(0.4375) = 0.0718932843 > 0

• So our new interval will be [0.375, 0.4375] since the function is negative at x =0.375 and positive at x = 0.4375

So without much work we know the location of a zero inside a range of length0.0625 = 2−4. Each iteration will halve the length of the range and we keep going un-til we reach the precision we need, though it is much easier to program a computerto do it.

101

Page 118: CLP-1 Differential Calculus

LIMITS 1.6 CONTINUITY

1.6.4 tt Exercises

Exercises — Stage 1

1. Give an example of a function (you can write a formula, or sketch a graph)that has infinitely many infinite discontinuities.

2. When I was born, I was less than one meter tall. Now, I am more than onemeter tall. What is the conclusion of the Intermediate Value Theorem aboutmy height?

3. Give an example (by sketch or formula) of a function f(x), defined onthe interval [0, 2], with f(0) = 0, f(2) = 2, and f(x) never equal to 1.Why does this not contradict the Intermediate Value Theorem?

4. Is the following a valid statement?

Suppose f is a continuous function over [10, 20], f(10) = 13,and f(20) = −13. Then f has a zero between x = 10 andx = 20.

5. Is the following a valid statement?

Suppose f is a continuous function over [10, 20], f(10) = 13,and f(20) = −13. Then f(15) = 0.

6. Is the following a valid statement?

Suppose f is a function over [10, 20], f(10) = 13, and f(20) =−13, and f takes on every value between −13 and 13. Thenf is continuous.

7. Suppose f(t) is continuous at t = 5. True or false: t = 5 is in the domain off(t).

8. Suppose limt→5

f(t) = 17, and suppose f(t) is continuous at t = 5. True or

false: f(5) = 17.

9. Suppose limt→5

f(t) = 17. True or false: f(5) = 17.

10. Suppose f(x) and g(x) are continuous at x = 0, and let h(x) =xf(x)

g2(x) + 1.

What is limx→0+

h(x)?

Exercises — Stage 2

102

Page 119: CLP-1 Differential Calculus

LIMITS 1.6 CONTINUITY

11. Find a constant k so that the function

a(x) =

x sin

(1x

)when x 6= 0

k when x = 0

is continuous at x = 0.

12. Use the Intermediate Value Theorem to show that the function f(x) = x3 +x2 + x+ 1 takes on the value 12345 at least once in its domain.

13. *. Describe all points for which the function is continuous: f(x) =1

x2 − 1.

14. *. Describe all points for which this function is continuous: f(x) =1√

x2 − 1.

15. *. Describe all points for which this function is continuous:1√

1 + cos(x).

16. *. Describe all points for which this function is continuous: f(x) =1

sinx.

17. *. Find all values of c such that the following function is continuous at x = c:

f(x) =

8− cx if x ≤ c

x2 if x > c

Use the definition of continuity to justify your answer.

18. *. Find all values of c such that the following function is continuous every-where:

f(x) =

x2 + c x ≥ 0

cos cx x < 0

Use the definition of continuity to justify your answer.

19. *. Find all values of c such that the following function is continuous:

f(x) =

x2 − 4 if x < c

3x if x ≥ c .

Use the definition of continuity to justify your answer.

20. *. Find all values of c such that the following function is continuous:

f(x) =

6− cx if x ≤ 2c

x2 if x > 2c

103

Page 120: CLP-1 Differential Calculus

LIMITS 1.7 (OPTIONAL) — MAKING THE INFORMAL A LITTLE MORE FORMAL

Use the definition of continuity to justify your answer.

Exercises — Stage 3

21. Show that there exists at least one real number x satisfying sinx = x− 1

22. *. Show that there exists at least one real number c such that 3c = c2.23. *. Show that there exists at least one real number c such that 2 tan(c) = c+1.

24. *. Show that there exists at least one real number c such that√

cos(πc) =sin(2πc) + 1

2.

25. *. Show that there exists at least one real number c such that1

(cosπc)2=

c+3

2.

26. Use the intermediate value theorem to find an interval of length one con-taining a root of f(x) = x7 − 15x6 + 9x2 − 18x+ 15.

27. Use the intermediate value theorem to give a decimal approximation of 3√

7that is correct to at least two decimal places. You may use a calculator, butonly to add, subtract, multiply, and divide.

28. Suppose f(x) and g(x) are functions that are continuous over the interval[a, b], with f(a) ≤ g(a) and g(b) ≤ f(b). Show that there exists some c ∈ [a, b]with f(c) = g(c).

1.7q (Optional) — Making the Informal a Little More For-mal

As we noted above, the definition of limits that we have been working with wasquite informal and not mathematically rigorous. In this (optional) section we willwork to understand the rigorous definition of limits.

Here is the formal definition — we will work through it all very slowly and care-fully afterwards, so do not panic.

Definition 1.7.1

Let a ∈ R and let f(x) be a function defined everywhere in a neighbourhoodof a, except possibly at a. We say that

the limit as x approaches a of f(x) is L

or equivalently

as x approaches a, f(x) approaches L

104

Page 121: CLP-1 Differential Calculus

LIMITS 1.7 (OPTIONAL) — MAKING THE INFORMAL A LITTLE MORE FORMAL

and write

limx→a

f(x) = L

if and only if for every ε > 0 there exists δ > 0 so that

|f(x)− L| < ε whenever 0 < |x− a| < δ

Note that an equivalent way of writing this very last statement is

if 0 < |x− a| < δ then |f(x)− L| < ε.

This is quite a lot to take in, so let us break it down into pieces.

Definition 1.7.2 The typical 3 pieces of a definition.

Usually a definition can be broken down into three pieces.

• Scene setting — define symbols and any restrictions on the objects thatwe are talking about.

• Naming — state the name and any notation for the property or objectthat the definition is about.

• Properties and restrictions — this is the heart of the definition where weexplain to the reader what it is that the object (in our case a function) hasto do in order to satisfy the definition.

Let us go back to the definition and look at each of these pieces in turn.

• Setting things up — The first sentence of the definition is really just setting upthe picture. It is telling us what the definition is about and sorting out a fewtechnical details.

Let a ∈ R — This simply tells us that the symbol “a” is a real number 1. Let f(x) be a function — This is just setting the scene so that we under-

stand all of the terms and symbols. defined everywhere in a neighbourhood of a, except possibly at a— This is

just a technical requirement; we need our function to be defined in a littleregion 2 around a. The function doesn’t have to be defined everywhere,but it must be defined for all x-values a little less than a and a little morethan a. The definition does not care about what the function does outsidethis little window, nor does it care what happens exactly at a.

1 The symbol “∈” is read as “is an element of” — it is definitely not the same as e or ε or ε. If youdo not recognise “R” or understand the difference between R and R, then please go back andread Chapter 0 carefully.

2 The term “neighbourhood of a” means a small open interval around a— for example (a−0.01, a+0.01). Typically we don’t really care how big this little interval is.

105

Page 122: CLP-1 Differential Calculus

LIMITS 1.7 (OPTIONAL) — MAKING THE INFORMAL A LITTLE MORE FORMAL

• Names, phrases and notation — The next part of the definition is simply nam-ing the property we are discussing and tells us how to write it down. i.e. we aretalking about “limits” and we write them down using the symbols indicated.

• The heart of things — we explain this at length below, but for now we will givea quick explanation. Work on these two points. They are hard.

for all ε > 0 there exists δ > 0 — It is important we read this in order.It means that we can pick any positive number ε we want and there willalways be another positive number δ that is going to make what ever fol-lows be true. if 0 < |x− a| < δ then |f(x)− L| < ε — From the previous point we have

our two numbers — any ε > 0 then based on that choice of ε we havea positive number δ. The current statement says that whenever we havechosen x so that it is very close to a, then f(x) has to be very close to L.How close it “very close”? Well 0 < |x − a| < δ means that x has to bewithin a distance δ of a (but not exactly a) and similarly |f(x) − L| < εmeans that f(x) has to be within a distance ε of L.

That is the definition broken up into pieces which hopefully now make more sense,but what does it actually mean? Consider a function we saw earlier

f(x) =

2x x 6= 3

9 x = 3

and sketch it again:

We know (from our earlier work) that limx→3 f(x) = 6, so zoom in around (x, y) =(3, 6). To make this look more like our definition, we have a = 3 and L = 6.

• Pick some small number ε > 0 and highlight the horizontal strip of all points(x, y) for which |y − L| < ε. This means all the y-values have to satisfy L− ε <y < L+ ε.

• You can see that the graph of the function passes through this strip for somex-values close to a. What we need to be able to do is to pick a vertical strip ofx-values around a so that the function lies inside the horizontal strip.

• That is, we must find a small number δ > 0 so that for any x-value inside thevertical strip a− δ < x < a+ δ, except exactly at x = a, the value of the functionlies inside the horizontal strip, namely L− ε < y = f(x) < L+ ε.

106

Page 123: CLP-1 Differential Calculus

LIMITS 1.7 (OPTIONAL) — MAKING THE INFORMAL A LITTLE MORE FORMAL

• We see (pictorially) that we can do this. If we were to choose a smaller valueof ε making the horizontal strip narrower, it is clear that we can choose thevertical strip to be narrower. Indeed, it doesn’t matter how small we make thehorizontal strip, we will always be able to construct the second vertical strip.

The above is a pictorial argument, but we can quite easily make it into a mathe-matical one. We want to show the limit is 6. That means for any ε we need to find aδ so that when

3− δ < x < 3 + δ with x 6= 3 we have 6− ε < f(x) < 6 + ε

Now we note that when x 6= 3, we have f(x) = 2x and so

6− ε < f(x) < 6 + ε implies that 6− ε < 2x < 6 + ε

this nearly specifies a range of x values, we just need to divide by 2

3− ε/2 < x < 3 + ε/2

Hence if we choose δ = ε/2 then we get the desired inequality

3− δ < x < 3 + δ

i.e. — no matter what ε > 0 is chosen, if we put δ = ε/2 then when 3− δ < x < 3 + δwith x 6= 3 we will have 6− ε < f(x) < 6 + ε. This is exactly what we need to satisfythe definition of “limit” above.

The above work gives us the argument we need, but it still needs to be writtenup properly. We do this below.

Example 1.7.3 Formal limit of a simple function.

Find the limit as x→ 3 of the following function

f(x) =

2x x 6= 3

9 x = 3

107

Page 124: CLP-1 Differential Calculus

LIMITS 1.7 (OPTIONAL) — MAKING THE INFORMAL A LITTLE MORE FORMAL

Example 1.7.3

Proof. We will show that the limit is equal to 6. Let ε > 0 and δ = ε/2. Itremains to show that |f(x)− 6| < ε whenever |x− 3| < δ.So assume that |x− 3| < δ, and so

3− δ < x < 3 + δ multiply both sides by 26− 2δ < 2x < 6 + 2δ

Recall that f(x) = 2x and that since δ = ε/2

6− ε < f(x) < 6 + ε.

We can conclude that |f(x)− 6| < ε as required.

Because of the ε and δ in the definition of limits, we need to have ε and δ in theproof. While ε and δ are just symbols playing particular roles, and could be replacedwith other symbols, this style of proof is usually called ε–δ proof.

In the above example everything works, but it can be very instructive to see whathappens in an example that doesn’t work.

Example 1.7.4 Formal limit where limit does not exist.

Look again at the function

f(x) =

x x < 2

−1 x = 2

x+ 3 x > 2

and let us see why, according to the definition of the limit, that limx→2

f(x) 6= 2. Again,

start by sketching a picture and zooming in around (x, y) = (2, 2):

108

Page 125: CLP-1 Differential Calculus

LIMITS 1.7 (OPTIONAL) — MAKING THE INFORMAL A LITTLE MORE FORMAL

Example 1.7.4

Try to proceed through the same steps as before:

• Pick some small number ε > 0 and highlight a horizontal strip that contains ally-values with |y − L| < ε. This means all the y-values have to satisfy L − ε <y < L+ ε.

• You can see that the graph of the function passes through this strip for somex-values close to a. To the left of a, we can always find some x-values that makethe function sit inside the horizontal-ε-strip. However, unlike the previous ex-ample, there is a problem to the right of a. Even for x-values just a little largerthan a, the value of f(x) lies well outside the horizontal-ε-strip.

• So given this choice of ε, we can find a δ > 0 so that for x inside the verticalstrip a− δ < x < a, the value of the function sits inside the horizontal-ε-strip.

• Unfortunately, there is no way to choose a δ > 0 so that for x inside the verticalstrip a < x < a + δ (with x 6= a) the value of the function sits inside thehorizontal-epsilon-strip.

• So it is impossible to choose δ so that for x inside the vertical strip a− δ < x <a+δ the value of the function sits inside the horizontal strip L−ε < y = f(x) <L+ ε.

• Thus the limit of f(x) as x→ 2 is not 2.

Doing things formally with ε’s and δ’s is quite painful for general functions. Itis far better to make use of the arithmetic of limits (Theorem 1.4.3) and some basicbuilding blocks (like those in Theorem 1.4.1). Thankfully for most of the problemswe deal with in calculus (at this level at least) can be approached in exactly this way.

This does leave the problem of proving the arithmetic of limits and the limitsof the basic building blocks. The proof of the Theorem 1.4.3 is quite involved andwe leave it to the very end of this Chapter. Before we do that we will prove The-orem 1.4.1 by a formal ε–δ proof. Then in the next section we will look at the for-mal definition of limits at infinity and prove Theorem 1.5.3. The proof of the Theo-rem 1.5.9, the arithmetic of infinite limits, is very similar to that of Theorem 1.4.3 andso we do not give it.

So let us now prove Theorem 1.4.1 in which we stated two simple limits:

limx→a

c = c and limx→a

x = a.

Here is the formal ε–δ proof:

Proof. Proof of Theorem 1.4.1Since there are two limits to prove, we do each in turn. Let a, c be real numbers.

• Let ε > 0 and set f(x) = c. Choose δ = 1, then for any x satisfying

109

Page 126: CLP-1 Differential Calculus

LIMITS 1.8 (OPTIONAL) — MAKING INFINITE LIMITS A LITTLE MORE FORMAL

|x−a| < δ (or indeed any real number x at all) we have |f(x)− c| = 0 < ε.Hence lim

x→ac = c as required.

• Let ε > 0 and set f(x) = x. Choose δ = ε, then for any x satisfying|x− a| < δ we have

a− δ < x < a+ δ but f(x) = x and δ = ε

a− ε < f(x) < a+ ε

Thus we have |f(x)− a| < ε. Hence limx→a

x = a as required.

This completes the proof.

1.8q (Optional) — Making Infinite Limits a Little More For-mal

For those of you who made it through the formal ε − δ definition of limits we givethe formal definition of limits involving infinity:

Definition 1.8.1 Limits involving infinity — formal.

a Let f be a function defined on the whole real line. We say that

the limit as x approaches∞ of f(x) is L

or equivalently

f(x) converges to L as x goes to∞

and write

limx→∞

f(x) = L

if and only if for every ε > 0 there exists M ∈ R so that |f(x) − L| < εwhenever x > M .

Similarly we write

limx→−∞

f(x) = K

if and only if for every ε > 0 there exists N ∈ R so that |f(x) − K| < εwhenever x < N .

110

Page 127: CLP-1 Differential Calculus

LIMITS 1.8 (OPTIONAL) — MAKING INFINITE LIMITS A LITTLE MORE FORMAL

b Let a be a real number and f(x) be a function defined for all x 6= a. Wewrite

limx→a

f(x) =∞

if and only if for every P > 0 there exists δ > 0 so that f(x) > P whenever0 < |x− a| < δ.

c Let f be a function defined on the whole real line. We write

limx→∞

f(x) =∞

if and only if for every P > 0 there exists M > 0 so that f(x) > Pwhenever x > M .

Note that we can loosen the above requirements on the domain of definition off — for example, in part (a) all we actually require is that f(x) be defined for all xlarger than some value. It would be sufficient to require “there is some x0 ∈ R sothat f is defined for all x > x0”. Also note that there are obvious variations of parts(b) and (c) with∞ replaced by −∞.

For completeness let’s prove Theorem 1.5.3 using this form definition. The layoutof the proof will be very similar to our proof of Theorem 1.4.1.

Proof. Proof of Theorem 1.5.3.There are four limits to prove in total and we do each in turn. Let c ∈ R.

• Let ε > 0 and set f(x) = c. ChooseM = 0, then for any x satisfying x > M(or indeed any real number x at all) we have |f(x) − c| = 0 < ε. Hencelimx→∞

c = c as required.

• The proof that limx→−∞

c = c is nearly identical. Again, let ε > 0 and set

f(x) = c. Choose N = 0, then for any x satisfying x < N we have|f(x)− c| = 0 < ε. Hence lim

x→−∞c = c as required.

• Let ε > 0 and set f(x) = x. Choose M = 1ε. Then when x > M we have

0 < M < x divide through by xM to get

0 <1

x<

1

M= ε

Since x > 0, 1x

= | 1x| = | 1

x− 0| < ε as required.

• Again, the proof in the limit to −∞ is similar but we have to be careful ofsigns. Let ε > 0 and set f(x) = x. Choose N = −1

ε. Then when x < N we

have

0 > N > x divide through by xN to get

111

Page 128: CLP-1 Differential Calculus

LIMITS 1.9 (OPTIONAL) — PROVING THE ARITHMETIC OF LIMITS

0 >1

x>

1

N= −ε

Notice that by assumption both x,N < 0, so xN > 0. Now since x < 0,1x

= −| 1x| = | 1

x− 0| < ε as required.

This completes the proof.

1.9q (Optional) — Proving the Arithmetic of Limits

Perhaps the most useful theorem of this chapter is Theorem 1.4.3 which shows howlimits interact with arithmetic. In this (optional) section we will prove both the arith-metic of limits Theorem 1.4.3 and the Squeeze Theorem 1.4.18. Before we get to theproofs it is very helpful to prove three technical lemmas that we’ll need. The first isa very general result about absolute values of numbers:

Lemma 1.9.1 The triangle inequality.

For any x, y ∈ R

|x+ y| ≤ |x|+ |y|

Proof. Notice that for any real number x, we always have −x, x ≤ |x| andeither |x| = x or |x| = −x. So now let x, y ∈ R. Then we must have either

|x+ y| = x+ y ≤ |x|+ |y|

or

|x+ y| = −x− y ≤ |x|+ |y|

In both cases we end up with |x+ y| ≤ |x|+ |y|.

The second lemma is more specialised. It proves that if we have a function f(x)→F as x→ a then there must be a small window around x = awhere the function f(x)must only take values not far from F . In particular it tells us that |f(x)| cannot bebigger than |F |+ 1 when x is very close to a.

112

Page 129: CLP-1 Differential Calculus

LIMITS 1.9 (OPTIONAL) — PROVING THE ARITHMETIC OF LIMITS

Lemma 1.9.2

Let a ∈ R and let f be a function so that limx→a

f(x) = F . Then there exists a δ > 0

so that if 0 < |x− a| < δ then we also have |f(x)| ≤ |F |+ 1.

The proof is mostly just manipulating the ε–δ definition of a limit with ε = 1.

Proof. Let ε = 1. Then since f(x) → F as x → a, there exists δ > 0 sothat when 0 < |x − a| < δ, we also have |f(x) − F | ≤ ε = 1. So now assume0 < |x− a| < δ. Then

−ε ≤ f(x)− F ≤ ε rearrange a little−ε+ F ≤ f(x) ≤ ε+ F

Now ε+ F ≤ ε+ |F | and −ε+ F ≥ −ε− |F |, so

−ε− |F | ≤ f(x) ≤ ε+ |F |

Hence we have |f(x)| ≤ ε+ |F | = |F |+ 1.

Finally our third technical lemma gives us a bound in the other direction; it tellsus that when x is close to a, the value of |f(x)| cannot be much smaller than |F |.

Lemma 1.9.3

Let a ∈ R and F 6= 0 and let f be a function so that limx→a

f(x) = F . Then there

exists δ > 0 so that when 0 < |x− a| < δ, we have |f(x)| > |F |2

.

Proof. Set ε = |F |2> 0. Since f(x)→ F , we know there exists a δ > 0 so that

when 0 < |x− a| < δ we have |f(x)−F | < ε. So now assume 0 < |x− a| < δ sothat |f(x)− F | < ε = |F |

2. Then

|F | = |F − f(x) + f(x)| sneaky trick≤ |f(x)− F |+ |f(x)| but |f(x)− F | < ε

< ε+ |f(x)|

Hence |f(x)| > |F | − ε = |F |2

as required.

113

Page 130: CLP-1 Differential Calculus

LIMITS 1.9 (OPTIONAL) — PROVING THE ARITHMETIC OF LIMITS

Now we are in a position to prove Theorem 1.4.3. The proof has more steps thanthe previous ε − δ proofs we have seen. This is mostly because we do not havespecific functions f(x) and g(x) and instead must play with them in the abstract —and make good use of the formal definition of limits.

We will break the proof into three pieces. The minimum that is required is toprove that

limx→a

(f(x) + g(x)) = F +G

limx→a

f(x) · g(x) = F ·G

limx→a

1

g(x)=

1

Gif G 6= 0.

From these three we can prove that

limx→a

f(x) · c = F · climx→a

(f(x)− g(x)) = F −G

limx→a

f(x)

g(x)=F

Gif G 6= 0.

The first follows by setting g(x) = c and using lim f(x) · g(x). The second follows bysetting c = −1, putting h(x) = (−1) · g(x) and then applying both lim f(x) · g(x) andlim f(x)+g(x). The third follows by setting h(x) = 1

g(x)and then using lim f(x) ·h(x).

Starting with addition, in order to satisfy the definition of limit, we are going tohave to show that

|(f(x) + g(x))− (F +G)| is small

when we know that |f(x) − F |, |g(x) − G| are small. To do this we use the triangleinequality above showing that

|(f(x) + g(x))− (F +G)| = |(f(x)− F ) + (g(x)−G)| ≤ |f(x)− F |+ |g(x)−G|

This is the key technical piece of the proof. So if we want the LHS of the above to besize ε, we need to make sure that each term on the RHS is of size ε

2. The rest of the

proof is setting up facts based on the definition of limits and then rearranging factsto reach the conclusion.

Proof. Proof of Theorem 1.4.3 — limit of a sum.Let a ∈ R and assume that

limx→a

f(x) = F and limx→a

g(x) = G.

We wish to show that

limx→a

f(x) + g(x) = F +G.

Let ε > 0 — we have to find a δ > 0 so that when |x − a| < δ we have |(f(x) +

114

Page 131: CLP-1 Differential Calculus

LIMITS 1.9 (OPTIONAL) — PROVING THE ARITHMETIC OF LIMITS

g(x))− (F +G)| < ε.Let ε > 0 and set ε1 = ε2 = ε

2. By the definition of limits, because f(x)→ F there

exists some δ1 > 0 so that whenever |x − a| < δ1, we also have |f(x) − F | <ε1. Similarly there exists δ2 > 0 so that if |x − a| < δ2, then we must have|g(x) − G| < ε2. So now choose δ = minδ1, δ2 and assume |x − a| < δ. Thenwe must have that |x− a| < δ1, δ2 and so we also have

|f(x)− F | < ε1 |g(x)−G| < ε2

Now consider |(f(x) + g(x))− (F +G)| and rearrange the terms:

|(f(x) + g(x))− (F +G)| = |(f(x)− F ) + (g(x)−G)|

now apply triangle inequality

≤ |f(x)− F |+ |g(x)−G| use facts from above< ε1 + ε2

= ε.

Hence we have shown that for any ε > 0 there exists some δ > 0 so that when|x − a| < δ we also have |(f(x) + g(x)) − (F + G)| < ε. Which is exactly theformal definition of the limit we needed to prove.

Let us do similarly for the limit of a product. Some of the details of the proofare very similar, but there is a little technical trick in the middle to make it work. Inparticular we need to show that

|f(x) · g(x)− F ·G| is small

when we know that |f(x)− F | and |g(x)−G| are both small. Notice that

f(x) · g(x)− F ·G = f(x) · g(x)− F ·G+ f(x) ·G− f(x) ·G︸ ︷︷ ︸=0

= f(x) · g(x)− f(x) ·G+ f(x) ·G− F ·G= f(x) · (g(x)−G) + (f(x)− F ) ·G

So if we know |f(x)− F | is small and |g(x)−G| is small then we are done — exceptthat we also need to know that f(x) doesn’t become really large near a — this isexactly why we needed to prove Lemma 1.9.2.

As was the case in the previous proof, we want the LHS to be of size at most ε, sowe want, for example, the two terms on the RHS to be of size at most ε

2. This means

• we need |G| · |f(x)− F | to be of size at most ε2, and

• we need |g(x) − G| to be of size at most ε2(|F |+1)

since we know that |f(x)| ≤|F |+ 1 when x is close to a.

Armed with these tricks we turn to the proofs.

115

Page 132: CLP-1 Differential Calculus

LIMITS 1.9 (OPTIONAL) — PROVING THE ARITHMETIC OF LIMITS

Proof. Proof of Theorem 1.4.3 — limit of a product.Let a ∈ R and assume that

limx→a

f(x) = F and limx→a

g(x) = G.

We wish to show that

limx→a

f(x) · g(x) = F ·G.

Let ε > 0. Set ε1 = ε2(|G|+1)

(the extra +1 in the denominator is just there tomake sure that ε1 is well–defined even if G = 0), and ε2 = ε

2(|F |+1). From this we

establish the existence of δ1, δ2, δ3 which we need below.

• By assumption f(x)→ F so there exists δ1 > 0 so that whenever |x− a| <δ1, we also have |f(x)− F | < ε1.

• Similarly because g(x)→ G, there exists δ2 > 0 so that whenever |x−a| <δ2, we also have |g(x)−G| < ε2.

• By Lemma 1.9.2 there exists δ3 > 0 so that whenever |x− a| < δ3, we alsohave |f(x)| ≤ |F |+ 1.

Let δ = minδ1, δ2, δ3, assume |x − a| < δ and consider |f(x) · g(x) − F · G|.Rearrange the terms as we did above:

|f(x) · g(x)− F ·G| = |f(x) · (g(x)−G) + (f(x)− F ) ·G|≤ |f(x)| · |g(x)−G|+ |G| · |f(x)− F |

By our three dot-points above we know that |f(x)−F | < ε1 and |g(x)−G| < ε2and |f(x)| ≤ |F |+ 1, so we have

|f(x) · g(x)− F ·G| < |f(x)| · ε2 + |G| · ε1

sub in ε1, ε2 and bound on f(x)

< (|F |+ 1) · ε

2(|F |+ 1)+ |G| · ε

2(|G|+ 1)

≤ ε

2+ε

2= ε.

Thus we have shown that for any ε > 0 there exists δ > 0 so that when |x−a| < δwe also have |f(x) · g(x)− F ·G| < ε. Hence f(x) · g(x)→ F ·G.

Finally we can prove the limit of a reciprocal. Notice that

1

g(x)− 1

G=G− g(x)

g(x) ·G

116

Page 133: CLP-1 Differential Calculus

LIMITS 1.9 (OPTIONAL) — PROVING THE ARITHMETIC OF LIMITS

We need to show the LHS is of size at most ε when x is close enough to a, so ifG− g(x) is small we are done — except if g(x) or G are close to zero. By assumption(go back and read Theorem 1.4.3) we have G 6= 0, and we know from Lemma 1.9.3that |g(x)| cannot be smaller than |G|

2. Together these imply that the denominator on

the RHS cannot be zero and indeed must be of magnitude at least |G|2

2. Thus we need

|G− g(x)| to be of size at most ε · |G|22

.

Proof. Proof of Theorem 1.4.3 — limit of a reciprocal.Let ε > 0 and set ε1 = ε|G|2 · 1

2. We now use this and Lemma 1.9.3 to establish

the existence of δ1, δ2.

• Since g(x)→ G we know that there exists δ1 > 0 so that when |x− a| < δ1

we also have |g(x)−G| < ε1.

• By Lemma 1.9.3 there exists δ2 so that when |x − a| < δ2 we also have|g(x)| > |G|

2. Equivalently, when |x− a| < δ2 we also have

∣∣∣ G2g(x)

∣∣∣ < 1.

Set δ = minδ1, δ2 and assume |x− a| < δ. Then∣∣∣∣ 1

g(x)− 1

G

∣∣∣∣ =

∣∣∣∣G− g(x)

g(x) ·G

∣∣∣∣= |g(x)−G| · 1

|G| · |g(x)| by assumption

<ε1

|G| · |g(x)| sub in ε1

= ε · |G|2|g(x)| since

∣∣∣∣ G

2g(x)

∣∣∣∣ < 1

< ε

Thus we have shown that for any ε > 0 there exists δ > 0 so that when |x−a| < δwe also have | 1

g(x)− 1

G| < ε. Hence 1

g(x)→ 1

G.

We can also now prove the Squeeze / sandwich / pinch theorem.

Proof. Proof of Theorem 1.4.18 — Squeeze / sandwich / pinch.In the squeeze theorem, we are given three functions f(x), g(x) and h(x) andare told that

f(x) ≤ g(x) ≤ h(x) and limx→a

f(x) = limx→a

h(x) = L

and we must conclude from this that limx→a

g(x) = L too. That is, we are givensome fixed, but unspecified, ε > 0 and it is up to us to find a δ > 0 with theproperty that |g(x)− L| < ε whenever |x− a| < δ. Now because we have beentold that f and h both converge to L, there exist δ1 > 0 and δ2 > 0 such that

117

Page 134: CLP-1 Differential Calculus

LIMITS 1.9 (OPTIONAL) — PROVING THE ARITHMETIC OF LIMITS

• |f(x)− L| < ε, i.e. L− ε < f(x) < L+ ε, whenever |x− a| < δ1, and

• |h(x)− L| < ε, i.e. L− ε < h(x) < L+ ε, whenever |x− a| < δ2

So set δ = minδ1, δ2 and assume |x− a| < δ. Then both L− ε < f(x) < L + εand L− ε < h(x) < L+ ε so that

L− ε < f(x) ≤ g(x) ≤ h(x) < L+ ε which implies thatL− ε <g(x) < L+ ε which in turn gives us

|g(x)− L| < ε

as desired.

118

Page 135: CLP-1 Differential Calculus

DERIVATIVES

Chapter 2

Calculus is built on two operations — differentiation, which is used to analyse in-stantaneous rate of change, and integration, which is used to analyse areas. Under-standing differentiation and using it to compute derivatives of functions is one ofthe main aims of this course.

We had a glimpse of derivatives in the previous chapter on limits — in particularSections 1.1 and 1.2 on tangents and velocities introduced derivatives in disguise.One of the main reasons that we teach limits is to understand derivatives. Fortu-nately, as we shall see, while one does need to understand limits in order to correctlyunderstand derivatives, one does not need the full machinery of limits in order tocompute and work with derivatives. The other main part of calculus, integration, we(mostly) leave until a later course.

The derivative finds many applications in many different areas of the sciences.Indeed the reason that calculus is taken by so many university students is so thatthey may then use the ideas both in subsequent mathematics courses and in otherfields. In almost any field in which you study quantitative data you can find calculuslurking somewhere nearby.

Its development 1 came about over a very long time, starting with the ancientGreek geometers. Indian, Persian and Arab mathematicians made significant contri-butions from around the 6th century. But modern calculus really starts with Newtonand Leibniz in the 17th century who developed independently based on ideas ofothers including Descartes. Newton applied his work to many physical problems(including orbits of moons and planets) but didn’t publish his work. When Leibnizsubsequently published his “calculus”, Newton accused him of plagiarism — thiscaused a huge rift between British and continental-European mathematicians whichwasn’t closed for another century.

1 A quick google will turn up many articles on the development and history of calculus. Wikipediahas a good one.

119

Page 136: CLP-1 Differential Calculus

DERIVATIVES 2.1 REVISITING TANGENT LINES

2.1q Revisiting Tangent Lines

2.1.1 tt Revisiting Tangent Lines

By way of motivation for the definition of the derivative, we return to the discussionof tangent lines that we started in the previous chapter on limits. We consider, inExamples 2.1.2 and 2.1.5, below, the problem of finding the slope of the tangent lineto a curve at a point. But let us start by recalling, in Example 2.1.1, what is meant bythe slope of a straight line.

Example 2.1.1 What is slope.

In this example, we recall what is meant by the slope of the straight line

y = 12x+ 3

2

• We claim that if, as we walk along this straight line, our x–coordinate changesby an amount ∆x, then our y–coordinate changes by exactly ∆y = 1

2∆x.

• For example, in the figure on the left below, we move from the point

(x0, y0) = (1 , 2 = 12× 1 + 3

2)

on the line to the point

(x1, y1) = (5 , 4 = 12× 5 + 3

2)

on the line. In this move our x–coordinate changes by

∆x = 5− 1 = 4

and our y–coordinate changes by

∆y = 4− 2 = 2

which is indeed 12× 4 = 1

2∆x, as claimed.

120

Page 137: CLP-1 Differential Calculus

DERIVATIVES 2.1 REVISITING TANGENT LINES

Example 2.1.1

• In general, when we move from the point

(x0, y0) = (x0,12x0 + 3

2)

on the line to the point

(x1, y1) = (x1,12x1 + 3

2)

on the line, our x–coordinate changes by

∆x = x1 − x0

and our y–coordinate changes by

∆y = y1 − y0

=[

12x1 + 3

2

]−[

12x0 + 3

2

]= 1

2(x1 − x0)

which is indeed 12∆x, as claimed.

• So, for the straight line y = 12x+ 3

2, the ratio ∆y

∆x= y1−y0

x1−x0 always takes the value12, regardless of the choice of initial point (x0, y0) and final point (x1, y1). This

constant ratio is the slope of the line y = 12x+ 3

2.

Straight lines are special in that for each straight line, there is a fixed numberm, called the slope of the straight line, with the property that if you take any twodifferent points, (x0, y0) and (x1, y1), on the line, the ratio ∆y

∆x= y1−y0

x1−x0 , which is calledthe rate of change of y per unit rate of change 1 of x, always takes the value m. Thisis the property that distinguishes lines from other curves.

Other curves do not have this property. In the next two examples we illustratethis point with the parabola y = x2. Recall that we studied this example back inSection 1.1. In Example 2.1.2 we find the slope of the tangent line to y = x2 at aparticular point. We generalise this in Example 2.1.5, to show that we can define “theslope of the curve y = x2” at an arbitrary point x = x0 by considering ∆y

∆x= y1−y0

x1−x0with (x1, y1) very close to (x0, y0).

Example 2.1.2 Slope of secants of y = x2.

In this example, let us fix (x0, y0) to be the point (2, 4) on the parabola y = x2. Now let(x1, y1) = (x1, x

21) be some other point on the parabola; that is, a point with x1 6= x0.

1 In the “real world” the phrase “rate of change” usually refers to rate of change per unit time. Inscience it used more generally.

121

Page 138: CLP-1 Differential Calculus

DERIVATIVES 2.1 REVISITING TANGENT LINES

• Draw the straight line through (x0, y0) and (x1, y1) — this is a secant line andwe saw these in Chapter 1 when we discussed tangent lines a.

• The following table gives the slope, y1−y0x1−x0 , of the secant line through (x0, y0) =

(2, 4) and (x1, y1), for various different choices of (x1, y1 = x21).

x1 1 1.5 1.9 1.99 1.999 2.001 2.01 2.1 2.5 3

y1 = x21 1 2.25 3.61 3.9601 3.9960 4.0040 4.0401 4.41 6.25 9

y1−y0x1−x0 3 3.5 3.9 3.99 3.999 4.001 4.01 4.1 4.5 5

• So now we have a big table of numbers — what do we do with them? Well,there are messages we can take away from this table.

Different choices of x1 give different values for the slope, y1−y0x1−x0 , of the

secant through (x0, y0) and (x1, y1). This is illustrated in Figure 2.1.3 below— the slope of the secant through (x0, y0) and (x1, y1) is different from theslope of the secant through (x0, y0) and (x′1, y

′1).

Figure 2.1.3: For a curvy curve, different secants have different slopes.

If the parabola were a straight line this would not be the case — the secantthrough any two different points on a line is always identical to the lineitself and so always has exactly the same slope as the line itself, as is il-lustrated in Figure 2.1.4 below — the (yellow) secant through (x0, y0) and(x1, y1) lies exactly on top of the (red) line y = 1

2x+ 3

2.

122

Page 139: CLP-1 Differential Calculus

DERIVATIVES 2.1 REVISITING TANGENT LINES

Example 2.1.4

Figure 2.1.4: For a straight line, all secants have the same slope.

Now look at the columns of the table closer to the middle. As x1 getscloser and closer to x0 = 2, the slope, y1−y0

x1−x0 , of the secant through (x0, y0)and (x1, y1) appears to get closer and closer to the value 4.

a If you do not remember this, then please revisit the first couple of sections of Chapter 1.

Example 2.1.5 More on secants of y = x2.

It is very easy to generalise what is happening in Example 2.1.2.

• Fix any point (x0, y0) on the parabola y = x2. If (x1, y1) is any other point on theparabola y = x2, then y1 = x2

1 and the slope of the secant through (x0, y0) and(x1, y1) is

slope =y1 − y0

x1 − x0

=x2

1 − x20

x1 − x0

since y = x2

=(x1 − x0)(x1 + x0)

x1 − x0

remember a2 − b2 = (a− b)(a+ b)

= x1 + x0

You should check the values given in the table of Example 2.1.2 above to con-vince yourself that the slope y1−y0

x1−x0 of the secant line really is x0 + x1 = 2 + x1

(since we set x0 = 2).

• Now as we move x1 closer and closer to x0, the slope should move closer andcloser to 2x0. Indeed if we compute the limit carefully — we now have the

123

Page 140: CLP-1 Differential Calculus

DERIVATIVES 2.1 REVISITING TANGENT LINES

Example 2.1.6

technology to do this — we see that in the limit as x1 → x0 the slope becomes2x0. That is

limx1→x0

y1 − y0

x1 − x0

= limx1→x0

(x1 + x0) by the work we did just above

= 2x0

Taking this limit gives us our first derivative. Of course we haven’t yet giventhe definition of a derivative, so we perhaps wouldn’t recognise it yet. Werectify this in the next section.

Figure 2.1.6: Secants approaching a tangent line

• So it is reasonable to say “as x1 approaches x0, the secant through (x0, y0) and(x1, y1) approaches the tangent line to the parabola y = x2 at (x0, y0)”. This iswhat we did back in Section 1.1.

The figure above shows four different secants through (x0, y0) for the curvey = x2. The four hollow circles are four different choices of (x1, y1). As (x1, y1)approaches (x0, y0), the corresponding secant does indeed approach the tan-gent to y = x2 at (x0, y0), which is the heavy (red) straight line in the figure.

Using limits we determined the slope of the tangent line to y = x2 at x0 to be2x0. Often we will be a little sloppy with our language and instead say “theslope of the parabola y = x2 at (x0, y0) is 2x0” — where we really mean theslope of the line tangent to the parabola at x0.

124

Page 141: CLP-1 Differential Calculus

DERIVATIVES 2.1 REVISITING TANGENT LINES

2.1.2 tt Exercises

Exercises — Stage 1

1. Shown below is the graph y = f(x). If we choose a point Q on the graph tothe left of the y-axis, is the slope of the secant line through P and Q positiveor negative? If we choose a point Q on the graph to the right of the y-axis, isthe slope of the secant line through P and Q positive or negative?

x

y

y = f(x)

P

2. Shown below is the graph y = f(x).

a If we want the slope of the secant line through P and Q to increase,should we slide Q closer to P , or further away?

b Which is larger, the slope of the tangent line at P , or the slope ofthe secant line through P and Q?

x

y

P

Q

3. Group the functions below into collections whose secant lines from x = −2to x = 2 all have the same slopes.

125

Page 142: CLP-1 Differential Calculus

DERIVATIVES 2.1 REVISITING TANGENT LINES

x

y

−2 2

(a)

x

y

−2 2

(b)

x

y

−2 2

(c)

x

y

−2 2

(d)

x

y

−2 2

(e)

x

y

−2 2

(f)

Exercises — Stage 2

4. Give your best approximation of the slope of the tangent line to the graphbelow at the point x = 5.

x

y

1 5

1

5. On the graph below, sketch the tangent line to y = f(x) at P . Then, findtwo points Q and R on the graph so that the secant line through Q andR has the same slope as the tangent line at P .

126

Page 143: CLP-1 Differential Calculus

DERIVATIVES 2.2 DEFINITION OF THE DERIVATIVE

x

y

y = f(x)

P

6. Mark the points where the curve shown below has a tangent line withslope 0.

x

y

y = f(x)

(Later on, we’ll learn how these points tell us a lot about the shape of agraph.)

2.2q Definition of the Derivative

We now define the “derivative” explicitly, based on the limiting slope ideas of theprevious section. Then we see how to compute some simple derivatives.

Let us now generalise what we did in the last section so as to find “the slope of

127

Page 144: CLP-1 Differential Calculus

DERIVATIVES 2.2 DEFINITION OF THE DERIVATIVE

the curve y = f(x) at (x0, y0)” for any smooth enough 1 function f(x).As before, let (x0, y0) be any point on the curve y = f(x). So we must have

y0 = f(x0). Now let (x1, y1) be any other point on the same curve. So y1 = f(x1) andx1 6= x0. Think of (x1, y1) as being pretty close to (x0, y0) so that the difference

∆x = x1 − x0

in x–coordinates is pretty small. In terms of this ∆x we have

x1 = x0 + ∆x and y1 = f(x0 + ∆x

)We can construct a secant line through (x0, y0) and (x1, y1) just as we did for theparabola above. It has slope

y1 − y0

x1 − x0

=f(x0 + ∆x

)− f(x0)

∆x

If f(x) is reasonably smooth 2, then as x1 approaches x0, i.e. as ∆x approaches 0, wewould expect the secant through (x0, y0) and (x1, y1) to approach the tangent line tothe curve y = f(x) at (x0, y0), just as happened in Figure 2.1.6. And more importantly,the slope of the secant through (x0, y0) and (x1, y1) should approach the slope of thetangent line to the curve y = f(x) at (x0, y0).

Thus we would expect 3 the slope of the tangent line to the curve y = f(x) at(x0, y0) to be

lim∆x→0

f(x0 + ∆x

)− f(x0)

∆x

When we talk of the “slope of the curve” at a point, what we really mean is the slopeof the tangent line to the curve at that point. So “the slope of the curve y = f(x) at(x0, y0)” is also the limit 4 expressed in the above equation. The derivative of f(x)at x = x0 is also defined to be this limit. Which leads 5 us to the most importantdefinition in this text:

Definition 2.2.1 Derivative at a point.

Let a ∈ R and let f(x) be defined on an open interval a that contains a.

• The derivative of f(x) at x = a is denoted f ′(a) and is defined by

f ′(a) = limh→0

f(a+ h

)− f(a)

h

if the limit exists.

1 The idea of “smooth enough” can be made quite precise. Indeed the word “smooth” has a veryprecise meaning in mathematics, which we won’t cover here. For now think of “smooth” asmeaning roughly just “smooth”.

2 Again the term “reasonably smooth” can be made more precise.3 Indeed, we don’t have to expect — it is!4 This is of course under the assumption that the limit exists — we will talk more about that below.5 We will rename “x0” to “a” and “∆x” to “h”.

128

Page 145: CLP-1 Differential Calculus

DERIVATIVES 2.2 DEFINITION OF THE DERIVATIVE

• When the above limit exists, the function f(x) is said to be differentiableat x = a. When the limit does not exist, the function f(x) is said to be notdifferentiable at x = a.

• We can equivalently define the derivative f ′(a) by the limit

f ′(a) = limx→a

f(x)− f(a)

x− a .

To see that these two definitions are the same, we set x = a+ h and thenthe limit as h goes to 0 is equivalent to the limit as x goes to a.

a Recall, from Definition 0.3.5, that the open interval (c, d) is just the set of all real numbersobeying c < x < d.

Lets now compute the derivatives of some very simple functions. This is ourfirst step towards building up a toolbox for computing derivatives of complicatedfunctions — this process will very much parallel what we did in Chapter 1 withlimits. The two simplest functions we know are f(x) = c and g(x) = x.

Example 2.2.2 Derivative of f(x) = c.

Let a, c ∈ R be a constants. Compute the derivative of the constant function f(x) = cat x = a.We compute the desired derivative by just substituting the function of interest intothe formal definition of the derivative.

f ′(a) = limh→0

f(a+ h)− f(a)

h(the definition)

= limh→0

c− ch

(substituted in the function)

= limh→0

0 (simplified things)

= 0

That was easy! What about the next most complicated function — arguably it’sthis one:

Example 2.2.3 Derivative of g(x) = x.

Let a ∈ R and compute the derivative of g(x) = x at x = a.Again, we compute the derivative of g by just substituting the function of interest

129

Page 146: CLP-1 Differential Calculus

DERIVATIVES 2.2 DEFINITION OF THE DERIVATIVE

Example 2.2.3

into the formal definition of the derivative and then evaluating the resulting limit.

g′(a) = limh→0

g(a+ h)− g(a)

h(the definition)

= limh→0

(a+ h)− ah

(substituted in the function)

= limh→0

h

h(simplified things)

= limh→0

1 (simplified a bit more)

= 1

That was a little harder than the first example, but still quite straight forward —start with the definition and apply what we know about limits.

Thanks to these two examples, we have our first theorem about derivatives:

Theorem 2.2.4 Easiest derivatives.

Let a, c ∈ R and let f(x) = c be the constant function and g(x) = x. Then

f ′(a) = 0

and

g′(a) = 1.

To ratchet up the difficulty a little bit more, let us redo the example we have al-ready done a few times f(x) = x2. To make it a little more interesting let’s changethe names of the function and the variable so that it is not exactly the same as Exam-ples 2.1.2 and 2.1.5.

Example 2.2.5 Derivative of h(t) = t2.

Compute the derivative of

h(t) = t2 at t = a

• This function isn’t quite like the ones we saw earlier — it’s a function of t ratherthan x. Recall that a function is a rule which assigns to each input value anoutput value. So far, we have usually called the input value x. But this “x”is just a dummy variable representing a generic input value. There is nothing

130

Page 147: CLP-1 Differential Calculus

DERIVATIVES 2.2 DEFINITION OF THE DERIVATIVE

Example 2.2.5

wrong with calling a generic input value t instead. Indeed, from time to timeyou will see functions that are not written as formulas involving x, but insteadare written as formulas in t (for example representing time — see Section 1.2),or z (for example representing height), or other symbols.

• So let us write the definition of the derivative

f ′(a) = limh→0

f(a+ h)− f(a)

h

and then translate it to the function names and variables at hand:

h′(a) = limh→0

h(a+ h)− h(a)

h

• But there is a problem — “h” plays two roles here — it is both the functionname and the small quantity that is going to zero in our limit. It is extremelydangerous to have a symbol represent two different things in a single compu-tation. We need to change one of them. So let’s rename the small quantity thatis going to zero in our limit from “h” to “∆t”:

h′(a) = lim∆t→0

h(a+ ∆t)− h(a)

∆t

• Now we are ready to begin. Substituting in what the function h is,

h′(a) = lim∆t→0

(a+ ∆t)2 − a2

∆t

= lim∆t→0

a2 + 2a∆t+ ∆t2 − a2

∆t

(just squared out (a+ ∆t)2

)= lim

∆t→0

2a∆t+ ∆t2

∆t= lim

∆t→0(2a+ ∆t)

= 2a

• You should go back check that this is what we got in Example 2.1.5 — just somenames have been changed.

2.2.1 tt An Important Point (and Some Notation)

Notice here that the answer we get depends on our choice of a — if we want toknow the derivative at a = 3 we can just substitute a = 3 into our answer 2a to get

131

Page 148: CLP-1 Differential Calculus

DERIVATIVES 2.2 DEFINITION OF THE DERIVATIVE

the slope is 6. If we want to know at a = 1 (like at the end of Section 1.1) we substitutea = 1 and get the slope is 2. The important thing here is that we can move from thederivative being computed at a specific point to the derivative being a function itself— input any value of a and it returns the slope of the tangent line to the curve atthe point x = a, y = h(a). The variable a is a dummy variable. We can rename a toanything we want, like x, for example. So we can replace every a in

h′(a) = 2a by x, giving h′(x) = 2x

where all we have done is replaced the symbol a by the symbol x.We can do this more generally and tweak the derivative at a specific point a to

obtain the derivative as a function of x. We replace

f ′(a) = limh→0

f(a+ h)− f(a)

h

with

f ′(x) = limh→0

f(x+ h)− f(x)

h

which gives us the following definition

Definition 2.2.6 Derivative as a function.

Let f(x) be a function.

• The derivative of f(x) with respect to x is

f ′(x) = limh→0

f(x+ h

)− f(x)

h

provided the limit exists.

• If the derivative f ′(x) exists for all x ∈ (a, b) we say that f is differentiableon (a, b).

• Note that we will sometimes be a little sloppy with our discussions andsimply write “f is differentiable” to mean “f is differentiable on an inter-val we are interested in” or “f is differentiable everywhere”.

Notice that we are no longer thinking of tangent lines, rather this is an operationwe can do on a function. For example:

Example 2.2.7 The derivative of f(x) = 1x.

Let f(x) = 1x

and compute its derivative with respect to x — think carefully aboutwhere the derivative exists.

• Our first step is to write down the definition of the derivative — at this stage,

132

Page 149: CLP-1 Differential Calculus

DERIVATIVES 2.2 DEFINITION OF THE DERIVATIVE

Example 2.2.7

we know of no other strategy for computing derivatives.

f ′(x) = limh→0

f(x+ h)− f(x)

h(the definition)

• And now we substitute in the function and compute the limit.

f ′(x) = limh→0

f(x+ h)− f(x)

h(the definition)

= limh→0

1

h

[1

x+ h− 1

x

](substituted in the function)

= limh→0

1

h

x− (x+ h)

x(x+ h)(wrote over a common denominator)

= limh→0

1

h

−hx(x+ h)

(started cleanup)

= limh→0

−1

x(x+ h)

= − 1

x2

• Notice that the original function f(x) = 1x

was not defined at x = 0 and thederivative is also not defined at x = 0. This does happen more generally — iff(x) is not defined at a particular point x = a, then the derivative will not existat that point either.

So we now have two slightly different ideas of derivatives:

• The derivative f ′(a) at a specific point x = a, being the slope of the tangent lineto the curve at x = a, and

• The derivative as a function, f ′(x) as defined in Definition 2.2.6.

Of course, if we have f ′(x) then we can always recover the derivative at a specificpoint by substituting x = a.

As we noted at the beginning of the chapter, the derivative was discovered inde-pendently by Newton and Leibniz in the late 17th century. Because their discover-ies were independent, Newton and Leibniz did not have exactly the same notation.Stemming from this, and from the many different contexts in which derivatives areused, there are quite a few alternate notations for the derivative:

Definition 2.2.8

The following notations are all used for “the derivative of f(x) with respect to

133

Page 150: CLP-1 Differential Calculus

DERIVATIVES 2.2 DEFINITION OF THE DERIVATIVE

x”

f ′(x)df

dx

d

dxf(x) f(x) Df(x) Dxf(x),

while the following notations are all used for “the derivative of f(x) at x = a”

f ′(a)df

dx(a)

d

dxf(x)

∣∣∣∣x=a

f(a) Df(a) Dxf(a).

Some things to note about these notations:

• We will generally use the first three, but you should recognise them all.The notation f ′(a) is due to Lagrange, while the notation df

dx(a) is due to

Leibniz. They are both very useful. Neither can be considered “better”.

• Leibniz notation writes the derivative as a “fraction” — however it isdefinitely not a fraction and should not be thought of in that way. It isjust shorthand, which is read as “the derivative of f with respect to x”.

• You read f ′(x) as “f–prime of x”, and dfdx

as “dee–f–dee–x”, and ddxf(x)

as “dee-by-dee–x of f”.

• Similarly you read dfdx

(a) as “dee–f–dee–x at a”, and ddxf(x)|x=a as “dee-

by-dee-x of f of x at x equals a”.

• The notation f is due to Newton. In physics, it is common to use f(t) todenote the derivative of f with respect to time.

2.2.2 tt Back to Computing Some Derivatives

At this point we could try to start working out how derivatives interact with arith-metic and make an “Arithmetic of derivatives” theorem just like the one we sawfor limits (Theorem 1.4.3). We will get there shortly, but before that it is importantthat we become more comfortable with computing derivatives using limits and thenunderstanding what the derivative actually means. So — more examples.

Example 2.2.9 ddx

√x.

Compute the derivative, f ′(a), of the function f(x) =√x at the point x = a for any

a > 0.

134

Page 151: CLP-1 Differential Calculus

DERIVATIVES 2.2 DEFINITION OF THE DERIVATIVE

• So again we start with the definition of derivative and go from there:

f ′(a) = limx→a

f(x)− f(a)

x− a = limx→a

√x−√ax− a

• As x tends to a, the numerator and denominator both tend to zero. But 00

isnot defined. So to get a well defined limit we need to exhibit a cancellationbetween the numerator and denominator — just as we saw in Examples 1.4.12and 1.4.17. Now there are two equivalent ways to proceed from here, bothbased on a similar “trick”.

• For the first, review Example 1.4.17, which concerned taking a limit involvingsquare-roots, and recall that we used “multiplication by the conjugate” there:

√x−√ax− a

=

√x−√ax− a ×

√x+√a√

x+√a

(multiplication by 1 =

conjugateconjugate

)=

(√x−√a)(

√x+√a)

(x− a)(√x+√a)

=x− a

(x− a)(√x+√a)

(since (A−B)(A+B) = A2 −B2)

)=

1√x+√a

• Alternatively, we can arrive at√x−√ax−a = 1√

x+√a

by using almost the same trickto factor the denominator. Just set A =

√x and B =

√a in A2 − B2 = (A −

B)(A+B) to get

x− a = (√x−√a)(

√x+√a)

and then substitute this little fact into our expression√x−√ax− a =

√x−√a

(√x−√a)(

√x+√a)

(now cancel common factors)

=1

(√x+√a)

• Once we know that√x−√ax−a = 1√

x+√a, we can take the limit we need:

f ′(a) = limx→a

√x−√ax− a

= limx→a

1√x+√a

=1

2√a

135

Page 152: CLP-1 Differential Calculus

DERIVATIVES 2.2 DEFINITION OF THE DERIVATIVE

Example 2.2.9

• We should think about the domain of f ′ here — that is, for which values of ais f ′(a) defined? The original function f(x) was defined for all x ≥ 0, howeverthe derivative f ′(a) = 1

2√a

is undefined at a = 0.

If we draw a careful picture of√x around x = 0 we can see why this has to

be the case. The figure below shows three different tangent lines to the graphof y = f(x) =

√x. As the point of tangency moves closer and closer to the

origin, the tangent line gets steeper and steeper. The slope of the tangent lineat(a,√a)

blows up as a→ 0.

Example 2.2.10 ddx|x|.

Compute the derivative, f ′(a), of the function f(x) = |x| at the point x = a.

• We should start this example by recalling the definition of |x| (we saw this backin Example 1.5.6):

|x| =

−x if x < 0

0 if x = 0

x if x > 0.

It is definitely not just “chop off the minus sign”.

136

Page 153: CLP-1 Differential Calculus

DERIVATIVES 2.2 DEFINITION OF THE DERIVATIVE

• This breaks our computation of the derivative into 3 cases depending onwhether x is positive, negative or zero.

• Assume x > 0. Then

df

dx= lim

h→0

f(x+ h)− f(x)

h

= limh→0

|x+ h| − |x|h

Since x > 0 and we are interested in the behaviour of this function as h→ 0 wecan assume h is much smaller than x. This means x+h > 0 and so |x+h| = x+h.

= limh→0

x+ h− xh

= limh→0

h

h= 1 as expected

• Assume x < 0. Then

df

dx= lim

h→0

f(x+ h)− f(x)

h

= limh→0

|x+ h| − |x|h

Since x < 0 and we are interested in the behaviour of this function as h→ 0 wecan assume h is much smaller than x. This means x + h < 0 and so |x + h| =−(x+ h).

= limh→0

−(x+ h)− (−x)

h

= limh→0

−hh

= −1

• When x = 0 we have

f ′(0) = limh→0

f(0 + h)− f(0)

h

137

Page 154: CLP-1 Differential Calculus

DERIVATIVES 2.2 DEFINITION OF THE DERIVATIVE

Example 2.2.10

= limh→0

|0 + h| − |0|h

= limh→0

|h|h

To proceed we need to know if h > 0 or h < 0, so we must use one-sided limits.The limit from above is:

limh→0+

|h|h

= limh→0+

h

hsince h > 0, |h| = h

= 1

Whereas, the limit from below is:

limh→0−

|h|h

= limh→0−

−hh

since h < 0, |h| = −h

= −1

Since the one-sided limits differ, the limit as h→ 0 does not exist. And thus thederivative does not exist as x = 0.

In summary:

d

dx|x| =

−1 if x < 0

DNE if x = 0

1 if x > 0

2.2.3 ttWhere is the Derivative Undefined?

According to Definition 2.2.1, the derivative f ′(a) exists precisely when the limitlimx→a

f(x)−f(a)x−a exists. That limit is also the slope of the tangent line to the curve y = f(x)

at x = a. That limit does not exist when the curve y = f(x) does not have a tangentline at x = a or when the curve does have a tangent line, but the tangent line has

138

Page 155: CLP-1 Differential Calculus

DERIVATIVES 2.2 DEFINITION OF THE DERIVATIVE

infinite slope. We have already seen some examples of this.

• In Example 2.2.7, we considered the function f(x) = 1x. This function “blows

up” (i.e. becomes infinite) at x = 0. It does not have a tangent line at x = 0 andits derivative does not exist at x = 0.

• In Example 2.2.10, we considered the function f(x) = |x|. This function doesnot have a tangent line at x = 0, because there is a sharp corner in the graphof y = |x| at x = 0. (Look at the graph in Example 2.2.10.) So the derivative off(x) = |x| does not exist at x = 0.

Here are a few more examples.

Example 2.2.11 Derivative at a discontinuity.

Visually, the function

H(x) =

0 if x ≤ 0

1 if x > 0

does not have a tangent line at (0, 0). Not surprisingly, when a = 0 and h tends to 0with h > 0,

H(a+ h)−H(a)

h=H(h)−H(0)

h=

1

h

blows up. The same sort of computation shows that f ′(a) cannot possibly existwhenever the function f is not continuous at a. We will formalize, and prove, thisstatement in Theorem 2.2.14, below.

Example 2.2.12 ddxx1/3.

Visually, it looks like the function f(x) = x1/3, sketched below, (this might be a goodpoint to recall that cube roots of negative numbers are negative — for example, since(−1)3 = −1, the cube root of −1 is −1),

139

Page 156: CLP-1 Differential Calculus

DERIVATIVES 2.2 DEFINITION OF THE DERIVATIVE

Example 2.2.12

has the y–axis as its tangent line at (0, 0). So we would expect that f ′(0) does notexist. Let’s check. With a = 0,

f ′(a) = limh→0

f(a+ h)− f(a)

h= lim

h→0

f(h)− f(0)

h= lim

h→0

h1/3

h

= limh→0

1

h2/3= DNE

as expected.

Example 2.2.13 ddx

√|x|.

We have already considered the derivative of the function√x in Example 2.2.9. We’ll

now look at the function f(x) =√|x|. Recall, from Example 2.2.10, the definition of

|x|.When x > 0, we have |x| = x and f(x) is identical to

√x. When x < 0, we have

|x| = −x and f(x) =√−x. So to graph y =

√|x| when x < 0, you just have to graph

y =√x for x > 0 and then send x → −x — i.e. reflect the graph in the y–axis. Here

is the graph.

The pointy thing at the origin is called a cusp. The graph of y = f(x) does not havea tangent line at (0, 0) and, correspondingly, f ′(0) does not exist because

limh→0+

f(h)− f(0)

h= lim

h→0+

√|h|h

= limh→0+

1√h

= DNE

140

Page 157: CLP-1 Differential Calculus

DERIVATIVES 2.2 DEFINITION OF THE DERIVATIVE

Theorem 2.2.14

If the function f(x) is differentiable at x = a, then f(x) is also continuous atx = a.

Proof. The function f(x) is continuous at x = a if and only if the limit of

f(a+ h)− f(a) =f(a+ h)− f(a)

hh

as h→ 0 exists and is zero. But if f(x) is differentiable at x = a, then, as h→ 0,the first factor, f(a+h)−f(a)

hconverges to f ′(a) and the second factor, h, converges

to zero. So the product provision of our arithmetic of limits Theorem 1.4.3implies that the product f(a+h)−f(a)

hh converges to f ′(a) · 0 = 0 too.

Notice that while this theorem is useful as stated, it is (arguably) more often ap-plied in its contrapositive 6 form:

Theorem 2.2.15 The contrapositive of Theorem 2.2.14.

If f(x) is not continuous at x = a then it is not differentiable at x = a.

As the above examples illustrate, this statement does not tell us what happens iff is continuous at x = a — we have to think!

2.2.4 tt Exercises

Exercises — Stage 1

1. The function f(x) is shown. Select all options below that describe its deriva-

tive,df

dx:

• (a) constant

• (b) increasing

• (c) decreasing

• (d) always positive

6 If you have forgotten what the contrapositive is, then quickly reread Footnote 1.3.5 in Section 1.3.

141

Page 158: CLP-1 Differential Calculus

DERIVATIVES 2.2 DEFINITION OF THE DERIVATIVE

• (e) always negative

x

y

y = f(x)

2. The function f(x) is shown. Select all options below that describe its deriva-

tive,df

dx:

• (a) constant

• (b) increasing

• (c) decreasing

• (d) always positive

• (e) always negative

x

y

y = f(x)

3. The function f(x) is shown. Select all options below that describe its

derivative,df

dx:

142

Page 159: CLP-1 Differential Calculus

DERIVATIVES 2.2 DEFINITION OF THE DERIVATIVE

• (a) constant

• (b) increasing

• (c) decreasing

• (d) always positive

• (e) always negative

x

y

y = f(x)

4. *. State, in terms of a limit, what it means for f(x) = x3 to be differentiableat x = 0.

5. For which values of x does f ′(x) not exist?

x

y

1

1

143

Page 160: CLP-1 Differential Calculus

DERIVATIVES 2.2 DEFINITION OF THE DERIVATIVE

6. Suppose f(x) is a function defined at x = a with

limh→0−

f(a+ h)− f(a)

h= lim

h→0+

f(a+ h)− f(a)

h= 1.

True or false: f ′(a) = 1.

7. Suppose f(x) is a function defined at x = a with

limx→a−

f ′(x) = limx→a+

f ′(x) = 1.

True or false: f ′(a) = 1.

8. Suppose s(t) is a function, with t measured in seconds, and s measuredin metres. What are the units of s′(t)?

Exercises — Stage 2

9. Use the definition of the derivative to find the equation of the tangent lineto the curve y(x) = x3 + 5 at the point (1, 6).

10. Use the definition of the derivative to find the derivative of f(x) = 1x.

11. *. Let f(x) = x|x|. Using the definition of the derivative, show that f(x) isdifferentiable at x = 0.

12. *. Use the definition of the derivative to compute the derivative of thefunction f(x) = 2

x+1.

13. *. Use the definition of the derivative to compute the derivative of the func-tion f(x) = 1

x2+3.

14. Use the definition of the derivative to find the slope of the tangent line tothe curve f(x) = x log10(2x+ 10) at the point x = 0.

15. *. Compute the derivative of f(x) = 1x2

directly from the definition.

16. *. Find the values of the constants a and b for which

f(x) =

x2 x ≤ 2

ax+ b x > 2

is differentiable everywhere.Remark: In the text, you have already learned the derivatives of x2 andax + b. In this question, you are only asked to find the values of a andb—not to justify how you got them—so you don’t have to use the def-inition of the derivative. However, on an exam, you might be asked to

144

Page 161: CLP-1 Differential Calculus

DERIVATIVES 2.2 DEFINITION OF THE DERIVATIVE

justify your answer, in which case you would show how to differentiatethe two branches of f(x) using the definition of a derivative.

17. *. Use the definition of the derivative to compute f ′(x) if f(x) =√

1 + x.Where does f ′(x) exist?

Exercises — Stage 3

18. Use the definition of the derivative to find the velocity of an object whoseposition is given by the function s(t) = t4 − t2.

19. *. Determine whether the derivative of following function exists at x =0.

f(x) =

x cosx if x ≥ 0√x2 + x4 if x < 0

You must justify your answer using the definition of a derivative.

20. *. Determine whether the derivative of the following function exists at x = 0

f(x) =

x cosx if x ≤ 0√

1 + x− 1 if x > 0

You must justify your answer using the definition of a derivative.

21. *. Determine whether the derivative of the following function exists at x = 0

f(x) =

x3 − 7x2 if x ≤ 0

x3 cos(

1x

)if x > 0

You must justify your answer using the definition of a derivative.

22. *. Determine whether the derivative of the following function exists at x = 1

f(x) =

4x2 − 8x+ 4 if x ≤ 1

(x− 1)2 sin

(1

x− 1

)if x > 1

You must justify your answer using the definition of a derivative.

23. . Sketch a function f(x) with f ′(0) = −1 that takes the following values:

x −1 −12−1

4−1

80 1

814

12

1

f(x) −1 −12−1

4−1

80 1

814

12

1

145

Page 162: CLP-1 Differential Calculus

DERIVATIVES 2.3 INTERPRETATIONS OF THE DERIVATIVE

Remark: you can’t always guess the behaviour of a function from itspoints, even if the points seem to be making a clear pattern.

24. Let p(x) = f(x) + g(x), for some functions f and g whose derivatives exist.Use limit laws and the definition of a derivative to show that p′(x) = f ′(x)+g′(x).Remark: this is called the sum rule, and we’ll learn more about it inLemma 2.4.1.

25. Let f(x) = 2x, g(x) = x, and p(x) = f(x) · g(x).

a Find f ′(x) and g′(x).

b Find p′(x).

c Is p′(x) = f ′(x) · g′(x)?

In Theorem 2.4.3, you’ll learn a rule for calculating the derivative of a prod-uct of two functions.

26. *. There are two distinct straight lines that pass through the point (1,−3)and are tangent to the curve y = x2. Find equations for these two lines.Remark: the point (1,−3) does not lie on the curve y = x2.

27. *. For which values of a is the function

f(x) =

0 x ≤ 0

xa sin 1x

x > 0

differentiable at 0?

2.3q Interpretations of the Derivative

In the previous sections we defined the derivative as the slope of a tangent line,using a particular limit. This allows us to compute “the slope of a curve” 1 andprovides us with one interpretation of the derivative. However, the main importanceof derivatives does not come from this application. Instead, (arguably) it comes fromthe interpretation of the derivative as the instantaneous rate of change of a quantity.

1 Again — recall that we are being a little sloppy with this term — we really mean “The slope ofthe tangent line to the curve”.

146

Page 163: CLP-1 Differential Calculus

DERIVATIVES 2.3 INTERPRETATIONS OF THE DERIVATIVE

2.3.1 tt Instantaneous Rate of Change

In fact we have already (secretly) used a derivative to compute an instantaneous rateof change in Section 1.2. For your convenience we’ll review that computation here,in Example 2.3.1, and then generalise it.

Example 2.3.1 Velocity as a derivative.

You drop a ball from a tall building. After t seconds the ball has fallen a distance ofs(t) = 4.9t2 metres. What is the velocity of the ball one second after it is dropped?

• In the time interval from t = 1 to t = 1 + h the ball travels a distance

s(1 + h)− s(1) = 4.9(1 + h)2 − 4.9(1)2 = 4.9[2h+ h2

]• So the average velocity over this time interval is

average velocity from t = 1 to t = 1 + h

=distance travelled from t = 1 to t = 1 + h

length of time from t = 1 to t = 1 + h

=s(1 + h)− s(1)

h

=4.9[2h+ h2

]h

= 4.9[2 + h]

• The instantaneous velocity at time t = 1 is then defined to be the limit

instantaneous velocity at time t = 1

= limh→0

[average velocity from t = 1 to t = 1 + h

]= lim

h→0

s(1 + h)− s(1)

h= s′(1)

= limh→0

4.9[2 + h]

= 9.8m/sec

• We conclude that the instantaneous velocity at time t = 1, which is the instan-taneous rate of change of distance per unit time at time t = 1, is the derivatives′(1) = 9.8m/sec.

Now suppose, more generally, that you are taking a walk and that as you walk,you are continuously measuring some quantity, like temperature, and that the mea-

147

Page 164: CLP-1 Differential Calculus

DERIVATIVES 2.3 INTERPRETATIONS OF THE DERIVATIVE

surement at time t is f(t). Then the

average rate of change of f(t) from t = a to t = a+ h

=change in f(t) from t = a to t = a+ h

length of time from t = a to t = a+ h

=f(a+ h)− f(a)

h

so the

instantaneous rate of change of f(t) at t = a

= limh→0

[average rate of change of f(t) from t = a to t = a+ h

]= lim

h→0

f(a+ h)− f(a)

h= f ′(a)

In particular, if you are walking along the x–axis and your x–coordinate at time t isx(t), then x′(a) is the instantaneous rate of change (per unit time) of your x–coordinateat time t = a, which is your velocity at time a. If v(t) is your velocity at time t, thenv′(a) is the instantaneous rate of change of your velocity at time a. This is called youracceleration at time a.

2.3.2 tt Slope

Suppose that y = f(x) is the equation of a curve in the xy–plane. That is, f(x) is they–coordinate of the point on the curve whose x–coordinate is x. Then, as we havealready seen,

[the slope of the secant through

(a, f(a)

)and

(a+ h, f(a+ h)

)]=f(a+ h)− f(a)

h

This is shown in Figure 2.3.2 below.

148

Page 165: CLP-1 Differential Calculus

DERIVATIVES 2.3 INTERPRETATIONS OF THE DERIVATIVE

Figure 2.3.2

In order to create the tangent line (as we have done a few times now) we squeezeh → 0. As we do this, the secant through

(a, f(a)

)and

(a + h, f(a + h)

)approaches

2 the tangent line to y = f(x) at x = a. Since the secant becomes the tangent line inthis limit, the slope of the secant becomes the slope of the tangent and

[the slope of the tangent line to y = f(x) at x = a

]= lim

h→0

f(a+ h)− f(a)

h= f ′(a).

Let us go a little further and work out a general formula for the equation of thetangent line to y = f(x) at x = a. We know that the tangent line

• has slope f ′(a) and

• passes through the point(a, f(a)

).

There are a couple of different ways to construct the equation of the tangent linefrom this information. One is to observe, as in Figure 2.3.3, that if (x, y) is any otherpoint on the tangent line then the line segment from

(a, f(a)

)to (x, y) is part of the

tangent line and so also has slope f ′(a). That is,

y − f(a)

x− a =[the slope of the tangent line

]= f ′(a)

Cross multiplying gives us the equation of the tangent line:

y − f(a) = f ′(a) (x− a) or y = f(a) + f ′(a) (x− a)

2 We are of course assuming that the curve is smooth enough to have a tangent line at a.

149

Page 166: CLP-1 Differential Calculus

DERIVATIVES 2.3 INTERPRETATIONS OF THE DERIVATIVE

Figure 2.3.3: A line segment of a tangent line

A second way to derive the same equation of the same tangent line is to recall thatthe general equation for a line, with finite slope, is y = mx + b, where m is the slopeand b is the y-intercept. We already know the slope — so m = f ′(a). To work out bwe use the other piece of information — (a, f(a)) is on the line. So (x, y) = (a, f(a))must solve y = f ′(a)x+ b. That is,

f(a) = f ′(a) · a+ b and so b = f(a)− af ′(a)

Hence our equation is, once again,

y = f ′(a) · x+ (f(a)− af ′(a)) or, after rearranging a little,y = f(a) + f ′(a) (x− a)

This is a very useful formula, so perhaps we should make it a theorem.

Theorem 2.3.4 Tangent line.

The tangent line to the curve y = f(x) at x = a is given by the equation

y = f(a) + f ′(a) (x− a)

provided the derivative f ′(a) exists.

The caveat at the end of the above theorem is necessary — there are certainlycases in which the derivative does not exist and so we do need to be careful.

150

Page 167: CLP-1 Differential Calculus

DERIVATIVES 2.3 INTERPRETATIONS OF THE DERIVATIVE

Example 2.3.5 A tangent line to the curve y =√x.

Find the tangent line to the curve y =√x at x = 4.

Rather than redoing everything from scratch, we can, and for efficiency, should, useTheorem 2.3.4. To write this up properly, we must ensure that we tell the readerwhat we are doing. So something like the following:

• By Theorem 2.3.4, the tangent line to the curve y = f(x) at x = a is given by

y = f(a) + f ′(a)(x− a)

provided f ′(a) exists.

• In Example 2.2.9, we found that, for any a > 0, the derivative of√x at x = a is

f ′(a) =1

2√a

• In the current example, a = 4 and we have

f(a) = f(4) =√x∣∣x=4

=√

4 = 2

and f ′(a) = f ′(4) =1

2√a

∣∣∣a=4

=1

2√

4=

1

4

• So the equation of the tangent line to y =√x at x = 4 is

y = 2 +1

4

(x− 4

)or y =

x

4+ 1

We don’t have to write it up using dot-points as above; we have used them here tohelp delineate each step in the process of computing the tangent line.

2.3.3 tt Exercises

Exercises — Stage 2

1. Suppose h(t) gives the height at time t of the water at a dam, where theunits of t are hours and the units of h are meters.

a What is the physical interpretation of the slope of the secant linethrough the points (0, h(0)) and (24, h(24))?

151

Page 168: CLP-1 Differential Calculus

DERIVATIVES 2.3 INTERPRETATIONS OF THE DERIVATIVE

b What is the physical interpretation of the slope of the tangent lineto the curve y = h(t) at the point (0, h(0))?

2. Suppose p(t) is a function that gives the profit generated by selling t wid-gets. What is the practical interpretation of p′(t)?

3. T (d) gives the temperature of water at a particular location d metres belowthe surface. What is the physical interpretation of T ′(d)? Would you expectthe magnitude of T ′(d) to be larger when d is near 0, or when d is very large?

4. C(w) gives the calories in w grams of a particular dish. What does C ′(w)describe?

5. The velocity of a moving object at time t is given by v(t). What is v′(t)?

6. The function T (j) gives the temperature in degrees Celsius of a cup ofwater after j joules of heat have been added. What is T ′(j)?

7. A population of bacteria, left for a fixed amount of time at temperatureT , grows to P (T ) individuals. Interpret P ′(T ).

Exercises — Stage 3

8. You hammer a small nail into a wooden wagon wheel. R(t) gives the num-ber of rotations the nail has undergone t seconds after the wagon startedto roll. Give an equation for how quickly the nail is rotating, measured indegrees per second.

9. A population of bacteria, left for a fixed amount of time at temperatureT , grows to P (T ) individuals. There is one ideal temperature wherethe bacteria population grows largest, and the closer the sample is tothat temperature, the larger the population is (unless the temperature isso extreme that it causes all the bacteria to die by freezing or boiling).How will P ′(T ) tell you whether you are colder or hotter than the idealtemperature?

152

Page 169: CLP-1 Differential Calculus

DERIVATIVES 2.4 ARITHMETIC OF DERIVATIVES - A DIFFERENTIATION TOOLBOX

2.4q Arithmetic of Derivatives - a Differentiation Toolbox

2.4.1 tt Arithmetic of Derivatives - a Differentiation Toolbox

So far, we have evaluated derivatives only by applying Definition 2.2.1 to the func-tion at hand and then computing the required limits directly. It is quite obvious thatas the function being differentiated becomes even a little complicated, this procedurequickly becomes extremely unwieldy. It is many orders of magnitude more efficientto have access to

• a list of derivatives of some simple functions and

• a collection of rules for breaking down complicated derivative computationsinto sequences of simple derivative computations.

This is precisely what we did to compute limits. We started with limits of simplefunctions and then used “arithmetic of limits” to computed limits of complicatedfunctions.

We have already started building our list of derivatives of simple functions. Wehave shown, in Examples 2.2.2, 2.2.3, 2.2.5 and 2.2.9, that

d

dx1 = 0

d

dxx = 1

d

dxx2 = 2x

d

dx

√x =

1

2√x

We’ll expand this list later.We now start building a collection of tools that help reduce the problem of com-

puting the derivative of a complicated function to that of computing the derivativesof a number of simple functions. In this section we give three derivative “rules” asthree separate theorems. We’ll give the proofs of these theorems in the next sectionand examples of how they are used in the following section.

As was the case for limits, derivatives interact very cleanly with addition, sub-traction and multiplication by a constant. The following result actually follows verydirectly from the first three points of Theorem 1.4.3.

Lemma 2.4.1 Derivative of sum and difference.

Let f(x), g(x) be differentiable functions and let c ∈ R be a constant. Then

d

dx

f(x) + g(x)

= f ′(x) + g′(x)

d

dx

f(x)− g(x)

= f ′(x)− g′(x)

d

dx

cf(x)

= cf ′(x)

That is, the derivative of the sum is the sum of the derivatives, and so forth.

153

Page 170: CLP-1 Differential Calculus

DERIVATIVES 2.4 ARITHMETIC OF DERIVATIVES - A DIFFERENTIATION TOOLBOX

Following this we can combine the three statements in this lemma into a singlerule which captures the “linearity of differentiation”.

Theorem 2.4.2 Linearity of differentiation.

Again, let f(x), g(x) be differentiable functions, let α, β ∈ R be constants anddefine the “linear combination”

S(x) = αf(x) + βg(x).

Then the derivative of S(x) at x = a exists and is

dS

dx= S ′(x) = αf ′(x) + βg′(x).

Note that we can recover the three rules in the previous lemma by setting α =β = 1 or α = 1, β = −1 or α = c, β = 0.

Unfortunately, the derivative does not act quite as simply on products or quo-tients. The rules for computing derivatives of products and quotients get their ownnames and theorems:

Theorem 2.4.3 The product rule.

Let f(x), g(x) be differentiable functions, then the derivative of the productf(x)g(x) exists and is given by

d

dx

f(x) g(x)

= f ′(x) g(x) + f(x) g′(x).

Before we proceed to the derivative of the ratio of two functions, it is worth notinga special case of the product rule when g(x) = f(x). In fact, since this is a usefulspecial case, let us call it a corollary 1:

Corollary 2.4.4 Derivative of a square.

Let f(x) be a differentiable function, then the derivative of its square is:

d

dx

f(x)2

= 2 f(x) f ′(x)

1 Recall that a corollary is an important result that follows from one or more theorems — typicallywithout too much extra work — as is the case here.

154

Page 171: CLP-1 Differential Calculus

DERIVATIVES 2.4 ARITHMETIC OF DERIVATIVES - A DIFFERENTIATION TOOLBOX

With a little work this can be generalised to other powers — but that is best doneonce we understand how to compute the derivative of the composition of two func-tions. That requires the chain rule (see Theorem 2.9.2 below). But before we get tothat, we need to see how to take the derivative of a quotient of two functions.

Theorem 2.4.5 The quotient rule.

Let f(x), g(x) be differentiable functions. Then the derivative of their quotientis

d

dx

f(x)

g(x)

=f ′(x) g(x)− f(x) g′(x)

g(x)2.

This derivative exists except at points where g(x) = 0.

There is a useful special case of this theorem which we obtain by setting f(x) = 1.In that case, the quotient rule tells us how to compute the derivative of the reciprocalof a function.

Corollary 2.4.6 Derivative of a reciprocal.

Let g(x) be a differentiable function. Then the derivative of the reciprocal of gis given by

d

dx

1

g(x)

= − g

′(x)

g(x)2

and exists except at those points where g(x) = 0.

So we have covered, sums, differences, products and quotients. This allows usto compute derivatives of many different functions — including polynomials andrational functions. However we are still missing trigonometric functions (for exam-ple), and a rule for computing derivatives of compositions. These will follow in thenear future, but there are a couple of things to do before that — understand wherethe above theorems come from, and practice using them.

2.4.2 tt Exercises

Exercises — Stage 1

1. True or false:d

dxf(x)+g(x) = f ′(x)+g′(x) when f and g are differentiable

functions.

155

Page 172: CLP-1 Differential Calculus

DERIVATIVES 2.4 ARITHMETIC OF DERIVATIVES - A DIFFERENTIATION TOOLBOX

2. True or false:d

dxf(x)g(x) = f ′(x)g′(x) when f and g are differentiable

functions.

3. True or false:d

dx

f(x

g(x)

=f ′(x)

g(x)− f(x)g′(x)

g2(x)when f and g are differen-

tiable functions.4. Let f be a differentiable function. Use at least three different rules to differ-

entiate g(x) = 3f(x), and verify that they all give the same answer.

Exercises — Stage 2

5. Differentiate f(x) = 3x5 + 4x2/3.

6. Use the product rule to differentiate f(x) = (2x+ 5)(8√x− 9x).

7. *. Find the equation of the tangent line to the graph of y = x3 at x =1

2.

8. *. A particle moves along the x–axis so that its position at time t is givenby x = t3 − 4t2 + 1 .

a At t = 2, what is the particle’s speed?

b At t = 2, in what direction is the particle moving?

c At t = 2, is the particle’s speed increasing or decreasing?

9. *. Calculate and simplify the derivative of2x− 1

2x+ 1

10. What is the slope of the graph y =

(3x+ 1

3x− 2

)2

when x = 1?

11. Find the equation of the tangent line to the curve f(x) =1√x+ 1

at the

point(1, 1

2

).

Exercises — Stage 3

12. A town is founded in the year 2000. After t years, it has had b(t) birthsand d(t) deaths. Nobody enters or leaves the town except by birth ordeath (whoa). Give an expression for the rate the population of the townis growing.

156

Page 173: CLP-1 Differential Calculus

DERIVATIVES 2.5 PROOFS OF THE ARITHMETIC OF DERIVATIVES

13. *. Find all points on the curve y = 3x2 where the tangent line passes through(2, 9).

14. *. Evaluate limy→0

(√100180 + y −

√100180

y

)by interpreting the limit as

a derivative.

15. A rectangle is growing. At time t = 0, it is a square with side length 1metre. Its width increases at a constant rate of 2 metres per second, andits length increases at a constant rate of 5 metres per second. How fastis its area increasing at time t > 0?

16. Let f(x) = x2g(x) for some differentiable function g(x). What is f ′(0)?

17. Verify that differentiating f(x) =g(x)

h(x)using the quotient rule gives the

same answer as differentiating f(x) =g(x)

k(x)· k(x)

h(x)using the product rule

and the quotient rule.

2.5q Proofs of the Arithmetic of Derivatives

The theorems of the previous section are not too difficult to prove from the definitionof the derivative (which we know) and the arithmetic of limits (which we also know).In this section we show how to construct these rules.

Throughout this section we will use our two functions f(x) and g(x). Since thetheorems we are going to prove all express derivatives of linear combinations, prod-ucts and quotients in terms of f, g and their derivatives, it is helpful to recall thedefinitions of the derivatives of f and g:

f ′(x) = limh→0

f(x+ h)− f(x)

hand g′(x) = lim

h→0

g(x+ h)− g(x)

h.

Our proofs, roughly speaking, involve doing algebraic manipulations to uncover theexpressions that look like the above.

2.5.1 tt Proof of the Linearity of Differentiation (Theorem 2.4.2)

Recall that in Theorem 2.4.2 we defined S(x) = α f(x) + β g(x), where α, β ∈ R areconstants. We wish to compute S ′(x), so we start with the definition:

S ′(x) = limh→0

S(x+ h)− S(x)

h

157

Page 174: CLP-1 Differential Calculus

DERIVATIVES 2.5 PROOFS OF THE ARITHMETIC OF DERIVATIVES

Let us concentrate on the numerator of the expression inside the limit and then comeback to the full limit in a moment. Substitute in the definition of S(x):

S(x+ h)− S(x) =[αf(x+ h) + βg(x+ h)

]−[αf(x) + βg(x)

]collect terms

= α[f(x+ h)− f(x)] + β

[g(x+ h)− g(x)

]Now it is easy to see the structures we need — namely, we almost have the expres-sions for the derivatives f ′(x) and g′(x). Indeed, all we need to do is divide by h andtake the limit. So let’s finish things off.

S ′(x) = limh→0

S(x+ h)− S(x)

hfrom above

= limh→0

α[f(x+ h)− f(x)] + β

[g(x+ h)− g(x)

]h

= limh→0

[αf(x+ h)− f(x)

h+ β

g(x+ h)− g(x)

h

]limit laws

= α limh→0

f(x+ h)− f(x)

h+ β lim

h→0

g(x+ h)− g(x)

h= αf ′(x) + βg′(x)

as required.

2.5.2 tt Proof of the Product Rule (Theorem 2.4.3)

After the warm-up above, we will just jump straight in. Let P (x) = f(x) g(x), theproduct of our two functions. The derivative of the product is given by

P ′(x) = limh→0

P (x+ h)− P (x)

h

Again we will focus on the numerator inside the limit and massage it into the formwe need. To simplify these manipulations, define

F (h) =f(x+ h)− f(x)

hand G(h) =

g(x+ h)− g(x)

h.

Then we can write

f(x+ h) = f(x) + hF (h) and g(x+ h) = g(x) + hG(h).

We can also write

f ′(x) = limh→0

F (h) and g′(x) = limh→0

G(h).

So back to that numerator:

P (x+ h)− P (x) = f(x+ h) · g(x+ h)− f(x) · g(x) substitute

158

Page 175: CLP-1 Differential Calculus

DERIVATIVES 2.5 PROOFS OF THE ARITHMETIC OF DERIVATIVES

= [f(x) + hF (h)] [g(x) + hG(h)]− f(x) · g(x) expand= f(x)g(x) + f(x) · hG(h) + hF (h) · g(x) + h2F (h) ·G(h)− f(x) · g(x)

= f(x) · hG(h) + hF (h) · g(x) + h2F (h) ·G(h).

Armed with this we return to the definition of the derivative:

P ′(x) = limh→0

P (x+ h)− P (x)

h

= limh→0

f(x) · hG(h) + hF (h) · g(x) + h2F (h) ·G(h)

h

=

(limh→0

f(x) · hG(h)

h

)+

(limh→0

hF (h) · g(x)

h

)+

(limh→0

h2F (h) ·G(h)

h

)=(

limh→0

f(x) ·G(h))

+(

limh→0

F (h) · g(x))

+(

limh→0

hF (h) ·G(h))

Now since f(x) and g(x) do not change as we send h to zero, we can pull themoutside. We can also write the third term as the product of 3 limits:

=(f(x) lim

h→0G(h)

)+(g(x) lim

h→0F (h)

)+(

limh→0

h)·(

limh→0

F (h))·(

limh→0

G(h))

= f(x) · g′(x) + g(x) · f ′(x) + 0 · f ′(x) · g′(x)

= f(x) · g′(x) + g(x) · f ′(x).

And so we recover the product rule.

2.5.3 tt (Optional) — Proof of the Quotient Rule (Theorem 2.4.5)

We now give the proof of the quotient rule in two steps1. We assume throughout thatg(x) 6= 0 and that f(x) and g(x) are differentiable, meaning that the limits definingf ′(x), g′(x) exist.

• In the first step, we prove the quotient rule under the assumption that f(x)/g(x)is differentiable.

• In the second step, we prove that 1/g(x) differentiable. Once we know that1/g(x) is differentiable, the product rule implies that f(x)/g(x) is differentiable.

Step 1: the proof of the quotient rule assumng that f(x)g(x)

is differentiable. WriteQ(x) = f(x)g(x)

.Then f(x) = g(x)Q(x) so that f ′(x) = g′(x)Q(x) + g(x)Q′(x), by the product rule,and

Q′(x) =f ′(x)− g′(x)Q(x)

g(x)=f ′(x)− g′(x) f(x)

g(x)

g(x)

=f ′(x)g(x)− f(x)g′(x)

g(x)2

1 We thank Serban Raianu for suggesting this approach.

159

Page 176: CLP-1 Differential Calculus

DERIVATIVES 2.6 USING THE ARITHMETIC OF DERIVATIVES – EXAMPLES

Step 2: the proof that 1/g(x) is differentiable. By definition

d

dx

1

g(x)= lim

h→0

1

h

[1

g(x+ h)− 1

g(x)

]= lim

h→0

g(x)− g(x+ h)

h g(x) g(x+ h)

= − limh→0

1

g(x)

1

g(x+ h)

g(x+ h)− g(x)

h

= − 1

g(x)limh→0

1

g(x+ h)limh→0

g(x+ h)− g(x)

h

= − 1

g(x)2g′(x)

2.6q Using the Arithmetic of Derivatives – Examples

2.6.1 tt Using the Arithmetic of Derivatives – Examples

In this section we illustrate the computation of derivatives using the arithmetic ofderivatives — Theorems 2.4.2, 2.4.3 and 2.4.5. To make it clear which rules we areusing during the examples we will note which theorem we are using:

•LIN to stand for “linearity” ddxα f(x) + β g(x) = α f ′(x) + β g′(x) Theorem 2.4.2

•PR to stand for “product rule” ddxf(x) g(x) = f ′(x) g(x) + f(x) g′(x) Theorem 2.4.3

•QR to stand for “quotient rule” ddx

f(x)g(x)

= f ′(x) g(x)−f(x) g′(x)

g(x)2Theorem 2.4.5

We’ll start with a really easy example.

Example 2.6.1 ddx4x+ 7.

d

dx4x+ 7 = 4 · d

dxx+ 7 · d

dx1 LIN

= 4 · 1 + 7 · 0 = 4

where we have used LIN with f(x) = x, g(x) = 1, α = 4, β = 7.

Example 2.6.2 ddx

x(4x+ 7)

.

Continuing on from the previous example, we can use the product rule and the pre-

160

Page 177: CLP-1 Differential Calculus

DERIVATIVES 2.6 USING THE ARITHMETIC OF DERIVATIVES – EXAMPLES

Example 2.6.2

vious result to compute

d

dx

x(4x+ 7)

= x · d

dx4x+ 7+ (4x+ 7)

d

dxx PR

= x · 4 + (4x+ 7) · 1= 8x+ 7

where we have used the product rule PR with f(x) = x and g(x) = 4x+ 7.

Example 2.6.3 ddx

x

4x+7

.

In the same vein as the previous example, we can use the quotient rule to compute

d

dx

x

4x+ 7

=

(4x+ 7) · ddxx − x · d

dx4x+ 7

(4x+ 7)2QR

=(4x+ 7) · 1− x · 4

(4x+ 7)2

=7

(4x+ 7)2

where we have used the quotient rule QR with f(x) = x and g(x) = 4x+ 7.

Now for a messier example.

Example 2.6.4 Some examples should be messy.

Differentiate

f(x) =x

2x+ 13x+1

This problem looks nasty. But it isn’t so hard if we just build it up a bit at a time.

• First, f(x) is the ratio of

f1(x) = x and f2(x) = 2x+1

3x+ 1

If we can find the derivatives of f1(x) and f2(x), we will be able to get thederivative of f(x) just by applying the quotient rule. The derivative, f ′1(x) = 1,of f1(x) is easy, so let’s work on f2(x).

• The function f2(x) is the linear combination

f2(x) = 2f3(x) + f4(x) with f3(x) = x and f4(x) =1

3x+ 1

161

Page 178: CLP-1 Differential Calculus

DERIVATIVES 2.6 USING THE ARITHMETIC OF DERIVATIVES – EXAMPLES

If we can find the derivatives of f3(x) and f4(x), we will be able to get thederivative of f2(x) just by applying linearity (Theorem 2.4.2). The derivative,f ′3(x) = 1, of f3(x) is easy. So let’s work of f4(x).

• The function f4(x) is the ratio

f4(x) =1

f5(x)with f5(x) = 3x+ 1

If we can find the derivative of f5(x), we will be able to get the derivative off4(x) just by applying the special case the quotient rule (Corollary 2.4.6). Thederivative of f5(x) is easy.

• So we have completed breaking down f(x) into easy pieces. It is now just amatter of reversing the break down steps, putting everything back together,starting with the easy pieces and working up to f(x). Here goes.

f5(x) = 3x+ 1

sod

dxf5(x) = 3

d

dxx+

d

dx1 = 3 · 1 + 0 = 3 LIN

f4(x) =1

f5(x)

sod

dxf4(x) = − f ′5(x)

f5(x)2= − 3

(3x+ 1)2QR

f2(x) = 2f3(x) + f4(x)

sod

dxf2(x) = 2f ′3(x) + f ′4(x) = 2− 3

(3x+ 1)2LIN

f(x) =f1(x)

f2(x)

sod

dxf(x) =

f ′1(x)f2(x)− f1(x)f ′2(x)

f2(x)2QR

=1[2x+ 1

3x+1

]− x[2− 3

(3x+1)2

][2x+ 1

3x+1

]2Oof!

• We now have an answer. But we really should clean it up, not only to makeit easier to read, but also because invariably such computations are just smallsteps inside much larger computations. Any future computations involvingthis expression will be a lot easier and less error prone if we clean it up now.Cancelling the 2x and the −2x in

1[2x+

1

3x+1

]− x[2− 3

(3x+1)2

]= 2x+

1

3x+1− 2x+

3x

(3x+1)2

162

Page 179: CLP-1 Differential Calculus

DERIVATIVES 2.6 USING THE ARITHMETIC OF DERIVATIVES – EXAMPLES

Example 2.6.4

=1

3x+1+

3x

(3x+1)2

and multiplying both the numerator and denominator by (3x+ 1)2 gives

f ′(x) =

13x+1

+ 3x(3x+1)2[

2x+ 13x+1

]2 (3x+ 1)2

(3x+ 1)2

=(3x+ 1) + 3x[

2x(3x+ 1) + 1]2

=6x+ 1

[6x2 + 2x+ 1]2.

While the linearity theorem (Theorem 2.4.2) is stated for a linear combination oftwo functions, it is not difficult to extend it to linear combinations of three or morefunctions as the following example shows.

Example 2.6.5 Linearity of the derivative of three or more functions.

We’ll start by generalising linearity to three functions.

d

dx

aF (x) + bG(x) + cH(x)

=

d

dx

a · [F (x)] + 1 · [bG(x) + cH(x)]

= aF ′(x) +

d

dxbG(x) + cH(x)

by LIN with α = a, f(x) = F (x), β = 1,and g(x) = bG(x) + cH(x)

= aF ′(x) + bG′(x) + cH ′(x)

by LIN with α = b, f(x) = G(x), β = c,

and g(x) = H(x)

This gives us linearity for three terms, namely (just replacing upper case names bylower case names)

d

dxaf(x) + bg(x) + ch(x) = af ′(x) + bg′(x) + ch′(x)

Just by repeating the above argument many times, we may generalise to linearity forn terms, for any natural number n:

d

dxa1f1(x) + a2f2(x) + · · ·+ anfn(x)

= a1f′1(x) + a2f

′2(x) + · · ·+ anf

′n(x)

163

Page 180: CLP-1 Differential Calculus

DERIVATIVES 2.6 USING THE ARITHMETIC OF DERIVATIVES – EXAMPLES

Similarly, while the product rule is stated for the product of two functions, it isnot difficult to extend it to the product of three or more functions as the followingexample shows.

Example 2.6.6 Extending the product rule to more than two factors.

Once again, we’ll start by generalising the product rule to three factors.

d

dxF (x)G(x)H(x) = F ′(x)G(x)H(x) + F (x)

d

dxG(x)H(x)

by PR withf(x) = F (x) and g(x) = G(x)H(x)

= F ′(x)G(x)H(x) + F (x)G′(x)H(x) +G(x)H ′(x)

by PR withf(x) = G(x) and g(x) = H(x)

This gives us a product rule for three factors, namely (just replacing upper casenames by lower case names)

d

dxf(x) g(x)h(x) = f ′(x) g(x)h(x) + f(x) g′(x)h(x) + f(x) g(x)h′(x)

Observe that when we differentiate a product of three factors, the answer is a sum ofthree terms and in each term the derivative acts on exactly one of the original factors.Just by repeating the above argument many times, we may generalise the productrule to give the derivative of a product of n factors, for any natural number n:

d

dxf1(x) f2(x) · · · fn(x) = f ′1(x) f2(x) · · · fn(x)

+f1(x) f ′2(x) · · · fn(x)

...+f1(x) f2(x) · · · f ′n(x)

We can also write the above as

d

dxf1(x) f2(x) · · · fn(x)

=

[f ′1(x)

f1(x)+f ′2(x)

f2(x)+ · · ·+ f ′n(x)

fn(x)

]· f1(x) f2(x) · · · fn(x)

When we differentiate a product of n factors, the answer is a sum of n terms andin each term the derivative acts on exactly one of the original factors. In the firstterm, the derivative acts on the first of the original factors. In the second term, thederivative acts on the second of the original factors. And so on.If we make f1(x) = f2(x) = · · · = fn(x) = f(x) then each of the n terms on the righthand side of the above equation is the product of f ′(x) and exactly n− 1 f(x)’s, andso is exactly f(x)n−1 f ′(x). So we get the following useful result

d

dxf(x)n = n · f(x)n−1 · f ′(x).

164

Page 181: CLP-1 Differential Calculus

DERIVATIVES 2.6 USING THE ARITHMETIC OF DERIVATIVES – EXAMPLES

This last result is quite useful, so let us write it as a lemma for future reference.

Lemma 2.6.7

Let n be a natural number and f be a differentiable function. Then

d

dxf(x)n = n · f(x)n−1 · f ′(x).

This immediately gives us another useful result.

Example 2.6.8 Derivative of xn.

We can now compute the derivative of xn for any natural number n. Start withLemma 2.6.7 and substitute f(x) = x and f ′(x) = 1:

d

dxxn = n · xn−1 · 1 = nxn−1

Again — this is a result we will come back to quite a few times in the future, sowe should make sure we can refer to it easily. However, at present this statementonly holds when n is a positive integer. With a little more work we can extend this tocompute xq where q is any positive rational number and then any rational numberat all (positive or negative). So let us hold off for a little longer. Instead we can makeit a lemma, since it will be an ingredient in quite a few of the examples followingbelow and in constructing the final corollary.

Lemma 2.6.9 Derivative of xn.

Let n be a positive integer then

d

dxxn = nxn−1

Back to more examples.

Example 2.6.10 Derivative of a polynomial.

d

dx

2x3 + 4x5

= 2

d

dxx3+ 4

d

dxx5

by LIN with α = 2, f(x) = x3, β = 4 and g(x) = x5

= 23x2+ 45x4

165

Page 182: CLP-1 Differential Calculus

DERIVATIVES 2.6 USING THE ARITHMETIC OF DERIVATIVES – EXAMPLES

Example 2.6.10

by Lemma 2.6.9, once with n = 3, and once with n = 5

= 6x2 + 20x4

Example 2.6.11 Derivative of product of polynomials.

In this example we’ll compute ddx

(3x+ 9)(x2 + 4x3)

in two different ways. For the

first, we’ll start with the product rule.

d

dx

(3x+ 9)(x2 + 4x3)

= d

dx(3x+ 9)

(x2 + 4x3) + (3x+ 9)

d

dxx2 + 4x3

=

3× 1 + 9× 0

(x2 + 4x3) + (3x+ 9) 2x+ 4(3x2)= 3(x2 + 4x3) + (3x+ 9) (2x+ 12x2)

= 3x2 + 12x3 + (6x2 + 18x+ 36x3 + 108x2)

= 18x+ 117x2 + 48x3

For the second, we expand the product first and then differentiate.

d

dx

(3x+ 9)(x2 + 4x3)

=

d

dx

9x2 + 39x3 + 12x4

= 9(2x) + 39(3x2) + 12(4x3)

= 18x+ 117x2 + 48x3

Example 2.6.12 Derivative of a rational function.

d

dx

4x3 − 7x

4x2 + 1

=

(12x2 − 7)(4x2 + 1)− (4x3 − 7x)(8x)

(4x2 + 1)2

by QR with f(x) = 4x3 − 7x, f ′(x) = 12x2 − 7,

and g(x) = 4x2 + 1, g′(x) = 8x

=(48x4 − 16x2 − 7)− (32x4 − 56x2)

(4x2 + 1)2

=16x4 + 40x2 − 7

(4x2 + 1)2

166

Page 183: CLP-1 Differential Calculus

DERIVATIVES 2.6 USING THE ARITHMETIC OF DERIVATIVES – EXAMPLES

Example 2.6.13 Derivative of a cube-root.

In this example, we’ll use a little trickery to find the derivative of 3√x. The trickery

consists of observing that, by the definition of the cube root,

x =(

3√x)3.

Since both sides of the expression are the same, they must have the same derivatives:

d

dxx =

d

dx

(3√x)3.

We already know by Theorem 2.2.4 that

d

dx

x

= 1

and that, by Lemma 2.6.7 with n = 3 and f(x) = 3√x,

d

dx

(3√x)3

= 3(

3√x)2 · d

dx

3√x

= 3x2/3 · d

dx

3√x.

Since we know that ddxx = d

dx( 3√x)

3, we must have

1 = 3x2/3 · d

dx

3√x

which we can rearrange to give the result we need

d

dx

3√x

= 13x−2/3

Example 2.6.14 Derivative of a positive rational power of x.

In this example, we’ll use the same trickery as in Example 2.6.13 to find the derivativexp/q for any two natural numbers p and q. By definition of the qth root,

xp =(xp/q

)q.

That is, xp and(xp/q

)q are the same function, and so have the same derivative. So wedifferentiate both of them. We already know that, by Lemma 2.6.9 with n = p,

d

dx

xp

= pxp−1

and that, by Lemma 2.6.7 with n = q and f(x) = xp/q,

d

dx

(xp/q

)q= q

(xp/q

)q−1 d

dx

xp/q

167

Page 184: CLP-1 Differential Calculus

DERIVATIVES 2.6 USING THE ARITHMETIC OF DERIVATIVES – EXAMPLES

Example 2.6.14

Remember that (xa)b = x(a·b). Now these two derivatives must be the same. So

pxp−1 = q · x(pq−p)/q d

dx

xp/q

and, rearranging things,

d

dx

xp/q

=p

qxp−1−(pq−p)/q

=p

qx(pq−q−pq+p)/q

=p

qxpq−1

So finallyd

dx

xp/q

=p

qxpq−1

Notice that this has the same form as Lemma 2.6.9, above, except with n = pq

allowedto be any positive rational number, not just a positive integer.

Example 2.6.15 Derivative of x−n.

In this example we’ll use the quotient rule to find the derivative of x−m, for anynatural number m.By the special case of the quotient rule (Corollary 2.4.6) with g(x) = xm and g′(x) =mxm−1

d

dx

x−m

=

d

dx

1

xm

= − mx

m−1

(xm)2 = −mx−m−1

Again, notice that this has the same form as Lemma 2.6.9, above, except with n = −mbeing a negative integer.

Example 2.6.16 Derivative of a negative rational power of x.

In this example we’ll use the quotient rule to find the derivative of x−p/q, for any pairof natural numbers p and q. By the special case the quotient rule (Corollary 2.4.6)with g(x) = x

pq and g′(x) = p

qxpq−1,

d

dx

x−

pq

=d

dx

1

xpq

= −

pqxpq−1

(xpq )

2 = −pqx−

pq−1

168

Page 185: CLP-1 Differential Calculus

DERIVATIVES 2.6 USING THE ARITHMETIC OF DERIVATIVES – EXAMPLES

Note that we have found, in Examples 2.2.2, 2.6.14 and 2.6.16, the derivative ofxa for any rational number a, whether 0, positive, negative, integer or fractional. Inall cases, the answer is

Corollary 2.6.17 Derivative of xa.

Let a be a rational number, then

d

dxxa = axa−1

We shall show, in Example 2.10.5, that the formula ddxxa = axa−1 in fact applies

for all real numbers a, not just rational numbers.Back in Example 2.2.9 we computed the derivative of

√x from the definition of

the derivative. The above corollary (correctly) gives

d

dxx

12 =

1

2x−

12

but with far less work.Here’s an (optional) messy example.

Example 2.6.18 Optional messy example.

Find the derivative of

f(x) =(√x− 1)(2− x)(1− x2)√

x(3 + 2x)

• As we seen before, the best strategy for dealing with nasty expressions is tobreak them up into easy pieces. We can think of f(x) as the five–fold product

f(x) = f1(x) · f2(x) · f3(x) · 1

f4(x)· 1

f5(x)

with

f1(x) =√x− 1 f2(x) = 2− x f3(x) = 1− x2

f4(x) =√x f5(x) = 3 + 2x

• By now, the derivatives of the fj’s should be easy to find:

f ′1(x) =1

2√x

f ′2(x) = −1 f ′3(x) = −2x

f ′4(x) =1

2√x

f ′5(x) = 2

169

Page 186: CLP-1 Differential Calculus

DERIVATIVES 2.6 USING THE ARITHMETIC OF DERIVATIVES – EXAMPLES

Example 2.6.18

• Now, to get the derivative f(x) we use the n–fold product rule which was de-veloped in Example 2.6.6, together with the special case of the quotient rule(Corollary 2.4.6).

f ′(x) = f ′1f2f31

f4

1

f5

+ f1f′2f3

1

f4

1

f5

+ f1f2f′3

1

f4

1

f5

− f1f2f3f ′4f 2

4

1

f5

− f1f2f31

f4

f ′5f 2

5

=[f ′1f1

+f ′2f2

+f ′3f3

− f ′4f4

− f ′5f5

]f1f2f3

1

f4

1

f5

=

[1

2√x(√x− 1)

− 1

2− x −2x

1− x2− 1

2x− 2

3 + 2x

](√x− 1)(2− x)(1− x2)√

x(3 + 2x)

The trick that we used in going from the first line to the second line, namelymultiplying term number j by fj(x)

fj(x)is often useful in simplifying the derivative

of a product of many factors a.

a Also take a look at “logarithmic differentiation” in Section 2.10.

2.6.2 tt Exercises

Exercises — Stage 1

1. Spot and correct the error(s) in the following calculation.

f(x) =2x

x+ 1

f ′(x) =2(x+ 1) + 2x

(x+ 1)2

=2(x+ 1)

(x+ 1)2

=2

x+ 1

170

Page 187: CLP-1 Differential Calculus

DERIVATIVES 2.6 USING THE ARITHMETIC OF DERIVATIVES – EXAMPLES

2. True or false:d

dx2x = x2x−1.

Exercises — Stage 2

3. Differentiate f(x) = 23x6 + 5x4 + 12x2 + 9 and factor the result.

4. Differentiate s(t) = 3t4 + 5t3 − 1t.

5. Differentiate x(y) =(

2y + 1y

)· y3.

6. Differentiate T (x) =

√x+ 1

x2 + 3.

7. *. Compute the derivative of(

7x+ 2

x2 + 3

).

8. What is f ′(0), when f(x) = (3x3 + 4x2 + x+ 1)(2x+ 5)?

9. Differentiate f(x) =3x3 + 1

x2 + 5x.

10. *. Compute the derivative of(

3x2 + 5

2− x

)11. *. Compute the derivative of

(2− x2

3x2 + 5

).

12. *. Compute the derivative of(

2x3 + 1

x+ 2

).

13. *. For what values of x does the derivative of√x

1− x2exist? Explain your

answer.

14. Differentiate f(x) = (3 5√x+ 15 3

√x+ 8) (3x2 + 8x− 5).

15. Differentiate f(x) =(x2 + 5x+ 1)(

√x+ 3√x)

x.

16. Find all x-values where f(x) =1

15x5 + x4 − 5

3x3

has a horizontal tangent

line.

Exercises — Stage 3

171

Page 188: CLP-1 Differential Calculus

DERIVATIVES 2.7 DERIVATIVES OF EXPONENTIAL FUNCTIONS

17. *. Find an equation of a line that is tangent to both of the curves y = x2 andy = x2 − 2x+ 2 (at different points).

18. [1998H] Find all lines that are tangent to both of the curves y = x2 andy = −x2 + 2x− 5. Illustrate your answer with a sketch.

19. *. Evaluate limx→2015

(cos(x)− cos(2015)

x− 2015

).

20. *. Evaluate limx→π/3

(cos(x)− 1/2

x− π/3

).

21. *. Evaluate limx→π

(sin(x)

x− π

).

22. *. Evaluate limx→2

(x2015 − 22015

x− 2

).

2.7q Derivatives of Exponential Functions

Now that we understand how derivatives interact with products and quotients, weare able to compute derivatives of

• polynomials,

• rational functions, and

• powers and roots of rational functions.

Notice that all of the above come from knowing 1 the derivative of xn and applyinglinearity of derivatives and the product rule.

There is still one more “rule” that we need to complete our toolbox and that isthe chain rule. However before we get there, we will add a few functions to our listof things we can differentiate 2. The first of these is the exponential function.

Let a > 0 and set f(x) = ax — this is what is known as an exponential function.Let’s see what happens when we try to compute the derivative of this function justusing the definition of the derivative.

df

dx= lim

h→0

f(x+ h)− f(x)

h= lim

h→0

ax+h − axh

1 Differentiating powers and roots of functions is actually quite a bit easier once one knows thechain rule — which we will discuss soon.

2 One reason we add these functions is that they interact very nicely with the derivative. Anotherreason is that they turn up in many “real world” examples.

172

Page 189: CLP-1 Differential Calculus

DERIVATIVES 2.7 DERIVATIVES OF EXPONENTIAL FUNCTIONS

= limh→0

ax · ah − 1

h= ax · lim

h→0

ah − 1

h

Unfortunately we cannot complete this computation because we cannot evaluate thelast limit directly. For the moment, let us assume this limit exists and name it

C(a) = limh→0

ah − 1

h

It depends only on a and is completely independent of x. Using this notation (whichwe will quickly improve upon below), our desired derivative is now

d

dxax = C(a) · ax.

Thus the derivative of ax is ax multiplied by some constant — i.e. the function ax

is nearly unchanged by differentiating. If we can tune a so that C(a) = 1 then thederivative would just be the original function! This turns out to be very useful.

To try finding an a that obeys C(a) = 1, let us investigate how C(a) changes witha. Unfortunately (though this fact is not at all obvious) there is no way to write C(a)as a finite combination of any of the functions we have examined so far 3. To getstarted, we’ll try to guess C(a), for a few values of a, by plugging in some smallvalues of h.

Example 2.7.1 Estimates of C(a).

Let a = 1 then C(1) = limh→0

1h − 1

h= 0. This is not surprising since 1x = 1 is constant,

and so its derivative must be zero everywhere. Let a = 2 then C(2) = limh→0

2h − 1

h.

Setting h to smaller and smaller numbers gives

h 0.1 0.01 0.001 0.0001 0.00001 0.000001 0.00000012h−1h

0.7177 0.6956 0.6934 0.6932 0.6931 0.6931 0.6931

Similarly when a = 3 we get

h 0.1 0.01 0.001 0.0001 0.00001 0.000001 0.00000013h−1h

1.1612 1.1047 1.0992 1.0987 1.0986 1.0986 1.0986

and a = 10

3 To a bit more be precise, we say that a number q is algebraic if we can write q as the zero of apolynomial with integer coefficients. When a is any positive algebraic number other 1, C(a) isnot algebraic. A number that is not algebraic is called transcendental. The best known exampleof a transcendental number is π (which follows from the Lindemann-Weierstrass Theorem —way beyond the scope of this course).

173

Page 190: CLP-1 Differential Calculus

DERIVATIVES 2.7 DERIVATIVES OF EXPONENTIAL FUNCTIONS

Example 2.7.1

h 0.1 0.01 0.001 0.0001 0.00001 0.000001 0.000000110h−1h

2.5893 2.3293 2.3052 2.3028 2.3026 2.3026 2.3026

From this example it appears that C(a) increases as we increase a, and that C(a) = 1for some value of a between 2 and 3.

We can learn a lot more about C(a), and, in particular, confirm the guesses thatwe made in the last example, by making use of logarithms — this would be a goodtime for you to review them.

2.7.1 ttWhirlwind Review of Logarithms

Before you read much further into this little review on logarithms, you should firstgo back and take a look at the review of inverse functions in Section 0.6.

2.7.1.1ttt Logarithmic Functions

We are about to define the “logarithm with base q”. In principle, q is allowed tobe any strictly positive real number, except q = 1. However we shall restrict ourattention to q > 1, because, in practice, the only q’s that are ever used are e (a numberthat we shall define in the next few pages), 10 and, if you are a computer scientist, 2.So, fix any q > 1 (if you like, pretend that q = 10). The function f(x) = qx

• increases as x increases (for example if x′ > x, then 10x′

= 10x · 10x′−x > 10x

since 10x′−x > 1)

• obeys limx→−∞

qx = 0 (for example 10−1000 is really small) and

• obeys limx→+∞

qx = +∞ (for example 10+1000 is really big).

Consequently, for any 0 < Y < ∞, the horizontal straight line y = Y crosses thegraph of y = f(x) = qx at exactly one point, as illustrated in the figure below.

174

Page 191: CLP-1 Differential Calculus

DERIVATIVES 2.7 DERIVATIVES OF EXPONENTIAL FUNCTIONS

The x–coordinate of that intersection point, denotedX in the figure, is logq(Y ). Sologq(Y ) is the power to which you have to raise q to get Y . It is the inverse functionof f(x) = qx. Of course we are free to rename the dummy variables X and Y . If, forexample, we wish to graph our logarithm function, it is natural to rename Y → xand X → y, giving

Definition 2.7.2

Let q > 1. Then the logarithm with base q is defined a by

y = logq(x)⇔ x = qy

a We can also define logarithms with base 0 < r < 1 but doing so is not necessary. To seethis, set q = 1/r > 1. Then it is reasonable to define logr(x) = − logq(x) since

rlogr(x) =

(1

q

)logr(x)

=

(1

q

)− logq(x)

= qlogq(x) = x

as required.

Obviously the power to which we have to raise q to get qx is x, so we have both

logq(qx) = x qlogq(x) = x

From the exponential properties

qlogq(xy) = xy = qlogq(x)qlogq(y) = qlogq(x)+logq(y)

qlogq(x/y) = x/y = qlogq(x)/qlogq(y) = qlogq(x)−logq(y)

qlogq(xr) = xr =

(qlogq(x)

)r= qrlogq(x)

we have

logq(xy) = logq(x) + logq(y)

logq(x/y) = logq(x)− logq(y)

logq(xr) = r logq(x)

175

Page 192: CLP-1 Differential Calculus

DERIVATIVES 2.7 DERIVATIVES OF EXPONENTIAL FUNCTIONS

Can we convert from logarithms in one base to logarithms in another? For example,if our calculator computes logarithms base 10 for us (which it very likely does), canwe also use it to compute a logarithm base q? Yes, using

logq(x) =log10 x

log10 q

How did we get this? Well, let’s start with a number x and suppose that we want tocompute

y = logq x

We can rearrange this by exponentiating both sides

qy = qlogq x = x

Now take log base 10 of both sides

log10 qy = log10 x

But recall that logq(xr) = r logq(x), so

y log10 q = log10 x

y =log10 x

log10 q

2.7.2 tt Back to that Limit

Recall that we are trying to choose a so that

limh→0

ah − 1

h= C(a) = 1.

We can estimate the correct value of a by using our numerical estimate of C(10)above. The way to do this is to first rewrite C(a) in terms of logarithms.

a = 10log10 a and so ah = 10h log10 a.

Using this we rewrite C(a) as

C(a) = limh→0

1

h

(10h log10 a − 1

)Now set H = h log10(a), and notice that as h→ 0 we also have H → 0

= limH→0

log10 a

H

(10H − 1

)176

Page 193: CLP-1 Differential Calculus

DERIVATIVES 2.7 DERIVATIVES OF EXPONENTIAL FUNCTIONS

= log10 a · limH→0

10H − 1

H= log10 a · C(10).

Below is a sketch of C(a) against a.

Figure 2.7.3

Remember that we are trying to find an a with C(a) = 1. We can do so by recog-nising that C(a) = C(10) (log10 a) has the following properties.

• When a = 1, log10(a) = log10 1 = 0 so that C(a) = C(10) log10(a) = 0. Of course,we should have expected this, because when a = 1 we have ax = 1x = 1 whichis just the constant function and d

dx1 = 0.

• log10 a increases as a increases, and hence C(a) = C(10) log10 a increases as aincreases.

• log10 a tends to +∞ as a→∞, and hence C(a) tends to +∞ as a→∞.

Hence the graph of C(a) passes through (1, 0), is always increasing as a increasesand goes off to +∞ as a goes off to +∞. See Figure 2.7.3. Consequently 4 there isexactly one value of a for which C(a) = 1.

The value of a for which C(a) = 1 is given the name e. It is called Euler’s constant5. In Example 2.7.1, we estimated C(10) ≈ 2.3026. So if we assume C(a) = 1 then the

4 We are applying the Intermediate Value Theorem here, but we have neglected to verify the hy-pothesis that log10(a) is a continuous function. Please forgive us — we could do this if we reallyhad to, but it would make a big mess without adding much understanding, if we were to do sohere in the text. Better to just trust us on this.

5 Unfortunately there is another Euler’s constant, γ, which is more properly called the Eu-ler–Mascheroni constant. Anyway like many mathematical discoveries, e was first found bysomeone else — Napier used the constant e in order to compute logarithms but only implicitly.Bernoulli was probably the first to approximate it when examining continuous compound inter-est. It first appeared explicitly in work of Leibniz, though he denoted it b. It was Euler, though,who established the notation we now use and who showed how important the constant is tomathematics.

177

Page 194: CLP-1 Differential Calculus

DERIVATIVES 2.7 DERIVATIVES OF EXPONENTIAL FUNCTIONS

above equation becomes

2.3026 · log10 a ≈ 1

log10 a ≈1

2.3026≈ 0.4343

a ≈ 100.4343 ≈ 2.7813

This gives us the estimate a ≈ 2.7813 which is not too bad. In fact 6

Equation 2.7.4 Euler’s constant.

e = 2.7182818284590452354 . . .

= 1 +1

1!+

1

2!+

1

3!+

1

4!+ · · ·

We will be able to explain this last formula once we develop Taylor polynomialslater in the course.

To summarise

Theorem 2.7.5

The constant e is the unique real number that satisfies

limh→0

eh − 1

h= 1

Further,

d

dx(ex) = ex

We plot ex in the graph below

6 Recall n factorial, written n! is the product n× (n− 1)× (n− 2)× · · · × 2× 1.

178

Page 195: CLP-1 Differential Calculus

DERIVATIVES 2.7 DERIVATIVES OF EXPONENTIAL FUNCTIONS

And just a reminder of some of its 7 properties. . .

1. e0 = 1

2. ex+y = exey

3. e−x = 1ex

4.(ex)y

= exy

5. limx→∞

ex =∞, limx→−∞

ex = 0

Now consider again the problem of differentiating ax. We saw above that

d

dxax = C(a) · ax and C(a) = C(10) · log10 a

which givesd

dxax = C(10) · log10 a · ax

We can eliminate the C(10) term with a little care. Since we know that ddxex = ex, we

have C(e) = 1. This allows us to express

1 = C(e) = C(10) · log10 e and so

C(10) =1

log10 e

Putting things back together gives

d

dxax =

log10 a

log10 e· ax

= loge a · ax.

7 The function ex is of course the special case of the function ax with a = e. So it inherits all theusual algebraic properties of ax.

179

Page 196: CLP-1 Differential Calculus

DERIVATIVES 2.7 DERIVATIVES OF EXPONENTIAL FUNCTIONS

There is more than one way to get to this result. For example, let f(x) = ax, then

loge f(x) = x loge a

f(x) = ex loge a

So if we write g(x) = ex then we are really attempting to differentiate the function

df

dx=

d

dxg(x · loge a).

In order to compute this derivative we need to know how to differentiate

d

dxg(qx)

where q is a constant. We’ll hold off on learning this for the moment until we haveintroduced the chain rule (see Section 2.9 and in particular Corollary 2.9.9). Similarlywe’d like to know how to differentiate logarithms — again this has to wait until wehave learned the chain rule.

Notice that the derivatives

d

dxxn = nxn−1 and

d

dxex = ex

are either nearly unchanged or actually unchanged by differentiating. It turns outthat some of the trigonometric functions also have this property of being “nearlyunchanged” by differentiation. That brings us to the next section.

2.7.3 tt Exercises

Exercises — Stage 1

1. Match the curves in the graph to the following functions:

(a) y =

(1

2

)x(b) y = 1x (c) y = 2x

(d) y = 2−x (e) y = 3x

180

Page 197: CLP-1 Differential Calculus

DERIVATIVES 2.7 DERIVATIVES OF EXPONENTIAL FUNCTIONS

x

y

D

CA B

2. The graph below shows an exponential function f(x) = ax and itsderivative f ′(x). Choose all the options that describe the constant a.

(a) a < 0 (b) a > 0 (c) a < 1

(d) a > 1 (e) a < e (f) a > e

x

y y = f(x)

y = f ′(x)

3. True or false:d

dxex = xex−1

4. A population of bacteria is described by P (t) = 100e0.2t, for 0 ≤ t ≤ 10.Over this time period, is the population increasing or decreasing?We will learn more about the uses of exponential functions to describe real-world phenomena in Section 3.3.

Exercises — Stage 2

5. Find the derivative of f(x) =ex

2x.

6. Differentiate f(x) = e2x.

181

Page 198: CLP-1 Differential Calculus

DERIVATIVES 2.8 DERIVATIVES OF TRIGONOMETRIC FUNCTIONS

7. Differentiate f(x) = ea+x, where a is a constant.

8. For which values of x is the function f(x) = xex increasing?

9. Differentiate e−x.

10. Differentiate f(x) = (ex + 1)(ex − 1).

11. A particle’s position is given by

s(t) = t2et.

When is the particle moving in the negative direction?

Exercises — Stage 3

12. Let g(x) = f(x)ex, for a differentiable function f(x). Give a simplifiedformula for g′(x).Functions of the form g(x) are relatively common. If you remember thisformula, you can save yourself some time when you need to differenti-ate them. We will explore this more in Question 2.14.2.19, Section 2.14.

13. Which of the following functions describe a straight line?

(a) y = e3 log x + 1 (b) 2y + 5 = e3+log x (c) y = e2x + 4

(d) y = elog x3e + log 2

14. *. Find constants a, b so that the following function is differentiable:

f(x) =

ax2 + b x ≤ 1

ex x > 1

2.8q Derivatives of Trigonometric Functions

We are now going to compute the derivatives of the various trigonometric functions,sinx, cosx and so on. The computations are more involved than the others that wehave done so far and will take several steps. Fortunately, the final answers will bevery simple.

Observe that we only need to work out the derivatives of sinx and cosx, since theother trigonometric functions are really just quotients of these two functions. Recall:

tanx =sinx

cosxcotx =

cosx

sinxcscx =

1

sinxsecx =

1

cosx.

182

Page 199: CLP-1 Differential Calculus

DERIVATIVES 2.8 DERIVATIVES OF TRIGONOMETRIC FUNCTIONS

The first steps towards computing the derivatives of sinx, cosx is to find theirderivatives at x = 0. The derivatives at general points x will follow quickly fromthese, using trig identities. It is important to note that we must measure angles inradians 1 , rather than degrees, in what follows. Indeed — unless explicitly statedotherwise, any number that is put into a trigonometric function is measured in radi-ans.

2.8.1 tt These Proofs are Optional, the Results are Not.

While we expect you to read and follow these proofs, we do not expect you to beable to reproduce them. You will be required to know the results, in particular The-orem 2.8.5 below.

2.8.2 tt Step 1: ddxsinx

∣∣x=0

By definition, the derivative of sinx evaluated at x = 0 is

d

dxsinx

∣∣∣x=0

= limh→0

sinh− sin 0

h= lim

h→0

sinh

h

We will prove this limit by use of the squeeze theorem (Theorem 1.4.18). To get therewe will first need to do some geometry. But first we will build some intuition.

The figure below contains part of a circle of radius 1. Recall that an arc of lengthh on such a circle subtends an angle of h radians at the centre of the circle. So thedarkened arc in the figure has length h and the darkened vertical line in the figurehas length sinh. We must determine what happens to the ratio of the lengths of thedarkened vertical line and darkened arc as h tends to zero.

1 In science, radians is the standard unit for measuring angles. While you may be more familiarwith degrees, radians should be used in any computation involving calculus. Using degrees willcause errors. Thankfully it is easy to translate between these two measures since 360 = 2πradians. See Appendix B.2.1.

183

Page 200: CLP-1 Differential Calculus

DERIVATIVES 2.8 DERIVATIVES OF TRIGONOMETRIC FUNCTIONS

Here is a magnified version of the part of the above figure that contains the dark-ened arc and vertical line.

This particular figure has been drawn with h = .4 radians. Here are three moresuch blow ups. In each successive figure, the value of h is smaller. To make thefigures clearer, the degree of magnification was increased each time hwas decreased.

As we make h smaller and smaller and look at the figure with ever increasingmagnification, the arc of length h and vertical line of length sinh look more andmore alike. We would guess from this that

limh→0

sinh

h= 1

The following tables of values

184

Page 201: CLP-1 Differential Calculus

DERIVATIVES 2.8 DERIVATIVES OF TRIGONOMETRIC FUNCTIONS

h sinh sinhh

0.4 .3894 .97350.2 .1987 .99340.1 .09983 .99830.05 .049979 .999580.01 .00999983 .9999830.001 .0099999983 .9999983

h sinh sinhh

−0.4 −.3894 .9735−0.2 −.1987 .9934−0.1 −.09983 .9983−0.05 −.049979 .99958−0.01 −.00999983 .999983−0.001 −.0099999983 .9999983

suggests the same guess. Here is an argument that shows that the guess really iscorrect.

2.8.3 tt Proof that limh→0

sinhh = 1

The circle in the figure above has radius 1. Hence

|OP | = |OR| = 1 |PS| = sinh

|OS| = cosh |QR| = tanh

Now we can use a few geometric facts about this figure to establish both an upperbound and a lower bound on sinh

hwith both the upper and lower bounds tending to

185

Page 202: CLP-1 Differential Calculus

DERIVATIVES 2.8 DERIVATIVES OF TRIGONOMETRIC FUNCTIONS

1 as h tends to 0. So the squeeze theorem will tell us that sinhh

also tends to 1 as htends to 0.

• The triangle OPR has base 1 and height sinh, and hence

area of4OPR =1

2× 1× sinh =

sinh

2.

• The triangle OQR has base 1 and height tanh, and hence

area of4OQR =1

2× 1× tanh =

tanh

2.

• The “piece of pie”OPR cut out of the circle is the fraction h2π

of the whole circle(since the angle at the corner of the piece of pie is h radians and the angle forthe whole circle is 2π radians). Since the circle has radius 1 we have

area of pie OPR =h

2π· (area of circle) =

h

2ππ · 12 =

h

2

Now the triangle OPR is contained inside the piece of pie OPR. and so the area ofthe triangle is smaller than the area of the piece of pie. Similarly, the piece of pieOPR is contained inside the triangle OQR. Thus we have

area of triangle OPR ≤ area of pie OPR ≤ area of triangle OQR

Substituting in the areas we worked out gives

sinh

2≤ h

2≤ tanh

2

which cleans up to give

sinh ≤ h ≤ sinh

cosh

We rewrite these two inequalities so that sinhh

appears in both.

• Since sinh ≤ h, we have thatsinh

h≤ 1.

• Since h ≤ sinh

coshwe have that cosh ≤ sinh

h.

Thus we arrive at the “squeezable” inequality

cosh ≤ sinh

h≤ 1

We know 2 that

limh→0

cosh = 1.

Since sinhh

is sandwiched between cosh and 1, we can apply the squeeze theorem forlimits (Theorem 1.4.18) to deduce the following lemma:

2 Again, refresh your memory by looking up Appendix A.5.

186

Page 203: CLP-1 Differential Calculus

DERIVATIVES 2.8 DERIVATIVES OF TRIGONOMETRIC FUNCTIONS

Lemma 2.8.1

limh→0

sinh

h= 1.

Since this argument took a bit of work, perhaps we should remind ourselves whywe needed it in the first place. We were computing

d

dxsinx

∣∣∣x=0

= limh→0

sinh− sin 0

h

= limh→0

sinh

h(This is why!)

= 1

This concludes Step 1. We now know that ddx

sinx∣∣x=0

= 1. The remaining stepsare easier.

2.8.4 tt Step 2: ddxcosx

∣∣x=0

By definition, the derivative of cosx evaluated at x = 0 is

limh→0

cosh− cos 0

h= lim

h→0

cosh− 1

h

Fortunately we don’t have to wade through geometry like we did for the previousstep. Instead we can recycle our work and massage the above limit to rewrite it interms of expressions involving sinh

h. Thanks to Lemma 2.8.1 the work is then easy.

We’ll show you two ways to proceed — one uses a method similar to “multiply-ing by the conjugate” that we have already used a few times (see Example 1.4.17and 2.2.9 ), while the other uses a nice trick involving the double–angle formula 3 .

2.8.4.1tttMethod 1 — Multiply by the “Conjugate”

Start by multiplying the expression inside the limit by 1, written ascosh+ 1

cosh+ 1:

cosh− 1

h=

cosh− 1

h· cosh+ 1

cosh+ 1

=cos2 h− 1

h(1 + cosh)

(since (a− b)(a+ b) = a2 − b2

)3 See Appendix A.14 if you have forgotten. You should also recall that sin2 θ + cos2 θ = 1. Sorry

for nagging.

187

Page 204: CLP-1 Differential Calculus

DERIVATIVES 2.8 DERIVATIVES OF TRIGONOMETRIC FUNCTIONS

= − sin2 h

h(1 + cosh)(since sin2 h+ cos2 h = 1)

= −sinh

h· sinh

1 + cosh

Now we can take the limit as h→ 0 via Lemma 2.8.1.

limh→0

cosh− 1

h= lim

h→0

(− sinh

h· sinh

1 + cosh

)= − lim

h→0

(sinh

h

)· limh→0

(sinh

1 + cosh

)= −1 · 0

2= 0

2.8.4.2tttMethod 2 — via the Double Angle Formula

The other way involves the double angle formula 4,

cos 2θ = 1− 2 sin2(θ) or cos 2θ − 1 = −2 sin2(θ)

Setting θ = h/2, we have

cosh− 1

h=−2(

sin h2

)2

h

Now this begins to look like sinh?h

, except that inside the sin(·) we have h/2. So,setting θ = h/2,

cosh− 1

h= −sin2 θ

θ= −θ · sin2 θ

θ2

= −θ · sin θ

θ· sin θ

θ

When we take the limit as h→ 0, we are also taking the limit as θ = h/2→ 0, and so

limh→0

cosh− 1

h= lim

θ→0

[−θ · sin θ

θ· sin θ

θ

]= lim

θ→0[−θ] · lim

θ→0

[sin θ

θ

]· limθ→0

[sin θ

θ

]= 0 · 1 · 1= 0

where we have used the fact that limh→0

sinh

h= 1 and that the limit of a product is the

product of limits (i.e. Lemma 2.8.1 and Theorem 1.4.3).

Thus we have now produced two proofs of the following lemma:

4 We hope you looked this up in in Appendix A.14. Nag.

188

Page 205: CLP-1 Differential Calculus

DERIVATIVES 2.8 DERIVATIVES OF TRIGONOMETRIC FUNCTIONS

Lemma 2.8.2

limh→0

cosh− 1

h= 0

Again, there has been a bit of work to get to here, so we should remind ourselveswhy we needed it. We were computing

d

dxcosx

∣∣∣x=0

= limh→0

cosh− cos 0

h

= limh→0

cosh− 1

h= 0

Armed with these results we can now build up the derivatives of sine and co-sine.

2.8.5 tt Step 3: ddxsinx and d

dxcosx for General x

To proceed to the general derivatives of sinx and cosx we are going to use the abovetwo results and a couple of trig identities. Remember the addition formulae 5

sin(a+ b) = sin(a) cos(b) + cos(a) sin(b)

cos(a+ b) = cos(a) cos(b)− sin(a) sin(b)

To compute the derivative of sin(x) we just start from the definition of the derivative:

d

dxsinx = lim

h→0

sin(x+ h)− sinx

h

= limh→0

sinx cosh+ cosx sinh− sinx

h

= limh→0

[sinx

cosh− 1

h+ cosx

sinh− 0

h

]= sinx lim

h→0

cosh− 1

h+ cosx lim

h→0

sinh− 0

h

= sinx

[d

dxcosx

]x=0︸ ︷︷ ︸

=0

+ cosx

[d

dxsinx

]x=0︸ ︷︷ ︸

=1

= cosx

5 You really should. Look this up in Appendix A.8 if you have forgotten.

189

Page 206: CLP-1 Differential Calculus

DERIVATIVES 2.8 DERIVATIVES OF TRIGONOMETRIC FUNCTIONS

The computation of the derivative of cosx is very similar.d

dxcosx = lim

h→0

cos(x+ h)− cosx

h

= limh→0

cosx cosh− sinx sinh− cosx

h

= limh→0

[cosx

cosh− 1

h− sinx

sinh− 0

h

]= cosx lim

h→0

cosh− 1

h− sinx lim

h→0

sinh− 0

h

= cosx

[d

dxcosx

]x=0︸ ︷︷ ︸

=0

− sinx

[d

dxsinx

]x=0︸ ︷︷ ︸

=1

= − sinx

We have now found the derivatives of both sinx and cosx, provided x is measured inradians.

Lemma 2.8.3

d

dxsinx = cosx

d

dxcosx = − sinx

The above formulas hold provided x is measured in radians.

These formulae are pretty easy to remember — applying ddx

to sinx and cosx justexchanges sinx and cosx, except for the minus sign 6 in the derivative of cosx.

Remark 2.8.4 Optional — Another derivation of ddx

cosx = − sinx. We remarkthat, once one knows that d

dxsinx = cosx, it is easy to use it and the trig identity

cos(x) = sin(π2− x)

to derive ddx

cosx = − sinx. Here is howa.

d

dxcosx = lim

h→0

cos(x+ h)− cosx

h= lim

h→0

sin(π2− x− h)− sin

(π2− x)

h

= − limh′→0

sin(x′ + h′)− sin(x′)

h′with x′ = π

2− x, h′ = −h

= − d

dx′sinx′

∣∣∣x′=

π2−x

= − cos(π2− x)

= − sinx

a We thank Serban Raianu for suggesting that we include this.

6 There is a bad pun somewhere in here about sine errors and sign errors.

190

Page 207: CLP-1 Differential Calculus

DERIVATIVES 2.8 DERIVATIVES OF TRIGONOMETRIC FUNCTIONS

Note that if x is measured in degrees, then the formulas of Lemma 2.8.3 arewrong. There are similar formulas, but we need the chain rule to build them —that is the subject of the next section. But first we should find the derivatives of theother trig functions.

2.8.6 tt Step 4: the Remaining Trigonometric Functions

It is now an easy matter to get the derivatives of the remaining trigonometric func-tions using basic trig identities and the quotient rule. Remember 7 that

tanx =sinx

cosxcotx =

cosx

sinx=

1

tanx

cscx =1

sinxsecx =

1

cosx

So, by the quotient rule,

d

dxtanx =

d

dx

sinx

cosx=

cosx︷ ︸︸ ︷(d

dxsinx

)cosx− sinx

− sinx︷ ︸︸ ︷(d

dxcosx

)cos2 x

= sec2 x

d

dxcscx =

d

dx

1

sinx= −

cosx︷ ︸︸ ︷(d

dxsinx

)sin2 x

= − cscx cotx

d

dxsecx =

d

dx

1

cosx= −

− sinx︷ ︸︸ ︷(d

dxcosx

)cos2 x

= secx tanx

d

dxcotx =

d

dx

cosx

sinx=

− sinx︷ ︸︸ ︷(d

dxcosx

)sinx− cosx

cosx︷ ︸︸ ︷(d

dxsinx

)sin2 x

= − csc2 x

2.8.7 tt Summary

To summarise all this work, we can write this up as a theorem:

Theorem 2.8.5 Derivatives of trigonometric functions.

The derivatives of sinx and cosx are

d

dxsinx = cosx

d

dxcosx = − sinx

7 You really should. If you do not then take a quick look at the appropriate appendix.

191

Page 208: CLP-1 Differential Calculus

DERIVATIVES 2.8 DERIVATIVES OF TRIGONOMETRIC FUNCTIONS

Consequently the derivatives of the other trigonometric functions are

d

dxtanx = sec2 x

d

dxcotx = − csc2 x

d

dxcscx = − cscx cotx

d

dxsecx = secx tanx

Of these 6 derivatives you should really memorise those of sine, cosine and tan-gent. We certainly expect you to be able to work out those of cotangent, cosecantand secant.

2.8.8 tt Exercises

Exercises — Stage 1

1. Graph sine and cosine on the same axes, from x = −2π to x = 2π. Markthe points where sinx has a horizontal tangent. What do these pointscorrespond to, on the graph of cosine?

2. Graph sine and cosine on the same axes, from x = −2π to x = 2π. Markthe points where sinx has a tangent line of maximum (positive) slope.What do these points correspond to, on the graph of cosine?

Exercises — Stage 2

3. Differentiate f(x) = sinx+ cosx+ tanx.

4. For which values of x does the function f(x) = sinx + cosx have ahorizontal tangent?

5. Differentiate f(x) = sin2 x+ cos2 x.

6. Differentiate f(x) = 2 sinx cosx.

7. Differentiate f(x) = ex cotx.

8. Differentiate f(x) =2 sinx+ 3 tanx

cosx+ tanx

192

Page 209: CLP-1 Differential Calculus

DERIVATIVES 2.8 DERIVATIVES OF TRIGONOMETRIC FUNCTIONS

9. Differentiate f(x) =5 secx+ 1

ex.

10. Differentiate f(x) = (ex + cotx)(5x6 − cscx).

11. Differentiate f(θ) = sin(π2− θ).

12. Differentiate f(x) = sin(−x) + cos(−x).

13. Differentiate s(θ) =cos θ + sin θ

cos θ − sin θ.

14. *. Find the values of the constants a and b for which

f(x) =

cos(x) x ≤ 0

ax+ b x > 0

is differentiable everywhere.

15. *. Find the equation of the line tangent to the graph of y = cos(x) + 2x

at x =π

2.

Exercises — Stage 3

16. *. Evaluate limx→π/3

(cos(x)− 1/2

x− π/3

). Use any method.

17. Show how you can use the quotient rule to find the derivative of tangent, ifyou already know the derivatives of sine and cosine.

18. *. The derivative of the function

f(x) =

ax+ b for x < 0

6 cosx2+sinx+cosx

for x ≥ 0

exists for all x. Determine the values of the constants a and b.19. *. For which values of x does the derivative of f(x) = tan x exist?

20. *. For what values of x does the derivative of10 sin(x)

x2 + x− 6exist? Explain

your answer.

21. *. For what values of x does the derivative ofx2 + 6x+ 5

sin(x)exist? Explain

your answer.

22. *. Find the equation of the line tangent to the graph of y = tan(x) at x =π

4.

23. *. Find the equation of the line tangent to the graph of y = sin(x)+cos(x)+ex

at x = 0.

193

Page 210: CLP-1 Differential Calculus

DERIVATIVES 2.9 ONE MORE TOOL – THE CHAIN RULE

24. For which values of x does the function f(x) = ex sinx have a horizontaltangent line?

25. Let

f(x) =

sinxx

, x 6= 0

1 , x = 0

Find f ′(0), or show that it does not exist.

26. *. Differentiate the function

h(x) = sin(|x|)

and give the domain where the derivative exists.

27. *. For the function

f(x) =

0 x ≤ 0sin(x)√

xx > 0

which of the following statements is correct?

i f is undefined at x = 0.

ii f is neither continuous nor differentiable at x = 0.

iii f is continuous but not differentiable at x = 0.

iv f is differentiable but not continuous at x = 0.

v f is both continuous and differentiable at x = 0.

28. *. Evaluate limx→0

sinx27 + 2x5ex99

sin5 x.

2.9q One More Tool – the Chain Rule

We have built up most of the tools that we need to express derivatives of complicatedfunctions in terms of derivatives of simpler known functions. We started by learninghow to evaluate

• derivatives of sums, products and quotients

• derivatives of constants and monomials

These tools allow us to compute derivatives of polynomials and rational functions.In the previous sections, we added exponential and trigonometric functions to our

194

Page 211: CLP-1 Differential Calculus

DERIVATIVES 2.9 ONE MORE TOOL – THE CHAIN RULE

list. The final tool we add is called the chain rule. It tells us how to take the deriva-tive of a composition of two functions. That is if we know f(x) and g(x) and theirderivatives, then the chain rule tells us the derivative of f

(g(x)

).

Before we get to the statement of the rule, let us look at an example showing howsuch a composition might arise (in the “real-world”).

Example 2.9.1 Walking towards a campfire.

You are out in the woods after a long day of mathematics and are walking towardsyour camp fire on a beautiful still night. The heat from the fire means that the airtemperature depends on your position. Let your position at time t be x(t). Thetemperature of the air at position x is f(x). What instantaneous rate of change oftemperature do you feel at time t?

• Because your position at time t is x = x(t), the temperature you feel at time t isF (t) = f

(x(t)

).

• The instantaneous rate of change of temperature that you feel is F ′(t). We havea complicated function, F (t), constructed by composing two simpler functions,x(t) and f(x).

• We wish to compute the derivative, F ′(t) = ddtf(x(t)), of the complicated func-

tion F (t) in terms of the derivatives, x′(t) and f ′(x), of the two simple functions.This is exactly what the chain rule does.

195

Page 212: CLP-1 Differential Calculus

DERIVATIVES 2.9 ONE MORE TOOL – THE CHAIN RULE

2.9.1 tt Statement of the Chain Rule

Theorem 2.9.2 The chain rule — version 1.

Let a ∈ R and let g(x) be a function that is differentiable at x = a. Nowlet f(u) be a function that is differentiable at u = g(a). Then the functionF (x) = f(g(x)) is differentiable at x = a and

F ′(a) = f ′(g(a)

)g′(a)

Here, as was the case earlier in this chapter, we have been very careful to give thepoint at which the derivative is evaluated a special name (i.e. a). But of course thisevaluation point can really be any point (where the derivative is defined). So it isvery common to just call the evaluation point “x” rather than give it a special namelike “a”, like this:

Theorem 2.9.3 The chain rule — version 2.

Let f and g be differentiable functions then

d

dxf(g(x)

)= f ′

(g(x)

)· g′(x)

Notice that when we form the composition f(g(x)

)there is an “outside” function

(namely f(x)) and an “inside” function (namely g(x)). The chain rule tells us thatwhen we differentiate a composition that we have to differentiate the outside andthen multiply by the derivative of the inside.

d

dxf(g(x)

)= f ′

(g(x)

)︸ ︷︷ ︸diff outside

· g′(x)︸︷︷︸diff inside

Here is another statement of the chain rule which makes this idea more explicit.

Theorem 2.9.4 The chain rule — version 3.

Let y = f(u) and u = g(x) be differentiable functions, then

dy

dx=

dy

du· du

dx

196

Page 213: CLP-1 Differential Calculus

DERIVATIVES 2.9 ONE MORE TOOL – THE CHAIN RULE

This particular form is easy to remember because it looks like we can just “cancel”the du between the two terms.

dy

dx=

dy

du·

du

dx

Of course, du is not, by itself, a number or variable 1 that can be cancelled. Butthis is still a good memory aid.

The hardest part about applying the chain rule is recognising when the functionyou are trying to differentiate is really the composition of two simpler functions.This takes a little practice. We can warm up with a couple of simple examples.

Example 2.9.5 Derivative of a power of sinx.

Let f(u) = u5 and g(x) = sin(x). Then set F (x) = f(g(x)

)=(

sin(x))5. To find the

derivative of F (x) we can simply apply the chain rule — the pieces of the composi-tion have been laid out for us. Here they are.

f(u) = u5 f ′(u) = 5u4

g(x) = sin(x) g′(x) = cos x

We now just put them together as the chain rule tells us

dF

dx= f ′

(g(x)

)· g′(x)

= 5(g(x)

)4 · cos(x) since f ′(u) = 5u4

= 5(

sin(x))4 · cos(x)

Notice that it is quite easy to extend this to any power. Set f(u) = un. Then followthe same steps and we arrive at

F (x) = (sin(x))n F ′(x) = n(

sin(x))n−1

cos(x)

This example shows one of the ways that the chain rule appears very frequently— when we need to differentiate the power of some simpler function. More gener-ally we have the following.

Example 2.9.6 Derivative of a power of a function.

Let f(u) = un and let g(x) be any differentiable function. Set F (x) = f(g(x)

)= g(x)n.

Then

dF

dx=

d

dx

(g(x)n

)= ng(x)n−1 · g′(x)

1 In this context du is called a differential. There are ways to understand and manipulate these incalculus but they are beyond the scope of this course.

197

Page 214: CLP-1 Differential Calculus

DERIVATIVES 2.9 ONE MORE TOOL – THE CHAIN RULE

Example 2.9.6

This is precisely the result in Example 2.6.6 and Lemma 2.6.7.

Example 2.9.7 Derivative of cos(3x− 2).

Let f(u) = cos(u) and g(x) = 3x− 2. Find the derivative of

F (x) = f(g(x)

)= cos(3x− 2).

Again we should approach this by first writing down f and g and their derivativesand then putting everything together as the chain rule tells us.

f(u) = cos(u) f ′(u) = − sin(u)

g(x) = 3x− 2 g′(x) = 3

So the chain rule says

F ′(x) = f ′(g(x)

)· g′(x)

= − sin(g(x)

)· 3

= −3 sin(3x− 2)

This example shows a second way that the chain rule appears very frequently —when we need to differentiate some function of ax + b. More generally we have thefollowing.

Example 2.9.8 Derivative of f(ax+ b).

Let a, b ∈ R and let f(x) be a differentiable function. Set g(x) = ax+ b. Then

d

dxf(ax+ b) =

d

dxf(g(x)

)= f ′

(g(x)

)· g′(x)

= f ′(ax+ b) · a

So the derivative of f(ax+ b) with respect to x is just af ′(ax+ b).

The above is a very useful result that follows from the chain rule, so let’s make ita corollary to highlight it.

198

Page 215: CLP-1 Differential Calculus

DERIVATIVES 2.9 ONE MORE TOOL – THE CHAIN RULE

Corollary 2.9.9

Let a, b ∈ R and let f(x) be a differentiable function, then

d

dxf(ax+ b) = af ′(ax+ b).

Example 2.9.10 2.9.1 continued.

Let us now go back to our motivating campfire example. There we had

f(x) = temperature at position xx(t) = position at time tF (t) = f(x(t)) = temperature at time t

The chain rule gave

F ′(t) = f ′(x(t)

)· x′(t)

Notice that the units of measurement on both sides of the equation agree — as in-deed they must. To see this, let us assume that t is measured in seconds, that x(t)is measured in metres and that f(x) is measured in degrees. Because of this F (x(t))must also be measured in degrees (since it is a temperature).What about the derivatives? These are rates of change. So

• F ′(t) has units degreessecond

,

• f ′(x) has units degreesmetre

, and

• x′(t) has units metresecond

.

Hence the product

f ′(x(t)

)· x′(t) has units =

degrees

metre· metre

second=

degrees

second.

has the same units as F ′(t). So the units on both sides of the equation agree. Check-ing that the units on both sides of an equation agree is a good check of consistency,but of course it does not prove that both sides are in fact the same.

199

Page 216: CLP-1 Differential Calculus

DERIVATIVES 2.9 ONE MORE TOOL – THE CHAIN RULE

2.9.2 tt (Optional) — Derivation of the Chain Rule

First, let’s review what our goal is. We have been given a function g(x), that is differ-entiable at some point x = a, and another function f(u), that is differentiable at thepoint u = b = g(a). We have defined the composite function F (x) = f

(g(x)

)and we

wish to show that

F ′(a) = f ′(g(a)

)· g′(a)

Before we can compute F ′(a), we need to set up some ground work, and in par-ticular the definitions of our given derivatives:

f ′(b) = limH→0

f(b+H)− f(b)

Hand g′(a) = lim

h→0

g(a+ h)− g(a)

h.

We are going to use similar manipulation tricks as we did back in the proofs of thearithmetic of derivatives in Section 2.5. Unfortunately, we have already used up thesymbols “F” and “H”, so we are going to make use the Greek letters γ, ϕ.

As was the case in our derivation of the product rule it is convenient to introducea couple of new functions. Set

ϕ(H) =f(b+H)− f(b)

H

Then we have

limH→0

ϕ(H) = f ′(b) = f ′(g(a)

)since b = g(a),

and we can also write (with a little juggling)

f(b+H) = f(b) +Hϕ(H)

Similarly set

γ(h) =g(a+ h)− g(a)

h

which gives us

limh→0

γ(h) = g′(a) and g(a+ h) = g(a) + hγ(h).

Now we can start computing

F ′(a) = limh→0

F (a+ h)− F (a)

h

= limh→0

f(g(a+ h)

)− f

(g(a)

)h

200

Page 217: CLP-1 Differential Calculus

DERIVATIVES 2.9 ONE MORE TOOL – THE CHAIN RULE

We know that g(a) = b and g(a+ h) = g(a) + hγ(h)), so

F ′(a) = limh→0

f(g(a) + hγ(h)

)− f

(g(a)

)h

= limh→0

f(b+ hγ(h))− f(b)

h

Now for the sneaky bit. We can turn f(b+ hγ(h)) into f(b+H) by setting

H = hγ(h)

Now notice that as h→ 0 we have

limh→0

H = limh→0

h · γ(h)

= limh→0

h · limh→0

γ(h)

= 0 · g′(a) = 0

So as h→ 0 we also have H → 0.We now have

F ′(a) = limh→0

f(b+H

)− f(b)

h

= limh→0

f(b+H

)− f(b)

H︸ ︷︷ ︸=ϕ(H)

· Hh︸︷︷︸

=γ(h)

if H = hγ(h) 6= 0

= limh→0

(ϕ(H) · γ(h)

)= lim

h→0ϕ(H) · lim

h→0γ(h) since H → 0 as h→ 0

= limH→0

ϕ(H) · limh→0

γ(h) = f ′(b) · g′(a)

This is exactly the RHS of the chain rule. It is possible to have H = 0 in the secondline above. But that possibility is easy to deal with:

• If g′(a) 6= 0, then, since limh→0 γ(h) = g′(a), H = hγ(h) cannot be 0 for smallnonzero h. Technically, there is an h0 > 0 such that H = hγ(h) 6= 0 for all 0 <|h| < h0. In taking the limit h → 0, above, we need only consider 0 < |h| < h0

and so, in this case, the above computation is completely correct.

• If g′(a) = 0, the above computation is still fine provided we exclude all h’s forwhich H = hγ(h) 6= 0. When g′(a) = 0, the right hand side, f ′

(g(a)

)· g′(a), of

the chain rule is 0. So the above computation gives

limh→0γ(h)6=0

f(b+H

)− f(b)

h= f ′

(g(a)

)· g′(a) = 0

On the other hand, when H = 0, we have f(b+H

)− f(b) = 0. So

limh→0γ(h)=0

f(b+H

)− f(b)

h= 0

too. That’s all we need.

201

Page 218: CLP-1 Differential Calculus

DERIVATIVES 2.9 ONE MORE TOOL – THE CHAIN RULE

2.9.3 tt Chain Rule Examples

We’ll now use the chain rule to compute some more derivatives.

Example 2.9.11 ddx

(1 + 3x

)75.

Find ddx

(1 + 3x

)75.This is a concrete version of Example 2.9.8. We are to find the derivative of a functionthat is built up by first computing 1+3x and then taking the 75th power of the result.So we set

f(u) = u75 f ′(u) = 75u74

g(x) = 1 + 3x g′(x) = 3

F (x) = f(g(x)

)= g(x)75 =

(1 + 3x

)75

By the chain rule

F ′(x) = f ′(g(x)

)g′(x) = 75 g(x)74 g′(x) = 75

(1 + 3x

)74 · 3= 225

(1 + 3x

)74

Example 2.9.12 ddx

sin(x2).

Find ddx

sin(x2).In this example we are to compute the derivative of sin with a (slightly) complicatedargument. So we apply the chain rule with f being sin and g(x) being the compli-cated argument. That is, we set

f(u) = sinu f ′(u) = cosu

g(x) = x2 g′(x) = 2x

F (x) = f(g(x)

)= sin

(g(x)

)= sin(x2)

By the chain rule

F ′(x) = f ′(g(x)

)g′(x) = cos

(g(x)

)g′(x) = cos(x2) · 2x

= 2x cos(x2)

202

Page 219: CLP-1 Differential Calculus

DERIVATIVES 2.9 ONE MORE TOOL – THE CHAIN RULE

Example 2.9.13 ddx

3√

sin(x2).

Find ddx

3√

sin(x2).In this example we are to compute the derivative of the cube root of a (moderately)complicated argument, namely sin(x2). So we apply the chain rule with f being“cube root” and g(x) being the complicated argument. That is, we set

f(u) = 3√u = u

13 f ′(u) = 1

3u−

23

g(x) = sin(x2) g′(x) = 2x cos(x2)

F (x) = f(g(x)

)= 3√g(x) = 3

√sin(x2)

In computing g′(x) here, we have already used the chain rule once (in Example2.9.12). By the chain rule

F ′(x) = f ′(g(x)

)y′(x) = 1

3g(x)−

23 · 2x cos(x2)

=2x

3

cos(x2)

[sin(x2)]23

Example 2.9.14 Derivative of a double-composition.

Find the derivative of ddxf(g(h(x))).

This is very similar to the previous example. Let us set F (x) = f(g(h(x))) withu = g(h(x)) then the chain rule tells us

dF

dx=

df

du· du

dx

= f ′(g(h(x))) · d

dxg(h(x))

We now just apply the chain rule again

= f ′(g(h(x))) · g′(h(x)) · h′(x).

Indeed it is not too hard to generalise further (in the manner of Example 2.6.6 tofind the derivative of the composition of 4 or more functions (though things start tobecome tedious to write down):

d

dxf1(f2(f3(f4(x)))) = f ′1(f2(f3(f4(x)))) · d

dxf2(f3(f4(x)))

= f ′1(f2(f3(f4(x)))) · f ′2(f3(f4(x))) · d

dxf3(f4(x))

= f ′1(f2(f3(f4(x)))) · f ′2(f3(f4(x))) · f ′3(f4(x)) · f ′4(x)

203

Page 220: CLP-1 Differential Calculus

DERIVATIVES 2.9 ONE MORE TOOL – THE CHAIN RULE

Example 2.9.15 Finding the quotient rule from the chain rule.

We can also use the chain rule to recover Corollary 2.4.6 and from there we can usethe product rule to recover the quotient rule.We want to differentiate F (x) = 1

g(x)so set f(u) = 1

uand u = g(x). Then the chain

rule tells us

d

dx

1

g(x)

=

dF

dx=

df

du· du

dx

=−1

u2· g′(x)

= − g′(x)

g(x)2.

Once we know this, a quick application of the product rule will give us the quotientrule.

d

dx

f(x)

g(x)

=

d

dx

f(x) · 1

g(x)

use PR

= f ′(x) · 1

g(x)+ f(x) · d

dx

1

g(x)

use the result from above

= f ′(x) · 1

g(x)− f(x) · g

′(x)

g(x)2place over a common denominator

=f ′(x) · g(x)− f(x) · g′(x)

g(x)2

which is exactly the quotient rule.

Example 2.9.16 A nice messy example.

Compute the following derivative:

d

dxcos

(x5√

3 + x6

(4 + x2)3

)This time we are to compute the derivative of cos with a really complicated argu-ment.

• So, to start, we apply the chain rule with g(x) = x5√

3+x6

(4+x2)3being the really compli-

cated argument and f being cos. That is, f(u) = cos(u). Since f ′(u) = − sin(u),the chain rule gives

d

dxcos

(x5√

3 + x6

(4 + x2)3

)= − sin

(x5√

3 + x6

(4 + x2)3

)d

dx

x5√

3 + x6

(4 + x2)3

204

Page 221: CLP-1 Differential Calculus

DERIVATIVES 2.9 ONE MORE TOOL – THE CHAIN RULE

• This reduced our problem to that of computing the derivative of the reallycomplicated argument x5

√3+x6

(4+x2)3. We can think of the argument as being built up

out of three pieces, namely x5, multiplied by√

3 + x6, divided by (4 + x2)3, or,

equivalently, multiplied by (4 + x2)−3. So we may rewrite x5

√3+x6

(4+x2)3as x5

(3 +

x6) 1

2 (4 + x2)−3, and then apply the product rule to reduce the problem to that

of computing the derivatives of the three pieces.

• Here goes (recall Example 2.6.6):

d

dx

[x5 (3 + x6)

12 (4 + x2)

−3]=

d

dx

[x5]· (3 + x6)

12 · (4 + x2)

−3

+ x5 · d

dx

[(3 + x6)

12]· (4 + x2)

−3

+ x5 · (3 + x6)12 · d

dx

[(4 + x2)

−3]This has reduced our problem to computing the derivatives of x5, which is easy,and of (3 + x6)

12 and (4 + x2)

−3, both of which can be done by the chain rule.Doing so,

d

dx

[x5 (3 + x6)

12 (4 + x2)

−3]=

5x4︷ ︸︸ ︷d

dx

[x5]·(3 + x6)

12 · (4 + x2)

−3

+ x5 ·

12

(3+x6)−12 ·6x5︷ ︸︸ ︷

d

dx

[(3 + x6)

12]·(4 + x2)

−3

+ x5 · (3 + x6)12 ·

−3(4+x2)−4·2x︷ ︸︸ ︷

d

dx

[(4 + x2)

−3]• Now we can clean things up in a sneaky way by observing

differentiating x5, to get 5x4, is the same as multiplying x5 by 5x, and

differentiating (3 + x6)12 to get 1

2(3 + x6)−

12 · 6x5 is the same as multiplying

(3 + x6)12 by 3x5

3+x6, and

differentiating (4 + x2)−3 to get−3(4 + x2)

−4 ·2x is the same as multiplying(4 + x2)

−3 by − 6x4+x2

.

Using these sneaky tricks we can write our solution quite neatly:

d

dxcos

(x5√

3 + x6

(4 + x2)3

)= − sin

(x5√

3 + x6

(4 + x2)3

)x5√

3 + x6

(4 + x2)3

5

x+

3x5

3 + x6− 6x

4 + x2

205

Page 222: CLP-1 Differential Calculus

DERIVATIVES 2.9 ONE MORE TOOL – THE CHAIN RULE

Example 2.9.16

• This method of cleaning up the derivative of a messy product is actually some-thing more systematic in disguise — namely logarithmic differentiation. Wewill come to this later.

2.9.4 tt Exercises

Exercises — Stage 1

1. Suppose the amount of kelp in a harbour depends on the number ofurchins. Urchins eat kelp: when there are more urchins, there is lesskelp, and when there are fewer urchins, there is more kelp. Supposefurther that the number of urchins in the harbour depends on the num-ber of otters, who find urchins extremely tasty: the more otters thereare, the fewer urchins there are.Let O, U , and K be the populations of otters, urchins, and kelp, respec-tively.

a Is dKdU

positive or negative?

b Is dUdO

positive or negative?

c Is dKdO

positive or negative?

Remark: An urchin barren is an area where unchecked sea urchin graz-ing has decimated the kelp population, which in turn causes the otherspecies that shelter in the kelp forests to leave. Introducing otters tourchin barrens is one intervention to increase biodiversity. A short videowith a more complex view of otters and urchins in Canadian waters isavailable on YouTube: https://youtu.be/ASJ82wyHisE

2. Suppose A,B,C,D and E are functions describing an interrelated system,with the following signs: dA

dB> 0, dB

dC> 0, dC

dD< 0, and dD

dE> 0. Is dA

dEpositive

or negative?

Exercises — Stage 2

3. Evaluate the derivative of f(x) = cos(5x+ 3).

4. Evaluate the derivative of f(x) = (x2 + 2)5.

5. Evaluate the derivative of T (k) = (4k4 + 2k2 + 1)17.

206

Page 223: CLP-1 Differential Calculus

DERIVATIVES 2.9 ONE MORE TOOL – THE CHAIN RULE

6. Evaluate the derivative of f(x) =

√x2 + 1

x2 − 1.

7. Evaluate the derivative of f(x) = ecos(x2).

8. *. Evaluate f ′(2) if f(x) = g(x/h(x)

), h(2) = 2, h′(2) = 3, g′(1) = 4.

9. *. Find the derivative of ex cos(x).

10. *. Evaluate f ′(x) if f(x) = ex2+cosx.

11. *. Evaluate f ′(x) if f(x) =

√x− 1

x+ 2.

12. *. Differentiate the function

f(x) =1

x2+√x2 − 1

and give the domain where the derivative exists.

13. *. Evaluate the derivative of f(x) =sin 5x

1 + x2

14. Evaluate the derivative of f(x) = sec(e2x+7).

15. Find the tangent line to the curve y = (tan2 x+ 1) (cos2 x) at the point x =π

4.

16. The position of a particle at time t is given by s(t) = et3−7t2+8t. For which

values of t is the velocity of the particle zero?

17. What is the slope of the tangent line to the curve y = tan(ex

2)

at the pointx = 1?

18. *. Differentiate y = e4x tanx. You do not need to simplify your answer.

19. *. Evaluate the derivative of the following function at x = 1: f(x) =x3

1 + e3x.

20. *. Differentiate esin2(x).21. *. Compute the derivative of y = sin (e5x)

22. *. Find the derivative of ecos(x2).

23. *. Compute the derivative of y = cos(x2 +

√x2 + 1

)24. *. Evaluate the derivative.

y = (1 + x2) cos2 x

207

Page 224: CLP-1 Differential Calculus

DERIVATIVES 2.9 ONE MORE TOOL – THE CHAIN RULE

25. *. Evaluate the derivative.

y =e3x

1 + x2

26. *. Find g′(2) if g(x) = x3h(x2), where h(4) = 2 and h′(4) = −2.

27. *. At what points (x, y) does the curve y = xe−(x2−1)/2 have a horizontaltangent?

28. A particle starts moving at time t = 1, and its position thereafter is givenby

s(t) = sin

(1

t

).

When is the particle moving in the negative direction?

29. Compute the derivative of f(x) =ex

cos3(5x− 7).

30. *. Evaluated

dx

xe2x cos 4x

.

Exercises — Stage 3

31. A particle moves along the Cartesian plane from time t = −π/2 to timet = π/2. The x-coordinate of the particle at time t is given by x = cos t, andthe y-coordinate is given by y = sin t, so the particle traces a curve in theplane. When does the tangent line to that curve have slope −1?

32. *. Show that, for all x > 0, ex+x2 > 1 + x.

33. We know that sin(2x) = 2 sin x cosx. What other trig identity can youderive from this, using differentiation?

34. Evaluate the derivative of f(x) = 3

√ecscx2

√x3 − 9 tanx

. You do not have to sim-

plify your answer.

35. Suppose a particle is moving in the Cartesian plane over time. For any realnumber t ≥ 0, the coordinate of the particle at time t is given by (sin t, cos2 t).

a Sketch a graph of the curve traced by the particle in the plane by plot-ting points, and describe how the particle moves along it over time.

b What is the slope of the curve traced by the particle at time t =10π

3?

208

Page 225: CLP-1 Differential Calculus

DERIVATIVES 2.10 THE NATURAL LOGARITHM

2.10q The Natural Logarithm

The chain rule opens the way to understanding derivatives of more complicatedfunction. Not only compositions of known functions as we have seen the examplesof the previous section, but also functions which are defined implicitly.

Consider the logarithm base e — loge(x) is the power that e must be raised to togive x. That is, loge(x) is defined by

eloge x = x

i.e. — it is the inverse of the exponential function with base e. Since this choice ofbase works so cleanly and easily with respect to differentiation, this base turns out tobe (arguably) the most natural choice for the base of the logarithm. And as we sawin our whirlwind review of logarithms in Section 2.7, it is easy to use logarithms ofone base to compute logarithms with another base:

logq x =loge x

loge q

So we are (relatively) free to choose a base which is convenient for our purposes.The logarithm with base e, is called the “natural logarithm”. The “naturalness”

of logarithms base e is exactly that this choice of base works very nicely in calculus(and so wider mathematics) in ways that other bases do not 1 . There are severaldifferent “standard” notations for the logarithm base e;

loge x = log x = lnx.

We recommend that you be able to recognise all of these.In this text we will write the natural logarithm as “log” with no base. The reason

for this choice is that base e is the standard choice of base for logarithms in mathe-matics 2

The natural logarithm inherits many properties of general logarithms 3 . So, forall x, y > 0 the following hold:

• elog x = x,

• for any real number X , log(eX)

= X ,

• for any a > 1, loga x = log xlog a

and log x = loga xloga e

• log 1 = 0, log e = 1

1 The interested reader should head to Wikipedia and look up the natural logarithm.2 In other disciplines other bases are natural; in computer science, since numbers are stored in

binary it makes sense to use the binary logarithm — i.e. base 2. While in some sciences andfinance, it makes sense to use the decimal logarithm — i.e. base 10.

3 Again take a quick look at the whirlwind review of logarithms in Section 2.7.

209

Page 226: CLP-1 Differential Calculus

DERIVATIVES 2.10 THE NATURAL LOGARITHM

• log(xy) = log x+ log y

• log(xy

)= log x− log y, log

(1y

)= − log y

• log(xX) = X log x

• limx→∞

log x =∞, limx→0

log x = −∞

And finally we should remember that log x has domain (i.e. is defined for) x > 0 andrange (i.e. takes all values in) −∞ < x <∞.

To compute the derivative of log x we could attempt to start with the limit defini-tion of the derivative

d

dxlog x = lim

h→0

log(x+ h)− log(x)

h

= limh→0

log((x+ h)/x)

h= um. . .

This doesn’t look good. But all is not lost — we have the chain rule, and we knowthat the logarithm satisfies the equation:

x = elog x

Since both sides of the equation are the same function, both sides of the equationhave the same derivative. i.e. we are using 4

if f(x) = g(x) for all x, then f ′(x) = g′(x)

4 Notice that just because the derivatives are the same, doesn’t mean the original functions are thesame. Both f(x) = x2 and g(x) = x2 + 3 have derivative f ′(x) = g′(x) = 2x, but f(x) 6= g(x).

210

Page 227: CLP-1 Differential Calculus

DERIVATIVES 2.10 THE NATURAL LOGARITHM

So now differentiate both sides:

d

dxx =

d

dxelog x

The left-hand side is easy, and the right-hand side we can process using the chainrule with f(u) = eu and u = log x.

1 =df

du· du

dx

= eu · d

dxlog x︸ ︷︷ ︸

what we want to compute

Recall that eu = elog x = x, so

1 = x · d

dxlog x︸ ︷︷ ︸

now what?

We can now just rearrange this equation to make the thing we want the subject:

d

dxlog x =

1

x

Thus we have proved:

Theorem 2.10.1

d

dxlog x =

1

x

where log x is the logarithm base e.

Example 2.10.2 The derivative of log 3x.

Let f(x) = log 3x. Find f ′(x).There are two ways to approach this — we can simplify then differentiate, or differ-entiate and then simplify. Neither is difficult.

• Simplify and then differentiate:

f(x) = log 3x log of a product= log 3 + log x

f ′(x) =d

dxlog 3 +

d

dxlog x

211

Page 228: CLP-1 Differential Calculus

DERIVATIVES 2.10 THE NATURAL LOGARITHM

Example 2.10.2

=1

x.

• Differentiation and then simplify:

f ′(x) =d

dxlog(3x) chain rule

=1

3x· 3

=1

x

Example 2.10.3 The derivative of log cx.

Notice that we can extend the previous example for any positive constant — not just3. Let c > 0 be a constant, then

d

dxlog cx =

d

dx(log c+ log x)

=1

x

Example 2.10.4 The derivative of log |x|.

We can push this further still. Let g(x) = log |x|, then a

• If x > 0, |x| = x and so

g′(x) =d

dxlog x =

1

x

• If x < 0 then |x| = −x. If |h| is strictly smaller than |x|, then we also have thatx + h < 0 and |x + h| = −(x + h) = |x| − h. Write X = |x| and H = −h. Then,by the definition of the derivative,

g′(x) = limh→0

log |x+ h| − log |x|h

= limh→0

log(|x| − h)− log |x|h

= limH→0

log(X +H)− logX

−H = − limH→0

log(X +H)− logX

H

= − d

dXlogX = − 1

X= − 1

|x|=

1

x

212

Page 229: CLP-1 Differential Calculus

DERIVATIVES 2.10 THE NATURAL LOGARITHM

Example 2.10.4

• Since log 0 is undefined, g′(0) does not exist.

Putting this together gives:

d

dxlog |x| = 1

x

a It’s probably a good moment to go back and look at Example 2.2.10.

Example 2.10.5 The derivative of xa.

Just after Corollary 2.6.17, we said that we would, in the future, find the derivativeof xa for all real numbers. The future is here. Let x > 0 and a be any real number.Exponentiating both sides of log

(xa)

= a log x gives us xa = ea log x and then

d

dxxa =

d

dxea log x = ea log x d

dx(a log x) by the chain rule

=a

xea log x =

a

xxa

= axa−1

as expected.

We can extend Theorem 2.10.1 to compute the derivative of logarithms of otherbases in a straightforward way. Since for any positive a 6= 1:

loga x =log x

log a=

1

log a· log x since a is a constant

d

dxloga x =

1

log a· 1

x

2.10.1 tt Back to ddxa

x

We can also now finally get around to computing the derivative of ax (which westarted to do back in Section 2.7).

f(x) = ax take log of both sideslog f(x) = x log a exponentiate both sides base e

f(x) = ex log a chain rule

f ′(x) = ex log a · log a

213

Page 230: CLP-1 Differential Calculus

DERIVATIVES 2.10 THE NATURAL LOGARITHM

= ax · log a

Notice that we could have also done the following:

f(x) = ax take log of both sideslog f(x) = x log a differentiate both sides

d

dx(log f(x)) = log a

We then process the left-hand side using the chain rule

f ′(x) · 1

f(x)= log a

f ′(x) = f(x) · log a = ax · log a

We will see ddx

log f(x) more below in the subsection on “logarithmic differentiation”.To summarise the results above:

Corollary 2.10.6

d

dxax = log a · ax for any a > 0

d

dxloga x =

1

x · log afor any a > 0, a 6= 1

where log x is the natural logarithm.

Recall that we need the caveat a 6= 1 because the logarithm base 1 is not welldefined. This is because 1x = 1 for any x. We do not need a similar caveat for thederivative of the exponential because we know (recall Example 2.7.1)

d

dx1x =

d

dx1 = 0 while the above corollary tells us

= log 1 · 1x = 0 · 1 = 0.

2.10.2 tt Logarithmic Differentiation

We want to go back to some previous slightly messy examples (Examples 2.6.6 and 2.6.18)and now show you how they can be done more easily.

214

Page 231: CLP-1 Differential Calculus

DERIVATIVES 2.10 THE NATURAL LOGARITHM

Example 2.10.7 Derivative of a triple product.

Consider again the derivative of the product of 3 functions:

P (x) = F (x) ·G(x) ·H(x)

Start by taking the logarithm of both sides:

logP (x) = log (F (x) ·G(x) ·H(x))

= logF (x) + logG(x) + logH(x)

Notice that the product of functions on the right-hand side has become a sum of func-tions. Differentiating sums is much easier than differentiating products. So when wedifferentiate we have

d

dxlogP (x) =

d

dxlogF (x) +

d

dxlogG(x) +

d

dxlogH(x)

A quick application of the chain rule shows that ddx

log f(x) = f ′(x)/f(x):

P ′(x)

P (x)=F ′(x)

F (x)+G′(x)

G(x)+H ′(x)

H(x)

Multiply through by P (x) = F (x)G(x)H(x):

P ′(x) =

(F ′(x)

F (x)+G′(x)

G(x)+H ′(x)

H(x)

)· F (x)G(x)H(x)

= F ′(x)G(x)H(x) + F (x)G′(x)H(x) + F (x)G(x)H ′(x)

which is what found in Example 2.6.6 by repeated application of the product rule.The above generalises quite easily to more than 3 functions.

This same trick of “take a logarithm and then differentiate” — or logarithmicdifferentiation — will work any time you have a product (or ratio) of functions.

Example 2.10.8 Derivative of a messy product.

Lets use logarithmic differentiation on the function from Example 2.6.18:

f(x) =(√x− 1)(2− x)(1− x2)√

x(3 + 2x)

Beware however, that we may only take the logarithm of positive numbers, and thisf(x) is often negative. For example, if 1 < x < 2, the factor (1 − x2) in the defini-tion of f(x) is negative while all of the other factors are positive, so that f(x) < 0.None–the–less, we can use logarithmic differentiation to find f ′(x), by exploiting

215

Page 232: CLP-1 Differential Calculus

DERIVATIVES 2.10 THE NATURAL LOGARITHM

Example 2.10.8

the observation that ddx

log |f(x)| = f ′(x)f(x)

. (To see this, use the chain rule and Exam-ple 2.10.4.) So we take the logarithm of |f(x)| and expand.

log |f(x)| = log|√x− 1| |2− x| |1− x2|√

x|3 + 2x|= log |√x−1|+ log |2−x|+ log |1−x2| − log(

√x)︸ ︷︷ ︸

= 12

log x

− log |3+2x|

Now we can essentially just differentiate term-by-term:

d

dxlog |f(x)| = d

dx

(log |√x− 1|+ log |2− x|+ log |1− x2|

− 1

2log |x| − log |3 + 2x|

)f ′(x)

f(x)=

1/(2√x)√

x− 1+−1

2− x +−2x

1− x2− 1

2x− 2

3 + 2x

f ′(x) = f(x) ·(

1

2√x(√x−1)

− 1

2−x −2x

1−x2− 1

2x− 2

3+2x

)=

(√x− 1)(2− x)(1− x2)√

x(3 + 2x)·(

1

2√x(√x− 1)

− 1

2− x −2x

1− x2− 1

2x− 2

3 + 2x

)just as we found previously.

2.10.3 tt Exercises

Reminder: in these notes, we use log x to mean loge x, which is also commonly writ-ten elsewhere as lnx.

Exercises — Stage 1

1. The volume in decibels (dB) of a sound is given by the formula:

V (P ) = 10 log10

(P

S

)where P is the intensity of the sound and S is the intensity of a standardbaseline sound. (That is: S is some constant.)How much noise will ten speakers make, if each speaker produces 3dB ofnoise? What about one hundred speakers?

216

Page 233: CLP-1 Differential Calculus

DERIVATIVES 2.10 THE NATURAL LOGARITHM

2. An investment of $1000 with an interest rate of 5% per year grows to

A(t) = 1000et/20

dollars after t years. When will the investment double?

3. Which of the following expressions, if any, is equivalent to log (cos2 x)?

(a) 2 log(cosx) (b) 2 log | cosx| (c) log2(cosx)

(d) log(cosx2))

Exercises — Stage 2

4. Differentiate f(x) = log(10x).

5. Differentiate f(x) = log(x2).

6. Differentiate f(x) = log(x2 + x).

7. Differentiate f(x) = log10 x.

8. *. Find the derivative of y =log x

x3.

9. Evaluated

dθlog(sec θ).

10. Differentiate the function f(x) = ecos(log x).

11. *. Evaluate the derivative. You do not need to simplify your answer.

y = log(x2 +√x4 + 1)

12. *. Differentiate√− log(cosx).

13. *. Calculate and simplify the derivative of log(x+√x2 + 4

).

14. *. Evaluate the derivative of g(x) = log(ex2

+√

1 + x4).

15. *. Evaluate the derivative of the following function at x = 1: g(x) =

log(2x− 1

2x+ 1

).

217

Page 234: CLP-1 Differential Calculus

DERIVATIVES 2.10 THE NATURAL LOGARITHM

16. Evaluate the derivative of the function f(x) = log

(√(x2 + 5)3

x4 + 10

).

17. Evaluate f ′(2) if f(x) = log(g(xh(x)

)), h(2) = 2, h′(2) = 3, g(4) = 3, g′(4) =

5.

18. *. Differentiate the function

g(x) = πx + xπ.

19. Differentiate f(x) = xx.

20. *. Find f ′(x) if f(x) = xx + log10 x.

21. Differentiate f(x) =4

√(x4 + 12)(x4 − x2 + 2)

x3.

22. Differentiate f(x) = (x+ 1)(x2 + 1)2(x3 + 1)3(x4 + 1)4(x5 + 1)5.

23. Differentiate f(x) =

(5x2 + 10x+ 15

3x4 + 4x3 + 5

)(1

10(x+ 1)

).

24. *. Let f(x) = (cos x)sinx, with domain 0 < x < π2. Find f ′(x).

25. *. Find the derivative of (tan(x))x, when x is in the interval (0, π/2).

26. *. Find f ′(x) if f(x) = (x2 + 1)(x2+1)

27. *. Differentiate f(x) = (x2 + 1)sin(x).

28. *. Let f(x) = xcos3(x), with domain (0,∞). Find f ′(x).

29. *. Differentiate f(x) = (3 + sin(x))x2−3.

Exercises — Stage 3

30. Let f(x) and g(x) be differentiable functions, with f(x) > 0. Evaluated

dx

[f(x)]g(x)

.

31. Let f(x) be a function whose range includes only positive numbers.Show that the curves y = f(x) and y = log(f(x)) have horizontal tan-gent lines at the same values of x.

218

Page 235: CLP-1 Differential Calculus

DERIVATIVES 2.11 IMPLICIT DIFFERENTIATION

2.11q Implicit Differentiation

2.11.1 tt Implicit Differentiation

Implicit differentiation is a simple trick that is used to compute derivatives of func-tions either

• when you don’t know an explicit formula for the function, but you know anequation that the function obeys or

• even when you have an explicit, but complicated, formula for the function, andthe function obeys a simple equation.

The trick is just to differentiate both sides of the equation and then solve for thederivative we are seeking. In fact we have already done this, without using thename “implicit differentiation”, when we found the derivative of log x in the pre-vious section. There we knew that the function f(x) = log x satisfied the equationef(x) = x for all x. That is, the functions ef(x) and x are in fact the same function andso have the same derivative. So we had

d

dxef(x) =

d

dxx = 1

We then used the chain rule to get ddxef(x) = ef(x)f ′(x), which told us that f ′(x) obeys

the equation

ef(x)f ′(x) = 1 and we can now solve for f ′(x)

f ′(x) = e−f(x) = e− log x =1

x.

The typical way to get used to implicit differentiation is to play with problemsinvolving tangent lines to curves. So here are a few examples finding the equationsof tangent lines to curves. Recall, from Theorem 2.3.4, that, in general, the tangentline to the curve y = f(x) at

(x0, y0

)is y = f(x0)+f ′(x0)(x−x0) = y0 +f ′(x0)(x−x0).

Example 2.11.1 Finding a tangent line using implicit differentiation.

Find the equation of the tangent line to y = y3 + xy + x3 at x = 1.This is a very standard sounding example, but made a little complicated by the factthat the curve is given by a cubic equation — which means we cannot solve directlyfor y in terms of x or vice versa. So we really do need implicit differentiation.

• First notice that when x = 1 the equation, y = y3+xy+x3, of the curve simplifiesto y = y3 + y + 1 or y3 = −1, which we can solve a : y = −1. So we know thatthe curve passes through (1,−1) when x = 1.

219

Page 236: CLP-1 Differential Calculus

DERIVATIVES 2.11 IMPLICIT DIFFERENTIATION

Example 2.11.1

• Now, to find the slope of the tangent line at (1,−1), pretend that our curve isy = f(x) so that f(x) obeys

f(x) = f(x)3 + xf(x) + x3

for all x. Differentiating both sides gives

f ′(x) = 3f(x)2f ′(x) + f(x) + xf ′(x) + 3x2

• At this point we could isolate for f ′(x) and write it in terms of f(x) and x,but since we only want answers when x = 1, let us substitute in x = 1 andf(1) = −1 (since the curve passes through (1,−1)) and clean things up beforedoing anything else.

• Subbing in x = 1, f(1) = −1 gives

f ′(1) = 3f ′(1)− 1 + f ′(1) + 3 and so f ′(1) = −2

3

• The equation of the tangent line is

y = y0 + f ′(x0)(x− x0) = −1− 2

3(x− 1) = −2

3x− 1

3

We can further clean up the equation of the line to write it as 2x+ 3y = −1.

a This type of luck rarely happens in the “real world”. But it happens remarkably frequently intextbooks, problem sets and tests.

In the previous example we replace y by f(x) in the middle of the computation.We don’t actually have to do this. When we are writing out our solution we canremember that y is a function of x. So we can start with

y = y3 + xy + x3

and differentiate remembering that y ≡ y(x)

y′ = 3y2y′ + xy′ + y + 3x2

And now substitute x = 1, y = −1 to get

y′(1) = 3 · y′(1) + y′(1)− 1 + 3 and so

y′(1) = −2

3

220

Page 237: CLP-1 Differential Calculus

DERIVATIVES 2.11 IMPLICIT DIFFERENTIATION

The next one is at the same time a bit easier (because it is a quadratic) and a bitharder (because we are asked for the tangent at a general point on the curve, not aspecific one).

Example 2.11.2 Another tangent line through implicit differentiation.

Let (x0, y0) be a point on the ellipse 3x2 + 5y2 = 7. Find the equation for the tangentlines when x = 1 and y is positive. Then find an equation for the tangent line to theellipse at a general point (x0, y0).Since we are not given an specific point x0 we are going to have to be careful withthe second half of this question.

• When x = 1 the equation simplifies to

3 + 5y2 = 7

5y2 = 4

y = ± 2√5.

We are only interested in positive y, so our point on the curve is (1, 2/√

5).

• Now we use implicit differentiation to find dydx

at this point. First we pretendthat we have solved the curve explicitly, for some interval of x’s, as y = f(x).The equation becomes

3x2 + 5f(x)2 = 7 now differentiate6x+ 10f(x)f ′(x) = 0

f ′(x) = − 3x

5f(x)

• When x = 1, y = 2/√

5 this becomes

f ′(1) = − 3

5 · 2/√

5= − 3

2√

5

So the tangent line passes through (1, 2/√

5) and has slope − 32√

5. Hence the

tangent line has equation

y = y0 + f ′(x0)(x− x0)

=2√5− 3

2√

5(x− 1)

=7− 3x

2√

5or equivalently

3x+ 2√

5y = 7

Now we should go back and do the same but for a general point on the curve (x0, y0):

221

Page 238: CLP-1 Differential Calculus

DERIVATIVES 2.11 IMPLICIT DIFFERENTIATION

• A good first step here is to sketch the curve. Since this is an ellipse, it is prettystraight-forward.

3x2 + 5y2 = 7

(x0, y0)

• Notice that there are two points on the ellipse — the extreme right and left

points (x0, y0) = ±(√

73, 0)

— at which the tangent line is vertical. In those twocases, the tangent line is just x = x0.

• Since this is a quadratic for y, we could solve it explicitly to get

y = ±√

7− 3x2

5

and choose the positive or negative branch as appropriate. Then we coulddifferentiate to find the slope and put things together to get the tangent line.

But even in this relatively easy case, it is computationally cleaner, and henceless vulnerable to mechanical errors, to use implicit differentiation. So that’swhat we’ll do.

• Now we could again “pretend” that we have solved the equation for the ellipsefor y = f(x) near (x0, y0), but let’s not do that. Instead (as we did just beforethis example) just remember that when we differentiate y is really a function ofx. So starting from

3x2 + 5y2 = 7 differentiating gives6x+ 5 · 2y · y′ = 0

We can then solve this for y′:

y′ = −3x

5y

where y′ and y are both functions of x.

• Hence at the point (x0, y0) we have

y′|(x0,y0) = −3x0

5y0

222

Page 239: CLP-1 Differential Calculus

DERIVATIVES 2.11 IMPLICIT DIFFERENTIATION

Example 2.11.2

This is the slope of the tangent line at (x0, y0) and so its equation is

y = y0 + y′ · (x− x0)

= y0 −3x0

5y0

(x− x0)

We can simplify this by multiplying through by 5y0 to get

5y0y = 5y20 − 3x0x+ 3x2

0

We can clean this up more by moving all the terms that contain x or y to theleft-hand side and everything else to the right:

3x0x+ 5y0y = 3x20 + 5y2

0

But there is one more thing we can do, our original equation is 3x2 + 5y2 = 7for all points on the curve, so we know that 3x2

0 + 5y20 = 7. This cleans up the

right-hand side.

3x0x+ 5y0y = 7

• In deriving this formula for the tangent line at (x0, y0) we have assumed thaty0 6= 0. But in fact the final answer happens to also work when y0 = 0 (which

means x0 = ±√

73), so that the tangent line is x = x0.

We can also check that our answer for general (x0, y0) reduces to our answer forx0 = 1.

• When x0 = 1 we worked out that y0 = 2/√

5.

• Plugging this into our answer above gives

3x0x+ 5y0y = 7 sub in (x0, y0) = (1, 2/√

5) :

3x+ 52√5y = 7 clean up a little

3x+ 2√

5y = 7

as required.

223

Page 240: CLP-1 Differential Calculus

DERIVATIVES 2.11 IMPLICIT DIFFERENTIATION

Example 2.11.3 A more involved example.

At which points does the curve x2 − xy + y2 = 3 cross the x–axis? Are the tangentlines to the curve at those points parallel?This is a 2 part question — first the x-intercepts and then we need to examine tangentlines.

• Finding where the curve crosses the x-axis is straight forward. It does so wheny = 0. This means x satisfies

x2 − x · 0 + 02 = 3 so x = ±√

3.

So the curve crosses the x–axis at two points(±√

3 , 0).

• Now we need to find the tangent lines at those points. But we don’t actuallyneed the lines, just their slopes. Again we can pretend that near one of thosepoints the curve is y = f(x). Applying d

dxto both sides of x2−xf(x)+f(x)2 = 3

gives

2x− f(x)− xf ′(x) + 2f(x)f ′(x) = 0

etc etc.

• But let us stop “pretending”. Just make sure we remember that y is a functionof x when we differentiate:

x2 − xy + y2 = 3 start with the curve, and differentiate2x− xy′ − y + 2yy′ = 0

Now substitute in the first point, x = +√

3, y = 0:

2√

3−√

3y′ + 0 = 0

y′ = 2

And now do the second point x = −√

3, y = 0:

−2√

3 +√

3y′ + 0 = 0

y′ = 2

Thus the slope is the same at x =√

3 and x = −√

3 and the tangent lines areparallel.

224

Page 241: CLP-1 Differential Calculus

DERIVATIVES 2.11 IMPLICIT DIFFERENTIATION

Example 2.11.3

Okay — let’s get away from curves and do something a little different.

Example 2.11.4 A rough game of baseball.

You are standing at the origin. At time zero a pitcher throws a ball at your head a.

Figure 2.11.5

The position of the (centre of the) ball at time t is x(t) = d−vt, where d is the distancefrom your head to the pitcher’s mound and v is the ball’s velocity. Your eye sees theball filling b an angle 2θ(t) with

sin(θ(t)

)=

r

d− vtwhere r is the radius of the baseball. The question is “How fast is θ growing at timet?” That is, what is dθ

dt?

225

Page 242: CLP-1 Differential Calculus

DERIVATIVES 2.11 IMPLICIT DIFFERENTIATION

Example 2.11.5

• We don’t know (yet) how to solve this equation to find θ(t) explicitly. So weuse implicit differentiation.

• To do so we apply ddt

to both sides of our equation. This gives

cos(θ(t)

)· θ′(t) =

rv

(d− vt)2

• Then we solve for θ′(t):

θ′(t) =rv

(d− vt)2 cos(θ(t)

)• As is often the case, when using implicit differentiation, this answer is not very

satisfying because it contains θ(t), for which we still do not have an explicitformula. However in this case we can get an explicit formula for cos

(θ(t)

),

without having an explicit formula for θ(t), just by looking at the right–angledtriangle in Figure 2.11.5, above.

• The hypotenuse of that triangle has length d− vt. By Pythagoras, the length ofthe side of the triangle adjacent of the angle θ(t) is

√(d− vt)2 − r2. So

cos(θ(t)

)=

√(d− vt)2 − r2

d− vtand

θ′(t) =rv

(d− vt)√

(d− vt)2 − r2

a It seems that it is not a friendly game today.b This is the “visual angle” or “angular size”.

Okay — just one more tangent-to-the-curve example and then we’ll go on tosomething different.

Example 2.11.6 The astroid (no that is not a typo).

Let (x0, y0) be a point on the astroid a b

x23 + y

23 = 1.

Find an equation for the tangent line to the astroid at (x0, y0).

226

Page 243: CLP-1 Differential Calculus

DERIVATIVES 2.11 IMPLICIT DIFFERENTIATION

• As was the case in examples above we can rewrite the equation of the astroidnear (x0, y0) in the form y = f(x), with an explicit f(x), by solving the equationx

23 + y

23 = 1. But again, it is computationally cleaner, and hence less vulnerable

to mechanical errors, to use implicit differentiation. So that’s what we’ll do.

• First up, since (x0, y0) lies on the curve, it satisfies

x230 + y

230 = 1.

• Now, no pretending that y = f(x), this time — just make sure we rememberwhen we differentiate that y changes with x.

x23 + y

23 = 1

Start with the curve, and then differentiate

2

3x−

13 +

2

3y−

13y′ = 0

• Note the derivative of x23 , namely 2

3x−

13 , and the derivative of y

23 , namely

23y−

13y′, are defined only when x 6= 0 and y 6= 0. We are interested in the

case that x = x0 and y = y0. So we better assume that x0 6= 0 and y0 6= 0.Probably something weird happens when x0 = 0 or y0 = 0. We’ll come back tothis shortly.

• To continue on, we set x = x0, y = y0 in the equation above, and then solve fory′:

2

3x− 1

30 +

2

3y− 1

30 y′(x) = 0 =⇒ y′(x0) = −

(y0

x0

) 13

This is the slope of the tangent line and its equation is

y = y0 + f ′(x0)(x− x0) = y0 −(y0

x0

) 13

(x− x0)

Now let’s think a little bit about what the tangent line slope of − 3

√y0x0

tells us aboutthe astroid.

• First, as a preliminary observation, note that since x230 ≥ 0 and y

230 ≥ 0 the

equation x230 + y

230 = 1 of the astroid forces 0 ≤ x

230 , y

230 ≤ 1 and hence −1 ≤

x0, y0 ≤ 1.

• For all x0, y0 > 0 the slope − 3

√y0x0

< 0. So at all points on the astroid thatare in the first quadrant, the tangent line has negative slope, i.e. is “leaningbackwards”.

227

Page 244: CLP-1 Differential Calculus

DERIVATIVES 2.11 IMPLICIT DIFFERENTIATION

Example 2.11.6

• As x0 tends to zero, y0 tends to ±1 and the tangent line slope tends to infinity.So at points on the astroid near (0,±1), the tangent line is almost vertical.

• As y0 tends to zero, x0 tends to ±1 and the tangent line slope tends to zero. Soat points on the astroid near (±1, 0), the tangent line is almost horizontal.

Here is a figure illustrating all this.

Sure enough, as we speculated earlier, something weird does happen to the astroidwhen x0 or y0 is zero. The astroid is pointy, and does not have a tangent there.

a Here is where is the astroid comes from. Imagine two circles, one of radius 1/4 and one of radius1. Paint a red dot on the smaller circle. Then imagine the smaller circle rolling around the insideof the larger circle. The curve traced by the red dot is our astroid. Google “astroid” (be carefulabout the spelling) to find animations showing this.

b The astroid was first discussed by Johann Bernoulli in 1691–92. It also appears in the work ofLeibniz.

2.11.2 tt Exercises

Exercises — Stage 1

1. If we implicitly differentiate x2 + y2 = 1, we get the equation 2x+ 2yy′ = 0.In the step where we differentiate y2 to obtain 2yy′, which rule(s) below arewe using? (a) power rule, (b) chain rule, (c) quotient rule , (d) derivativesof exponential functions

228

Page 245: CLP-1 Differential Calculus

DERIVATIVES 2.11 IMPLICIT DIFFERENTIATION

2. Using the picture below, estimatedy

dxat the three points where the curve

crosses the y-axis.

x

y

1 2 3 4

1

2

3

4

−4 −3 −2 −1

−4

−3

−2

−1

Remark: for this curve, one value of x may correspond to multiple val-ues of y. So, we cannot express this curve as y = f(x) for any function x.This is one typical situation where we might use implicit differentiation.

3. Consider the unit circle, formed by all points (x, y) that satisfy x2 + y2 = 1.

x

y

a Is there a function f(x) so that y = f(x) completely describes the unitcircle? That is, so that the points (x, y) that make the equation y = f(x)true are exactly the same points that make the equation x2 + y2 = 1true?

b Is there a function f ′(x) so that y = f ′(x) completely describes theslope of the unit circle? That is, so that for every point (x, y) on the

229

Page 246: CLP-1 Differential Calculus

DERIVATIVES 2.11 IMPLICIT DIFFERENTIATION

unit circle, the slope of the tangent line to the circle at that point isgiven by f ′(x)?

c Use implicit differentiation to find an expression fordy

dx. Simplify until

the expression is a function in terms of x only (not y), or explain whythis is impossible.

Exercises — Stage 2

4. *. Finddy

dxif xy + ex + ey = 1.

5. *. If ey = xy2 + x, computedy

dx.

6. *. If x2 tan(πy/4) + 2x log(y) = 16, then find y′ at the points where y = 1.

7. *. If x3 + y4 = cos(x2 + y) compute dydx

.

8. *. If x2ey + 4x cos(y) = 5, then find y′ at the points where y = 0.

9. *. If x2 + y2 = sin(x+ y) compute dydx

.

10. *. If x2 cos(y) + 2xey = 8, then find y′ at the points where y = 0.

11. At what points on the ellipse x2 + 3y2 = 1 is the tangent line parallel tothe line y = x?

12. *. For the curve defined by the equation√xy = x2y − 2, find the slope of

the tangent line at the point (1, 4).

13. *. If x2y2 + x sin(y) = 4, finddy

dx.

Exercises — Stage 3

14. *. If x2 + (y + 1)ey = 5, then find y′ at the points where y = 0.

15. For what values of x do the circle x2 + y2 = 1 and the ellipse x2 + 3y2 = 1have parallel tangent lines?

230

Page 247: CLP-1 Differential Calculus

DERIVATIVES 2.12 INVERSE TRIGONOMETRIC FUNCTIONS

2.12q Inverse Trigonometric Functions

One very useful application of implicit differentiation is to find the derivatives ofinverse functions. We have already used this approach to find the derivative of theinverse of the exponential function — the logarithm.

We are now going to consider the problem of finding the derivatives of the in-verses of trigonometric functions. Now is a very good time to go back and rereadSection 0.6 on inverse functions — especially Definition 0.6.4. Most importantly,given a function f(x), its inverse function f−1(x) only exists, with domain D, whenf(x) passes the “horizontal line test”, which says that for each Y in D the horizontalline y = Y intersects the graph y = f(x) exactly once. (That is, f(x) is a one-to-onefunction.)

Let us start by playing with the sine function and determine how to restrict thedomain of sinx so that its inverse function exists.

Example 2.12.1 The inverse of sinx.

Let y = f(x) = sin(x). We would like to find the inverse function which takes y andreturns to us a unique x-value so that sin(x) = y.

• For each real number Y , the number of x-values that obey sin(x) = Y , is exactlythe number of times the horizontal straight line y = Y intersects the graph ofsin(x).

• When −1 ≤ Y ≤ 1, the horizontal line intersects the graph infinitely manytimes. This is illustrated in the figure above by the line y = 0.3.

• On the other hand, when Y < −1 or Y > 1, the line y = Y never intersects thegraph of sin(x). This is illustrated in the figure above by the line y = −1.2.

This is exactly the horizontal line test and it shows that the sine function is not one-

231

Page 248: CLP-1 Differential Calculus

DERIVATIVES 2.12 INVERSE TRIGONOMETRIC FUNCTIONS

Example 2.12.2

to-one.Now consider the function

y = sin(x) with domain − π

2≤ x ≤ π

2

This function has the same formula but the domain has been restricted so that, aswe’ll now show, the horizontal line test is satisfied.

As we saw above when |Y | > 1 no x obeys sin(x) = Y and, for each −1 ≤ Y ≤ 1, theline y = Y (illustrated in the figure above with y = 0.3) crosses the curve y = sin(x)infinitely many times, so that there are infinitely many x’s that obey f(x) = sin x = Y .However exactly one of those crossings (the dot in the figure) has −π

2≤ x ≤ π

2.

That is, for each −1 ≤ Y ≤ 1, there is exactly one x, call it X , that obeys both

sinX = Y and −π2≤ X ≤ π

2

That unique value, X , is typically denoted arcsin(Y ). That is

sin(arcsin(Y )) = Y and −π2≤ arcsin(Y ) ≤ π

2

Renaming Y → x, the inverse function arcsin(x) is defined for all −1 ≤ x ≤ 1 and isdetermined by the equation

Equation 2.12.2

sin(

arcsin(x))

= x and − π

2≤ arcsin(x) ≤ π

2.

Note that many texts will use sin−1(x) to denote arcsine, however we will usearcsin(x) since we feel that it is clearer a; the reader should recognise both.

a The main reason being that people frequently confuse sin−1(x) with (sin(x))−1 = 1sin x . We feel

that prepending the prefix “arc” less likely to lead to such confusion. The notations asin(x) andArcsin(x) are also used.

232

Page 249: CLP-1 Differential Calculus

DERIVATIVES 2.12 INVERSE TRIGONOMETRIC FUNCTIONS

Example 2.12.3 More on the inverse of sinx.

Sincesin

π

2= 1 sin

π

6=

1

2and −π

2≤ π

6, π

2≤ π

2, we have

arcsin 1 =π

2arcsin

1

2=π

6

Even thoughsin(2π) = 0

it is not true that arcsin 0 = 2π, and it is not true that arcsin(

sin(2π))

= 2π, because2π is not between −π

2and π

2. More generally

arcsin(

sin(x))

= the unique angle θ between − π

2and

π

2obeying sin θ = sinx

= x if and only if −π2≤ x ≤ π

2

So, for example, arcsin(

sin(

11π16

))cannot be 11π

16because 11π

16is bigger than π

2. So how

do we find the correct answer? Start by sketching the graph of sin(x).

It looks like the graph of sinx is symmetric about x = π2. The mathematical way to

say that “the graph of sinx is symmetric about x = π2” is “sin(π

2− θ) = sin(π

2+ θ)” for

all θ. That is indeed true a .Now 11π

16= π

2+ 3π

16so

sin(11π

16

)= sin

(π2

+3π

16

)= sin

(π2− 3π

16

)= sin

(5π

16

)and, since 5π

16is indeed between −π

2and π

2,

arcsin(

sin(11π

16

))=

16

(and not

11π

16

).

a Indeed both are equal to cos θ. You can see this by playing with the trig identities in Ap-pendix A.8.

233

Page 250: CLP-1 Differential Calculus

DERIVATIVES 2.12 INVERSE TRIGONOMETRIC FUNCTIONS

2.12.1 tt Derivatives of Inverse Trig Functions

Now that we have explored the arcsine function we are ready to find its derivative.Lets call

arcsin(x) = θ(x),

so that the derivative we are seeking is dθdx

. The above equation is (after taking sineof both sides) equivalent to

sin(θ) = x

Now differentiate this using implicit differentiation (we just have to remember thatθ varies with x and use the chain rule carefully):

cos(θ) · dθ

dx= 1

dx=

1

cos(θ)substitute θ = arcsinx

d

dxarcsinx =

1

cos(arcsinx)

This doesn’t look too bad, but it’s not really very satisfying because the right handside is expressed in terms of arcsin(x) and we do not have an explicit formula forarcsin(x).

However even without an explicit formula for arcsin(x), it is a simple matterto get an explicit formula for cos

(arcsin(x)

), which is all we need. Just draw a

right–angled triangle with one angle being arcsin(x). This is done in the figure below1.

Since sin(θ) = x (see 2.12.2), we have made the side opposite the angle θ of lengthx and the hypotenuse of length 1. Then, by Pythagoras, the side adjacent to θ haslength

√1− x2 and so

cos(

arcsin(x))

= cos(θ) =√

1− x2

which in turn gives us the answer we need:

d

dxarcsin(x) =

1√1− x2

1 The figure is drawn for the case that 0 ≤ arcsin(x) ≤ π2 . Virtually the same argument works for

the case −π2 ≤ arcsin(x) ≤ 0

234

Page 251: CLP-1 Differential Calculus

DERIVATIVES 2.12 INVERSE TRIGONOMETRIC FUNCTIONS

The definitions for arccos, arctan and arccot are developed in the same way. Hereare the graphs that are used.

x

y

π2

π

y = cot(x)

y = 0.8

The definitions for the remaining two inverse trigonometric functions may also

235

Page 252: CLP-1 Differential Calculus

DERIVATIVES 2.12 INVERSE TRIGONOMETRIC FUNCTIONS

be developed in the same way23. But it’s a little easier to use

cscx =1

sinxsecx =

1

cosx

Definition 2.12.4

arcsinx is defined for |x| ≤ 1. It is the unique number obeying

sin(

arcsin(x))

= x and −π2≤ arcsin(x) ≤ π

2

arccosx is defined for |x| ≤ 1. It is the unique number obeying

cos(

arccos(x))

= x and 0 ≤ arccos(x) ≤ π

arctanx is defined for all x ∈ R. It is the unique number obeying

tan(

arctan(x))

= x and −π2< arctan(x) <

π

2

arccscx = arcsin 1x

is defined for |x| ≥ 1. It is the unique number obeying

csc(

arccsc(x))

= x and −π2≤ arccsc(x) ≤ π

2

Because csc(0) is undefined, arccsc(x) never takes the value 0.

arcsecx = arccos 1x

is defined for |x| ≥ 1. It is the unique number obeying

sec(

arcsec(x))

= x and 0 ≤ arcsec(x) ≤ π

Because sec(π/2) is undefined, arcsec(x) never takes the value π/2.

arccotx is defined for all x ∈ R. It is the unique number obeying

cot(

arccot(x))

= x and 0 < arccot(x) < π

2 In fact, there are two different widely used definitions of arcsecx. Under our definition, below,θ = arcsecx takes values in 0 ≤ θ ≤ π. Some people, perfectly legitimately, define θ = arcsecxto take values in the union of 0 ≤ θ < π

2 and π ≤ θ < 3π2 . Our definition is sometimes called

the “trigonometry friendly” definition. The definition itself has the advantage of simplicity. Theother definition is sometimes called the “calculus friendly” definition. It eliminates some abso-lute values and hence simplifies some computations. Similarly there are two different widelyused definitions of arccscx.

3 One could also define arccot(x) = arctan(1/x) with arccot(0) = π2 . We have chosen not to do so,

because the definition we have chosen is both continuous and standard.

236

Page 253: CLP-1 Differential Calculus

DERIVATIVES 2.12 INVERSE TRIGONOMETRIC FUNCTIONS

Example 2.12.5 The derivative of arccosx.

To find the derivative of arccos we can follow the same steps:

• Write arccos(x) = θ(x) so that cos θ = x and the desired derivative is dθdx

.

• Differentiate implicitly, remembering that θ is a function of x:

− sin θdθ

dx= 1

dx= − 1

sin θd

dxarccosx = − 1

sin(arccosx).

• To simplify this expression, again draw the relevant triangle

from which we see

sin(arccosx) = sin θ =√

1− x2.

• Thus

d

dxarccosx = − 1√

1− x2.

Example 2.12.6 The derivative of arctanx.

Very similar steps give the derivative of arctanx:

• Start with θ = arctanx, so tan θ = x.

• Differentiate implicitly:

sec2 θdθ

dx= 1

dx=

1

sec2 θ= cos2 θ

d

dxarctanx = cos2(arctanx).

237

Page 254: CLP-1 Differential Calculus

DERIVATIVES 2.12 INVERSE TRIGONOMETRIC FUNCTIONS

Example 2.12.6

• To simplify this expression, we draw the relevant triangle

from which we see

cos2(arctanx) = cos2 θ =1

1 + x2

• Thus

d

dxarctanx =

1

1 + x2.

An almost identical computation gives the derivative of arccotx:

• Start with θ = arccotx, so cot θ = x.

• Differentiate implicitly:

− csc2 θdθ

dx= 1

d

dxarccotx =

dx= − 1

csc2 θ= − sin2 θ = − 1

1 + x2

from the triangle

θ

√1 + x2

1

x

Example 2.12.7 The derivative of arccscx.

To find the derivative of arccsc we can use its definition and the chain rule.

θ = arccscx take cosecant of both sides

csc θ = x but csc θ =1

sin θ, so flip both sides

238

Page 255: CLP-1 Differential Calculus

DERIVATIVES 2.12 INVERSE TRIGONOMETRIC FUNCTIONS

Example 2.12.7

sin θ =1

xnow take arcsine of both sides

θ = arcsin

(1

x

)Now just differentiate, carefully using the chain rule :

dx=

d

dxarcsin

(1

x

)=

1√1− x−2

· −1

x2

To simplify further we will factor x−2 out of the square root. We need to be a littlecareful doing that. Take another look at examples 1.5.6 and 1.5.7 and the discussionbetween them before proceeding.

=1√

x−2(x2 − 1)· −1

x2

=1

|x−1| ·√x2 − 1

· −1

x2note that x2 · |x−1| = |x|.

= − 1

|x|√x2 − 1

In the same way we can find the derivative of the remaining inverse trig function.We just use its definition, a derivative we already know and the chain rule.

d

dxarcsec(x) =

d

dxarccos

(1

x

)= − 1√

1− 1x2

·(− 1

x2

)=

1

|x|√x2 − 1

By way of summary, we have

Theorem 2.12.8

The derivatives of the inverse trigonometric functions are

d

dxarcsin(x) =

1√1− x2

d

dxarccsc(x) = − 1

|x|√x2 − 1

d

dxarccos(x) = − 1√

1− x2

d

dxarcsec(x) =

1

|x|√x2 − 1

d

dxarctan(x) =

1

1 + x2

d

dxarccot(x) = − 1

1 + x2

239

Page 256: CLP-1 Differential Calculus

DERIVATIVES 2.12 INVERSE TRIGONOMETRIC FUNCTIONS

2.12.2 tt Exercises

Exercises — Stage 1

1. Give the domains of each of the following functions.

(a) f(x) = arcsin(cos x) (b) g(x) = arccsc(cos x)

(c) h(x) = sin(arccos x)

2. A particle starts moving at time t = 10, and it bobs up and down, so that itsheight at time t ≥ 10 is given by cos t. True or false: the particle has height1 at time t = arccos(1).

3. The curve y = f(x) is shown below, for some function f . Restrict f tothe largest possible interval containing 0 over which it is one–to–one,and sketch the curve y = f−1(x).

x

y

1

4. Let a be some constant. Where does the curve y = ax + cosx have a hori-zontal tangent line?

5. Define a function f(x) = arcsinx + arccscx. What is the domain of f(x)?Where is f(x) differentiable?

Exercises — Stage 2

6. Differentiate f(x) = arcsin(x

3

). What is the domain of f(x)?

7. Differentiate f(t) =arccos t

t2 − 1. What is the domain of f(t)?

8. Differentiate f(x) = arcsec(−x2 − 2). What is the domain of f(x)?

240

Page 257: CLP-1 Differential Calculus

DERIVATIVES 2.12 INVERSE TRIGONOMETRIC FUNCTIONS

9. Differentiate f(x) =1

aarctan

(xa

), where a is a nonzero constant. What is

the domain of f(x)?

10. Differentiate f(x) = x arcsinx+√

1− x2. What is the domain of f(x)?

11. For which values of x is the tangent line to y = arctan(x2) horizontal?

12. Evaluated

dxarcsinx+ arccosx.

13. *. Find the derivative of y = arcsin(

1x

).

14. *. Find the derivative of y = arctan(

1x

).

15. *. Calculate and simplify the derivative of (1 + x2) arctanx.

16. Show thatd

dxsin (arctan(x)) = (x2 + 1)−3/2.

17. Show thatd

dxcot (arcsin(x)) =

−1

x2√

1− x2.

18. *. Determine all points on the curve y = arcsinx where the tangent line isparallel to the line y = 2x+ 9.

19. For which values of x does the function f(x) = arctan(csc x) have a hori-zontal tangent line?

Exercises — Stage 3

20. *. Let f(x) = x+ cos x, and let g(y) = f−1(y) be the inverse function. Deter-mine g′(y).

21. *. f(x) = 2x− sin(x) is one–to–one. Find(f−1)′

(π − 1).

22. *. f(x) = ex + x is one–to–one. Find(f−1)′

(e+ 1).

23. Differentiate f(x) = [sinx+ 2]arcsecx. What is the domain of this function?

24. Suppose you can’t remember whether the derivative of arcsine is1√

1− x2or

1√x2 − 1

. Describe how the domain of arcsine suggests that

one of these is wrong.

25. Evaluate limx→1

((x− 1)−1

(arctanx− π

4

)).

241

Page 258: CLP-1 Differential Calculus

DERIVATIVES 2.13 THE MEAN VALUE THEOREM

26. Suppose f(2x+ 1) =5x− 9

3x+ 7. Evaluate f−1(7).

27. Suppose f−1(4x− 1) =2x+ 3

x+ 1. Evaluate f(0).

28. Suppose a curve is defined implicitly by

arcsin(x+ 2y) = x2 + y2

Solve for y′ in terms of x and y.

2.13q The Mean Value Theorem

Consider the following situation. Two towns are separated by a 120km long stretchof road. The police in town A observe a car leaving at 1pm. Their colleagues intown B see the car arriving at 2pm. After a quick phone call between the two policestations, the driver is issued a fine for going 120km/h at some time between 1pmand 2pm. It is intuitively obvious 1 that, because his average velocity was 120km/h,the driver must have been going at least 120km/h at some point. From a knowl-edge of the average velocity of the car, we are able to deduce something about aninstantaneous velocity 2 .

Let us turn this around a little bit. Consider the premise of a 90s action film 3

— a bus must travel at a velocity of no less than 80km/h. Being a bus, it is unableto go faster than, say, 120km/h. The film runs for about 2 hours, and let’s assumethat there is about thirty minutes of non-action — so the bus’ velocity is constrainedbetween 80 and 120km/h for a total of 1.5 hours.

It is again obvious that the bus must have travelled between 80 × 1.5 = 120 and120×1.5 = 180km during the film. This time, from a knowledge of the instantaneousrate of change of position — the derivative — throughout a 90 minute time interval,we are able to say something about the net change of position during the 90 minutes.

In both of these scenarios we are making use of a piece of mathematics called theMean Value Theorem. It says that, under appropriate hypotheses, the average rateof change f(b)−f(a)

b−a of a function over an interval is achieved exactly by the instanta-

1 Unfortunately there are many obvious things that are decidedly false — for example “There aremore rational numbers than integers.” or “Viking helmets had horns on them”.

2 Recall that speed and velocity are not the same. Velocity specifies the direction of motion as wellas the rate of change. Objects moving along a straight line have velocities that are positive ornegative numbers indicating which direction the object is moving along the line. Speed, on theother hand, is the distance travelled per unit time and is always a non-negative number — it isthe absolute value of velocity.

3 The sequel won a Raspberry award for “Worst remake or sequel”.

242

Page 259: CLP-1 Differential Calculus

DERIVATIVES 2.13 THE MEAN VALUE THEOREM

neous rate of change f ′(c) of the function at some 4 (unknown) point a ≤ c ≤ b. Weshall get to a precise statement in Theorem 2.13.5. We start working up to it by firstconsidering the special case in which f(a) = f(b).

2.13.1 tt Rolle’s Theorem

Theorem 2.13.1 Rolle’s theorem.

Let a and b be real numbers with a < b. And let f be a function so that

• f(x) is continuous on the closed interval a ≤ x ≤ b,

• f(x) is differentiable on the open interval a < x < b, and

• f(a) = f(b)

then there is a c strictly between a and b, i.e. obeying a < c < b, such that

f ′(c) = 0.

Again, like the two scenarios above, this theorem says something intuitively ob-vious. Consider — if you throw a ball straight up into the air and then catch it, atsome time in between the throw and the catch it must be stationary. Translating thisinto mathematical statements, let s(t) be the height of the ball above the ground inmetres, and let t be time from the moment the ball is thrown in seconds. Then wehave

s(0) = 1 we release the ball at about hip-heights(4) = 1 we catch the ball 4s later at hip-height

Then we know there is some time in between — say at t = c — when the ball isstationary (in this case when the ball is at the top of its trajectory). I.e.

v(c) = s′(c) = 0.

Rolle’s theorem guarantees that for any differentiable function that starts and endsat the same value, there will always be at least one point between the start and finishwhere the derivative is zero.

4 There must be at least one such point — there could be more than one — but there cannot bezero.

243

Page 260: CLP-1 Differential Calculus

DERIVATIVES 2.13 THE MEAN VALUE THEOREM

There can, of course, also be multiple points at which the derivative is zero — butthere must always be at least one. Notice, however, the theorem 5

does not tell us the value of c, just that such a c must exist.

Example 2.13.2 A simple application of Rolle’s theorem.

We can use Rolle’s theorem to show that the function

f(x) = sin(x)− cos(x)

has a point c between 0 and 3π2

so that f ′(c) = 0.To apply Rolle’s theorem we first have to show the function satisfies the conditionsof the theorem on the interval [0, 3π

2].

• Since f is the sum of sine and cosine it is continuous on the interval and alsodifferentiable on the interval.

• Further, since

f(0) = sin 0− cos 0 = 0− 1 = −1

f

(3π

2

)= sin

2− cos

2= −1− 0 = −1

we can now apply Rolle’s theorem.

• Rolle’s theorem implies that there must be a point c ∈ (0, 3π/2) so that f ′(c) = 0.

While Rolle’s theorem doesn’t tell us the value of c, this example is sufficiently sim-ple that we can find it directly.

f ′(x) = cos x+ sinx

f ′(c) = cos c+ sin c = 0 rearrangesin c = − cos c and divide by cos c

tan c = −1

Hence c = 3π4

. We have sketched the function and the relevant points below.

5 Notice this is very similar to the intermediate value theorem (see Theorem 1.6.12)

244

Page 261: CLP-1 Differential Calculus

DERIVATIVES 2.13 THE MEAN VALUE THEOREM

Example 2.13.2

A more substantial application of Rolle’s theorem (in conjunction with the inter-mediate value theorem — Theorem 1.6.12) is to show that a function does not havemultiple zeros in an interval:

Example 2.13.3 Showing an equation has exactly 1 solution.

Show that the equation 2x− 1 = sin(x) has exactly 1 solution.

• Start with a rough sketch of each side of the equation

This seems like it should be true.

• Notice that the problem we are trying to solve is equivalent to showing thatthe function

f(x) = 2x− 1− sin(x)

has only a single zero.

• Since f(x) is the sum of a polynomial and a sine function, it is continuousand differentiable everywhere. Thus we can apply both the IVT and Rolle’stheorem.

245

Page 262: CLP-1 Differential Calculus

DERIVATIVES 2.13 THE MEAN VALUE THEOREM

Example 2.13.3

• Notice that f(0) = −1 and f(2) = 4 − 1 − sin(2) = 3 − sin(2) ≥ 2, since−1 ≤ sin(2) ≤ 1. Thus by the IVT we know there is at least one number cbetween 0 and 2 so that f(c) = 0.

• But our job is only half done — this shows that there is at least one zero, butit does not tell us there is no more than one. We have more work to do, andRolle’s theorem is the tool we need.

• Consider what would happen if f(x) is zero in 2 places — that is, there arenumbers a, b so that f(a) = f(b) = 0.

Since f(x) is differentiable everywhere and f(a) = f(b) = 0, we can applyRolle’s theorem.

Hence we know there is a point c between a and b so that f ′(c) = 0.

But let us examine f ′(x):

f ′(x) = 2− cosx

Since −1 ≤ cosx ≤ 1, we must have that f ′(x) ≥ 1.

But this contradicts Rolle’s theorem which tells us there must be a pointat which the derivative is zero.

Thus the function cannot be zero at two different places — otherwise we’d havea contradiction.

We can actually nail down the value of c using the bisection approach we used inexample 1.6.15. If we do this carefully we find that c ≈ 0.887862 . . .

2.13.2 tt Back to the MVT

Rolle’s theorem can be generalised in a straight-forward way; given a differentiablefunction f(x) we can still say something about df

dx, even if f(a) 6= f(b). Consider the

following sketch:

246

Page 263: CLP-1 Differential Calculus

DERIVATIVES 2.13 THE MEAN VALUE THEOREM

Figure 2.13.4

All we have done is tilt the picture so that f(a) < f(b). Now we can no longerguarantee that there will be a point on the graph where the tangent line is horizon-tal, but there will be a point where the tangent line is parallel to the secant joining(a, f(a)) to (b, f(b)).

To state this in terms of our first scenario back at the beginning of this section,suppose that you are driving along the x–axis. At time t = a you are at x = f(a)and at time t = b you are at x = f(b). For simplicity, let’s suppose that b > a andf(b) ≥ f(a), just like in the above sketch. Then during the time interval in questionyou travelled a net distance of f(b)−f(a). It took you b−a units of time to travel thatdistance, so your average velocity was f(b)−f(a)

b−a . You may very well have been goingfaster than f(b)−f(a)

b−a part of the time and slower than f(b)−f(a)b−a part of the time. But it

is reasonable to guess that at some time between t = a and t = b your instantaneousvelocity was exactly f(b)−f(a)

b−a . The mean value theorem says that, under reasonableassumptions about f , this is indeed the case.

Theorem 2.13.5 The mean value theorem.

Let a and b be real numbers with a < b. And let f(x) be a function so that

• f(x) is continuous on the closed interval a ≤ x ≤ b, and

• f(x) is differentiable on the open interval a < x < b

then there is a c ∈ (a, b), such that

f ′(c) =f(b)− f(a)

b− awhich we can also express as

f(b) = f(a) + f ′(c)(b− a).

Let us start to explore the mean value theorem — which is very frequently known

247

Page 264: CLP-1 Differential Calculus

DERIVATIVES 2.13 THE MEAN VALUE THEOREM

as the MVT. A simple example to start:

Example 2.13.6 Apply MVT to a polynomial.

Consider the polynomial f(x) = 3x2 − 4x+ 2 on [−1, 1].

• Since f is a polynomial it is continuous on the interval and also differentiableon the interval. Hence we can apply the MVT.

• The MVT tells us that there is a point c ∈ (−1, 1) so that

f ′(c) =f(1)− f(−1)

1− (−1)=

1− 9

2= −4

This example is sufficiently simple that we can find the point c and the correspondingtangent line:

• The derivative is

f ′(x) = 6x− 4

• So we need to solve f ′(c) = −4:

6c− 4 = −4

which tells us that c = 0.

• The tangent line has slope −4 and passes through (0, f(0)) = (0, 2), and so isgiven by

y = −4x+ 2

• The secant line joining (−1, f(−1)) = (−1, 9) to (1, f(1)) = (1, 1) is just

y = 5− 4x

• Here is a sketch of the curve and the two lines:

−1 −0.5 0.5 1

3

6

9 3x2 − 4x+ 25− 4x2− 4x

248

Page 265: CLP-1 Differential Calculus

DERIVATIVES 2.13 THE MEAN VALUE THEOREM

Example 2.13.7 MVT, speed and distance.

We can return to our initial car-motivated examples. Say you are driving along astraight road in a car that can go at most 80km/h. How far can you go in 2 hours? —the answer is easy, but we can also solve this using MVT.

• Let s(t) be the position of the car in km at time t measured in hours.

• Then s(0) = 0 and s(2) = q, where q is the quantity that we need to bound.

• We are told that |s′(t)| ≤ 80, or equivalently

−80 ≤ s′(t) ≤ 80

• By the MVT there is some c between 0 and 2 so that

s′(c) =q − 0

2=q

2

• Now since −80 ≤ s′(c) ≤ 80 we must have −80 ≤ q/2 ≤ 80 and hence −160 ≤q = s(2) ≤ 160.

More generally if we have some information about the derivative, then we canuse the MVT to leverage this information to tell us something about the function.

Example 2.13.8 Using MVT to bound a function.

Let f(x) be a differentiable function so that

f(1) = 10 and −1 ≤ f ′(x) ≤ 2 everywhere

Obtain upper and lower bounds on f(5).Okay — what do we do?

• Since f(x) is differentiable we can use the MVT.

• Say f(5) = q, then the MVT tells us that there is some c between 1 and 5 suchthat

f ′(c) =q − 10

5− 1=q − 10

4

• But we know that −1 ≤ f ′(c) ≤ 2, so

−1 ≤ f ′(c) ≤ 2

−1 ≤ q − 10

4≤ 2

−4 ≤ q − 10 ≤ 8

6 ≤ q ≤ 18

249

Page 266: CLP-1 Differential Calculus

DERIVATIVES 2.13 THE MEAN VALUE THEOREM

Example 2.13.8

• Thus we must have 6 ≤ f(5) ≤ 18.

2.13.3 tt (Optional) — Why is the MVT True

We won’t give a real proof for this theorem, but we’ll look at a picture which showswhy it is true. Here is the picture. It contains a sketch of the graph of f(x), with xrunning from a to b, as well as a line segment which is the secant of the graph fromthe point

(a , f(a)

)to the point

(b , f(b)

). The slope of the secant is exactly f(b)−f(a)

b−a .

Remember that we are looking for a point,(c , f(c)

), on the graph of f(x) with the

property that f ′(c) = f(b)−f(a)b−a , i.e. with the property that the slope of the tangent line

at(c , f(c)

)is the same as the slope of the secant. So imagine that you start moving

the secant upward, carefully keeping the moved line segment parallel to the secant.So the slope of the moved line segment is always exactly f(b)−f(a)

b−a . When we first startmoving the line segment it is not tangent to the curve — it crosses the curve. Thisis illustrated in the figure by the second line segment from the bottom. If we movethe line segment too far it does not touch the curve at all. This is illustrated in thefigure by the top segment. But if we stop moving the line segment just before it stopsintersecting the curve at all, we get exactly the tangent line to the curve at the pointon the curve that is farthest from the secant. This tangent line has exactly the desiredslope. This is illustrated in the figure by the third line segment from the bottom.

2.13.4 tt Be Careful with Hypotheses

The mean value theorem has hypotheses — f(x) has to be continuous for a ≤ x ≤ band has to be differentiable for a < x < b. If either hypothesis is violated, theconclusion of the mean value theorem can fail. That is, the curve y = f(x) neednot have a tangent line at some x = c between a and b whose slope, f ′(c), is the same

250

Page 267: CLP-1 Differential Calculus

DERIVATIVES 2.13 THE MEAN VALUE THEOREM

as the slope, f(b)−f(a)b−a , of the secant joining the points

(a , f(a)

)and

(b , f(b)

)on the

curve. If f ′(x) fails to exist for even a single value of x between a and b, all bets areoff. The following two examples illustrate this.

Example 2.13.9 MVT doesn’t work here.

For the first “bad” example, a = 0, b = 2 and

f(x) =

0 if x ≤ 1

1 if x > 1

For this example, f ′(x) = 0 at every x where it is defined. That is, at every x 6= 1. Butthe slope of the secant joining

(a , f(a)

)= (0, 0) and

(b , f(b)

)= (2, 1) is 1

2.

Example 2.13.10 MVT doesn’t work here either.

For the second “bad” example, a = −1, b = 1 and f(x) = |x|. For this function

f ′(x) =

−1 if x < 0

undefined if x = 0

1 if x > 0

For this example, f ′(x) = ±1 at every x where it is defined. That is, at every x 6= 0.But the slope of the secant joining

(a , f(a)

)= (−1, 1) and

(b , f(b)

)= (1, 1) is 0.

Example 2.13.11 MVT does work on this one.

Here is one “good” example, where the hypotheses of the mean value theorem aresatisfied. Let f(x) = x2. Then f ′(x) = 2x. For any a < b,

f(b)− f(a)

b− a =b2 − a2

b− a = b+ a

So f ′(c) = 2c is exactly f(b)−f(a)b−a when c = a+b

2, which, in this example, happens to be

exactly half way between x = a and x = b.

251

Page 268: CLP-1 Differential Calculus

DERIVATIVES 2.13 THE MEAN VALUE THEOREM

Example 2.13.11

A simple consequence of the mean value theorem is that if you know the sign off ′(c) for all c’s between a and b, with b > a, then f(b)− f(a) = f ′(c)(b− a) must havethe same sign.

Corollary 2.13.12 Consequences of the mean value theorem.

Let A and B be real numbers with A < B. Let function f(x) be defined andcontinuous on the closed interval A ≤ x ≤ B and be differentiable on the openinterval A < x < B.

a If f ′(c) = 0 for all A < c < B, then f(b) = f(a) for all A ≤ a < b ≤ B.

— That is, f(x) is constant on A ≤ x ≤ B.

b If f ′(c) ≥ 0 for all A < c < B, then f(b) ≥ f(a) for all A ≤ a ≤ b ≤ B.

— That is, f(x) is increasing on A ≤ x ≤ B.

c If f ′(c) ≤ 0 for all A < c < B, then f(b) ≤ f(a) for all A ≤ a ≤ b ≤ B.

— That is, f(x) is decreasing on A ≤ x ≤ B.

It is not hard to see why the above is true:

• Say f ′(x) = 0 at every point in the interval [A,B]. Now pick any a, b ∈ [A,B]with a < b. Then the MVT tells us that there is c ∈ (a, b) so that

f ′(c) =f(b)− f(a)

b− aIf f(b) 6= f(a) then we must have that f ′(c) 6= 0 — contradicting what we aretold about f ′(x). Thus we must have that f(b) = f(a).

• Similarly, say f ′(x) ≥ 0 at every point in the interval [A,B]. Now pick anya, b ∈ [A,B] with a < b. Then the MVT tells us that there is c ∈ (a, b) so that

f ′(c) =f(b)− f(a)

b− a

252

Page 269: CLP-1 Differential Calculus

DERIVATIVES 2.13 THE MEAN VALUE THEOREM

Since b > a, the denominator is positive. Now if f(b) < f(a) the numeratorwould be negative, making the right-hand side negative, and contradictingwhat we are told about f ′(x). Hence we must have f(b) ≥ f(a).

A nice corollary of the above corollary is the following:

Corollary 2.13.13

If f ′(x) = g′(x) for all x in the open interval (a, b), then f − g is a constant on(a, b). That is f(x) = g(x) + c, where c is some constant.

We can prove this by setting h(x) = f(x) − g(x). Then h′(x) = 0 and so theprevious corollary tells us that h(x) is constant.

Example 2.13.14 Summing arcsin and arccos.

Using this corollary we can prove results like the following:

arcsinx+ arccosx =π

2for all − 1 < x < 1

How does this work? Let f(x) = arcsin x+ arccosx. Then

f ′(x) =1√

1− x2+

−1√1− x2

= 0

Thus f must be a constant. To find out which constant, we can just check its value ata convenient point, like x = 0.

arcsin(0) + arccos(0) = π/2 + 0 = π/2

Since the function is constant, this must be the value.

2.13.5 tt Exercises

Exercises — Stage 1

1. Suppose a particular caribou has a top speed of 70 kph, and in one yearit migrates 5000 km. What do you know about the amount of time thecaribou spent travelling during its migration?

253

Page 270: CLP-1 Differential Calculus

DERIVATIVES 2.13 THE MEAN VALUE THEOREM

2. Suppose a migrating sandhill crane flew 240 kilometres in one day.What does the mean value theorem tell you about its speed during thatday?

3. Below is the graph of y = f(x), where x is continuous on [a, b] anddifferentiable on (a, b). Mark on the graph the approximate location ofa value c guaranteed by the mean value theorem.

x

y

a b

4. Give a function f(x) with the properties that:

• f(x) is differentiable on the open interval 0 < x < 10

• f(0) = 0, f(10) = 10

but for all c ∈ (0, 10), f ′(c) = 0.

5. For each of the parts below, sketch a function f(x) (different in eachpart) that is continuous and differentiable over all real numbers, withf(1) = f(2) = 0, and with the listed property, or explain why no suchfunction exists.

a f ′(c) = 0 for no point c ∈ (1, 2)

b f ′(c) = 0 for exactly one point c ∈ (1, 2)

c f ′(c) = 0 for exactly five points c ∈ (1, 2)

d f ′(c) = 0 for infinitely many points c ∈ (1, 2)

6. Suppose you want to show that a point exists where the function f(x) =√|x| has a tangent line with slope 1

13. Find the mistake(s) in the following

254

Page 271: CLP-1 Differential Calculus

DERIVATIVES 2.13 THE MEAN VALUE THEOREM

work, and give a correct proof.

The function f(x) is continuous and differentiable over all realnumbers, so the mean value theorem applies. f(−4) = 2 andf(9) = 3, so by the mean value theorem, there exists some c ∈(−4, 9) such that f ′(x) =

3− 2

9− (−4)=

1

13.

Exercises — Stage 2

7. *. Let f(x) = x2− 2πx+ cos(x)− 1. Show that there exists a real numberc such that f ′(c) = 0.

8. *. Let f(x) = ex + (1− e)x2 − 1. Show that there exists a real number c suchthat f ′(c) = 0.

9. *. Let f(x) =√

3 + sin(x) + (x− π)2. Show that there exists a real number csuch that f ′(c) = 0.

10. *. Let f(x) = x cos(x)−x sin(x). Show that there exists a real number c suchthat f ′(c) = 0.

11. How many roots does the function f(x) = 3x− sinx have?

12. How many roots does the function f(x) =(4x+ 1)4

16+ x have?

13. How many roots does the function f(x) = x3 + sin (x5) have?

14. How many positive-valued solutions does the equation ex = 4 cos(2x)have?

15. *. Let f(x) = 3x5 − 10x3 + 15x+ a, where a is some constant.

a Prove that, regardless of the value a, f ′(x) > 0 for all x in (−1, 1).

b Prove that, regardless of the value a, f(x) = 3x5 − 10x3 + 15x + ahas at most one root in [−1, 1].

16. *. Find the point promised by the Mean Value Theorem for the functionex on the interval [0, T ].

17. Use Corollary 2.13.12 and Theorem 2.12.8 to show that arcsecx = C −arccscx for some constant C; then find C.

Exercises — Stage 3

255

Page 272: CLP-1 Differential Calculus

DERIVATIVES 2.14 HIGHER ORDER DERIVATIVES

18. *. Suppose f(0) = 0 and f ′(x) =1

1 + e−f(x). Prove that f(100) < 100.

Remark: an equation relating a function to its own derivative is called adifferential equation. We’ll see some very basic differential equations inSection 3.3.

19. Let f(x) = 2x + sinx. What is the largest interval containing x = 0 overwhich f(x) is one–to–one? What are the domain and range of f−1(x)?

20. Let f(x) =x

2+ sin x. What is the largest interval containing x = 0 over

which f(x) is one–to–one? What are the domain and range of f−1(x), if werestrict f to this interval?

21. Suppose f(x) and g(x) are functions that are continuous over the interval[a, b] and differentiable over the interval (a, b). Suppose further that f(a) <g(a) and g(b) < f(b). Show that there exists some c ∈ [a, b] with f ′(c) > g′(c).

22. Suppose f(x) is a function that is differentiable over all real numbers,and f ′(x) has precisely two roots. What is the maximum number ofdistinct roots that f(x) may have?

23. How many roots does f(x) = sinx+ x2 + 5x+ 1 have?

2.14q Higher Order Derivatives

2.14.1 tt Higher Order Derivatives

The operation of differentiation takes as input one function, f(x), and produces asoutput another function, f ′(x). Now f ′(x) is once again a function. So we can dif-ferentiate it again, assuming that it is differentiable, to create a third function, calledthe second derivative of f . And we can differentiate the second derivative again tocreate a fourth function, called the third derivative of f . And so on.

Definition 2.14.1

• f ′′(x) and f (2)(x) and d2fdx2

(x) all mean ddx

(d

dxf(x)

)• f ′′′(x) and f (3)(x) and d3f

dx3(x) all mean d

dx

(d

dx

(d

dxf(x)

))• f (4)(x) and d4f

dx4(x) both mean d

dx

(d

dx

(d

dx

(d

dxf(x)

)))256

Page 273: CLP-1 Differential Calculus

DERIVATIVES 2.14 HIGHER ORDER DERIVATIVES

• and so on.

Here is a simple example. Then we’ll think a little about the significance of secondorder derivatives. Then we’ll do a more a computationally complex example.

Example 2.14.2 Derivatives of xn.

Let n be a natural number and let f(x) = xn. Then

d

dxxn = nxn−1

d2

dx2xn =

d

dx

(nxn−1

)= n(n− 1)xn−2

d3

dx3xn =

d

dx

(n(n− 1)xn−2

)= n(n− 1)(n− 2)xn−3

Each time we differentiate, we bring down the exponent, which is exactly onesmaller than the previous exponent brought down, and we reduce the exponent byone. By the time we have differentiated n − 1 times, the exponent has decreased ton− (n− 1) = 1 and we have brought down the factors n(n− 1)(n− 2) · · · 2. So

dn−1

dxn−1xn = n(n− 1)(n− 2) · · · 2x

and

dn

dxnxn = n(n− 1)(n− 2) · · · 1

The product of the first n natural numbers, 1 · 2 · 3 · · · · · n, is called “n factorial” andis denoted n!. So we can also write

dn

dxnxn = n!

If m > n, then

dm

dxmxn = 0

Example 2.14.3 Position, velocity and acceleration.

Recall that the derivative v′(a) is the (instantaneous) rate of change of the functionv(t) at t = a. Suppose that you are walking on the x–axis and that x(t) is yourx–coordinate at time t. Also suppose, for simplicity, that you are moving from left toright. Then v(t) = x′(t) is your velocity at time t and v′(a) = x′′(a) is the rate at which

257

Page 274: CLP-1 Differential Calculus

DERIVATIVES 2.14 HIGHER ORDER DERIVATIVES

Example 2.14.3

your velocity is changing at time t = a. It is called your acceleration. In particular,if x′′(a) > 0, then your velocity is increasing, i.e. you are speeding up, at time a. Ifx′′(a) < 0, then your velocity is decreasing, i.e. you are slowing down, at time a.That’s one interpretation of the second derivative.

Example 2.14.4 2.11.1 continued.

Find y′′ if y = y3 + xy + x3.Solution This problem concerns some function y(x) that is not given to us explicitly.All that we are told is that y(x) satisfies

y(x) = y(x)3 + xy(x) + x3 (E1)

for all x. We are asked to find y′′(x). We cannot solve this equation to get an explicitformula for y(x). So we use implicit differentiation, as we did in Example 2.11.1.That is, we apply d

dxto both sides of (E1). This gives

y′(x) = 3y(x)2 y′(x) + y(x) + x y′(x) + 3x2 (E2)

which we can solve for y′(x), by moving all y′(x)’s to the left hand side, giving[1− x− 3y(x)2

]y′(x) = y(x) + 3x2

and then dividing across.

y′(x) =y(x) + 3x2

1− x− 3y(x)2(E3)

To get y′′(x), we have two options.Method 1. Apply d

dxto both sides of (E2). This gives

y′′(x) = 3y(x)2 y′′(x) + 6y(x) y′(x)2 + 2y′(x) + x y′′(x) + 6x

We can now solve for y′′(x), giving

y′′(x) =6x+ 2y′(x) + 6y(x)y′(x)2

1− x− 3y(x)2(E4)

Then we can substitute in (E3), giving

y′′(x) = 23x+ y(x)+3x2

1−x−3y(x)2+ 3y(x)

( y(x)+3x2

1−x−3y(x)2

)2

1− x− 3y(x)2

= 23x[1−x−3y(x)2]

2+ [y(x)+3x2][1−x−3y(x)2] + 3y(x)[y(x)+3x2]

2

[1− x− 3y(x)2]3

258

Page 275: CLP-1 Differential Calculus

DERIVATIVES 2.14 HIGHER ORDER DERIVATIVES

Example 2.14.4

Method 2. Alternatively, we can also differentiate (E3).

y′′(x) =[y′(x) + 6x][1−x−3y(x)2]− [y(x) + 3x2][−1− 6y(x)y′(x)]

[1− x− 3y(x)2]2

=

[ y(x)+3x2

1−x−3y(x)2+ 6x

][1− x− 3y(x)2]− [y(x) + 3x2][−1− 6y(x) y(x)+3x2

1−x−3y(x)2]

[1− x− 3y(x)2]2

=2[y(x) + 3x2][1−x−3y(x)2] + 6x[1−x−3y(x)2]

2+ 6y(x)[y(x)+3x2]

2

[1− x− 3y(x)2]3

Remark 1. We have now computed y′′(x) — sort of. The answer is in terms of y(x),which we don’t know. Since we cannot get an explicit formula for y(x), there’s not agreat deal that we can do, in general.Remark 2. Even though we cannot solve y = y3+xy+x3 explicitly for y(x), for generalx, it is sometimes possible to solve equations like this for some special values of x.In fact, we saw in Example 2.11.1 that when x = 1, the given equation reduces toy(1) = y(1)3 + 1 · y(1) + 13, or y(1)3 = −1, which we can solve to get y(1) = −1.Substituting into (E2), as we did in Example 2.11.1 gives

y′(1) =−1 + 3

1− 1− 3(−1)2= −2

3

and substituting into (E4) gives

y′′(1) =6 + 2

(− 2

3

)+ 6(−1)

(− 2

3

)2

1− 1− 3(−1)2=

6− 43− 8

3

−3= −2

3

(It’s a fluke that, in this example, y′(1) and y′′(1) happen to be equal.) So we nowknow that, even though we can’t solve y = y3 + xy + x3 explicitly for y(x), the graphof the solution passes through (1,−1) and has slope −2

3(i.e. is sloping downwards

by between 30 and 45) there and, furthermore, the slope of the graph decreases asx increases through x = 1.

Here is a sketch of the part of the graph very near (1,−1). The tangent line to thegraph at (1,−1) is also shown. Note that the tangent line is sloping down to theright, as we expect, and that the graph lies below the tangent line near (1,−1). That’sbecause the slope f ′(x) is decreasing (becoming more negative) as x passes through1.

259

Page 276: CLP-1 Differential Calculus

DERIVATIVES 2.14 HIGHER ORDER DERIVATIVES

Warning 2.14.5

Many people will suppress the (x) in y(x) when doing computations like thosein Example 2.14.4. This gives shorter, easier to read formulae, like y′ = y+3x2

1−x−3y2.

If you do this, you must never forget that y is a function of x and is not aconstant. If you do forget, you’ll make the very serious error of saying thatdydx

= 0, which is false.

2.14.2 tt Exercises

Exercises — Stage 1

1. What is the 180th derivative of the function f(x) = ex?

2. Suppose f(x) is a differentiable function, with f ′(x) > 0 and f ′′(x) > 0for every x ∈ (a, b). Which of the following must be true?

i f(x) is positive over (a, b)

ii f(x) is increasing over (a, b)

iii f(x) is increasing at a constant rate over (a, b)

iv f(x) is increasing faster and faster over (a, b)

v f ′′′′(x) > 0 for some x ∈ (a, b)

3. Let f(x) = ax15 for some constant a. Which value of a results in f (15)(x) = 3?

4. Find the mistake(s) in the following work, and provide a corrected an-swer.

Suppose−14x2 +2xy+y2 = 1. We findd2y

dx2at the point (1, 3).

Differentiating implicitly:

−28x+ 2y + 2xy′ + 2yy′ = 0

Plugging in x = 1, y = 3:

−28 + 6 + 2y′ + 6y′ = 0

y′ =11

4

260

Page 277: CLP-1 Differential Calculus

DERIVATIVES 2.14 HIGHER ORDER DERIVATIVES

Differentiating:

y′′ = 0

Exercises — Stage 2

5. Let f(x) = (log x− 1)x. Evaluate f ′′(x).

6. Evaluated2

dx2arctanx.

7. The unit circle consists of all point x2 + y2 = 1. Give an expression ford2y

dx2

in terms of y.

8. Suppose the position of a particle at time t is given by s(t) =et

t2 + 1.

Find the acceleration of the particle at time t = 1.

9. Evaluated3

dx3log(5x2 − 12).

10. The height of a particle at time t seconds is given by h(t) = − cos t. Is theparticle speeding up or slowing down at t = 1?

11. The height of a particle at time t seconds is given by h(t) = t3−t2−5t+10.Is the particle’s motion getting faster or slower at t = 1?

12. Suppose a curve is defined implicitly by

x2 + x+ y = sin(xy)

What isd2y

dx2at the point (0, 0)?

13. Which statements below are true, and which false?

ad4

dx4sinx = sinx

bd4

dx4cosx = cosx

cd4

dx4tanx = tanx

Exercises — Stage 3

261

Page 278: CLP-1 Differential Calculus

DERIVATIVES 2.14 HIGHER ORDER DERIVATIVES

14. A function f(x) satisfies f ′(x) < 0 and f ′′(x) > 0 over (a, b). Which ofthe following curves below might represent y = f(x)?

x

y

a b

(i)

x

y

a b

(ii)

x

y

a b

(iii)

x

y

a b

(iv)

x

y

a b

(v)

15. Let f(x) = 2x. What is f (n)(x), if n is a whole number?

16. Let f(x) = ax3 + bx2 + cx+ d, where a, b, c, and d are nonzero constants.

What is the smallest integer n so thatdnf

dxn= 0 for all x?

17. *.f(x) = ex+x2 h(x) = 1 + x+

3

2x2

a Find the first and second derivatives of both functions

b Evaluate both functions and their first and second derivatives at 0.

c Show that for all x > 0, f(x) > h(x).

Remark: for some applications, we only need to know that a function is“big enough.” Since f(x) is a difficult function to evaluate, it may be usefulto know that it is bigger than h(x) when x is positive.

18. *. The equation x3y + y3 = 10x defines y implicitly as a function of x nearthe point (1, 2).

a Compute y′ at this point.

b It can be shown that y′′ is negative when x = 1. Use this fact and youranswer to 2.14.2.18.a to make a sketch showing the relationship of thecurve to its tangent line at (1, 2).

262

Page 279: CLP-1 Differential Calculus

DERIVATIVES 2.15 (OPTIONAL) — IS limx→c f ′(x) EQUAL TO f ′(c)?

19. Let g(x) = f(x)ex. In Question 2.7.3.12, Section 2.7, we learned that g′(x) =[f(x) + f ′(x)]ex.

a What is g′′(x)?

b What is g′′′(x)?

c Based on your answers above, guess a formula for g(4)(x). Check it bydifferentiating.

20. Suppose f(x) is a function whose first n derivatives exist over all real num-bers, and f (n)(x) has precisely m roots. What is the maximum number ofroots that f(x) may have?

21. How many roots does the function f(x) = (x+1) log(x+1)+sinx−x2 have?

22. *. Let f(x) = x|x|.

a Show that f(x) is differentiable at x = 0, and find f ′(0).

b Find the second derivative of f(x). Explicitly state, with justifica-tion, the point(s) at which f ′′(x) does not exist, if any.

2.15q (Optional) — Is limx→c f ′(x) Equal to f ′(c)?

Consider the function

f(x) =

sinx2

xif x 6= 0

0 if x = 0

For any x 6= 0 we can easily use our differentiation rules to find

f ′(x) =2x2 cosx2 − sinx2

x2

But for x = 0 none of our usual differentation rules apply. So how do we findf ′(0)? One obviously legitimate strategy is to directly apply the Definition 2.2.1 ofthe derivative. As an alternative, we will now consider the question “Can one findf ′(0) by taking the limit of f ′(x) as x tends to zero?”. There is bad news and there isgood news.

• The bad news is that, even for functions f(x) that are differentiable for all x,f ′(x) need not be continuous. That is, it is not always true that limx→0 f

′(x) =f ′(0). We will see a function for which limx→0 f

′(x) 6= f ′(0) in Example 2.15.1,below.

263

Page 280: CLP-1 Differential Calculus

DERIVATIVES 2.15 (OPTIONAL) — IS limx→c f ′(x) EQUAL TO f ′(c)?

• The good news is that Theorem 2.15.2, below provides conditions which aresufficient to guarantee that f(x) is differentiable at x = 0 and that limx→0 f

′(x) =f ′(0).

Example 2.15.1

Consider the function

f(x) =

x2 sin 1

xif x 6= 0

0 if x = 0

For x 6= 0 we have, by the product and chain rules,

f ′(x) = 2x sin1

x+ x2

(cos

1

x

)(− 1

x2

)= 2x sin

1

x− cos

1

x

As∣∣sin 1

x

∣∣ ≤ 1, we have

limx→0

2x sin1

x= 0

On the other hand, as x tends to zero, 1x

goes to ±∞. So

limx→0

cos1

x= DNE =⇒ lim

x→0f ′(x) = DNE

We will now see that, despite this, f ′(0) is perfectly well defined. By definition

f ′(0) = limh→0

f(h)− f(0)

h

= limh→0

h2 sin 1h− 0

h

= limh→0

h sin1

h

= 0 since∣∣∣∣sin 1

h

∣∣∣∣ ≤ 1

So f ′(0) exists, but is not equal to limx→0 f′(x), which does not exist.

Now for the good news.

Theorem 2.15.2

Let a < c < b. Assume that

• the function f(x) is continous on the interval a < x < b and

264

Page 281: CLP-1 Differential Calculus

DERIVATIVES 2.15 (OPTIONAL) — IS limx→c f ′(x) EQUAL TO f ′(c)?

• is differentiable at every x in the intervals a < x < c and c < x < b and

• the limit limx→c f ′(x) exists.

Then f is differentiable at x = c and

f ′(c) = limx→c

f ′(x)

Proof. By hypothesis, there is a number L such that

limx→c

f ′(x) = L

By definition

f ′(c) = limh→0

f(c+ h)− f(c)

h

By the Mean Value Theorem (Theorem 2.13.5 ) there is, for each h, an (un-known) number xh between c and c+ h such that f ′(xh) = f(c+h)−f(c)

h. So

f ′(c) = limh→0

f ′(xh)

As h tends to zero, c + h tends to c, and so xh is forced to tend to c, and f ′(xh)is forced to tend to L so that

f ′(c) = limh→0

f ′(xh) = L

In the next example we evaluate f ′(0) by applying Theorem 2.15.2.

Example 2.15.3

Let

f(x) =

sinx2

xif x 6= 0

0 if x = 0

We have already observed above that, for x 6= 0,

f ′(x) =2x2 cosx2 − sinx2

x2= 2 cos x2 − sinx2

x2

We use Theorem 2.15.2 with c = 0 to show that f(x) is differentiable at x = 0 andto evaluate f ′(0). That theorem has two hypotheses that we have not yet verified,namely the continuity of f(x) at x = 0, and the existence of the limit limx→0 f

′(x). Weverify them now.

265

Page 282: CLP-1 Differential Calculus

DERIVATIVES 2.15 (OPTIONAL) — IS limx→c f ′(x) EQUAL TO f ′(c)?

Example 2.15.3

• We already know, by Lemma 2.8.1, that limh→0sinhh

= 1. So

limx→0

sinx2

x2= lim

h→0+

sinh

hwith h = x2

= 1

and

limx→0

f(x) = limx→0

sinx2

x= lim

x→0x

sinx2

x2= lim

x→0x lim

x→0

sinx2

x2= 0× 1 = 0

and f(x) is continuous at x = 0.

• The limit of the derivative is

limx→0

f ′(x) = limx→0

[2 cosx2 − sinx2

x2

]= 2× 1− 1 = 1

So, by Theorem 2.15.2, f(x) is differentiable at x = 0 and f ′(0) = 1.

266

Page 283: CLP-1 Differential Calculus

APPLICATIONS OF DERIVATIVES

Chapter 3

In Section 2.2 we defined the derivative at x = a, f ′(a), of an abstract function f(x),to be its instantaneous rate of change at x = a:

f ′(a) = limx→a

f(x)− f(a)

x− aThis abstract definition, and the whole theory that we have developed to deal with it,turns out be extremely useful simply because “instantaneous rate of change” appearsin a huge number of settings. Here are a few examples.

• If you are moving along a line and x(t) is your position on the line at timet, then your rate of change of position, x′(t), is your velocity. If, instead, v(t)is your velocity at time t, then your rate of change of velocity, v′(t), is youracceleration. We shall explore this further in Section 3.1.

• If P (t) is the size of some population (say the number of humans on the earth)at time t, then P ′(t) is the rate at which the size of that population is changing.It is called the net birth rate. We shall explore it further in Section 3.3.3.

• Radiocarbon dating, a procedure used to determine the age of, for example,archaeological materials, is based on an understanding of the rate at whichan unstable isotope of carbon decays. We shall look at this procedure in Sec-tion 3.3.1

• A capacitor is an electrical component that is used to repeatedly store andrelease electrical charge (say electrons) in an electronic circuit. If Q(t) is thecharge on a capacitor at time t, then Q′(t) is the instantaneous rate at whichcharge is flowing into the capacitor. That’s called the current. The standardunit of charge is the coulomb. One coulomb is the magnitude of the charge ofapproximately 6.241× 1018 electrons. The standard unit for current is the amp.One amp represents one coulomb per second.

267

Page 284: CLP-1 Differential Calculus

APPLICATIONS OF DERIVATIVES 3.1 VELOCITY AND ACCELERATION

3.1q Velocity and Acceleration

3.1.1 tt Velocity and Acceleration

If you are moving along the x–axis and your position at time t is x(t), then yourvelocity at time t is v(t) = x′(t) and your acceleration at time t is a(t) = v′(t) = x′′(t).

Example 3.1.1 Velocity as derivative of position.

Suppose that you are moving along the x–axis and that at time t your position isgiven by

x(t) = t3 − 3t+ 2.

We’re going to try and get a good picture of what your motion is like. We can learnquite a bit just by looking at the sign of the velocity v(t) = x′(t) at each time t.

• If x′(t) > 0, then at that instant x is increasing, i.e. you are moving to the right.

• If x′(t) = 0, then at that instant you are not moving at all.

• If x′(t) < 0, then at that instant x is decreasing, i.e. you are moving to the left.

From the given formula for x(t) it is straight forward to work out the velocity

v(t) = x′(t) = 3t2 − 3 = 3(t2 − 1) = 3(t+ 1)(t− 1)

This is zero only when t = −1 and when t = +1; at no other value a of t can thispolynomial be equal zero. Consequently in any time interval that does not includeeither t = −1 or t = +1, v(t) takes only a single sign b . So

• For all t < −1, both (t+1) and (t−1) are negative (sub in, for example, t = −10)so the product v(t) = x′(t) = 3(t+ 1)(t− 1) > 0.

• For all −1 < t < 1, the factor (t + 1) > 0 and the factor (t − 1) < 0 (sub in, forexample, t = 0) so the product v(t) = x′(t) = 3(t+ 1)(t− 1) < 0.

• For all t > 1, both (t+ 1) and (t− 1) are positive (sub in, for example, t = +10)so the product v(t) = x′(t) = 3(t+ 1)(t− 1) > 0.

The figure below gives a summary of the sign information we have about t− 1, t+ 1and x′(t).

268

Page 285: CLP-1 Differential Calculus

APPLICATIONS OF DERIVATIVES 3.1 VELOCITY AND ACCELERATION

It is now easy to put together a mental image of your trajectory.

• For t large and negative (i.e. far in the past), x(t) is large and negative and v(t)is large and positive. For example c , when t = −106, x(t) ≈ t3 = −1018 andv(t) ≈ 3t2 = 3 · 1012. So you are moving quickly to the right.

• For t < −1, v(t) = x′(t) > 0 so that x(t) is increasing and you are moving to theright.

• At t = −1, v(−1) = 0 and you have come to a halt at position x(−1) = (−1)3 −3(−1) + 2 = 4.

• For −1 < t < 1, v(t) = x′(t) < 0 so that x(t) is decreasing and you are movingto the left.

• At t = +1, v(1) = 0 and you have again come to a halt, but now at positionx(1) = 13 − 3 + 2 = 0.

• For t > 1, v(t) = x′(t) > 0 so that x(t) is increasing and you are again movingto the right.

• For t large and positive (i.e. in the far future), x(t) is large and positive andv(t) is large and positive. For example d , when t = 106, x(t) ≈ t3 = 1018 andv(t) ≈ 3t2 = 3 · 1012. So you are moving quickly to the right.

Here is a sketch of the graphs of x(t) and v(t). The heavy lines in the graphs indicatewhen you are moving to the right — that is where v(t) = x′(t) is positive.

269

Page 286: CLP-1 Differential Calculus

APPLICATIONS OF DERIVATIVES 3.1 VELOCITY AND ACCELERATION

Example 3.1.1

And here is a schematic picture of the whole trajectory.

a This is because the equation ab = 0 is only satisfied for real numbers a and b when either a = 0or b = 0 or both a = b = 0. Hence if a polynomial is the product of two (or more) factors, then itis only zero when at least one of those factors is zero. There are more complicated mathematicalenvironments in which you have what are called “zero divisors” but they are beyond the scopeof this course.

b This is because if v(ta) < 0 and v(tb) > 0 then, by the intermediate value theorem, the continuousfunction v(t) = x′(t) must take the value 0 for some t between ta and tb.

c Notice here we are using the fact that when t is very large t3 is much bigger than t2 and t1. So wecan approximate the value of the polynomial x(t) by the largest term — in this case t3. We cando similarly with v(t) — the largest term is 3t2.

d We are making a similar rough approximation here.

Example 3.1.2 Position and velocity from acceleration.

In this example we are going to figure out how far a body falling from rest will fallin a given time period.

• We should start by defining some variables and their units. Denote

time in seconds by t,

mass in kilograms by m,

distance fallen (in metres) at time t by s(t), velocity (in m/sec) by v(t) =s′(t) and acceleration (in m/sec2) by a(t) = v′(t) = s′′(t).

It makes sense to choose a coordinate system so that the body starts to fall att = 0.

• We will use Newton’s second law of motion

the force applied to the body at time t = m · a(t).

together with the assumption that the only force acting on the body is gravity(in particular, no air resistance). Note that near the surface of the Earth,

the force due to gravity acting on a body of mass m = m · g.

The constant g, called the acceleration of gravity a , is about 9.8m/sec2.

270

Page 287: CLP-1 Differential Calculus

APPLICATIONS OF DERIVATIVES 3.1 VELOCITY AND ACCELERATION

• Since the body is falling from rest, we know that its initial velocity is zero. Thatis

v(0) = 0.

Newton’s second law then implies that

m · a(t) = force due to gravitym · v′(t) = m · g cancel the m

v′(t) = g

• In order to find the velocity, we need to find a function of t whose derivative isconstant. We are simply going to guess such a function and then we will verifythat our guess has all of the desired properties. It’s easy to guess a functionwhose derivative is the constant g. Certainly gt has the correct derivative. Sodoes

v(t) = gt+ c

for any constant c. One can then verify b that v′(t) = g. Using the fact thatv(0) = 0 we must then have c = 0 and so

v(t) = gt.

• Since velocity is the derivative of position, we know that

s′(t) = v(t) = g · t.

To find s(t) we are again going to guess and check. It’s not hard to see that wecan use

s(t) =g

2t2 + c

where again c is some constant. Again we can verify that this works simply bydifferentiating c. Since we have defined s(t) to be the distance fallen, it followsthat s(0) = 0 which in turn tells us that c = 0. Hence

s(t) =g

2t2 but g = 9.8, so

= 4.9t2,

which is exactly the s(t) used way back in Section 1.2.

a It is also called the standard acceleration due to gravity or standard gravity. For those of youwho prefer imperial units (or US customary units), it is about 32 ft/sec2, 77165 cubits/minute2,or 631353 furlongs/hour2.

271

Page 288: CLP-1 Differential Calculus

APPLICATIONS OF DERIVATIVES 3.1 VELOCITY AND ACCELERATION

Example 3.1.2

b While it is clear that this satisfies the equation we want, it is less clear that it is the only functionthat works. To see this, assume that there are two functions f(t) and h(t) which both satisfyv′(t) = g. Then f ′(t) = h′(t) = g and so f ′(t) − h′(t) = 0. Equivalently d

dt (f(t)− h(t)) = 0. Theonly function whose derivative is zero everywhere is the constant function (see Section 2.13 andTheorem 2.13.12). Thus f(t)− h(t) = constant. So all the functions that satisfy v′(t) = g must beof the form gt+ constant.

c To show that any solution of s′(t) = gv must be of this form we can use the same reasoning weused to get v(t) = gt+ constant.

Let’s now do a similar but more complicated example.

Example 3.1.3 Stopping distance of a braking car.

A car’s brakes can decelerate the car at 64000km/hr2. How fast can the car be drivenif it must be able to stop within a distance of 50m?Solution Before getting started, notice that there is a small “trick” in this problem —several quantities are stated but their units are different. The acceleration is stated inkilometres per hour2, but the distance is stated in metres. Whenever we come acrossa “real world” problem a we should be careful of the units used.

• We should first define some variables and their units. Denote

time (in hours) by t, the position of the car (in kilometres) at time t by x(t), and the velocity (in kilometres per hour) by is v(t).

We can also choose a coordinate system such that x(0) = 0 and the car startsbraking at time t = 0.

• Now let us rewrite the information in the problem in terms of these variables.

We are told that, at maximum braking, the acceleration v′(t) = x′′(t) of thecar is −64000. We need to determine the maximum initial velocity v(0) so that the stop-

ping distance is at most 50m = 0.05km (being careful with our units). Letus call the stopping distance xstop which is really x(tstop) where tstop is thestopping time.

• In order to determine xstop we first need to determine tstop, which we will doby assuming maximum braking from a, yet to be determined, initial velocityof v(0) = q m/sec.

• Assuming that the car undergoes a constant acceleration at this maximumbraking power, we have

v′(t) = −64000

272

Page 289: CLP-1 Differential Calculus

APPLICATIONS OF DERIVATIVES 3.1 VELOCITY AND ACCELERATION

This equation is very similar to the ones we had to solve in Example 3.1.2 justabove.

As we did there b , we are going to just guess v(t). First, we just guess onefunction whose derivative is −64000, namely −64000t. Next we observe that,since the derivative of a constant is zero, any function of the form

v(t) = −64000 t+ c

with constant c, has the correct derivative. Finally, the requirement that theinitial velocity v(0) = q" forces c = q, so

v(t) = q − 64000 t

• From this we can easily determine the stopping time tstop, when the initial ve-locity is q, since this is just when v(t) = 0:

0 = v(tstop) = q − 64000 · tstop and so

tstop =q

64000.

• Armed with the stopping time, how do we get at the stopping distance? Weneed to find the formula satisfied by x(t). Again (as per Example 3.1.2) wemake use of the fact that

x′(t) = v(t) = q − 64000t.

So we need to guess a function x(t) so that x′(t) = q − 64000t. It is not hard tosee that

x(t) = qt− 32000t2 + constant

works. Since we know that x(0) = 0, this constant is just zero and

x(t) = qt− 32000t2.

• We are now ready to compute the stopping distance (in terms of the, still yet tobe determined, initial velocity q):

xstop = x(tstop) = qtstop − 32000t2stop

=q2

64000− 32000q2

640002

=q2

64000

(1− 1

2

)=

q2

2× 64000

Notice that the stopping distance is a quadratic function of the initial velocity— if you go twice as fast, you need four times the distance to stop.

273

Page 290: CLP-1 Differential Calculus

APPLICATIONS OF DERIVATIVES 3.1 VELOCITY AND ACCELERATION

Example 3.1.3

• But we are told that the stopping distance must be less than 50m = 0.05km.This means that

xstop =q2

2× 64000≤ 5

100

q2 ≤ 2× 64000× 5

100=

64000× 10

100= 6400

Thus we must have q ≤ 80. Hence the initial velocity can be no greater than80km/h.

a Well — “realer world” would perhaps be a betterer term.b Now is a good time to go back and have a read of that example.

3.1.2 tt Exercises

Exercises — Stage 1

1. Suppose you throw a ball straight up in the air, and its height from t = 0to t = 4 is given by h(t) = −4.9t2 + 19.6t. True or false: at time t = 2, theacceleration of the ball is 0.

2. Suppose an object is moving with a constant acceleration. It takes tenseconds to accelerate from 1 m

sto 2 m

s. How long does it take to accel-

erate from 2 ms

to 3 ms

? How long does it take to accelerate from 3 ms

to13 m

s?

3. Let s(t) be the position of a particle at time t. True or false: if s′′(a) > 0for some a, then the particle’s speed is increasing when t = a.

4. Let s(t) be the position of a particle at time t. True or false: if s′(a) > 0 ands′′(a) > 0 for some a, then the particle’s speed is increasing when t = a.

Exercises — Stage 2 For this section, we will ask you a number of questions thathave to do with objects falling on Earth. Unless otherwise stated, you should assumethat an object falling through the air has an acceleration due to gravity of 9.8 metersper second per second.

5. A flower pot rolls out of a window 10m above the ground. How fast is itfalling just as it smacks into the ground?

274

Page 291: CLP-1 Differential Calculus

APPLICATIONS OF DERIVATIVES 3.1 VELOCITY AND ACCELERATION

6. You want to know how deep a well is, so you drop a stone down andcount the seconds until you hear it hit bottom.

a If the stone took x seconds to hit bottom, how deep is the well?

b Suppose you think you dropped the stone down the well, but actu-ally you tossed it down, so instead of an initial velocity of 0 metresper second, you accidentally imparted an initial speed of 1 metresper second. What is the actual depth of the well, if the stone fellfor x seconds?

7. You toss a key to your friend, standing two metres away. The keys initiallymove towards your friend at 2 metres per second, but slow at a rate of 0.25metres per second per second. How much time does your friend have toreact to catch the keys? That is–how long are the keys flying before theyreach your friend?

8. A car is driving at 100 kph, and it brakes with a deceleration of 50000 kmhr2

.How long does the car take to come to a complete stop?

9. You are driving at 120 kph, and need to stop in 100 metres. How muchdeceleration do your brakes need to provide? You may assume thebrakes cause a constant deceleration.

10. You are driving at 100 kph, and apply the brakes steadily, so that your cardecelerates at a constant rate and comes to a stop in exactly 7 seconds. Whatwas your speed one second before you stopped?

11. About 8.5 minutes after liftoff, the US space shuttle has reached orbital ve-locity, 17 500 miles per hour. Assuming its acceleration was constant, howfar did it travel in those 8.5 minutes?Source: http://www.nasa.gov/mission_pages/shuttle/

shuttlemissions/sts121/launch/qa-leinbach.html

12. A pitching machine has a dial to adjust the speed of the pitch. You rotate itso that it pitches the ball straight up in the air. How fast should the ball exitthe machine, in order to stay in the air exactly 10 seconds?You may assume that the ball exits from ground level, and is acted on onlyby gravity, which causes a constant deceleration of 9.8 metres per second.

13. A peregrine falcon can dive at a speed of 325 kph. If you were to drop astone, how high up would you have to be so that the stone reached thesame speed in its fall?

14. You shoot a cannon ball into the air with initial velocity v0, and thengravity brings it back down (neglecting all other forces). If the cannonball made it to a height of 100m, what was v0?

275

Page 292: CLP-1 Differential Calculus

APPLICATIONS OF DERIVATIVES 3.2 RELATED RATES

15. Suppose you are driving at 120 kph, and you start to brake at a decelerationof 50000 kph per hour. For three seconds you steadily increase your decel-eration to 60000 kph per hour. (That is, for three seconds, the rate of changeof your deceleration is constant.) How fast are you driving at the end ofthose three seconds?

Exercises — Stage 3

16. You jump up from the side of a trampoline with an initial upward ve-locity of 1 metre per second. While you are in the air, your decelerationis a constant 9.8 metres per second per second due to gravity. Once youhit the trampoline, as you fall your speed decreases by 4.9 metres persecond per second. How many seconds pass between the peak of yourjump and the lowest part of your fall on the trampoline?

17. Suppose an object is moving so that its velocity doubles every second. Givean expression for the acceleration of the object.

3.2q Related Rates

3.2.1 tt Related Rates

Consider the following problem

A spherical balloon is being inflated at a rate of 13cm3/sec. How fast isthe radius changing when the balloon has radius 15cm?

There are several pieces of information in the statement:

276

Page 293: CLP-1 Differential Calculus

APPLICATIONS OF DERIVATIVES 3.2 RELATED RATES

• The balloon is spherical

• The volume is changing at a rate of 13cm3/sec — so we need variables forvolume (in cm3) and time (in sec). Good choices are V and t.

• We are asked for the rate at which the radius is changing — so we need avariable for radius and units. A good choice is r, measured in cm — sincevolume is measured in cm3.

Since the balloon is a sphere we know 1 that

V =4

3πr3

Since both the volume and radius are changing with time, both V and r are implicitlyfunctions of time; we could really write

V (t) =4

3πr(t)3.

We are told the rate at which the volume is changing and we need to find the rate atwhich the radius is changing. That is, from a knowledge of dV

dt, find the related rate

2 drdt

.In this case, we can just differentiate our equation by t to get

dV

dt= 4πr2 dr

dt

This can then be rearranged to give

dr

dt=

1

4πr2

dV

dt.

Now we were told that dVdt

= 13, so

dr

dt=

13

4πr2.

We were also told that the radius is 15cm, so at that moment in time

dr

dt=

13

π4× 152.

This is a very typical example of a related rate problem. This section is really justa collection of problems, but all will follow a similar pattern.

• The statement of the problem will tell you quantities that must be related (aboveit was volume, radius and, implicitly, time).

1 If you don’t know the formula for the volume of a sphere, now is a good time to revise by lookingat Appendix A.11.

2 Related rate problems are problems in which you are given the rate of change of one quantityand are to determine the rate of change of another, related, quantity.

277

Page 294: CLP-1 Differential Calculus

APPLICATIONS OF DERIVATIVES 3.2 RELATED RATES

• Typically a little geometry (or some physics or. . . ) will allow you to relate thesequantities (above it was the formula that links the volume of a sphere to itsradius).

• Implicit differentiation will then allow you to link the rate of change of onequantity to another.

Another balloon example

Example 3.2.1 A rising balloon.

Consider a helium balloon rising vertically from a fixed point 200m away from you.You are trying to work out how fast it is rising. Now — computing the velocitydirectly is difficult, but you can measure angles. You observe that when it is at anangle of π/4 its angle is changing by 0.05 radians per second.

• Start by drawing a picture with the relevant variables

• So denote the angle to be θ (in radians), the height of the balloon (in m) by hand time (in seconds) by t. Then trigonometry tells us

h = 200 · tan θ

• Differentiating allows us to relate the rates of change

dh

dt= 200 sec2 θ · dθ

dt

• We are told that when θ = π/4 we observe dθdt

= 0.05, so

dh

dt= 200 · sec2(π/4) · 0.05

278

Page 295: CLP-1 Differential Calculus

APPLICATIONS OF DERIVATIVES 3.2 RELATED RATES

Example 3.2.1

= 200 · 0.05 ·(√

2)2

= 200 · 5

100· 2

= 20m/s

• So the balloon is rising at a rate of 20m/s.

The following problem is perhaps the classic related rate problem.

Example 3.2.2 A sliding ladder.

A 5m ladder is leaning against a wall. The floor is quite slippery and the base of theladder slides out from the wall at a rate of 1m/s. How fast is the top of the laddersliding down the wall when the base of the ladder is 3m from the wall?

• A good first step is to draw a picture stating all relevant quantities. This willalso help us define variables and units.

• So now define x(t) to be the distance between the bottom of the ladder and thewall, at time t, and let y(t) be the distance between the top of the ladder andthe ground at time t. Measure time in seconds, but both distances in meters.

• We can relate the quantities using Pythagoras:

x2 + y2 = 52

279

Page 296: CLP-1 Differential Calculus

APPLICATIONS OF DERIVATIVES 3.2 RELATED RATES

Example 3.2.2

• Differentiating with respect to time then gives

2xdx

dt+ 2y

dy

dt= 0

• We know that dxdt

= 1 and x = 3, so

6 · 1 + 2ydy

dt= 0

but we need to determine y before we can go further. Thankfully we know thatx2 + y2 = 25 and x = 3, so y2 = 25− 9 = 16 and a so y = 4.

• So finally putting everything together

6 · 1 + 8dy

dt= 0

dy

dt= −3

4m/s.

Thus the top of the ladder is sliding towards the floor at a rate of 34m/s.

a Since the ladder isn’t buried in the ground, we can discard the solution y = −4.

The next example is complicated by the rates of change being stated not just as“the rate of change per unit time” but instead being stated as “the percentage rate ofchange per unit time”. If a quantity f is changing with rate df

dt, then we can say that

f is changing at a rate of 100 ·dfdt

fpercent.

Thus if, at time t, f has rate of change r%, then

100f ′(t)

f(t)= r =⇒ f ′(t) =

r

100f(t)

so that if h is a very small time increment

f(t+ h)− f(t)

h≈ r

100f(t) =⇒ f(t+ h) ≈ f(t) +

rh

100f(t)

That is, over a very small time interval h, f increases by the fraction rh100

of its valueat time t.

So armed with this, let’s look at the problem.

280

Page 297: CLP-1 Differential Calculus

APPLICATIONS OF DERIVATIVES 3.2 RELATED RATES

Example 3.2.3 Percentage rate of change of R = PQ.

The quantities P, Q and R are functions of time and are related by the equationR = PQ. Assume that P is increasing instantaneously at the rate of 8% per year(meaning that 100P

P= 8) and that Q is decreasing instantaneously at the rate of 2%

per year (meaning that 100Q′

Q= −2). Determine the percentage rate of change for R.

Solution This one is a little different — we are given the variables and the formula,so no picture drawing or defining required. Though we do need to define a timevariable — let t denote time in years.

• Since R(t) = P (t) ·Q(t) we can differentiate with respect to t to get

dR

dt= PQ′ +QP ′

• But we need the percentage change in R, namely

100R′

R= 100

PQ′ +QP ′

R

but R = PQ, so rewrite it as

= 100PQ′ +QP ′

PQ

= 100PQ′

PQ+ 100

QP ′

PQ

= 100Q′

Q+ 100

P ′

P

so we have stated the instantaneous percentage rate of change in R as the sumof the percentage rate of change in P and Q.

• We know the percentage rate of change of P and Q, so

100R′

R= −2 + 8 = 6

That is, the instantaneous percentage rate of change of R is 6% per year.

Yet another falling object example.

Example 3.2.4 The shadow of a falling ball.

A ball is dropped from a height of 49m above level ground. The height of the ballat time t is h(t) = 49 − 4.9t2 m. A light, which is also 49m above the ground, is 10mto the left of the ball’s original position. As the ball descends, the shadow of the ball

281

Page 298: CLP-1 Differential Calculus

APPLICATIONS OF DERIVATIVES 3.2 RELATED RATES

Example 3.2.4

caused by the light moves across the ground. How fast is the shadow moving onesecond after the ball is dropped?Solution There is quite a bit going on in this example, so read carefully.

• First a diagram; the one below is perhaps a bit over the top.

• Let’s call s(t) the distance from the shadow to the point on the ground directlyunderneath the ball.

• By similar triangles we see that

4.9t2

10=

49− 4.9t2

s(t)

We can then solve for s(t) by just multiplying both sides by 104.9t2

s(t). This gives

s(t) = 1049− 4.9t2

4.9t2=

100

t2− 10

• Differentiating with respect to t will then give us the rates,

s′(t) = −2100

t3

• So, at t = 1, s′(1) = −200m/sec. That is, the shadow is moving to the left at200m/sec.

282

Page 299: CLP-1 Differential Calculus

APPLICATIONS OF DERIVATIVES 3.2 RELATED RATES

A more nautical example.

Example 3.2.5 The distance between moving boats.

Two boats spot each other in the ocean at midday — Boat A is 15km west of Boat B.Boat A is travelling east at 3km/h and boat B is travelling north at 4km/h. At 3pmhow fast is the distance between the boats changing.

• First we draw a picture.

• Let x(t) be the distance at time t, in km, from boat A to the original position ofboat B (i.e. to the position of boat B at noon). And let y(t) be the distance attime t, in km, of boat B from its original position. And let z(t) be the distancebetween the two boats at time t.

• Additionally we are told that x′ = −3 and y′ = 4 — notice that x′ < 0 sincethat distance is getting smaller with time, while y′ > 0 since that distance isincreasing with time.

• Further at 3pm boat A has travelled 9km towards the original position of boatB, so x = 15 − 9 = 6, while boat B has travelled 12km away from its originalposition, so y = 12.

• The distances x, y and z form a right-angled triangle, and Pythagoras tells usthat

z2 = x2 + y2.

At 3pm we know x = 6, y = 12 so

z2 = 36 + 144 = 180

z =√

180 = 6√

5.

283

Page 300: CLP-1 Differential Calculus

APPLICATIONS OF DERIVATIVES 3.2 RELATED RATES

Example 3.2.5

• Differentiating then gives

2zdz

dt= 2x

dx

dt+ 2y

dy

dt= 12 · (−3) + 24 · (4)

= 60.

Dividing through by 2z = 12√

5 then gives

dz

dt=

60

12√

5=

5√5

=√

5

So the distance between the boats is increasing at√

5km/h.

One last one before we move on to another topic.

Example 3.2.6 Fuel level in a cylindrical tank.

Consider a cylindrical fuel tank of radius r and length L (in some appropriate units)that is lying on its side. Suppose that fuel is being pumped into the tank at a rate q.At what rate is the fuel level rising?

Solution If the tank were vertical everything would be much easier. Unfortunatelythe tank is on its side, so we are going to have to work a bit harder to establish therelation between the depth and volume. Also notice that we have not been suppliedwith units for this problem — so we do not need to state the units of our variables.

• Again — draw a picture. Here is an end view of the tank; the shaded part ofthe circle is filled with fuel.

284

Page 301: CLP-1 Differential Calculus

APPLICATIONS OF DERIVATIVES 3.2 RELATED RATES

• Let us denote by V (t) the volume of fuel in the tank at time t and by h(t) thefuel level at time t.

• We have been told that V ′(t) = q and have been asked to determine h′(t). Whileit is possible to do so by finding a formula relating V (t) and h(t), it turns out tobe quite a bit easier to first find a formula relating V and the angle θ shown inthe end view. We can then translate this back into a formula in terms of h usingthe relation

h(t) = r − r cos θ(t).

Once we know θ′(t), we can easily obtain h′(t) by differentiating the aboveequation.

• The computation that follows below gets a little involved in places, so we willdrop the “(t)” on the variables V, h and θ. The reader must never forget thatthese three quantities are really functions of time, while r and L are constantsthat do not depend on time.

• The volume of fuel is L times the cross–sectional area filled by the fuel. That is,

V =L× Area of

While we do not have a canned formula for the area of a chord of a circle likethis, it is easy to express the area of the chord in terms of two areas that we cancompute.

V = L×Area of

= L×[

Area of- Area of

]

285

Page 302: CLP-1 Differential Calculus

APPLICATIONS OF DERIVATIVES 3.2 RELATED RATES

• The piece of pie

is the fraction 2θ2π

of the full circle, so its area is 2θ2ππr2 = θr2.

• The triangle

has height r cos θ and base 2r sin θ and hence has area 12(r cos θ)(2r sin θ) =

r2 sin θ cos θ = r2

2sin(2θ), where we have used a double-angle formula (see Ap-

pendix A.14).

• Subbing these two areas into the above expression for V gives

V = L×[θr2 − r2

2sin 2θ

]=Lr2

2

[2θ − sin 2θ

]Oof!

• Now we can differentiate to find the rate of change. Recalling that V = V (t)and θ = θ(t), while r and L are constants,

V ′ =Lr2

2[2θ′ − 2 cos 2θ · θ′]

= Lr2 · θ′ · [1− cos 2θ]

Solving this for θ′ and using V ′ = q gives

θ′ =q

Lr2(1− cos 2θ)

This is the rate at which θ is changing, but we need the rate at which h ischanging. We get this from

h = r − r cos θ differentiating this givesh′ = r sin θ · θ′

Substituting our expression for θ′ into the expression for h′ gives

h′ = r sin θ · q

Lr2(1− cos 2θ)

286

Page 303: CLP-1 Differential Calculus

APPLICATIONS OF DERIVATIVES 3.2 RELATED RATES

Example 3.2.6

• We can clean this up a bit more — recall more double-angle formulas a

h′ = r sin θ · q

Lr2(1− cos 2θ)substitute cos 2θ = 1− 2 sin2 θ

= r sin θ · q

Lr2 · 2 sin2 θnow cancel r’s and a sin θ

=q

2Lr sin θ

• But we can clean this up even more — instead of writing this rate in terms of θit is more natural to write it in terms of h (since the initial problem is stated interms of h). From the triangle

and Pythagoras we have

sin θ =

√r2 − (r − h)2

r=

√2rh− h2

r

and hence

h′ =q

2L√

2rh− h2.

• As a check, notice that h′ becomes undefined when h < 0 and also when h > 2r,because then the argument of the square root in the denominator is negative.Both make sense — the fuel level in the tank must obey 0 ≤ h ≤ 2r.

a Take another look at Appendix A.14.

3.2.2 tt Exercises

Exercises — Stage 1

1. Suppose the quantities P and Q are related by the formula P = Q3.P and Q are changing with respect to time, t. Given this information,which of the following are problems you could solve?

287

Page 304: CLP-1 Differential Calculus

APPLICATIONS OF DERIVATIVES 3.2 RELATED RATES

i GivendP

dt(0), find

dQ

dt(0). (Remember: the notation

dP

dt(0) means

the derivative of P with respect to t at the time t = 0.)

ii GivendP

dt(0) and the value of Q when t = 0, find

dQ

dt(0).

iii GivendQ

dt(0), find

dP

dt(0).

iv GivendQ

dt(0) and the value of P when t = 0, find

dP

dt(0).

Exercises — Stage 2 For problems 3.2.2.2 through 3.2.2.4, the relationship betweenseveral variables is explicitly given. Use this information to relate their rates ofchange.For Questions 3.2.2.5 through 3.2.2.9, look for a way to use the PythagoreanTheorem.For Questions 3.2.2.10 through 3.2.2.14, look for tricks from trigonome-try.For Questions 3.2.2.15 through 3.2.2.20, you’ll need to know formulas for volumeor area.

2. *. A point is moving on the unit circle (x, y) : x2 + y2 = 1 in the xy–plane. At (2/

√5, 1/√

5), its y–coordinate is increasing at rate 3. What isthe rate of change of its x–coordinate?

3. *. The quantities P, Q and R are functions of time and are related by theequation R = PQ. Assume that P is increasing instantaneously at the rateof 8% per year and that Q is decreasing instantaneously at the rate of 2%

per year. That is,P ′

P= 0.08 and

Q′

Q= −0.02. Determine the percentage rate

of change for R.

4. *. Three quantities, F , P and Q all depend upon time t and are related bythe equation

F =P

Q

a Assume that at a particular moment in time P = 25 and P is increasingat the instantaneous rate of 5 units/min. At the same moment, Q = 5and Q is increasing at the instantaneous rate of 1 unit/min. What isthe instantaneous rate of change in F at this moment?

b Assume that at another moment in time P is increasing at the instan-taneous rate of 10% and Q is decreasing at the instantaneous rate 5%.What can you conclude about the rate of change of F at this moment?

5. *. Two particles move in the Cartesian plane. Particle A travels on the x-axis starting at (10, 0) and moving towards the origin with a speed of 2 unitsper second. Particle B travels on the y-axis starting at (0, 12) and movingtowards the origin with a speed of 3 units per second. What is the rate of

288

Page 305: CLP-1 Differential Calculus

APPLICATIONS OF DERIVATIVES 3.2 RELATED RATES

change of the distance between the two particles when particle A reachesthe point (4, 0)?

6. *. Two particles A and B are placed on the Cartesian plane at (0, 0) and(3, 0) respectively. At time 0, both start to move in the +y direction. ParticleA moves at 3 units per second, while B moves at 2 units per second. Howfast is the distance between the particles changing when particle A is at adistance of 5 units from B.

7. *. Ship A is 400 miles directly south of Hawaii and is sailing south at20 miles/hour. Ship B is 300 miles directly east of Hawaii and is sailingwest at 15 miles/hour. At what rate is the distance between the shipschanging?

8. *. Two tall sticks are vertically planted into the ground, separated by adistance of 30 cm. We simultaneously put two snails at the base of eachstick. The two snails then begin to climb their respective sticks. The firstsnail is moving with a speed of 25 cm per minute, while the second snailis moving with a speed of 15 cm per minute. What is the rate of changeof the distance between the two snails when the first snail reaches 100cm above the ground?

9. *. A 20m long extension ladder leaning against a wall starts collapsing inon itself at a rate of 2m/s, while the foot of the ladder remains a constant5m from the wall. How fast is the ladder moving down the wall after 3.5seconds?

10. A watering trough has a cross section shaped like an isosceles trapezoid.The trough is 2 metres long, 50 cm high, 1 metre wide at the top, and 60cm wide at the bottom.

60 cm

1 m

50 cm

2 m

A pig is drinking water from the trough at a rate of 3 litres per minute.When the height of the water is 25 cm, how fast is the height decreasing?

11. A tank is 5 metres long, and has a trapezoidal cross section with the dimen-sions shown below.

289

Page 306: CLP-1 Differential Calculus

APPLICATIONS OF DERIVATIVES 3.2 RELATED RATES

1.25 m

3 m3 m

1 m

5 m

A hose is filling the tank up at a rate of one litre per second. How fast is theheight of the water increasing when the water is 10 centimetres deep?

12. A rocket is blasting off, 2 kilometres away from you. You and the rocketstart at the same height. The height of the rocket in kilometres, t hours afterliftoff, is given by

h(t) = 61750t2

How fast (in radians per second) is your line of sight rotating to keep look-ing at the rocket, one minute after liftoff?

13. *. A high speed train is traveling at 2 km/min along a straight track.The train is moving away from a movie camera which is located 0.5 kmfrom the track.

a How fast is the distance between the train and the camera increas-ing when they are 1.3 km apart?

b Assuming that the camera is always pointed at the train, how fast(in radians per min) is the camera rotating when the train and thecamera are 1.3 km apart?

14. A clock has a minute hand that is 10 cm long, and an hour hand that is 5 cmlong. Let D be the distance between the tips of the two hands. How fast isD decreasing at 4:00?

D

290

Page 307: CLP-1 Differential Calculus

APPLICATIONS OF DERIVATIVES 3.2 RELATED RATES

15. *. Find the rate of change of the area of the annulus (x, y) : r2 ≤ x2 + y2 ≤R2. (i.e. the points inside the circle of radius R but outside the circle of

radius r) if R = 3 cm, r = 1 cm,dR

dt= 2

cm

s, and

dr

dt= 7

cm

s.

R

r

16. Two spheres are centred at the same point. The radius R of the biggersphere at time t is given by R(t) = 10 + 2t, while the radius r of the smallersphere is given by r(t) = 6t, t ≥ 0. How fast is the volume between thespheres (inside the big sphere and outside the small sphere) changing whenthe bigger sphere has a radius twice as large as the smaller?

17. You attach two sticks together at their ends, and stick the other ends inthe mud. One stick is 150 cm long, and the other is 200 cm.

150 cm 200 cm1.4 m

The structure starts out being 1.4 metres high at its peak, but the sticksslide, and the height decreases at a constant rate of three centimetresper minute. How quickly is the area of the triangle (formed by the twosticks and the level ground) changing when the height of the structureis 120 cm?

18. The circular lid of a salt shaker has radius 8. There is a cut-out to allow thesalt to pour out of the lid, and a door that rotates around to cover the cut-out. The door is a quarter-circle of radius 7 cm. The cut-out has the shapeof a quarter-annulus with outer radius 6 cm and inner radius 1 cm. If theuncovered area of the cut-out is A cm2, then the salt flows out at 1

5A cm3 per

second.

291

Page 308: CLP-1 Differential Calculus

APPLICATIONS OF DERIVATIVES 3.2 RELATED RATES

salt shaker lidcut-out uncovered

salt shaker lidcut-out partially covered

Recall: an annulus is the set of points inside one circle and outside another,like a flat doughnut (see Question 3.2.2.15).

annulusquarter annulus

While pouring out salt, you spin the door around the lid at a constant rate ofπ6

radians per second, covering more and more of the cut-out. When exactlyhalf of the cut-out is covered, how fast is the flow of salt changing?

19. A cylindrical sewer pipe with radius 1 metre has a vertical rectangulardoor that slides in front of it to block the flow of water, as shown below.If the uncovered area of the pipe isAm2, then the flow of water throughthe pipe is 1

5A cubic metres per second.

The door slides over the pipe, moving vertically at a rate of 1 centime-tre per second. How fast is the flow of water changing when the doorcovers the top 25 centimetres of the pipe?

292

Page 309: CLP-1 Differential Calculus

APPLICATIONS OF DERIVATIVES 3.2 RELATED RATES

20. A martini glass is shaped like a cone, with top diameter 10 cm and sidelength 10 cm.

10

1010

When the liquid in the glass is 7 cm high, it is evaporating at a rate of 5mL per minute. How fast is the height of the liquid decreasing?

Exercises — Stage 3

21. A floating buoy is anchored to the bottom of a river. As the river flows,the buoy is pulled in the direction of flow until its 2-metre rope is taut.A sensor at the anchor reads the angle θ between the rope and theriverbed, as shown in the diagram below. This data is used to measurethe depth D of water in the river, which depends on time.

293

Page 310: CLP-1 Differential Calculus

APPLICATIONS OF DERIVATIVES 3.2 RELATED RATES

θ

D

a If θ =π

4and

dt= 0.25

rad

hr, how fast is the depth D of the water

changing?

b A measurement showsdθ

dt= 0, but

dD

dt6= 0. Under what circum-

stances does this occur?

c A measurement showsdθ

dt> 0, but

dD

dt< 0. Under what circum-

stances does this occur?

22. A point is moving in the xy-plane along the quadrilateral shown below.

x

y

1

1

a When the point is at (0,−2), it is moving to the right. An observerstationed at the origin must turn at a rate of one radian per second tokeep looking directly at the point. How fast is the point moving?

b When the point is at (0, 2), its x-coordinate is increasing at a rate ofone unit per second. How fast it its y-coordinate changing? How fastis the point moving?

23. You have a cylindrical water bottle 20 cm high, filled with water. Its crosssection is a circle of radius 5. You slowly smoosh the sides, so the cross

294

Page 311: CLP-1 Differential Calculus

APPLICATIONS OF DERIVATIVES 3.2 RELATED RATES

section becomes an ellipse with major axis (widest part) 2a and minor axis(skinniest part) 2b.

5

5

20

ab

20

After t seconds of smooshing the bottle, a = 5 + t cm. The perimeter of thecross section is unchanged as the bottle deforms. The perimeter of an ellipseis actually quite difficult to calculate, but we will use an approximationderived by Ramanujan and assume that the perimeter p of our ellipse is

p ≈ π[3(a+ b)−

√(a+ 3b)(3a+ b)

].

The area of an ellipse is πab.

a Give an equation that relates a and b (and no other variables).

b Give an expression for the volume of the bottle as it is beingsmooshed, in terms of a and b (and no other variables).

c Suppose the bottle was full when its cross section was a circle. Howfast is the water spilling out when a is twice as big as b?

24. The quantities A, B, C, and D all depend on time, and are related by theformula

AB = log(C2 +D2 + 1

).

At time t = 10, the following values are known:

• A = 0

•dA

dt= 2 units per second

What is B when t = 10?

295

Page 312: CLP-1 Differential Calculus

APPLICATIONS OF DERIVATIVES 3.3 EXPONENTIAL GROWTH AND DECAY

3.3q Exponential Growth and Decay — a First Look at Dif-ferential Equations

A differential equation is an equation for an unknown function that involves thederivative of the unknown function. For example, Newton’s law of cooling says:

The rate of change of temperature of an object is proportional to the dif-ference in temperature between the object and its surroundings.

We can write this more mathematically using a differential equation — an equationfor the unknown function T (t) that also involves its derivative dT

dt(t). If we denote by

T (t) the temperature of the object at time t and by A the temperature of its surround-ings, Newton’s law of cooling says that there is some constant of proportionality, K,such that

dT

dt(t) = K

[T (t)− A

]Differential equations play a central role in modelling a huge number of different

phenomena, including the motion of particles, electromagnetic radiation, financialoptions, ecosystem populations and nerve action potentials. Most universities offerhalf a dozen different undergraduate courses on various aspects of differential equa-tions. We are barely going to scratch the surface of the subject. At this point we aregoing to restrict ourselves to a few very simple differential equations for which wecan just guess the solution. In particular, we shall learn how to solve systems obey-ing Newton’s law of cooling in Section 3.3.2, below. But first, here is another slightlysimpler example.

3.3.1 tt Carbon Dating

Scientists can determine the age of objects containing organic material by a methodcalled carbon dating or radiocarbon dating 1. Cosmic rays hitting the atmosphere con-vert nitrogen into a radioactive isotope of carbon, 14C, with a half–life of about 5730years 2. Vegetation absorbs carbon dioxide from the atmosphere through photosyn-thesis and animals acquire 14C by eating plants. When a plant or animal dies, it stopsreplacing its carbon and the amount of 14C begins to decrease through radioactivedecay. More precisely, let Q(t) denote the amount of 14C in the plant or animal tyears after it dies. The number of radioactive decays per unit time, at time t, is pro-portional to the amount of 14C present at time t, which is Q(t). Thus

1 Willard Libby, of Chicago University was awarded the Nobel Prize in Chemistry in 1960, fordeveloping radiocarbon dating.

2 A good question to ask yourself is “How can a scientist (who presumably doesn’t live 60 cen-turies) measure this quantity?” One way exploits the little piece of calculus we are about todiscuss.

296

Page 313: CLP-1 Differential Calculus

APPLICATIONS OF DERIVATIVES 3.3 EXPONENTIAL GROWTH AND DECAY

Equation 3.3.1 Radioactive decay.

dQ

dt(t) = −kQ(t)

Here k is a constant of proportionality that is determined by the half–life. Weshall explain what half-life is and also determine the value of k in Example 3.3.3,below. Before we do so, let’s think about the sign in equation 3.3.1.

• Recall that Q(t) denotes a quantity, namely the amount of 14C present at timet. There cannot be a negative amount of 14C, nor can this quantity be zero(otherwise we wouldn’t use carbon dating, so we must have Q(t) > 0.

• As the time t increases, Q(t) decreases, because 14C is being continuously con-verted into 14N by radioactive decay 3. Thus dQ

dt(t) < 0.

• The signs Q(t) > 0 and dQdt

(t) < 0 are consistent with equation 3.3.1 providedthe constant of proportionality k > 0.

• In equation 3.3.1, we chose to call the constant of proportionality “−k”. We didso in order to make k > 0. We could just as well have chosen to call the con-stant of proportionality “K”. That is, we could have replaced equation 3.3.1 bydQdt

(t) = KQ(t). The constant of proportionality K would have to be negative,(and K and k would be related by K = −k).

Now, let’s guess some solutions to equation 3.3.1. We wish to guess a functionQ(t) whose derivative is just a constant times itself. Here is a short table of deriva-tives. It is certainly not complete, but it contains the most important derivatives thatwe know.

F (t) 1 ta sin t cos t tan t et log t arcsin t arctan tddtF (t) 0 ata−1 cos t − sin t sec2 t et 1

t1√

1−t21

1+t2

There is exactly one function in this table whose derivative is just a (nonzero)constant times itself. Namely, the derivative of et is exactly et = 1×et. This is almost,but not quite what we want. We want the derivative of Q(t) to be the constant −k(rather than the constant 1) times Q(t). We want the derivative to “pull a constant”out of our guess. That is exactly what happens when we differentiate eat, where a isa constant. Differentiating gives

d

dteat = aeat

3 The precise transition is 14C → 14N + e− + νe where e− is an electron and νe is an electronneutrino.

297

Page 314: CLP-1 Differential Calculus

APPLICATIONS OF DERIVATIVES 3.3 EXPONENTIAL GROWTH AND DECAY

i.e. “pulls the constant a out of eat”.We have succeeded in guessing a single function, namely e−kt, that obeys equa-

tion 3.3.1. Can we guess any other solutions? Yes. If C is any constant, Ce−kt alsoobeys equation 3.3.1:

d

dt(Ce−kt) = C

d

dte−kt = Ce−kt(−k) = −k(Ce−kt)

You can try guessing some more solutions, but you won’t find any, because with alittle trickery we can prove that a function Q(t) obeys equation 3.3.1 if and only ifQ(t) is of the form Ce−kt, where C is some constant.

The trick 4 is to imagine that Q(t) is any (at this stage, unknown) solution toequation 3.3.1 and to compare Q(t) and our known solution e−kt by studying theratio Q(t)/e−kt. We will show that Q(t) obeys equation 3.3.1 if and only if the ratioQ(t)/e−kt is a constant, i.e. if and only if the derivative of the ratio is zero. By theproduct rule

d

dt

[Q(t)/e−kt

]=

d

dt

[ektQ(t)

]= kektQ(t) + ektQ′(t)

Since ekt is never 0, the right hand side is zero if and only if kQ(t) +Q′(t) = 0; that isQ′(t) = −kQ(t). Thus

d

dtQ(t) = −kQ(t) ⇐⇒ d

dt

[Q(t)/e−kt

]= 0

as required.We have succeed in finding all functions that obey 3.3.1. That is we have found

the general solution to 3.3.1. This is worth stating as a theorem.

Theorem 3.3.2

A differentiable function Q(t) obeys the differential equation

dQ

dt(t) = −kQ(t)

if and only if there is a constant C such that

Q(t) = Ce−kt

Before we start to apply the above theorem, we take this opportunity to remindthe reader that in this text we will use log x with no base to indicate the naturallogarithm. That is

log x = loge x = lnx

Both of the notations log(x) and ln(x) are used widely and the reader should becomfortable with both.

4 Notice that is very similar to what we needed in Example 3.1.2, except that here the constant ismultiplicative rather than additive. That is const× f(t) rather than const+ f(t).

298

Page 315: CLP-1 Differential Calculus

APPLICATIONS OF DERIVATIVES 3.3 EXPONENTIAL GROWTH AND DECAY

Example 3.3.3 Carbon dating and half–life.

In this example, we determine the value of the constant of proportionality k in equa-tion 3.3.1 that corresponds to the half–life of 14C, which is 5730 years.

• Imagine that some plant or animal contains a quantity Q0 of 14C at its time ofdeath. Let’s choose the zero point of time t = 0 to be the instant that the plantor animal died.

• Denote by Q(t) the amount of 14C in the plant or animal t years after it died.Then Q(t) must obey both equation 3.3.1 and Q(0) = Q0.

• Since Q(t) must obey equation 3.3.1, Theorem 3.3.2 tells us that there must bea constant C such that Q(t) = Ce−kt. To also have Q0 = Q(0) = Ce−k×0, theconstant C must be Q0. That is, Q(t) = Q0e

−kt for all t ≥ 0.

• By definition, the half–life of 14C is the length of time that it takes for half ofthe 14C to decay. That is, the half–life t1/2 is determined by

Q(t1/2) =1

2Q(0) =

1

2Q0 but we know Q(t) = Q0e

−kt

Q0e−kt1/2 =

1

2Q0 now cancel Q0

e−kt1/2 =1

2

Taking the logarithm of both sides gives

−kt1/2 = log1

2= − log 2 and so

k =log 2

t1/2.

We are told that, for 14C, the half–life t1/2 = 5730, so

k =log 2

5730= 0.000121 to 6 digits

From the work in the above example we have accumulated enough new facts tomake a corollary to Theorem 3.3.2.

Corollary 3.3.4

The function Q(t) satisfies the equation

dQ

dt= −kQ(t)

299

Page 316: CLP-1 Differential Calculus

APPLICATIONS OF DERIVATIVES 3.3 EXPONENTIAL GROWTH AND DECAY

if and only if

Q(t) = Q(0) · e−kt.

The half-life is defined to be the time t1/2 which obeys

Q(t1/2) =1

2·Q(0).

The half-life is related to the constant k by

t1/2 =log 2

k

Now here is a typical problem that is solved using Corollary 3.3.4.

Example 3.3.5 Determining the age of an artefact.

A particular piece of parchment contains about 64% as much 14C as plants do today.Estimate the age of the parchment.Solution Let Q(t) denote the amount of 14C in the parchment t years after it was firstcreated.By equation 3.3.1 and Example 3.3.3,

dQ

dt= −kQ(t) with k =

log 2

5730= 0.000121.

By Corollary 3.3.4

Q(t) = Q(0) · e−kt

The time at which Q(t) reaches 0.64Q(0) is determined by

Q(t) = 0.64Q(0) but Q(t) = Q(0)e−kt

Q(0)e−kt = 0.64Q(0) cancel Q(0)

e−kt = 0.64 take logarithms−kt = log 0.64

t =log 0.64

−k =log 0.64

−0.000121= 3700 to 2significant digits.

That is, the parchment a is about 37 centuries old.

a The British Museum has an Egyptian mathematical text from the seventeenth century B.C.

300

Page 317: CLP-1 Differential Calculus

APPLICATIONS OF DERIVATIVES 3.3 EXPONENTIAL GROWTH AND DECAY

We have stated that the half-life of 14C is 5730 years. How can this be determined?We can explain this using the following example.

Example 3.3.6 Computing a half-life.

A scientist in a B-grade science fiction film is studying a sample of the rare andfictitious element, implausium a. With great effort he has produced a sample of pureimplausium. The next day — 17 hours later — he comes back to his lab and discoversthat his sample is now only 37% pure. What is the half-life of the element?Solution We can again set up our problem using Corollary 3.3.4. Let Q(t) denote thequantity of implausium at time t, measured in hours. Then we know

Q(t) = Q(0) · e−kt

We also know that

Q(17) = 0.37Q(0).

That enables us to determine k via

Q(17) = 0.37Q(0) = Q(0)e−17k divide both sides by Q(0)

0.37 = e−17k

and so

k = − log 0.37

17= 0.05849

We can then convert this to the half life using Corollary 3.3.4:

t1/2 =log 2

k≈ 11.85 hours

While this example is entirely fictitious, one really can use this approach to measurethe half-life of materials.

a Implausium leads to even weaker plots than unobtainium.

3.3.2 tt Newton’s Law of Cooling

Recall Newton’s law of cooling from the start of this section:

The rate of change of temperature of an object is proportional to the dif-ference in temperature between the object and its surroundings.

301

Page 318: CLP-1 Differential Calculus

APPLICATIONS OF DERIVATIVES 3.3 EXPONENTIAL GROWTH AND DECAY

The temperature of the surroundings is sometimes called the ambient temperature.We then translated this statement into the following differential equation

Equation 3.3.7 Newton’s law of cooling.

dT

dt(t) = K

[T (t)− A

]

where T (t) is the temperature of the object at time t, A is the temperature ofits surroundings, and K is a constant of proportionality. This mathematical modelof temperature change works well when studying a small object in a large, fixedtemperature, environment. For example, a hot cup of coffee in a large room 5.

Before we worry about solving this equation, let’s think a little about the sign ofthe constant of proportionality. At any time t, there are three possibilities.

• If T (t) > A, that is, if the body is warmer than its surroundings, we would ex-pect heat to flow from the body into its surroundings and so we would expectthe body to cool off so that dT

dt(t) < 0. For this expectation to be consistent with

equation 3.3.7, we need K < 0.

• If T (t) < A, that is the body is cooler than its surroundings, we would expectheat to flow from the surroundings into the body and so we would expect thebody to warm up so that dT

dt(t) > 0. For this expectation to be consistent with

equation 3.3.7, we again need K < 0.

• Finally if T (t) = A, that is the body and its environment have the same temper-ature, we would not expect any heat to flow between the two and so we wouldexpect that dT

dt(t) = 0. This does not impose any condition on K.

In conclusion, we would expect K < 0. Of course, we could have chosen to call theconstant of proportionality −k, rather than K. Then the differential equation wouldbe dT

dt= −k

(T − A

)and we would expect k > 0.

Now to find the general solution to equation 3.3.7. Since this equation is so sim-ilar in form to equation 3.3.1, we might expect a similar solution. Start by tryingT (t) = CeKt and let’s see what goes wrong. Substitute it into the equation:

dT

dt= K(T (t)− A)

KCeKt = KCeKT −KA?0 = −KA? the constant A causes problems!

5 It does not work so well when the object is of a similar size to its surroundings since the tempera-ture of the surroundings will rise as the object cools. It also fails when there are phase transitionsinvolved — for example, an ice-cube melting in a warm room does not obey Newton’s law ofcooling.

302

Page 319: CLP-1 Differential Calculus

APPLICATIONS OF DERIVATIVES 3.3 EXPONENTIAL GROWTH AND DECAY

Let’s try something a little different — recall that the derivative of a constant is zero.So we can add or subtract a constant from T (t) without changing its derivative. SetQ(t) = T (t) +B, then

dQ

dt(t) =

dT

dt(t) by Newton’s law of cooling

= K(T (t)− A) = K(Q(t)−B − A)

So if we choose B = −A then we will have

dQ

dt(t) = KQ(t)

which is exactly the same form as equation 3.3.1, but with K = −k. So by Theo-rem 3.3.2

Q(t) = Q(0)eKt

We can translate back to T (t), since Q(t) = T (t)−A and Q(0) = T (0)−A. This givesus the solution.

Corollary 3.3.8

A differentiable function T (t) obeys the differential equation

dT

dt(t) = K

[T (t)− A

]if and only if

T (t) = [T (0)− A] eKt + A

Just before we put this into action, we remind the reader that log x = loge x = lnx.

Example 3.3.9 Warming iced tea.

The temperature of a glass of iced tea is initially 5. After 5 minutes, the tea hasheated to 10 in a room where the air temperature is 30.

a Determine the temperature as a function of time.

b What is the temperature after 10 minutes?

c Determine when the tea will reach a temperature of 20.

Solution Part (a)

• Denote by T (t) the temperature of the tea t minutes after it was removed fromthe fridge, and let A = 30 be the ambient temperature.

303

Page 320: CLP-1 Differential Calculus

APPLICATIONS OF DERIVATIVES 3.3 EXPONENTIAL GROWTH AND DECAY

Example 3.3.9

• By Newton’s law of cooling,

dT

dt= K(T − A) = K(T − 30)

for some, as yet unknown, constant of proportionality K.

• By Corollary 3.3.8,

T (t) = [T (0)− 30] eKt + 30 = 30− 25eKt

since the initial temperature T (0) = 5.

• This solution is not complete because it still contains an unknown constant,namely K. We have not yet used the given data that T (5) = 10. We can use itto determine K. At t = 5,

T (5) = 30− 25e5K = 10 rearrange

e5K =20

25

5K = log20

25and so

K =1

5log

4

5= −0.044629 to 6 digits

Part (b)

• To find the temperature at 10 minutes we can just use the solution we havedetermined above.

T (10) = 30− 25e10K

= 30− 25e10× 15

log 45

= 30− 25e2 log 45 = 30− 25elog 16

25

= 30− 16 = 14

Part (c)

• We can find when the temperature is 20 by solving T (t) = 20:

20 = 30− 25eKt rearrange

eKt =10

25=

2

5

Kt = log2

5

t =log 2

5

K= 20.5 minutes to 1 decimal place

304

Page 321: CLP-1 Differential Calculus

APPLICATIONS OF DERIVATIVES 3.3 EXPONENTIAL GROWTH AND DECAY

A slightly more gruesome example.

Example 3.3.10 Determining a time from temperatures.

A dead body is discovered at 3:45pm in a room where the temperature is 20C. Atthat time the temperature of the body is 27C. Two hours later, at 5:45pm, the tem-perature of the body is 25.3 C. What was the time of death? Note that the normal(adult human) body temperature is 37.Solution We will assume a that the body’s temperature obeys Newton’s law of cool-ing.

• Denote by T (t) the temperature of the body at time t, with t = 0 correspondingto 3:45pm. We wish to find the time of death — call it td.

• There is a lot of data in the statement of the problem; we are told that

the ambient temperature: A = 20

the temperature of the body when discovered: T (0) = 27

the temperature of the body 2 hours later: T (2) = 25.3

assuming the person was a healthy adult right up until he died, the tem-perature at the time of death: T (td) = 37.

• Since we assume the temperature of the body obeys Newton’s law of cooling,we use Corollary 3.3.8 to find,

T (t) = [T (0)− A] eKt + A = 20 + 7eKt

Two unknowns remain, K and td.

• We can find the constant K by using T (2) = 25.3:

25.3 = T (2) = 20 + 7e2K rearrange

7e2K = 5.3 rearrange a bit more

2K = log(

5.37

)K = 1

2log(

5.37

)= −0.139 to 3 decimal places

• Since we know b that td is determined by T (td) = 37, we have

37 = T (td) = 20 + 7e−0.139td rearrangee−0.139td = 17

7

−0.139td = log(

177

)td = − 1

0.139log(

177

)= −6.38 to 2 decimal places

Now 6.38 hours is 6 hours and 0.38 × 60 = 23 minutes. So the time of deathwas 6 hours and 23 minutes before 3:45pm, which is 9:22am.

305

Page 322: CLP-1 Differential Calculus

APPLICATIONS OF DERIVATIVES 3.3 EXPONENTIAL GROWTH AND DECAY

Example 3.3.10

a We don’t know any other method!b Actually, we are assuming again.

A slightly tricky example — we need to determine the ambient temperature fromthree measurements at different times.

Example 3.3.11 Finding the temperature outside.

A glass of room-temperature water is carried out onto a balcony from an apartmentwhere the temperature is 22C. After one minute the water has temperature 26Cand after two minutes it has temperature 28C. What is the outdoor temperature?Solution We will assume that the temperature of the thermometer obeys Newton’slaw of cooling.

• Let A be the outdoor temperature and T (t) be the temperature of the water tminutes after it is taken outside.

• By Newton’s law of cooling,

T (t) = A+(T (0)− A

)eKt

by Corollary 3.3.8. Notice there are 3 unknowns here — A, T (0) and K — sowe need three pieces of information to find them all.

• We are told T (0) = 22, so

T (t) = A+(22− A

)eKt.

• We are also told T (1) = 26, which gives

26 = A+(22− A

)eK rearrange things

eK =26− A22− A

• Finally, T (2) = 28, so

28 = A+(22− A

)e2K rearrange

e2K =28− A22− A but eK =

26− A22− A , so(

26− A22− A

)2

=28− A22− A multiply through by (22− A)2

(26− A)2 = (28− A)(22− A)

306

Page 323: CLP-1 Differential Calculus

APPLICATIONS OF DERIVATIVES 3.3 EXPONENTIAL GROWTH AND DECAY

Example 3.3.11

We can expand out both sides and collect up terms to get

262︸︷︷︸=676

−52A+ A2 = 28× 22︸ ︷︷ ︸=616

−50A+ A2

60 = 2A

30 = A

So the temperature outside is 30.

3.3.3 tt Population Growth

Suppose that we wish to predict the size P (t) of a population as a function of the timet. In the most naive model of population growth, each couple produces β offspring(for some constant β) and then dies. Thus over the course of one generation β P (t)

2

children are produced and P (t) parents die so that the size of the population growsfrom P (t) to

P (t+ tg) = P (t) + βP (t)

2︸ ︷︷ ︸parents+offspring

− P (t)︸︷︷︸parents die

2P (t)

where tg denotes the lifespan of one generation. The rate of change of the size of thepopulation per unit time is

P (t+ tg)− P (t)

tg=

1

tg

[β2P (t)− P (t)

]= bP (t)

where b = β−22tg

is the net birthrate per member of the population per unit time. If weapproximate

P (t+tg)−P (t)

tg≈ dP

dt(t)

we get the differential equation

Equation 3.3.12 Simple population model.

dP

dt= bP (t)

307

Page 324: CLP-1 Differential Calculus

APPLICATIONS OF DERIVATIVES 3.3 EXPONENTIAL GROWTH AND DECAY

By Corollary 3.3.4, with −k replaced by b,

P (t) = P (0) · ebt

This is called the Malthusian 6 growth model. It is, of course, very simplistic. Oneof its main characteristics is that, since P (t + T ) = P (0) · eb(t+T ) = P (t) · ebT , everytime you add T to the time, the population size is multiplied by ebT . In particular, thepopulation size doubles every log 2

bunits of time. The Malthusian growth model can

be a reasonably good model only when the population size is very small comparedto its environment 7. A more sophisticated model of population growth, that takesinto account the “carrying capacity of the environment” is considered in the optionalsubsection below.

Example 3.3.13 A simple prediction of future population.

In 1927 the population of the world was about 2 billion. In 1974 it was about 4 billion.Estimate when it reached 6 billion. What will the population of the world be in 2100,assuming the Malthusian growth model?Solution We follow our usual pattern for dealing with such problems.

• Let P (t) be the world’s population t years after 1927. Note that 1974 corre-sponds to t = 1974− 1927 = 47.

• We are assuming that P (t) obeys equation 3.3.12. So, by Corollary 3.3.4 with−k replaced by b,

P (t) = P (0) · ebt

Notice that there are 2 unknowns here — b and P (0) — so we need two piecesof information to find them.

• We are told P (0) = 2, so

P (t) = 2 · ebt

• We are also told P (47) = 4, which gives

4 = 2 · e47b clean up

e47b = 2 take the log and clean up

b =log 2

47= 0.0147 to 3 significant digits

• We now know P (t) completely, so we can easily determine the predicted pop-ulation a in 2100, i.e. at t = 2100− 1927 = 173.

P (173) = 2e173b = 2e173×0.0147 = 25.4 billion

6 This is named after Rev. Thomas Robert Malthus. He described this model in a 1798 paper called“An essay on the principle of population”.

7 That is, the population has plenty of food and space to grow.

308

Page 325: CLP-1 Differential Calculus

APPLICATIONS OF DERIVATIVES 3.3 EXPONENTIAL GROWTH AND DECAY

Example 3.3.13

• Finally, our crude model predicts that the population is 6 billion at the time tthat obeys

P (t) = 2ebt = 6 clean up

ebt = 3 take the log and clean up

t =log 3

b= 47

log 3

log 2= 74.5

which corresponds b to the middle of 2001.

a The 2015 Revision of World Population, a publication of the United Nations, predicts that theworld’s population in 2100 will be about 11 billion. They are predicting a reduction in the worldpopulation growth rate due to lower fertility rates, which the Malthusian growth model does nottake into account.

b The world population really reached 6 billion in about 1999.

3.3.3.1ttt (Optional) — Logistic Population Growth

Logistic growth adds one more wrinkle to the simple population model. It assumesthat the population only has access to limited resources. As the size of the populationgrows the amount of food available to each member decreases. This in turn causesthe net birth rate b to decrease. In the logistic growth model b = b0

(1− P

K

), where K

is called the carrying capacity of the environment, so that

P ′(t) = b0

(1− P (t)

K

)P (t)

We can learn quite a bit about the behaviour of solutions to differential equationslike this, without ever finding formulae for the solutions, just by watching the signof P ′(t). For concreteness, we’ll look at solutions of the differential equation

dP

dt(t) =

(6000− 3P (t)

)P (t)

We’ll sketch the graphs of four functions P (t) that obey this equation.

• For the first function, P (0) = 0.

• For the second function, P (0) = 1000.

• For the third function, P (0) = 2000.

• For the fourth function, P (0) = 3000.

309

Page 326: CLP-1 Differential Calculus

APPLICATIONS OF DERIVATIVES 3.3 EXPONENTIAL GROWTH AND DECAY

The sketches will be based on the observation that (6000−3P )P = 3(2000−P )P

• is zero for P = 0, 2000,

• is strictly positive for 0 < P < 2000 and

• is strictly negative for P > 2000.

Consequently

dP

dt(t)

= 0 if P (t) = 0

> 0 if 0 < P (t) < 2000

= 0 if P (t) = 2000

< 0 if P (t) > 2000

Thus if P (t) is some function that obeys dPdt

(t) =(6000−3P (t)

)P (t), then as the graph

of P (t) passes through(t, P (t)

)

the graph has

slope zero, i.e. is horizontal, if P (t) = 0

positive slope, i.e. is increasing, if 0 < P (t) < 2000

slope zero, i.e. is horizontal, if P (t) = 2000

negative slope, i.e. is decreasing, if 0 < P (t) < 2000

as illustrated in the figure

As a result,

• if P (0) = 0, the graph starts out horizontally. In other words, as t starts toincrease, P (t) remains at zero, so the slope of the graph remains at zero. Thepopulation size remains zero for all time. As a check, observe that the functionP (t) = 0 obeys dP

dt(t) =

(6000− 3P (t)

)P (t) for all t.

• Similarly, if P (0) = 2000, the graph again starts out horizontally. So P (t) re-mains at 2000 and the slope remains at zero. The population size remains 2000for all time. Again, the function P (t) = 2000 obeys dP

dt(t) =

(6000− 3P (t)

)P (t)

for all t.

310

Page 327: CLP-1 Differential Calculus

APPLICATIONS OF DERIVATIVES 3.3 EXPONENTIAL GROWTH AND DECAY

• If P (0) = 1000, the graph starts out with positive slope. So P (t) increaseswith t. As P (t) increases towards 2000, the slope (6000 − 3P (t)

)P (t), while

remaining positive, gets closer and closer to zero. As the graph approachesheight 2000, it becomes more and more horizontal. The graph cannot actuallycross from below 2000 to above 2000, because to do so, it would have to havestrictly positive slope for some value of P above 2000, which is not allowed.

• If P (0) = 3000, the graph starts out with negative slope. So P (t) decreaseswith t. As P (t) decreases towards 2000, the slope (6000 − 3P (t)

)P (t), while

remaining negative, gets closer and closer to zero. As the graph approachesheight 2000, it becomes more and more horizontal. The graph cannot actuallycross from above 2000 to below 2000, because to do so, it would have to havenegative slope for some value of P below 2000. which is not allowed.

These curves are sketched in the figure below. We conclude that for any initialpopulation size P (0), except P (0) = 0, the population size approaches 2000 as t→∞.

3.3.4 tt Exercises

ttt Exercises for § 3.3.1

Exercises — Stage 1

1. Which of the following is a differential equation for an unknown function yof x?

(a) y =dy

dx(b)

dy

dx= 3 [y − 5] (c) y = 3

[y − dx

dx

](d) ex = ey + 1 (e) y = 10ex

2. Which of the following functions Q(t) satisfy the differential equation

Q(t) = 5dQ

dt?

(a) Q(t) = 0 (b) Q(t) = 5et (c) Q(t) = e5t

311

Page 328: CLP-1 Differential Calculus

APPLICATIONS OF DERIVATIVES 3.3 EXPONENTIAL GROWTH AND DECAY

(d) Q(t) = et/5 (e) Q(t) = et/5 + 1

3. Suppose a sample starts out with C grams of a radioactive isotope, andthe amount of the radioactive isotope left in the sample at time t is givenby

Q(t) = Ce−kt

for some positive constant k. When will Q(t) = 0?

Exercises — Stage 2

4. *. Consider a function of the form f(x) = Aekx where A and k are constants.If f(0) = 5 and f(7) = π, find the constants A and k.

5. *. Find the function y(t) ifdy

dt+ 3y = 0, y(1) = 2.

6. A sample of bone belongs to an animal that died 10,000 years ago. Ifthe bone contained 5 µg of Carbon-14 when the animal died, how muchCarbon-14 do you expect it to have now?

7. A sample containing one gram of Radium-226 was stored in a lab 100years ago; now the sample only contains 0.9576 grams of Radium-226.What is the half-life of Radium-226?

8. *. The mass of a sample of Polonium–210, initially 6 grams, decreases at arate proportional to the mass. After one year, 1 gram remains. What is thehalf–life (the time it takes for the sample to decay to half its original mass)?

9. Radium-221 has a half-life of 30 seconds. How long does it take for only0.01% of an original sample to be left?

Exercises — Stage 3

10. Polonium-210 has a half life of 138 days. What percentage of a sampleof Polonium-210 decays in a day?

11. A sample of ore is found to contain 7.2 ± 0.3 µg of Uranium-232, the half-life of which is between 68.8 and 70 years. How much Uranium-232 willremain undecayed in the sample in 10 years?

ttt Exercises for § 3.3.2

Exercises — Stage 1

312

Page 329: CLP-1 Differential Calculus

APPLICATIONS OF DERIVATIVES 3.3 EXPONENTIAL GROWTH AND DECAY

1. Which of the following functions T (t) satisfy the differential equationdT

dt= 5 [T − 20]?

(a) T (t) = 20 (b) T (t) = 20e5t − 20 (c) T (t) = e5t + 20

(d) T (t) = 20e5t + 20

2. At time t = 0, an object is placed in a room, of temperature A. After tseconds, Newton’s Law of Cooling gives the temperature of the object is as

T (t) = 35eKt − 10

What is the temperature of the room? Is the room warmer or colder thanthe object?

3. A warm object is placed in a cold room. The temperature of the object,over time, approaches the temperature of the room it is in. The temper-ature of the object at time t is given by

T (t) = [T (0)− A]eKt + A.

Can K be a positive number? Can K be a negative number? Can K bezero?

4. Suppose an object obeys Newton’s Law of Cooling, and its temperature isgiven by

T (t) = [T (0)− A]ekt + A

for some constant k. At what time is T (t) = A?

Exercises — Stage 2

5. A piece of copper at room temperature (25) is placed in a boiling pot ofwater. After 10 seconds, it has heated to 90. When will it be 99.9?

6. Today is a chilly day. We heated up a stone to 500 C in a bonfire,then took it out and left it outside, where the temperature is 0 C. Af-ter 10 minutes outside of the bonfire, the stone had cooled to a still-untouchable 100 C. Now the stone is at a cozy 50 C. How long agowas the stone taken out of the fire?

Exercises — Stage 3

7. *. Isaac Newton drinks his coffee with cream. To be exact, 9 parts coffee to1 part cream. His landlady pours him a cup of coffee at 95 C into whichNewton stirs cream taken from the icebox at 5 C. When he drinks the mix-ture ten minutes later, he notes that it has cooled to 54 C. Newton wondersif his coffee would be hotter (and by how much) if he waited until just be-

313

Page 330: CLP-1 Differential Calculus

APPLICATIONS OF DERIVATIVES 3.3 EXPONENTIAL GROWTH AND DECAY

fore drinking it to add the cream. Analyze this question, assuming that:

i The temperature of the dining room is constant at 22 C.

ii When a volume V1 of liquid at temperature T1 is mixed with a volume

V2 at temperature T2, the temperature of the mixture isV1T1 + V2T2

V1 + V2

.

iii Newton’s Law of Cooling: The temperature of an object cools at a rateproportional to the difference in temperature between the object andits surroundings.

iv The constant of proportionality is the same for the cup of coffee withcream as for the cup of pure coffee.

8. *. The temperature of a glass of iced tea is initially 5. After 5 minutes,the tea has heated to 10 in a room where the air temperature is 30.

a Use Newton’s law of cooling to obtain a differential equation forthe temperature T (t) at time t.

b Determine when the tea will reach a temperature of 20.

9. Suppose an object is changing temperature according to Newton’s Lawof Cooling, and its temperature at time t is given by

T (t) = 0.8kt + 15

Is k positive or negative?

ttt Exercises for § 3.3.3

Exercises — Stage 1

1. Let a population at time t be given by the Malthusian model,

P (t) = P (0)ebt for some positive constant b.

Evaluate limt→∞

P (t). Does this model make sense for large values of t?

Exercises — Stage 2

2. In the 1950s, pure-bred wood bison were thought to be extinct. How-ever, a small population was found in Canada. For decades, a cap-

314

Page 331: CLP-1 Differential Calculus

APPLICATIONS OF DERIVATIVES 3.3 EXPONENTIAL GROWTH AND DECAY

tive breeding program has been working to increase their numbers, andfrom time to time wood bison are released to the wild. Suppose in 2015,a released herd numbered 121 animals, and a year later, there were 136 a.If the wood bison adhere to the Malthusian model (a big assumption!),and if there are no more releases of captive animals, how many animalswill the herd have in 2020?

a These numbers are loosely based on animals actually released near Shageluk,Alaska in 2015. Watch the first batch being released here.

3. A founding colony of 1,000 bacteria is placed in a petri dish of yummybacteria food. After an hour, the population has doubled. Assuming theMalthusian model, how long will it take for the colony to triple its originalpopulation?

4. A single pair of rats comes to an island after a shipwreck. They multiplyaccording to the Malthusian model. In 1928, there were 1,000 rats on theisland, and the next year there were 1500. When was the shipwreck?

5. A farmer wants to farm cochineals, which are insects used to make red dye.The farmer raises a small number of cochineals as a test. In three months,a test population of cochineals will increase from 200 individuals to 1000,given ample space and food.The farmer’s plan is to start with an initial population of P (0) cochineals,and after a year have 1 000 000 + P (0) cochineals, so that one million can beharvested, and P (0) saved to start breeding again. What initial populationP (0) does the Malthusian model suggest?

Exercises — Stage 3

6. Let f(t) = 100ekt, for some constant k.

a If f(t) is the amount of a decaying radioactive isotope in a sampleat time t, what is the amount of the isotope in the sample whent = 0? What is the sign of k?

b If f(t) is the number of individuals in a population that is grow-ing according to the Malthusian model, how many individuals arethere when t = 0? What is the sign of k?

c If f(t) is the temperature of an object at time t, given by Newton’sLaw of Cooling, what is the ambient temperature surrounding theobject? What is the sign of k?

315

Page 332: CLP-1 Differential Calculus

APPLICATIONS OF DERIVATIVES 3.3 EXPONENTIAL GROWTH AND DECAY

ttt Further problems for § 3.3

1. *. Find f(2) if f ′(x) = πf(x) for all x, and f(0) = 2.

2. Which functions T (t) satisfy the differential equationdT

dt= 7T + 9?

3. *. It takes 8 days for 20% of a particular radioactive material to decay. Howlong does it take for 100 grams of the material to decay to 40 grams?

4. A glass of boiling water is left in a room. After 15 minutes, it has cooled to 85

C, and after 30 minutes it is 73 C. What temperature is the room?

5. *. A 25-year-old graduate of UBC is given $50,000 which is invested at 5% peryear compounded continuously. The graduate also intends to deposit moneycontinuously at the rate of $2000 per year. Assuming that the interest rateremains 5%, the amount A(t) of money at time t satisfies the equation

dA

dt= 0.05A+ 2000

a Solve this equation and determine the amount of money in the accountwhen the graduate is 65.

b At age 65, the graduate will withdraw money continuously at the rate ofW dollars per year. If the money must last until the person is 85, what isthe largest possible value of W ?

6. *. An investor puts $120,000 which into a bank account which pays 6% annualinterest, compounded continuously. She plans to withdraw money continu-ously from the account at the rate of $9000 per year. If A(t) is the amount ofmoney at time t, then

dA

dt= 0.06A− 9000

a Solve this equation for A(t).

b When will the money run out?

7. *. A particular bacterial culture grows at a rate proportional to the number ofbacteria present. If the size of the culture triples every nine hours, how longdoes it take the culture to double?

8. *. An object falls under gravity near the surface of the earth and its motion isimpeded by air resistance proportional to its speed. Its velocity v satisfies thedifferential equation

dv

dt= −g − kv

where g and k are positive constants.

a Find the velocity of the object as a function of time t, given that it was v0

316

Page 333: CLP-1 Differential Calculus

APPLICATIONS OF DERIVATIVES 3.4 TAYLOR POLYNOMIALS

at t = 0.

b Find limt→∞

v(t).

3.4qApproximating Functions Near a Specified Point — Tay-lor Polynomials

Suppose that you are interested in the values of some function f(x) for x near somefixed point a. When the function is a polynomial or a rational function we can usesome arithmetic (and maybe some hard work) to write down the answer. For exam-ple:

f(x) =x2 − 3

x2 − 2x+ 4

f(1/5) =125− 3

125− 2

5+ 4

=1−75

251−10+100

25

=−74

91

Tedious, but we can do it. On the other hand if you are asked to compute sin(1/10)then what can we do? We know that a calculator can work it out

sin(1/10) = 0.09983341 . . .

but how does the calculator do this? How did people compute this before calculators1 ? A hint comes from the following sketch of sin(x) for x around 0.

−1 −0.5 0.5 1

−1

−0.5

0.5

1

x

xsinx

The above figure shows that the curves y = x and y = sinx are almost the samewhen x is close to 0. Hence if we want the value of sin(1/10) we could just use thisapproximation y = x to get

sin(1/10) ≈ 1/10.

1 Originally the word “calculator” referred not to the software or electronic (or even mechanical)device we think of today, but rather to a person who performed calculations.

317

Page 334: CLP-1 Differential Calculus

APPLICATIONS OF DERIVATIVES 3.4 TAYLOR POLYNOMIALS

Of course, in this case we simply observed that one function was a good approxima-tion of the other. We need to know how to find such approximations more system-atically.

More precisely, say we are given a function f(x) that we wish to approximateclose to some point x = a, and we need to find another function F (x) that

• is simple and easy to compute 2

• is a good approximation to f(x) for x values close to a.

Further, we would like to understand how good our approximation actually is.Namely we need to be able to estimate the error |f(x)− F (x)|.

There are many different ways to approximate a function and we will discussone family of approximations: Taylor polynomials. This is an infinite family of everimproving approximations, and our starting point is the very simplest.

3.4.1 tt Zeroth Approximation — the Constant Approximation

The simplest functions are those that are constants. And our zeroth 3 approximationwill be by a constant function. That is, the approximating function will have theform F (x) = A, for some constant A. Notice that this function is a polynomial ofdegree zero.

To ensure that F (x) is a good approximation for x close to a, we choose A so thatf(x) and F (x) take exactly the same value when x = a.

F (x) = A so F (a) = A = f(a) =⇒ A = f(a)

Our first, and crudest, approximation rule is

Equation 3.4.1 Constant approximation.

f(x) ≈ f(a)

An important point to note is that we need to know f(a) — if we cannot computethat easily then we are not going to be able to proceed. We will often have to choosea (the point around which we are approximating f(x)) with some care to ensure thatwe can compute f(a).

Here is a figure showing the graphs of a typical f(x) and approximating functionF (x).

2 It is no good approximating a function with something that is even more difficult to work with.3 It barely counts as an approximation at all, but it will help build intuition. Because of this, and

the fact that a constant is a polynomial of degree 0, we’ll start counting our approximations fromzero rather than 1.

318

Page 335: CLP-1 Differential Calculus

APPLICATIONS OF DERIVATIVES 3.4 TAYLOR POLYNOMIALS

At x = a, f(x) and F (x) take the same value. For x very near a, the values of f(x)and F (x) remain close together. But the quality of the approximation deterioratesfairly quickly as x moves away from a. Clearly we could do better with a straightline that follows the slope of the curve. That is our next approximation.

But before then, an example:

Example 3.4.2 A (weak) approximation of e0.1.

Use the constant approximation to estimate e0.1.Solution First set f(x) = ex.

• Now we first need to pick a point x = a to approximate the function. Thispoint needs to be close to 0.1 and we need to be able to evaluate f(a) easily.The obvious choice is a = 0.

• Then our constant approximation is just

F (x) = f(0) = e0 = 1

F (0.1) = 1

Note that e0.1 = 1.105170918 . . ., so even this approximation isn’t too bad..

3.4.2 tt First Approximation — the Linear Approximation

Our first 4 approximation improves on our zeroth approximation by allowing the ap-proximating function to be a linear function of x rather than just a constant function.That is, we allow F (x) to be of the form A+Bx, for some constants A and B.

To ensure that F (x) is a good approximation for x close to a, we still require thatf(x) and F (x) have the same value at x = a (that was our zeroth approximation).Our additional requirement is that their tangent lines at x = a have the same slope

4 Recall that we started counting from zero.

319

Page 336: CLP-1 Differential Calculus

APPLICATIONS OF DERIVATIVES 3.4 TAYLOR POLYNOMIALS

— that the derivatives of f(x) and F (x) are the same at x = a. Hence

F (x) = A+Bx =⇒ F (a) = A+Ba = f(a)

F ′(x) = B =⇒ F ′(a) = B = f ′(a)

So we must have B = f ′(a). Substituting this into A + Ba = f(a) we get A =f(a)− af ′(a). So we can write

F (x) = A+Bx =

A︷ ︸︸ ︷f(a)− af ′(a) +f ′(a) · x

= f(a) + f ′(a) · (x− a)

We write it in this form because we can now clearly see that our first approximationis just an extension of our zeroth approximation. This first approximation is alsooften called the linear approximation of f(x) about x = a.

Equation 3.4.3 Linear approximation.

f(x) ≈ f(a) + f ′(a)(x− a)

We should again stress that in order to form this approximation we need to knowf(a) and f ′(a) — if we cannot compute them easily then we are not going to be ableto proceed.

Recall, from Theorem 2.3.4, that y = f(a) + f ′(a)(x− a) is exactly the equation ofthe tangent line to the curve y = f(x) at a. Here is a figure showing the graphs of atypical f(x) and the approximating function F (x).

Observe that the graph of f(a)+f ′(a)(x−a) remains close to the graph of f(x) for amuch larger range of x than did the graph of our constant approximation, f(a). Onecan also see that we can improve this approximation if we can use a function thatcurves down rather than being perfectly straight. That is our next approximation.

But before then, back to our example:

320

Page 337: CLP-1 Differential Calculus

APPLICATIONS OF DERIVATIVES 3.4 TAYLOR POLYNOMIALS

Example 3.4.4 A better approximation of e0.1.

Use the linear approximation to estimate e0.1.Solution First set f(x) = ex and a = 0 as before.

• To form the linear approximation we need f(a) and f ′(a):

f(x) = ex f(0) = 1

f ′(x) = ex f ′(0) = 1

• Then our linear approximation is

F (x) = f(0) + xf ′(0) = 1 + x

F (0.1) = 1.1

Recall that e0.1 = 1.105170918 . . ., so the linear approximation is almost correct to 3digits.

It is worth doing another simple example here.

Example 3.4.5 A linear approximation of√

4.1.

Use a linear approximation to estimate√

4.1.Solution First set f(x) =

√x. Hence f ′(x) = 1

2√x. Then we are trying to approximate

f(4.1). Now we need to choose a sensible a value.

• We need to choose a so that f(a) and f ′(a) are easy to compute.

We could try a = 4.1 — but then we need to compute f(4.1) and f ′(4.1) —which is our original problem and more!

We could try a = 0 — then f(0) = 0 and f ′(0) = DNE.

Setting a = 1 gives us f(1) = 1 and f ′(1) = 12. This would work, but we

can get a better approximation by choosing a is closer to 4.1.

Indeed we can set a to be the square of any rational number and we’ll geta result that is easy to compute.

Setting a = 4 gives f(4) = 2 and f ′(4) = 14. This seems good enough.

• Substitute this into equation 3.4.3 to get

f(4.1) ≈ f(4) + f ′(4) · (4.1− 4)

= 2 +0.1

4= 2 + 0.025 = 2.025

Notice that the true value is√

4.1 = 2.024845673 . . ..

321

Page 338: CLP-1 Differential Calculus

APPLICATIONS OF DERIVATIVES 3.4 TAYLOR POLYNOMIALS

3.4.3 tt Second Approximation — the Quadratic Approximation

We next develop a still better approximation by now allowing the approximatingfunction be to a quadratic function of x. That is, we allow F (x) to be of the formA+Bx+Cx2, for some constants A, B and C. To ensure that F (x) is a good approx-imation for x close to a, we choose A, B and C so that

• f(a) = F (a) (just as in our zeroth approximation),

• f ′(a) = F ′(a) (just as in our first approximation), and

• f ′′(a) = F ′′(a) — this is a new condition.

These conditions give us the following equations

F (x) = A+Bx+ Cx2 =⇒ F (a) = A+Ba+ Ca2 = f(a)

F ′(x) = B + 2Cx =⇒ F ′(a) = B + 2Ca = f ′(a)

F ′′(x) = 2C =⇒ F ′′(a) = 2C = f ′′(a)

Solve these for C first, then B and finally A.

C =1

2f ′′(a) substitute

B = f ′(a)− 2Ca = f ′(a)− af ′′(a) substitute again

A = f(a)−Ba− Ca2 = f(a)− a[f ′(a)− af ′′(a)]− 1

2f ′′(a)a2

Then put things back together to build up F (x):

F (x) = f(a)− f ′(a)a+1

2f ′′(a)a2 (this line is A)

+ f ′(a)x − f ′′(a)ax (this line is Bx)

+1

2f ′′(a)x2 (this line is Cx2)

= f(a) + f ′(a)(x− a) +1

2f ′′(a)(x− a)2

Oof! We again write it in this form because we can now clearly see that our secondapproximation is just an extension of our first approximation.

Our second approximation is called the quadratic approximation:

Equation 3.4.6 Quadratic approximation.

f(x) ≈ f(a) + f ′(a)(x− a) +1

2f ′′(a)(x− a)2

322

Page 339: CLP-1 Differential Calculus

APPLICATIONS OF DERIVATIVES 3.4 TAYLOR POLYNOMIALS

Here is a figure showing the graphs of a typical f(x) and approximating functionF (x).

This new approximation looks better than both the first and second.Now there is actually an easier way to derive this approximation, which we show

you now. Let us rewrite 5

F (x) so that it is easy to evaluate it and its derivatives at x = a:

F (x) = α + β · (x− a) + γ · (x− a)2

Then

F (x) = α + β · (x− a) + γ · (x− a)2 F (a) = α = f(a)

F ′(x) = β + 2γ · (x− a) F ′(a) = β = f ′(a)

F ′′(x) = 2γ F ′′(a) = 2γ = f ′′(a)

And from these we can clearly read off the values of α, β and γ and so recover ourfunction F (x). Additionally if we write things this way, then it is quite clear how toextend this to a cubic approximation and a quartic approximation and so on.

Return to our example:

Example 3.4.7 An even better approximation of e0.1.

Use the quadratic approximation to estimate e0.1.Solution Set f(x) = ex and a = 0 as before.

• To form the quadratic approximation we need f(a), f ′(a) and f ′′(a):

f(x) = ex f(0) = 1

f ′(x) = ex f ′(0) = 1

f ′′(x) = ex f ′′(0) = 1

• Then our quadratic approximation is

F (x) = f(0) + xf ′(0) +1

2x2f ′′(0) = 1 + x+

x2

2

5 Any polynomial of degree two can be written in this form. For example, when a = 1, 3+2x+x2 =6 + 4(x− 1) + (x− 1)2.

323

Page 340: CLP-1 Differential Calculus

APPLICATIONS OF DERIVATIVES 3.4 TAYLOR POLYNOMIALS

Example 3.4.7

F (0.1) = 1.105

Recall that e0.1 = 1.105170918 . . ., so the quadratic approximation is quite accuratewith very little effort.

Before we go on, let us first introduce (or revise) some notation that will makeour discussion easier.

3.4.4 ttWhirlwind Tour of Summation Notation

In the remainder of this section we will frequently need to write sums involvinga large number of terms. Writing out the summands explicitly can become quiteimpractical — for example, say we need the sum of the first 11 squares:

1 + 22 + 32 + 42 + 52 + 62 + 72 + 82 + 92 + 102 + 112

This becomes tedious. Where the pattern is clear, we will often skip the middle fewterms and instead write

1 + 22 + · · ·+ 112.

A far more precise way to write this is using Σ (capital-sigma) notation. For example,we can write the above sum as

11∑k=1

k2

This is read as

The sum from k equals 1 to 11 of k2.

More generally

Definition 3.4.8

Let m ≤ n be integers and let f(x) be a function defined on the integers. Thenwe write

n∑k=m

f(k)

to mean the sum of f(k) for k from m to n:

f(m) + f(m+ 1) + f(m+ 2) + · · ·+ f(n− 1) + f(n).

324

Page 341: CLP-1 Differential Calculus

APPLICATIONS OF DERIVATIVES 3.4 TAYLOR POLYNOMIALS

Similarly we write

n∑i=m

ai

to mean

am + am+1 + am+2 + · · ·+ an−1 + an

for some set of coefficients am, . . . , an.

Consider the example7∑

k=3

1

k2=

1

32+

1

42+

1

52+

1

62+

1

72

It is important to note that the right hand side of this expression evaluates to a num-ber 6; it does not contain “k”. The summation index k is just a “dummy” variableand it does not have to be called k. For example

7∑k=3

1

k2=

7∑i=3

1

i2=

7∑j=3

1

j2=

7∑`=3

1

`2

Also the summation index has no meaning outside the sum. For example

k7∑

k=3

1

k2

has no mathematical meaning; It is gibberish 7 .

3.4.5 tt Still Better Approximations — Taylor Polynomials

We can use the same strategy to generate still better approximations by polynomials8 of any degree we like. As was the case with the approximations above, we de-termine the coefficients of the polynomial by requiring, that at the point x = a, theapproximation and its first n derivatives agree with those of the original function.

Rather than simply moving to a cubic polynomial, let us try to write things in amore general way. We will consider approximating the function f(x) using a poly-

6 Some careful addition shows it is 46181176400 .

7 Or possibly gobbledygook. For a discussion of statements without meaning and why one shouldavoid them we recommend the book “Bendable learnings: the wisdom of modern management”by Don Watson.

8 Polynomials are generally a good choice for an approximating function since they are so easy towork with. Depending on the situation other families of functions may be more appropriate. Forexample if you are approximating a periodic function, then sums of sines and cosines might be abetter choice; this leads to Fourier series.

325

Page 342: CLP-1 Differential Calculus

APPLICATIONS OF DERIVATIVES 3.4 TAYLOR POLYNOMIALS

nomial, Tn(x), of degree n — where n is a non-negative integer. As we discussedabove, the algebra is easier if we write

Tn(x) = c0 + c1(x− a) + c2(x− a)2 + · · ·+ cn(x− a)n

=n∑k=0

ck(x− a)k using Σ notation

The above form 9 10 makes it very easy to evaluate this polynomial and its deriva-tives at x = a. Before we proceed, we remind the reader of some notation (seeNotation 2.2.8):

• Let f(x) be a function and k be a positive integer. We can denote its kth deriva-tive with respect to x by

dkf

dxk

(d

dx

)kf(x) f (k)(x)

Additionally we will need

Definition 3.4.9 Factorial.

Let n be a positive integer a , then n-factorial, denoted n!, is the product

n! = n× (n− 1)× · · · × 3× 2× 1

Further, we use the convention that

0! = 1

The first few factorials are

1! = 1 2! = 2 3! = 6

4! = 24 5! = 120 6! = 720

a It is actually possible to define the factorial of positive real numbers and even negativenumbers but it requires more advanced calculus and is outside the scope of this course.The interested reader should look up the Gamma function.

Now consider Tn(x) and its derivatives:

Tn(x) =c0 + c1(x− a)+c2(x− a)2 + c3(x− a)3 + · · ·+cn(x− a)n

9 Any polynomial in x of degree n can also be expressed as a polynomial in (x − a) of the samedegree n and vice versa. So Tn(x) really still is a polynomial of degree n.

10 Furthermore when x is close to a, (x − a)k decreases very quickly as k increases, which oftenmakes the "high k" terms in Tn(x) very small. This can be a considerable advantage when build-ing up approximations by adding more and more terms. If we were to rewrite Tn(x) in the formn∑k=0

bkxk the "high k" terms would typically not be very small when x is close to a.

326

Page 343: CLP-1 Differential Calculus

APPLICATIONS OF DERIVATIVES 3.4 TAYLOR POLYNOMIALS

T ′n(x) = c1 +2c2(x− a) + 3c3(x− a)2+ · · ·+ncn(x− a)n−1

T ′′n (x) = 2c2 + 6c3(x− a) + · · ·+n(n− 1)cn(x− a)n−2

T ′′′n (x) = 6c3 + · · ·+n(n− 1)(n− 2)cn(x− a)n−3

...

T (n)n (x) = n! · cn

Now notice that when we substitute x = a into the above expressions only the con-stant terms survive and we get

Tn(a) = c0

T ′n(a) = c1

T ′′n (a) = 2 · c2

T ′′′n (a) = 6 · c3

...

T (n)n (a) = n! · cn

So now if we want to set the coefficients of Tn(x) so that it agrees with f(x) atx = a then we need

Tn(a) = c0 = f(a) c0 = f(a) =1

0!f(a)

We also want the first n derivatives of Tn(x) to agree with the derivatives of f(x) atx = a, so

T ′n(a) = c1 = f ′(a) c1 = f ′(a) =1

1!f ′(a)

T ′′n (a) = 2 · c2 = f ′′(a) c2 =1

2f ′′(a) =

1

2!f ′′(a)

T ′′′n (a) = 6 · c3 = f ′′′(a) c3 =1

6f ′′′(a) =

1

3!f ′′′(a)

More generally, making the kth derivatives agree at x = a requires :

T (k)n (a) = k! · ck = f (k)(a) ck =

1

k!f (k)(a)

And finally the nth derivative:

T (n)n (a) = n! · cn = f (n)(a) cn =

1

n!f (n)(a)

Putting this all together we have

327

Page 344: CLP-1 Differential Calculus

APPLICATIONS OF DERIVATIVES 3.4 TAYLOR POLYNOMIALS

Equation 3.4.10 Taylor polynomial.

f(x) ≈ Tn(x) = f(a) + f ′(a)(x− a) +1

2f ′′(a) · (x− a)2 + · · ·

+1

n!f (n)(a) · (x− a)n

=n∑k=0

1

k!f (k)(a) · (x− a)k

Let us formalise this definition.Definition 3.4.11 Taylor polynomial.

Let a be a constant and let n be a non-negative integer. The nth degree Taylorpolynomial for f(x) about x = a is

Tn(x) =n∑k=0

1

k!f (k)(a) · (x− a)k.

The special case a = 0 is called a Maclaurin a polynomial.

a The polynomials are named after Brook Taylor who devised a general method for con-structing them in 1715. Slightly later, Colin Maclaurin made extensive use of the specialcase a = 0 (with attribution of the general case to Taylor) and it is now named after him.The special case of a = 0 was worked on previously by James Gregory and Isaac Newton,and some specific cases were known to the 14th century Indian mathematician Madhavaof Sangamagrama.

Before we proceed with some examples, a couple of remarks are in order.

• While we can compute a Taylor polynomial about any a-value (providing thederivatives exist), in order to be a useful approximation, we must be able tocompute f(a), f ′(a), · · · , f (n)(a) easily. This means we must choose the point awith care. Indeed for many functions the choice a = 0 is very natural — hencethe prominence of Maclaurin polynomials.

• If we have computed the approximation Tn(x), then we can readily extend thisto the next Taylor polynomial Tn+1(x) since

Tn+1(x) = Tn(x) +1

(n+ 1)!f (n+1)(a) · (x− a)n+1

This is very useful if we discover that Tn(x) is an insufficient approximation,because then we can produce Tn+1(x) without having to start again from scratch.

328

Page 345: CLP-1 Differential Calculus

APPLICATIONS OF DERIVATIVES 3.4 TAYLOR POLYNOMIALS

3.4.6 tt Some Examples

Let us return to our running example of ex:

Example 3.4.12 Taylor approximations of ex.

The constant, linear and quadratic approximations we used above were the first fewMaclaurin polynomial approximations of ex. That is

T0(x) = 1 T1(x) = 1 + x T2(x) = 1 + x+x2

2

Since ddxex = ex, the Maclaurin polynomials are very easy to compute. Indeed this

invariance under differentiation means that

f (n)(x) = ex n = 0, 1, 2, . . . so

f (n)(0) = 1

Substituting this into equation 3.4.10 we get

Tn(x) =n∑k=0

1

k!xk

Thus we can write down the seventh Maclaurin polynomial very easily:

T7(x) = 1 + x+x2

2+x3

6+x4

24+

x5

120+

x6

720+

x7

5040

The following figure contains sketches of the graphs of ex and its Taylor polynomialsTn(x) for n = 0, 1, 2, 3, 4.

329

Page 346: CLP-1 Differential Calculus

APPLICATIONS OF DERIVATIVES 3.4 TAYLOR POLYNOMIALS

Example 3.4.12

x

y

1 2−1

1

2

3

4

5

6

7

y = T0(x) = 1

y = T1(x) = 1 + x

y = T2(x) = 1 + x+ x2

2

y = T3(x)

y = T4(x)

y = ex

Also notice that if we use T7(1) to approximate the value of e1 we obtain:

e1 ≈ T7(1) = 1 + 1 +1

2+

1

6+

1

24+

1

120+

1

720+

1

5040

=685

252= 2.718253968 . . .

The true value of e is 2.718281828 . . ., so the approximation has an error of about3× 10−5.Under the assumption that the accuracy of the approximation improves with n (anassumption we examine in Subsection 3.4.9 below) we can see that the approxima-tion of e above can be improved by adding more and more terms. Indeed this is howthe expression for e in equation 2.7.4 in Section 2.7 comes about.

Now that we have examined Maclaurin polynomials for ex we should take a lookat log x. Notice that we cannot compute a Maclaurin polynomial for log x since it isnot defined at x = 0.

330

Page 347: CLP-1 Differential Calculus

APPLICATIONS OF DERIVATIVES 3.4 TAYLOR POLYNOMIALS

Example 3.4.13 Taylor approximation of log x.

Compute the 5th Taylor polynomial for log x about x = 1.Solution We have been told a = 1 and fifth degree, so we should start by writingdown the function and its first five derivatives:

f(x) = log x f(1) = log 1 = 0

f ′(x) =1

xf ′(1) = 1

f ′′(x) =−1

x2f ′′(1) = −1

f ′′′(x) =2

x3f ′′′(1) = 2

f (4)(x) =−6

x4f (4)(1) = −6

f (5)(x) =24

x5f (5)(1) = 24

Substituting this into equation 3.4.10 gives

T5(x) = 0 + 1 · (x− 1) +1

2· (−1) · (x− 1)2 +

1

6· 2 · (x− 1)3

+1

24· (−6) · (x− 1)4 +

1

120· 24 · (x− 1)5

= (x− 1)− 1

2(x− 1)2 +

1

3(x− 1)3 − 1

4(x− 1)4 +

1

5(x− 1)5

Again, it is not too hard to generalise the above work to find the Taylor polynomialof degree n: With a little work one can show that

Tn(x) =n∑k=1

(−1)k+1

k(x− 1)k.

For cosine:

Example 3.4.14 Maclaurin polynomial for cosx.

Find the 4th degree Maclaurin polynomial for cosx.Solution We have a = 0 and we need to find the first 4 derivatives of cosx.

f(x) = cos x f(0) = 1

f ′(x) = − sinx f ′(0) = 0

f ′′(x) = − cosx f ′′(0) = −1

f ′′′(x) = sin x f ′′′(0) = 0

331

Page 348: CLP-1 Differential Calculus

APPLICATIONS OF DERIVATIVES 3.4 TAYLOR POLYNOMIALS

f (4)(x) = cos x f (4)(0) = 1

Substituting this into equation 3.4.10 gives

T4(x) = 1 + 1 · (0) · x+1

2· (−1) · x2 +

1

6· 0 · x3 +

1

24· (1) · x4

= 1− x2

2+x4

24

Notice that since the 4th derivative of cosx is cosx again, we also have that the fifthderivative is the same as the first derivative, and the sixth derivative is the same asthe second derivative and so on. Hence the next four derivatives are

f (4)(x) = cos x f (4)(0) = 1

f (5)(x) = − sinx f (5)(0) = 0

f (6)(x) = − cosx f (6)(0) = −1

f (7)(x) = sinx f (7)(0) = 0

f (8)(x) = cos x f (8)(0) = 1

Using this we can find the 8th degree Maclaurin polynomial:

T8(x) = 1− x2

2+x4

24− x6

6!+x8

8!

Continuing this process gives us the 2nth Maclaurin polynomial

T2n(x) =n∑k=0

(−1)k

(2k)!· x2k

Warning 3.4.15

The above formula only works when x is measured in radians, because all ofour derivative formulae for trig functions were developed under the assump-tion that angles are measured in radians.

Below we plot cosx against its first few Maclaurin polynomial approximations:

332

Page 349: CLP-1 Differential Calculus

APPLICATIONS OF DERIVATIVES 3.4 TAYLOR POLYNOMIALS

Example 3.4.15

The above work is quite easily recycled to get the Maclaurin polynomial for sine:

Example 3.4.16 Maclaurin polynomial for sinx.

Find the 5th degree Maclaurin polynomial for sinx.Solution We could simply work as before and compute the first five derivatives ofsinx. But set g(x) = sinx and notice that g(x) = −f ′(x), where f(x) = cos x. Then wehave

g(0) = −f ′(0) = 0

g′(0) = −f ′′(0) = 1

g′′(0) = −f ′′′(0) = 0

g′′′(0) = −f (4)(0) = −1

g(4)(0) = −f (5)(0) = 0

g(5)(0) = −f (6)(0) = 1

333

Page 350: CLP-1 Differential Calculus

APPLICATIONS OF DERIVATIVES 3.4 TAYLOR POLYNOMIALS

Example 3.4.17

Hence the required Maclaurin polynomial is

T5(x) = x− x3

3!+x5

5!

Just as we extended to the 2nth Maclaurin polynomial for cosine, we can also extendour work to compute the (2n+ 1)th Maclaurin polynomial for sine:

T2n+1(x) =n∑k=0

(−1)k

(2k + 1)!· x2k+1

Warning 3.4.17

The above formula only works when x is measured in radians, because all ofour derivative formulae for trig functions were developed under the assump-tion that angles are measured in radians.

Below we plot sinx against its first few Maclaurin polynomial approximations.

334

Page 351: CLP-1 Differential Calculus

APPLICATIONS OF DERIVATIVES 3.4 TAYLOR POLYNOMIALS

To get an idea of how good these Taylor polynomials are at approximating sinand cos, let’s concentrate on sinx and consider x’s whose magnitude |x| ≤ 1. Thereare tricks that you can employ 11 to evaluate sine and cosine at values of x outsidethis range.

If |x| ≤ 1 radians 12, then the magnitudes of the successive terms in the Taylorpolynomials for sinx are bounded by

|x| ≤ 1 13!|x|3 ≤ 1

615!|x|5 ≤ 1

120≈ 0.0083

17!|x|7 ≤ 1

7!≈ 0.0002 1

9!|x|9 ≤ 1

9!≈ 0.000003 1

11!|x|11 ≤ 1

11!≈ 0.000000025

From these inequalities, and the graphs on the previous pages, it certainly looks like,for x not too large, even relatively low degree Taylor polynomials give very goodapproximations. In Section 3.4.9 we’ll see how to get rigorous error bounds on ourTaylor polynomial approximations.

3.4.7 tt Estimating Change and ∆x, ∆y Notation

Suppose that we have two variables x and y that are related by y = f(x), for somefunction f . One of the most important applications of calculus is to help us under-stand what happens to y when we make a small change in x.

Definition 3.4.18

Let x, y be variables related by a function f . That is y = f(x). Then we denotea small change in the variable x by ∆x (read as “delta x”). The correspondingsmall change in the variable y is denoted ∆y (read as “delta y”).

∆y = f(x+ ∆x)− f(x)

In many situations we do not need to compute ∆y exactly and are instead happywith an approximation. Consider the following example.

Example 3.4.19 Estimate the increase in cost for a given change in production.

Let x be the number of cars manufactured per week in some factory and let y thecost of manufacturing those x cars. Given that the factory currently produces a carsper week, we would like to estimate the increase in cost if we make a small changein the number of cars produced.Solution We are told that a is the number of cars currently produced per week; thecost of production is then f(a).

11 If you are writing software to evaluate sinx, you can always use the trig identity sin(x) = sin(x−2nπ), to easily restrict to |x| ≤ π. You can then use the trig identity sin(x) = − sin(x±π) to reduceto |x| ≤ π

2 . Finally you can use the trig identity sin(x) = ∓ cos(π2 ± x)) to reduce to |x| ≤ π4 < 1.

12 Recall that the derivative formulae that we used to derive the Taylor polynomials are valid onlywhen x is in radians. The restriction −1 ≤ x ≤ 1 radians translates to angles bounded by180π ≈ 57.

335

Page 352: CLP-1 Differential Calculus

APPLICATIONS OF DERIVATIVES 3.4 TAYLOR POLYNOMIALS

Example 3.4.21

• Say the number of cars produced is changed from a to a + ∆x (where ∆x issome small number.

• As x undergoes this change, the costs change from y = f(a) to f(a+∆x). Hence

∆y = f(a+ ∆x)− f(a)

• We can estimate this change using a linear approximation. Substituting x =a+ ∆x into the equation 3.4.3 yields the approximation

f(a+ ∆x) ≈ f(a) + f ′(a)(a+ ∆x− a)

and consequently the approximation

∆y = f(a+ ∆x)− f(a) ≈ f(a) + f ′(a)∆x− f(a)

simplifies to the following neat estimate of ∆y:

Equation 3.4.20 Linear approximation of ∆y.

∆y ≈ f ′(a)∆x

• In the automobile manufacturing example, when the production level is acars per week, increasing the production level by ∆x will cost approximatelyf ′(a)∆x. The additional cost per additional car, f ′(a), is called the “marginalcost” of a car.

• If we instead use the quadratic approximation (given by equation 3.4.6) thenwe estimate

f(a+ ∆x) ≈ f(a) + f ′(a)∆x+1

2f ′′(a)∆x2

and so

∆y = f(a+ ∆x)− f(a) ≈ f(a) + f ′(a)∆x+1

2f ′′(a)∆x2 − f(a)

which simplifies to

Equation 3.4.21 Quadratic approximation of ∆y.

∆y ≈ f ′(a)∆x+1

2f ′′(a)∆x2

336

Page 353: CLP-1 Differential Calculus

APPLICATIONS OF DERIVATIVES 3.4 TAYLOR POLYNOMIALS

3.4.8 tt Further Examples

In this subsection we give further examples of computation and use of Taylor ap-proximations.

Example 3.4.22 Estimating tan 46.

Estimate tan 46, using the constant-, linear- and quadratic-approximations (equa-tions 3.4.1, 3.4.3 and 3.4.6).Solution Note that we need to be careful to translate angles measured in degrees toradians.

• Set f(x) = tanx, x = 46 π180

radians and a = 45 π180

= π4

radians. This is a goodchoice for a because

a = 45 is close to x = 46. As noted above, it is generally the case that thecloser x is to a, the better various approximations will be.

We know the values of all trig functions at 45.

• Now we need to compute f and its first two derivatives at x = a. It is a goodtime to recall the special 1 : 1 :

√2 triangle

So

f(x) = tan x f(π/4) = 1

f ′(x) = sec2 x =1

cos2 xf ′(π/4) =

1

1/√

22 = 2

f ′′(x) =2 sinx

cos3 xf ′′(π/4) =

2/√

2

1/√

23 = 4

• As x− a = 46 π180− 45 π

180= π

180radians, the three approximations are

f(x) ≈ f(a)

= 1

f(x) ≈ f(a) + f ′(a)(x− a) = 1 + 2 π180

= 1.034907

337

Page 354: CLP-1 Differential Calculus

APPLICATIONS OF DERIVATIVES 3.4 TAYLOR POLYNOMIALS

Example 3.4.22

f(x) ≈ f(a) + f ′(a)(x−a) +1

2f ′′(a)(x−a)2 = 1 + 2 π

180+

1

24(π

180

)2

= 1.035516

For comparison purposes, tan 46 really is 1.035530 to 6 decimal places.

Warning 3.4.23

All of our derivative formulae for trig functions were developed under theassumption that angles are measured in radians. Those derivatives appearedin the approximation formulae that we used in Example 3.4.22, so we wereobliged to express x− a in radians.

Example 3.4.24 Error inferring a height from an angle.

Suppose that you are ten meters from a vertical pole. You were contracted to mea-sure the height of the pole. You can’t take it down or climb it. So you measure theangle subtended by the top of the pole. You measure θ = 30, which gives

h = 10 tan 30 = 10√3≈ 5.77m

This is just standard trigonometry — if we know the angle exactly then we know theheight exactly.However, in the “real world” angles are hard to measure with such precision. If thecontract requires you the measurement of the pole to be accurate within 10 cm, howaccurate does your measurement of the angle θ need to be?Solution For simplicity a , we are going to assume that the pole is perfectly straightand perfectly vertical and that your distance from the pole was exactly 10 m.

• Write θ = θ0 + ∆θ where θ is the exact angle, θ0 is the measured angle and ∆θis the error.

• Similarly write h = h0 + ∆h, where h is the exact height and h0 = 10√3

is thecomputed height. Their difference, ∆h, is the error.

• Then

h0 = 10 tan θ0 h0 + ∆h = 10 tan(θ0 + ∆θ)

∆h = 10 tan(θ0 + ∆θ)− 10 tan θ0

We could attempt to solve this equation for ∆θ in terms of ∆h — but it is farsimpler to approximate ∆h using the linear approximation in equation 3.4.20.

338

Page 355: CLP-1 Differential Calculus

APPLICATIONS OF DERIVATIVES 3.4 TAYLOR POLYNOMIALS

Example 3.4.24

• To use equation 3.4.20, replace y with h, x with θ and a with θ0. Our functionf(θ) = 10 tan θ and θ0 = 30 = π/6 radians. Then

∆y ≈ f ′(a)∆x becomes ∆h ≈ f ′(θ0)∆θ

Since f(θ) = 10 tan θ, f ′(θ) = 10 sec2 θ and

f ′(θ0) = 10 sec2(π/6) = 10 ·(

2√3

)2

=40

3

• Putting things together gives

∆h ≈ f ′(θ0)∆θ becomes ∆h ≈ 40

3∆θ

We can then solve this equation for ∆θ in terms of ∆h:

∆θ ≈ 3

40∆h

• We are told that we must have |∆h| < 0.1, so we must have

|∆θ| ≤ 3

400

This is measured in radians, so converting back to degrees

3

400· 180

π= 0.43

a Mathematicians love assumptions that let us tame the real world.

Definition 3.4.25

Suppose that you measure, approximately, some quantity. Suppose that theexact value of that quantity isQ0 and that your measurement yieldedQ0 +∆Q.Then |∆Q| is called the absolute error of the measurement and 100 |∆Q|

Q0is called

the percentage error of the measurement. As an example, if the exact value is4 and the measured value is 5, then the absolute error is |5 − 4| = 1 and thepercentage error is 100 |5−4|

4= 25. That is, the error, 1, was 25% of the exact

value, 4.

339

Page 356: CLP-1 Differential Calculus

APPLICATIONS OF DERIVATIVES 3.4 TAYLOR POLYNOMIALS

Example 3.4.26 Error inferring the area and volume from the radius.

Suppose that the radius of a sphere has been measured with a percentage error of atmost ε%. Find the corresponding approximate percentage errors in the surface areaand volume of the sphere.Solution We need to be careful in this problem to convert between absolute andpercentage errors correctly.

• Suppose that the exact radius is r0 and that the measured radius is r0 + ∆r.

• Then the absolute error in the measurement is |∆r| and, by definition, the per-centage error is 100 |∆r|

r0. We are told that 100 |∆r|

r0≤ ε.

• The surface area a of a sphere of radius r isA(r) = 4πr2. The error in the surfacearea computed with the measured radius is

∆A = A(r0 + ∆r)− A(r0) ≈ A′(r0)∆r

= 8πr0∆r

where we have made use of the linear approximation, equation 3.4.20.

• The corresponding percentage error is then

100|∆A|A(r0)

≈ 100|A′(r0)∆r|A(r0)

= 1008πr0|∆r|

4πr20

= 2× 100|∆r|r0

≤ 2ε

• The volume of a sphere b of radius r is V (r) = 43πr3. The error in the volume

computed with the measured radius is

∆V = V (r0 + ∆r)− V (r0) ≈ V ′(r0)∆r

= 4πr20∆r

where we have again made use of the linear approximation, equation 3.4.20.

• The corresponding percentage error is

100|∆V |V (r0)

≈ 100|V ′(r0)∆r|V (r0)

= 1004πr2

0|∆r|4πr3

0/3= 3× 100

|∆r|r0

≤ 3ε

We have just computed an approximation to ∆V . This problem is actually suffi-ciently simple that we can compute ∆V exactly:

∆V = V (r0 + ∆r)− V (r0) = 43π(r0 + ∆r)3 − 4

3πr3

0

• Applying (a+ b)3 = a3 + 3a2b+ 3ab2 + b3 with a = r0 and b = ∆r, gives

V (r0 + ∆r)− V (r0) = 43π[r3

0 + 3r20∆r + 3r0 (∆r)2 + (∆r)3

]− 4

3πr3

0

= 43π[3r2

0∆r + 3r0 (∆r)2 + (∆r)3]

340

Page 357: CLP-1 Differential Calculus

APPLICATIONS OF DERIVATIVES 3.4 TAYLOR POLYNOMIALS

Example 3.4.26

• Thus the difference between the exact error and the linear approximation to theerror is obtained by retaining only the last two terms in the square brackets.This has magnitude

43π∣∣3r0 (∆r)2 + (∆r)3

∣∣ = 43π∣∣3r0 + ∆r

∣∣(∆r)2

or in percentage terms

100 · 143πr3

0

· 43π∣∣3r0 (∆r)2 + (∆r)3

∣∣ = 100

∣∣∣∣3∆r2

r20

+∆r3

r30

∣∣∣∣=

(100

3∆r

r0

)·(

∆r

r0

) ∣∣∣∣1 +∆r

3r0

∣∣∣∣≤ 3ε

( ε

100

)·(

1 +ε

300

)Since ε is small, we can assume that 1 + ε

300≈ 1. Hence the difference between

the exact error and the linear approximation of the error is roughly a factor ofε

100smaller than the linear approximation 3ε.

• As an aside, notice that if we argue that ∆r is very small and so we can ignoreterms involving (∆r)2 and (∆r)3 as being really really small, then we obtain

V (r0 + ∆r)− V (r0) = 43π[3r2

0∆r+3r0 (∆r)2 + (∆r)3︸ ︷︷ ︸really really small

]

≈ 43π · 3r2

0∆r = 4πr20∆r

which is precisely the result of our linear approximation above.

a We do not expect you to remember the surface areas of solids for this course.b We do expect you to remember the formula for the volume of a sphere.

Example 3.4.27 Percentage error inferring a height.

To compute the height h of a lamp post, the length s of the shadow of a two meterpole is measured. The pole is 6 m from the lamp post. If the length of the shadowwas measured to be 4 m, with an error of at most one cm, find the height of the lamppost and estimate the percentage error in the height.Solution We should first draw a picture a

341

Page 358: CLP-1 Differential Calculus

APPLICATIONS OF DERIVATIVES 3.4 TAYLOR POLYNOMIALS

• By similar triangles we see that

2

s=

h

6 + s

from which we can isolate h as a function of s:

h =2(6 + s)

s=

12

s+ 2

• The length of the shadow was measured to be s0 = 4 m. The correspondingheight of the lamp post is

h0 =12

4+ 2 = 5m

• If the error in the measurement of the length of the shadow was ∆s, then theexact shadow length was s = s0 + ∆s and the exact lamp post height is h =f(s0 + ∆s), where f(s) = 12

s+ 2. The error in the computed lamp post height is

∆h = h− h0 = f(s0 + ∆s)− f(s0)

• We can then make a linear approximation of this error using equation 3.4.20:

∆h ≈ f ′(s0)∆s = −12

s20

∆s = −12

42∆s

• We are told that |∆s| ≤ 1100

m. Consequently, approximately,

|∆h| ≤ 12

42

1

100=

3

400

The percentage error is then approximately

100|∆h|h0

≤ 1003

400× 5= 0.15%

342

Page 359: CLP-1 Differential Calculus

APPLICATIONS OF DERIVATIVES 3.4 TAYLOR POLYNOMIALS

Example 3.4.27

a We get to reuse that nice lamp post picture from Example 3.2.4.

3.4.9 tt The Error in the Taylor Polynomial Approximations

Any time you make an approximation, it is desirable to have some idea of the sizeof the error you introduced. That is, we would like to know the difference R(x)between the original function f(x) and our approximation F (x):

R(x) = f(x)− F (x).

Of course if we know R(x) exactly, then we could recover f(x) = F (x) + R(x) — sothis is an unrealistic hope. In practice we would simply like to bound R(x):

|R(x)| = |f(x)− F (x)| ≤M

where (hopefully)M is some small number. It is worth stressing that we do not needthe tightest possible value of M , we just need a relatively easily computed M thatisn’t too far off the true value of |f(x)− F (x)|.

We will now develop a formula for the error introduced by the constant approx-imation, equation 3.4.1 (developed back in Section 3.4.1)

f(x) ≈ f(a) = T0(x) 0th Taylor polynomial

The resulting formula can be used to get an upper bound on the size of the error|R(x)|.

The main ingredient we will need is the Mean-Value Theorem (Theorem 2.13.5)— so we suggest you quickly revise it. Consider the following obvious statement:

f(x) = f(x) now some sneaky manipulations= f(a) + (f(x)− f(a))

= f(a)︸︷︷︸=T0(x)

+(f(x)− f(a)) · x− ax− a︸ ︷︷ ︸

=1

= T0(x) +f(x)− f(a)

x− a︸ ︷︷ ︸looks familiar

·(x− a)

Indeed, this equation is important in the discussion that follows, so we’ll highlight it

343

Page 360: CLP-1 Differential Calculus

APPLICATIONS OF DERIVATIVES 3.4 TAYLOR POLYNOMIALS

Equation 3.4.28 We will need it again soon.

f(x) = T0(x) +

[f(x)− f(a)

x− a

](x− a)

The coefficientf(x)− f(a)

x− a of (x− a) is the average slope of f(t) as t moves from

t = a to t = x. We can picture this as the slope of the secant joining the points(a, f(a)) and (x, f(x)) in the sketch below.

As t moves from a to x, the instantaneous slope f ′(t) keeps changing. Sometimesf ′(t) might be larger than the average slope f(x)−f(a)

x−a , and sometimes f ′(t) mightbe smaller than the average slope f(x)−f(a)

x−a . However, by the Mean-Value Theorem(Theorem 2.13.5), there must be some number c, strictly between a and x, for which

f ′(c) =f(x)− f(a)

x− a exactly.

Substituting this into formula 3.4.28 gives

Equation 3.4.29 Towards the error.

f(x) = T0(x) + f ′(c)(x− a) for some c strictly between a and x

Notice that this expression as it stands is not quite what we want. Let us massagethis around a little more into a more useful form

Equation 3.4.30 The error in constant approximation.

f(x)− T0(x) = f ′(c) · (x− a) for some c strictly between a and x

344

Page 361: CLP-1 Differential Calculus

APPLICATIONS OF DERIVATIVES 3.4 TAYLOR POLYNOMIALS

Notice that the MVT doesn’t tell us the value of c, however we do know that itlies strictly between x and a. So if we can get a good bound on f ′(c) on this intervalthen we can get a good bound on the error.

Example 3.4.31 Error in the approximation in 3.4.2.

Let us return to Example 3.4.2, and we’ll try to bound the error in our approximationof e0.1.

• Recall that f(x) = ex, a = 0 and T0(x) = e0 = 1.

• Then by equation 3.4.30

e0.1 − T0(0.1) = f ′(c) · (0.1− 0) with 0 < c < 0.1

• Now f ′(c) = ec, so we need to bound ec on (0, 0.1). Since ec is an increasingfunction, we know that

e0 < f ′(c) < e0.1 when 0 < c < 0.1

So one is tempted to write that

|e0.1 − T0(0.1)| = |R(x)| = |f ′(c)| · (0.1− 0)

< e0.1 · 0.1

And while this is true, it is rather circular. We have just bounded the error inour approximation of e0.1 by 1

10e0.1 — if we actually knew e0.1 then we wouldn’t

need to estimate it!

• While we don’t know e0.1 exactly, we do know a that 1 = e0 < e0.1 < e1 < 3.This gives us

|R(0.1)| < 3× 0.1 = 0.3

That is — the error in our approximation of e0.1 is no greater than 0.3. Recallthat we don’t need the error exactly, we just need a good idea of how large itactually is.

• In fact the real error here is

|e0.1 − T0(0.1)| = |e0.1 − 1| = 0.1051709 . . .

so we have over-estimated the error by a factor of 3.

But we can actually go a little further here — we can bound the error above andbelow. If we do not take absolute values, then since

e0.1 − T0(0.1) = f ′(c) · 0.1 and 1 < f ′(c) < 3

345

Page 362: CLP-1 Differential Calculus

APPLICATIONS OF DERIVATIVES 3.4 TAYLOR POLYNOMIALS

Example 3.4.31

we can write

1× 0.1 ≤ (e0.1 − T0(0.1)) ≤ 3× 0.1

so

T0(0.1) + 0.1 ≤ e0.1 ≤ T0(0.1) + 0.3

1.1 ≤ e0.1 ≤ 1.3

So while the upper bound is weak, the lower bound is quite tight.

a Oops! Do we really know that e < 3? We haven’t proved it. We will do so soon.

There are formulae similar to equation 3.4.29, that can be used to bound the errorin our other approximations; all are based on generalisations of the MVT. The nextone — for linear approximations — is

f(x) = f(a) + f ′(a)(x− a)︸ ︷︷ ︸=T1(x)

+1

2f ′′(c)(x− a)2 for some c strictly between a and x

which we can rewrite in terms of T1(x):

Equation 3.4.32 The error in linear approximation.

f(x)− T1(x) =1

2f ′′(c)(x− a)2 for some c strictly between a and x

It implies that the error that we make when we approximate f(x) by T1(x) =f(a) + f ′(a) (x− a) is exactly 1

2f ′′(c) (x− a)2 for some c strictly between a and x.

More generally

f(x) = f(a)+f ′(a) · (x−a)+ · · ·+ 1

n!f (n)(a) · (x−a)n︸ ︷︷ ︸

=Tn(x)

+1

(n+1)!f (n+1)(c) · (x−a)n+1

for some c strictly between a and x. Again, rewriting this in terms of Tn(x) gives

346

Page 363: CLP-1 Differential Calculus

APPLICATIONS OF DERIVATIVES 3.4 TAYLOR POLYNOMIALS

Equation 3.4.33

f(x)− Tn(x) =1

(n+ 1)!f (n+1)(c) · (x− a)n+1 for some c strictly between a and x

That is, the error introduced when f(x) is approximated by its Taylor polynomialof degree n, is precisely the last term of the Taylor polynomial of degree n + 1, butwith the derivative evaluated at some point between a and x, rather than exactly ata. These error formulae are proven in the optional Section 3.4.10 later in this chapter.

Example 3.4.34 Approximate sin 46 and estimate the error.

Approximate sin 46 using Taylor polynomials about a = 45, and estimate the re-sulting error.Solution

• Start by defining f(x) = sin x and

a = 45 = 45 π180

radians x = 46 = 46 π180

radians

x− a = π180

radians

• The first few derivatives of f at a are

f(x) = sin x f(a) =1√2

f ′(x) = cos x

f ′(a) =1√2

f ′′(x) = − sinx

f ′′(a) = − 1√2

f (3)(x) = − cosx f (3)(a) = − 1√2

• The constant, linear and quadratic Taylor approximations for sin(x) about π4

are

T0(x) = f(a) =1√2

T1(x) = T0(x) + f ′(a) · (x−a) =1√2

+1√2

(x−π

4

)T2(x) = T1(x)+

1

2f ′′(a) · (x−a)2 =

1√2

+1√2

(x−π

4

)− 1

2√

2

(x−π

4

)2

347

Page 364: CLP-1 Differential Calculus

APPLICATIONS OF DERIVATIVES 3.4 TAYLOR POLYNOMIALS

Example 3.4.34

• So the approximations for sin 46 are

sin 46 ≈ T0

(46π

180

)=

1√2

= 0.70710678

sin 46 ≈ T1

(46π

180

)=

1√2

+1√2

( π

180

)= 0.71944812

sin 46 ≈ T2

(46π

180

)=

1√2

+1√2

( π

180

)− 1

2√

2

( π

180

)2

= 0.71934042

• The errors in those approximations are (respectively)

error in 0.70710678 = f ′(c)(x− a) = cos c ·( π

180

)error in 0.71944812 =

1

2f ′′(c)(x− a)2 = −1

2· sin c ·

( π

180

)2

error in 0.71923272 =1

3!f (3)(c)(x− a)3 = − 1

3!· cos c ·

( π

180

)3

In each of these three cases c must lie somewhere between 45 and 46.

• Rather than carefully estimating sin c and cos c for c in that range, we makeuse of a simpler (but much easier bound). No matter what c is, we know that| sin c| ≤ 1 and | cos c| ≤ 1. Hence∣∣error in 0.70710678

∣∣ ≤ ( π

180

)< 0.018∣∣error in 0.71944812

∣∣ ≤ 1

2

( π

180

)2

< 0.00015∣∣error in 0.71934042∣∣ ≤ 1

3!

( π

180

)3

< 0.0000009

Example 3.4.35 Showing e < 3.

In Example 3.4.31 above we used the fact that e < 3 without actually proving it. Let’sdo so now.

• Consider the linear approximation of ex about a = 0.

T1(x) = f(0) + f ′(0) · x = 1 + x

348

Page 365: CLP-1 Differential Calculus

APPLICATIONS OF DERIVATIVES 3.4 TAYLOR POLYNOMIALS

So at x = 1 we have

e ≈ T1(1) = 2

• The error in this approximation is

ex − T1(x) =1

2f ′′(c) · x2 =

ec

2· x2

So at x = 1 we have

e− T1(1) =ec

2

where 0 < c < 1.

• Now since ex is an increasing a function, it follows that ec < e. Hence

e− T1(1) =ec

2<e

2

Moving the e2

to the left hand side and the T1(1) to the right hand side gives

e

2≤ T1(1) = 2

So e < 4.

• This isn’t as tight as we would like — so now do the same with the quadraticapproximation with a = 0:

ex ≈ T2(x) = 1 + x+x2

2

So when x = 1 we have

e ≈ T2(1) = 1 + 1 +1

2=

5

2

• The error in this approximation is

ex − T2(x) =1

3!f ′′′(c) · x3 =

ec

6· x3

So at x = 1 we have

e− T2(1) =ec

6

where 0 < c < 1.

349

Page 366: CLP-1 Differential Calculus

APPLICATIONS OF DERIVATIVES 3.4 TAYLOR POLYNOMIALS

Example 3.4.35

• Again since ex is an increasing function we have ec < e. Hence

e− T2(1) =ec

6<e

6

That is

5e

6< T2(1) =

5

2

So e < 3 as required.

a Since the derivative of ex is ex which is positive everywhere, the function is increasing every-where.

Example 3.4.36 More on ex.

We wrote down the general nth degree Maclaurin polynomial approximation of ex

in Example 3.4.12 above.

• Recall that

Tn(x) =n∑k=0

1

k!xk

• The error in this approximation is (by equation 3.4.33)

ex − Tn(x) =1

(n+ 1)!ec

where c is some number between 0 and x.

• So setting x = 1 in this gives

e− Tn(1) =1

(n+ 1)!ec

where 0 < c < 1.

• Since ex is an increasing function we know that 1 = e0 < ec < e1 < 3, so theabove expression becomes

1

(n+ 1)!≤ e− Tn(1) =

1

(n+ 1)!ec ≤ 3

(n+ 1)!

350

Page 367: CLP-1 Differential Calculus

APPLICATIONS OF DERIVATIVES 3.4 TAYLOR POLYNOMIALS

Example 3.4.36

• So when n = 9 we have

1

10!≤ e−

(1 + 1 +

1

2+ · · ·+ 1

9!

)≤ 3

10!

• Now 1/10! < 3/10! < 10−6, so the approximation of e by

e ≈ 1 + 1 +1

2+ · · ·+ 1

9!=

98641

36288= 2.718281 . . .

is correct to 6 decimal places.

• More generally we know that using Tn(1) to approximate e will have an errorof at most 3

(n+1)!— so it converges very quickly.

Example 3.4.37 Example 3.4.24 Revisited.

Recall a that in Example 3.4.24 (measuring the height of the pole), we used the linearapproximation

f(θ0 + ∆θ) ≈ f(θ0) + f ′(θ0)∆θ

with f(θ) = 10 tan θ and θ0 = 30π

180to get

∆h = f(θ0 + ∆θ)− f(θ0) ≈ f ′(θ0)∆θ which implies that ∆θ ≈ ∆h

f ′(θ0)

• While this procedure is fairly reliable, it did involve an approximation. So thatyou could not 100% guarantee to your client’s lawyer that an accuracy of 10 cmwas achieved.

• On the other hand, if we use the exact formula 3.4.29, with the replacementsx→ θ0 + ∆θ and a→ θ0

f(θ0 + ∆θ) = f(θ0) + f ′(c)∆θ for some c between θ0 and θ0 + ∆θ

in place of the approximate formula 3.4.3, this legality is taken care of:

∆h = f(θ0+∆θ)− f(θ0) = f ′(c)∆θ for some c between θ0 and θ0 + ∆θ

We can clean this up a little more since in our example f ′(θ) = 10 sec2 θ. Thusfor some c between θ0 and θ0 + ∆θ:

|∆h| = 10 sec2(c)|∆θ|

351

Page 368: CLP-1 Differential Calculus

APPLICATIONS OF DERIVATIVES 3.4 TAYLOR POLYNOMIALS

Example 3.4.37

• Of course we do not know exactly what c is. But suppose that we know thatthe angle was somewhere between 25 and 35. In other words suppose that,even though we don’t know precisely what our measurement error was, it wascertainly no more than 5.

• Now on the range 25 < c < 35, sec(c) is an increasing and positive function.Hence on this range

1.217 · · · = sec2 25 ≤ sec2 c ≤ sec2 35 = 1.490 . . . < 1.491

So

12.17 · |∆θ| ≤ |∆h| = 10 sec2(c) · |∆θ| ≤ 14.91 · |∆θ|

• Since we require |∆h| < 0.1, we need 14.91|∆θ| < 0.1, that is

|∆θ| < 0.1

14.91= 0.0067 . . .

So we must measure angles with an accuracy of no less than 0.0067 radians —which is

180

π· 0.0067 = 0.38.

Hence a measurement error of 0.38 or less is acceptable.

a Now is a good time to go back and re-read it.

3.4.10 tt (Optional) — Derivation of the Error Formulae

In this section we will derive the formula for the error that we gave in equation 3.4.33— namely

Rn(x) = f(x)− Tn(x) =1

(n+ 1)!f (n+1)(c) · (x− a)n+1

for some c strictly between a and x, and where Tn(x) is the nth degree Taylor polyno-mial approximation of f(x) about x = a:

Tn(x) =n∑k=0

1

k!f (k)(a).

352

Page 369: CLP-1 Differential Calculus

APPLICATIONS OF DERIVATIVES 3.4 TAYLOR POLYNOMIALS

Recall that we have already proved a special case of this formula for the constant ap-proximation using the Mean-Value Theorem (Theorem 2.13.5). To prove the generalcase we need the following generalisation 13 of that theorem:

Theorem 3.4.38 Generalised Mean-Value Theorem.

Let the functions F (x) and G(x) both be defined and continuous on a ≤ x ≤ band both be differentiable on a < x < b. Furthermore, suppose that G′(x) 6= 0for all a < x < b. Then, there is a number c obeying a < c < b such that

F (b)− F (a)

G(b)−G(a)=F ′(c)

G′(c)

Notice that setting G(x) = x recovers the original Mean-Value Theorem. It turnsout that this theorem is not too difficult to prove from the MVT using some sneakyalgebraic manipulations:

Proof.

• First we construct a new function h(x) as a linear combination of F (x)and G(x) so that h(a) = h(b) = 0. Some experimentation yields

h(x) =[F (b)− F (a)

]·[G(x)−G(a)

]−[G(b)−G(a)

]·[F (x)− F (a)

]• Since h(a) = h(b) = 0, the Mean-Value theorem (actually Rolle’s theorem)

tells us that there is a number c obeying a < c < b such that h′(c) = 0:

h′(x) =[F (b)− F (a)

]·G′(x)−

[G(b)−G(a)

]· F ′(x) so

0 =[F (b)− F (a)

]·G′(c)−

[G(b)−G(a)

]· F ′(c)

Now move the G′(c) terms to one side and the F ′(c) terms to the other:[F (b)− F (a)

]·G′(c) =

[G(b)−G(a)

]· F ′(c).

• Since we haveG′(x) 6= 0, we know thatG′(c) 6= 0. Further the Mean-Valuetheorem ensures a that G(a) 6= G(b). Hence we can move terms about toget

[F (b)− F (a)

]=[G(b)−G(a)

]· F′(c)

G′(c)

F (b)− F (a)

G(b)−G(a)=F ′(c)

G′(c)

as required.

13 It is not a terribly creative name for the generalisation, but it is an accurate one.

353

Page 370: CLP-1 Differential Calculus

APPLICATIONS OF DERIVATIVES 3.4 TAYLOR POLYNOMIALS

a Otherwise if G(a) = G(b) the MVT tells us that there is some point c between a and b sothat G′(c) = 0.

Armed with the above theorem we can now move on to the proof of the Taylorremainder formula.

Proof of equation 3.4.33. We begin by proving the remainder formula forn = 1. That is

f(x)− T1(x) =1

2f ′′(c) · (x− a)2

• Start by setting

F (x) = f(x)− T1(x) G(x) = (x− a)2

Notice that, since T1(a) = f(a) and T ′1(x) = f ′(a),

F (a) = 0 G(a) = 0

F ′(x) = f ′(x)− f ′(a) G′(x) = 2(x− a)

• Now apply the generalised MVT with b = x: there exists a point q be-tween a and x such that

F (x)− F (a)

G(x)−G(a)=F ′(q)

G′(q)

F (x)− 0

G(x)− 0=f ′(q)− f ′(a)

2(q − a)

2 · F (x)

G(x)=f ′(q)− f ′(a)

q − a

• Consider the right-hand side of the above equation and set g(x) = f ′(x).Then we have the term g(q)−g(a)

q−a — this is exactly the form needed to applythe MVT. So now apply the standard MVT to the right-hand side of theabove equation — there is some c between q and a so that

f ′(q)− f ′(a)

q − a =g(q)− g(a)

q − a = g′(c) = f ′′(c)

Notice that here we have assumed that f ′′(x) exists.

• Putting this together we have that

2 · F (x)

G(x)=f ′(q)− f ′(a)

q − a = f ′′(c)

354

Page 371: CLP-1 Differential Calculus

APPLICATIONS OF DERIVATIVES 3.4 TAYLOR POLYNOMIALS

2f(x)− T1(x)

(x− a)2= f ′′(c)

f(x)− T1(x) =1

2!f ′′(c) · (x− a)2

as required.

Oof! We have now proved the cases n = 1 (and we did n = 0 earlier).To proceed — assume we have proved our result for n = 1, 2, · · · , k. We realisethat we haven’t done this yet, but bear with us. Using that assumption we willprove the result is true for n = k + 1. Once we have done that, then

• we have proved the result is true for n = 1, and

• we have shown if the result is true for n = k then it is true for n = k + 1

Hence it must be true for all n ≥ 1. This style of proof is called mathematicalinduction. You can think of the process as something like climbing a ladder:

• prove that you can get onto the ladder (the result is true for n = 1), and

• if I can stand on the current rung, then I can step up to the next rung (ifthe result is true for n = k then it is also true for n = k + 1)

Hence I can climb as high as like.

• Let k > 0 and assume we have proved

f(x)− Tk(x) =1

(k + 1)!f (k+1)(c) · (x− a)k+1

for some c between a and x.

• Now set

F (x) = f(x)− Tk+1(x) G(x) = (x− a)k+1

and notice that, since Tk+1(a) = f(a),

F (a) = f(a)− Tk+1(a) = 0 G(a) = 0 G′(x) = (k + 1)(x− a)k

and apply the generalised MVT with b = x: hence there exists a q betweena and x so that

F (x)− F (a)

G(x)−G(a)=F ′(q)

G′(q)which becomes

F (x)

(x− a)k+1=

F ′(q)

(k + 1)(q − a)krearrange

F (x) =(x− a)k+1

(k + 1)(q − a)k· F ′(q)

355

Page 372: CLP-1 Differential Calculus

APPLICATIONS OF DERIVATIVES 3.4 TAYLOR POLYNOMIALS

• We now examine F ′(q). First carefully differentiate F (x):

F ′(x) =d

dx

[f(x)−

(f(a) + f ′(a)(x− a) +

1

2f ′′(a)(x− a)2 + · · ·

+1

k!f (k)(x− a)k

)]= f ′(x)−

(f ′(a) +

2

2f ′′(a)(x− a) +

3

3!f ′′′(a)(x− a)2 + · · ·

+k

k!f (k)(a)(x− a)k−1

)= f ′(x)−

(f ′(a) + f ′′(a)(x− a) +

1

2f ′′′(a)(x− a)2 + · · ·

+1

(k − 1)!f (k)(a)(x− a)k−1

)Now notice that if we set f ′(x) = g(x) then this becomes

F ′(x) = g(x)−(g(a) + g′(a)(x− a) +

1

2g′′(a)(x− a)2 + · · ·

+1

(k − 1)!g(k−1)(a)(x− a)k−1

)So F ′(x) is then exactly the remainder formula but for a degree k − 1approximation to the function g(x) = f ′(x).

• Hence the function F ′(q) is the remainder when we approximate f ′(q)with a degree k − 1 Taylor polynomial. The remainder formula, equa-tion 3.4.33, then tells us that there is a number c between a and q so that

F ′(q) = g(q)−(g(a) + g′(a)(q − a) +

1

2g′′(a)(q − a)2 + · · ·

+1

(k − 1)!g(k−1)(a)(q − a)k−1

)=

1

k!g(k)(c)(q − a)k =

1

k!f (k+1)(c)(q − a)k

Notice that here we have assumed that f (k+1)(x) exists.

• Now substitute this back into our equation above

F (x) =(x− a)k+1

(k + 1)(q − a)k· F ′(q)

=(x− a)k+1

(k + 1)(q − a)k· 1

k!f (k+1)(c)(q − a)k

356

Page 373: CLP-1 Differential Calculus

APPLICATIONS OF DERIVATIVES 3.4 TAYLOR POLYNOMIALS

=1

(k + 1)k!· f (k+1)(c) · (x− a)k+1(q − a)k

(q − a)k

=1

(k + 1)!· f (k+1)(c) · (x− a)k+1

as required.

So we now know that

• if, for some k, the remainder formula (with n = k) is true for all k timesdifferentiable functions,

• then the remainder formula is true (with n = k + 1) for all k + 1 timesdifferentiable functions.

Repeatedly applying this for k = 1, 2, 3, 4, · · · (and recalling that we have shownthe remainder formula is true when n = 0, 1) gives equation 3.4.33 for all n =0, 1, 2, · · ·.

3.4.11 tt Exercises

ttt Exercises for § 3.4.1

Exercises — Stage 1

1. The graph below shows three curves. The black curve is y = f(x), thered curve is y = g(x) = 1 + 2 sin(1 + x), and the blue curve is y = h(x) =0.7. If you want to estimate f(0), what might cause you to use g(0)?What might cause you to use h(0)?

357

Page 374: CLP-1 Differential Calculus

APPLICATIONS OF DERIVATIVES 3.4 TAYLOR POLYNOMIALS

x

y

y = f(x)y = g(x)

y = h(x)

Exercises — Stage 2 In this and following sections, we will ask you to approximatethe value of several constants, such as log(0.93). A valid question to consider is whywe would ask for approximations of these constants that take lots of time, and areless accurate than what you get from a calculator.

One answer to this question is historical: people were approximating logarithmsbefore they had calculators, and these are some of the ways they did that. Pretendyou’re on a desert island without any of your usual devices and that you want tomake a number of quick and dirty approximate evaluations.

Another reason to make these approximations is technical: how does the calcula-tor get such a good approximation of log(0.93)? The techniques you will learn lateron in this chapter give very accurate formulas for approximating functions like log xand sinx, which are sometimes used in calculators.

A third reason to make simple approximations of expressions that a calculatorcould evaluate is to provide a reality check. If you have a ballpark guess for youranswer, and your calculator gives you something wildly different, you know todouble-check that you typed everything in correctly.For now, questions like Ques-tion 3.4.11.2 through Question 3.4.11.4 are simply for you to practice the fundamentalideas we’re learning.

2. Use a constant approximation to estimate the value of log(x) when x = 0.93.Sketch the curve y = f(x) and your constant approximation.(Remember we use log x to mean the natural logarithm of x, loge x.)

3. Use a constant approximation to estimate arcsin(0.1).

4. Use a constant approximation to estimate√

3 tan(1).

358

Page 375: CLP-1 Differential Calculus

APPLICATIONS OF DERIVATIVES 3.4 TAYLOR POLYNOMIALS

Exercises — Stage 3

5. Use a constant approximation to estimate the value of 10.13. Your esti-mation should be something you can calculate in your head.

ttt Exercises for § 3.4.2

Exercises — Stage 1

1. Suppose f(x) is a function, and we calculated its linear approximationnear x = 5 to be f(x) ≈ 3x− 9.

a What is f(5)?

b What is f ′(5)?

c What is f(0)?

2. The curve y = f(x) is shown below. Sketch the linear approximation off(x) about x = 2.

x

y

y = f(x)

2

3. What is the linear approximation of the function f(x) = 2x+ 5 about x = a?

Exercises — Stage 2

4. Use a linear approximation to estimate log(x) when x = 0.93. Sketch thecurve y = f(x) and your linear approximation.

5. Use a linear approximation to estimate√

5.

359

Page 376: CLP-1 Differential Calculus

APPLICATIONS OF DERIVATIVES 3.4 TAYLOR POLYNOMIALS

6. Use a linear approximation to estimate 5√

30

Exercises — Stage 3

7. Use a linear approximation to estimate 10.13, then compare your esti-mation with the actual value.

8. Imagine f(x) is some function, and you want to estimate f(b). To do this,you choose a value a and take an approximation (linear or constant) of f(x)about a. Give an example of a function f(x), and values a and b, where theconstant approximation gives a more accurate estimation of f(b) than thelinear approximation.

9. The function

L(x) =1

4x+

4π −√

27

12

is the linear approximation of f(x) = arctan x about what point x = a?

ttt Exercises for § 3.4.3

Exercises — Stage 1

1. The quadratic approximation of a function f(x) about x = 3 is

f(x) ≈ −x2 + 6x

What are the values of f(3), f ′(3), f ′′(3), and f ′′′(3)?

2. Give a quadratic approximation of f(x) = 2x+ 5 about x = a.

Exercises — Stage 2

3. Use a quadratic approximation to estimate log(0.93).

4. Use a quadratic approximation to estimate cos

(1

15

).

5. Calculate the quadratic approximation of f(x) = e2x about x = 0.

6. Use a quadratic approximation to estimate 543 .

7. Evaluate the expressions below.

360

Page 377: CLP-1 Differential Calculus

APPLICATIONS OF DERIVATIVES 3.4 TAYLOR POLYNOMIALS

a30∑n=5

1

b3∑

n=1

[2(n+ 3)− n2

]

c10∑n=1

[1

n− 1

n+ 1

]

d4∑

n=1

5 · 2n4n+1

8. Write the following in sigma notation:

a 1 + 2 + 3 + 4 + 5

b 2 + 4 + 6 + 8

c 3 + 5 + 7 + 9 + 11

d 9 + 16 + 25 + 36 + 49

e 9 + 4 + 16 + 5 + 25 + 6 + 36 + 7 + 49 + 8

f 8 + 15 + 24 + 35 + 48

g 3− 6 + 9− 12 + 15− 18

Exercises — Stage 3

9. Use a quadratic approximation of f(x) = 2 arcsinx about x = 0 to ap-proximate f(1). What number are you approximating?

10. Use a quadratic approximation of ex to estimate e as a decimal.

11. Group the expressions below into collections of equivalent expressions.

a10∑n=1

2n

b10∑n=1

2n

361

Page 378: CLP-1 Differential Calculus

APPLICATIONS OF DERIVATIVES 3.4 TAYLOR POLYNOMIALS

c10∑n=1

n2

d 210∑n=1

n

e 211∑n=2

(n− 1)

f14∑n=5

(n− 4)2

g1

4

10∑n=1

(4n+1

2n

)

ttt Exercises for § 3.4.4

Exercises — Stage 1

1. The 3rd degree Taylor polynomial for a function f(x) about x = 1 is

T3(x) = x3 − 5x2 + 9x

What is f ′′(1)?

2. The nth degree Taylor polynomial for f(x) about x = 5 is

Tn(x) =n∑k=0

2k + 1

3k − 9(x− 5)k

What is f (10)(5)?

Exercises — Stage 3

3. The 4th-degree Maclaurin polynomial for f(x) is

T4(x) = x4 − x3 + x2 − x+ 1

What is the third-degree Maclaurin polynomial for f(x)?

362

Page 379: CLP-1 Differential Calculus

APPLICATIONS OF DERIVATIVES 3.4 TAYLOR POLYNOMIALS

4. The 4th degree Taylor polynomial for f(x) about x = 1 is

T4(x) = x4 + x3 − 9

What is the third degree Taylor polynomial for f(x) about x = 1?

5. For any even number n, suppose the nth degree Taylor polynomial forf(x) about x = 5 is

n/2∑k=0

2k + 1

3k − 9(x− 5)2k

What is f (10)(5)?

6. The third-degree Taylor polynomial for f(x) = x3

[2 log x− 11

3

]about

x = a isT3(x) = −2

3

√e3 + 3ex− 6

√ex2 + x3

What is a?

ttt Exercises for § 3.4.5

Exercises — Stage 1

1. Give the 16th degree Maclaurin polynomial for f(x) = sinx+ cosx.

2. Give the 100th degree Taylor polynomial for s(t) = 4.9t2 − t + 10 aboutt = 5.

3. Write the nth-degree Taylor polynomial for f(x) = 2x about x = 1 in sigmanotation.

4. Find the 6th degree Taylor polynomial of f(x) = x2 log x+ 2x2 + 5 aboutx = 1, remembering that log x is the natural logarithm of x, loge x.

5. Give the nth degree Maclaurin polynomial for1

1− x in sigma notation.

Exercises — Stage 3

363

Page 380: CLP-1 Differential Calculus

APPLICATIONS OF DERIVATIVES 3.4 TAYLOR POLYNOMIALS

6. Calculate the 3rd-degree Taylor Polynomial for f(x) = xx about x = 1.

7. Use a 5th-degree Maclaurin polynomial for 6 arctanx to approximate π.

8. Write the 100th-degree Taylor polynomial for f(x) = x(log x−1) about x = 1in sigma notation.

9. Write the (2n)th-degree Taylor polynomial for f(x) = sinx about x =π

4in

sigma notation.

10. Estimate the sum below

1 +1

2+

1

3!+

1

4!+ · · ·+ 1

157!

by interpreting it as a Maclaurin polynomial.

11. Estimate the sum below

100∑k=0

(−1)k

2k!

(5π

4

)2k

by interpreting it as a Maclaurin polynomial.

ttt Exercises for § 3.4.6

Exercises — Stage 1

1. In the picture below, label the following:

f(x) f (x+ ∆x) ∆x ∆y

x

y

y = f(x)

x x+ ∆x

364

Page 381: CLP-1 Differential Calculus

APPLICATIONS OF DERIVATIVES 3.4 TAYLOR POLYNOMIALS

2. At this point in the book, every homework problem takes you about 5minutes. Use the terms you learned in this section to answer the ques-tion: if you spend 15 minutes more, how many more homework prob-lems will you finish?

Exercises — Stage 2

3. Let f(x) = arctan x.

a Use a linear approximation to estimate f(5.1)− f(5).

b Use a quadratic approximation to estimate f(5.1)− f(5).

4. When diving off a cliff from x metres above the water, your speed as youhit the water is given by

s(x) =√

19.6xm

sec

Your last dive was from a height of 4 metres.

a Use a linear approximation of ∆y to estimate how much faster youwill be falling when you hit the water if you jump from a height of 5metres.

b A diver makes three jumps: the first is from x metres, the second fromx + ∆x metres, and the third from x + 2∆x metres, for some fixedpositive values of x and ∆x. Which is bigger: the increase in terminalspeed from the first to the second jump, or the increase in terminalspeed from the second to the third jump?

ttt Exercises for § 3.4.7

Exercises — Stage 1

1. Let f(x) = 7x2−3x+4. Suppose we measure x to be x0 = 2 but that the realvalue of x is x0 + ∆x. Suppose further that the error in our measurement is∆x = 1. Let ∆y be the change in f(x) corresponding to a change of ∆x inx0. That is, ∆y = f (x0 + ∆x)− f(x0).True or false: ∆y = f ′(2)(1) = 25

2. Suppose the exact amount you are supposed to tip is $5.83, but you approx-imate and tip $6. What is the absolute error in your tip? What is the percenterror in your tip?

365

Page 382: CLP-1 Differential Calculus

APPLICATIONS OF DERIVATIVES 3.4 TAYLOR POLYNOMIALS

3. Suppose f(x) = 3x2 − 5. If you measure x to be 10, but its actual value is11, estimate the resulting error in f(x) using the linear approximation, andthen the quadratic approximation.

Exercises — Stage 2

4. A circular pen is being built on a farm. The pen must contain A0 squaremetres, with an error of no more than 2%. Estimate the largest percentageerror allowable on the radius.

5. A circle with radius 3 has a sector cut out of it. It’s a smallish sector, nomore than a quarter of the circle. You want to find out the area of the sector.

a Suppose the angle of the sector is θ. What is the area of the sector?

b Unfortunately, you don’t have a protractor, only a ruler. So, you mea-sure the chord made by the sector (marked d in the diagram above).What is θ in terms of d?

c Suppose you measured d = 0.7, but actually d = 0.68. Estimate theabsolute error in your calculation of the area removed.

6. A conical tank, standing on its pointy end, has height 2 metres and radius0.5 metres. Estimate change in volume of the water in the tank associatedto a change in the height of the water from 50 cm to 45 cm.

0.5

2

Exercises — Stage 3

366

Page 383: CLP-1 Differential Calculus

APPLICATIONS OF DERIVATIVES 3.4 TAYLOR POLYNOMIALS

7. A sample begins with precisely 1 µg of a radioactive isotope, and after 3years is measured to have 0.9 µg remaining. If this measurement is cor-rect to within 0.05 µg, estimate the corresponding accuracy of the half-lifecalculated using it.

ttt Exercises for § 3.4.8

Exercises — Stage 1

1. Suppose f(x) is a function that we approximated by F (x). Further,suppose f(10) = −3, while our approximation was F (10) = 5. LetR(x) = f(x)− F (x).

a True or false: |R(10)| ≤ 7

b True or false: |R(10)| ≤ 8

c True or false: |R(10)| ≤ 9

d True or false: |R(10)| ≤ 100

2. Let f(x) = ex, and let T3(x) be the third-degree Maclaurin polynomialfor f(x),

T3(x) = 1 + x+1

2x2 +

1

3!x3

Use Equation 3.4.33 to give a reasonable bound on the error |f(2) −T3(2)|. Then, find the error |f(2)− T3(2)| using a calculator.

3. Let f(x) = 5x3 − 24x2 + ex − π4, and let T5(x) be the fifth-degree Taylorpolynomial for f(x) about x = 1. Give the best bound you can on the error|f(37)− T (37)|.

4. You and your friend both want to approximate sin(33). Your friend usesthe first-degree Maclaurin polynomial for f(x) = sinx, while you use thezeroth-degree (constant) Maclaurin polynomial for f(x) = sin x. Who has abetter approximation, you or your friend?

Exercises — Stage 2

5. Suppose a function f(x) has sixth derivative

f (6)(x) =6!(2x− 5)

x+ 3.

367

Page 384: CLP-1 Differential Calculus

APPLICATIONS OF DERIVATIVES 3.4 TAYLOR POLYNOMIALS

Let T5(x) be the 5th-degree Taylor polynomial for f(x) about x = 11.Give a bound for the error |f(11.5)− T5(11.5)|.

6. Let f(x) = tanx, and let T2(x) be the second-degree Taylor polynomial forf(x) about x = 0. Give a reasonable bound on the error |f(0.1) − T (0.1)|using Equation 3.4.33.

7. Let f(x) = log(1 − x), and let T5(x) be the fifth-degree Maclaurin poly-nomial for f(x). Use Equation 3.4.33 to give a bound on the error|f(−1

4

)− T5

(−1

4

)|.

(Remember log x = loge x, the natural logarithm of x.)

8. Let f(x) = 5√x, and let T3(x) be the third-degree Taylor polynomial for f(x)

about x = 32. Give a bound on the error |f(30)− T3(30)|.9. Let

f(x) = sin

(1

x

),

and let T1(x) be the first-degree Taylor polynomial for f(x) about x =1

π.

Give a bound on the error |f(0.01) − T1(0.01)|, using Equation 3.4.33. Youmay leave your answer in terms of π.Then, give a reasonable bound on the error |f(0.01)− T1(0.01)|.

10. Let f(x) = arcsinx, and let T2(x) be the second-degree Maclau-rin polynomial for f(x). Give a reasonable bound on the error∣∣f (1

2

)− T2

(12

)∣∣ using Equation 3.4.33. What is the exact value of theerror

∣∣f (12

)− T2

(12

)∣∣?Exercises — Stage 3

11. Let f(x) = log(x), and let Tn(x) be the nth-degree Taylor polynomial forf(x) about x = 1. You use Tn(1.1) to estimate log(1.1). If your estimationneeds to have an error of no more than 10−4, what is an acceptable value ofn to use?

12. Give an estimation of 7√

2200 using a Taylor polynomial. Your estimationshould have an error of less than 0.001.

13. Use Equation 3.4.33 to show that

4241

5040≤ sin(1) ≤ 4243

5040

14. In this question, we use the remainder of a Maclaurin polynomial to ap-proximate e.

368

Page 385: CLP-1 Differential Calculus

APPLICATIONS OF DERIVATIVES 3.4 TAYLOR POLYNOMIALS

a Write out the 4th degree Maclaurin polynomial T4(x) of the functionex.

b Compute T4(1).

c Use your answer from 3.4.11.14.b to conclude326

120< e <

325

119.

ttt Further problems for § 3.4

Exercises — Stage 1

1. *. Consider a function f(x) whose third-degree Maclaurin polynomial is4 + 3x2 + 1

2x3. What is f ′(0)? What is f ′′(0)?

2. *. Consider a function h(x) whose third-degree Maclaurin polynomial is

1 + 4x− 1

3x2 +

2

3x3. What is h(3)(0)?

3. *. The third-degree Taylor polynomial of h(x) about x = 2 is 3 +1

2(x− 2) +

2(x− 2)3.What is h′(2)? What is h′′(2)?

Exercises — Stage 2

4. *. The function f(x) has the property that f(3) = 2, f ′(3) = 4 andf ′′(3) = −10.

a Use the linear approximation to f(x) centred at x = 3 to approxi-mate f(2.98).

b Use the quadratic approximation to f(x) centred at x = 3 to ap-proximate f(2.98).

5. *. Use the tangent line to the graph of y = x1/3 at x = 8 to find anapproximate value for 101/3. Is the approximation too large or too small?

6. *. Estimate√

2 using a linear approximation.

7. *. Estimate 3√

26 using a linear approximation.

8. *. Estimate (10.1)5 using a linear approximation.

9. *. Estimate sin

(101π

100

)using a linear approximation. (Leave your answer

in terms of π.)

369

Page 386: CLP-1 Differential Calculus

APPLICATIONS OF DERIVATIVES 3.4 TAYLOR POLYNOMIALS

10. *. Use a linear approximation to estimate arctan(1.1), using arctan 1 =π

4.

11. *. Use a linear approximation to estimate (2.001)3. Write your answer in theform n/1000 where n is an integer.

12. *. Using a suitable linear approximation, estimate (8.06)2/3. Give your an-swer as a fraction in which both the numerator and denominator are inte-gers.

13. *. Find the third–order Taylor polynomial for f(x) = (1 − 3x)−1/3 aroundx = 0.

14. *. Consider a function f(x) which has f (3)(x) =x

22− x2. Show that when

we approximate f(2) using its second degree Taylor polynomial at a = 1,the absolute value of the error is less than 1

50= 0.02.

15. *. Consider a function f(x) which has f (4)(x) =cos(x2)

3− x . Show that

when we approximate f(0.5) using its third-degree Maclaurin polyno-mial, the absolute value of the error is less than 1

500= 0.002.

16. *. Consider a function f(x) which has f (3)(x) =e−x

8 + x2. Show that when

we approximate f(1) using its second degree Maclaurin polynomial, theabsolute value of the error is less than 1/40.

17. *.

a By using an appropriate linear approximation for f(x) = x1/3, esti-mate 52/3.

b Improve your answer in 3.4.11.17.a by making a quadratic approxi-mation.

c Obtain an error estimate for your answer in 3.4.11.17.a (not just bycomparing with your calculator’s answer for 52/3).

Exercises — Stage 3

18. The 4th degree Maclaurin polynomial for f(x) is

T4(x) = 5x2 − 9

What is the third degree Maclaurin polynomial for f(x)?

370

Page 387: CLP-1 Differential Calculus

APPLICATIONS OF DERIVATIVES 3.5 OPTIMISATION

19. *. The equation y4 + xy = x2 − 1 defines y implicitly as a function of xnear the point x = 2, y = 1.

a Use the tangent line approximation at the given point to estimatethe value of y when x = 2.1.

b Use the quadratic approximation at the given point to estimate thevalue of y when x = 2.1.

c Make a sketch showing how the curve relates to the tangent lineat the given point.

20. *. The equation x4 + y + xy4 = 1 defines y implicitly as a function of x nearthe point x = −1, y = 1.

a Use the tangent line approximation at the given point to estimate thevalue of y when x = −0.9.

b Use the quadratic approximation at the given point to get another es-timate of y when x = −0.9.

c Make a sketch showing how the curve relates to the tangent line at thegiven point.

21. *. Given that log 10 ≈ 2.30259, estimate log 10.3 using a suitable tangentline approximation. Give an upper and lower bound for the error in yourapproximation by using a suitable error estimate.

22. *. Consider f(x) = eex .

a Give the linear approximation for f near x = 0 (call this L(x)).

b Give the quadratic approximation for f near x = 0 (call this Q(x)).

c Prove that L(x) < Q(x) < f(x) for all x > 0.

d Find an interval of length at most 0.01 that is guaranteed to containthe number e0.1.

3.5q Optimisation

One important application of differential calculus is to find the maximum (or mini-mum) value of a function. This often finds real world applications in problems suchas the following.

371

Page 388: CLP-1 Differential Calculus

APPLICATIONS OF DERIVATIVES 3.5 OPTIMISATION

Example 3.5.1 Enclosing a paddock.

A farmer has 400m of fencing materials. What is the largest rectangular paddockthat can be enclosed?Solution We will describe a general approach to these sorts of problems in Sec-tions 3.5.2 and 3.5.3 below, but here we can take a stab at starting the problem.

• Begin by defining variables and their units (more generally we might draw apicture too); let the dimensions of the paddock be x by y metres.

• The area enclosed is then Am2 where

A = x · y

At this stage we cannot apply the calculus we have developed since the area isa function of two variables and we only know how to work with functions ofa single variable. We need to eliminate one variable.

• We know that the perimeter of the rectangle (and hence the dimensions x andy) are constrained by the amount of fencing materials the farmer has to hand:

2x+ 2y ≤ 400

and so we have

y ≤ 200− x

Clearly the area of the paddock is maximised when we use all the fencing pos-sible, so

y = 200− x

• Now substitute this back into our expression for the area

A = x · (200− x)

Since the area cannot be negative (and our lengths x, y cannot be negative ei-ther), we must also have

0 ≤ x ≤ 200

• Thus the question of the largest paddock enclosed becomes the problem offinding the maximum value of

A = x · (200− x) subject to the constraint 0 ≤ x ≤ 200.

The above example is sufficiently simple that we can likely determine the answer

372

Page 389: CLP-1 Differential Calculus

APPLICATIONS OF DERIVATIVES 3.5 OPTIMISATION

by several different methods. In general, we will need more systematic methods forsolving problems of the form

Find the maximum value of y = f(x) subject to a ≤ x ≤ b

To do this we need to examine what a function looks like near its maximum andminimum values.

3.5.1 tt Local and Global Maxima and Minima

We start by asking:

Suppose that the maximum (or minimum) value of f(x) is f(c) then whatdoes that tell us about c?

Notice that we have not yet made the ideas of maximum and minimum very precise.For the moment think of maximum as “the biggest value” and minimum as “thesmallest value”.

Warning 3.5.2

It is important to distinguish between “the smallest value” and “the smallestmagnitude”. For example, because

−5 < −1

the number−5 is smaller than−1. But the magnitude of−1, which is |−1| = 1,is smaller than the magnitude of −5, which is | − 5| = 5. Thus the smallestnumber in the set −1,−5 is −5, while the number in the set −1,−5 thathas the smallest magnitude is −1.

Now back to thinking about what happens around a maximum. Suppose thatthe maximum value of f(x) is f(c), then for all “nearby” points, the function shouldbe smaller.

Consider the derivative of f ′(c):

f ′(c) = limh→0

f(c+ h)− f(c)

h.

373

Page 390: CLP-1 Differential Calculus

APPLICATIONS OF DERIVATIVES 3.5 OPTIMISATION

Split the above limit into the left and right limits:

• Consider points to the right of x = c, For all h > 0,

f(c+ h) ≤ f(c) which implies thatf(c+ h)− f(c) ≤ 0 which also impliesf(c+ h)− f(c)

h≤ 0 since

negativepositive

= negative.

But now if we squeeze h→ 0 we get

limh→0+

f(c+ h)− f(c)

h≤ 0

(provided the limit exists).

• Consider points to the left of x = c. For all h < 0,

f(c+ h) ≤ f(c) which implies thatf(c+ h)− f(c) ≤ 0 which also impliesf(c+ h)− f(c)

h≥ 0 since

negativenegative

= positive.

But now if we squeeze h→ 0 we get

limh→0−

f(c+ h)− f(c)

h≥ 0

(provided the limit exists).

• So if the derivative f ′(c) exists, then the above right- and left-hand limits mustagree, which forces f ′(c) = 0.

Thus we can conclude that

If the maximum value of f(x) is f(c) and f ′(c) exists, then f ′(c) = 0.

Using similar reasoning one can also see that

If the minimum value of f(x) is f(c) and f ′(c) exists, then f ′(c) = 0.

Notice two things about the above reasoning:

• Firstly, in order for the argument to work we only need that f(x) < f(c) for xclose to c — it does not matter what happens for x values far from c.

• Secondly, in the above argument we needed to consider f(x) for x both to theleft of and to the right of c. If the function f(x) is defined on a closed interval[a, b], then the above argument only applies when a < c < b — not when c iseither of the endpoints a and b.

374

Page 391: CLP-1 Differential Calculus

APPLICATIONS OF DERIVATIVES 3.5 OPTIMISATION

Consider the function below

This function has only 1 maximum value (the middle green point in the graph)and 1 minimum value (the rightmost blue point), however it has 4 points at whichthe derivative is zero. In the small intervals around those points where the derivativeis zero, we can see that function is locally a maximum or minimum, even if it is notthe global maximum or minimum. We clearly need to be more careful distinguishingbetween these cases.

Definition 3.5.3

Let a ≤ b and let the function f(x) be defined for all x ∈ [a, b]. Now let a ≤ c ≤b, then

• We say that f(x) has a global (or absolute) minimum at x = c if f(x) ≥f(c) for all a ≤ x ≤ b.

• Similarly, we say that f(x) has a global (or absolute) maximum at x = cif f(x) ≤ f(c) for all a ≤ x ≤ b.

Now let a < c < b (note the strict inequalities), then

• We say that f(x) has a local minimum at x = c if there are a′ and b′

obeying a ≤ a′ < c < b′ ≤ b such that f(x) ≥ f(c) for all x obeyinga′ < x < b′. Note the strict inequalities in a′ < c < b′.

• Similarly, we say that f(x) has a local maximum at x = c if there are a′

and b′ obeying a ≤ a′ < c < b′ ≤ b such that f(x) ≤ f(c) for all x obeyinga′ < x < b′. Note the strict inequalities in a′ < c < b′.

The global maxima and minima of a function are called the global extrema ofthe function, while the local maxima and minima are called the local extrema.

Consider again the function we showed in the figure above

375

Page 392: CLP-1 Differential Calculus

APPLICATIONS OF DERIVATIVES 3.5 OPTIMISATION

It has 2 local maxima and 2 local minima. The global maximum occurs at themiddle green point (which is also a local maximum), while the global minimumoccurs at the rightmost blue point (which is not a local minimum).

Using the above definition we can summarise what we have learned above as thefollowing theorem 1:

Theorem 3.5.4

If a function f(x) has a local maximum or local minimum at x = c and if f ′(c)exists, then f ′(c) = 0.

• It is often (but not always) the case that, when f(x) has a local maximum atx = c, the function f(x) increases strictly as x approaches c from the left anddecreases strictly as x leaves c to the right. That is, f ′(x) > 0 for x just to the leftof c and f ′(x) < 0 for x just to the right of c. Then, it is often the case, becausef ′(x) is decreasing as x increases through c, that f ′′(c) < 0.

• Conversely, if f ′(c) = 0 and f ′′(c) < 0, then, just to the right of c f ′(x) mustbe negative, so that f(x) is decreasing, and just to the left of c f ′(x) must bepositive, so that f(x) is increasing. So f(x) has a local maximum at c.

• Similarly, it is often the case that, when f(x) has a local minimum at x = c,f ′(x) < 0 for x just to the left of c and f ′(x) > 0 for x just to the right of c andf ′′(x) > 0.

• Conversely, if f ′(c) = 0 and f ′′(c) > 0, then, just to the right of c f ′(x) mustbe positive, so that f(x) is increasing, and, just to the left of c f ′(x) must benegative, so that f(x) is decreasing. So f(x) has a local minimum at c.

1 This is one of several important mathematical contributions made by Pierre de Fermat, a Frenchgovernment lawyer and amateur mathematician, who lived in the first half of the seventeenthcentury.

376

Page 393: CLP-1 Differential Calculus

APPLICATIONS OF DERIVATIVES 3.5 OPTIMISATION

Theorem 3.5.5

If f ′(c) = 0 and f ′′(c) < 0, then f(x) has a local maximum at c.If f ′(c) = 0 and f ′′(c) > 0, then f(x) has a local minimum at c.Note the strict inequalities.

Theorem 3.5.4 says that, when f(x) has a local maximum or minimum at x = c, thereare two possibilities.

• The derivative f ′(c) = 0. This case is illustrated in the following figure.

Observe that, in this example, f ′(x) changes continuously from negative topositive at the local minimum, taking the value zero at the local minimum (thered dot).

• The derivative f ′(c) does not exist. This case is illustrated in the followingfigure.

Observe that, in this example, f ′(x) changes discontinuously from negative topositive at the local minimum (x = 0) and f ′(0) does not exist.

This theorem demonstrates that the points at which the derivative is zero or doesnot exist are very important. It simplifies the discussion that follows if we give thesepoints names.

377

Page 394: CLP-1 Differential Calculus

APPLICATIONS OF DERIVATIVES 3.5 OPTIMISATION

Definition 3.5.6

Let f(x) be a function and let c be a point in its domain. Then

• if f ′(c) exists and is zero we call x = c a critical point of the function, and

• if f ′(c) does not exist then we call x = c a singular point of the function.

Warning 3.5.7

Note that some people (and texts) will combine both of these cases and callx = c a critical point when either the derivative is zero or does not exist. Thereader should be aware of the lack of convention on this point a and should becareful to understand whether the more inclusive definition of critical point isbeing used, or if the text is using the more precise definition that distinguishescritical and singular points.

a No pun intended.

We’ll now look at a few simple examples involving local maxima and minima,critical points and singular points. Then we will move on to global maxima andminima.

Example 3.5.8 Local max and min of x3 − 6x.

In this example, we’ll look for local maxima and minima of the function f(x) =x3 − 6x on the interval −2 ≤ x ≤ 3.

• First compute the derivative

f ′(x) = 3x2 − 6.

Since this is a polynomial it is defined everywhere on the domain and so therewill not be any singular points. So we now look for critical points.

• To do so we look for zeroes of the derivative

f ′(x) = 3x2 − 6 = 3(x2 − 2) = 3(x−√

2)(x+√

2).

This derivative takes the value 0 at two different values of x. Namely x = c− =−√

2 and x = c+ =√

2. Here is a sketch of the graph of f(x).

378

Page 395: CLP-1 Differential Calculus

APPLICATIONS OF DERIVATIVES 3.5 OPTIMISATION

Example 3.5.8

From the figure we see that

f(x) has a local minimum at x = c+ (i.e. we have f(x) ≥ f(c+) wheneverx is close to c+) and

f(x) has a local maximum at x = c− (i.e. we have f(x) ≤ f(c−) wheneverx is close to c−) and

the global minimum of f(x), for x in the interval −2 ≤ x ≤ 3, is at x = c+

(i.e. we have f(x) ≥ f(c+) whenever −2 ≤ x ≤ 3) and

the global maximum of f(x), for x in the interval −2 ≤ x ≤ 3, is at x = 3(i.e. we have f(x) ≤ f(3) whenever −2 ≤ x ≤ 3).

• Note that we have carefully constructed this example to illustrate that theglobal maximum (or minimum) of a function on an interval may or may notalso be a local maximum (or minimum) of the function.

Example 3.5.9 Local max and min of x3.

In this example, we’ll look for local maxima and minima of the function f(x) = x3

on the interval −1 ≤ x ≤ 1.

• First compute the derivative:

f ′(x) = 3x2.

Again, this is a polynomial and so defined on all of the domain. The functionwill not have singular points, but may have critical points.

• The derivative is zero only when x = 0, so x = c = 0 is the only critical point ofthe function.

379

Page 396: CLP-1 Differential Calculus

APPLICATIONS OF DERIVATIVES 3.5 OPTIMISATION

Example 3.5.9

• The graph of f(x) is sketched below. From that sketch we see that f(x) hasneither a local maximum nor a local minimum at x = c despite the fact thatf ′(c) = 0 — we have f(x) < f(c) = 0 for all x < c = 0 and f(x) > f(c) = 0 forall x > c = 0.

• Note that this example has been constructed to illustrate that a critical point(or singular point) of a function need not be a local maximum or minimum for thefunction.

• Reread Theorem 3.5.4. It says a that, “if f(x) has a local maximum/minimumat x = c and if f is differentiable at x = c, then f ′(c) = 0”. It does not say that“if f ′(c) = 0 then f has a local maximum/minimum at x = c”.

a A very common error of logic that people make is “Affirming the consequent”. “If P then Q” istrue, then observe Q and conclude P — this is false. “If he is Shakespeare then he is dead” “Thatman is dead” “He must be Shakespeare”. Or you may have also seen someone use this reasoning:“If a person is a genius before their time then they are misunderstood.” “I am misunderstood”“So I must be a genius before my time.”.

Example 3.5.10 Local max and min of |x|.

In this example, we’ll look for local maxima and minima of the function

f(x) = |x| =x if x ≥ 0

−x if x < 0

on the interval −1 ≤ x ≤ 1.

380

Page 397: CLP-1 Differential Calculus

APPLICATIONS OF DERIVATIVES 3.5 OPTIMISATION

Example 3.5.10

• Again, start by computing the derivative (reread Example 2.2.10):

f ′(x) =

1 if x > 0

undefined if x = 0

−1 if x < 0

• This derivative never takes the value 0, so the function does not have any criticalpoints. However the derivative does not exist at the point x = 0, so that pointis a singular point.

• Here is a sketch of the graph of f(x).

From the figure we see that f(x) has a local (and in fact global) minimum atx = 0 despite the fact that f ′(0) is not a critical point.

• Reread Theorem 3.5.4 yet again. It says that, “if f(x) has a local maxi-mum/minimum at x = c and if f is differentiable at x = c, then f ′(c) = 0”.It says nothing about what happens at points where the derivative does notexist. Indeed that is why we have to consider both critical points and singularpoints when we look for maxima and minima.

3.5.2 tt Finding Global Maxima and Minima

We now have a technique for finding local maxima and minima — just look forvalues of x for which either f ′(x) = 0 or f ′(x) does not exist. What about findingglobal maxima and minima? We’ll start by stating explicitly that, under appropriatehypotheses, global maxima and minima are guaranteed to exist.

381

Page 398: CLP-1 Differential Calculus

APPLICATIONS OF DERIVATIVES 3.5 OPTIMISATION

Theorem 3.5.11

Let the function f(x) be defined and continuous on the closed, finite interval a

−∞ < a ≤ x ≤ b < ∞. Then f(x) attains a maximum and a minimum at leastonce. That is, there exist numbers a ≤ xm, xM ≤ b such that

f(xm) ≤ f(x) ≤ f(xM) for all a ≤ x ≤ b

a The hypotheses that f(x) be continuous and that the interval be finite and closed are allessential. We suggest that you find three functions f1(x), f2(x) and f3(x) with f1 definedbut not continuous on 0 ≤ x ≤ 1, f2 defined and continuous on −∞ < x < ∞, and f3defined and continuous on 0 < x < 1, and with none of f1, f2 and f3 attaining either aglobal maximum or a global minimum.

So let’s again consider the question

Suppose that the maximum (or minimum) value of f(x), for a ≤ x ≤ b, isf(c). What does that tell us about c?

If c obeys a < c < b (note the strict inequalities), then f has a local maximum (orminimum) at x = c and Theorem 3.5.4 tells us that either f ′(c) = 0 or f ′(c) does notexist. The only other place that a maximum or minimum can occur are at the endsof the interval. We can summarise this as:

Theorem 3.5.12

If f(x) has a global maximum or global minimum, for a ≤ x ≤ b, at x = c thenthere are 3 possibilities. Either

• f ′(c) = 0, or

• f ′(c) does not exist, or

• c = a or c = b.

That is, a global maximum or minimum must occur either at a critical point, asingular point or at the endpoints of the interval.

This theorem provides the basis for a method to find the maximum and minimumvalues of f(x) for a ≤ x ≤ b:

382

Page 399: CLP-1 Differential Calculus

APPLICATIONS OF DERIVATIVES 3.5 OPTIMISATION

Corollary 3.5.13

Let f(x) be a function on the interval a ≤ x ≤ b. Then to find the globalmaximum and minimum of the function:

• Make a list of all values of c, with a ≤ c ≤ b, for which

f ′(c) = 0, or

f ′(c) does not exist, or

c = a or c = b.

That is — compute the function at all the critical points, singular points,and endpoints.

• Evaluate f(c) for each c in that list. The largest (or smallest) of thosevalues is the largest (or smallest) value of f(x) for a ≤ x ≤ b.

Let’s now demonstrate how to use this strategy. The function in this first exampleis not too simple — but it is a good example of a function that contains both a singularpoint and a critical point.

Example 3.5.14 Find max and min of 2x5/3 + 3x2/3.

Find the largest and smallest values of the function f(x) = 2x5/3 +3x2/3 for−1 ≤ x ≤1.Solution We will apply the method in Corollary 3.5.13. It is perhaps easiest to findthe values at the endpoints of the intervals and then move on to the values at anycritical or singular points.

• Before we get into things, notice that we can rewrite the function by factoringit:

f(x) = 2x5/3 + 3x2/3 = x2/3 · (2x+ 3)

• Let’s compute the function at the endpoints of the interval:

f(1) = 2 + 3 = 5

f(−1) = 2 · (−1)5/3 + 3 · (−1)2/3 = −2 + 3 = 1

• To compute the function at the critical and singular points we first need to findthe derivative:

f ′(x) = 2 · 5

3x2/3 + 3 · 2

3x−1/3

383

Page 400: CLP-1 Differential Calculus

APPLICATIONS OF DERIVATIVES 3.5 OPTIMISATION

=10

3x2/3 + 2x−1/3

=10x+ 6

3x1/3

• Notice that the numerator and denominator are defined for all x. The onlyplace the derivative is undefined is when the denominator is zero. Hence theonly singular point is at x = 0. The corresponding function value is

f(0) = 0

• To find the critical points we need to solve f ′(x) = 0:

0 =10x+ 6

3x1/3

Hence we must have 10x = −6 or x = −3/5. The corresponding function valueis

f(x) = x2/3 · (2x+ 3) recall this from above, then

f(−3/5) = (−3/5)2/3 ·(

2 · −3

5+ 3

)=

(9

25

)1/3

· −6 + 15

5

=

(9

25

)1/3

· 9

5≈ 1.28

Note that if we do not want to approximate the root (if, for example, we do nothave a calculator handy), then we can also write

f(−3/5) =

(9

25

)1/3

· 9

5

=

(9

25

)1/3

· 9

25· 5

= 5 ·(

9

25

)4/3

Since 0 < 9/25 < 1, we know that 0 <(

925

)4/3< 1, and hence

0 < f(−3/5) = 5 ·(

9

25

)4/3

< 5.

• We summarise our work in this table

384

Page 401: CLP-1 Differential Calculus

APPLICATIONS OF DERIVATIVES 3.5 OPTIMISATION

Example 3.5.14

c −35

0 −1 1

type critical point singular point endpoint endpoint

f(c) 95

3

√925≈ 1.28 0 1 5

• The largest value of f in the table is 5 and the smallest value of f in the table is0.

• Thus on the interval −1 ≤ x ≤ 1 the global maximum of f is 5, and is taken atx = 1, while the global minimum value of f(x) is 0, and is taken at x = 0.

• For completeness we also sketch the graph of this function on the same inter-val.

Later (in Section 3.6) we will see how to construct such a sketch without usinga calculator or computer.

3.5.3 ttMax/Min Examples

As noted at the beginning of this section, the problem of finding maxima and minimais a very important application of differential calculus in the real world. We now turnto a number of examples of this process. But to guide the reader we will describe ageneral procedure to follow for these problems.

1 Read — read the problem carefully. Work out what information is given in thestatement of the problem and what we are being asked to compute.

2 Diagram — draw a diagram. This will typically help you to identify what youknow about the problem and what quantities you need to work out.

385

Page 402: CLP-1 Differential Calculus

APPLICATIONS OF DERIVATIVES 3.5 OPTIMISATION

3 Variables — assign variables to the quantities in the problem along with theirunits. It is typically a good idea to make sensible choices of variable names: Afor area, h for height, t for time etc.

4 Relations — find relations between the variables. By now you should know thequantity we are interested in (the one we want to maximise or minimise) andwe need to establish a relation between it and the other variables.

5 Reduce — the relation down to a function of one variable. In order to applythe calculus we know, we must have a function of a single variable. To do thiswe need to use all the information we have to eliminate variables. We shouldalso work out the domain of the resulting function.

6 Maximise or minimise — we can now apply the methods of Corollary 3.5.13to find the maximum or minimum of the quantity we need (as the problemdictates).

7 Be careful — make sure your answer makes sense. Make sure quantities arephysical. For example, lengths and areas cannot be negative.

8 Answer the question — be sure your answer really answers the question askedin the problem.

Let us start with a relatively simple problem:

Example 3.5.15 Constructing a container of maximal volume.

A closed rectangular container with a square base is to be made from two differentmaterials. The material for the base costs $5 per square meter, while the material forthe other five sides costs $1 per square meter. Find the dimensions of the containerwhich has the largest possible volume if the total cost of materials is $72.Solution We can follow the steps we outlined above to find the solution.

• We need to determine the area of the two types of materials used and the cor-responding total cost.

• Draw a picture of the box.

386

Page 403: CLP-1 Differential Calculus

APPLICATIONS OF DERIVATIVES 3.5 OPTIMISATION

The more useful picture is the unfolded box on the right.

• In the picture we have already introduced two variables. The square base hasside-length b metres and it has height h metres. Let the area of the base be Aband the area of the other fives sides be As (both in m2), and the total cost be C(in dollars). Finally let the volume enclosed be V m3.

• Some simple geometry tells us that

Ab = b2

As = 4bh+ b2

V = b2h

C = 5 · Ab + 1 · As = 5b2 + 4bh+ b2 = 6b2 + 4bh.

• To eliminate one of the variables we use the fact that the total cost is $72.

C = 6b2 + 4bh = 72 rearrange4bh = 72− 6b2 isolate h

h =72− 6b2

4b=

3

2· 12− b2

b

Substituting this into the volume gives

V = b2h =3b

2(12− b2) = 18b− 3

2b3

Now note that since b is a length it cannot be negative, so b ≥ 0. Further sincevolume cannot be negative, we must also have

12− b2 ≥ 0

and so b ≤√

12.

• Now we can apply Corollary 3.5.13 on the above expression for the volumewith 0 ≤ b ≤

√12. The endpoints give:

V (0) = 0

V (√

12) = 0

The derivative is

V ′(b) = 18− 9b2

2

Since this is a polynomial there are no singular points. However we can solveV ′(b) = 0 to find critical points:

18− 9b2

2= 0 divide by 9 and multiply by 2

387

Page 404: CLP-1 Differential Calculus

APPLICATIONS OF DERIVATIVES 3.5 OPTIMISATION

Example 3.5.15

4− b2 = 0

Hence b = ±2. Thus the only critical point in the domain is b = 2. The corre-sponding volume is

V (2) = 18× 2− 3

2× 23

= 36− 12 = 24.

So by Corollary 3.5.13, the maximum volume is when 24 when b = 2 and

h =3

2· 12− b2

b=

3

2

12− 4

2= 6.

• All our quantities make sense; lengths, areas and volumes are all non-negative.

• Checking the question again, we see that we are asked for the dimensions ofthe container (rather than its volume) so we can answer with

The container with dimensions 2×2×6mwill be the largest possible.

Example 3.5.16 Constructing another box.

A rectangular sheet of cardboard is 6 inches by 9 inches. Four identical squaresare cut from the corners of the cardboard, as shown in the figure below, and theremaining piece is folded into an open rectangular box. What should the size of thecut out squares be in order to maximize the volume of the box?Solution This one is quite similar to the previous one, so we perhaps don’t need togo into so much detail.

• After reading carefully we produce the following picture:

388

Page 405: CLP-1 Differential Calculus

APPLICATIONS OF DERIVATIVES 3.5 OPTIMISATION

Example 3.5.16

• Let the height of the box be x inches, and the base be `×w inches. The volumeof the box is then V cubic inches.

• Some simple geometry tells us that ` = 9− 2x,w = 6− 2x and so

V = x(9− 2x)(6− 2x)cubic inches= 54x− 30x2 + 4x3.

Notice that since all lengths must be non-negative, we must have

x, `, w ≥ 0

and so 0 ≤ x ≤ 3 (if x > 3 then w < 0).

• We can now apply Corollary 3.5.13. First the endpoints of the interval give

V (0) = 0 V (3) = 0

The derivative is

V ′(x) = 54− 60x+ 12x2

= 6(9− 10x+ 2x2)

Since this is a polynomial there are no singular points. To find critical pointswe solve V ′(x) = 0 to get

x± =10±

√100− 4× 2× 9

4

=10±

√28

4=

10± 2√

7

4=

5±√

7

2We can then use a calculator to approximate

x+ ≈ 3.82 x− ≈ 1.18.

So x− is inside the domain, while x+ lies outside.

Alternatively a , we can bound x± by first noting that 2 ≤√

7 ≤ 3. From thiswe know that

1 =5− 3

2≤ x− =

5−√

7

2≤ 5− 2

2= 1.5

3.5 =5 + 2

2≤ x+ =

5 +√

7

2≤ 5 + 3

2= 4

• Since the volume is zero when x = 0, 3, it must be the case that the volume ismaximised when x = x− = 5−

√7

2.

• Notice that since 0 < x− < 3 we know that the other lengths are positive, soour answer makes sense. Further, the question only asks for the length x andnot the resulting volume so we have answered the question.

a Say if we do not have a calculator to hand, or your instructor insists that the problem be donewithout one.

389

Page 406: CLP-1 Differential Calculus

APPLICATIONS OF DERIVATIVES 3.5 OPTIMISATION

There is a new wrinkle in the next two examples. Each involves finding the min-imum value of a function f(x) with x running over all real numbers, rather thanjust over a finite interval as in Corollary 3.5.13. Both in Example 3.5.18 and in Ex-ample 3.5.19 the function f(x) tends to +∞ as x tends to either +∞ or −∞. So theminimum value of f(x) will be achieved for some finite value of x, which will be alocal minimum as well as a global minimum.

Theorem 3.5.17

Let f(x) be defined and continuous for all −∞ < x < ∞. Let c be a finite realnumber.

a If limx→+∞

f(x) = +∞ and limx→−∞

f(x) = +∞ and if f(x) has a global mini-

mum at x = c, then there are 2 possibilities. Either

• f ′(c) = 0, or

• f ′(c) does not exist

That is, a global minimum must occur either at a critical point or at asingular point.

b If limx→+∞

f(x) = −∞ and limx→−∞

f(x) = −∞ and if f(x) has a global maxi-

mum at x = c, then there are 2 possibilities. Either

• f ′(c) = 0, or

• f ′(c) does not exist

That is, a global maximum must occur either at a critical point or at asingular point.

Example 3.5.18 How far from a point to a line.

Find the point on the line y = 6− 3x that is closest to the point (7, 5).Solution In this problem

• A simple picture

390

Page 407: CLP-1 Differential Calculus

APPLICATIONS OF DERIVATIVES 3.5 OPTIMISATION

• Some notation is already given to us. Let a point on the line have coordinates(x, y), and we do not need units. And let ` be the distance from the point (x, y)to the point (7, 5).

• Since the points are on the line the coordinates (x, y) must obey

y = 6− 3x

Notice that x and y have no further constraints. The distance ` is given by

`2 = (x− 7)2 + (y − 5)2

• We can now eliminate the variable y:

`2 = (x− 7)2 + (y − 5)2

= (x− 7)2 + (6− 3x− 5)2 = (x− 7)2 + (1− 3x)2

= x2 − 14x+ 49 + 1− 6x+ 9x2 = 10x2 − 20x+ 50

= 10(x2 − 2x+ 5)

` =√

10 ·√x2 − 2x+ 5

Notice that as x→ ±∞ the distance `→ +∞.

• We can now apply Theorem 3.5.17

Since the distance is defined for all real x, we do not have to check theendpoints of the domain — there are none.

391

Page 408: CLP-1 Differential Calculus

APPLICATIONS OF DERIVATIVES 3.5 OPTIMISATION

Example 3.5.18

Form the derivative:

d`

dx=√

102x− 2

2√x2 − 2x+ 5

It is zero when x = 1, and undefined if x2 − 2x+ 5 < 0. However, since

x2 − 2x+ 5 = (x2 − 2x+ 1) + 4 = (x− 1)2︸ ︷︷ ︸≥0

+4

we know that x2 − 2x + 5 ≥ 4. Thus the function has no singular pointsand the only critical point occurs at x = 1. The corresponding functionvalue is then

`(1) =√

10√

1− 2 + 5 = 2√

10.

Thus the minimum value of the distance is ` = 2√

10 and occurs at x = 1.

• This answer makes sense — the distance is not negative.

• The question asks for the point that minimises the distance, not that minimumdistance. Hence the answer is x = 1, y = 6− 3 = 3. I.e.

The point that minimises the distance is (1, 3).

Notice that we can make the analysis easier by observing that the point that min-imises the distance also minimises the squared-distance. So that instead of minimis-ing the function `, we can just minimise `2:

`2 = 10(x2 − 2x+ 5)

The resulting algebra is a bit easier and we don’t have to hunt for singular points.

Example 3.5.19 How far from a point to a curve.

Find the minimum distance from (2, 0) to the curve y2 = x2 + 1.Solution This is very much like the previous question.

• After reading the problem carefully we can draw a picture

392

Page 409: CLP-1 Differential Calculus

APPLICATIONS OF DERIVATIVES 3.5 OPTIMISATION

Example 3.5.19

• In this problem we do not need units and the variables x, y are supplied. Wedefine the distance to be ` and it is given by

`2 = (x− 2)2 + y2.

As noted in the previous problem, we will minimise the squared-distance sincethat also minimises the distance.

• Since x, y satisfy y2 = x2 + 1, we can write the distance as a function of x:

`2 = (x− 2)2 + y2 = (x− 2)2 + (x2 + 1)

Notice that as x→ ±∞ the squared-distance `2 → +∞.

• Since the squared-distance is a polynomial it will not have any singular points,only critical points. The derivative is

d

dx`2 = 2(x− 2) + 2x = 4x− 4

so the only critical point occurs at x = 1.

• When x = 1, y = ±√

2 and the distance is

`2 = (1− 2)2 + (1 + 1) = 3 ` =√

3

and thus the minimum distance from the curve to (2, 0) is√

3.

393

Page 410: CLP-1 Differential Calculus

APPLICATIONS OF DERIVATIVES 3.5 OPTIMISATION

Example 3.5.20 Constructing a trough.

A water trough is to be constructed from a metal sheet of width 45 cm by bendingup one third of the sheet on each side through an angle θ. Which θ will allow thetrough to carry the maximum amount of water?Solution Clearly 0 ≤ θ ≤ π, so we are back in the domain a of Corollary 3.5.13.

• After reading the problem carefully we should realise that it is really asking usto maximise the cross-sectional area. A figure really helps.

• From this we are led to define the height h cm and cross-sectional area A cm2.Both are functions of θ.

h = 15 sin θ

while the area can be computed as the sum of the central 15× h rectangle, plustwo triangles. Each triangle has height h and base 15 cos θ. Hence

A = 15h+ 2 · 1

2· h · 15 cos θ

= 15h (1 + cos θ)

• Since h = 15 sin θ we can rewrite the area as a function of just θ:

A(θ) = 225 sin θ (1 + cos θ)

where 0 ≤ θ ≤ π.

• Now we use Corollary 3.5.13. The ends of the interval give

A(0) = 225 sin 0(1 + cos 0) = 0

A(π) = 225 sin π(1 + cos π) = 0

The derivative is

A′(θ) = 225 cos θ · (1 + cos θ) + 225 sin θ · (− sin θ)

= 225[cos θ + cos2 θ − sin2 θ

]recall sin2 θ = 1− cos2 θ

= 225[cos θ + 2 cos2 θ − 1

]

394

Page 411: CLP-1 Differential Calculus

APPLICATIONS OF DERIVATIVES 3.5 OPTIMISATION

Example 3.5.20

This is a continuous function, so there are no singular points. However we canstill hunt for critical points by solving A′(θ) = 0. That is

2 cos2 θ + cos θ − 1 = 0 factor carefully(2 cos θ − 1)(cos θ + 1) = 0

Hence we must have cos θ = −1 or cos θ = 12. On the domain 0 ≤ θ ≤ π, this

means θ = π/3 or θ = π.

A(π) = 0

A(π/3) = 225 sin(π/3)(1 + cos(π/3))

= 225 ·√

3

2·(

1 +1

2

)= 225 · 3

√3

4≈ 292.28

• Thus the cross-sectional area is maximised when θ =π

3.

a Again, no pun intended.

Example 3.5.21 Closest and farthest points on a curve to a given point.

Find the points on the ellipse x2

4+ y2 = 1 that are nearest to and farthest from the

point (1, 0).Solution While this is another distance problem, the possible values of x, y arebounded, so we need Corollary 3.5.13 rather than Theorem 3.5.17.

• We start by drawing a picture:

395

Page 412: CLP-1 Differential Calculus

APPLICATIONS OF DERIVATIVES 3.5 OPTIMISATION

Example 3.5.21

• Let ` be the distance from the point (x, y) on the ellipse to the point (1, 0). Aswas the case above, we will maximise the squared-distance.

`2 = (x− 1)2 + y2.

• Since (x, y) lie on the ellipse we have

x2

4+ y2 = 1

Note that this also shows that −2 ≤ x ≤ 2 and −1 ≤ y ≤ 1.

Isolating y2 and substituting this into our expression for `2 gives

`2 = (x− 1)2 + 1− x2/4︸ ︷︷ ︸=y2

.

• Now we can apply Corollary 3.5.13. The endpoints of the domain give

`2(−2) = (−2− 1)2 + 1− (−2)2/4 = 32 + 1− 1 = 9

`2(2) = (2− 1)2 + 1− 22/4 = 1 + 1− 1 = 1

The derivative is

d

dx`2 = 2(x− 1)− x/2 =

3x

2− 2

Thus there are no singular points, but there is a critical point at x = 4/3. Thecorresponding squared-distance is

`2(4/3) =

(4

3− 1

)2

+ 1− (4/3)2

4

= (1/3)2 + 1− (4/9) = 6/9 = 2/3.

• To summarise (and giving distances and coordinates of points):

x (x, y) `

−2 (−2, 0) 343

(43,±√

53

) √23

2 (2, 0) 1

The point of maximum distance is (−2, 0), and the point of minimum distanceis(

43,±√

53

).

396

Page 413: CLP-1 Differential Calculus

APPLICATIONS OF DERIVATIVES 3.5 OPTIMISATION

Example 3.5.22 Largest rectangle inside a triangle.

Find the dimensions of the rectangle of largest area that can be inscribed in an equi-lateral triangle of side a if one side of the rectangle lies on the base of the triangle.Solution Since the rectangle must sit inside the triangle, its dimensions are boundedand we will end up using Corollary 3.5.13.

• Carefully draw a picture:

We have drawn (on the left) the triangle in the xy-plane with its base on thex-axis. The base has been drawn running from (−a/2, 0) to (a/2, 0) so its centrelies at the origin. A little Pythagoras (or a little trigonometry) tells us that theheight of the triangle is

√a2 − (a/2)2 =

√3

2· a = a · sin π

3

Thus the vertex at the top of the triangle lies at(

0,√

32· a)

.

• If we construct a rectangle that does not touch the sides of the triangle, then wecan increase the dimensions of the rectangle until it touches the triangle andso make its area larger. Thus we can assume that the two top corners of therectangle touch the triangle as drawn in the right-hand figure above.

• Now let the rectangle be 2x wide and y high. And let A denote its area. Clearly

A = 2xy.

where 0 ≤ x ≤ a/2 and 0 ≤ y ≤√

32a.

• Our construction means that the top-right corner of the rectangle will havecoordinates (x, y) and lie on the line joining the top vertex of the triangle at(0,√

3a/2) to the bottom-right vertex at (a/2, 0). In order to write the area as a

397

Page 414: CLP-1 Differential Calculus

APPLICATIONS OF DERIVATIVES 3.5 OPTIMISATION

Example 3.5.22

function of x alone, we need the equation for this line since it will tell us howto write y as a function of x. The line has slope

slope =

√3a/2− 0

0− a/2 = −√

3.

and passes through the point (0,√

3a/2), so any point (x, y) on that line satis-fies:

y = −√

3x+

√3

2a.

• We can now write the area as a function of x alone

A(x) = 2x

(−√

3x+

√3

2a

)=√

3x(a− 2x).

with 0 ≤ x ≤ a/2.

• The ends of the domain give:

A(0) = 0 A(a/2) = 0.

The derivative is

A′(x) =√

3 (x · (−2) + 1 · (a− 2x)) =√

3(a− 4x).

Since this is a polynomial there are no singular points, but there is a criticalpoint at x = a/4. There

A(a/4) =√

3 · a4· (a− a/2) =

√3 · a

2

8.

y = −√

3 · (a/4) +

√3

2a =√

3 · a4.

• Checking the question again, we see that we are asked for the dimensionsrather than the area, so the answer is 2x× y:

The largest such rectangle has dimensions a2×√

3a4

.

This next one is a good physics example. In it we will derive Snell’s Law 2 from

2 Snell’s law is named after the Dutch astronomer Willebrord Snellius who derived it in around1621, though it was first stated accurately in 984 by Ibn Sahl.

398

Page 415: CLP-1 Differential Calculus

APPLICATIONS OF DERIVATIVES 3.5 OPTIMISATION

Fermat’s principle 3 .

Example 3.5.23 Snell’s law.

Consider the figure below which shows the trajectory of a ray of light as it passesthrough two different mediums (say air and water).

Let ca be the speed of light in air and cw be the speed of light in water. Fermat’sprinciple states that a ray of light will always travel along a path that minimises thetime taken. So if a ray of light travels from P (in air) to Q (in water) then it will“choose” the point O (on the interface) so as to minimise the total time taken. Usethis idea to show Snell’s law,

sin θisin θr

=cacw

where θi is the angle of incidence and θr is the angle of refraction (as illustrated inthe figure above).Solution This problem is a little more abstract than the others we have examined,but we can still apply Theorem 3.5.17.

• We are given a figure in the statement of the problem and it contains all therelevant points and angles. However it will simplify things if we decide on acoordinate system. Let’s assume that the point O lies on the x-axis, at coor-dinates (x, 0). The point P then lies above the axis at (XP ,+YP ), while Q liesbelow the axis at (XQ,−YQ). This is drawn below.

3 Named after Pierre de Fermat who described it in a letter in 1662. The beginnings of the idea,however, go back as far as Hero of Alexandria in around 60CE. Hero is credited with manyinventions including the first vending machine, and a precursor of the steam engine called anaeolipile.

399

Page 416: CLP-1 Differential Calculus

APPLICATIONS OF DERIVATIVES 3.5 OPTIMISATION

• The statement of Snell’s law contains terms sin θi and sin θr, so it is a good ideafor us to see how to express these in terms of the coordinates we have justintroduced:

sin θi =opposite

hypotenuse=

(x−XP )√(XP − x)2 + Y 2

P

sin θr =opposite

hypotenuse=

(XQ − x)√(XQ − x)2 + Y 2

Q

• Let `P denote the distance PO, and `Q denote the distance OQ. Then we have

`P =√

(XP − x)2 + Y 2P

`Q =√

(XQ − x)2 + Y 2Q

If we then denote the total time taken by T , then

T =`Pca

+`Qcw

=1

ca

√(XP − x)2 + Y 2

P +1

cw

√(XQ − x)2 + Y 2

Q

which is written as a function of x since all the other terms are constants.

• Notice that as x→ +∞ or x→ −∞ the total time T →∞ and so we can applyTheorem 3.5.17. The derivative is

dT

dx=

1

ca

−2(XP − x)

2√

(XP − x)2 + Y 2P

+1

cw

−2(XQ − x)

2√

(XQ − x)2 + Y 2Q

Notice that the terms inside the square-roots cannot be zero or negative sincethey are both sums of squares and YP , YQ > 0. So there are no singular points,but there is a critical point when T ′(x) = 0, namely when

0 =1

ca

XP − x√(XP − x)2 + Y 2

P

+1

cw

XQ − x√(XQ − x)2 + Y 2

Q

400

Page 417: CLP-1 Differential Calculus

APPLICATIONS OF DERIVATIVES 3.5 OPTIMISATION

Example 3.5.23

=− sin θica

+sin θrcw

Rearrange this to get

sin θica

=sin θrcw

move sines to one side

sin θisin θr

=cacw

which is exactly Snell’s law.

Example 3.5.24 Finding the best viewing angle.

The Statue of Liberty has height 46m and stands on a 47m tall pedestal. How farfrom the statue should an observer stand to maximize the angle subtended by thestatue at the observer’s eye, which is 1.5m above the base of the pedestal?Solution Obviously if we stand too close then all the observer sees is the pedestal,while if they stand too far then everything is tiny. The best spot for taking a photo-graph is somewhere in between.

• Draw a careful picture a

and we can put in the relevant lengths and angles.

401

Page 418: CLP-1 Differential Calculus

APPLICATIONS OF DERIVATIVES 3.5 OPTIMISATION

• The height of the statue is h = 46m, and the height of the pedestal (above theeye) is p = 47− 1.5 = 45.5m. The horizontal distance from the statue to the eyeis x. There are two relevant angles. First θ is the angle subtended by the statue,while ϕ is the angle subtended by the portion of the pedestal above the eye.

• Some trigonometry gives us

tanϕ =p

x

tan(ϕ+ θ) =p+ h

x

Thus

ϕ = arctanp

x

ϕ+ θ = arctanp+ h

x

and so

θ = arctanp+ h

x− arctan

p

x.

• If we allow the viewer to stand at any point in front of the statue, then 0 ≤ x <∞. Further observe that as x→∞ or x→ 0 the angle θ → 0, since

limx→0

arctanp+ h

x=π

2

limx→0

arctanp

x=π

2

Clearly the largest value of θ will be strictly positive and so has to be taken forsome 0 < x <∞. (Note the strict inequalities.) This x will be a local maximumas well as a global maximum. As θ is not singular at any 0 < x < ∞, we needonly search for critical points.

A careful application of the chain rule shows that the derivative is

dx=

1

1 + (p+hx

)2·(−(p+ h)

x2

)− 1

1 + ( px)2·(−px2

)=

−(p+ h)

x2 + (p+ h)2+

p

x2 + p2

So a critical point occurs when

(p+ h)

x2 + (p+ h)2=

p

x2 + p2cross multiply

(p+ h)(x2 + p2) = p(x2 + (p+ h)2) collect x terms

402

Page 419: CLP-1 Differential Calculus

APPLICATIONS OF DERIVATIVES 3.5 OPTIMISATION

Example 3.5.24

x2(p+ h− p) = p(p+ h)2 − p2(p+ h) clean uphx2 = p(p+ h)(p+ h− p) = ph(p+ h)

cancel common factorsx2 = p(p+ h)

x = ±√p(p+ h) ≈ ±64.9m

• Thus the best place to stand approximately 64.9m in front or behind the statue.At that point θ ≈ 0.348 radians or 19.9.

a And make some healthy use of public domain clip art.

Example 3.5.25 Moving objects around corners.

Find the length of the longest rod that can be carried horizontally (no tilting allowed)from a corridor 3m wide into a corridor 2m wide. The two corridors are perpendic-ular to each other.Solution

• Suppose that we are carrying the rod around the corner, then if the rod is aslong as possible it must touch the corner and the outside walls of both corri-dors. A picture of this is show below.

You can see that this gives rise to two similar triangles, one inside each corridor.Also the maximum length of the rod changes with the angle it makes with thewalls of the corridor.

• Suppose that the angle between the rod and the inner wall of the 3m corridoris θ, as illustrated in the figure above. At the same time it will make an angleof π

2− θ with the outer wall of the 2m corridor. Denote by `1(θ) the length of

the part of the rod forming the hypotenuse of the upper triangle in the figure

403

Page 420: CLP-1 Differential Calculus

APPLICATIONS OF DERIVATIVES 3.5 OPTIMISATION

above. Similarly, denote by `2(θ) the length of the part of the rod forming thehypotenuse of the lower triangle in the figure above. Then

`1(θ) =3

sin θ`2(θ) =

2

cos θ

and the total length is

`(θ) = `1(θ) + `2(θ) =3

sin θ+

2

cos θ

where 0 ≤ θ ≤ π2.

• The length of the longest rod we can move through the corridor in this way isthe minimum of `(θ). Notice that `(θ) is not defined at θ = 0, π

2. Indeed we find

that as θ → 0+ or θ → π2−, the length ` → +∞. (You should be able to picture

what happens to our rod in those two limits). Clearly the minimum allowed`(θ) is going to be finite and will be achieved for some 0 < θ < π

2(note the strict

inequalities) and so will be a local minimum as well as a global minimum. Sowe only need to find zeroes of `′(θ).

Differentiating ` gives

d`

dθ= −3 cos θ

sin2 θ+

2 sin θ

cos2 θ=−3 cos3 θ + 2 sin3 θ

sin2 θ cos2 θ.

This does not exist at θ = 0, π2

(which we have already analysed) but does existat every 0 < θ < π

2and is equal to zero when the numerator is zero. Namely

when

2 sin3 θ = 3 cos3 θ divide by cos3 θ

2 tan3 θ = 3

tan θ =3

√3

2

• From this we can recover sinθ and cosθ, without having to compute θ itself. Wecan, for example, construct a right-angle triangle with adjacent length 3

√2 and

opposite length 3√

3 (so that tan θ = 3√

3/2):

404

Page 421: CLP-1 Differential Calculus

APPLICATIONS OF DERIVATIVES 3.5 OPTIMISATION

Example 3.5.25

It has hypotenuse√

32/3 + 22/3, and so

sin θ =31/3

√32/3 + 22/3

cos θ =21/3

√32/3 + 22/3

Alternatively could use the identities:

1 + tan2 θ = sec2 θ 1 + cot2 θ = csc2 θ

to obtain expressions for 1/ cos θ and 1/ sin θ.

• Using the above expressions for sin θ, cos θ we find the minimum of ` (which isthe longest rod that we can move):

` =3

sin θ+

2

cos θ=

33√3√

223 +3

23

+23√2√

223 +3

23

=

√2

23 + 3

23

[3

23 + 2

23

]=[2

23 + 3

23

] 32 ≈ 7.02m

3.5.4 tt Exercises

ttt Exercises for § 3.5.1

Exercises — Stage 1

1. Identify every critical point and every singular point of f(x) shown onthe graph below. Which correspond to local extrema?

405

Page 422: CLP-1 Differential Calculus

APPLICATIONS OF DERIVATIVES 3.5 OPTIMISATION

x

y

y = f(x)

2. Identify every critical point and every singular point of f(x) on thegraph below. Which correspond to local extrema? Which correspondto global extrema over the interval shown?

x

y

y = f(x)

3. Draw a graph y = f(x) where a f(2) is a local maximum, but it is not aglobal maximum.

Exercises — Stage 2

4. Suppose f(x) =x2 − 10

x− 5.

a Find all critical points.

b Find all singular points.

406

Page 423: CLP-1 Differential Calculus

APPLICATIONS OF DERIVATIVES 3.5 OPTIMISATION

c What are the possible points where local extrema of f(x) may ex-ist?

Exercises — Stage 3

5. Below are a number of curves, all of which have a singular point at x =2. For each, label whether x = 2 is a local maximum, a local minimum,or neither.

x

y

2

x

y

2

x

y

2

x

y

2

6. Draw a graph y = f(x) where f(2) is a local maximum, but x = 2 is not acritical point.

7.f(x) =

√|(x− 5)(x+ 7)|

Find all critical points and all singular points of f(x). You do not have tospecify whether a point is critical or singular.

8. Suppose f(x) is the constant function f(x) = 4. What are the critical pointsand singular points of f(x)? What are its local and global maxima andminima?

ttt Exercises for § 3.5.2

Exercises — Stage 1

1. Sketch a function f(x) such that:

• f(x) is defined over all real numbers

• f(x) has a global max but no global min.

2. Sketch a function f(x) such that:

• f(x) is defined over all real numbers

• f(x) is always positive

• f(x) has no global max and no global min.

407

Page 424: CLP-1 Differential Calculus

APPLICATIONS OF DERIVATIVES 3.5 OPTIMISATION

3. Sketch a function f(x) such that:

• f(x) is defined over all real numbers

• f(x) has a global minimum at x = 5

• f(x) has a global minimum at x = −5, too.

Exercises — Stage 2

4. f(x) = x2 + 6x− 10. Find all global extrema on the interval [−5, 5]

5. f(x) =2

3x3−2x2−30x+7. Find all global extrema on the interval [−4, 0].

ttt Exercises for § 3.5.3

Exercises — Stage 1 For Questions 3.5.4.1 through 3.5.4.3, the quantity to optimizeis already given to you as a function of a single variable.For Questions 3.5.4.4 and3.5.4.5, you can decide whether a critical point is a local extrema by considering thederivative of the function.For Questions 3.5.4.6 through 3.5.4.13, you will have tofind an expression for the quantity you want to optimize as a function of a singlevariable.

1. *. Find the global maximum and the global minimum for f(x) = x5−5x+2on the interval [−2, 0].

2. *. Find the global maximum and the global minimum for f(x) = x5−5x−10on the interval [0, 2].

3. *. Find the global maximum and the global minimum for f(x) = 2x3−6x2−2 on the interval [1, 4].

4. *. Consider the function h(x) = x3 − 12x + 4. What are the coordinatesof the local maximum of h(x)? What are the coordinates of the localminimum of h(x)?

5. *. Consider the function h(x) = 2x3 − 24x + 1. What are the coordinates ofthe local maximum of h(x)? What are the coordinates of the local minimumof h(x)?

6. *. You are in a dune buggy at a point P in the desert, 12 km due southof the nearest point A on a straight east-west road. You want to get to atown B on the road 18 km east of A. If your dune buggy can travel at anaverage speed of 15 km/hr through the desert and 30 km/hr along the

408

Page 425: CLP-1 Differential Calculus

APPLICATIONS OF DERIVATIVES 3.5 OPTIMISATION

road, towards what point Q on the road should you head to minimizeyour travel time from P to B?

A Q B

P

12 km

7. *. A closed three dimensional box is to be constructed in such a way thatits volume is 4500 cm3. It is also specified that the length of the base is 3times the width of the base. Find the dimensions of the box that satisfiesthese conditions and has the minimum possible surface area. Justify youranswer.

8. *. A closed rectangular container with a square base is to be made fromtwo different materials. The material for the base costs $5 per squaremetre, while the material for the other five sides costs $1 per square me-tre. Find the dimensions of the container which has the largest possiblevolume if the total cost of materials is $72.

9. *. Find a point X on the positive x–axis and a point Y on the positive y–axissuch that (taking O = (0, 0))

i The triangle XOY contains the first quadrant portion of the unit circlex2 + y2 = 1 and

ii the area of the triangle XOY is as small as possible.

A complete and careful mathematical justification of property 3.5.4.9.i isrequired.

10. *. A rectangle is inscribed in a semicircle of radius R so that one sideof the rectangle lies along a diameter of the semicircle. Find the largestpossible perimeter of such a rectangle, if it exists, or explain why it doesnot. Do the same for the smallest possible perimeter.

409

Page 426: CLP-1 Differential Calculus

APPLICATIONS OF DERIVATIVES 3.5 OPTIMISATION

11. *. Find the maximal possible volume of a cylinder with surface area A.a

a Food is often packaged in cylinders, and companies wouldn’t want to waste themetal they are made out of. So, you might expect the dimensions you find in thisproblem to describe a tin of, say, cat food. Read here about why this isn’t the case.

12. *. What is the largest possible area of a window, with perimeter P , inthe shape of a rectangle with a semicircle on top (so the diameter of thesemicircle equals the width of the rectangle)?

13. *. Consider an open-top rectangular baking pan with base dimensions xcentimetres by y centimetres and height z centimetres that is made from Asquare centimetres of tin plate. Suppose y = px for some fixed constant p.

a Find the dimensions of the baking pan with the maximum capacity(i.e., maximum volume). Prove that your answer yields the bakingpan with maximum capacity. Your answer will depend on the valueof p.

b Find the value of the constant p that yields the baking pan with max-imum capacity and give the dimensions of the resulting baking pan.Prove that your answer yields the baking pan with maximum capac-ity.

Exercises — Stage 3

14. *. Let f(x) = xx for x > 0.

a Find f ′(x).

b At what value of x does the curve y = f(x) have a horizontaltangent line?

c Does the function f have a local maximum, a local minimum, orneither of these at the point x found in part 3.5.4.14.b?

15. *. A length of wire is cut into two pieces, one of which is bent to forma circle, the other to form a square. How should the wire be cut if thearea enclosed by the two curves is maximized? How should the wirebe cut if the area enclosed by the two curves is minimized? Justify youranswers.

410

Page 427: CLP-1 Differential Calculus

APPLICATIONS OF DERIVATIVES 3.6 SKETCHING GRAPHS

3.6q Sketching Graphs

One of the most obvious applications of derivatives is to help us understand theshape of the graph of a function. In this section we will use our accumulated knowl-edge of derivatives to identify the most important qualitative features of graphsy = f(x). The goal of this section is to highlight features of the graph y = f(x)that are easily

• determined from f(x) itself, and

• deduced from f ′(x), and

• read from f ′′(x).

We will then use the ideas to sketch several examples.

3.6.1 tt Domain, Intercepts and Asymptotes

Given a function f(x), there are several important features that we can determinefrom that expression before examining its derivatives.

• The domain of the function — take note of values where f does not exist. Ifthe function is rational, look for where the denominator is zero. Similarly becareful to look for roots of negative numbers or other possible sources of dis-continuities.

• Intercepts — examine where the function crosses the x-axis and the y-axis bysolving f(x) = 0 and computing f(0).

• Vertical asymptotes — look for values of x at which f(x) blows up. If f(x)approaches either +∞ or −∞ as x approaches a (or possibly as x approaches afrom one side) then x = a is a vertical asymptote to y = f(x). When f(x) is arational function (written so that common factors are cancelled), then y = f(x)has vertical asymptotes at the zeroes of the denominator.

• Horizontal asymptotes — examine the limits of f(x) as x→ +∞ and x→ −∞.Often f(x) will tend to +∞ or to −∞ or to a finite limit L. If, for example,lim

x→+∞f(x) = L, then y = L is a horizontal asymptote to y = f(x) as x→∞.

411

Page 428: CLP-1 Differential Calculus

APPLICATIONS OF DERIVATIVES 3.6 SKETCHING GRAPHS

Example 3.6.1 Domain, intercepts and asymptotes of x+1(x+3)(x−2)

.

Consider the function

f(x) =x+ 1

(x+ 3)(x− 2)

• We see that it is defined on all real numbers except x = −3,+2.

• Since f(0) = −1/6 and f(x) = 0 only when x = −1, the graph has y-intercept(0,−1/6) and x-intercept (−1, 0).

• Since the function is rational and its denominator is zero at x = −3,+2 it willhave vertical asymptotes at x = −3,+2. To determine the shape around thoseasymptotes we need to examine the limits

limx→−3

f(x) limx→2

f(x)

Notice that when x is close to−3, the factors (x+1) and (x−2) are both negative,so the sign of f(x) = x+1

x−2· 1x+3

is the same as the sign of x+ 3. Hence

limx→−3+

f(x) = +∞ limx→−3−

f(x) = −∞

A similar analysis when x is near 2 gives

limx→2+

f(x) = +∞ limx→2−

f(x) = −∞

• Finally since the numerator has degree 1 and the denominator has degree 2,we see that as x→ ±∞, f(x)→ 0. So y = 0 is a horizontal asymptote.

• Since we know the behaviour around the asymptotes and we know the loca-tions of the intercepts (as shown in the left graph below), we can then join upthe pieces and smooth them out to get the a good sketch of this function (belowright).

412

Page 429: CLP-1 Differential Calculus

APPLICATIONS OF DERIVATIVES 3.6 SKETCHING GRAPHS

3.6.2 tt First Derivative — Increasing or Decreasing

Now we move on to the first derivative, f ′(x). This is a good time to revisit the mean-value theorem (Theorem 2.13.5) and some of its consequences (Corollary 2.13.12). Inparticular, let us assume that f(x) is continuous on an interval [A,B] and differen-tiable on (A,B). Then

• if f ′(x) > 0 for all A < x < B, then f(x) is increasing on (A,B)

— that is, for all A < a < b < B, f(a) < f(b).

• if f ′(x) < 0 for all A < x < B, then f(x) is decreasing on (A,B)

— that is, for all A < a < b < B, f(a) > f(b).

Thus the sign of the derivative indicates to us whether the function is increasing ordecreasing. Further, as we discussed in Section 3.5.1, we should also examine pointsat which the derivative is zero — critical points — and where the derivative does notexist — singular points. These points may indicate a local maximum or minimum.

After studying the function f(x) as described above, we should compute itsderivative f ′(x).

• Critical points — determine where f ′(x) = 0. At a critical point, f has a hori-zontal tangent.

• Singular points — determine where f ′(x) is not defined. If f ′(x) approaches±∞ as x approaches a singular point a, then f has a vertical tangent there whenf approaches a finite value as x approaches a (or possibly approaches a fromone side) and a vertical asymptote when f(x) approaches ±∞ as x approachesa (or possibly approaches a from one side).

• Increasing and decreasing — where is the derivative positive and where is itnegative. Notice that in order for the derivative to change sign, it must eitherpass through zero (a critical point) or have a singular point. Thus neighbour-ing regions of increase and decrease will be separated by critical and singularpoints.

Example 3.6.2 A simple polynomial.

Consider the function

f(x) = x4 − 6x3

• Before we move on to derivatives, let us first examine the function itself as wedid above.

As f(x) is a polynomial its domain is all real numbers.

413

Page 430: CLP-1 Differential Calculus

APPLICATIONS OF DERIVATIVES 3.6 SKETCHING GRAPHS

Its y-intercept is at (0, 0). We find its x-intercepts by factoring

f(x) = x4 − 6x3 = x3(x− 6)

So it crosses the x-axis at x = 0, 6.

Again, since the function is a polynomial it does not have any verticalasymptotes. And since

limx→±∞

f(x) = limx→±∞

x4(1− 6/x) = +∞

it does not have horizontal asymptotes — it blows up to +∞ as x goes to±∞.

We can also determine where the function is positive or negative since weknow it is continuous everywhere and zero at x = 0, 6. Thus we mustexamine the intervals

(−∞, 0) (0, 6) (6,∞)

When x < 0, x3 < 0 and x − 6 < 0 so f(x) = x3(x − 6) =(negative)(negative) > 0. Similarly when x > 6, x3 > 0, x − 6 > 0 wemust have f(x) > 0. Finally when 0 < x < 6, x3 > 0 but x − 6 < 0 sof(x) < 0. Thus

interval (−∞, 0) 0 (0, 6) 6 (6,∞)

f(x) positive 0 negative 0 positive

Based on this information we can already construct a rough sketch.

• Now we compute its derivative

f ′(x) = 4x3 − 18x2 = 2x2(2x− 9)

414

Page 431: CLP-1 Differential Calculus

APPLICATIONS OF DERIVATIVES 3.6 SKETCHING GRAPHS

• Since the function is a polynomial, it does not have any singular points, but itdoes have two critical points at x = 0, 9/2. These two critical points split thereal line into 3 open intervals

(−∞, 0) (0, 9/2) (9/2,∞)

We need to determine the sign of the derivative in each intervals.

When x < 0, x2 > 0 but (2x − 9) < 0, so f ′(x) < 0 and the function isdecreasing.

When 0 < x < 9/2, x2 > 0 but (2x − 9) < 0, so f ′(x) < 0 and the functionis still decreasing.

When x > 9/2, x2 > 0 and (2x − 9) > 0, so f ′(x) > 0 and the function isincreasing.

We can then summarise this in the following table

interval (−∞, 0) 0 (0, 9/2) 9/2 (9/2,∞)

f ′(x) negative 0 negative 0 positive

decreasing horizontaltangent

decreasing minimum increasing

Since the derivative changes sign from negative to positive at the critical pointx = 9/2, this point is a minimum. Its y-value is

y = f(9/2) =93

23

(9

2− 6

)=

36

23·(−3

2

)= −37

24

On the other hand, at x = 0 the derivative does not change sign; while thispoint has a horizontal tangent line it is not a minimum or maximum.

• Putting this information together we arrive at a quite reasonable sketch.

415

Page 432: CLP-1 Differential Calculus

APPLICATIONS OF DERIVATIVES 3.6 SKETCHING GRAPHS

Example 3.6.2

To improve upon this further we will examine the second derivative.

3.6.3 tt Second Derivative — Concavity

The second derivative f ′′(x) tells us the rate at which the derivative changes. Perhapsthe easiest way to understand how to interpret the sign of the second derivative isto think about what it implies about the slope of the tangent line to the graph of thefunction. Consider the following sketches of y = 1 + x2 and y = −1− x2.

• In the case of y = f(x) = 1+x2 , f ′′(x) = 2 > 0. Notice that this means the slope,f ′(x), of the line tangent to the graph at x increases as x increases. Looking atthe figure on the left above, we see that the graph always lies above the tangentlines.

416

Page 433: CLP-1 Differential Calculus

APPLICATIONS OF DERIVATIVES 3.6 SKETCHING GRAPHS

• For y = f(x) = −1 − x2 , f ′′(x) = −2 < 0. The slope, f ′(x), of the line tangentto the graph at x decreases as x increases. Looking at the figure on the rightabove, we see that the graph always lies below the tangent lines.

Similarly consider the following sketches of y = x−1/2 and y =√

4− x:

Both of their derivatives, −12x−3/2 and −1

2(4 − x)−1/2, are negative, so they are

decreasing functions. Examining second derivatives shows some differences.

• For the first function, y′′(x) = 34x−5/2 > 0, so the slopes of tangent lines are

increasing with x and the graph lies above its tangent lines.

• However, the second function has y′′(x) = −14(4−x)−3/2 < 0 so the slopes of the

tangent lines are decreasing with x and the graph lies below its tangent lines.

More generally

Definition 3.6.3

Let f(x) be a continuous function on the interval [a, b] and suppose its first andsecond derivatives exist on that interval.

• If f ′′(x) > 0 for all a < x < b, then the graph of f lies above its tangentlines for a < x < b and it is said to be concave up.

• If f ′′(x) < 0 for all a < x < b, then the graph of f lies below its tangentlines for a < x < b and it is said to be concave down.

417

Page 434: CLP-1 Differential Calculus

APPLICATIONS OF DERIVATIVES 3.6 SKETCHING GRAPHS

• If f ′′(c) = 0 for some a < c < b, and the concavity of f changes acrossx = c, then we call (c, f(c)) an inflection point.

Note that one might also see the terms

• “convex” or “convex up” used in place of “concave up”, and

• “concave” or “convex down” used to mean “concave down”.

To avoid confusion we recommend the reader stick with the terms “concave up” and“concave down”.

Let’s now continue Example 3.6.2 by discussing the concavity of the curve.

Example 3.6.4 Continuation of 3.6.2.

Consider again the function

f(x) = x4 − 6x3

• Its first derivative is f ′(x) = 4x3 − 18x2, so

f ′′(x) = 12x2 − 36x = 12x(x− 3)

• Thus the second derivative is zero (and potentially changes sign) at x = 0, 3.Thus we should consider the sign of the second derivative on the followingintervals

(−∞, 0) (0, 3) (3,∞)

418

Page 435: CLP-1 Differential Calculus

APPLICATIONS OF DERIVATIVES 3.6 SKETCHING GRAPHS

Example 3.6.4

A little algebra gives us

interval (−∞, 0) 0 (0, 3) 3 (3,∞)

f ′′(x) positive 0 negative 0 positiveconcavity up inflection down inflection up

Since the concavity changes at both x = 0 and x = 3, the following are inflectionpoints

(0, 0) (3, 34 − 6× 33) = (3,−34)

• Putting this together with the information we obtained earlier gives us the fol-lowing sketch

3.6.4 tt Symmetries

Before we proceed to some examples, we should examine some simple symmetriespossessed by some functions. We’ll look at three symmetries — evenness, oddnessand periodicity. If a function possesses one of these symmetries then it can be ex-ploited to reduce the amount of work required to sketch the graph of the function.

Let us start with even and odd functions.

419

Page 436: CLP-1 Differential Calculus

APPLICATIONS OF DERIVATIVES 3.6 SKETCHING GRAPHS

Definition 3.6.5

A function f(x) is said to be even if f(−x) = f(x) for all x.

Definition 3.6.6

A function f(x) is said to be odd if f(−x) = −f(x) for all x.

Example 3.6.7 An even function and an odd funtion.

Let f(x) = x2 and g(x) = x3. Then

f(−x) = (−x)2 = x2 = f(x)

g(−x) = (−x)3 = −x3 = −g(x)

Hence f(x) is even and g(x) is odd.Notice any polynomial involving only even powers of x will be even

f(x) = 7x6 + 2x4 − 3x2 + 5 remember that 5 = 5x0

f(−x) = 7(−x)6 + 2(−x)4 − 3(−x)2 + 5

= 7x6 + 2x4 − 3x2 + 5 = f(x)

Similarly any polynomial involving only odd powers of x will be odd

g(x) = 2x5 − 8x3 − 3x

g(−x) = 2(−x)5 − 8(−x)3 − 3(−x)

= −2x5 + 8x3 + 3x = −g(x)

Not all even and odd functions are polynomials. For example

|x| cosx and (ex + e−x)

are all even, while

sinx tanx and (ex − e−x)

are all odd. Indeed, given any function f(x), the function

g(x) = f(x) + f(−x) will be even, andh(x) = f(x)− f(−x) will be odd.

Now let us see how we can make use of these symmetries to make graph sketch-ing easier. Let f(x) be an even function. Then

the point (x0, y0) lies on the graph of y = f(x)

420

Page 437: CLP-1 Differential Calculus

APPLICATIONS OF DERIVATIVES 3.6 SKETCHING GRAPHS

if and only if y0 = f(x0) = f(−x0) which is the case if and only if

the point (−x0, y0) lies on the graph of y = f(x).

Notice that the points (x0, y0) and (−x0, y0) are just reflections of each other acrossthe y-axis. Consequently, to draw the graph y = f(x), it suffices to draw the part ofthe graph with x ≥ 0 and then reflect it in the y–axis. Here is an example. The partwith x ≥ 0 is on the left and the full graph is on the right.

Very similarly, when f(x) is an odd function then

(x0, y0) lies on the graph of y = f(x)

if and only if

(−x0,−y0) lies on the graph of y = f(x)

421

Page 438: CLP-1 Differential Calculus

APPLICATIONS OF DERIVATIVES 3.6 SKETCHING GRAPHS

Now the symmetry is a little harder to interpret pictorially. To get from (x0, y0)to (−x0,−y0) one can first reflect (x0, y0) in the y–axis to get to (−x0, y0) and thenreflect the result in the x–axis to get to (−x0,−y0). Consequently, to draw the graphy = f(x), it suffices to draw the part of the graph with x ≥ 0 and then reflect it firstin the y–axis and then in the x–axis. Here is an example. First, here is the part of thegraph with x ≥ 0.

Next, as an intermediate step (usually done in our heads rather than on paper),we add in the reflection in the y–axis.

Finally to get the full graph, we reflect the dashed line in the x–axis

and then remove the dashed line.

422

Page 439: CLP-1 Differential Calculus

APPLICATIONS OF DERIVATIVES 3.6 SKETCHING GRAPHS

Let’s do a more substantial example of an even function

Example 3.6.8 An even rational function.

Consider the function

g(x) =x2 − 9

x2 + 3

• The function is even since

g(−x) =(−x)2 − 9

(−x)2 + 3=x2 − 9

x2 + 3= g(x)

Thus it suffices to study the function for x ≥ 0 because we can then use theeven symmetry to understand what happens for x < 0.

• The function is defined on all real numbers since its denominator x2+3 is neverzero. Hence it has no vertical asymptotes.

• The y-intercept is g(0) = −93

= −3. And x-intercepts are given by the solutionof x2 − 9 = 0, namely x = ±3. Note that we only need to establish x = 3 as anintercept. Then since g is even, we know that x = −3 is also an intercept.

• To find the horizontal asymptotes we compute the limit as x→ +∞

limx→∞

g(x) = limx→∞

x2 − 9

x2 + 3

= limx→∞

x2(1− 9/x2)

x2(1 + 3/x2)

= limx→∞

1− 9/x2

1 + 3/x2= 1

Thus y = 1 is a horizontal asymptote. Indeed, this is also the asymptote asx→ −∞ since by the even symmetry

limx→−∞

g(x) = limx→∞

g(−x) = limx→∞

g(x).

423

Page 440: CLP-1 Differential Calculus

APPLICATIONS OF DERIVATIVES 3.6 SKETCHING GRAPHS

• We can already produce a quite reasonable sketch just by putting in the hor-izontal asymptote and the intercepts and drawing a smooth curve betweenthem.

Note that we have drawn the function as never crossing the asymptote y = 1,however we have not yet proved that. We could by trying to solve g(x) = 1.

x2 − 9

x2 + 3= 1

x2 − 9 = x2 + 3

−9 = 3 so no solutions.

Alternatively we could analyse the first derivative to see how the function ap-proaches the asymptote.

• Now we turn to the first derivative:

g′(x) =(x2 + 3)(2x)− (x2 − 9)(2x)

(x2 + 3)2

=24x

(x2 + 3)2

There are no singular points since the denominator is nowhere zero. The onlycritical point is at x = 0. Thus we must find the sign of g′(x) on the intervals

(−∞, 0) (0,∞)

• When x > 0, 24x > 0 and (x2 + 3) > 0, so g′(x) > 0 and the function isincreasing. By even symmetry we know that when x < 0 the function must bedecreasing. Hence the critical point x = 0 is a local minimum of the function.

• Notice that since the function is increasing for x > 0 and the function mustapproach the horizontal asymptote y = 1 from below. Thus the sketch above isquite accurate.

424

Page 441: CLP-1 Differential Calculus

APPLICATIONS OF DERIVATIVES 3.6 SKETCHING GRAPHS

Example 3.6.8

• Now consider the second derivative:

g′′(x) =d

dx

24x

(x2 + 3)2

=(x2 + 3)2 · 24− 24x · 2(x2 + 3) · 2x

(x2 + 3)4

cancel a factor of (x2 + 3)

=(x2 + 3) · 24− 96x2

(x2 + 3)3

=72(1− x2)

(x2 + 3)3

• It is clear that g′′(x) = 0 when x = ±1. Note that, again, we can infer the zero atx = −1 from the zero at x = 1 by the even symmetry. Thus we need to examinethe sign of g′′(x) the intervals

(−∞,−1) (−1, 1) (1,∞)

• When |x| < 1 we have (1−x2) > 0 so that g′′(x) > 0 and the function is concaveup. When |x| > 1 we have (1 − x2) < 0 so that g′′(x) < 0 and the function isconcave down. Thus the points x = ±1 are inflection points. Their coordinatesare (±1, g(±1)) = (±1,−2).

• Putting this together gives the following sketch:

Another symmetry we should consider is periodicity.

425

Page 442: CLP-1 Differential Calculus

APPLICATIONS OF DERIVATIVES 3.6 SKETCHING GRAPHS

Definition 3.6.9

A function f(x) is said to be periodic, with period P > 0, if f(x+P ) = f(x) forall x.

Note that if f(x+ P ) = f(x) for all x, then replacing x by x+ P , we have

f(x+ 2P ) = f(x+ P + P ) = f(x+ P ) = f(x).

More generally f(x + kP ) = f(x) for all integers k. Thus if f has period P , thenit also has period nP for all natural numbers n. The smallest period is called thefundamental period.

Example 3.6.10 sinx is periodic.

The classic example of a periodic function is f(x) = sinx, which has period 2π sincef(x+ 2π) = sin(x+ 2π) = sinx = f(x).

If f(x) has period P then

(x0, y0) lies on the graph of y = f(x)

if and only if y0 = f(x0) = f(x0 + P ) which is the case if and only if

(x0 + P, y0) lies on the graph of y = f(x)

and, more generally,

(x0, y0) lies on the graph of y = f(x)

if and only if

(x0 + nP, y0) lies on the graph of y = f(x)

for all integers n.Note that the point (x0 +P, y0) can be obtained by translating (x0, y0) horizontally

by P . Similarly the point (x0 +nP, y0) can be found by repeatedly translating (x0, y0)horizontally by P .

Consequently, to draw the graph y = f(x), it suffices to draw one period of thegraph, say the part with 0 ≤ x ≤ P , and then translate it repeatedly. Here is anexample. Here is a sketch of one period

426

Page 443: CLP-1 Differential Calculus

APPLICATIONS OF DERIVATIVES 3.6 SKETCHING GRAPHS

and here is the full sketch.

3.6.5 tt A Checklist for Sketching

Above we have described how we can use our accumulated knowledge of deriva-tives to quickly identify the most important qualitative features of graphs y = f(x).Here we give the reader a quick checklist of things to examine in order to producean accurate sketch based on properties that are easily read off from f(x), f ′(x) andf ′′(x).

3.6.5.1ttt A Sketching Checklist

1 Features of y = f(x) that are read off of f(x):

• First check where f(x) is defined. Then

• y=f(x) is plotted only for x’s in the domain of f(x), i.e. where f(x) isdefined.

• y = f(x) has vertical asymptotes at the points where f(x) blows up to±∞.

• Next determine whether the function is even, odd, or periodic.

• y = f(x) is first plotted for x ≥ 0 if the function is even or odd. The rest ofthe sketch is then created by reflections.

• y = f(x) is first plotted for a single period if the function is periodic. Therest of the sketch is then created by translations.

427

Page 444: CLP-1 Differential Calculus

APPLICATIONS OF DERIVATIVES 3.6 SKETCHING GRAPHS

• Next compute f(0), limx→∞ f(x) and limx→−∞ f(x) and look for solutionsto f(x) = 0 that you can easily find. Then

• y = f(x) has y–intercept(0, f(0)

).

• y = f(x) has x–intercept (a, 0) whenever f(a) = 0

• y = f(x) has horizontal asymptote y = Y if limx→∞ f(x) = L or limx→−∞ f(x) =L.

2 Features of y = f(x) that are read off of f ′(x):

• Compute f ′(x) and determine its critical points and singular points, then

• y = f(x) has a horizontal tangent at the points where f ′(x) = 0.

• y = f(x) is increasing at points where f ′(x) > 0.

• y = f(x) is decreasing at points where f ′(x) < 0.

• y = f(x) has vertical tangents or vertical asymptotes at the points wheref ′(x) = ±∞.

3 Features of y = f(x) that are read off of f ′′(x):

• Compute f ′′(x) and determine where f ′′(x) = 0 or does not exist, then

• y = f(x) is concave up at points where f ′′(x) > 0.

• y = f(x) is concave down at points where f ′′(x) < 0.

• y = f(x) may or may not have inflection points where f ′′(x) = 0.

3.6.6 tt Sketching Examples

Example 3.6.11 Sketch f(x) = x3 − 3x+ 1.

1 Reading from f(x):

• The function is a polynomial so it is defined everywhere.• Since f(−x) = −x3 + 3x + 1 6= ±f(x), it is not even or odd. Nor is it

periodic.• The y-intercept is y = 1. The x-intercepts are not easily computed since it

is a cubic polynomial that does not factor nicely a . So for this example wedon’t worry about finding them.

• Since it is a polynomial it has no vertical asymptotes.• For very large x, both positive and negative, the x3 term in f(x) dominates

the other two terms so that

f(x)→

+∞ as x→ +∞−∞ as x→ −∞

428

Page 445: CLP-1 Differential Calculus

APPLICATIONS OF DERIVATIVES 3.6 SKETCHING GRAPHS

and there are no horizontal asymptotes.

2 We now compute the derivative:

f ′(x) = 3x2 − 3 = 3(x2 − 1) = 3(x+ 1)(x− 1)

• The critical points (where f ′(x) = 0) are at x = ±1. Further since thederivative is a polynomial it is defined everywhere and there are nosingular points. The critical points split the real line into the intervals(−∞,−1), (−1, 1) and (1,∞).

• When x < −1, both factors (x+ 1), (x− 1) < 0 so f ′(x) > 0.

• Similarly when x > 1, both factors (x+ 1), (x− 1) > 0 so f ′(x) > 0.

• When −1 < x < 1, (x− 1) < 0 but (x+ 1) > 0 so f ′(x) < 0.

• Summarising all this

(−∞,−1) -1 (-1,1) 1 (1,∞)

f ′(x) positive 0 negative 0 positiveincreasing maximum decreasing minimum increasing

So (−1, f(−1)) = (−1, 3) is a local maximum and (1, f(1)) = (1,−1) is alocal minimum.

3 Compute the second derivative:

f ′′(x) = 6x

• The second derivative is zero when x = 0, and the problem is quite easyto analyse. Clearly, f ′′(x) < 0 when x < 0 and f ′′(x) > 0 when x > 0.

• Thus f is concave down for x < 0, concave up for x > 0 and has aninflection point at x = 0.

Putting this all together gives:

429

Page 446: CLP-1 Differential Calculus

APPLICATIONS OF DERIVATIVES 3.6 SKETCHING GRAPHS

Example 3.6.11

a With the aid of a computer we can find the x-intercepts numerically: x ≈−1.879385242, 0.3472963553, and 1.532088886. If you are interested in more details checkout Appendix C.

Example 3.6.12 Sketch f(x) = x4 − 4x3.

1 Reading from f(x):

• The function is a polynomial so it is defined everywhere.

• Since f(−x) = x4 + 4x3 6= ±f(x), it is not even or odd. Nor is it periodic.

• The y-intercept is y = f(0) = 0, while the x-intercepts are given by thesolution of

f(x) = x4 − 4x3 = 0

x3(x− 4) = 0

Hence the x-intercepts are 0, 4.

• Since f is a polynomial it does not have any vertical asymptotes.

• For very large x, both positive and negative, the x4 term in f(x) dominates

430

Page 447: CLP-1 Differential Calculus

APPLICATIONS OF DERIVATIVES 3.6 SKETCHING GRAPHS

the other term so that

f(x)→

+∞ as x→ +∞+∞ as x→ −∞

and the function has no horizontal asymptotes.

2 Now compute the derivative f ′(x):

f ′(x) = 4x3 − 12x2 = 4(x− 3)x2

• The critical points are at x = 0, 3. Since the function is a polynomial thereare no singular points. The critical points split the real line into the inter-vals (−∞, 0), (0, 3) and (3,∞).

• When x < 0, x2 > 0 and x− 3 < 0, so f ′(x) < 0.

• When 0 < x < 3, x2 > 0 and x− 3 < 0, so f ′(x) < 0.

• When 3 < x, x2 > 0 and x− 3 > 0, so f ′(x) > 0.

• Summarising all this

(−∞, 0) 0 (0,3) 3 (3,∞)

f ′(x) negative 0 negative 0 positive

decreasing horizontaltangent

decreasing minimum increasing

So the point (3, f(3)) = (3,−27) is a local minimum. The point (0, f(0)) =(0, 0) is neither a minimum nor a maximum, even though f ′(0) = 0.

3 Now examine f ′′(x):

f ′′(x) = 12x2 − 24x = 12x(x− 2)

• So f ′′(x) = 0 when x = 0, 2. This splits the real line into the intervals(−∞, 0), (0, 2) and (2,∞).

• When x < 0, x− 2 < 0 and so f ′′(x) > 0.

• When 0 < x < 2, x > 0 and x− 2 < 0 and so f ′′(x) < 0.

• When 2 < x, x > 0 and x− 2 > 0 and so f ′′(x) > 0.

• Thus the function is convex up for x < 0, then convex down for 0 < x <2, and finally convex up again for x > 2. Hence (0, f(0)) = (0, 0) and(2, f(2)) = (2,−16) are inflection points.

Putting all this information together gives us the following sketch.

431

Page 448: CLP-1 Differential Calculus

APPLICATIONS OF DERIVATIVES 3.6 SKETCHING GRAPHS

Example 3.6.12

Example 3.6.13 f(x) = x3 − 6x2 + 9x− 54.

1 Reading from f(x):

• The function is a polynomial so it is defined everywhere.

• Since f(−x) = −x3 − 6x2 − 9x− 54 6= ±f(x), it is not even or odd. Nor isit periodic.

• The y-intercept is y = f(0) = −54, while the x-intercepts are given by thesolution of

f(x) = x3 − 6x2 + 9x− 54 = 0

x2(x− 6) + 9(x− 6) = 0

(x2 + 9)(x− 6) = 0

Hence the only x-intercept is 6.

• Since f is a polynomial it does not have any vertical asymptotes.

• For very large x, both positive and negative, the x3 term in f(x) dominatesthe other term so that

f(x)→

+∞ as x→ +∞−∞ as x→ −∞

and the function has no horizontal asymptotes.

432

Page 449: CLP-1 Differential Calculus

APPLICATIONS OF DERIVATIVES 3.6 SKETCHING GRAPHS

2 Now compute the derivative f ′(x):

f ′(x) = 3x2 − 12x+ 9

= 3(x2 − 4x+ 3) = 3(x− 3)(x− 1)

• The critical points are at x = 1, 3. Since the function is a polynomial thereare no singular points. The critical points split the real line into the inter-vals (−∞, 1), (1, 3) and (3,∞).

• When x < 1, (x− 1) < 0 and (x− 3) < 0, so f ′(x) > 0.

• When 1 < x < 3, (x− 1) > 0 and (x− 3) < 0, so f ′(x) < 0.

• When 3 < x, (x− 1) > 0 and (x− 3) > 0, so f ′(x) > 0.

• Summarising all this

(−∞, 1) 1 (1,3) 3 (3,∞)

f ′(x) positive 0 negative 0 positiveincreasing maximum decreasing minimum increasing

So the point (1, f(1)) = (1,−50) is a local maximum. The point (3, f(3)) =(3,−54) is a local minimum.

3 Now examine f ′′(x):

f ′′(x) = 6x− 12

• So f ′′(x) = 0 when x = 2. This splits the real line into the intervals (−∞, 2)and (2,∞).

• When x < 2, f ′′(x) < 0.

• When x > 2, f ′′(x) > 0.

• Thus the function is convex down for x < 2, then convex up for x > 2.Hence (2, f(2)) = (2,−52) is an inflection point.

Putting all this information together gives us the following sketch.

433

Page 450: CLP-1 Differential Calculus

APPLICATIONS OF DERIVATIVES 3.6 SKETCHING GRAPHS

Example 3.6.13

and if we zoom in around the interesting points (minimum, maximum and inflectionpoint), we have

An example of sketching a simple rational function.

Example 3.6.14 f(x) =x

x2 − 4.

1 Reading from f(x):

• The function is rational so it is defined except where its denominator is

434

Page 451: CLP-1 Differential Calculus

APPLICATIONS OF DERIVATIVES 3.6 SKETCHING GRAPHS

zero — namely at x = ±2.

• Since f(−x) =−x

x2 − 4= −f(x), it is odd. Indeed this means that we only

need to examine what happens to the function for x ≥ 0 and we can theninfer what happens for x ≤ 0 using f(−x) = −f(x). In practice we willsketch the graph for x ≥ 0 and then infer the rest from this symmetry.

• The y-intercept is y = f(0) = 0, while the x-intercepts are given by thesolution of f(x) = 0. So the only x-intercept is 0.

• Since f is rational, it may have vertical asymptotes where its denominatoris zero — at x = ±2. Since the function is odd, we only have to analysethe asymptote at x = 2 and we can then infer what happens at x = −2 bysymmetry.

limx→2+

f(x) = limx→2+

x

(x− 2)(x+ 2)= +∞

limx→2−

f(x) = limx→2−

x

(x− 2)(x+ 2)= −∞

• We now check for horizontal asymptotes:

limx→+∞

f(x) = limx→+∞

x

x2 − 4

= limx→+∞

1

x− 4/x= 0

2 Now compute the derivative f ′(x):

f ′(x) =(x2 − 4) · 1− x · 2x

(x2 − 4)2

=−(x2 + 4)

(x2 − 4)2

• Hence there are no critical points. There are singular points where the de-nominator is zero, namely x = ±2. Before we proceed, notice that the nu-merator is always negative and the denominator is always positive. Hencef ′(x) < 0 except at x = ±2 where it is undefined.

• The function is decreasing except at x = ±2.

• We already know that at x = 2 we have a vertical asymptote and thatf ′(x) < 0 for all x. So

limx→2

f ′(x) = −∞

• Summarising all this

435

Page 452: CLP-1 Differential Calculus

APPLICATIONS OF DERIVATIVES 3.6 SKETCHING GRAPHS

[0,2) 2 (2,∞)

f ′(x) negative DNE negative

decreasing verticalasymptote

decreasing

Remember — we will draw the graph for x ≥ 0 and then use the oddsymmetry to infer the graph for x < 0.

3 Now examine f ′′(x):

f ′′(x) = −(x2 − 4)2 · (2x)− (x2 + 4) · 2 · 2x · (x2 − 4)

(x2 − 4)4

= −(x2 − 4) · (2x)− (x2 + 4) · 4x(x2 − 4)3

= −2x3 − 8x− 4x3 − 16x

(x2 − 4)3

=2x(x2 + 12)

(x2 − 4)3

• So f ′′(x) = 0 when x = 0 and does not exist when x = ±2. This splits thereal line into the intervals (−∞,−2), (−2, 0), (0, 2) and (2,∞). Howeverwe only need to consider x ≥ 0 (because of the odd symmetry).

• When 0 < x < 2, x > 0, (x2 + 12) > 0 and (x2 − 4) < 0 so f ′′(x) < 0.

• When x > 2, x > 0, (x2 + 12) > 0 and (x2 − 4) > 0 so f ′′(x) > 0.

Putting all this information together gives the following sketch for x ≥ 0:

We can then draw in the graph for x < 0 using f(−x) = −f(x):

436

Page 453: CLP-1 Differential Calculus

APPLICATIONS OF DERIVATIVES 3.6 SKETCHING GRAPHS

Example 3.6.14

Notice that this means that the concavity changes at x = 0, so the point (0, f(0)) =(0, 0) is a point of inflection (as indicated).

This final example is more substantial since the function has singular points(points where the derivative is undefined). The analysis is more involved.

Example 3.6.15 f(x) = 3

√x2

(x−6)2.

1 Reading from f(x):

• First notice that we can rewrite

f(x) = 3

√x2

(x− 6)2= 3

√x2

x2 · (1− 6/x)2= 3

√1

(1− 6/x)2

• The function is the cube root of a rational function. The rational functionis defined except at x = 6, so the domain of f is all reals except x = 6.

• Clearly the function is not periodic, and examining

f(−x) = 3

√1

(1− 6/(−x))2

= 3

√1

(1 + 6/x)26= ±f(x)

shows the function is neither even nor odd.

437

Page 454: CLP-1 Differential Calculus

APPLICATIONS OF DERIVATIVES 3.6 SKETCHING GRAPHS

• To compute horizontal asymptotes we examine the limit of the portion ofthe function inside the cube-root

limx→±∞

1

(1− 6x)2

= 1

This means we have

limx→±∞

f(x) = 1

That is, the line y = 1 will be a horizontal asymptote to the graph y = f(x)both for x→ +∞ and for x→ −∞.

• Our function f(x)→ +∞ as x→ 6, because of the (1−6/x)2 in its denom-inator. So y = f(x) has x = 6 as a vertical asymptote.

2 Now compute f ′(x). Since we rewrote

f(x) = 3

√1

(1− 6/x)2=

(1− 6

x

)− 23

we can use the chain rule

f ′(x) = −2

3

(1− 6

x

)− 53 6

x2

= −4

(x− 6

x

)− 53 1

x2

= −4

(1

x− 6

) 53 1

x13

• Notice that the derivative is nowhere equal to zero, so the function has nocritical points. However there are two places the derivative is undefined.The terms (

1

x− 6

) 53 1

x13

are undefined at x = 6, 0 respectively. Hence x = 0, 6 are singular points.These split the real line into the intervals (−∞, 0), (0, 6) and (6,∞).

• When x < 0, (x − 6) < 0, we have that (x − 6)−53 < 0 and x−

13 < 0 and so

f ′(x) = −4 · (negative) · (negative) < 0.

• When 0 < x < 6, (x− 6) < 0, we have that (x− 6)−53 < 0 and x−

13 > 0 and

so f ′(x) > 0.

• When x > 6, (x − 6) > 0, we have that (x − 6)−53 > 0 and x−

13 > 0 and so

f ′(x) < 0.

438

Page 455: CLP-1 Differential Calculus

APPLICATIONS OF DERIVATIVES 3.6 SKETCHING GRAPHS

• We should also examine the behaviour of the derivative as x → 0 andx→ 6.

limx→0−

f ′(x) = −4

(limx→0−

(x− 6)−53

)(limx→0−

x−13

)= −∞

limx→0+

f ′(x) = −4

(limx→0+

(x− 6)−53

)(limx→0+

x−13

)= +∞

limx→6−

f ′(x) = −4

(limx→6−

(x− 6)−53

)(limx→6−

x−13

)= +∞

limx→6+

f ′(x) = −4

(limx→6+

(x− 6)−53

)(limx→6+

x−13

)= −∞

We already know that x = 6 is a vertical asymptote of the function, so itis not surprising that the lines tangent to the graph become vertical as weapproach 6. The behavior around x = 0 is less standard, since the linestangent to the graph become vertical, but x = 0 is not a vertical asymptoteof the function. Indeed the function takes a finite value y = f(0) = 0.

• Summarising all this

(−∞, 0) 0 (0,6) 6 (6,∞)

f ′(x) negative DNE positive DNE negative

decreasing verticaltangents

increasing verticalasymptote

decreasing

3 Now look at f ′′(x):

f ′′(x) = −4d

dx

[(1

x− 6

) 53 1

x13

]

= −4

[−5

3

(1

x− 6

) 83 1

x13

− 1

3

(1

x− 6

) 53 1

x43

]

=4

3

(1

x− 6

) 83 1

x43

[5x+ (x− 6)]

= 8

(1

x− 6

) 83 1

x43

[x− 1]

Oof!

• Both of the factors(

1x−6

) 83

=(

13√x−6

)8

and 1

x43

=(

13√x

)4

are even powersand so are positive (though possibly infinite). So the sign of f ′′(x) is thesame as the sign of the factor x− 1. Thus

439

Page 456: CLP-1 Differential Calculus

APPLICATIONS OF DERIVATIVES 3.6 SKETCHING GRAPHS

(−∞, 1) 1 (1,∞)

f ′′(x) negative 0 positive

concave down inflectionpoint

concave up

Here is a sketch of the graph y = f(x).

It is hard to see the inflection point at x = 1, y = f(1) = 13√25

in the above sketch. Sohere is a blow up of the part of the sketch around x = 1.

And if we zoom in even more we have

440

Page 457: CLP-1 Differential Calculus

APPLICATIONS OF DERIVATIVES 3.6 SKETCHING GRAPHS

Example 3.6.15

3.6.7 tt Exercises

ttt Exercises for § 3.6.1

Exercises — Stage 1

1. Suppose f(x) is a function given by

f(x) =g(x)

x2 − 9

where g(x) is also a function. True or false: f(x) has a vertical asymptoteat x = −3.

Exercises — Stage 2

2. Match the functions f(x), g(x), h(x), and k(x) to the curves y = A(x)through y = D(x).

f(x) =√x2 + 1 g(x) =

√x2 − 1

h(x) =√x2 + 4 k(x) =

√x2 − 4

441

Page 458: CLP-1 Differential Calculus

APPLICATIONS OF DERIVATIVES 3.6 SKETCHING GRAPHS

x

y

y = A(x)

1−1 2−2

1

2

x

y

y = B(x)

1−1 2−2

1

2

x

y

y = C(x)

1−1 2−2

1

2

x

y

y = D(x)

1−1 2−2

1

2

3. Below is the graph of

y = f(x) =

√log2(x+ p)

a What is p?

b What is b (marked on the graph)?

c What is the x-intercept of f(x)?

Remember log(x+ p) is the natural logarithm of x+ p, loge(x+ p).

x

y

b

1

2

3

−6 −5 −4 −3 −2 −1 1 2−8

442

Page 459: CLP-1 Differential Calculus

APPLICATIONS OF DERIVATIVES 3.6 SKETCHING GRAPHS

4. Find all asymptotes of f(x) =x(2x+ 1)(x− 7)

3x3 − 81.

5. Find all asymptotes of f(x) = 103x−7.

ttt Exercises for § 3.6.2

Exercises — Stage 1

1. Match each function graphed below to its derivative from the list. (Forexample, which function on the list corresponds to A′(x)?)The y-axes have been scaled to make the curve’s behaviour clear, so thevertical scales differ from graph to graph.l(x) = (x− 2)4

m(x) = (x− 2)4(x+ 2)n(x) = (x− 2)2(x+ 2)2

o(x) = (x− 2)(x+ 2)3

p(x) = (x+ 2)4

x

y

y = A(x)

2−2

443

Page 460: CLP-1 Differential Calculus

APPLICATIONS OF DERIVATIVES 3.6 SKETCHING GRAPHS

x

yy = B(x)

2−2

x

y

y = C(x)

2−2

x

y

y = D(x)

2−2

444

Page 461: CLP-1 Differential Calculus

APPLICATIONS OF DERIVATIVES 3.6 SKETCHING GRAPHS

x

y

y = E(x)

2−2

Exercises — Stage 2

2. *. Find the interval(s) where f(x) =ex

x+ 3is increasing.

3. *. Find the interval(s) where f(x) =

√x− 1

2x+ 4is increasing.

4. *. Find the interval(s) where f(x) = 2 arctan(x)−log(1+x2) is increasing.

ttt Exercises for § 3.6.3

Exercises — Stage 1

1. On the graph below, mark the intervals where f ′′(x) > 0 (i.e. f(x) isconcave up) and where f ′′(x) < 0 (i.e. f(x) is concave down).

445

Page 462: CLP-1 Differential Calculus

APPLICATIONS OF DERIVATIVES 3.6 SKETCHING GRAPHS

x

y

2. Sketch a curve that is:

• concave up when |x| > 5,

• concave down when |x| < 5,

• increasing when x < 0, and

• decreasing when x > 0.

3. Suppose f(x) is a function whose second derivative exists and is continuousfor all real numbers.True or false: if f ′′(3) = 0, then x = 3 is an inflection point of f(x).Remark: compare to Question 3.6.7.7

Exercises — Stage 2

4. *. Find all inflection points for the graph of f(x) = 3x5 − 5x4 + 13x.

Exercises — Stage 3 Questions 3.6.7.5 through 3.6.7.7 ask you to show that certainthings are true. Give a clear explanation using concepts and theorems from thissemester.

5. *. Let

f(x) =x5

20+

5x3

6− 10x2 + 500x+ 1000

446

Page 463: CLP-1 Differential Calculus

APPLICATIONS OF DERIVATIVES 3.6 SKETCHING GRAPHS

Show that f(x) has exactly one inflection point.

6. *. Let f(x) be a function whose first two derivatives exist everywhere,and f ′′(x) > 0 for all x.

a Show that f(x) has at most one critical point and that any criticalpoint is an absolute minimum for f(x).

b Show that the maximum value of f(x) on any finite interval occursat one of the endpoints of the interval.

7. Suppose f(x) is a function whose second derivative exists and is continu-ous for all real numbers, and x = 3 is an inflection point of f(x). Use theIntermediate Value Theorem to show that f ′′(3) = 0.Remark: compare to Question 3.6.7.3.

ttt Exercises for § 3.6.4

Exercises — Stage 1

1. What symmetries (even, odd, periodic) does the function graphed be-low have?

x

y

y = f(x)

2. What symmetries (even, odd, periodic) does the function graphed belowhave?

447

Page 464: CLP-1 Differential Calculus

APPLICATIONS OF DERIVATIVES 3.6 SKETCHING GRAPHS

x

y

y = f(x)

3. Suppose f(x) is an even function defined for all real numbers. Below is thecurve y = f(x) when x < 0. Complete the sketch of the curve.

x

y

4. Suppose f(x) is an odd function defined for all real numbers. Below isthe curve y = f(x) when x < 0. Complete the sketch of the curve.

448

Page 465: CLP-1 Differential Calculus

APPLICATIONS OF DERIVATIVES 3.6 SKETCHING GRAPHS

x

y

Exercises — Stage 2 In Questions 3.6.7.7 through 3.6.7.10, find the symmetries of afunction from its equation.

5.

f(x) =x4 − x6

ex2

Show that f(x) is even.

6.f(x) = sin(x) + cos

(x2

)Show that f(x) is periodic.

7.f(x) = x4 + 5x2 + cos

(x3)

What symmetries (even, odd, periodic) does f(x) have?

8.f(x) = x5 + 5x4

What symmetries (even, odd, periodic) does f(x) have?

9.f(x) = tan (πx)

What is the period of f(x)?

Exercises — Stage 3

449

Page 466: CLP-1 Differential Calculus

APPLICATIONS OF DERIVATIVES 3.6 SKETCHING GRAPHS

10.f(x) = tan (3x) + sin (4x)

What is the period of f(x)?

ttt Exercises for § 3.6.6

Exercises — Stage 1 In Questions 3.6.7.2 through 3.6.7.4, you will sketch the graphsof rational functions.In Questions 3.6.7.6 and 3.6.7.7, you will sketch the graphs offunctions with an exponential component. In the next section, you will learn howto find their horizontal asymptotes, but for now these are given to you.In Ques-tions 3.6.7.8 and 3.6.7.9, you will sketch the graphs of functions that have a trigono-metric component.

1. *. Let f(x) = x√

3− x.

a Find the domain of f(x).

b Determine the x-coordinates of the local maxima and minima (ifany) and intervals where f(x) is increasing or decreasing.

c Determine intervals where f(x) is concave upwards or down-wards, and the x coordinates of inflection points (if any). You mayuse, without verifying it, the formula f ′′(x) = (3x−12)(3−x)−3/2/4.

d There is a point at which the tangent line to the curve y = f(x) isvertical. Find this point.

e Sketch the graph y = f(x), showing the features given in items (a)to (d) above and giving the (x, y) coordinates for all points occur-ring above.

2. *. Sketch the graph of

f(x) =x3 − 2

x4.

Indicate the critical points, local and absolute maxima and minima, verticaland horizontal asymptotes, inflection points and regions where the curve isconcave upward or downward.

3. *. The first and second derivatives of the function f(x) =x4

1 + x3are:

f ′(x) =4x3 + x6

(1 + x3)2and f ′′(x) =

12x2 − 6x5

(1 + x3)3

Graph f(x). Include local and absolute maxima and minima, regions wheref(x) is increasing or decreasing, regions where the curve is concave upward

450

Page 467: CLP-1 Differential Calculus

APPLICATIONS OF DERIVATIVES 3.6 SKETCHING GRAPHS

or downward, and any asymptotes.

4. *. The first and second derivatives of the function f(x) =x3

1− x2are:

f ′(x) =3x2 − x4

(1− x2)2and f ′′(x) =

6x+ 2x3

(1− x2)3

Graph f(x). Include local and absolute maxima and minima, regionswhere the curve is concave upward or downward, and any asymptotes.

5. *. The function f(x) is defined by

f(x) =

ex x < 0x2+3

3(x+1)x ≥ 0

a Explain why f(x) is continuous everywhere.

b Determine all of the following if they are present:

i x–coordinates of local maxima and minima, intervals wheref(x) is increasing or decreasing;

ii intervals where f(x) is concave upwards or downwards;

iii equations of any horizontal or vertical asymptotes.

c Sketch the graph of y = f(x), giving the (x, y) coordinates for allpoints of interest above.

6. *. The function f(x) and its derivative are given below:

f(x) = (1 + 2x)e−x2

and f ′(x) = 2(1− x− 2x2)e−x2

Sketch the graph of f(x). Indicate the critical points, local and/or absolutemaxima and minima, and asymptotes. Without actually calculating the in-flection points, indicate on the graph their approximate location.Note: lim

x→±∞f(x) = 0.

7. *. Consider the function f(x) = xe−x2/2.

Note: limx→±∞

f(x) = 0.

a Find all inflection points and intervals of increase, decrease, con-vexity up, and convexity down. You may use without proof theformula f ′′(x) = (x3 − 3x)e−x

2/2.

451

Page 468: CLP-1 Differential Calculus

APPLICATIONS OF DERIVATIVES 3.6 SKETCHING GRAPHS

b Find local and global minima and maxima.

c Use all the above to draw a graph for f . Indicate all special pointson the graph.

8. Use the techniques from this section to sketch the graph of f(x) = x+2 sinx.

9. *.f(x) = 4 sinx− 2 cos 2x

Graph the equation y = f(x), including all important features. (In particu-lar, find all local maxima and minima and all inflection points.) Addition-ally, find the maximum and minimum values of f(x) on the interval [0, π].

10. Sketch the curve y = 3

√x+ 1

x2.

You may use the facts y′(x) =−(x+ 2)

3x5/3(x+ 1)2/3and y′′(x) =

4x2 + 16x+ 10

9x8/3(x+ 1)5/3.

Exercises — Stage 3

11. *. A function f(x) defined on the whole real number line satisfies the fol-lowing conditions

f(0) = 0 f(2) = 2 limx→+∞

f(x) = 0 f ′(x) = K(2x− x2)e−x

for some positive constant K. (Read carefully: you are given the derivativeof f(x), not f(x) itself.)

a Determine the intervals on which f is increasing and decreasing andthe location of any local maximum and minimum values of f .

b Determine the intervals on which f is concave up or down and thex–coordinates of any inflection points of f .

c Determine limx→−∞

f(x).

d Sketch the graph of y = f(x), showing any asymptotes and the infor-mation determined in parts 3.6.7.11.a and 3.6.7.11.b.

12. *. Let f(x) = e−x , x ≥ 0.

a Sketch the graph of the equation y = f(x). Indicate any local extremaand inflection points.

b Sketch the graph of the inverse function y = g(x) = f−1(x).

452

Page 469: CLP-1 Differential Calculus

APPLICATIONS OF DERIVATIVES3.7 L’HÔPITAL’S RULE AND INDETERMINATE FORMS

c Find the domain and range of the inverse function g(x) = f−1(x).

d Evaluate g′(12).

13. *.

a Sketch the graph of y = f(x) = x5 − x, indicating asymptotes, localmaxima and minima, inflection points, and where the graph is con-cave up/concave down.

b Consider the function f(x) = x5 − x+ k, where k is a constant, −∞ <k <∞. How many roots does the function have? (Your answer mightdepend on the value of k.)

14. *. The hyperbolic trigonometric functions sinh(x) and cosh(x) are definedby

sinh(x) =ex − e−x

2cosh(x) =

ex + e−x

2

They have many properties that are similar to corresponding properties ofsin(x) and cos(x). In particular, it is easy to see that

d

dxsinh(x) = cosh(x)

d

dxcosh(x) = sinh(x) cosh2(x)− sinh2(x) = 1

You may use these properties in your solution to this question.

a Sketch the graphs of sinh(x) and cosh(x).

b Define inverse hyperbolic trigonometric functions sinh−1(x) andcosh−1(x), carefully specifing their domains of definition. Sketch thegraphs of sinh−1(x) and cosh−1(x).

c Find ddx

cosh−1(x)

.

3.7q L’Hôpital’s Rule and Indeterminate Forms

3.7.1 tt L’Hôpital’s Rule and Indeterminate Forms

Let us return to limits (Chapter 1) and see how we can use derivatives to simplifycertain families of limits called indeterminate forms. We know, from Theorem 1.4.3on the arithmetic of limits, that if

limx→a

f(x) = F limx→a

g(x) = G

453

Page 470: CLP-1 Differential Calculus

APPLICATIONS OF DERIVATIVES3.7 L’HÔPITAL’S RULE AND INDETERMINATE FORMS

and G 6= 0, then

limx→a

f(x)

g(x)=F

G

The requirement that G 6= 0 is critical — we explored this in Example 1.4.7. Pleasereread that example.

Of course 1 it is not surprising that if F 6= 0 and G = 0, then

limx→a

f(x)

g(x)= DNE

and if F = 0 but G 6= 0 then

limx→a

f(x)

g(x)= 0

However when both F,G = 0 then, as we saw in Example 1.4.7, almost anything canhappen

f(x) = x g(x) = x2 limx→0

x

x2= lim

x→0

1

x= DNE

f(x) = x2 g(x) = x limx→0

x2

x= lim

x→0x = 0

f(x) = x g(x) = x limx→0

x

x= lim

x→01 = 1

f(x) = 7x2 g(x) = 3x2 limx→0

7x2

3x2= lim

x→0

7

3=

7

3

Indeed after exploring Example 1.4.12 and 1.4.14 we gave ourselves the rule of thumbthat if we found 0/0, then there must be something that cancels.

Because the limit that results from these 0/0 situations is not immediately obvi-ous, but also leads to some interesting mathematics, we should give it a name.

Definition 3.7.1 First indeterminate forms.

Let a ∈ R and let f(x) and g(x) be functions. If

limx→a

f(x) = 0 and limx→a

g(x) = 0

then the limit

limx→a

f(x)

g(x)

is called a 00

indeterminate form.

There are quite a number of mathematical tools for evaluating such indeterminateforms — Taylor series for example. A simpler method, which works in quite a fewcases, is L’Hôpital’s rule 2 .

1 Now it is not so surprising, but perhaps back when we started limits, this was not so obvious.2 Named for the 17th century mathematician, Guillaume de l’Hôpital, who published the first

454

Page 471: CLP-1 Differential Calculus

APPLICATIONS OF DERIVATIVES3.7 L’HÔPITAL’S RULE AND INDETERMINATE FORMS

Theorem 3.7.2 L’Hôpital’s Rule.

Let a ∈ R and assume that

limx→a

f(x) = limx→a

g(x) = 0

Then

a if f ′(a) and g′(a) exist and g′(a) 6= 0, then

limx→a

f(x)

g(x)=f ′(a)

g′(a),

b while, if f ′(x) and g′(x) exist, with g′(x) nonzero, on an open interval thatcontains a, except possibly at a itself, and if the limit

limx→a

f ′(x)

g′(x)exists or is +∞ or is −∞

then

limx→a

f(x)

g(x)= lim

x→af ′(x)

g′(x)

Proof. We only give the proof for part (a). The proof of part (b) is notvery difficult, but uses the Generalised Mean–Value Theorem (Theorem 3.4.38),which is optional and most readers have not seen it.

• First note that we must have f(a) = g(a) = 0. To see this note that sincederivative f ′(a) exists, we know that the limit

limx→a

f(x)− f(a)

x− a exists

Since we know that the denominator goes to zero, we must also have

textbook on differential calculus. The eponymous rule appears in that text, but is believed tohave been developed by Johann Bernoulli. The book was the source of some controversy since itcontained many results by Bernoulli, which l’Hôpital acknowledged in the preface, but Bernoullifelt that l’Hôpital got undue credit.

Note that around that time l’Hôpital’s name was commonly spelled l’Hospital, but the spellingof silent s in French was changed subsequently; many texts spell his name l’Hospital. If you findyourself in Paris, you can hunt along Boulevard de l’Hôpital for older street signs carved intothe sides of buildings which spell it “l’Hospital” — though arguably there are better things to dothere.

455

Page 472: CLP-1 Differential Calculus

APPLICATIONS OF DERIVATIVES3.7 L’HÔPITAL’S RULE AND INDETERMINATE FORMS

that the numerator goes to zero (otherwise the limit would be undefined).Hence we must have

limx→a

(f(x)− f(a)) =(

limx→a

f(x))− f(a) = 0

We are told that limx→a

f(x) = 0 so we must have f(a) = 0. Similarly we

know that g(a) = 0.

• Now consider the indeterminate form

limx→a

f(x)

g(x)= lim

x→af(x)− 0

g(x)− 0use 0 = f(a) = g(a)

= limx→a

f(x)− f(a)

g(x)− g(a)multiply by 1 =

(x− a)−1

(x− a)−1

= limx→a

f(x)− f(a)

g(x)− g(a)· (x− a)−1

(x− a)−1rearrange

= limx→a

f(x)− f(a)

x− ag(x)− g(a)

x− a

use arithmetic of limits

=limx→a

f(x)− f(a)

x− alimx→a

g(x)− g(a)

x− a

=f ′(a)

g′(a)

We can justify this step and apply Theorem 1.4.3, since the limits in thenumerator and denominator exist, because they are just f ′(a) and g′(a).

3.7.1.1tttOptional — Proof of Part (b) of l’Hôpital’s Rule

To prove part (b) we must work around the possibility that f ′(a) and g′(a) do notexist or that f ′(x) and g′(x) are not continuous at x = a. To do this, we make useof the Generalised Mean-Value Theorem (Theorem 3.4.38) that was used to proveEquation 3.4.33. We recommend you review the GMVT before proceeding.

For simplicity we consider the limit

limx→a+

f(x)

g(x)

By assumption, we know that

limx→a+

f(x) = limx→a+

g(x) = 0

456

Page 473: CLP-1 Differential Calculus

APPLICATIONS OF DERIVATIVES3.7 L’HÔPITAL’S RULE AND INDETERMINATE FORMS

For simplicity, we also assume that f(a) = g(a) = 0. This allows us to write

f(x)

g(x)=f(x)− f(a)

g(x)− g(a)

which is the right form for an application of the GMVT.By assumption f ′(x) and g′(x) exist, with g′(x) nonzero, in some open interval

around a, except possibly at a itself. So we know that they exist, with g′(x) 6= 0, insome interval (a, b] with b > a. Then the GMVT (Theorem 3.4.38) tells us that forx ∈ (a, b]

f(x)

g(x)=f(x)− f(a)

g(x)− g(a)=f ′(c)

g′(c)

where c ∈ (a, x). As we take the limit as x→ a, we also have that c→ a, and so

limx→a+

f(x)

g(x)= lim

x→a+f ′(c)

g′(c)= lim

c→a+f ′(c)

g′(c)

as required.

3.7.2 tt Standard Examples

Here are some simple examples using L’Hôpital’s rule.

Example 3.7.3 Find limx→0

sinx

x.

Consider the limit

limx→0

sinx

x

• Notice that

limx→0

sinx = 0

limx→0

x = 0

so this is a 00

indeterminate form, and suggests we try l’Hôpital’s rule.

• To apply the rule we must first check the limits of the derivatives.

f(x) = sinx f ′(x) = cos x and f ′(0) = 1

g(x) = x g′(x) = 1 and g′(0) = 1

• So by l’Hôpital’s rule

limx→0

sinx

x=f ′(0)

g′(0)=

1

1= 1.

457

Page 474: CLP-1 Differential Calculus

APPLICATIONS OF DERIVATIVES3.7 L’HÔPITAL’S RULE AND INDETERMINATE FORMS

Example 3.7.4 Compute limx→0

sin(x)

sin(2x).

Consider the limit

limx→0

sin(x)

sin(2x)

• First check

limx→0

sin 2x = 0

limx→0

sinx = 0

so we again have a 00

indeterminate form.

• Set f(x) = sinx and g(x) = sin 2x, then

f ′(x) = cos x f ′(0) = 1

g′(x) = 2 cos 2x g′(0) = 2

• And by l’Hôpital’s rule

limx→0

sinx

sin 2x=f ′(0)

g′(0)=

1

2.

Example 3.7.5 limx→0

qx − 1

x.

Let q > 1 and compute the limit

limx→0

qx − 1

x

This limit arose in our discussion of exponential functions in Section 2.7.

• First check

limx→0

(qx − 1) = 1− 1 = 0

limx→0

x = 0

so we have a 00

indeterminate form.

• Set f(x) = qx− 1 and g(x) = x, then (maybe after a quick review of Section 2.7)

f ′(x) =d

dx(qx − 1) = qx · log q f ′(0) = log q

458

Page 475: CLP-1 Differential Calculus

APPLICATIONS OF DERIVATIVES3.7 L’HÔPITAL’S RULE AND INDETERMINATE FORMS

Example 3.7.5

g′(x) = 1 g′(0) = 1

• And by l’Hôpital’s rule a

limh→0

qh − 1

h= log q.

a While it might not be immediately obvious, this example relies on circular reasoning. In order toapply l’Hôpital’s rule, we need to compute the derivative of qx. However in order to computethat limit (see Section 2.7) we needed to evaluate this limit.

A more obvious example of this sort of circular reasoning can be seen if we use l’Hôpital’s ruleto compute the derivative of f(x) = xn at x = a using the limit

f ′(a) = limx→a

xn − anx− a = lim

x→a

nxn−1 − 0

1− 0= nan−1.

We have used the result ddxx

n = nxn−1 to prove itself!

In this example, we shall apply L’Hôpital’s rule twice before getting the answer.

Example 3.7.6 Double L’Hôpital.

Compute the limit

limx→0

sin(x2)

1− cosx

• Again we should check

limx→0

sin(x2) = sin 0 = 0

limx→0

(1− cosx) = 1− cos 0 = 0

and we have a 00

indeterminate form.

• Let f(x) = sin(x2) and g(x) = 1− cosx then

f ′(x) = 2x cos(x2) f ′(0) = 0

g′(x) = sinx g′(0) = 0

So if we try to apply l’Hôpital’s rule naively we will get

limx→0

sin(x2)

1− cosx=f ′(0)

g′(0)=

0

0.

which is another 00

indeterminate form.

459

Page 476: CLP-1 Differential Calculus

APPLICATIONS OF DERIVATIVES3.7 L’HÔPITAL’S RULE AND INDETERMINATE FORMS

Example 3.7.6

• It appears that we are stuck until we remember that l’Hôpital’s rule (as statedin Theorem 3.7.2) has a part (b) — now is a good time to reread it.

• It says that

limx→0

f(x)

g(x)= lim

x→0

f ′(x)

g′(x)

provided this second limit exists. In our case this requires us to compute

limx→0

2x cos(x2)

sin(x)

which we can do using l’Hôpital’s rule again. Now

h(x) = 2x cos(x2) h′(x) = 2 cos(x2)− 4x2 sin(x2) h′(0) = 2

`(x) = sin(x) `′(x) = cos(x) `′(0) = 1

By l’Hôpital’s rule

limx→0

2x cos(x2)

sin(x)=h′(0)

`′(0)= 2

• Thus our original limit is

limx→0

sin(x2)

1− cosx= lim

x→0

2x cos(x2)

sin(x)= 2.

• We can succinctly summarise the two applications of L’Hôpital’s rule in thisexample by

limx→0

sin(x2)

1− cosx︸ ︷︷ ︸num→0den→0

= limx→0

2x cos(x2)

sinx︸ ︷︷ ︸num→0den→0

= limx→0

2 cos(x2)− 4x2 sin(x2)

cosx︸ ︷︷ ︸num→2den→1

= 2

Here “num” and “den” are used as abbreviations of “numerator” and “denom-inator” respectively."

One must be careful to ensure that the hypotheses of l’Hôpital’s rule are satisfiedbefore applying it. The following “warnings” show the sorts of things that can gowrong.

460

Page 477: CLP-1 Differential Calculus

APPLICATIONS OF DERIVATIVES3.7 L’HÔPITAL’S RULE AND INDETERMINATE FORMS

Warning 3.7.7 Denominator limit nonzero.

If

limx→a

f(x) = 0 but limx→a

g(x) 6= 0

then

limx→a

f(x)

g(x)need not be the same as

f ′(a)

g′(a)or lim

x→af ′(x)

g′(x).

Here is an example. Take

a = 0 f(x) = 3x g(x) = 4 + 5x

Then

limx→0

f(x)

g(x)= lim

x→0

3x

4 + 5x=

3× 0

4 + 5× 0= 0

limx→0

f ′(x)

g′(x)=f ′(0)

g′(0)=

3

5

Warning 3.7.8 Numerator limit nonzero.

If

limx→a

g(x) = 0 but limx→a

f(x) 6= 0

then

limx→a

f(x)

g(x)need not be the same as lim

x→af ′(x)

g′(x).

Here is an example. Take

a = 0 f(x) = 4 + 5x g(x) = 3x

Then

limx→0

f(x)

g(x)= lim

x→0

4 + 5x

3x= DNE

limx→0

f ′(x)

g′(x)= lim

x→0

5

3=

5

3

461

Page 478: CLP-1 Differential Calculus

APPLICATIONS OF DERIVATIVES3.7 L’HÔPITAL’S RULE AND INDETERMINATE FORMS

This next one is more subtle; the limits of the original numerator and denomina-tor functions both go to zero, but the limit of the ratio their derivatives does not exist.

Warning 3.7.9 Limit of ratio of derivatives DNE.

If

limx→a

f(x) = 0 and limx→a

g(x) = 0

but

limx→a

f ′(x)

g′(x)does not exist

then it is still possible that

limx→a

f(x)

g(x)exists.

Here is an example. Take

a = 0 f(x) = x2 sin1

xg(x) = x

Then (with an application of the squeeze theorem)

limx→0

f(x) = 0 and limx→0

g(x) = 0.

If we attempt to apply l’Hôptial’s rule then we have g′(x) = 1 and

f ′(x) = 2x sin1

x− cos

1

x

and we then try to compute the limit

limx→0

f ′(x)

g′(x)= lim

x→0

(2x sin

1

x− cos

1

x

)However, this limit does not exist. The first term converges to 0 (by the squeezetheorem), but the second term cos(1/x) just oscillates wildly between ±1. Allwe can conclude from this is

Since the limit of the ratio of derivatives does not exist, we cannotapply l’Hôpital’s rule.

Instead we should go back to the original limit and apply the squeeze theorem:

limx→0

f(x)

g(x)= lim

x→0

x2 sin 1x

x= lim

x→0x sin

1

x= 0,

since |x sin(1/x)| < |x| and |x| → 0 as x→ 0.

462

Page 479: CLP-1 Differential Calculus

APPLICATIONS OF DERIVATIVES3.7 L’HÔPITAL’S RULE AND INDETERMINATE FORMS

It is also easy to construct an example in which the limits of numerator and de-nominator are both zero, but the limit of the ratio and the limit of the ratio of thederivatives do not exist. A slight change of the previous example shows that it ispossible that

limx→a

f(x) = 0 and limx→a

g(x) = 0

but neither of the limits

limx→a

f(x)

g(x)or lim

x→af ′(x)

g′(x)

exist. Take

a = 0 f(x) = x sin1

xg(x) = x

Then (with a quick application of the squeeze theorem)

limx→0

f(x) = 0 and limx→0

g(x) = 0.

However,

limx→0

f(x)

g(x)= lim

x→0

x sin 1x

x= lim

x→0sin

1

x

does not exist. And similarly

limx→0

f ′(x)

g′(x)= lim

x→0

sin 1x− 1

xcos 1

x

x2

does not exist.

3.7.3 tt Variations

Theorem 3.7.2 is the basic form of L’Hôpital’s rule, but there are also many variations.Here are a bunch of them.

3.7.3.1ttt Limits at ±∞

L’Hôpital’s rule also applies when the limit of x → a is replaced by limx→a+

or by limx→a−

or by limx→+∞

or by limx→−∞

.

We can justify adapting the rule to the limits to ±∞ via the following reasoning

limx→∞

f(x)

g(x)= lim

y→0+

f(1/y)

g(1/y)substitute x = 1/y

463

Page 480: CLP-1 Differential Calculus

APPLICATIONS OF DERIVATIVES3.7 L’HÔPITAL’S RULE AND INDETERMINATE FORMS

= limy→0+

− 1y2f ′(1/y)

− 1y2g′(1/y)

,

where we have used l’Hôpital’s rule (assuming this limit exists) and the fact thatddyf(1/y) = − 1

y2f ′(1/y) (and similarly for g). Cleaning this up and substituting y =

1/x gives the required result:

limx→∞

f(x)

g(x)= lim

y→0+

f ′(1/y)

g′(1/y)= lim

x→∞f ′(x)

g′(x).

Example 3.7.10 L’Hôpital at infinity.

Consider the limit

limx→∞

arctanx− π2

1x

Both numerator and denominator go to 0 as x → ∞, so this is an 00

indeterminateform. We find

limx→+∞

arctanx− π2

1x︸ ︷︷ ︸

num→0den→0

= limx→+∞

11+x2

− 1x2

= − limx→+∞

1

1 + 1x2︸ ︷︷ ︸

num→1den→1

= −1

We have applied L’Hôpital’s rule with

f(x) = arctan x− π

2g(x) =

1

x

f ′(x) =1

1 + x2g′(x) = − 1

x2

3.7.3.2ttt ∞∞ indeterminate form

L’Hôpital’s rule also applies when limx→a

f(x) = 0, limx→a

g(x) = 0 is replaced by limx→a

f(x) =

±∞, limx→a

g(x) = ±∞.

Example 3.7.11 Compute limx→∞

log x

x.

Consider the limit

limx→∞

log x

x

464

Page 481: CLP-1 Differential Calculus

APPLICATIONS OF DERIVATIVES3.7 L’HÔPITAL’S RULE AND INDETERMINATE FORMS

Example 3.7.11

The numerator and denominator both blow up towards infinity so this is an ∞∞ inde-terminate form. An application of l’Hôpital’s rule gives

limx→∞

log x

x︸ ︷︷ ︸num→∞den→∞

= limx→∞

1/x

1

= limx→∞

1

x= 0

Example 3.7.12 Find limx→∞

5x2 + 3x− 3

x2 + 1.

Consider the limit

limx→∞

5x2 + 3x− 3

x2 + 1

Then by two applications of l’Hôpital’s rule we get

limx→∞

5x2 + 3x− 3

x2 + 1︸ ︷︷ ︸num→∞den→∞

= limx→∞

10x+ 3

2x︸ ︷︷ ︸num→∞den→∞

= limx→∞

10

2= 5.

Example 3.7.13 A messier double l’Hôpital.

Compute the limit

limx→0+

log x

tan(π2− x)

We can compute this using l’Hôpital’s rule twice:

limx→0+

log x

tan(π2− x)︸ ︷︷ ︸

num→−∞den→+∞

= limx→0+

1x

− sec2(π2− x)

= − limx→0+

cos2(π2− x)

x︸ ︷︷ ︸num→0den→0

= − limx→0+

2 cos(π2− x) sin(π

2− x)

1︸ ︷︷ ︸num→0den→1

= 0

The first application of L’Hôpital’s was with

f(x) = log x g(x) = tan(π

2− x)

465

Page 482: CLP-1 Differential Calculus

APPLICATIONS OF DERIVATIVES3.7 L’HÔPITAL’S RULE AND INDETERMINATE FORMS

Example 3.7.13

f ′(x) =1

xg′(x) = − sec2

(π2− x)

and the second time with

f(x) = cos2(π

2− x)

g(x) = x

f ′(x) = 2 cos(π

2− x)[− sin

(π2− x)]

(−1) g′(x) = 1

Sometimes things don’t quite work out as we would like and l’Hôpital’s rule canget stuck in a loop. Remember to think about the problem before you apply any rule.

Example 3.7.14 Stuck in a loop.

Consider the limit

limx→∞

ex + e−x

ex − e−x

Clearly both numerator and denominator go to ∞, so we have a ∞∞ indeterminateform. Naively applying l’Hôpital’s rule gives

limx→∞

ex + e−x

ex − e−x = limx→∞

ex − e−xex + e−x

which is again a ∞∞ indeterminate form. So apply l’Hôpital’s rule again:

limx→∞

ex − e−xex + e−x

= limx→∞

ex + e−x

ex − e−x

which is right back where we started!The correct approach to such a limit is to apply the methods we learned in Chapter 1and rewrite

ex + e−x

ex − e−x =ex(1 + e−2x)

ex(1− e−2x)=

1 + e−2x

1− e−2x

and then take the limit.A similar sort of l’Hôpital-rule-loop will occur if you naively apply l’Hôpital’s ruleto the limit

limx→∞

√4x2 + 1

5x− 1

which appeared in Example 1.5.6.

466

Page 483: CLP-1 Differential Calculus

APPLICATIONS OF DERIVATIVES3.7 L’HÔPITAL’S RULE AND INDETERMINATE FORMS

3.7.3.3tttOptional — Proof of l’Hôpital’s Rule for ∞∞

We can justify this generalisation of l’Hôpital’s rule with some careful manipula-tions. Since the derivatives f ′, g′ exist in some interval around a, we know that f, gare continuous in some interval around a; let x, t be points inside that interval. Nowrewrite 3

f(x)

g(x)=f(x)

g(x)+

(f(t)

g(x)− f(t)

g(x)

)︸ ︷︷ ︸

=0

+

(f(x)− f(t)

g(x)− g(t)− f(x)− f(t)

g(x)− g(t)

)︸ ︷︷ ︸

=0

=f(x)− f(t)

g(x)− g(t)︸ ︷︷ ︸ready for GMVT

+f(t)

g(x)+

(f(x)

g(x)− f(t)

g(x)− f(x)− f(t)

g(x)− g(t)

)︸ ︷︷ ︸

we can clean it up

=f(x)− f(t)

g(x)− g(t)+f(t)

g(x)+

(f(x)− f(t)

g(x)− f(x)− f(t)

g(x)− g(t)

)=f(x)− f(t)

g(x)− g(t)+f(t)

g(x)+

(1

g(x)− 1

g(x)− g(t)

)· (f(x)− f(t))

=f(x)− f(t)

g(x)− g(t)+f(t)

g(x)+

(g(x)− g(t)− g(x)

g(x)(g(x)− g(t))

)· (f(x)− f(t))

=f(x)− f(t)

g(x)− g(t)︸ ︷︷ ︸ready for GMVT

+f(t)

g(x)− g(t)

g(x)· f(x)− f(t)

g(x)− g(t)︸ ︷︷ ︸ready for GMVT

Oof! Now the generalised mean-value theorem (Theorem 3.4.38) tells us there is a cbetween x and t so that

f(x)− f(t)

g(x)− g(t)=f ′(c)

g′(c)

Now substitute this into the large expression we derived above:

f(x)

g(x)=f ′(c)

g′(c)+

1

g(x)

(f(t)− f ′(c)

g′(c)· g(t)

)At first glance this does not appear so useful, however if we fix t and take the limitas x→ a, then it becomes

limx→a

f(x)

g(x)= lim

x→af ′(c)

g′(c)+ lim

x→a1

g(x)

(f(t)− f ′(c)

g′(c)· g(t)

)Since g(x)→∞ as x→ a, this last term goes to zero

= limx→a

f ′(c)

g′(c)+ 0

3 This is quite a clever argument, but it is not immediately obvious why one rewrites things thisway. After the fact it becomes clear that it is done to massage the expression into the form wherewe can apply the generalised mean-value theorem (Theorem 3.4.38).

467

Page 484: CLP-1 Differential Calculus

APPLICATIONS OF DERIVATIVES3.7 L’HÔPITAL’S RULE AND INDETERMINATE FORMS

Now take the limit as t→ a. The left-hand side is unchanged since it is independentof t. The right-hand side, however, does change; the number c is trapped between xand t. Since we have already taken the limit x→ a, so when we take the limit t→ a,we are effectively taking the limit c→ a. Hence

limx→a

f(x)

g(x)= lim

c→af ′(c)

g′(c)

which is the desired result.

3.7.3.4ttt 0 · ∞ indeterminate form

When limx→a

f(x) = 0 and limx→a

g(x) = ∞. We can use a little algebra to manipulate this

into either a 00

or ∞∞ form:

limx→a

f(x)

1/g(x)limx→a

g(x)

1/f(x)

Example 3.7.15 limx→0+

x · log x.

Consider the limit

limx→0+

x · log x

Here the function f(x) = x goes to zero, while g(x) = log x goes to−∞. If we rewritethis as the fraction

x · log x =log x

1/x

then the 0 · ∞ form has become an ∞∞ form.The result is then

limx→0+

x︸︷︷︸→0

log x︸︷︷︸→−∞

= limx→0+

log x1x︸ ︷︷ ︸

num→−∞den→∞

= limx→0+

1x

− 1x2

= − limx→0+

x = 0

Example 3.7.16 Computing limx→+∞

xne−x.

In this example we’ll evaluate limx→+∞

xne−x, for all natural numbers n. We’ll start with

n = 1 and n = 2 and then, using what we have learned from those cases, move on to

468

Page 485: CLP-1 Differential Calculus

APPLICATIONS OF DERIVATIVES3.7 L’HÔPITAL’S RULE AND INDETERMINATE FORMS

Example 3.7.16

general n.

limx→+∞

x︸︷︷︸→∞

e−x︸︷︷︸→0

= limx→+∞

x

ex︸︷︷︸num→+∞den→+∞

= limx→+∞

1

ex︸︷︷︸num→1

den→+∞

= limx→+∞

e−x = 0

Applying l’Hôpital twice,

limx→+∞

x2︸︷︷︸→∞

e−x︸︷︷︸→0

= limx→+∞

x2

ex︸︷︷︸num→+∞den→+∞

= limx→+∞

2x

ex︸︷︷︸num→∞den→+∞

= limx→+∞

2

ex︸︷︷︸num→2

den→+∞

= limx→+∞

2e−x

= 0

Indeed, for any natural number n, applying l’Hôpital n times gives

limx→+∞

xn︸︷︷︸→∞

e−x︸︷︷︸→0

= limx→+∞

xn

ex︸︷︷︸num→+∞den→+∞

= limx→+∞

nxn−1

ex︸ ︷︷ ︸num→∞den→+∞

= limx→+∞

n(n− 1)xn−2

ex︸ ︷︷ ︸num→∞den→+∞

= · · · = limx→+∞

n!

ex︸︷︷︸num→n!den→+∞

= 0

3.7.3.5ttt∞−∞ indeterminate form

When limx→a

f(x) = ∞ and limx→a

g(x) = ∞. We rewrite the difference as a fraction usinga common denominator

f(x)− g(x) =h(x)

`(x)

which is then a 00

or ∞∞ form.

469

Page 486: CLP-1 Differential Calculus

APPLICATIONS OF DERIVATIVES3.7 L’HÔPITAL’S RULE AND INDETERMINATE FORMS

Example 3.7.17 Compute limx→π

2−

(secx− tanx).

Consider the limit

limx→π

2−

(secx− tanx)

Since the limit of both secx and tanx is +∞ as x→ π2−, this is an∞−∞ indeterminate

form. However we can rewrite this as

secx− tanx =1

cosx− sinx

cosx=

1− sinx

cosx

which is then a 00

indeterminate form. This then gives

limx→π

2−

(secx︸︷︷︸→+∞

− tanx︸ ︷︷ ︸→+∞

)= lim

x→π2−

1− sinx

cosx︸ ︷︷ ︸num→0den→0

= limx→π

2−

− cosx

− sinx︸ ︷︷ ︸num→0den→−1

= 0

In the last example, Example 3.7.17, we converted an∞−∞ indeterminate forminto a 0

0indeterminate form by exploiting the fact that the two terms, secx and tanx,

in the∞−∞ indeterminate form shared a common denominator, namely cosx. Inthe “real world” that will, of course, almost never happen. However as the next cou-ple of examples show, you can often massage these expressions into suitable forms.

Here is another, much more complicated, example, where it doesn’t happen.

Example 3.7.18 A complicated∞−∞ example.

In this example, we evaluate the∞−∞ indeterminate form

limx→0

( 1

x︸︷︷︸→±∞

− 1

log(1 + x)︸ ︷︷ ︸→±∞

)

We convert it into a 00

indeterminate form simply by putting the two fractions, 1x

and1

log(1+x)over a common denominator.

limx→0

( 1

x︸︷︷︸→±∞

− 1

log(1 + x)︸ ︷︷ ︸→±∞

)= lim

x→0

log(1 + x)− xx log(1 + x)︸ ︷︷ ︸

num→0den→0

(E1)

Now we apply L’Hôpital’s rule, and simplify

limx→0

log(1 + x)− xx log(1 + x)︸ ︷︷ ︸

num→0den→0

= limx→0

11+x− 1

log(1 + x) + x1+x

470

Page 487: CLP-1 Differential Calculus

APPLICATIONS OF DERIVATIVES3.7 L’HÔPITAL’S RULE AND INDETERMINATE FORMS

Example 3.7.18

= limx→0

1− (1 + x)

(1 + x) log(1 + x) + x

= − limx→0

x

(1 + x) log(1 + x) + x︸ ︷︷ ︸num→0

den→1×0+0=0

(E2)

Then we apply L’Hôpital’s rule a second time

− limx→0

x

(1 + x) log(1 + x) + x︸ ︷︷ ︸num→0

den→1×0+0=0

= − limx→0

1

log(1 + x) + 1+x1+x

+ 1︸ ︷︷ ︸num→1

den→0+1+1=2

= −1

2(E3)

Combining (E1), (E2) and (E3) gives our final answer

limx→0

(1

x− 1

log(1 + x)

)= −1

2

The following example can be done by l’Hôpital’s rule, but it is actually far sim-pler to multiply by the conjugate and take the limit using the tools of Chapter 1.

Example 3.7.19 Compute limx→∞

√x2 + 4x−

√x2 − 3x.

Consider the limit

limx→∞

√x2 + 4x−

√x2 − 3x

Neither term is a fraction, but we can write√x2 + 4x−

√x2 − 3x = x

√1 + 4/x− x

√1− 3/x assuming x > 0

= x(√

1 + 4/x−√

1− 3/x)

=

√1 + 4/x−

√1− 3/x

1/x

which is now a 00

form with f(x) =√

1 + 4/x−√

1− 3/x and g(x) = 1/x. Then

f ′(x) =−4/x2

2√

1 + 4/x− 3/x2

2√

1− 3/xg′(x) = − 1

x2

Hencef ′(x)

g′(x)=

4

2√

1 + 4/x+

3√1− 3/x

471

Page 488: CLP-1 Differential Calculus

APPLICATIONS OF DERIVATIVES3.7 L’HÔPITAL’S RULE AND INDETERMINATE FORMS

Example 3.7.19

And so in the limit as x→∞

limx→∞

f ′(x)

g′(x)=

4

2+

3

2=

7

2

and so our original limit is also 7/2.By comparison, if we multiply by the conjugate we have

√x2+4x−

√x2−3x =

(√x2+4x−

√x2−3x

)·√x2+4x+

√x2−3x√

x2+4x+√x2−3x

=x2 + 4x− (x2 − 3x)√x2 + 4x+

√x2 − 3x

=7x√

x2 + 4x+√x2 − 3x

=7√

1 + 4/x+√

1− 3/xassuming x > 0

Now taking the limit as x → ∞ gives 7/2 as required. Just because we knowl’Hôpital’s rule, it does not mean we should use it everywhere it might be applied.

3.7.3.6ttt 1∞ indeterminate form

We can use l’Hôpital’s rule on limits of the form

limx→a

f(x)g(x) with

limx→a

f(x) = 1 and limx→a

g(x) =∞

by considering the logarithm of the limit 4 :

log(

limx→a

f(x)g(x))

= limx→a

log(f(x)g(x)

)= lim

x→alog (f(x)) · g(x)

which is now an 0 · ∞ form. This can be further transformed into a 00

or ∞∞ form:

log(

limx→a

f(x)g(x))

= limx→a

log (f(x)) · g(x)

= limx→a

log (f(x))

1/g(x).

4 We are using the fact that the logarithm is a continuous function and Theorem 1.6.10.

472

Page 489: CLP-1 Differential Calculus

APPLICATIONS OF DERIVATIVES3.7 L’HÔPITAL’S RULE AND INDETERMINATE FORMS

Example 3.7.20 Find limx→0

(1 + x)ax .

The following limit appears quite naturally when considering systems which displayexponential growth or decay.

limx→0

(1 + x)ax with the constant a 6= 0

Since (1 + x)→ 1 and a/x→∞ this is an 1∞ indeterminate form.By considering its logarithm we have

log(

limx→0

(1 + x)ax

)= lim

x→0log((1 + x)

ax

)= lim

x→0

a

xlog(1 + x)

= limx→0

a log(1 + x)

x

which is now a 00

form. Applying l’Hôpital’s rule gives

limx→0

a log(1 + x)

x︸ ︷︷ ︸num→0den→0

= limx→0

a1+x

1︸︷︷︸num→aden→1

= a

Since (1 + x)a/x = exp[log(

(1 + x)a/x)]

and the exponential function is continuous,our original limit is ea.

Here is a more complicated example of a 1∞ indeterminate form.

Example 3.7.21 A more complicated example.

In the limit

limx→0

(sinx

x

) 1x2

the base, sinxx

, converges to 1 (see Example 3.7.3) and the exponent, 1x2

, goes to ∞.But if we take logarithms then

log(sinx

x

) 1x2

=log sinx

x

x2

then, in the limit x → 0, we have a 00

indeterminate form. One application of

473

Page 490: CLP-1 Differential Calculus

APPLICATIONS OF DERIVATIVES3.7 L’HÔPITAL’S RULE AND INDETERMINATE FORMS

Example 3.7.21

l’Hôpital’s rule gives

limx→0

log sinxx

x2︸ ︷︷ ︸num→0den→0

= limx→0

xsinx

x cosx−sinxx2

2x= lim

x→0

x cosx−sinxx sinx

2x= lim

x→0

x cosx− sinx

2x2 sinx

which is another 00

form. Applying l’Hôpital’s rule again gives:

limx→0

x cosx− sinx

2x2 sinx︸ ︷︷ ︸num→0den→0

= limx→0

cosx− x sinx− cosx

4x sinx+ 2x2 cosx

= − limx→0

x sinx

4x sinx+ 2x2 cosx= − lim

x→0

sinx

4 sinx+ 2x cosx

which is yet another 00

form. Once more with l’Hôpital’s rule:

− limx→0

sinx

4 sinx+ 2x cosx︸ ︷︷ ︸num→0den→0

= − limx→0

cosx

4 cosx+ 2 cosx− 2x sinx︸ ︷︷ ︸num→1den→6

= −1

6

Oof! We have just shown that the logarithm of our original limit is −16. Hence the

original limit itself is e−1/6.This was quite a complicated example. However it does illustrate the importance ofcleaning up your algebraic expressions. This will both reduce the amount of workyou have to do and will also reduce the number of errors you make.

3.7.3.7ttt 00indeterminate form

Like the 1∞ form, this can be treated by considering its logarithm.

Example 3.7.22 Compute limx→0+

xx.

For example, in the limit

limx→0+

xx

both the base, x, and the exponent, also x, go to zero. But if we consider the logarithmthen we have

log xx = x log x

474

Page 491: CLP-1 Differential Calculus

APPLICATIONS OF DERIVATIVES3.7 L’HÔPITAL’S RULE AND INDETERMINATE FORMS

Example 3.7.22

which is a 0 · ∞ indeterminate form, which we already know how to treat. In fact,we already found, in Example 3.7.15, that

limx→0+

x log x = 0

Since the exponential is a continuous function

limx→0+

xx = limx→0+

exp(x log x

)= exp

(limx→0+

x log x)

= e0 = 1

3.7.3.8ttt∞0 indeterminate form

Again, we can treat this form by considering its logarithm.

Example 3.7.23 Find limx→+∞

x1x .

For example, in the limit

limx→+∞

x1x

the base, x, goes to infinity and the exponent, 1x, goes to zero. But if we take loga-

rithms

log x1x =

log x

x

which is an ∞∞ form, which we know how to treat.

limx→+∞

log x

x︸ ︷︷ ︸num→∞den→∞

= limx→+∞

1x

1︸︷︷︸num→0den→1

= 0

Since the exponential is a continuous function

limx→+∞

x1x = lim

x→+∞exp

( log x

x

)= exp

(limx→∞

log x

x

)= e0 = 1

475

Page 492: CLP-1 Differential Calculus

APPLICATIONS OF DERIVATIVES3.7 L’HÔPITAL’S RULE AND INDETERMINATE FORMS

3.7.4 tt Exercises

Exercises — Stage 1 In Questions 3.7.4.1 to 3.7.4.3, you are asked to give pairs offunctions that combine to make indeterminate forms. Remember that an indetermi-nate form is indeterminate precisely because its limit can take on a number of values.

1. Give two functions f(x) and g(x) with the following properties:

i limx→∞

f(x) =∞

ii limx→∞

g(x) =∞

iii limx→∞

f(x)

g(x)= 2.5

2. Give two functions f(x) and g(x) with the following properties:

i limx→∞

f(x) =∞

ii limx→∞

g(x) =∞

iii limx→∞

f(x)

g(x)= 0

3. Give two functions f(x) and g(x) with the following properties:

i limx→∞

f(x) = 1

ii limx→∞

g(x) =∞

iii limx→∞

[f(x)]g(x) = 5

Exercises — Stage 2

4. *. Evaluate limx→1

x3 − ex−1

sin(πx).

5. *. Evaluate limx→0+

log x

x. (Remember: in these notes, log means logarithm

base e.)

6. *. Evaluate limx→∞

(log x)2e−x.

476

Page 493: CLP-1 Differential Calculus

APPLICATIONS OF DERIVATIVES3.7 L’HÔPITAL’S RULE AND INDETERMINATE FORMS

7. *. Evaluate limx→∞

x2e−x.

8. *. Evaluate limx→0

x− x cosx

x− sinx.

9. Evaluate limx→0

√x6 + 4x4

x2 cosx.

10. *. Evaluate limx→∞

(log x)2

x.

11. *. Evaluate limx→0

1− cosx

sin2 x.

12. Evaluate limx→0

x

secx.

13. Evaluate limx→0

cscx · tanx · (x2 + 5)

ex.

14. Evaluate limx→∞

√2x2 + 1−

√x2 + x.

15. *. Evaluate limx→0

sin(x3 + 3x2)

sin2 x.

16. *. Evaluate limx→1

log(x3)

x2 − 1.

17. *. Evaluate limx→0

e−1/x2

x4.

18. *. Evaluate limx→0

xex

tan(3x).

19. Evaluate limx→0

x2√

sin2 x.

20. Evaluate limx→0

x2√

cosx.

21. Evaluate limx→0+

ex log x.

22. Evaluate limx→0

[− log(x2)

]x.

23. *. Find c so that limx→0

1 + cx− cosx

ex2 − 1exists.

477

Page 494: CLP-1 Differential Calculus

APPLICATIONS OF DERIVATIVES3.7 L’HÔPITAL’S RULE AND INDETERMINATE FORMS

24. *. Evaluate limx→0

ek sin(x2) − (1 + 2x2)

x4, where k is a constant.

Exercises — Stage 3

25. Suppose an algorithm, given an input with with n variables, will terminatein at most S(n) = 5n4 − 13n3 − 4n + log(n) steps. A researcher writes thatthe algorithm will terminate in roughly at most A(n) = 5n4 steps. Show thatthe percentage error involved in using A(n) instead of S(n) tends to zero asn gets very large. What happens to the absolute error?Remark: this is a very common kind of approximation. When people dealwith functions that give very large numbers, often they don’t care about theexact large number–they only want a ballpark. So, a complicated functionmight be replaced by an easier function that doesn’t give a large relativeerror. If you would like to know more about the ways people describe func-tions that give very large numbers, you can read about “Big O Notation” inSection 3.6.3 of the CLP2 textbook.

478

Page 495: CLP-1 Differential Calculus

TOWARDS INTEGRAL CALCULUS

Chapter 4

We have now come to the final topic of the course — antiderivatives. This is only ashort section since it is really just to give a taste of the next calculus subject: integralcalculus.

4.1q Introduction to Antiderivatives

4.1.1 tt Introduction to Antiderivatives

So far in the course we have learned how to determine the rate of change (i.e. thederivative) of a given function. That is

given a function f(x) finddf

dx.

Along the way we developed an understanding of limits, which allowed us to de-fine instantaneous rates of change — the derivative. We then went on to develop anumber of applications of derivatives to modelling and approximation. In this lastsection we want to just introduce the idea of antiderivatives. That is

given a derivativedf

dxfind the original function f(x).

For example — say we know that

df

dx= x2

479

Page 496: CLP-1 Differential Calculus

TOWARDS INTEGRAL CALCULUS 4.1 INTRODUCTION TO ANTIDERIVATIVES

and we want to find f(x). From our previous experience differentiating we knowthat derivatives of polynomials are again polynomials. So we guess that our un-known function f(x) is a polynomial. Further we know that when we differentiatexn we get nxn−1 — multiply by the exponent and reduce the exponent by 1. So toend up with a derivative of x2 we need to have differentiated an x3. But d

dxx3 = 3x2,

so we need to divide both sides by 3 to get the answer we want. That is

d

dx

(1

3x3

)= x2

However we know that the derivative of a constant is zero, so we also have

d

dx

(1

3x3 + 1

)= x2

and

d

dx

(1

3x3 − π

)= x2

At this point it will really help the discussion to give a name to what we are doing.

Definition 4.1.1

A function F (x) that satisfies

d

dxF (x) = f(x)

is called an antiderivative of f(x).

Notice the use of the indefinite article there — an antiderivative. This is preciselybecause we can always add or subtract a constant to an antiderivative and whenwe differentiate we’ll get the same answer. We can write this as a lemma, but it isactually just Corollary 2.13.13 (from back in the section on the mean-value theorem)in disguise.

Lemma 4.1.2

Let F (x) be an antiderivative of f(x), then for any constant c, the functionF (x) + c is also an antiderivative of f(x).

Because of this lemma we typically write antiderivatives with “+c” tacked on theend. That is, if we know that F ′(x) = f(x), then we would state that the antideriva-tive of f(x) is

F (x) + c

480

Page 497: CLP-1 Differential Calculus

TOWARDS INTEGRAL CALCULUS 4.1 INTRODUCTION TO ANTIDERIVATIVES

where this “+c” is there to remind us that we can always add or subtract some con-stant and it will still be an antiderivative of f(x). Hence the antiderivative of x2

is1

3x3 + c

Similarly, the antiderivative of x4 is

1

5x5 + c

and for√x = x1/2 it is

2

3x3/2 + c

This last one is tricky (at first glance) — but we can always check our answer bydifferentiating.

d

dx

(2

3x3/2 + c

)=

2

3· 3

2x1/2 + 0 X

Now in order to determine the value of c we need more information. For exam-ple, we might be asked

Given that g′(t) = t2 and g(3) = 7 find g(t).

We are given the derivative and one piece of additional information and from thesetwo facts we need to find the original function. From our work above we know that

g(t) =1

3t3 + c

and we can find c from the other piece of information

7 = g(3) =1

3· 27 + c = 9 + c

Hence c = −2 and so

g(t) =1

3t3 − 2

We can then very easily check our answer by recomputing g(3) and g′(t). This is agood habit to get into.

Finding antiderivatives of polynomials is generally not too hard. We just need touse the rule

if f(x) = xn then F (x) =1

n+ 1xn+1 + c.

Of course this breaks down when n = −1. In order to find an antiderivative forf(x) = 1

xwe need to remember that d

dxlog x = 1

x, and more generally that d

dxlog |x| =

1x. See Example 2.10.4. So

if f(x) =1

xthen F (x) = log |x|+ c

481

Page 498: CLP-1 Differential Calculus

TOWARDS INTEGRAL CALCULUS 4.1 INTRODUCTION TO ANTIDERIVATIVES

Example 4.1.3 Antiderivative of 3x5 − 7x2 + 2x+ 3 + x−1 − x−2.

Let f(x) = 3x5 − 7x2 + 2x+ 3 + x−1 − x−2. Then the antiderivative of f(x) is

F (x) =3

6x6 − 7

3x3 +

2

2x2 + 3x+ log |x| − 1

−1x−1 + c clean it up

=1

2x6 − 7

3x3 + x2 + 3x+ log |x|+ x−1 + c

Now to check we should differentiate and hopefully we get back to where we started

F ′(x) =6

2x5 − 7

3· 3x2 + 2x+ 3 +

1

x− x−2

= 3x5 − 7x2 + 2x+ 3 +1

x− x−2 X

In your next calculus course you will develop a lot of machinery to help you findantiderivatives. At this stage about all that we can do is continue the sort of thingwe have done. Think about the derivatives we know and work backwards. So, forexample, we can take a list of derivatives

F (x) 1 xn sinx cosx tanx ex ln |x| arcsinx arctanx

f(x) = ddxF (x) 0 nxn−1 cosx − sinx sec2 x ex 1

x1√

1−x21

1+x2

and flip it upside down to give the tables of antiderivatives.

f(x) = ddxF (x) 0 nxn−1 cosx − sinx sec2 x ex 1

x

F (x) c xn + c sinx+ c cosx+ c tanx+ c ex + c ln |x|+ c

f(x) = ddxF (x) 1√

1−x21

1+x2

F (x) arcsinx+ c arctanx+ c

Here c is just a constant — any constant. But we can do a little more; clean upxn by dividing by n and then replacing n by n + 1. Similarly we can tweak sinx bymultiplying by −1:

f(x) = ddxF (x) 0 xn cosx sinx sec2 x ex 1

x

F (x) c 1n+1

xn+1 + c sinx+ c − cosx+ c tanx+ c ex + c ln |x|+ c

f(x) = ddxF (x) 1√

1−x21

1+x2

F (x) arcsinx+ c arctanx+ c

Here are a couple more examples.

482

Page 499: CLP-1 Differential Calculus

TOWARDS INTEGRAL CALCULUS 4.1 INTRODUCTION TO ANTIDERIVATIVES

Example 4.1.4 Antiderivatives of sinx, cos 2x and 11+4x2

.

Consider the functions

f(x) = sinx+ cos 2x g(x) =1

1 + 4x2

Find their antiderivatives.Solution The first one we can almost just look up our table. Let F be the antideriva-tive of f , then

F (x) = − cosx+ sin 2x+ c is not quite right.

When we differentiate to check things, we get a factor of two coming from the chainrule. Hence to compensate for that we multiply sin 2x by 1

2:

F (x) = − cosx+1

2sin 2x+ c

Differentiating this shows that we have the right answer.Similarly, if we use G to denote the antiderivative of g, then it appears that G isnearly arctanx. To get this extra factor of 4 we need to substitute x 7→ 2x. So we try

G(x) = arctan(2x) + c which is nearly correct.

Differentiating this gives us

G′(x) =2

1 + (2x)2= 2g(x)

Hence we should multiply by 12. This gives us

G(x) =1

2arctan(2x) + c.

We can then check that this is, in fact, correct just by differentiating.

Now let’s do a more substantial example.

Example 4.1.5 Position as antiderivative of velocity.

Suppose that we are driving to class. We start at x = 0 at time t = 0. Our velocity isv(t) = 50 sin(t). The class is at x = 25. When do we get there?Solution Let’s denote by x(t) our position at time t. We are told that

• x(0) = 0

• x′(t) = 50 sin t

483

Page 500: CLP-1 Differential Calculus

TOWARDS INTEGRAL CALCULUS 4.1 INTRODUCTION TO ANTIDERIVATIVES

Example 4.1.5

We have to determine x(t) and then find the time T that obeys x(T ) = 25. Nowarmed with our table above we know that the antiderivative of sin t is just − cos t.We can check this:

d

dt(− cos t) = sin t

We can then get the factor of 50 by multiplying both sides of the above equation by50:

d

dt(−50 cos t) = 50 sin t

And of course, this is just an antiderivative of 50 sin t; to write down the generalantiderivative we just add a constant c:

d

dt(−50 cos t+ c) = 50 sin t

Since v(t) = ddtx(t), this antiderivative is x(t):

x(t) = −50 cos t+ c

To determine c we make use of the other piece of information we are given, namely

x(0) = 0.

Substituting this in gives us

x(0) = −50 cos 0 + c = −50 + c

Hence we must have c = 50 and so

x(t) = −50 cos t+ 50 = 50(1− cos t).

Now that we have our position as a function of time, we can determine how long ittakes us to arrive there. That is, we can find the time T so that x(T ) = 25.

25 = x(T ) = 50(1− cosT ) so1

2= 1− cosT

−1

2= − cosT

1

2= cosT.

Recalling our special triangles, we see that T = π3.

484

Page 501: CLP-1 Differential Calculus

TOWARDS INTEGRAL CALCULUS 4.1 INTRODUCTION TO ANTIDERIVATIVES

The example below shows how antiderivatives arise naturally when studyingdifferential equations.

Example 4.1.6 Theorem 3.3.2 revisited..

Back in Section 3.3 we encountered a simple differential equation, namely equa-tion 3.3.1. We were able to solve this equation by guessing the answer and thenchecking it carefully. We can derive the solution more systematically by using an-tiderivatives.Recall equation 3.3.1:

dQ

dt= −kQ

where Q(t) is the amount of radioactive material at time t and we assume Q(t) > 0.Take this equation and divide both sides by Q(t) to get

1

Q(t)

dQ

dt= −k

At this point we should a think that the left-hand side is familiar. Now is a goodmoment to look back at logarithmic differentiation in Section 2.10.The left-hand side is just the derivative of logQ(t):

d

dt(logQ(t)) =

1

Q(t)

dQ

dt

= −k

So to solve this equation, we are really being asked to find all functions logQ(t)having derivative −k. That is, we need to find all antiderivatives of −k. Of coursethat is just −kt+ c. Hence we must have

logQ(t) = −kt+ c

and then taking the exponential of both sides gives

Q(t) = e−kt+c = ec · e−kt = Ce−kt

where C = ec. This is precisely Theorem 3.3.2.

a Well — perhaps it is better to say “notice that”. Let’s not make this a moral point.

The above is a small example of the interplay between antiderivatives and differ-ential equations.

Here is another example of how we might use antidifferentiation to computeareas or volumes.

485

Page 502: CLP-1 Differential Calculus

TOWARDS INTEGRAL CALCULUS 4.1 INTRODUCTION TO ANTIDERIVATIVES

Example 4.1.7 Volume of a cone.

We know (especially if one has revised the material in the appendix and Ap-pendix B.5.2 in particular) that the volume of a right-circular cone is

V =π

3r2h

where h is the height of the cone and r is the radius of its base. Now, the derivationof this formula given in Appendix B.5.2 is not too simple. We present an alternateproof here that uses antiderivatives.

Consider cutting off a portion of the cone so that its new height is x (rather than h).Call the volume of the resulting smaller cone V (x). We are going to determine V (x)for all x ≥ 0, including x = h, by first evaluating V ′(x) and V (0) (which is obviously0).Call the radius of the base of the new smaller cone y (rather than r). By similartriangles we know that

r

h=y

x.

Now keep x and y fixed and consider cutting off a little more of the cone so its heightisX . When we do so, the radius of the base changes from y to Y and again by similartriangles we know that

Y

X=y

x=r

h

The change in volume is then

V (x)− V (X)

Of course if we knew the formula for the volume of a cone, then we could computethe above exactly. However, even without knowing the volume of a cone, it is easyto derive upper and lower bounds on this quantity. The piece removed has bottomradius y and top radius Y . Hence its volume is bounded above and below by thecylinders of height x−X and with radius y and Y respectively. Hence

πY 2(x−X) ≤ V (x)− V (X) ≤ πy2(x−X)

486

Page 503: CLP-1 Differential Calculus

TOWARDS INTEGRAL CALCULUS 4.1 INTRODUCTION TO ANTIDERIVATIVES

Example 4.1.7

since the volume of a cylinder is just the area of its base times its height. Now mas-sage this expression a little

πY 2 ≤ V (x)− V (X)

x−X ≤ πy2

The middle term now looks like a derivative; all we need to do is take the limit asX → x:

limX→x

πY 2 ≤ limX→x

V (x)− V (X)

x−X ≤ limX→x

πy2

The rightmost term is independent of X and so is just πy2. In the leftmost term, asX → x, we must have that Y → y. Hence the leftmost term is just πy2. Then by thesqueeze theorem (Theorem 1.4.18) we know that

dV

dx= lim

X→x

V (x)− V (X)

x−X = πy2.

But we know that

y =r

h· x

so

dV

dx= π

( rh

)2

x2

Now we can antidifferentiate to get back to V :

V (x) =π

3

( rh

)2

x3 + c

To determine c notice that when x = 0 the volume of the cone is just zero and soc = 0. Thus

V (x) =π

3

( rh

)2

x3

and so when x = h we are left with

V (h) =π

3

( rh

)2

h3 =π

3r2h

as required.

487

Page 504: CLP-1 Differential Calculus

TOWARDS INTEGRAL CALCULUS 4.1 INTRODUCTION TO ANTIDERIVATIVES

4.1.2 tt Exercises

Exercises — Stage 1

1. Let f(x) be a function with derivative f ′(x). What is the most general an-tiderivative of f ′(x)?

2. On the graph below, the black curve is y = f(x). Which of the colouredcurves is an antiderivative of f(x)?

x

y

f(x)

C(x)

A(x)

B(x)

Exercises — Stage 2 In Questions 4.1.2.3 through 4.1.2.12, you are asked to find theantiderivative of a function. Phrased like this, we mean the most general antideriva-tive. These will all include some added constant. The table after Example 4.1.3might be of help.In Questions 4.1.2.13 through 4.1.2.16, you are asked to find a spe-cific antiderivative of a function. In this case, you should be able to solve for theentire function–no unknown constants floating about.In Questions 4.1.2.17 through4.1.2.19, we will explore some simple situations where antiderivatives might arise.

3. Find the antiderivative of f(x) = 3x2 + 5x4 + 10x− 9.

4. Find the antiderivative of f(x) =3

5x7 − 18x4 + x.

5. Find the antiderivative of f(x) = 4 3√x− 9

2x2.7.

488

Page 505: CLP-1 Differential Calculus

TOWARDS INTEGRAL CALCULUS 4.1 INTRODUCTION TO ANTIDERIVATIVES

6. Find the antiderivative of f(x) =1

7√x

.

7. Find the antiderivative of f(x) = e5x+11.

8. Find the antiderivative of f(x) = 3 sin(5x) + 7 cos(13x).

9. Find the antiderivative of f(x) = sec2(x+ 1).

10. Find the antiderivative of f(x) =1

x+ 2.

11. Find the antiderivative of f(x) =7√

3− 3x2.

12. Find the antiderivative of f(x) =1

1 + 25x2

13. Find the function f(x) with f ′(x) = 3x2 − 9x+ 4 and f(1) = 10.

14. Find the function f(x) with f ′(x) = cos(2x) and f(π) = π.

15. Find the function f(x) with f ′(x) =1

xand f(−1) = 0.

16. Find the function f(x) with f ′(x) =1√

1− x2+ 1 and f(1) = −π

2.

17. Suppose a population of bacteria at time t (measured in hours) is grow-ing at a rate of 100e2t individuals per hour. Starting at time t = 0, howlong will it take the initial colony to increase by 300 individuals?

18. Your bank account at time t (measured in years) is growing at a rate of

1500et

50

dollars per year. How much money is in your account at time t?

19. At time t during a particular day, 0 ≤ t ≤ 24, your house consumes energyat a rate of

0.5 sin( π

24t)

+ 0.25

kW. (Your consumption was smallest in the middle of the night, and peakedat noon.) How much energy did your house consume in that day?

489

Page 506: CLP-1 Differential Calculus

TOWARDS INTEGRAL CALCULUS 4.1 INTRODUCTION TO ANTIDERIVATIVES

Exercises — Stage 3 For Questions 4.1.2.21 through 4.1.2.26, you are again asked tofind the antiderivatives of certain functions. In general, finding antiderivatives canbe extremely difficult–indeed, it will form the main topic of next semester’s calculuscourse. However, you can work out the antiderivatives of the functions below usingwhat you’ve learned so far about derivatives.

20. *. Let f(x) = 2 sin−1√x and g(x) = sin−1(2x − 1). Find the derivative off(x) − g(x) and simplify your answer. What does the answer imply aboutthe relation between f(x) and g(x)?

21. Find the antiderivative of f(x) = 2 cos(2x) cos(3x)− 3 sin(2x) sin(3x).

22. Find the antiderivative of f(x) =(x2 + 1)ex − ex(2x)

(x2 + 1)2.

23. Find the antiderivative of f(x) = 3x2ex3 .

24. Find the antiderivative of f(x) = 5x sin(x2).

25. Find the antiderivative of f(x) = elog x.

26. Find the antiderivative of f(x) =7√

3− x2.

27. Imagine forming a solid by revolving the parabola y = x2 + 1 around thex-axis, as in the picture below.

y = x2 + 1

y

x

Use the method of Example 4.1.7 to find the volume of such an object if thesegment of the parabola that we rotate runs from x = −H to x = H .

490

Page 507: CLP-1 Differential Calculus

HIGH SCHOOL MATERIAL

Appendix A

This chapter is really split into two parts.

• Sections A.1 to A.13 we expect you to understand and know.

• The very last section, Section A.14, contains results that we don’t expect you tomemorise, but that we think you should be able to quickly derive from otherresults you know.

A.1q Similar Triangles

Two triangles T1, T2 are similar when

• (AAA — angle angle angle) The angles of T1 are the same as the angles of T2.

• (SSS — side side side) The ratios of the side lengths are the same. That is

A

a=B

b=C

c

491

Page 508: CLP-1 Differential Calculus

HIGH SCHOOL MATERIAL A.3 TRIGONOMETRY — DEFINITIONS

• (SAS — side angle side) Two sides have lengths in the same ratio and the anglebetween them is the same. For example

A

a=C

cand angle β is same

A.2q Pythagoras

For a right-angled triangle the length of the hypotenuse is related to the lengths ofthe other two sides by

(adjacent)2 + (opposite)2 = (hypotenuse)2

A.3q Trigonometry — Definitions

492

Page 509: CLP-1 Differential Calculus

HIGH SCHOOL MATERIAL A.5 TRIGONOMETRY — GRAPHS

sin θ =opposite

hypotenusecsc θ =

1

sin θ

cos θ =adjacent

hypotenusesec θ =

1

cos θ

tan θ =oppositeadjacent

cot θ =1

tan θ

A.4q Radians, Arcs and Sectors

For a circle of radius r and angle of θ radians:

• Arc length L(θ) = rθ.

• Area of sector A(θ) = θ2r2.

A.5q Trigonometry — Graphs

sin θ cos θ tan θ

−π −π2

π2

π 3π2

−1

1

−π −π2

π2

π 3π2

−1

1

−π −π2

π2

π 3π2

493

Page 510: CLP-1 Differential Calculus

HIGH SCHOOL MATERIAL A.7 TRIGONOMETRY — SIMPLE IDENTITIES

A.6q Trigonometry — Special Triangles

From the above pair of special triangles we have

sinπ

4=

1√2

sinπ

6=

1

2sin

π

3=

√3

2

cosπ

4=

1√2

cosπ

6=

√3

2cos

π

3=

1

2

tanπ

4= 1 tan

π

6=

1√3

tanπ

3=√

3

A.7q Trigonometry — Simple Identities

• Periodicity

sin(θ + 2π) = sin(θ) cos(θ + 2π) = cos(θ)

• Reflection

sin(−θ) = − sin(θ) cos(−θ) = cos(θ)

• Reflection around π/4

sin(π2− θ)

= cos θ cos(π2− θ)

= sin θ

• Reflection around π/2

sin (π − θ) = sin θ cos (π − θ) = − cos θ

• Rotation by π

sin (θ + π) = − sin θ cos (θ + π) = − cos θ

• Pythagoras

sin2 θ + cos2 θ = 1

494

Page 511: CLP-1 Differential Calculus

HIGH SCHOOL MATERIAL A.9 INVERSE TRIG FUNCTIONS

A.8q Trigonometry — Add and Subtract Angles

• Sine

sin(α± β) = sin(α) cos(β)± cos(α) sin(β)

• Cosine

cos(α± β) = cos(α) cos(β)∓ sin(α) sin(β)

A.9q Inverse Trigonometric Functions

Some of you may not have studied inverse trigonometric functions in highschool,however we still expect you to know them by the end of the course.

arcsinx arccosx arctanxDomain: −1 ≤ x ≤ 1 Domain: −1 ≤ x ≤ 1 Domain: all real numbersRange: −π

2≤ arcsinx ≤ π

2Range: 0 ≤ arccosx ≤ π Range: −π

2< arctanx < π

2

−1 1

−π2

π2

−1 1

π2

π

−π2

π2

Since these functions are inverses of each other we have

arcsin(sin θ) = θ −π2≤ θ ≤ π

2arccos(cos θ) = θ 0 ≤ θ ≤ π

arctan(tan θ) = θ −π2≤ θ ≤ π

2

and also

sin(arcsinx) = x −1 ≤ x ≤ 1

cos(arccosx) = x −1 ≤ x ≤ 1

tan(arctanx) = x any real x

495

Page 512: CLP-1 Differential Calculus

HIGH SCHOOL MATERIAL A.10 AREAS

arccscx arcsecx arccotxDomain: |x| ≥ 1 Domain: |x| ≥ 1 Domain: all real numbersRange: −π

2≤ arccscx ≤ π

2Range: 0 ≤ arcsecx ≤ π Range: 0 < arccotx < π

arccscx 6= 0 arcsecx 6= π

2

−1 1

−π2

π2

−1 1

π2

π

π2

π

Again

arccsc(csc θ) = θ −π2≤ θ ≤ π

2, θ 6= 0

arcsec(sec θ) = θ 0 ≤ θ ≤ π, θ 6= π

2arccot(cot θ) = θ 0 < θ < π

and

csc(arccscx) = x |x| ≥ 1

sec(arcsecx) = x |x| ≥ 1

cot(arccotx) = x any real x

A.10q Areas

• Area of a rectangle

A = bh

496

Page 513: CLP-1 Differential Calculus

HIGH SCHOOL MATERIAL A.11 VOLUMES

• Area of a triangle

A =1

2bh =

1

2ab sin θ

• Area of a circle

A = πr2

• Area of an ellipse

A = πab

A.11q Volumes

• Volume of a rectangular prism

V = lwh

• Volume of a cylinder

V = πr2h

• Volume of a cone

V =1

3πr2h

• Volume of a sphere

V =4

3πr3

497

Page 514: CLP-1 Differential Calculus

HIGH SCHOOL MATERIAL A.13 LOGARITHMS

A.12q Powers

In the following, x and y are arbitrary real numbers, and q is an arbitrary constantthat is strictly bigger than zero.

• q0 = 1

• qx+y = qxqy, qx−y = qx

qy

• q−x = 1qx

•(qx)y

= qxy

• limx→∞

qx =∞, limx→−∞

qx = 0 if q > 1

• limx→∞

qx = 0, limx→−∞

qx =∞ if 0 < q < 1

• The graph of 2x is given below. The graph of qx, for any q > 1, is similar.

A.13q Logarithms

In the following, x and y are arbitrary real numbers that are strictly bigger than 0,and p and q are arbitrary constants that are strictly bigger than one.

• qlogq x = x, logq(qx)

= x

• logq x =logp x

logp q

• logq 1 = 0, logq q = 1

• logq(xy) = logq x+ logq y

• logq(xy

)= logq x− logq y

498

Page 515: CLP-1 Differential Calculus

HIGH SCHOOL MATERIAL A.14 YOU SHOULD BE ABLE TO DERIVE

• logq(

1y

)= − logq y,

• logq(xy) = y logq x

• limx→∞

logq x =∞, limx→0

logq x = −∞

• The graph of log10 x is given below. The graph of logq x, for any q > 1, is similar.

A.14q Highschool Material You Should be Able to Derive

• Graphs of csc θ, sec θ and cot θ:

csc θ sec θ cot θ

−π −π2

π2

π 3π2

2π−1

1

−π −π2

π2

π 3π2

2π−1

1

−π −π2

π2

π 3π2

• More Pythagoras

sin2 θ + cos2 θ = 1divide by cos2 θ7−−−−−−−−→ tan2 θ + 1 = sec2 θ

sin2 θ + cos2 θ = 1divide by sin2 θ7−−−−−−−−→ 1 + cot2 θ = csc2 θ

• Sine — double angle (set β = α in sine angle addition formula)

sin(2α) = 2 sin(α) cos(α)

499

Page 516: CLP-1 Differential Calculus

HIGH SCHOOL MATERIAL A.14 YOU SHOULD BE ABLE TO DERIVE

• Cosine — double angle (set β = α in cosine angle addition formula)

cos(2α) = cos2(α)− sin2(α)

= 2 cos2(α)− 1 (use sin2(α) = 1− cos2(α))= 1− 2 sin2(α) (use cos2(α) = 1− sin2(α))

• Composition of trigonometric and inverse trigonometric functions:

cos(arcsinx) =√

1− x2 sec(arctanx) =√

1 + x2

and similar expressions.

500

Page 517: CLP-1 Differential Calculus

ORIGIN OF TRIG, AREAAND VOLUME FORMULAS

Appendix B

B.1q Theorems about Triangles

B.1.1 tt Thales’ Theorem

We want to get at right-angled triangles. A classic construction for this is to draw atriangle inside a circle, so that all three corners lie on the circle and the longest sideforms the diameter of the circle. See the figure below in which we have scaled thecircle to have radius 1 and the triangle has longest side 2.

Thales theorem states that the angle at C is always a right-angle. The proof isquite straight-forward and relies on two facts:

• the angles of a triangle add to π, and

• the angles at the base of an isosceles triangle are equal.

501

Page 518: CLP-1 Differential Calculus

ORIGIN OF TRIG, AREA AND VOLUME FORMULASB.1 THEOREMS ABOUT TRIANGLES

So we split the triangleABC by drawing a line from the centre of the circle to C. Thiscreates two isosceles triangles OAC and OBC. Since they are isosceles, the anglesat their bases α and β must be equal (as shown). Adding the angles of the originaltriangle now gives

π = α + (α + β) + β = 2(α + β)

So the angle at C = π − (α + β) = π/2.

B.1.2 tt Pythagoras

Since trigonometry, at its core, is the study of lengths and angles in right-angledtriangles, we must include a result you all know well, but likely do not know howto prove.

The lengths of the sides of any right-angled triangle are related by the famousresult due to Pythagoras

c2 = a2 + b2.

There are many ways to prove this, but we can do so quite simply by studying thefollowing diagram:

We start with a right-angled triangle with sides labeled a, b and c. Then we con-struct a square of side-length a+b and draw inside it 4 copies of the triangle arrangedas shown in the centre of the above figure. The area in white is then a2 + b2. Nowmove the triangles around to create the arrangement shown on the right of the above

502

Page 519: CLP-1 Differential Calculus

ORIGIN OF TRIG, AREA AND VOLUME FORMULAS B.2 TRIGONOMETRY

figure. The area in white is bounded by a square of side-length c and so its area is c2.The area of the outer square didn’t change when the triangles were moved, nor didthe area of the triangles, so the white area cannot have changed either. This provesa2 + b2 = c2.

B.2q Trigonometry

B.2.1 tt Angles — Radians vs Degrees

For mathematics, and especially in calculus, it is much better to measure angles inunits called radians rather than degrees. By definition, an arc of length θ on a circleof radius one subtends an angle of θ radians at the centre of the circle.

The circle on the left has radius 1, and the arc swept out by an angle of θ radianshas length θ. Because a circle of radius one has circumference 2π we have

2π radians = 360 π radians = 180π

2radians = 90

π

3radians = 60

π

4radians = 45

π

6radians = 30

More generally, consider a circle of radius r. Let L(θ) denote the length of thearc swept out by an angle of θ radians and let A(θ) denote the area of the sector (orwedge) swept out by the same angle. Since the angle sweeps out the fraction θ

2πof a

whole circle, we have

L(θ) = 2πr · θ2π

= θr and

A(θ) = πr2 · θ2π

2r2

503

Page 520: CLP-1 Differential Calculus

ORIGIN OF TRIG, AREA AND VOLUME FORMULAS B.2 TRIGONOMETRY

B.2.2 tt Trig Function Definitions

The trigonometric functions are defined as ratios of the lengths of the sides of a right-angle triangle as shown in the left of the diagram below . These ratios depend onlyon the angle θ.

The trigonometric functions sine, cosine and tangent are defined as ratios of thelengths of the sides

sin θ =opposite

hypotenusecos θ =

adjacenthypotenuse

tan θ =oppositeadjacent

=sin θ

cos θ.

These are frequently abbreviated as

sin θ =oh

cos θ =ah

tan θ =oa

which gives rise to the mnemonic

SOH CAH TOA

If we scale the triangle so that they hypotenuse has length 1 then we obtain thediagram on the right. In that case, sin θ is the height of the triangle, cos θ the lengthof its base and tan θ is the length of the line tangent to the circle of radius 1 as shown.

Since the angle 2π sweeps out a full circle, the angles θ and θ + 2π are really thesame.

504

Page 521: CLP-1 Differential Calculus

ORIGIN OF TRIG, AREA AND VOLUME FORMULAS B.2 TRIGONOMETRY

Hence all the trigonometric functions are periodic with period 2π. That is

sin(θ + 2π) = sin(θ) cos(θ + 2π) = cos(θ) tan(θ + 2π) = tan(θ)

The plots of these functions are shown below

sin θ cos θ tan θ

−π −π2

π2

π 3π2

−1

1

−π −π2

π2

π 3π2

−1

1

−π −π2

π2

π 3π2

The reciprocals (cosecant, secant and cotangent) of these functions also play im-portant roles in trigonometry and calculus:

csc θ =1

sin θ=

ho

sec θ =1

cos θ=

ha

cot θ =1

tan θ=

cos θ

sin θ=

ao

The plots of these functions are shown below

csc θ sec θ cot θ

−π −π2

π2

π 3π2

2π−1

1

−π −π2

π2

π 3π2

2π−1

1

−π −π2

π2

π 3π2

505

Page 522: CLP-1 Differential Calculus

ORIGIN OF TRIG, AREA AND VOLUME FORMULAS B.2 TRIGONOMETRY

These reciprocal functions also have geometric interpretations:

Since these are all right-angled triangles we can use Pythagoras to obtain thefollowing identities:

sin2 θ + cos2 θ = 1 tan2 θ + 1 = sec2 θ 1 + cot2 θ = csc2 θ

Of these it is only necessary to remember the first

sin2 θ + cos2 θ = 1

The second can then be obtained by dividing this by cos2 θ and the third by dividingby sin2 θ.

B.2.3 tt Important Triangles

Computing sine and cosine is non-trivial for general angles — we need Taylor series(or similar tools) to do this. However there are some special angles (usually smallinteger fractions of π) for which we can use a little geometry to help. Consider thefollowing two triangles.

The first results from cutting a square along its diagonal, while the second isobtained by cutting an equilateral triangle from one corner to the middle of the op-posite side. These, together with the angles 0, π

2and π give the following table of

values

506

Page 523: CLP-1 Differential Calculus

ORIGIN OF TRIG, AREA AND VOLUME FORMULAS B.2 TRIGONOMETRY

θ sin θ cos θ tan θ csc θ sec θ cot θ

0 rad 0 1 0 DNE 1 DNEπ2

rad 1 0 DNE 1 DNE 0π rad 0 -1 0 DNE -1 DNEπ4

rad 1√2

1√2

1√

2√

2 1π6

rad 12

√3

21√3

2 2√3

√3

π3

rad√

32

12

√3 2√

32 1√

3

B.2.4 tt Some More Simple Identities

Consider the figure below

The pair triangles on the left shows that there is a simple relationship betweentrigonometric functions evaluated at θ and at −θ:

sin(−θ) = − sin(θ) cos(−θ) = cos(θ)

That is — sine is an odd function, while cosine is even. Since the other trigonometricfunctions can be expressed in terms of sine and cosine we obtain

tan(−θ) = − tan(θ) csc(−θ) = − csc(θ) sec(−θ) = sec(θ) cot(−θ) = − cot(θ)

Now consider the triangle on the right — if we consider the angle π2− θ the side-

lengths of the triangle remain unchanged, but the roles of “opposite” and “adjacent”are swapped. Hence we have

sin(π2− θ)

= cos θ cos(π2− θ)

= sin θ

Again these imply that

tan(π2− θ)

= cot θ csc(π2− θ)

= sec θ sec(π2− θ)

= csc θ cot(π2− θ)

= tan θ

We can go further. Consider the following diagram:

507

Page 524: CLP-1 Differential Calculus

ORIGIN OF TRIG, AREA AND VOLUME FORMULAS B.2 TRIGONOMETRY

This implies that

sin(π − θ) = sin(θ) cos(π − θ) = − cos(θ)

sin(π + θ) = − sin(θ) cos(π + θ) = − cos(θ)

From which we can get the rules for the other four trigonometric functions.

B.2.5 tt Identities — Adding Angles

We wish to explain the origins of the identity

sin(α + β) = sin(α) cos(β) + cos(α) sin(β).

A very geometric demonstration uses the figure below and an observation aboutareas.

• The left-most figure shows two right-angled triangles with angles α and β andboth with hypotenuse length 1.

• The next figure simply rearranges the triangles — translating and rotating thelower triangle so that it lies adjacent to the top of the upper triangle.

• Now scale the lower triangle by a factor of q so that edges opposite the anglesα and β are flush. This means that q cos β = cosα. ie

q =cosα

cos β

508

Page 525: CLP-1 Differential Calculus

ORIGIN OF TRIG, AREA AND VOLUME FORMULAS B.2 TRIGONOMETRY

Now compute the areas of these (blue and red) triangles

Ared =1

2q2 sin β cos β

Ablue =1

2sinα cosα

So twice the total area is

2Atotal = sinα cosα + q2 sin β cos β

• But we can also compute the total area using the rightmost triangle:

2Atotal = q sin(α + β)

Since the total area must be the same no matter how we compute it we have

q sin(α + β) = sinα cosα + q2 sin β cos β

sin(α + β) =1

qsinα cosα + q sin β cos β

=cos β

cosαsinα cosα +

cosα

cos βsin β cos β

= sinα cos β + cosα sin β

as required.We can obtain the angle addition formula for cosine by substituting α 7→ π/2− α

and β 7→ −β into our sine formula:

sin(α + β) = sin(α) cos(β) + cos(α) sin(β) becomessin(π/2− α− β)︸ ︷︷ ︸

cos(α+β)

= sin(π/2− α)︸ ︷︷ ︸cos(α)

cos(−β) + cos(π/2− α)︸ ︷︷ ︸sin(α)

sin(−β)

cos(α + β) = cos(α) cos(β)− sin(α) sin(β)

where we have used sin(π/2− θ) = cos(θ) and cos(π/2− θ) = sin(θ).It is then a small step to the formulas for the difference of angles. From the rela-

tion

sin(α + β) = sin(α) cos(β) + cos(α) sin(β)

we can substitute β 7→ −β and so obtain

sin(α− β) = sin(α) cos(−β) + cos(α) sin(−β)

= sin(α) cos(β)− cos(α) sin(β)

The formula for cosine can be obtained in a similar manner. To summarise

sin(α± β) = sin(α) cos(β)± cos(α) sin(β)

cos(α± β) = cos(α) cos(β)∓ sin(α) sin(β)

509

Page 526: CLP-1 Differential Calculus

ORIGIN OF TRIG, AREA AND VOLUME FORMULAS B.2 TRIGONOMETRY

The formulas for tangent are a bit more work, but

tan(α + β) =sin(α + β)

cos(α + β)

=sin(α) cos(β) + cos(α) sin(β)

cos(α) cos(β)− sin(α) sin(β)

=sin(α) cos(β) + cos(α) sin(β)

cos(α) cos(β)− sin(α) sin(β)· sec(α) sec(β)

sec(α) sec(β)

=sin(α) sec(α) + sin(β) sec(β)

1− sin(α) sec(α) sin(β) sec(β)

=tan(α) + tan(β)

1− tan(α) tan(β)

and similarly we get

tan(α− β) =tan(α)− tan(β)

1 + tan(α) tan(β)

B.2.6 tt Identities — Double-angle Formulas

If we set β = α in the angle-addition formulas we get

sin(2α) = 2 sin(α) cos(α)

cos(2α) = cos2(α)− sin2(α)

= 2 cos2(α)− 1 since sin2 θ = 1− cos2 θ

= 1− 2 sin2(α) since cos2 θ = 1− sin2 θ

tan(2α) =2 tan(α)

1− tan2(α)

=2

cot(α)− tan(α)divide top and bottom by tan(α)

B.2.7 tt Identities — Extras

B.2.7.1ttt Sums to Products

Consider the identities

sin(α + β) = sin(α) cos(β) + cos(α) sin(β) sin(α− β) = sin(α) cos(β)− cos(α) sin(β)

510

Page 527: CLP-1 Differential Calculus

ORIGIN OF TRIG, AREA AND VOLUME FORMULAS B.2 TRIGONOMETRY

If we add them together some terms on the right-hand side cancel:

sin(α + β) + sin(α− β) = 2 sin(α) cos(β).

If we now set u = α + β and v = α− β (i.e. α = u+v2, β = u−v

2) then

sin(u) + sin(v) = 2 sin

(u+ v

2

)cos

(u− v

2

)This transforms a sum into a product. Similarly:

sin(u)− sin(v) = 2 sin

(u− v

2

)cos

(u+ v

2

)cos(u) + cos(v) = 2 cos

(u+ v

2

)cos

(u− v

2

)cos(u)− cos(v) = −2 sin

(u+ v

2

)sin

(u− v

2

)

B.2.7.2ttt Products to sums

Again consider the identities

sin(α + β) = sin(α) cos(β) + cos(α) sin(β) sin(α− β) = sin(α) cos(β)− cos(α) sin(β)

and add them together:

sin(α + β) + sin(α− β) = 2 sin(α) cos(β).

Then rearrange:

sin(α) cos(β) =sin(α + β) + sin(α− β)

2

In a similar way, start with the identities

cos(α + β) = cos(α) cos(β)− sin(α) sin(β) cos(α− β) = cos(α) cos(β) + sin(α) sin(β)

If we add these together we get

2 cos(α) cos(β) = cos(α + β) + cos(α− β)

while taking their difference gives

2 sin(α) sin(β) = cos(α− β)− cos(α + β)

Hence

sin(α) sin(β) =cos(α− β)− cos(α + β)

2

cos(α) cos(β) =cos(α− β) + cos(α + β)

2

511

Page 528: CLP-1 Differential Calculus

ORIGIN OF TRIG, AREA AND VOLUME FORMULASB.3 INVERSE TRIGONOMETRIC FUNCTIONS

B.3q Inverse Trigonometric Functions

In order to construct inverse trigonometric functions we first have to restrict theirdomains so as to make them one-to-one (or injective). We do this as shown below

sin θ cos θ tan θDomain: −π

2≤ θ ≤ π

2Domain: 0 ≤ θ ≤ π Domain: −π

2< θ < π

2

Range: −1 ≤ sin θ ≤ 1 Range: −1 ≤ cos θ ≤ 1 Range: all real numbers

−π2

π2

−1

1

π2

π

−1

1

−π −π2

arcsinx arccosx arctanxDomain: −1 ≤ x ≤ 1 Domain: −1 ≤ x ≤ 1 Domain: all real numbersRange: −π

2≤ arcsinx ≤ π

2Range: 0 ≤ arccosx ≤ π Range: −π

2< arctanx < π

2

−1 1

−π2

π2

−1 1

π2

π

−π2

π2

Since these functions are inverses of each other we have

arcsin(sin θ) = θ −π2≤ θ ≤ π

2arccos(cos θ) = θ 0 ≤ θ ≤ π

arctan(tan θ) = θ −π2≤ θ ≤ π

2

and also

sin(arcsinx) = x −1 ≤ x ≤ 1

cos(arccosx) = x −1 ≤ x ≤ 1

tan(arctanx) = x any real x

We can read other combinations of trig functions and their inverses, like, for exam-ple, cos(arcsinx), off of triangles like

512

Page 529: CLP-1 Differential Calculus

ORIGIN OF TRIG, AREA AND VOLUME FORMULASB.3 INVERSE TRIGONOMETRIC FUNCTIONS

We have chosen the hypotenuse and opposite sides of the triangle to be of length1 and x, respectively, so that sin(θ) = x. That is, θ = arcsinx. We can then read off ofthe triangle that

cos(arcsinx) = cos(θ) =√

1− x2

We can reach the same conclusion using trig identities, as follows.

• Write arcsinx = θ. We know that sin(θ) = x and we wish to compute cos(θ). Sowe just need to express cos(θ) in terms of sin(θ).

• To do this we make use of one of the Pythagorean identities

sin2 θ + cos2 θ = 1

cos θ = ±√

1− sin2 θ

• Thus

cos(arcsinx) = cos θ = ±√

1− sin2 θ

• To determine which branch we should use we need to consider the domainand range of arcsinx:

Domain: − 1 ≤ x ≤ 1 Range: − π

2≤ arcsinx ≤ π

2

Thus we are applying cosine to an angle that always lies between −π2

and π2.

Cosine is non-negative on this range. Hence we should take the positive branchand

cos(arcsinx) =√

1− sin2 θ =√

1− sin2(arcsinx)

=√

1− x2

In a very similar way we can simplify tan(arccosx).

• Write arccosx = θ, and then

tan(arccosx) = tan θ =sin θ

cos θ

• Now the denominator is easy since cos θ = cos arccosx = x.

513

Page 530: CLP-1 Differential Calculus

ORIGIN OF TRIG, AREA AND VOLUME FORMULAS B.4 COSINE AND SINE LAWS

• The numerator is almost the same as the previous computation.

sin θ = ±√

1− cos2 θ

= ±√

1− x2

• To determine which branch we again consider domains and and ranges:

Domain: − 1 ≤ x ≤ 1 Range: 0 ≤ arccosx ≤ π

Thus we are applying sine to an angle that always lies between 0 and π. Sine isnon-negative on this range and so we take the positive branch.

• Putting everything back together gives

tan(arccosx) =

√1− x2

x

Completing the 9 possibilities gives:

sin(arcsinx) = x sin(arccosx) =√

1− x2 sin(arctanx) =x√

1 + x2

cos(arcsinx) =√

1− x2 cos(arccosx) = x cos(arctanx) =1√

1 + x2

tan(arcsinx) =x√

1− x2tan(arccosx) =

√1− x2

xtan(arctanx) = x

B.4q Cosine and Sine Laws

B.4.1 tt Cosine Law or Law of Cosines

The cosine law says that, if a triangle has sides of length a, b and c and the angleopposite the side of length c is γ, then

c2 = a2 + b2 − 2ab cos γ

Observe that, when γ = π2, this reduces to, (surpise!) Pythagoras’ theorem c2 =

a2 + b2. Let’s derive the cosine law.

514

Page 531: CLP-1 Differential Calculus

ORIGIN OF TRIG, AREA AND VOLUME FORMULAS B.4 COSINE AND SINE LAWS

Consider the triangle on the left. Now draw a perpendicular line from the side oflength c to the opposite corner as shown. This demonstrates that

c = a cos β + b cosα

Multiply this by c to get an expression for c2:

c2 = ac cos β + bc cosα

Doing similarly for the other corners gives

a2 = ac cos β + ab cos γ

b2 = bc cosα + ab cos γ

Now combining these:

a2 + b2 − c2 = (bc− bc) cosα + (ac− ac) cos β + 2ab cos γ

= 2ab cos γ

as required.

B.4.2 tt Sine Law or Law of Sines

The sine law says that, if a triangle has sides of length a, b and c and the anglesopposite those sides are α, β and γ, then

a

sinα=

b

sin β=

c

sin γ.

This rule is best understood by computing the area of the triangle using the for-mula A = 1

2ab sin θ of Appendix A.10. Doing this three ways gives

2A = bc sinα

2A = ac sin β

2A = ab sin γ

Dividing these expressions by abc gives

2A

abc=

sinα

a=

sin β

b=

sin γ

c

as required.

515

Page 532: CLP-1 Differential Calculus

ORIGIN OF TRIG, AREA AND VOLUME FORMULASB.5 CIRCLES, CONES AND SPHERES

B.5q Circles, cones and spheres

B.5.1 ttWhere Does the Formula for the Area of a Circle Come From?

Typically when we come across π for the first time it is as the ratio of the circumfer-ence of a circle to its diameter

π =C

d=C

2r

Indeed this is typically the first definition we see of π. It is easy to build an intuitionthat the area of the circle should be propotional to the square of its radius. For ex-ample we can draw the largest possible square inside the circle (an inscribed square)and the smallest possible square outside the circle (a circumscribed square):

The smaller square has side-length√

2r and the longer has side-length 2r. Hence

2r2 ≤ A ≤ 4r2 or 2 ≤ A

r2≤ 4

That is, the area of the circle is between 2 and 4 times the square of the radius. Whatis perhaps less obvious (if we had not been told this in school) is that the constant ofpropotionality for area is also π:

π =A

r2.

We will show this using Archimedes’ proof. He makes use of these inscribed andcircumscribed polygons to make better and better approximations of the circle. Thesteps of the proof are somewhat involved and the starting point is to rewrite the areaof a circle as

A =1

2Cr

where C is (still) the circumference of the circle. This suggests that this area is thesame as that of a triangle of height r and base length C

T =1

2Cr

516

Page 533: CLP-1 Differential Calculus

ORIGIN OF TRIG, AREA AND VOLUME FORMULASB.5 CIRCLES, CONES AND SPHERES

Archimedes’ proof then demonstrates that indeed this triangle and the circle havethe same area. It relies on a “proof by contradiction” — showing that T < A andT > A cannot be true and so the only possibility is that A = T .

We will first show that T < A cannot happen. Construct an n-sided “inscribed”polygon as shown below:

Let pn be the inscribed polygon as shown.

We need 4 steps.

• The area of pn is smaller than that of the circle — this follows since we canconstruct pn by cutting slices from the circle.

• Let En be the difference between the area of the circle and pn: En = A − A(pn)(see the left of the previous figure). By the previous point we know En > 0.Now as we increase the number of sides, this difference becomes smaller. Tobe more precise

E2n ≤1

2En.

The error En is made up of n “lobes”. In the centre-left of the previous figurewe draw one such lobe and surround it by a rectangle of dimensions a× 2b —

517

Page 534: CLP-1 Differential Calculus

ORIGIN OF TRIG, AREA AND VOLUME FORMULASB.5 CIRCLES, CONES AND SPHERES

we could determine these more precisely using a little trigonometry, but it isnot necessary.

This diagram shows the lobe is smaller than the rectangle of base 2b and heighta Since there are n copies of the lobe, we have

En ≤ n× 2ab rewrite asEn2≤ nab

Now draw in the polygon p2n and consider the associated “error” E2n. If wefocus on the two lobes shown then we see that the area of these two new lobesis equal to that of the old lobe (shown in centre-left) minus the area of thetriangle with base 2b and height a (drawn in purple). Since there are n copiesof this picture we have

E2n = En − nab now use that nab ≥ En/2

≤ En −En2

=En2

• The area of pn is smaller than T . To see this decompose pn into n isoscelestriangles. Each of these has base shorter than C/n; the straight line is shorterthan the corresponding arc — though strictly speaking we should prove this.The height of each triangle is shorter than r. Thus

A(pn) = n× 1

2(base)× (height)

≤ n× Cr

2n= T

• If we assume that T < A, then A − T = d where d is some positive number.However we know from point 2 that we can make n large enough so that En <d (each time we double n we halve the error). But now we have a contradictionto step 3, since we have just shown that

En = A− A(pn) < A− T which implies thatA(pn) > T.

Thus we cannot have T < A.If we now assume that T > A we will get a similar contradiction by a similar

construction. Now we use regular n-sided circumscribed polygons, Pn.

518

Page 535: CLP-1 Differential Calculus

ORIGIN OF TRIG, AREA AND VOLUME FORMULASB.5 CIRCLES, CONES AND SPHERES

The proof can be broken into 4 similar steps.

1. The area of Pn is greater than that of the circle — this follows since we canconstruct the circle by trimming the polygon Pn.

2. Let En be the difference between the area of the polygon and the circle: En =A(Pn)− A (see the left of the previous figure). By the previous point we knowEn > 0. Now as we increase the number of sides, this difference becomessmaller. To be more precise we will show

E2n ≤1

2En.

The errorEn is made up of n “lobes”. In the centre-left of the previous figure wedraw one such lobe. Let Ln denote the area of one of these lobes, so En = nLn.In the centre of the previous figure we have labelled this lobe carefully andalso shown how it changes when we create the polygon P2n. In particular, theoriginal lobe is bounded by the straight lines ~ad, ~af and the arc f bd. We createP2n from Pn by cutting away the corner triangle4aec. Accordingly the lines ~ecand ~ba are orthogonal and the segments |bc| = |cd|.By the construction of P2n from Pn, we have

2L2n = Ln − A(4aec) or equivalently L2n =1

2Ln − A(4abc)

And additionally

L2n ≤ A(4bcd)

Now consider the triangle 4abd (centre-right of the previous figure) and thetwo triangles within it 4abc and 4bcd. We know that ~ab and ~cb form a right-angle. Consequently ~ac is the hypotenuse of a right-angled triangle, so |ac| >|bc| = |cd|. So now, the triangles4abc and4bcd have the same heights, but thebase of ~ac is longer than ~cd. Hence the area of 4abc is strictly larger than thatof4bcd.

Thus we have

L2n ≤ A(4bcd) < A(4abc)

519

Page 536: CLP-1 Differential Calculus

ORIGIN OF TRIG, AREA AND VOLUME FORMULASB.5 CIRCLES, CONES AND SPHERES

But now we can write

L2n =1

2Ln − A(4abc) < 1

2Ln − L2n rearrange

2L2n <1

2Ln there are n such lobes, so

2nL2n <n

2Ln since En = nLn, we have

E2n <1

2En which is what we wanted to show.

3. The area of Pn is greater than T . To see this decompose Pn into n isoscelestriangles. The height of each triangle is r, while the base of each is longer thanC/n (this is a subtle point and its proof is equivalent to showing that tan θ > θ).Thus

A(Pn) = n× 1

2(base)× (height)

≥ n× Cr

2n= T

4. If we assume that T > A, then T − A = d where d is some positive number.However we know from point 2 that we can make n large enough so that En <d (each time we double n we halve the error). But now we have a contradictionsince we have just shown that

En = A(Pn)− A < T − A which implies thatA(pn) > T.

Thus we cannot have T > A. The only possibility that remains is that T = A.

B.5.2 ttWhere Do These Volume Formulas Come From?

We can establish the volumes of cones and spheres from the formula for the volumeof a cylinder and a little work with limits and some careful summations. We firstneed a few facts.

• Every square number can be written as a sum of consecutive odd numbers.More precisely

n2 = 1 + 3 + · · ·+ (2n− 1)

• The sum of the first n positive integers is 12n(n+ 1). That is

1 + 2 + 3 + · · ·+ n =1

2n(n+ 1)

520

Page 537: CLP-1 Differential Calculus

ORIGIN OF TRIG, AREA AND VOLUME FORMULASB.5 CIRCLES, CONES AND SPHERES

• The sum of the squares of the first n positive integers is 16n(n+ 1)(2n+ 1).

12 + 22 + 32 + · · ·+ n2 =1

6n(n+ 1)(2n+ 1)

We will not give completely rigorous proofs of the above identities (since we arenot going to assume that the reader knows mathematical induction), rather we willexplain them using pictorial arguments. The first two of these we can explain bysome quite simple pictures:

We see that we can decompose any square of unit-squares into a sequence ofstrips, each of which consists of an odd number of unit-squares. This is really justfrom the fact that

n2 − (n− 1)2 = 2n− 1

Similarly, we can represent the sum of the first n integers as a triangle of unit squaresas shown. If we make a second copy of that triangle and arrange it as shown, it givesa rectangle of dimensions n by n + 1. Hence the rectangle, being exactly twice thesize of the original triangle, contains n(n+ 1) unit squares.

The explanation of the last formula takes a little more work and a carefully con-structed picture:

Let us break these pictures down step by step

521

Page 538: CLP-1 Differential Calculus

ORIGIN OF TRIG, AREA AND VOLUME FORMULASB.5 CIRCLES, CONES AND SPHERES

• Leftmost represents the sum of the squares of the first n integers.

• Centre — We recall from above that each square number can be written as asum of consecutive odd numbers, which have been represented as colouredbands of unit-squares.

• Make three copies of the sum and arrange them carefully as shown. The firstand third copies are obvious, but the central copy is rearranged considerably;all bands of the same colour have the same length and have been arranged intorectangles as shown.

Putting everything from the three copies together creates a rectangle of dimen-sions (2n+ 1)× (1 + 2 + 3 + · · ·+ n).

We know (from above) that 1 + 2 + 3 + · · ·+ n = 12n(n+ 1) and so

(12 + 22 + · · ·+ n2) =1

3× 1

2n(n+ 1)(2n+ 1)

as required.Now we can start to look at volumes. Let us start with the volume of a cone;

consider the figure below. We bound the volume of the cone above and below bystacks of cylinders. The cross-sections of the cylinders and cone are also shown.

To obtain the bounds we will construct two stacks of n cylinders, C1, C2, . . . , Cn.Each cylinder has height h/n and radius that varies with height. In particular, wedefine cylinderCk to have height h/n and radius k×r/n. This radius was determinedusing similar triangles so that cylinder Cn has radius r. Now cylinder Ck has volume

Vk = π × radius2 × height = π

(kr

n

)2

· hn

522

Page 539: CLP-1 Differential Calculus

ORIGIN OF TRIG, AREA AND VOLUME FORMULASB.5 CIRCLES, CONES AND SPHERES

=πr2h

n3k2

We obtain an upper bound by stacking cylinders C1, C2, . . . , Cn as shown. Thisobject has volume

V = V1 + V2 + · · ·+ Vn

=πr2h

n3

(12 + 22 + 32 + · · ·+ n2

)=πr2h

n3· n(n+ 1)(2n+ 1)

6

A similar lower bound is obtained by stacking cylinders C1, . . . , Cn−1 which gives avolume of

V = V1 + V2 + · · ·+ Vn−1

=πr2h

n3

(12 + 22 + 32 + · · ·+ (n− 1)2

)=πr2h

n3· (n− 1)(n)(2n− 1)

6

Thus the true volume of the cylinder is bounded between

πr2h

n3· (n− 1)(n)(2n− 1)

6≤ correct volume ≤ πr2h

n3· n(n+ 1)(2n+ 1)

6

We can now take the limit as the number of cylinders, n, goes to infinity. The upperbound becomes

limn→∞

πr2h

n3

n(n+ 1)(2n+ 1)

6=πr2h

6limn→∞

n(n+ 1)(2n+ 1)

n3

=πr2h

6limn→∞

(1 + 1/n)(2 + 1/n)

1

=πr2h

6× 2

=πr2h

3

The other limit is identical, so by the squeeze theorem we have

Volume of cone =1

3πr2h

Now the sphere — though we will do the analysis for a hemisphere of radiusR. Again we bound the volume above and below by stacks of cylinders. The cross-sections of the cylinders and cone are also shown.

523

Page 540: CLP-1 Differential Calculus

ORIGIN OF TRIG, AREA AND VOLUME FORMULASB.5 CIRCLES, CONES AND SPHERES

To obtain the bounds we will construct two stacks of n cylinders, C1, C2, . . . , Cn.Each cylinder has height R/n and radius that varies with its position in the stack. Todescribe the position, define

yk = k × R

n

That is, yk, is k steps of distance Rn

from the top of the hemisphere. Then we set thekth cylinder, Ck to have height R/n and radius rk given by

r2k = R2 − (R− yk)2 = R2 −R2(1− k/n)2

= R2(2k/n− k2/n2)

as shown in the top-right and bottom-left illustrations. The volume of Ck is then

Vk = π × radius2 × height = π ×R2(2k/n− k2/n2

)× R

n

= πR3 ·(

2k

n2− k2

n3

)We obtain an upper bound by stacking cylinders C1, C2, . . . , Cn as shown. This

object has volume

V = V1 + V2 + · · ·+ Vn

= πR3 ·(

2

n2(1 + 2 + 3 + · · ·+ n)− 1

n3

(12 + 22 + 32 + · · ·+ n2

))Now recall from above that

1 + 2 + 3 + · · ·+ n =1

2n(n+ 1) 12 + 22 + 32 + · · ·+ n2 =

1

6n(n+ 1)(2n+ 1)

so

V = πR3 ·(n(n+ 1)

n2− n(n+ 1)(2n+ 1)

6n3

)524

Page 541: CLP-1 Differential Calculus

ORIGIN OF TRIG, AREA AND VOLUME FORMULASB.5 CIRCLES, CONES AND SPHERES

Again, a lower bound is obtained by stacking cylindersC1, . . . , Cn−1 and a similaranalysis gives

V = πR3 ·(n(n− 1)

(n− 1)2− n(n− 1)(2n− 1)

6(n− 1)3

)Thus the true volume of the hemisphere is bounded between

πR3 ·(n(n+ 1)

n2− n(n+ 1)(2n+ 1)

6n3

)≤ correct volume ≤ πR3 ·

(n(n+ 1)

n2− n(n+ 1)(2n+ 1)

6n3

)We can now take the limit as the number of cylinders, n, goes to infinity. The upperbound becomes

limn→∞

πR3 ·(n(n+ 1)

n2− n(n+ 1)(2n+ 1)

6n3

)= πR3

(limn→∞

n(n+ 1)

n2− n(n+ 1)(2n+ 1)

6n3

)= πR3

(1− 2

6

)=

2

3πR3.

The other limit is identical, so by the squeeze theorem we have

Volume of hemisphere =2

3πR3 and so

Volume of sphere =4

3πR3

525

Page 542: CLP-1 Differential Calculus

ROOT FINDING

Appendix C

To this point you have found solutions to equations almost exclusively by algebraicmanipulation. This is possible only for the artificially simple equations of problemsets and tests. In the “real world” it is very common to encounter equations thatcannot be solved by algebraic manipulation. For example, you found, by completinga square, that the solutions to the quadratic equation ax2 + bx + c = 0 are x =(−b±

√b2 − 4ac

)/2a. But it is known that there simply does not exist a corresponding

formula for the roots of a general polynomial of degree five or more. Fortunately,encountering such an equation is not the end of the world, because usually one doesnot need to know the solutions exactly. One only needs to know them to within somespecified degree of accuracy. For example, one rarely needs to know π to more than afew decimal places. There is a whole subject, called numerical analysis, that concernsusing algorithms to solve equations (and perform other tasks) approximately, to anydesired degree of accuracy.

We have already had, in Examples 1.6.14 and 1.6.15, and the lead up to them, a re-ally quick introduction to the bisection method, which is a crude, but effective, algo-rithm for finding approximate solutions to equations of the form f(x) = 0. We shallshortly use a little calculus to derive a very efficient algorithm for finding approxi-mate solutions to such equations. But first here is a simple example which providesa review of some of the basic ideas of root finding and the bisection method.

Example C.0.1 Bisection method.

Suppose that we are given some function f(x) and we have to find solutions to theequation f(x) = 0. To be concrete, suppose that f(x) = 8x3 + 12x2 + 6x − 15. Howdo we go about solving f(x) = 0? To get a rough idea of the lay of the land, sketchthe graph of f(x). First observe that

• when x is very large and negative, f(x) is very large and negative

• when x is very large and positive, f(x) is very large and positive

526

Page 543: CLP-1 Differential Calculus

ROOT FINDING

• when x = 0, f(x) = f(0) = −15 < 0

• when x = 1, f(x) = f(1) = 11 > 0

• f ′(x) = 24x2 + 24x + 6 = 24(x2 + x + 1

4

)= 24

(x + 1

2

)2 ≥ 0 for all x. So f(x)increases monotonically with x. The graph has a tangent of slope 0 at x = −1

2

and tangents of strictly positive slope everywhere else.

This tells us that the graph of f(x) looks like

x

y

1

y = f(x)

Since f(x) strictly increasesa as x increases, f(x) can take the value zero for at mostone value of x.

• Since f(0) < 0 and f(1) > 0 and f is continuous, f(x) must pass through 0as x travels from x = 0 to x = 1, by Theorem 1.6.12 (the intermediate valuetheorem). So f(x) takes the value zero for some x between 0 and 1. We willoften write this as “the root is x = 0.5± 0.5” to indicate the uncertainty.

• To get closer to the root, we evaluate f(x) halfway between 0 and 1.

f(

12

)= 8(

12

)3+ 12

(12

)2+ 6(

12

)− 15 = −8

Since f(

12

)< 0 and f(1) > 0 and f is continuous, f(x) must take the value zero

for some x between 12

and 1. The root is 0.75± 0.25.

• To get still closer to the root, we evaluate f(x) halfway between 12

and 1.

f(

34

)= 8(

34

)3+ 12

(34

)2+ 6(

34

)− 15 = −3

8

Since f(

34

)< 0 and f(1) > 0 and f is continuous, f(x) must take the value zero

for some x between 34

and 1. The root is 0.875± 0.125.

• And so on.

527

Page 544: CLP-1 Differential Calculus

ROOT FINDING C.1 NEWTON’S METHOD

Example C.0.1

a By “f(x) is strictly increasing” we mean that f(a) < f(b) whenever a < b. As f ′(x) > 0 for allx 6= − 1

2 , f(x) is strictly increasing even as x passes through − 12 . For example, for any x > − 1

2 ,the mean value theorem (Theorem 2.13.5) tells us that there is a c strictly between − 1

2 and x suchthat f(x)− f

(− 1

2

)= f ′(c)

(x+ 1

2

)> 0.

The root finding strategy used in Example C.0.1 is called the bisection method.The bisection method will home in on a root of the function f(x) whenever

• f(x) is continuous (f(x) need not have a derivative) and

• you can find two numbers a1 < b1 with f(a1) and f(b1) being of opposite sign.

Denote by I1 the interval [a1, b1] =x∣∣ a1 ≤ x ≤ b1

. Once you have found the

interval I1, the bisection method generates a sequence I1, I2, I3, · · · of intervals bythe following rule.

Equation C.0.2 (bisection method).

Denote by cn = an+bn2

the midpoint of the interval In = [an, bn]. If f(cn) has thesame sign as f(an), then

In+1 = [an+1, bn+1] with an+1 = cn, bn+1 = bn

and if f(cn) and f(an) have opposite signs, then

In+1 = [an+1, bn+1] with an+1 = an, bn+1 = cn

This rule was chosen so that f(an) and f(bn) have opposite sign for every n. Sincef(x) is continuous, f(x) has a zero in each interval In. Thus each step reduces theerror bars by a factor of 2. That isn’t too bad, but we can come up with somethingthat is much more efficient. We just need a little calculus.

C.1q Newton’s Method

Newton’s method1, also known as the Newton-Raphson method, is another tech-nique for generating numerical approximate solutions to equations of the form f(x) =

1 The algorithm that we are about to describe grew out of a method that Newton wrote about in1669. But the modern method incorporates substantial changes introduced by Raphson in 1690and Simpson in 1740.

528

Page 545: CLP-1 Differential Calculus

ROOT FINDING C.1 NEWTON’S METHOD

0. For example, one can easily get a good approximation to√

2 by applying New-ton’s method to the equation x2 − 2 = 0. This will be done in Example C.1.2, below.

Here is the derivation of Newton’s method. We start by simply making a guessfor the solution. For example, we could base the guess on a sketch of the graph off(x). Call the initial guess x1. Next recall, from Theorem 2.3.4, that the tangent lineto y = f(x) at x = x1 is y = F (x), where

F (x) = f(x1) + f ′(x1) (x− x1)

Usually F (x) is a pretty good approximation to f(x) for x near x1. So, instead oftrying to solve f(x) = 0, we solve the linear equation F (x) = 0 and call the solutionx2.

0 = F (x) = f(x1) + f ′(x1) (x− x1) ⇐⇒ x− x1 = − f(x1)

f ′(x1)

⇐⇒ x = x2 = x1 −f(x1)

f ′(x1)

Note that if f(x) were a linear function, then F (x) would be exactly f(x) and x2

would solve f(x) = 0 exactly.

x

y

x1 x2

y = F (x)

y = f(x)

(x1, f(x1))

Now we repeat, but starting with the (second) guess x2 rather than x1. This givesthe (third) guess x3 = x2 − f(x2)

f ′(x2). And so on. By way of summary, Newton’s method

is

1 Make a preliminary guess x1.

2 Define x2 = x1 − f(x1)f ′(x1)

.

3 Iterate. That is, for each natural number n, once you have computed xn, define

Equation C.1.1 (Newton’s method).

xn+1 = xn −f(xn)

f ′(xn)

529

Page 546: CLP-1 Differential Calculus

ROOT FINDING C.1 NEWTON’S METHOD

Example C.1.2 (Approximating√

2).

In this example we compute, approximately, the square root of two. We will of coursepretend that we do not already know that

√2 = 1.41421 · · ·. So we cannot find it by

solving, approximately, the equation f(x) = x−√

2 = 0. Instead we apply Newton’smethod to the equation

f(x) = x2 − 2 = 0

Since f ′(x) = 2x, Newton’s method says that we should generate approximate solu-tions by iteratively applying

xn+1 = xn −f(xn)

f ′(xn)= xn −

x2n − 2

2xn=xn2

+1

xn

We need a starting point. Since 12 = 1 < 2 and 22 = 4 > 2, the square root oftwo must be between 1 and 2, so let’s start Newton’s method with the initial guessx1 = 1.5. Here goesa:

x1 = 1.5

x2 =1

2x1 +

1

x1

=1

2(1.5) +

1

1.5

= 1.416666667

x3 =1

2x2 +

1

x2

=1

2(1.416666667) +

1

1.416666667

= 1.414215686

x4 =1

2x3 +

1

x3

=1

2(1.414215686) +

1

1.414215686

= 1.414213562

x5 =1

2x4 +

1

x4

=1

2(1.414213562) +

1

1.414213562

= 1.414213562

It looks like the xn’s, rounded to nine decimal places, have stabilized to 1.414213562.So it is reasonable to guess that

√2, rounded to nine decimal places, is ex-

actly 1.414213562. Recalling that all numbers 1.4142135615 ≤ y < 1.4142135625round to 1.414213562, we can check our guess by evaluating f(1.4142135615) andf(1.4142135625). Since f(1.4142135615) = −2.5 × 10−9 < 0 and f(1.4142135625) =3.6 × 10−10 > 0 the square root of two must indeed be between 1.4142135615 and1.4142135625.

a The following computations have been carried out in double precision, which is computer speakfor about 15 significant digits. We are displaying each xn rounded to 10 significant digits (9decimal places). So each displayed xn has not been impacted by roundoff error, and still containsmore decimal places than are usually needed.

530

Page 547: CLP-1 Differential Calculus

ROOT FINDING C.1 NEWTON’S METHOD

Example C.1.3 (Approximating π).

In this example we compute, approximately, π by applying Newton’s method to theequation

f(x) = sin x = 0

starting with x1 = 3. Since f ′(x) = cos x, Newton’s method says that we shouldgenerate approximate solutions by iteratively applying

xn+1 = xn −f(xn)

f ′(xn)= xn −

sinxncosxn

= xn − tanxn

Here goes

x1 = 3

x2 = x1 − tanx1 = 3− tan 3

= 3.142546543

x3 = 3.142546543− tan 3.142546543

= 3.141592653

x4 = 3.141592653− tan 3.141592653

= 3.141592654

x5 = 3.141592654− tan 3.141592654

= 3.141592654

Since f(3.1415926535) = 9.0 × 10−11 > 0 and f(3.1415926545) = −9.1 × 10−11 < 0, πmust be between 3.1415926535 and 3.1415926545. Of course to compute π in this way,we (or at least our computers) have to be able to evaluate tanx for various values ofx. Taylor expansions can help us do that. See Example 3.4.22.

Example C.1.4 wild instability.

This example illustrates how Newton’s method can go badly wrong if your initialguess is not good enough. We’ll try to solve the equation

f(x) = arctan x = 0

starting with x1 = 1.5. (Of course the solution to f(x) = 0 is just x = 0; we chosex1 = 1.5 for demonstration purposes.) Since the derivative f ′(x) = 1

1+x2, Newton’s

method gives

xn+1 = xn −f(xn)

f ′(xn)= xn − (1 + x2

n) arctanxn

Soa

x1 = 1.5

531

Page 548: CLP-1 Differential Calculus

ROOT FINDING C.1 NEWTON’S METHOD

x2 = 1.5− (1 + 1.52) arctan 1.5 = −1.69

x3 = −1.69− (1 + 1.692) arctan(−1.69) = 2.32

x4 = 2.32− (1 + 2.322) arctan(2.32) = −5.11

x5 = −5.11− (1 + 5.112) arctan(−5.11) = 32.3

x6 = 32.3− (1 + 32.32) arctan(32.3) = −1575

x7 = 3, 894, 976

Looks pretty bad! Our xn’s are not settling down at all!The figure below shows what went wrong. In this figure, y = F1(x) is the tangentline to y = arctanx at x = x1. Under Newton’s method, this tangent line crosses thex-axis at x = x2. Then y = F2(x) is the tangent to y = arctan x at x = x2. UnderNewton’s method, this tangent line crosses the x-axis at x = x3. And so on.The problem arose because the xn’s were far enough from the solution, x = 0, thatthe tangent line approximations, while good approximations to f(x) for x ≈ xn, werevery poor approximations to f(x) for x ≈ 0. In particular, y = F1(x) (i.e. the tangentline at x = x1) was a bad enough approximation to y = arctanx for x ≈ 0 that x = x2

(i.e. the value of x where y = F1(x) crosses the x-axis) is farther from the solutionx = 0 than our original guess x = x1.

x

y

y = f(x) = tan−1 x

x1

y = F1(x)

(x1,f(x1))

x2

y = F2(x)

(x2,f(x2))

x3

y = F3(x)

x4

y = F4(x)

(x4,f(x4))

If we had started with x1 = 0.5 instead of x1 = 1.5, Newton’s method would havesucceeded very nicely:

x1 = 0.5 x2 = −0.0796 x3 = 0.000335 x4 = −2.51× 10−11

a Once again, the following computations have been carried out in double precision. This time, it

532

Page 549: CLP-1 Differential Calculus

ROOT FINDING C.1 NEWTON’S METHOD

Example C.1.4

is clear that the xn’s are growing madly as n increases. So there is not much point to displayingmany decimal places and we have not done so.

Example C.1.5 interest rate.

A car dealer sells a new car for $23,520. He also offers to finance the same car forpayments of $420 per month for five years. What interest rate is this dealer charging?Solution. By way of preparation, we’ll start with a simpler problem. Suppose thatyou will have to make a single $420 payment n months in the future. The simplerproblem is to determine how much money you have to deposit now in an accountthat pays an interest rate of 100r% per month, compounded monthlya, in order to beable to make the $420 payment in n months.Let’s denote by P the initial deposit. Because the interest rate is 100r% per month,compounded monthly,

• the first month’s interest is P×r. So at the end of month #1, the account balanceis P + P r = P (1 + r).

• The second month’s interest is [P (1 + r)] × r. So at the end of month #2, theaccount balance is P (1 + r) + P (1 + r) r = P (1 + r)2.

• And so on.

• So at the end of n months, the account balance is P (1 + r)n.

In order for the balance at the end of n months, P (1 + r)n, to be $420, the initialdeposit has to be P = 420(1 + r)−n. That is what is meant by the statement “Thepresent valueb of a $420 payment made n months in the future, when the interestrate is 100r% per month, compounded monthly, is 420(1 + r)−n.”Now back to the original problem. We will be making 60 monthly payments of $420.The present value of all 60 payments isc

420(1 + r)−1 + 420(1 + r)−2 + · · ·+ 420(1 + r)−60

= 420(1 + r)−1 − (1 + r)−61

1− (1 + r)−1

= 4201− (1 + r)−60

(1 + r)− 1

= 4201− (1 + r)−60

r

The interest rate 100r% being charged by the car dealer is such that the present valueof 60 monthly payments of $420 is $23520. That is, the monthly interest rate beingcharged by the car dealer is the solution of

23520 = 4201− (1 + r)−60

ror 56 =

1− (1 + r)−60

r

533

Page 550: CLP-1 Differential Calculus

ROOT FINDING C.1 NEWTON’S METHOD

Example C.1.5

or 56r = 1− (1 + r)−60

or 56r(1 + r)60 = (1 + r)60 − 1

or (1− 56r)(1 + r)60 = 1

Set f(r) = (1− 56r)(1 + r)60 − 1. Then

f ′(r) = −56(1 + r)60 + 60(1− 56r)(1 + r)59

orf ′(r) =

[− 56(1 + r) + 60(1− 56r)

](1 + r)59 = (4− 3416r)(1 + r)59

Apply Newton’s method with an initial guess of r1 = .002. (That’s 0.2% per monthor 2.4% per year.) Then

r2 = r1 −(1− 56r1)(1 + r1)60 − 1

(4− 3416r1)(1 + r1)59= 0.002344

r3 = r2 −(1− 56r2)(1 + r2)60 − 1

(4− 3416r2)(1 + r2)59= 0.002292

r4 = r3 −(1− 56r3)(1 + r3)60 − 1

(4− 3416r3)(1 + r3)59= 0.002290

r5 = r4 −(1− 56r4)(1 + r4)60 − 1

(4− 3416r4)(1 + r4)59= 0.002290

So the interest rate is 0.229% per month or 2.75% per year.

a “Compounded monthly”, means that, each month, interest is paid on the accumulated interestthat was paid in all previous months.

b Inflation means that prices of goods (typically) increase with time, and hence $100 now is worthmore than $100 in 10 years time. The term “present value” is widely used in economics andfinance to mean “the current amount of money that will have a specified value at a specifiedtime in the future”. It takes inflation into account. If the money is invested, it takes into accountthe rate of return of the investment. We recommend that the interested reader do some search-engining to find out more.

c Don’t worry if you don’t know how to evaluate such sums. They are called geometric sums, andwill be covered in the CLP-2 text. (See (1.1.3) in the CLP-2 text. In any event, you can check thatthis is correct, by multiplying the whole equation by 1 − (1 + r)−1. When you simplify the lefthand side, you should get the right hand side.

534

Page 551: CLP-1 Differential Calculus

ROOT FINDING C.2 THE ERROR BEHAVIOUR OF NEWTON’S METHOD

C.2q The Error Behaviour of Newton’s Method

Newton’s method usually works spectacularly well, provided your initial guess isreasonably close to a solution of f(x) = 0. A good way to select this initial guess is tosketch the graph of y = f(x). We now explain why “Newton’s method usually worksspectacularly well, provided your initial guess is reasonably close to a solution off(x) = 0”.

Let r be any solution of f(x) = 0. Then f(r) = 0. Suppose that we have alreadycomputed xn. The error in xn is

∣∣xn − r∣∣. We now derive a formula that relates theerror after the next step,

∣∣xn+1 − r∣∣, to

∣∣xn − r∣∣. We have seen in (3.4.32) that

f(x) = f(xn) + f ′(xn)(x− xn) +1

2f ′′(c)(x− xn)2

for some c between xn and x. In particular, choosing x = r,

0 = f(r) = f(xn) + f ′(xn)(r − xn) +1

2f ′′(c)(r − xn)2 (E1)

Recall that xn+1 is the solution of 0 = f(xn) + f ′(xn)(x− xn). So

0 = f(xn) + f ′(xn)(xn+1 − xn) (E2)

We need to get an expression for xn+1 − r. Subtracting (E2) from (E1) gives

0 = f ′(xn)(r − xn+1) +1

2f ′′(c)(r − xn)2 =⇒ xn+1 − r =

f ′′(c)

2f ′(xn)(xn − r)2

=⇒∣∣xn+1 − r

∣∣ =|f ′′(c)|

2|f ′(xn)| |xn − r|2

If the guess xn is close to r, then c, which must be between xn and r, is also close to rand we will have f ′′(c) ≈ f ′′(r) and f ′(xn) ≈ f ′(r) and

∣∣xn+1 − r∣∣ ≈ |f ′′(r)|

2|f ′(r)| |xn − r|2 (E3)

Even when xn is not close to r, if we know that there are two numbers L,M > 0 suchthat f obeys:

(H1)∣∣f ′(xn)

∣∣ ≥ L

(H2)∣∣f ′′(c)∣∣ ≤M

(we’ll see examples of this below) then we will have

∣∣xn+1 − r∣∣ ≤ M

2L|xn − r|2 (E4)

535

Page 552: CLP-1 Differential Calculus

ROOT FINDING C.2 THE ERROR BEHAVIOUR OF NEWTON’S METHOD

Let’s denote by ε1 the error, |x1− r|, of our initial guess. In fact, let’s denote by εn theerror, |xn − r|, in xn. Then (E4) says

εn+1 ≤M

2Lε2n

In particular

ε2 ≤M

2Lε2

1

ε3 ≤M

2Lε2

2 ≤M

2L

(M

2Lε2

1

)2

=

(M

2L

)3

ε41

ε4 ≤M

2Lε2

3 ≤M

2L

[(M

2L

)3

ε41

]2

=

(M

2L

)7

ε81

ε5 ≤M

2Lε2

4 ≤M

2L

[(M

2L

)7

ε81

]2

=

(M

2L

)15

ε161

By now we can see a pattern forming, that is easily verified by induction1.

εn ≤(M

2L

)2n−1−1

ε2n−1

1 =2L

M

(M

2Lε1

)2n−1

(E5)

As long as M2Lε1 < 1 (which gives us a quantitative idea as to how good our first

guess has to be in order for Newton’s method to work), this goes to zero extremelyquickly as n increases. For example, suppose that M

2Lε1 ≤ 1

2. Then

εn ≤2L

M

(1

2

)2n−1

≤ 2L

0.25 if n = 2

0.0625 if n = 3

0.0039 = 3.9× 10−3 if n = 4

0.000015 = 1.5× 10−5 if n = 5

0.00000000023 = 2.3× 10−10 if n = 6

0.000000000000000000054 = 5.4× 10−20 if n = 7

Each time you increase n by one, the number of zeroes after the decimal place roughlydoubles. You can see why from (E5). Since(

M

2Lε1

)2(n+1)−1

=

(M

2Lε1

)2n−1×2

=

[(M

2Lε1

)2n−1]2

we have, very roughly speaking, εn+1 ≈ ε2n. This quadratic behaviour is the reason

that Newton’s method is so useful.

1 Mathematical induction is a technique for proving a sequence S1, S2, S3, · · · of statements. Thattechnique consists of first proving that S1 is true, and then proving that, for any natural numbern, if Sn is true then Sn+1 is true.

536

Page 553: CLP-1 Differential Calculus

ROOT FINDING C.2 THE ERROR BEHAVIOUR OF NEWTON’S METHOD

Example C.2.1 (Example C.1.2, continued).

Let’s consider, as we did in Example C.1.2, f(x) = x2 − 2, starting with x1 = 32. Then

f ′(x) = 2x f ′′(x) = 2

Recalling, from (H1) and (H2), that L is a lower bound on |f ′| and M is an upperbound on |f ′′|, we may certainly take M = 2 and if, for example, xn ≥ 1 for all n (ashappened in Example C.1.2), we may take L = 2 too. While we do not know what ris, we do know that 1 ≤ r ≤ 2 (since f(1) = 11 − 2 < 0 and f(2) = 22 − 2 > 0). As wetook x1 = 3

2, we have ε1 = |x1 − r| ≤ 1

2, so that M

2Lε1 ≤ 1

4and

εn+1 ≤2L

M

(M

2Lε1

)2n−1

≤ 2

(1

4

)2n−1

(E6)

This tends to zero very quickly as n increases. Furthermore this is an upper boundon the error and not the actual error. In fact (E6) is a very crude upper bound. Forexample, setting n = 3 gives the bound

ε4 ≤ 2

(1

4

)22

= 7× 10−3

and we saw in Example C.1.2 that the actual error in x4 was smaller than 5× 10−10.

Example C.2.2 (Example C.1.3, continued).

Let’s consider, as we did in Example C.1.3, f(x) = sinx, starting with x1 = 3. Then

f ′(x) = cos x f ′′(x) = − sinx

As | − sinx| ≤ 1, we may certainly take M = 1. In Example C.1.3, all xn’s werebetween 3 and 3.2. Since (to three decimal places)

sin(3) = 0.141 > 0 sin(3.2) = −0.058 < 0

the IVT (intermediate value theorem) tells us that 3 < r < 3.2 and ε1 = |x1− r| < 0.2.So r and all xn’s and hence all c’s lie in the interval (3, 3.2). Since

−0.9990 = cos(3) < cos c < cos(3.2) = −0.9983

we necessarily have∣∣f ′(c)∣∣ =

∣∣ cos c∣∣ ≥ 0.9 and we may take L = 0.9. So

εn+1 ≤2L

M

(M

2Lε1

)2n−1

≤ 2× 0.9

1

(1

2× 0.90.2

)2n−1

≤ 2

(1

9

)2n−1

This tends to zero very quickly as n increases.

537

Page 554: CLP-1 Differential Calculus

ROOT FINDING C.3 THE FALSE POSITION (REGULA FALSI) METHOD

We have now seen two procedures for finding roots of a function f(x) — the bi-section method (which does not use the derivative of f(x), but which is not veryefficient) and Newton’s method (which does use the derivative of f(x), and which isvery efficient). In fact, there is a whole constellation of other methods2 and the inter-ested reader should search engine their way to, for example, Wikipedia’s article onroot finding algorithms. Here, we will just mention two other methods, one being avariant of the bisection method and the other being a variant of Newton’s method.

C.3q The false position (regula falsi) method

Let f(x) be a continuous function and let a1 < b1 with f(a1) and f(b1) being ofopposite sign.

As we have seen, the bisection method generates a sequence of intervals In =[an, bn], n = 1, 2, 3, · · · with, for each n, f(an) and f(bn) having opposite sign (so that,by continuity, f has a root in In). Once we have In, we choose In+1 based on the signof f at the midpoint, an+bn

2, of In. Since we always test the midpoint, the possible

error decreases by a factor of 2 each step.The false position method tries to make the whole procedure more efficient by

testing the sign of f at a point that is closer to the end of In where the magnitude off is smaller. To be precise, we approximate y = f(x) by the equation of the straightline through

(an, f(an)

)and

(bn, f(bn)

).

xan bncn

y = f(x)

y = F (x)

(an, f(an))

(bn, f(bn))

The equation of that straight line is

y = F (x) = f(an) +f(bn)− f(an)

bn − an(x− an)

2 What does it say about mathematicians that they have developed so many ways of finding zero?

538

Page 555: CLP-1 Differential Calculus

ROOT FINDING C.4 THE SECANT METHOD

Then the false position method tests the sign of f(x) at the value of xwhere F (x) = 0.

F (x) = f(an) +f(bn)− f(an)

bn − an(x− an) = 0

⇐⇒ x = an −bn − an

f(bn)− f(an)f(an) =

anf(bn)− bnf(an)

f(bn)− f(an)

So once we have the interval In, the false position method generates the interval In+1

by the following rule.1

Equation C.3.1 fale position method.

Set cn = anf(bn)−bnf(an)f(bn)−f(an)

. If f(cn) has the same sign as f(an), then

In+1 = [an+1, bn+1] with an+1 = cn, bn+1 = bn

and if f(cn) and f(an) have opposite signs, then

In+1 = [an+1, bn+1] with an+1 = an, bn+1 = cn

C.4q The secant method

Let f(x) be a continuous function. The secant method is a variant of Newton’smethod that avoids the use of the derivative of f(x) — which can be very helpfulwhen dealing with the derivative is not easy. It avoids the use of the derivative byapproximating f ′(x) by f(x+h)−f(x)

hfor some h. That is, it approximates the tangent

line to f at x by a secant line for f that passes through x. To limit the number ofevaluations of f(x) required, it uses x = xn−1 and x+ h = xn. Here is how it works.

Suppose that we have already found xn. Then we denote by y = F (x) the equa-tion of the (secant) line that passes through

(xn−1, f(xn−1)

)and

(xn, f(xn)

)and we

choose xn+1 to be the value of x where F (x) = 0.

1 The convergence behaviour of the false position method is relatively complicated. So we do notdiscuss it here. As always, we invite the interested reader to visit their favourite search engine.

539

Page 556: CLP-1 Differential Calculus

ROOT FINDING C.4 THE SECANT METHOD

xxn−1 xn xn+1

y = f(x)

y = F (x)

(xn−1, f(xn−1))

(xn, f(xn))

The equation of the secant line is

y = F (x) = f(xn−1) +f(xn)− f(xn−1)

xn − xn−1

(x− xn−1)

so that xn+1 is determined by

0 = F (xn+1) = f(xn−1) +f(xn)− f(xn−1)

xn − xn−1

(xn+1 − xn−1)

⇐⇒ xn+1 = xn−1 −xn − xn−1

f(xn)− f(xn−1)f(xn−1)

or, simplifying,

Equation C.4.1 secant method.

xn+1 =xn−1f(xn)− xnf(xn−1)

f(xn)− f(xn−1)

Of course, to get started with n = 1, we need two initial guesses, x0 and x1, forthe root.

Example C.4.2 Approximating√

2, again.

In this example we compute, approximately, the square root of two by applying thesecant method to the equation

f(x) = x2 − 2 = 0

and we’ll compare the secant method results with the corresponding Newton’smethod results. (See Example C.1.2.)Since f ′(x) = 2x, (C.1.1) says that, under Newton’s method, we should iteratively

540

Page 557: CLP-1 Differential Calculus

ROOT FINDING C.4 THE SECANT METHOD

Example C.4.2

apply

xn+1 = xn −f(xn)

f ′(xn)= xn −

x2n − 2

2xn=xn2

+1

xn

while (C.4.1) says that, under the secant method, we should iteratively apply (aftera little simplifying algebra)

xn+1 =xn−1f(xn)− xnf(xn−1)

f(xn)− f(xn−1)=xn−1[x2

n − 2]− xn[x2n−1 − 2]

x2n − x2

n−1

=xn−1xn[xn − xn−1] + 2[xn − xn−1]

x2n − x2

n−1

=xn−1xn + 2

xn−1 + xn

Here are the results, starting Newton’s method with x1 = 4 and starting the secantmethod with x0 = 4, x1 = 3. (So we are giving the secant method a bit of a headstart.)

secant method Newton’s methodx0 4

x1 3 4

x2 2 2.25

x3 1.6 1.57

x4 1.444 1.422

x5 1.4161 1.414234

x6 1.414233 1.414213562525

x7 1.414213575 1.414213562373095

For comparison purposes, the square root of 2, to 15 decimal places, is1.414213562373095. So the secant method x7 is accurate to 7 decimal places and theNewton’s method x7 is accurate to at least 15 decimal places.

The advantage that the secant method has over Newton’s method is that it doesnot use the derivative of f . This can be a substantial advantage, for example whenevaluation of the derivative is computationally difficult or expensive. On the otherhand, the above example suggests that the secant method is not as fast as Newton’smethod. The following subsection shows that this is indeed the case.

541

Page 558: CLP-1 Differential Calculus

ROOT FINDING C.5 THE ERROR BEHAVIOUR OF THE SECANT METHOD

C.5q The Error Behaviour of the Secant Method

Let f(x) have two continuous derivatives, and let r be any solution of f(x) = 0. Wewill now get a pretty good handle on the error behaviour of the secant method nearr.

Denote by εn = xn − r the (signed) error in xn and by εn = |xn − r| the (absolute)error in xn. Then, xn = r + εn, and, by (C.4.1),

εn+1 =xn−1f(xn)− xnf(xn−1)

f(xn)− f(xn−1)− r

=[r + εn−1]f(xn)− [r + εn]f(xn−1)

f(xn)− f(xn−1)− r

=εn−1f(xn)− εnf(xn−1)

f(xn)− f(xn−1)

By the Taylor expansion (3.4.32) and the mean value theorem (Theorem 2.13.5),

f(xn) = f(r) + f ′(r)εn +1

2f ′′(c1)ε2

n

= f ′(r)εn +1

2f ′′(c1)ε2

n

f(xn)− f(xn−1) = f ′(c2)[xn − xn−1]

= f ′(c2)[εn − εn−1]

for some c1 between r and xn and some c2 between xn−1 and xn. So, for xn−1 and xnnear r, c1 and c2 also have to be near r and

f(xn) ≈ f ′(r)εn +1

2f ′′(r)ε2

n

f(xn−1) ≈ f ′(r)εn−1 +1

2f ′′(r)ε2

n−1

f(xn)− f(xn−1) ≈ f ′(r)[εn − εn−1]

and

εn+1 =εn−1f(xn)− εnf(xn−1)

f(xn)− f(xn−1)

≈ εn−1[f ′(r)εn + 12f ′′(r)ε2

n]− εn[f ′(r)εn−1 + 12f ′′(r)ε2

n−1]

f ′(r)[εn − εn−1]

=12εn−1εnf

′′(r)[εn − εn−1]

f ′(r)[εn − εn−1]

=f ′′(r)

2f ′(r)εn−1εn

542

Page 559: CLP-1 Differential Calculus

ROOT FINDING C.5 THE ERROR BEHAVIOUR OF THE SECANT METHOD

Taking absolute values, we have

εn+1 ≈ Kεn−1εn with K =

∣∣∣∣ f ′′(r)2f ′(r)

∣∣∣∣ (E7)

We have seen that Newton’s method obeys a similar formula — (E3) says that, whenxn is near r, Newton’s method obeys εn+1 ≈ Kε2

n, also with K =∣∣∣ f ′′(r)2f ′(r)

∣∣∣. As we shallnow see, the change from ε2

n, in εn+1 ≈ Kε2n, to εn−1εn, in εn+1 ≈ Kεn−1εn, does have

a substantial impact on the behaviour of εn for large n.To see the large n behaviour, we now iterate (E7). The formulae will look simpler

if we multiply (E7) by K and write δn = Kεn. Then (E7) becomes δn+1 ≈ δn−1δn (andwe have eliminated K). The first iterations are

δ2 ≈ δ0δ1

δ3 ≈ δ1δ2 ≈ δ0δ21

δ4 ≈ δ2δ3 ≈ δ20δ

31

δ5 ≈ δ3δ4 ≈ δ30δ

51

δ6 ≈ δ4δ5 ≈ δ50δ

81

δ7 ≈ δ5δ6 ≈ δ80δ

131

Notice that every δn is of the form δαn0 δβn1 . Substituting δn = δαn0 δβn1 into δn+1 ≈ δn−1δngives

δαn+1

0 δβn+1

1 ≈ δαn−1

0 δβn−1

1 δαn0 δβn1

and we haveαn+1 = αn−1 + αn βn+1 = βn−1 + βn (E8)

The recursion rule in (E8) is famous1. The Fibonacci2 sequence (which is 0, 1, 1, 2,3, 5, 8, 13, · · ·), is defined by

F0 = 0

F1 = 1

Fn = Fn−1 + Fn−2 for n > 1

So, for n ≥ 2, αn = Fn−1 and βn = Fn and

δn ≈ δαn0 δβn1 = δFn−1

0 δFn1

One of the known properties of the Fibonacci sequence is that, for large n,

Fn ≈ϕn√

5where ϕ =

1 +√

5

2≈ 1.61803

1 Plug “Fibonacci sequence in nature” into your search engine of choice.2 Fibonacci (1170-1250) was an Italian mathematician who was also known as Leonardo of Pisa,

Leonardo Bonacci and Leonardo Biglio Pisano.

543

Page 560: CLP-1 Differential Calculus

ROOT FINDING C.5 THE ERROR BEHAVIOUR OF THE SECANT METHOD

This ϕ is the golden ratio3. So, for large n,

Kεn = δn ≈ δFn−1

0 δFn1 ≈ δϕn−1√5

0 δϕn√

5

1 = δ1√5ϕ×ϕn

0 δ1√5×ϕn

1

= dϕn

where d = δ1√5ϕ

0 δ1√5

1

≈ d1.6n

Assuming that 0 < δ0 = Kε0 < 1 and 0 < δ1 = Kε1 < 1, we will have 0 < d < 1.By way of contrast, for Newton’s method, for large n,

Kεn ≈ d2n where d = (Kε1)1/2

As 2n grows quite a bit more quickly than 1.6n (for example, when n=5, 2n = 32and 1.6n = 10.5, and when n = 10, 2n = 1024 and 1.6n = 110) Newton’s methodhomes in on the root quite a bit faster than the secant method, assuming that youstart reasonably close to the root.

3 Also worth a quick trip to your search engine.

544

Page 561: CLP-1 Differential Calculus

HINTS FOR EXERCISES

Appendix D

1 · Limits1.1 · Drawing Tangents and a First Limit1.1.2 · Exercises

Exercises — Stage 11.1.2.2. Hint. The tangent line to a curve at point P passes through P .

1.1.2.3. Hint. Try drawing tangent lines to the following curves, at the givenpoints P :

x

y y = f(x)

P

x

yy = f(x)

P

x

y

y = f(x)

P

1.2 · Another Limit and Computing Velocity1.2.2 · Exercises

Exercises — Stage 11.2.2.3. Hint. Where did you start, and where did you end?

1.2.2.4. Hint. Is the object falling faster and faster, slower and slower, or at aconstant rate?

545

Page 562: CLP-1 Differential Calculus

HINTS FOR EXERCISES

1.2.2.5. Hint. Slope is change in vertical component over change in horizontalcomponent.

1.2.2.6. Hint. Sign of velocity gives direction of motion: the velocity is positiveat time t if s(t) is increasing at time t.

Exercises — Stage 21.2.2.7. Hint. Velocity is distance over time.

1.2.2.8. Hint. Use that√a−bc

=√a−bc·(√

a+b√a+b

)= a−b2

c(√a+b)

.

1.3 · The Limit of a Function1.3.2 · Exercises

Exercises — Stage 11.3.2.2. Hint. Consider the difference between a limit and a one-sided limit.

1.3.2.3. Hint. Pay careful attention to which limits are one-sided and which arenot.

1.3.2.5. Hint. The function doesn’t have to be continuous.

1.3.2.6. Hint. See Question 1.3.2.5

1.3.2.7. Hint. See Question 1.3.2.5

1.3.2.8. Hint. What is the relationship between the limit and the two one-sidedlimits?

1.3.2.9. Hint. What is the relationship between the limit and the two one-sidedlimits?

Exercises — Stage 21.3.2.14. Hint. What are the one-sided limits?

1.3.2.16. Hint. Think about what it means that x does not appear in the function

f(x) =1

10.

1.3.2.17. Hint. We only care about what happens really, really close to x = 3.

1.4 · Calculating Limits with Limit Laws1.4.2 · Exercises

Exercises — Stage 11.4.2.2. Hint. Try to make two functions with factors that will cancel.

1.4.2.3. Hint. Try to make g(x) cancel out.

546

Page 563: CLP-1 Differential Calculus

HINTS FOR EXERCISES

1.4.2.5. Hint. See Questions 1.4.2.2, 1.4.2.3, and 1.4.2.4.

Exercises — Stage 21.4.2.6. Hint. Find the limit of the numerator and denominator separately.

1.4.2.7. Hint. Break it up into smaller pieces, evaluate the limits of the pieces.

1.4.2.8. Hint. First find the limit of the “inside” function,4x− 2

x+ 2.

1.4.2.9. *. Hint. Is cos(−3) zero?

1.4.2.10. *. Hint. Expand, then simplify.

1.4.2.14. *. Hint. Try the simplest method first.

1.4.2.15. *. Hint. Factor the denominator.

1.4.2.16. *. Hint. Factor the numerator and the denominator.

1.4.2.17. *. Hint. Factor the numerator.

1.4.2.18. *. Hint. Simplify first by factoring the numerator.

1.4.2.19. Hint. The function is a polynomial.

1.4.2.20. *. Hint. Multiply both the numerator and the denominator by theconjugate of the numerator,

√x2 + 8 + 3.

1.4.2.21. *. Hint. Multiply both the numerator and the denominator by theconjugate of the numerator,

√x+ 7 +

√11− x.

1.4.2.22. *. Hint. Multiply both the numerator and the denominator by theconjugate of the numerator,

√x+ 2 +

√4− x.

1.4.2.23. *. Hint. Multiply both the numerator and the denominator by theconjugate of the numerator,

√x− 2 +

√4− x.

1.4.2.24. *. Hint. Multiply both the numerator and the denominator by theconjugate of the denominator, 2 +

√5− t.

1.4.2.25. Hint. Consider the factors x2 and cos(

3x

)separately. Review the

squeeze theorem.

1.4.2.26. Hint. Look for a reason to ignore the trig. Review the squeeze theorem.

1.4.2.27. *. Hint. As in the previous questions, we want to use the SqueezeTheorem. If x < 0, then −x is positive, so x < −x. Use this fact when you boundyour expressions.

1.4.2.28. Hint. Factor the numerator.

1.4.2.29. Hint. Factor the denominator; pay attention to signs.

1.4.2.30. Hint. First find the limit of the “inside” function.

547

Page 564: CLP-1 Differential Calculus

HINTS FOR EXERCISES

1.4.2.31. Hint. Factor; pay attention to signs.

1.4.2.32. Hint. Look for perfect squares

1.4.2.33. Hint. Think about what effect changing d has on the function x5 −32x+ 15.

1.4.2.34. Hint. There’s an easy way.

1.4.2.35. *. Hint. What can you do to safely ignore the sine function?

1.4.2.36. *. Hint. Factor

1.4.2.37. Hint. If you’re looking at the hints for this one, it’s probably easierthan you think.

1.4.2.38. Hint. You’ll want to simplify this, since t = 12

is not in the domain ofthe function. One way to start your simplification is to add the fractions in thenumerator by finding a common denominator.

1.4.2.39. Hint. If you’re not sure how|x|x

behaves, try plugging in a few valuesof x, like x = ±1 and x = ±2.

1.4.2.40. Hint. Look to Question 1.4.2.39 to see how a function of the form|X|X

behaves.

1.4.2.41. Hint. Is anything weird happening to this function at x = 0?

1.4.2.42. Hint. Use the limit laws.

1.4.2.43. *. Hint. The denominator goes to zero; what must the numerator goto?

Exercises — Stage 31.4.2.45. Hint. Try plotting points. If you can’t divide by f(x), take a limit.

1.4.2.46. Hint. There is a close relationship between f and g. Fill in the follow-ing table:

x f(x) g(x)f(x)

g(x)−3

−2

−1

−0

1

2

3

548

Page 565: CLP-1 Differential Calculus

HINTS FOR EXERCISES

1.4.2.47. Hint. Velocity of white ball when t = 1 is limh→0

s(1 + h)− s(1)

h.

1.4.2.49. Hint. When you’re evaluating limx→0−

f(x), you’re only considering val-

ues of x that are less than 0.

1.4.2.50. Hint. When you’re considering limx→−4−

f(x), you’re only considering

values of x that are less than−4. When you’re considering limx→−4+

f(x), think about

the domain of the rational function in the top line.

1.5 · Limits at Infinity1.5.2 · Exercises

Exercises — Stage 11.5.2.1. Hint. It might not look like a traditional polynomial.

1.5.2.2. Hint. The degree of the polynomial matters.

Exercises — Stage 21.5.2.3. Hint. What does a negative exponent do?

1.5.2.4. Hint. You can think about the behaviour of this function by remember-ing how you first learned to describe exponentiation.

1.5.2.5. Hint. The exponent will be a negative number.

1.5.2.6. Hint. What single number is the function approaching?

1.5.2.7. Hint. The highest-order term dominates when x is large.

1.5.2.8. Hint. Factor the highest power of x out of both the numerator and thedenominator. You can factor through square roots (carefully).

1.5.2.9. *. Hint. Multiply and divide by the conjugate,√x2 + 5x+

√x2 − x.

1.5.2.10. *. Hint. Divide both the numerator and the denominator by the high-est power of x that is in the denominator.Remember that√ is defined to be the positive square root. Consequently, if x < 0,then

√x2, which is positive, is not the same as x, which is negative.

1.5.2.11. *. Hint. Factor out the highest power of the denominator.

1.5.2.12. *. Hint. The conjugate of (√x2 + x− x) is (

√x2 + x+ x).

Multiply by 1 =

√x2 + x+ x√x2 + x+ x

to coax your function into a fraction.

1.5.2.13. *. Hint. Divide both the numerator and the denominator by the high-est power of x that is in the denominator.

549

Page 566: CLP-1 Differential Calculus

HINTS FOR EXERCISES

1.5.2.14. *. Hint. Divide both the numerator and the denominator by the high-est power of x that is in the denominator.

1.5.2.15. *. Hint. Divide both the numerator and the denominator by the high-est power of x that is in the denominator.

1.5.2.16. Hint. Divide both the numerator and the denominator by x (which isthe largest power of x in the denominator). In the numerator, move the resultingfactor of 1/x inside the two roots. Be careful about the signs when you do so.Even and odd roots behave differently– see Question 1.5.2.10.

1.5.2.17. *. Hint. Divide both the numerator and the denominator by the high-est power of x that is in the denominator.

1.5.2.18. Hint. Divide both the numerator and the denominator by the highestpower of x that is in the denominator. It is not always true that

√x2 = x.

1.5.2.19. Hint. Simplify.

1.5.2.20. *. Hint. What is a simpler version of |x|when you know x < 0?

1.5.2.22. *. Hint. Divide both the numerator and the denominator by the high-est power of x that is in the denominator. When is

√x = x, and when is

√x = −x?

1.5.2.23. Hint. Divide both the numerator and the denominator by the highestpower of x that is in the denominator. Pay careful attention to signs.

1.5.2.24. *. Hint. Multiply and divide the expression by its conjugate,(√n2 + 5n+ n

).

1.5.2.25. Hint. Consider what happens to the function as a becomes very, verysmall. You shouldn’t need to do much calculation.

1.5.2.26. Hint. Since x = 3 is not in the domain of the function, we need to be alittle creative. Try simplifying the function.

Exercises — Stage 31.5.2.27. Hint. This is a bit of a trick question. Consider what happens to arational function as x→ ±∞ in each of these three cases:

• the degree of the numerator is smaller than the degree of the denominator,

• the degree of the numerator is the same as the degree of the denominator,and

• the degree of the numerator is larger than the degree of the denominator.

1.5.2.28. Hint. We tend to conflate “infinity” with “some really large number.”

1.6 · Continuity1.6.4 · Exercises

550

Page 567: CLP-1 Differential Calculus

HINTS FOR EXERCISES

Exercises — Stage 11.6.4.1. Hint. Try a repeating pattern.

1.6.4.2. Hint. f is my height.

1.6.4.3. Hint. The intermediate value theorem only works for a certain kind offunction.

1.6.4.7. Hint. Compare what is given to you to the definition of continuity.

1.6.4.8. Hint. Compare what is given to you to the definition of continuity.

1.6.4.9. Hint. What if the function is discontinuous?

1.6.4.10. Hint. What is h(0)?

Exercises — Stage 21.6.4.11. Hint. Use the definition of continuity.

1.6.4.12. Hint. If this is your password, you might want to change it.

1.6.4.13. *. Hint. Find the domain: when is the denominator zero?

1.6.4.14. *. Hint. When is the denominator zero? When is the argument of thesquare root negative?

1.6.4.15. *. Hint. When is the denominator zero? When is the argument of thesquare root negative?

1.6.4.16. *. Hint. There are infinitely many points where it is not continuous.

1.6.4.17. *. Hint. x = c is the important point.

1.6.4.18. *. Hint. The important place is x = 0.

1.6.4.19. *. Hint. The important point is x = c.

1.6.4.20. *. Hint. The important point is x = 2c.

Exercises — Stage 31.6.4.21. Hint. Consider the function f(x) = sin x− x+ 1.

1.6.4.22. *. Hint. Consider the function f(x) = 3x− x2, and how it relates to theproblem and the IVT.

1.6.4.23. *. Hint. Consider the function 2 tanx− x− 1 and its roots.

1.6.4.24. *. Hint. Consider the function f(x) =√

cos(πx)− sin(2πx)− 1/2, andbe careful about where it is continuous.

1.6.4.25. *. Hint. Consider the function f(x) = 1/ cos2(πx) − x − 32, paying

attention to where it is continuous.

551

Page 568: CLP-1 Differential Calculus

HINTS FOR EXERCISES

1.6.4.26. Hint. We want f(x) to be 0; 0 is between a positive number and anegative number. Try evaluating f(x) for some integer values of x.

1.6.4.27. Hint. 3√

7 is the value where x3 = 7.

1.6.4.28. Hint. You need to consider separately the cases where f(a) < g(a) andf(a) = g(a). Let h(x) = f(x)− g(x). What is h(c)?

2 · Derivatives2.1 · Revisiting Tangent Lines2.1.2 · Exercises

Exercises — Stage 12.1.2.2. Hint. You can use 2.1.2.2.a to explain 2.1.2.2.b.

2.1.2.3. Hint. Your calculations for slope of the secant lines will all have thesame denominators; to save yourself some time, you can focus on the numera-tors.

Exercises — Stage 22.1.2.4. Hint. You can do this by calculating several secant lines. You can alsodo this by getting out a ruler and trying to draw the tangent line very carefully.

2.1.2.5. Hint. There are many possible values for Q and R.

2.1.2.6. Hint. A line with slope 0 is horizontal.

2.2 · Definition of the Derivative2.2.4 · Exercises

Exercises — Stage 12.2.4.1. Hint. What are the properties of f ′ when f is a line?

2.2.4.2. Hint. Be very careful not to confuse f and f ′.

2.2.4.3. Hint. Be very careful not to confuse f and f ′.

2.2.4.5. Hint. The slope has to look “the same” from the left and the right.

2.2.4.6. Hint. Use the definition of the derivative, and what you know aboutlimits.

2.2.4.7. Hint. Consider continuity.

2.2.4.8. Hint. Look at the definition of the derivative. Your answer will be afraction.

Exercises — Stage 2

552

Page 569: CLP-1 Differential Calculus

HINTS FOR EXERCISES

2.2.4.9. Hint. You need a point (given), and a slope (derivative).

2.2.4.10. Hint. You’ll need to add some fractions.

2.2.4.11. *. Hint. You don’t have to take the limit from the left and rightseparately–things will cancel nicely.

2.2.4.12. *. Hint. You might have to add fractions.

2.2.4.14. Hint. Your limit should be easy.

2.2.4.15. *. Hint. Add fractions.

2.2.4.16. *. Hint. For f to be differentiable at x = 2, two things must be true:it must be continuous at x = 2, and the derivative from the right must equal thederivative from the left.

2.2.4.17. *. Hint. After you plug in f(x) to the definition of a derivative, you’llwant to multiply and divide by the conjugate

√1 + x+ h+

√1 + x.

Exercises — Stage 32.2.4.18. Hint. From Section 1.2, compare the definition of velocity to the defi-nition of a derivative. When you’re finding the derivative, you’ll need to cancela lot on the numerator, which you can do by expanding the polynomials.

2.2.4.19. *. Hint. You’ll need to look at limits from the left and right. The factthat f(0) = 0 is useful for your computation. Recall that if x < 0 then

√x2 = |x| =

−x.

2.2.4.20. *. Hint. You’ll need to look at limits from the left and right. The factthat f(0) = 0 is useful for your computation.

2.2.4.21. *. Hint. You’ll need to look at limits from the left and right. The factthat f(0) = 0 is useful for your computation.

2.2.4.22. *. Hint. You’ll need to look at limits from the left and right. The factthat f(1) = 0 is useful for your computation.

2.2.4.23. . Hint. There’s lots of room between 0 and 18; see what you can do with

it.

2.2.4.24. Hint. Set up your usual limit, then split it into two pieces

2.2.4.25. Hint. You don’t need the definition of the derivative for a line.

2.2.4.26. *. Hint. A generic point on the curve has coordinates (α, α2). In termsof α, what is the equation of the tangent line to the curve at the point (α, α2)?What does it mean for (1,−3) to be on that line?

2.2.4.27. *. Hint. Remember for a constant n,

limh→0

hn =

0 n > 0

1 n = 0

DNE n < 0

553

Page 570: CLP-1 Differential Calculus

HINTS FOR EXERCISES

2.3 · Interpretations of the Derivative2.3.3 · Exercises

Exercises — Stage 22.3.3.1. Hint. Think about units.

Exercises — Stage 32.3.3.8. Hint. There are 360 degrees in one rotation.

2.3.3.9. Hint. P ′(t) was discussed in Question 2.3.3.7.

2.4 · Arithmetic of Derivatives - a Differentiation Toolbox2.4.2 · Exercises

Exercises — Stage 12.4.2.1. Hint. Look at the Sum rule.

2.4.2.2. Hint. Try an example, like f(x) = g(x) = x.

2.4.2.3. Hint. Simplify.

2.4.2.4. Hint. g(x) = f(x) + f(x) + f(x)

Exercises — Stage 22.4.2.5. Hint. Use linearity.

2.4.2.6. Hint. You have already seen ddx√x.

2.4.2.7. *. Hint. The equation of a line can be determined using a point, and theslope.

2.4.2.8. *. Hint. Be careful to distinguish between speed and velocity.

2.4.2.10. Hint. How do you take care of that power?

2.4.2.11. Hint. You know how to take the derivative of a reciprocal; this mightbe faster than using the quotient rule.

554

Page 571: CLP-1 Differential Calculus

HINTS FOR EXERCISES

Exercises — Stage 32.4.2.12. Hint. Population growth is rate of change of population.

2.4.2.14. *. Hint. Interpret it as a derivative that you know how to compute.

2.4.2.15. Hint. The answer is not 10 square metres per second.

2.4.2.16. Hint. You don’t need to know g(0) or g′(0).

2.6 · Using the Arithmetic of Derivatives – Examples2.6.2 · Exercises

Exercises — Stage 12.6.2.1. Hint. Check signs.

2.6.2.2. Hint. Read Lemma 2.6.9 carefully.

Exercises — Stage 22.6.2.3. Hint. First, factor an x out of the derivative. What’s left over looks likea quadratic equation, if you take x2 to be your variable, instead of x.

2.6.2.4. Hint. 1t

= t−1

2.6.2.5. Hint. First simplify. Don’t be confused by the role reversal of x and y: xis just the name of the function

(2y+ 1

y

)· y3, which is a function of the variable y.

You are to differentiate with respect to y.

2.6.2.6. Hint.√x = x1/2

2.6.2.8. Hint. You don’t need to multiply through.

2.6.2.9. Hint. You can use the quotient rule.

2.6.2.13. *. Hint. There are two pieces of the given function that could causeproblems.

2.6.2.14. Hint. 3√x = x1/3

2.6.2.15. Hint. Simplify first.

Exercises — Stage 32.6.2.17. *. Hint. Let m be the slope of such a tangent line, and let P1 and P2

be the points where the tangent line is tangent to the two curves, respectively.There are three equations m fulfils: it has the same slope as the curves at thegiven points, and it is the slope of the line passing through the points P1 and P2.

2.6.2.18. Hint. A line has equation y = mx + b, for some constants m and b.What has to be true for y = mb + x to be tangent to the first curve at the pointx = α, and to the second at the point x = β?

555

Page 572: CLP-1 Differential Calculus

HINTS FOR EXERCISES

2.6.2.19. *. Hint. Compare this to one of the forms given in the text for thedefinition of the derivative.

2.6.2.20. *. Hint. Compare this to one of the forms given in the text for thedefinition of the derivative.

2.6.2.21. *. Hint. Compare this to one of the forms given in the text for thedefinition of the derivative.

2.6.2.22. *. Hint. Compare this to one of the forms given in the text for thedefinition of the derivative.

2.7 · Derivatives of Exponential Functions2.7.3 · Exercises

Exercises — Stage 12.7.3.1. Hint. Two of the functions are the same.

2.7.3.3. Hint. When can you use the power rule?

2.7.3.4. Hint. What is the shape of the curve eax, when a is a positive consant?

Exercises — Stage 22.7.3.5. Hint. Quotient rule

2.7.3.6. Hint. e2x = (ex)2

2.7.3.7. Hint. ea+x = eaex

2.7.3.8. Hint. Figure out where the derivative is positive.

2.7.3.9. Hint. e−x = 1ex

2.7.3.10. Hint. Product rule will work nicely here. Alternately, review the resultof Question 2.7.3.6.

2.7.3.11. Hint. To find the sign of a product, compare the signs of each factor.The function et is always positive.

Exercises — Stage 32.7.3.12. Hint. After you differentiate, factor out ex.

2.7.3.13. Hint. Simplify.

2.7.3.14. *. Hint. In order to be differentiable, a function should be continuous.To determine the differentiability of the function at x = 1, use the definition ofthe derivative.

2.8 · Derivatives of Trigonometric Functions2.8.8 · Exercises

556

Page 573: CLP-1 Differential Calculus

HINTS FOR EXERCISES

Exercises — Stage 12.8.8.1. Hint. A horizontal tangent line is where the graph appears to “leveloff.”

2.8.8.2. Hint. You are going to mark there points on the sine graph where thegraph is the steepest, going up.

Exercises — Stage 22.8.8.3. Hint. You need to memorize the derivatives of sine, cosine, and tangent.

2.8.8.4. Hint. There are infinitely many values. You need to describe them all.

2.8.8.5. Hint. Simplify first.

2.8.8.6. Hint. The identity won’t help you.

2.8.8.8. Hint. Quotient rule

2.8.8.11. Hint. Use an identity.

2.8.8.12. Hint. How can you move the negative signs to a location that you canmore easily deal with?

2.8.8.13. Hint. Apply the quotient rule.

2.8.8.14. *. Hint. The only spot to worry about is when x = 0. For f(x) tobe differentiable, it must be continuous, so first find the value of b that makes fcontinuous at x = 0. Then, find the value of a that makes the derivatives fromthe left and right of x = 0 equal to each other.

Exercises — Stage 32.8.8.16. *. Hint. This looks like a derivative that you know how to compute.

2.8.8.17. Hint. tan θ =sin θ

cos θ

2.8.8.18. *. Hint. In order for a derivative to exist, the function must be contin-uous, and the derivative from the left must equal the derivative from the right.

2.8.8.19. *. Hint. There are infinitely many places where it does not exist.

2.8.8.25. Hint. You can set up the derivative using the limit definition: f ′(0) =

limh→0

f(h)− f(0)

h. If the limit exists, it gives you f ′(0); if the limit does not exist,

you conclude f ′(0) does not exist.To evaluate the limit, recall that when we differentiated sine, we learned that forh near 0,

cosh ≤ sinh

h≤ 1

557

Page 574: CLP-1 Differential Calculus

HINTS FOR EXERCISES

2.8.8.26. *. Hint. Recall |x| =

x x ≥ 0

−x x < 0. To determine whether h(x) is

differentiable at x = 0, use the definition of the derivative.

2.8.8.27. *. Hint. To decide whether the function is differentiable, use the defi-nition of the derivative.

2.8.8.28. *. Hint. In this chapter, we learned limx→0

sinx

x= 1. If you divide the

numerator and denominator by x5, you can make use of this knowledge.

2.9 · One More Tool – the Chain Rule2.9.4 · Exercises

Exercises — Stage 12.9.4.1. Hint. For parts 2.9.4.1.a and 2.9.4.1.b, remember the definition of aderivative:

dK

dU= lim

h→0

K(U + h)−K(U)

h.

When h is positive, U + h is an increased urchin population; what is the sign ofK(U + h)−K(U)?For part 2.9.4.1.c, use the chain rule!

2.9.4.2. Hint. Remember that Leibniz notation suggests fractional cancellation.

Exercises — Stage 22.9.4.3. Hint. If g(x) = cos x and h(x) = 5x + 3, then f(x) = g(h(x)). So weapply the chain rule, with “outside” function cosx and “inside” function 5x+ 3.

2.9.4.4. Hint. You can expand this into a polynomial, but it’s easier to use thechain rule. If g(x) = x5, and h(x) = x2 + 2, then f(x) = g(h(x)).

2.9.4.5. Hint. You can expand this into a polynomial, but it’s easier to use thechain rule. If g(k) = k17, and h(k) = 4k4 + 2k2 + 1, then T (k) = g(h(k)).

2.9.4.6. Hint. If we define g(x) =√x and h(x) =

x2 + 1

x2 − 1, then f(x) = g(h(x)).

To differentiate the square root function:d

dx√x =

d

dx

x1/2

=

1

2x−1/2 =

1

2√x

.

2.9.4.7. Hint. You’ll need to use the chain rule twice.

2.9.4.8. *. Hint. Use the chain rule.

2.9.4.9. *. Hint. Use the chain rule.

2.9.4.10. *. Hint. Use the chain rule.

2.9.4.11. *. Hint. Use the chain rule.

2.9.4.12. *. Hint. Recall1

x2= x−2 and

√x2 − 1 = (x2 − 1)1/2.

558

Page 575: CLP-1 Differential Calculus

HINTS FOR EXERCISES

2.9.4.14. Hint. If we let g(x) = sec x and h(x) = e2x+7, then f(x) = g(h(x)), so bythe chain rule, f ′(x) = g′(h(x)) · h′(x). However, in order to evaluate h′(x), we’llneed to use the chain rule again.

2.9.4.15. Hint. What trig identity can you use to simplify the first factor in theequation?

2.9.4.16. Hint. Velocity is the derivative of position with respect to time. In thiscase, the velocity of the particle is given by s′(t).

2.9.4.17. Hint. The slope of the tangent line is the derivative. You’ll need to usethe chain rule twice.

2.9.4.18. *. Hint. Start with the product rule, then use the chain rule to differ-entiate e4x.

2.9.4.19. *. Hint. Start with the quotient rule; you’ll need the chain rule only todifferentiate e3x.

2.9.4.20. *. Hint. More than one chain rule needed here.

2.9.4.21. *. Hint. More than one chain rule application is needed here.

2.9.4.22. *. Hint. More than one chain rule application is needed here.

2.9.4.23. *. Hint. More than one chain rule application is needed here.

2.9.4.24. *. Hint. What rule do you need, besides chain? Also, remember thatcos2 x = [cosx]2.

2.9.4.27. *. Hint. The product of two functions is zero exactly when at least oneof the functions is zero.

2.9.4.28. Hint. If t ≥ 1, then 0 < 1t≤ 1.

2.9.4.29. Hint. The notation cos3(5x − 7) means [cos(5x− 7)]3. So, if g(x) = x3

and h(x) = cos(5x− 7), then g(h(x)) = [cos(5x+ 7)]3 = cos3(5x+ 7).

2.9.4.30. *. Hint. In Example 2.6.6, we generalized the product rule to threefactors:

d

dxf(x)g(x)h(x) = f ′(x)g(x)h(x) + f(x)g′(x)h(x)

+ f(x)g(x)h′(x)

This isn’t strictly necessary, but it will simplify your computations.

Exercises — Stage 32.9.4.31. Hint. At time t, the particle is at the point

(x(t), y(t)

), with x(t) = cos t

and y(t) = sin t. Over time, the particle traces out a curve; let’s call that curvey = f(x). Then y(t) = f

(x(t)

), so the slope of the curve at the point

(x(t), y(t)

)is

f ′(x(t)

). You are to determine the values of t for which f ′

(x(t)

)= −1.

559

Page 576: CLP-1 Differential Calculus

HINTS FOR EXERCISES

2.9.4.32. *. Hint. Set f(x) = ex+x2 and g(x) = 1 +x. Compare f(0) and g(0), andcompare f ′(x) and g′(x).

2.9.4.33. Hint. If sin 2x and 2 sinx cosx are the same, then they also have thesame derivatives.

2.9.4.34. Hint. This is a long, nasty problem, but it doesn’t use anything youhaven’t seen before. Be methodical, and break the question into as many parts asyou have to. At the end, be proud of yourself for your problem-solving abilitiesand tenaciousness!

2.9.4.35. Hint. To sketch the curve, you can start by plotting points. Alternately,consider x2 + y.

2.10 · The Natural Logarithm2.10.3 · Exercises

Exercises — Stage 12.10.3.1. Hint. Each speaker produces 3dB of noise, so if P is the power of onespeaker, 3 = V (P ) = 10 log10

(PS

). Use this to find V (10P ) and V (100P ).

2.10.3.2. Hint. The question asks you when A(t) = 2000. So, solve 2000 =1000et/20 for t.

2.10.3.3. Hint. What happens when cosx is a negative number?

Exercises — Stage 22.10.3.4. Hint. There are two easy ways: use the chain rule, or simplify first.

2.10.3.5. Hint. There are two easy ways: use the chain rule, or simplify first.

2.10.3.6. Hint. Don’t be fooled by a common mistake: log(x2 +x) is not the sameas log(x2) + log x.

2.10.3.7. Hint. Use the base-change formula to convert this to natural logarithm(base e).

2.10.3.9. Hint. Use the chain rule.

2.10.3.10. Hint. Use the chain rule twice.

2.10.3.11. *. Hint. You’ll need to use the chain rule twice.

2.10.3.12. *. Hint. Use the chain rule.

2.10.3.13. *. Hint. Use the chain rule to differentiate.

2.10.3.14. *. Hint. You can differentiate this by using the chain rule severaltimes.

2.10.3.15. *. Hint. Using logarithm rules before you differentiate will make thiseasier.

560

Page 577: CLP-1 Differential Calculus

HINTS FOR EXERCISES

2.10.3.16. Hint. Using logarithm rules before you differentiate will make thiseasier.

2.10.3.17. Hint. First, differentiate using the chain rule and any other necessaryrules. Then, plug in x = 2.

2.10.3.18. *. Hint. In the text, you are given the derivatived

dxax, where a is a

constant.

2.10.3.19. Hint. You’ll need to use logarithmic differentiation. Set g(x) =log(f(x)), and find g′(x). Then, use that to find f ′(x). This is the method used

in the text to findd

dxax.

2.10.3.20. *. Hint. Use Question 2.10.3.19 and the base-change formula,

logb(a) =log a

log b.

2.10.3.21. Hint. To make this easier, use logarithmic differentiation. Set g(x) =log(f(x)), and find g′(x). Then, use that to find f ′(x). This is the method used in

the text to findd

dxax, and again in Question 2.10.3.19.

2.10.3.22. Hint. To make this easier, use logarithmic differentiation. Set g(x) =log(f(x)), and find g′(x). Then, use that to find f ′(x). This is the method used in

the text to findd

dxax, and again in Question 2.10.3.19.

2.10.3.23. Hint. It’s not going to come out nicely, but there’s a better way thanblindly applying quotient and product rules, or expanding giant polynomials.

2.10.3.24. *. Hint. You’ll need to use logarithmic differentiation. Set g(x) =log(f(x)), and find g′(x). Then, use that to find f ′(x). This is the method used in

the text to findd

dxax, and again in Question 2.10.3.19.

2.10.3.25. *. Hint. You’ll need to use logarithmic differentiation. Set g(x) =log(f(x)), and find g′(x). Then, use that to find f ′(x). This is the method used in

the text to findd

dxax, and again in Question 2.10.3.19.

2.10.3.26. *. Hint. You’ll need to use logarithmic differentiation. Set g(x) =log(f(x)), and find g′(x). Then, use that to find f ′(x). This is the method used in

the text to findd

dxax, and again in Question 2.10.3.19.

2.10.3.27. *. Hint. You’ll need to use logarithmic differentiation. Differentiate

log(f(x)), then solve for f ′(x). This is the method used in the text to findd

dxax.

2.10.3.28. *. Hint. You’ll need to use logarithmic differentiation. Differentiate

log(f(x)), then solve for f ′(x). This is the method used in the text to findd

dxax.

561

Page 578: CLP-1 Differential Calculus

HINTS FOR EXERCISES

2.10.3.29. *. Hint. You’ll need to use logarithmic differentiation. Differentiate

log(f(x)), then solve for f ′(x). This is the method used in the text to findd

dxax.

Exercises — Stage 3

2.10.3.30. Hint. Evaluated

dx

log(

[f(x)]g(x))

.

2.10.3.31. Hint. Differentiate y = log(f(x)). When is the derivative equal tozero?

2.11 · Implicit Differentiation2.11.2 · Exercises

Exercises — Stage 12.11.2.1. Hint. Where did the y′ come from?

2.11.2.2. Hint. The three points to look at are (0,−4), (0, 0), and (0, 4). Whatdoes the slope of the tangent line look like there?

2.11.2.3. Hint. A function must pass the vertical line test: one input cannot resultin two different outputs.

Exercises — Stage 22.11.2.4. *. Hint. Remember that y is a function of x. Use implicit differentia-

tion, then collect all the terms containingdy

dxon one side of the equation to solve

fordy

dx.

2.11.2.5. *. Hint. Differentiate implicitly, then solve for y′.

2.11.2.6. *. Hint. Remember that y is a function of x. You can determine explic-itly the values of x for which y(x) = 1.

2.11.2.8. *. Hint. Plug in y = 0 at a strategic point in your work to simplify yourcomputation.

2.11.2.10. *. Hint. Plug in y = 0 at a strategic point in your work to simplifyyour computation.

2.11.2.11. Hint. If the tangent line has slope y′, and it is parallel to y = x, theny′ = 1.

2.11.2.12. *. Hint. You don’t need to solve for y′ in general: only at a singlepoint.

2.11.2.13. *. Hint. After you differentiate implicitly, get all the terms containingy′ onto one side so you can solve for y′.

562

Page 579: CLP-1 Differential Calculus

HINTS FOR EXERCISES

Exercises — Stage 32.11.2.14. *. Hint. You don’t need to solve for dy

dxfor all values of x–only when

y = 0.

2.12 · Inverse Trigonometric Functions2.12.2 · Exercises

Exercises — Stage 12.12.2.1. Hint. Remember that only certain numbers can come out of sine andcosine, but any numbers can go in.

2.12.2.2. Hint. What is the range of the arccosine function?

2.12.2.3. Hint. A one-to-one function passes the horizontal line test. To graphthe inverse of a function, reflect it across the line y = x.

2.12.2.4. Hint. Your answer will depend on a. The arcsine function alone won’tgive you every value.

2.12.2.5. Hint. In order for x to be in the domain of f , you must be able to plugx into both arcsine and arccosecant.

Exercises — Stage 22.12.2.6. Hint. For the domain of f , remember the domain of arcsine is [−1, 1].

2.12.2.7. Hint. The domain of arccos(t) is [−1, 1], but you also have to make sureyou aren’t dividing by zero.

2.12.2.8. Hint.d

dxarcsecx =

1

|x|√x2 − 1

, and the domain of arcsecx is |x| ≥ 1.

2.12.2.9. Hint. The domain of arctan(x) is all real numbers.

2.12.2.10. Hint. The domain of arcsinx is [−1, 1], and the domain of√x is x ≥ 0.

2.12.2.11. Hint. This occurs only once.

2.12.2.12. Hint. The answer is a very simple expression.

2.12.2.13. *. Hint. chain rule

2.12.2.16. Hint. You can simplify the expression before you differentiate to re-move the trigonometric functions. If arctanx = θ, then fill in the sides of thetriangle below using the definition of arctangent and the Pythagorean theorem:

563

Page 580: CLP-1 Differential Calculus

HINTS FOR EXERCISES

θ

With the sides labeled, you can figure out sin (arctanx) = sin (θ).

2.12.2.17. Hint. You can simplify the expression before you differentiate to re-move the trigonometric functions. If arcsinx = θ, then fill in the sides of thetriangle below using the definition of arctangent and the Pythagorean theorem:

θ

With the sides labeled, you can figure out cot (arcsinx) = cot (θ).

2.12.2.18. *. Hint. What is the slope of the line y = 2x+ 9?

2.12.2.19. Hint. Differentiate using the chain rule.

Exercises — Stage 32.12.2.20. *. Hint. If g(y) = f−1(y), then f(g(y)) = f (f−1(y)) = y. Differentiatethis last equality using the chain rule.

2.12.2.21. *. Hint. To simplify notation, let g(y) = f−1(y). Simplify and differ-entiate g(f(x)).

2.12.2.22. *. Hint. To simplify notation, let g(y) = f−1(y). Simplify and differ-entiate g(f(x)).

2.12.2.23. Hint. Use logarithmic differentiation.

2.12.2.24. Hint. Where are those functions defined?

2.12.2.25. Hint. Compare this to one of the forms given in the text for the defi-nition of the derivative.

2.12.2.26. Hint. f−1(7) is the number y that satisfies f(y) = 7.

2.12.2.27. Hint. If f−1(y) = 0, that means f(0) = y. So, we’re looking for thenumber that we plug into f−1 to get 0.

564

Page 581: CLP-1 Differential Calculus

HINTS FOR EXERCISES

2.12.2.28. Hint. As usual, after you differentiate implicitly, get all the termscontaining y′ onto one side of the equation, so you can factor out y′.

2.13 · The Mean Value Theorem2.13.5 · Exercises

Exercises — Stage 12.13.5.1. Hint. How long would it take the caribou to travel 5000 km, travellingat its top speed?

2.13.5.2. Hint. Let f(x) be the position of the crane, where x is the hour of theday.

2.13.5.3. Hint. For an example, look at Figure 2.13.4.

2.13.5.4. Hint. How does this question differ from the statement of the meanvalue theorem?

2.13.5.6. Hint. Where is f(x) differentiable?

Exercises — Stage 22.13.5.7. *. Hint. To use Rolle’s Theorem, you will want two values where thefunction is zero. If you’re stuck finding one of them, think about when x2 − 2πxis equal to zero.

2.13.5.11. Hint. To show that there are exactly n roots, you need to not onlyshow that n exist, but also that there are not more than n.

2.13.5.12. Hint. To show that there are exactly n roots, you need to not onlyshow that n exist, but also that there are not more than n. If you can’t explicitlyfind the root(s), you can use the intermediate value theorem to show they exist.

2.13.5.13. Hint. If f(x) = 0, then |x3| = |sin (x5)| ≤ 1. When |x| < 1, is cos(x5)positive or negative?

2.13.5.14. Hint. Let f(x) = ex − 4 cos(2x), and use Rolle’s Theorem. What is theinterval where f(x) can have a positive root?

2.13.5.15. *. Hint. For 2.13.5.15.b, what does Rolle’s Theorem tell you has tohappen in order for f(x) to have more than one root in [−1, 1]?

2.13.5.16. *. Hint. Since f(x) = ex is a continuous and differentiable function,the MVT promises that there exists some number c such that

f ′(c) =f(T )− f(0)

T.

Find that c, in terms of T .

2.13.5.17. Hint. Let f(x) = arcsec x+ arccsc c− C. What is f ′(x)?

565

Page 582: CLP-1 Differential Calculus

HINTS FOR EXERCISES

Exercises — Stage 32.13.5.18. *. Hint. Show that f is differentiable by showing that f ′(x) exists forevery x. Then, the Mean Value Theorem applies. What is the largest f ′(x) can be,for any x? If f(100) < 100, what does the MVT tell you must be true of f ′(c) forsome c?

2.13.5.19. Hint. In order for f−1(x) to be defined over an interval, f(x) must beone–to–one over that interval.

2.13.5.20. Hint. In order for f−1(x) to be defined over an interval, f(x) must beone–to–one over that interval.

2.13.5.21. Hint. Let h(x) = f(x)−g(x). What does the Mean Value Theorem tellyou about the derivative of h?

2.13.5.22. Hint. Rolle’s Theorem relates the roots of a function to the roots of itsderivative.

2.13.5.23. Hint. To show that there are exactly n distinct roots, you need to notonly show that n exist, but also that there are not more than n.

2.14 · Higher Order Derivatives2.14.2 · Exercises

Exercises — Stage 12.14.2.1. Hint. If you know the first derivative, this should be easy.

2.14.2.2. Hint. Exactly two of the statements must be true.

2.14.2.3. Hint. Use factorials, as in Example 2.14.2.

2.14.2.4. Hint. The problem isn’t with any of the algebra.

Exercises — Stage 2

2.14.2.5. Hint. Recalld

dxlog x =

1

x.

2.14.2.6. Hint. Recalld

dxarctanx =

1

1 + x2= (1 + x2)−1.

2.14.2.7. Hint. Use implicit differentiation.

2.14.2.8. Hint. The acceleration is given by s′′(t).

2.14.2.9. Hint. Remember to use the chain rule.

2.14.2.10. Hint. h′(t) gives the velocity of the particle, and h′′(t) gives itsacceleration–the rate the velocity is changing.

2.14.2.11. Hint. h′(t) gives the velocity of the particle, and h′′(t) gives itsacceleration–the rate the velocity is changing. Be wary of signs–as in legends,they may be misleading.

566

Page 583: CLP-1 Differential Calculus

HINTS FOR EXERCISES

2.14.2.12. Hint. You don’t need to solve for y′′ in general–only when x = y = 0.To do this, you also need to find y′ at the point (0, 0).

2.14.2.13. Hint. To show that two functions are unequal, you can show that oneinput results in different outputs.

Exercises — Stage 32.14.2.14. Hint. Only one of the curves could possibly represent y = f(x).

2.14.2.15. Hint. Rememberd

dx2x = 2x log 2.

2.14.2.16. Hint. Differentiate a few times until you get zero, remembering thata, b, c, and d are all constants.

2.14.2.17. *. Hint. Use a similar method to Question 2.9.4.32, Section 2.9.

2.14.2.18. *. Hint. For 2.14.2.18.b, you know a point where the curve and tan-gent line intersect, and you know what the tangent line looks like. What do thederivatives tell you about the shape of the curve?

2.14.2.19. Hint. Review Pascal’s Triangle.

2.14.2.20. Hint. Rolle’s Theorem relates the roots of a function to the roots of itsderivative. So, the fifth derivative tells us something about the fourth, the fourthderivative tells us something about the third, and so on.

2.14.2.21. Hint. You’ll want to use Rolle’s Theorem, but the first derivativewon’t be very tractable–use the idea behind Question 2.14.2.20.

2.14.2.22. *. Hint. You can re-write this function as a piecewise function, withbranches x ≥ 0 and x < 0. To figure out the derivatives at x = 0, use the definitionof a derivative.

3 · Applications of derivatives3.1 · Velocity and Acceleration3.1.2 · Exercises

Exercises — Stage 13.1.2.1. Hint. Is the velocity changing at t = 2?

3.1.2.2. Hint. The acceleration (rate of change of velocity) is constant.

3.1.2.3. Hint. Remember the difference between speed and velocity.

3.1.2.4. Hint. How is this different from the wording of Question 3.1.2.3?

Exercises — Stage 2

567

Page 584: CLP-1 Differential Calculus

HINTS FOR EXERCISES

3.1.2.5. Hint. The equation of an object falling from rest on the earth is derivedin Example 3.1.2. It would be difficult to use exactly the version given for s(t),but using the same logic, you can find an equation for the height of the flowerpot at time t.

3.1.2.6. Hint. Remember that a falling object has an acceleration of 9.8 ms2

.

3.1.2.7. Hint. Acceleration is constant, so finding a formula for the distanceyour keys have travelled is a similar problem to finding a formula for somethingfalling.

3.1.2.8. Hint. See Example 3.1.3.

3.1.2.9. Hint. Be careful to match up the units.

3.1.2.10. Hint. Think about what it means for the car to decelerate at a constantrate. You might also review Question 3.1.2.2.

3.1.2.11. Hint. Let a be the acceleration of the shuttle. Start by finding a, thenfind the position function of the shuttle.

3.1.2.12. Hint. Review Example 3.1.2, but account for the fact that your initialvelocity is not zero.

3.1.2.13. Hint. Be very careful with units. The acceleration of gravity you’reused to is 9.8 metres per second squared, so you might want to convert 325 kpmto metres per second.

3.1.2.14. Hint. Since gravity alone brings it down, its acceleration is a constant−9.8 m

s2.

3.1.2.15. Hint. First, find an equation for a(t), the acceleration of the car, notingthat a′(t) is constant. Then, use this to find an equation for the velocity of the car.Be careful about seconds versus hours.

Exercises — Stage 33.1.2.16. Hint. We recommend using two different functions to describe yourheight: h1(t) while you are in the air, not yet touching the trampoline, and h2(t)while you are in the trampoline, going down.Both h1(t) and h2(t) are quadratic equations, since your acceleration is constantover both intervals, but be very careful about signs.

3.1.2.17. Hint. First, find an expression for the speed of the object. You can letv0 be its velocity at time t = 0.

3.2 · Related Rates3.2.2 · Exercises

568

Page 585: CLP-1 Differential Calculus

HINTS FOR EXERCISES

Exercises — Stage 13.2.2.1. Hint. If you know P , you can figure out Q.

Exercises — Stage 23.2.2.2. *. Hint. Since the point moves along the unit circle, we know that x2 +y2 = 1, where x and y are functions of time.

3.2.2.3. *. Hint. You’ll need some implicit differentiation: what should yourvariable be? Example 3.2.3 shows how to work with percentage rate of change.

3.2.2.4. *. Hint. For 3.2.2.4.b, refer to Example 3.2.3 for percentage rate ofchange.

3.2.2.5. *. Hint. Pay attention to direction, and what it means for the sign(plus/minus) of the velocities of the particles.

3.2.2.6. *. Hint. You’ll want to think about the difference in the y-coordinates ofthe two particles.

3.2.2.7. *. Hint. Draw a picture, and be careful about signs.

3.2.2.8. *. Hint. You’ll want to think about the difference in height of the twosnails.

3.2.2.9. *. Hint. The length of the ladder is changing.

3.2.2.10. Hint. If a trapezoid has height h and (parallel) bases b1 and b2, then itsarea is h

(b1+b2

2

). To figure out how wide the top of the water is when the water is

at height h, you can cut the trapezoid up into a rectangle and two triangles, andmake use of similar triangles.

3.2.2.11. Hint. Be careful with units. One litre is 1000 cm3, which is not thesame as 10 m3.

3.2.2.12. Hint. You, the rocket, and the rocket’s original position form a righttriangle.

3.2.2.13. *. Hint. Your picture should be a triangle.

3.2.2.14. Hint. Let θ be the angle between the two hands. Using the Law of

Cosines, you can get an expression for D in terms of θ. To finddθ

dt, use what you

know about how fast clock hands move.

3.2.2.15. *. Hint. The area in the annulus is the area of the outer circle minusthe area of the inner circle.

3.2.2.16. Hint. The volume of a sphere with radius R is4

3πr3.

3.2.2.17. Hint. The area of a triangle is half its base times its height. To find thebase, split the triangle into two right triangles.

569

Page 586: CLP-1 Differential Calculus

HINTS FOR EXERCISES

3.2.2.18. Hint. The easiest way to figure out the area of the sector of an annulus(or a circle) is to figure out the area of the entire annulus, then multiply by whatproportion of the entire annulus the sector is. For example, if your sector is 1

10

of the entire annulus, then its area is 110

of the area of the entire annulus. (SeeSection A.4 to see how this works out for circles.)

3.2.2.19. Hint. Think about the ways in which this problem is similar to anddifferent from Example 3.2.6 and Question 3.2.2.18.

3.2.2.20. Hint. The volume of a cone with height h and radius r is 13πr2h. Also,

one millilitre is the same as one cubic centimetre.

Exercises — Stage 33.2.2.21. Hint. If you were to install the buoy, how would you choose the lengthof rope? For which values of θ do sin θ and cos θ have different signs? How wouldthose values of θ look on the diagram?

3.2.2.22. Hint. At both points of interest, the point is moving along a straightline. From the diagram, you can figure out the equation of that line.For the question “How fast is the point moving?” in part (b), remember that thevelocity of an object can be found by differentiating (with respect to time) theequation that gives the position of the object. The complicating factors in thiscase are that (1) the position of our object is not given as a function of time, and(2) the position of our object is given in two dimensions, not one.

3.2.2.23. Hint. (a) Since the perimeter of the cross section of the bottle does notchange, p (the perimeter of the ellipse) is the same as the perimeter of the circleof radius 5.(b) The volume of the bottle will be the area of its cross section times its height.This is always the case when you have some two-dimensional shape, and turnit into a three-dimensional object by “pulling” the shape straight up. (For exam-ple, you can think of a cylinder as a circle that has been “pulled” straight up.To understand why this formula works, think about what is means to measurethe area of a shape in square centimetres, and the volume of an object in cubiccentimetres.)(c) You can use what you know about a and the formula from (a) to find b anddb

dt. Then use the formula from (b).

3.2.2.24. Hint. IfA = 0, you can figure out C andD from the relationship given.

3.3 · Exponential Growth and Decay — a First Look atDifferential Equations3.3.4 · Exercises· Exercises for § 3.3.1

570

Page 587: CLP-1 Differential Calculus

HINTS FOR EXERCISES

Exercises — Stage 13.3.4.1. Hint. Review the definition of a differential equation at the beginningof this section.

3.3.4.2. Hint. You can test whether a given function solves a differential equa-tion by substituting the function into the equation.

3.3.4.3. Hint. Solve 0 = Ce−kt for t.

Exercises — Stage 23.3.4.4. *. Hint. No calculus here–just a review of the algebra of exponentials.

3.3.4.5. *. Hint. Use Theorem 3.3.2.

3.3.4.6. Hint. From the text, we see the half-life of Carbon-14 is 5730 years. Amicrogram (µg) is one-millionth of a gram, but you don’t need to know that tosolve this problem.

3.3.4.7. Hint. The quantity of Radium-226 in the sample at time t will be Q(t) =Ce−kt for some positive constants C and k. You can use the given information tofind C and e−k.In the following work, remember we use log to mean natural logarithm, loge.

3.3.4.8. *. Hint. The fact that the mass of the sample decreases at a rate propor-tional to its mass tells us that, if Q(t) is the mass of Polonium-201, the followingdifferential equation holds:

dQ

dt= −kQ(t)

where k is some positive constant. Compare this to Theorem 3.3.2.

3.3.4.9. Hint. The amount of Radium-221 in a sample at time t will be Q(t) =Ce−kt for some positive constants C and k. You can leave C as a variable–it’s theoriginal amount in the sample, which isn’t specified. What you want to find isthe value of t such that Q(t) = 0.0001Q(0) = 0.0001C.

Exercises — Stage 33.3.4.10. Hint. You don’t need to know the original amount of Polonium-210 inorder to answer this question: you can leave it as some constant C, or you cancall it 100%.

3.3.4.11. Hint. Try to find the most possible and least possible remainingUranium-232, given the bounds in the problem.

· Exercises for § 3.3.2

Exercises — Stage 13.3.4.1. Hint. You can refer to Corollary 3.3.8, but you can also just differentiate

571

Page 588: CLP-1 Differential Calculus

HINTS FOR EXERCISES

the various proposed functions and see whether, in fact,dT

dtis the same as 5[T −

20].

3.3.4.2. Hint. From Newton’s Law of Cooling and Corollary 3.3.8, the tempera-ture of the object will be

T (t) = [T (0)− A]eKt + A

where A is the ambient temperature, T (0) is the initial temperature of the copper,and K is some constant.

3.3.4.3. Hint. What is limt→∞

eKt when K is positive, negative, or zero?

3.3.4.4. Hint. Solve A = [T (0)− A]ekt for t.

Exercises — Stage 23.3.4.5. Hint. From Newton’s Law of Cooling and Corollary 3.3.8, we know thetemperature of the copper will be

T (t) = [T (0)− A]eKt + A

where A is the ambient temperature, T (0) is the initial temperature of the copper,and K is some constant. Use the given information to find an expression for T (t)not involving any unknown constants.

3.3.4.6. Hint. From Newton’s Law of Cooling and Corollary 3.3.8, we know thetemperature of the stone t minutes after it leaves the fire is

T (t) = [T (0)− A]eKt + A

where A is the ambient temperature, T (0) is the temperature of the stone theinstant it left the fire, and K is some constant.

Exercises — Stage 33.3.4.8. *. Hint. Newton’s Law of Cooling models the temperature of the teaafter t minutes as

T (t) = [T (0)− A]eKt + A

where A is the ambient temperature, T (0) is the initial temperature of the tea,and K is some constant.

3.3.4.9. Hint. What is limt→∞

T (t)?

· Exercises for § 3.3.3

572

Page 589: CLP-1 Differential Calculus

HINTS FOR EXERCISES

Exercises — Stage 13.3.4.1. Hint. P (0) is also (probably) a positive constant.

Exercises — Stage 23.3.4.2. Hint. The assumption that the animals grow according to the Malthu-sian model tells us that their population t years after 2015 is given by P (t) =121ebt for some constant b.

3.3.4.3. Hint. The Malthusian model says that the population of bacteria t hoursafter being placed in the dish will be P (t) = 1000ebt for some constant b.

3.3.4.4. Hint. If 1928 is a years after the shipwreck, you might want to make useof the fact that eb(a+1) = ebaeb.

3.3.4.5. Hint. If the population has a net birthrate per individual per unit timeof b, then the Malthusian model predicts that the number of individuals at time twill be P (t) = P (0)ebt. You can use the test population to find eb.

Exercises — Stage 33.3.4.6. Hint. One way to investigate the sign of k is to think about f ′(t): is itpositive or negative?

· Further problems for § 3.33.3.4.1. *. Hint. Use Theorem 3.3.2 to figure out what f(x) looks like.

3.3.4.2. Hint. To use Corollary 3.3.8, you need to re-write the differential equationas

dT

dt= 7

[T −

(−9

7

)].

3.3.4.3. *. Hint. The amount of the material at time t will be Q(t) = Ce−kt for someconstants C and k.

3.3.4.4. Hint. In your calculations, it might come in handy that e30K =(e15K

)2.

3.3.4.5. *. Hint. The differential equation in the problem has the same form as thedifferential equation from Newton’s Law of Cooling.

3.3.4.6. *. Hint. We know the form of the solution A(t) from Corollary 3.3.8.

3.3.4.7. *. Hint. If a function’s rate of change is proportional to the function itself,what does the function looks like?

3.3.4.8. *. Hint. The equation from Newton’s Law of Cooling, in Corollary 3.3.8,has a similar form to the differential equation in this question.

3.4 · Approximating Functions Near a Specified Point —Taylor Polynomials

573

Page 590: CLP-1 Differential Calculus

HINTS FOR EXERCISES

3.4.11 · Exercises· Exercises for § 3.4.1

Exercises — Stage 13.4.11.1. Hint. An approximation should be something you can actually figureout–otherwise it’s no use.

Exercises — Stage 23.4.11.2. Hint. You’ll need some constant a to approximation log(0.93) ≈ log(a).This a should have two properties: it should be close to 0.93, and you should beable to easily evaluate log(a).

3.4.11.3. Hint. You’ll need some constant a to approximate arcsin(0.1) ≈arcsin(a). This a should have two properties: it should be close to 0.1, and youshould be able to easily evaluate arcsin(a).

3.4.11.4. Hint. You’ll need some constant a to approximate√

3 tan(1) ≈√3 tan(a). This a should have two properties: it should be close to 1, and you

should be able to easily evaluate√

3 tan(a).

Exercises — Stage 33.4.11.5. Hint. We could figure out 10.13 exactly, if we wanted, with pen andpaper. Since we’re asking for an approximation, we aren’t after perfect accuracy.Rather, we’re after ease of calculation.

· Exercises for § 3.4.2

Exercises — Stage 13.4.11.1. Hint. The linear approximation L(x) is chosen so that f(5) = L(5) andf ′(5) = L′(5).

3.4.11.2. Hint. The graph of the linear approximation is a line, passing through(2, f(2)), with slope f ′(2).

3.4.11.3. Hint. It’s an extremely accurate approximation.

Exercises — Stage 23.4.11.4. Hint. You’ll need to centre your approximation about some x = a,which should have two properties: you can easily compute log(a), and a is closeto 0.93.

3.4.11.5. Hint. Approximate the function f(x) =√x.

3.4.11.6. Hint. Approximate the function f(x) = 5√x.

574

Page 591: CLP-1 Differential Calculus

HINTS FOR EXERCISES

Exercises — Stage 33.4.11.7. Hint. Approximate the function f(x) = x3.

3.4.11.8. Hint. One possible choice of f(x) is f(x) = sinx.

3.4.11.9. Hint. Compare the derivatives.

· Exercises for § 3.4.3

Exercises — Stage 13.4.11.1. Hint. If Q(x) is the quadratic approximation of f about 3, then Q(3) =f(3), Q′(3) = f ′(3), and Q′′(3) = f ′′(3).

3.4.11.2. Hint. It is a very good approximation.

Exercises — Stage 23.4.11.3. Hint. Approximate f(x) = log x.

3.4.11.4. Hint. You’ll probably want to centre your approximation about x = 0.

3.4.11.5. Hint. The quadratic approximation of a function f(x) about x = a is

f(x) ≈ f(a) + f ′(a)(x− a) +1

2f ′′(a)(x− a)2

3.4.11.6. Hint. One way to go about this is to approximate the function f(x) =5 · x1/3 , because then 54/3 = 5 · 51/3 = f(5).

3.4.11.7. Hint. For 3.4.11.7.c, look for cancellations.

3.4.11.8. Hint. Compare (c) to (b).Compare (e) and (f) to (d).To get an alternating sign, consider powers of (−1).

Exercises — Stage 3

3.4.11.9. Hint. You can evaluate f(1) exactly. Recalld

dxarcsinx =

1√1− x2

.

3.4.11.10. Hint. Let f(x) = ex, and use the quadratic approximation of f(x)about x = 0 (given in your text, or you can reproduce it) to approximate f(1).

3.4.11.11. Hint. Be wary of indices: for example3∑

n=1

n =7∑

n=5

(n− 4).

· Exercises for § 3.4.4

Exercises — Stage 1

575

Page 592: CLP-1 Differential Calculus

HINTS FOR EXERCISES

3.4.11.1. Hint. T ′′3 (x) and f ′′(x) agree when x = 1.

3.4.11.2. Hint. The nth degree Taylor polynomial for f(x) about x = 5 is

Tn(x) =n∑k=0

f (k)(5)

k!(x− 5)k

Match up the terms.

Exercises — Stage 33.4.11.3. Hint. The fourth-degree Maclaurin polynomial for f(x) is

T4(x) = f(0) + f ′(0)x+1

2f ′′(0)x2 +

1

3!f ′′′(0)x3 +

1

4!f (4)(0)x4

while the third-degree Maclaurin polynomial for f(x) is

T3(x) = f(0) + f ′(0)x+1

2f ′′(0)x2 +

1

3!f ′′′(0)x3

3.4.11.4. Hint. The third-degree Maclaurin polynomial for f(x) about x = 1 is

T3(x) = f(1) + f ′(1)(x− 1) +1

2f ′′(1)(x− 1)2 +

1

3!f ′′′(1)(x− 1)3

How can you recover f(1), f ′(1), f ′′(1), and f ′′′(1) from T4(x)?

3.4.11.5. Hint. Compare the given polynomial to the more standard form of thenth degree Taylor polynomial,

n∑k=0

1

k!f (k)(5)(x− 5)k

and notice that the term you want (containing f (10)(5)) corresponds to k = 10 inthe standard form, but is not the term corresponding to k = 10 in the polynomialgiven in the question.

3.4.11.6. Hint. T ′′′3 (a) = f ′′′(a)

· Exercises for § 3.4.5

Exercises — Stage 13.4.11.1. Hint. The derivatives of f(x) repeat themselves.

3.4.11.2. Hint. You are approximating a polynomial with a polynomial.

3.4.11.3. Hint. Recalld

dx2x = 2x log 2, where log 2 is the constant loge 2.

576

Page 593: CLP-1 Differential Calculus

HINTS FOR EXERCISES

3.4.11.4. Hint. Just keep differentiating–it gets easier!

3.4.11.5. Hint. Start by differentiating, and finding the pattern for f (k)(0). Re-member the chain rule!

Exercises — Stage 33.4.11.6. Hint. You’ll need to differentiate xx. This is accomplished using loga-rithmic differentiation, covered in Section 2.10.

3.4.11.7. Hint. What is 6 arctan

(1√3

)?

3.4.11.8. Hint. After a few derivatives, this will be very similar to Exam-ple 3.4.13.

3.4.11.9. Hint. You’ll have to deal with “doubles” of coefficients in the Taylorpolynomial. The following is a way of getting them into sigma notation:

1 + 1− 1− 1 + 1 + 1− 1− 1

=(1 + 1) + (−1− 1) + (1 + 1) + (−1− 1)

=3∑

k=0

[(−1)k + (−1)k

]3.4.11.10. Hint. Compare this to the Maclaurin polynomial for ex.

3.4.11.11. Hint. Compare this to the Maclaurin polynomial for cosine.

· Exercises for § 3.4.6

Exercises — Stage 13.4.11.1. Hint. ∆x and ∆y represent changes in x and y, respectively, while f(x)and f (x+ ∆x) are the y-values the function takes.

3.4.11.2. Hint. Let f(x) be the number of problems finished after x minutes ofwork.

Exercises — Stage 23.4.11.3. Hint. ∆y = f(5.1)− f(5)

3.4.11.4. Hint. Use the approximation ∆y ≈ s′(4)∆x when x is near 4.

· Exercises for § 3.4.7

Exercises — Stage 13.4.11.1. Hint. Is the linear approximation exact, or approximate?

577

Page 594: CLP-1 Differential Calculus

HINTS FOR EXERCISES

3.4.11.2. Hint. When an exact value Q0 is measured as Q0 + ∆Q, Defini-tion 3.4.25gives us the absolute error as |∆Q|, and the percentage error as 100

|∆Q|Q0

.

3.4.11.3. Hint. Let ∆y is the change in f(x) associated to a change in x from ato a+ ∆x. The linear approximation tells us

∆y ≈ f ′(a)∆x

while the quadratic approximation tells us

∆y ≈ f ′(a)∆x+1

2f ′′(a) (∆x)2

Exercises — Stage 23.4.11.4. Hint. The exact area desired is A0. Let the corresponding exact radiusdesired be r0. The linear approximation tells us ∆A ≈ A′(r0)∆r. Use this rela-tionship, and what you know about the error allowable in A, to find the errorallowable in r.

3.4.11.5. Hint. For part (b), cut the triangle (with angle θ and side d) into tworight triangles.

3.4.11.6. Hint. The volume of a cone of height h and radius r is 13πr2h.

Exercises — Stage 33.4.11.7. Hint. Remember that the amount of the isotope present at time t isQ(t) = Q(0)e−kt for some constant k. The measured quantity after 3 years willallow you to replace k in the equation, then solving Q(t) = 1

2Q(0) for t will give

you the half-life of the isotope.

· Exercises for § 3.4.8

Exercises — Stage 13.4.11.1. Hint. R(10) = f(10)− F (10) = −3− 5

3.4.11.2. Hint. Equation 3.4.33 tells us

|f(2)− T3(2)| =∣∣∣∣f (4)(c)

4!(2− 0)4

∣∣∣∣for some c strictly between 0 and 2.

3.4.11.3. Hint. You are approximating a third-degree polynomial with a fifth-degree Taylor polynomial. You should be able to tell how good your approxima-tion will be without a long calculation.

578

Page 595: CLP-1 Differential Calculus

HINTS FOR EXERCISES

3.4.11.4. Hint. Draw a picture — it should be clear how the two approximationsbehave.

Exercises — Stage 23.4.11.5. Hint. In this case, Equation 3.4.33 tells us that

|f(11.5)− T5(11.5)| =∣∣∣∣f (6)(c)

6!(11.5− 11)6

∣∣∣∣for some c strictly between 11 and 11.5.

3.4.11.6. Hint. In this case, Equation 3.4.33 tells us that |f(0.1)− T2(0.1)| =∣∣∣∣f (3)(c)

3!(0.1− 0)3

∣∣∣∣ for some c strictly between 0 and 0.1.

3.4.11.7. Hint. In our case, Equation 3.4.33 tells us∣∣∣∣f (−1

4

)− T5

(−1

4

)∣∣∣∣ =

∣∣∣∣∣f (6)(c)

6!

(−1

4− 0

)6∣∣∣∣∣

for some c between −1

4and 0.

3.4.11.8. Hint. In this case, Equation 3.4.33 tells us that |f(30)− T3(30)| =∣∣∣∣f (4)(c)

4!(30− 32)4

∣∣∣∣ for some c strictly between 30 and 32.

3.4.11.9. Hint. In our case, Equation 3.4.33 tells us |f (0.01)− T1 (0.01)| =∣∣∣∣f (2)(c)

2!

(0.01− 1

π

)2

∣∣∣∣ for some c between 0.01 and1

π.

3.4.11.10. Hint. Using Equation 3.4.33,∣∣∣∣f (1

2

)− T2

(1

2

)∣∣∣∣ =

∣∣∣∣∣f (3)(c)

3!

(1

2− 0

)3∣∣∣∣∣

for some c in(

0,1

2

).

Exercises — Stage 33.4.11.11. Hint. It helps to have a formula for f (n)(x). You can figure it out bytaking several derivatives and noticing the pattern, but also this has been givenpreviously in the text.

3.4.11.12. Hint. You can approximate the function f(x) = x17 .

It’s a good bit of trivia to know 37 = 2187.A low-degree Taylor approximation will give you a good enough estimation. Ifyou guess a degree, and take that Taylor polynomial, the error will probably beless than 0.001 (but you still need to check).

579

Page 596: CLP-1 Differential Calculus

HINTS FOR EXERCISES

3.4.11.13. Hint. Use the 6th-degree Maclaurin approximation for f(x) = sinx.

3.4.11.14. Hint. For part 3.4.11.14.c, after you plug in the appropriate valuesto Equation 3.4.33, simplify the upper and lower bounds for e separately. Inparticular, for the upper bound, you’ll have to solve for e.

· Further problems for § 3.4

Exercises — Stage 13.4.11.1. *. Hint. Compare the given polynomial to the definition of a Maclaurinpolynomial.

3.4.11.3. *. Hint. Compare the given polynomial to the definition of a Taylorpolynomial.

Exercises — Stage 23.4.11.5. *. Hint. You can use the error formula to determine whether the ap-proximation is too large or too small.

3.4.11.6. *. Hint. Use the function f(x) =√x.

3.4.11.7. *. Hint. Use the function f(x) = x1/3. What is a good choice of centre?

3.4.11.8. *. Hint. Try using the function f(x) = x5.

3.4.11.9. *. Hint. If you use the function f(x) = sin(x), what is a good centre?

3.4.11.10. *. Hint. Recalld

dxarctanx =

1

1 + x2.

3.4.11.11. *. Hint. Try using the function f(x) = (2 + x)3.

3.4.11.12. *. Hint. You can try using f(x) = (8 + x)1/3. What is a suitable centrefor your approximation?

3.4.11.13. *. Hint. This is the same as the Maclaurin polynomial.

3.4.11.14. *. Hint. This is a straightforward application of Equation 3.4.33.

3.4.11.17. *. Hint. 52/3 = f(52) = f(25)

Exercises — Stage 33.4.11.18. Hint. The fourth-degree Maclaurin polynomial for f(x) is

T4(x) = f(0) + f ′(0)x+1

2f ′′(0)x2 +

1

3!f ′′′(0)x3 +

1

4!f (4)(0)x4

while the third-degree Maclaurin polynomial for f(x) is

T3(x) = f(0) + f ′(0)x+1

2f ′′(0)x2 +

1

3!f ′′′(0)x3

580

Page 597: CLP-1 Differential Calculus

HINTS FOR EXERCISES

3.4.11.19. *. Hint. For part 3.4.11.19.c, think about what the quadratic approxi-mation looks like — is it pointing up or down?

3.4.11.22. *. Hint. For (c), you can write f(x) as the sum of Q(x) and its errorterm.For (d), you can use the linear approximation of ex centred at 0, with its errorterm when x = 0.1.

3.5 · Optimisation3.5.4 · Exercises· Exercises for § 3.5.1

Exercises — Stage 13.5.4.1. Hint. Estimate f ′(0).

3.5.4.2. Hint. If the graph is discontinuous at a point, it is not differentiable atthat point.

3.5.4.3. Hint. Try making a little bump at x = 2, the letting the function getquite large somewhere else.

Exercises — Stage 23.5.4.4. Hint. Critical points are those values of x in the domain of f(x) forwhich f ′(x) = 0.Singular points are those values of x in the domain of f(x) for which f(x) is notdifferentiable.

Exercises — Stage 33.5.4.5. Hint. We’re only after local extrema, not global. Let f(x) be our func-tion. If there is some interval around x = 2 where nothing is bigger than f(2),then f(2) is a local maximum, whether or not it is a maximum overall.

3.5.4.6. Hint. By Theorem 3.5.4, if x = 2 not a critical point, then it must be asingular point. Remember that endpoints can’t be local extrema.

3.5.4.7. Hint. You should be able to figure out the global minima of f(x) in yourhead.

Remember with absolute values, |X| =X X ≥ 0

−X X < 0.

3.5.4.8. Hint. Review the definitions of critical points and extrema: Defini-tion 3.5.6 and Definition 3.5.3.

· Exercises for § 3.5.2

Exercises — Stage 1

581

Page 598: CLP-1 Differential Calculus

HINTS FOR EXERCISES

3.5.4.1. Hint. One way to avoid a global minimum is to have limx→∞

f(x) = −∞.

Since f(x) keeps getting lower and lower, there is no one value that is the lowest.

3.5.4.2. Hint. Try allowing the function to approach the x-axis without evertouching it.

3.5.4.3. Hint. Since the global minimum value occurs at x = 5 and x = −5, itmust be true that f(5) = f(−5).

Exercises — Stage 23.5.4.4. Hint. Global extrema will either occur at critical points in the interval(−5, 5) or at the endpoints x = 5, x = −5.

3.5.4.5. Hint. You only need to consider critical points that are in the interval(−4, 0).

· Exercises for § 3.5.3

Exercises — Stage 13.5.4.1. *. Hint. Factor the derivative.

3.5.4.2. *. Hint. Remember to test endpoints.

3.5.4.4. *. Hint. One way to decide whether a critical point x = c is a localextremum is to consider the first derivative. For example: if f ′(x) is negativefor all x just to the left of c, and positive for all x just to the right of c, then f(x)decreases up till c, then increases after c, so f(x) has a local minimum at c.

3.5.4.5. *. Hint. One way to decide whether a critical point x = c is a localextremum is to consider the first derivative. For example: if f ′(x) is negativefor all x just to the left of c, and positive for all x just to the right of c, then f(x)decreases up till c, then increases after c, so f(x) has a local minimum at c.

3.5.4.6. *. Hint. Start with a formula for travel time from P to B. You mightwant to assign a variable to the distance from A where your buggy first reachesthe road.

3.5.4.7. *. Hint. A box has three dimensions; make variables for them, and writethe relations given in the problem in terms of these variables.

3.5.4.8. *. Hint. Find a formula for the cost of the base, and another formula forthe cost of the other sides. The total cost is the sum of these two formulas.

3.5.4.9. *. Hint. The setup is this:

582

Page 599: CLP-1 Differential Calculus

HINTS FOR EXERCISES

x

y

X

Y

O

3.5.4.10. *. Hint. Put the whole system on xy-axes, so that you can easily de-scribe the pieces using (x, y)-coordinates.

3.5.4.11. *. Hint. The surface area consists of two discs and a strip. Find theareas of these pieces.The volume of a cylinder with radius r and height h is πr2h.

h

r

r

h

3.5.4.12. *. Hint. If the circle has radius r, and the entire window has perimeterP , what is the height of the rectangle?

Exercises — Stage 33.5.4.14. *. Hint. Use logarithmic differentiation to find f ′(x).

3.5.4.15. *. Hint. When you are finding the global extrema of a function, re-member to check endpoints as well as critical points.

3.6 · Sketching Graphs

583

Page 600: CLP-1 Differential Calculus

HINTS FOR EXERCISES

3.6.7 · Exercises· Exercises for § 3.6.1

Exercises — Stage 13.6.7.1. Hint. What happens if g(x) = x+ 3?

Exercises — Stage 23.6.7.2. Hint. Use domains and intercepts to distinguish between the functions.

3.6.7.3. Hint. To find p, the equation f(0) = 2 gives you two possible values ofp. Consider the domain of f(x) to decide between them.

3.6.7.4. Hint. Check for horizontal asymptotes by evaluating limx→±∞

f(x), and

check for vertical asymptotes by finding any value of x near which f(x) blowsup.

3.6.7.5. Hint. Check for horizontal asymptotes by evaluating limx→±∞

f(x), and

check for vertical asymptotes by finding any value of x near which f(x) blowsup.

· Exercises for § 3.6.2

Exercises — Stage 13.6.7.1. Hint. For each of the graphs, consider where the derivative is positive,negative, and zero.

Exercises — Stage 23.6.7.2. *. Hint. Where is f ′(x) > 0?

3.6.7.3. *. Hint. Consider the signs of the numerator and the denominator off ′(x).

3.6.7.4. *. Hint. Rememberd

dxarctanx =

1

1 + x2.

· Exercises for § 3.6.3

Exercises — Stage 13.6.7.1. Hint. There are two intervals where the function is concave up, and twowhere it is concave down.

3.6.7.2. Hint. Try allowing your graph to have horizontal asymptotes. For ex-ample, let the function get closer and closer to the x-axis (or another horizontalline) without touching it.

584

Page 601: CLP-1 Differential Calculus

HINTS FOR EXERCISES

3.6.7.3. Hint. Consider f(x) = (x− 3)4.

Exercises — Stage 33.6.7.5. *. Hint. You must show it has at least one inflection point (try theIntermediate Value Theorem), and at most one inflection point (consider whetherthe second derivative is increasing or decreasing).

3.6.7.6. *. Hint. Use 3.6.7.6.a in proving 3.6.7.6.b.

3.6.7.7. Hint. Since x = 3 is an inflection point, we know the concavity of f(x)changes at x = 3. That is, there is some interval around 3, with endpoints a andb, such that

• f ′′(a) < 0 and f ′′(x) < 0 for every x between a and 3, and

• f ′′(b) > 0 and f ′′(x) > 0 for every x between b and 3.

Use the IVT to show that f ′′(x) = 3 for some x between a and b; then show thatthis value of x can’t be anything except x = 3.

· Exercises for § 3.6.4

Exercises — Stage 13.6.7.1. Hint. This function is symmetric across the y-axis.

3.6.7.2. Hint. There are two.

3.6.7.3. Hint. Since the function is even, you only have to reflect the portionshown across the y-axis to complete the sketch.

3.6.7.4. Hint. Since the function is odd, to complete the sketch, reflect the por-tion shown across the y-axis, then the x-axis.

Exercises — Stage 23.6.7.5. Hint. A function is even if f(−x) = f(x).

3.6.7.6. Hint. Its period is not 2π.

3.6.7.7. Hint. Simplify f(−x) to see whether it is the same as f(x), −f(x), orneither.

3.6.7.8. Hint. Simplify f(−x) to see whether it is the same as f(x), −f(x), orneither.

3.6.7.9. Hint. Find the smallest value k such that f(x + k) = f(x) for any x inthe domain of f .You may use the fact that the period of g(X) = tanX is π.

Exercises — Stage 3

585

Page 602: CLP-1 Differential Calculus

HINTS FOR EXERCISES

3.6.7.10. Hint. It is true that f(x) = f(x+ 2π) for every x in the domain of f(x),but the period is not 2π.

· Exercises for § 3.6.6

Exercises — Stage 13.6.7.1. *. Hint. You’ll find the intervals of increase and decrease. These willgive you a basic outline of the behaviour of the function. Use concavity to refineyour picture.

3.6.7.2. *. Hint. The local maximum is also a global maximum.

3.6.7.3. *. Hint. The sign of the first derivative is determined entirely by thenumerator, but the sign of the second derivative depends on both the numeratorand the denominator.

3.6.7.4. *. Hint. The function is odd.

3.6.7.5. *. Hint. The function is continuous at x = 0, but its derivative is not.

3.6.7.6. *. Hint. Since you aren’t asked to find the intervals of concavity exactly,sketch the intervals of increase and decrease, and turn them into a smooth curve.You might not get exactly the intervals of concavity that are given in the solution,but there should be the same number of intervals as the solution, and they shouldhave the same positions relative to the local extrema.

3.6.7.7. *. Hint. Use intervals of increase and decrease, concavity, and asymp-totes to sketch the curve.

3.6.7.8. Hint. Although the function exhibits a certain kind of repeating be-haviour, it is not periodic.

3.6.7.9. *. Hint. The period of this function is 2π. So, it’s enough to graph thecurve y = f(x) over the interval [−π, π], because that figure will simply repeat.Use trigonometric identities to write f ′′(x) = −4(4 sin2 x + sin x − 2). Then youcan find where f ′′(x) = 0 by setting y = sinx and solving 0 = 4y2 + y − 2.

3.6.7.10. Hint. There is one point where the curve is continuous but has a verti-cal tangent line.

Exercises — Stage 33.6.7.11. *. Hint. Use lim

x→−∞f ′(x) to determine lim

x→−∞f(x).

3.6.7.12. *. Hint. Once you have the graph of a function, reflect it over the liney = x to graph its inverse. Be careful of the fact that f(x) is only defined in thisproblem for x ≥ 0.

3.6.7.14. *. Hint. For (a), don’t be intimidated by the new names: we can graphthese functions using the methods learned in this section.For (b), remember that to define an inverse of a function, we need to restrict the

586

Page 603: CLP-1 Differential Calculus

HINTS FOR EXERCISES

domain of that function to an interval where it is one-to-one. Then to graph theinverse, we can simply reflect the original function over the line y = x.For (c), set y(x) = cosh−1(x), so cosh(y(x)) = x. The differentiate using thechain rule. To get your final answer in terms of x (instead of y), use the iden-tity cosh2(y)− sinh2(y) = 1.

3.7 · L’Hôpital’s Rule and Indeterminate Forms3.7.4 · Exercises

Exercises — Stage 13.7.4.1. Hint. Try making one function a multiple of the other.

3.7.4.2. Hint. Try making one function a multiple of the other, but not a constantmultiple.

3.7.4.3. Hint. Try modifying the function from Example 3.7.20.

Exercises — Stage 23.7.4.4. *. Hint. Plugging in x = 1 to the numerator and denominator makesboth zero. This is exactly one of the indeterminate forms where l’Hôpital’s rulecan be directly applied.

3.7.4.5. *. Hint. Is this an indeterminate form?

3.7.4.6. *. Hint. First, rearrange the expression to a more natural form (withouta negative exponent).

3.7.4.7. *. Hint. If at first you don’t succeed, try, try again.

3.7.4.8. *. Hint. Keep at it!

3.7.4.9. Hint. Rather than use l’Hôpital, try factoring out x2 from the numeratorand denominator.

3.7.4.10. *. Hint. Keep going!

3.7.4.12. Hint. Try plugging in x = 0. Is this an indeterminate form?

3.7.4.13. Hint. Simplify the trigonometric part first.

3.7.4.14. Hint. Rationalize, then remember your training.

3.7.4.15. *. Hint. If it is too difficult to take a derivative for l’Hôpital’s Rule,try splitting up the function into smaller chunks and evaluating their limits in-dependently.

3.7.4.17. *. Hint. Try manipulating the function to get it into a nicer form

3.7.4.19. Hint. limx→0

x2√

sin2 x = (sin2 x)1x2 ; what form is this?

587

Page 604: CLP-1 Differential Calculus

HINTS FOR EXERCISES

3.7.4.20. Hint. limx→0

x2√

cosx = limx→0

(cosx)1x2

3.7.4.21. Hint. logarithms

3.7.4.22. Hint. Introduce yet another logarithm.

3.7.4.23. *. Hint. If the denominator tends to zero, and the limit exists, whatmust be the limit of the numerator?

3.7.4.24. *. Hint. Start with one application of l’Hôpital’s Rule. After that, youneed to consider three distinct cases: k > 2, k < 2, and k = 2.

Exercises — Stage 33.7.4.25. Hint. Percentage error: 100

∣∣∣exact−approxexact

∣∣∣. Absolute error: |exact −approx|. (We’ll see these definitions again in 3.4.25.)

4 · Towards Integral Calculus4.1 · Introduction to Antiderivatives4.1.2 · Exercises

Exercises — Stage 14.1.2.1. Hint. The function f(x) is an antiderivative of f ′(x), but it is not themost general one.

4.1.2.2. Hint. When f(x) is positive, its antiderivative F (x) is increasing. Whenf(x) is negative, its antiderivative F (x) is decreasing. When f(x) = 0, F (x) has ahorizontal tangent line.

Exercises — Stage 24.1.2.3. Hint. For any constant n 6= −1, an antiderivative of xn is 1

n+1xn+1.

4.1.2.4. Hint. For any constant n 6= −1, an antiderivative of xn is 1n+1

xn+1.

4.1.2.5. Hint. For any constant n 6= −1, an antiderivative of xn is 1n+1

xn+1. Theconstant n does not have to be an integer.

4.1.2.6. Hint. What is the derivative of√x?

4.1.2.7. Hint. The derivative of e5x+11 is close to, but not exactly the same as,f(x). Don’t be afraid to just make a guess. But be sure to check by differentiat-ing your guess. If the derivative isn’t what you want, you will often still learnenough to be able to then guess the correct antiderivative.

4.1.2.8. Hint. From the table in the text, an antiderivative of sinx is − cosx, andan antiderivative of cosx is sinx.

4.1.2.9. Hint. What is the derivative of tangent?

588

Page 605: CLP-1 Differential Calculus

HINTS FOR EXERCISES

4.1.2.10. Hint. What is an antiderivative of1

x?

4.1.2.11. Hint.7√

3− 3x2=

7√3

(1√

1− x2

)4.1.2.12. Hint. How is this similar to the derivative of the arctangent function?

4.1.2.13. Hint. First, find the antiderivative of f ′(x). Your answer will have anunknown +C in it. Figure out which value of C gives f(1) = 10.

4.1.2.14. Hint. Remember that one antiderivative of cosx is sinx (not − sinx).

4.1.2.15. Hint. An antiderivative of1

xis log(x) + C, but only for x > 0.

4.1.2.16. Hint. What is the derivative of the arcsine function?

4.1.2.17. Hint. If P (t) is the population at time t, then the information given inthe problem is P ′(t) = 100e2t.

4.1.2.18. Hint. You can’t know exactly —there will be a constant involved.

4.1.2.19. Hint. If P (t) is the amount of power in kW-hours the house has usedsince time t = 0, where t is measured in hours, then the information given is thatP ′(t) = 0.5 sin

( π24t)

+ 0.25 kW.

Exercises — Stage 34.1.2.20. *. Hint. The derivatives should match. Remember sin−1 is anotherway of writing arcsine.

4.1.2.21. Hint. Think about the product rule.

4.1.2.22. Hint. Think about the quotient rule for derivatives.

4.1.2.23. Hint. Notice that the derivative of x3 is 3x2.

4.1.2.24. Hint. Think about the chain rule for derivatives. You might need tomultiply your first guess by a constant.

4.1.2.25. Hint. Simplify.

4.1.2.26. Hint. This problem is similar to Question 4.1.2.11, but you’ll have to

do some harder factoring. Try getting f(x) into the form a

(1√

1− (bx)2

)for

some constants a and b.

4.1.2.27. Hint. Following Example 4.1.7 let V (H) be the volume of the solidformed by rotating the segment of the parabola from x = −H to x = H . The planis to evaluate

V ′(H) = limh→H

V (H)− V (h)

H − h

589

Page 606: CLP-1 Differential Calculus

HINTS FOR EXERCISES

and then antidifferentiate V ′(H) to find V (H). Since you don’t know V (H)−V (h)(yet), first find upper and lower bounds on it when h < H . These bounds canbe the volumes of two cylinders, one with radius H (and what height?) and theother with radius h.

590

Page 607: CLP-1 Differential Calculus

ANSWERS TO EXERCISES

Appendix E

1 · Limits1.1 · Drawing Tangents and a First Limit1.1.2 · Exercises

Exercises — Stage 11.1.2.1. Answer.

x

yy = f(x)

P

Q

(a)

(b)

(c)

1.1.2.2. Answer.

a True

b In general, this is false. For most functions f(x) this will be false, but thereare some functions for which it is true.

1.1.2.3. Answer. At least once.

591

Page 608: CLP-1 Differential Calculus

ANSWERS TO EXERCISES

1.2 · Another Limit and Computing Velocity1.2.2 · Exercises

Exercises — Stage 11.2.2.1. Answer. Speed is nonnegative; velocity has a sign (positive or negative)that indicates direction.

1.2.2.2. Answer. Yes–an object that is not moving has speed 0.

1.2.2.3. Answer. 0 kph

1.2.2.4. Answer. The speed at the one second mark is larger than the averagespeed.

1.2.2.5. Answer. The slope of a curve is given bychange in vertical component

change in horizontal component. The change in the vertical component is

exactly s(b)− s(a), and the change in the horizontal component is exactly b− a.

1.2.2.6. Answer. (0, 2) ∪ (6, 7)

Exercises — Stage 21.2.2.7. Answer.

a 24 units per second.

b 6 units per second

1.2.2.8. Answer.

a 14

units per second

b 12

units per second

c 16

units per second

Remark: the average velocity is not the average of the two instantaneous veloci-ties.

1.3 · The Limit of a Function1.3.2 · Exercises

Exercises — Stage 11.3.2.1. Answer.

a limx→−2

f(x) = 1

b limx→0

f(x) = 0

592

Page 609: CLP-1 Differential Calculus

ANSWERS TO EXERCISES

c limx→2

f(x) = 2

1.3.2.2. Answer. DNE

1.3.2.3. Answer.

a limx→−1−

f(x) = 2

b limx→−1+

f(x) = −2

c limx→−1

f(x) = DNE

d limx→−2+

f(x) = 0

e limx→2−

f(x) = 0

1.3.2.4. Answer. Many answers are possible; here is one.

x

yy = f(x)

3

10

1.3.2.5. Answer. Many answers are possible; here is one.

593

Page 610: CLP-1 Differential Calculus

ANSWERS TO EXERCISES

x

yy = f(x)

3

10

1.3.2.6. Answer. In general, this is false.

1.3.2.7. Answer. False

1.3.2.8. Answer. limx→−2−

f(x) = 16

1.3.2.9. Answer. Not enough information to say.

Exercises — Stage 21.3.2.10. Answer. lim

t→0sin t = 0

1.3.2.11. Answer. limx→0+

log x = −∞

1.3.2.12. Answer. limy→3

y2 = 9

1.3.2.13. Answer. limx→0−

1

x= −∞

1.3.2.14. Answer. limx→0

1

x= DNE

1.3.2.15. Answer. limx→0

1

x2=∞

1.3.2.16. Answer. limx→3

1

10=

1

10

1.3.2.17. Answer. 9

1.4 · Calculating Limits with Limit Laws1.4.2 · Exercises

594

Page 611: CLP-1 Differential Calculus

ANSWERS TO EXERCISES

Exercises — Stage 11.4.2.1. Answer. (1.4.2.1.a) and (1.4.2.1.d)

1.4.2.2. Answer. There are many possible answers; one is f(x) = 10(x − 3),g(x) = x− 3.

1.4.2.3. Answer. There are many possible answers; one is f(x) = (x − 3)2 andg(x) = x− 3. Another is f(x) = 0 and g(x) = x− 3.

1.4.2.4. Answer. There are many possible answers; one is f(x) = x − 3, g(x) =(x− 3)3.

1.4.2.5. Answer. Any real number; positive infinity; negative infinity; does notexist.

Exercises — Stage 21.4.2.6. Answer. 0

1.4.2.7. Answer. 6

1.4.2.8. Answer. 16

1.4.2.9. *. Answer. 4/ cos(3)

1.4.2.10. *. Answer. 2

1.4.2.11. *. Answer. −7/2

1.4.2.12. *. Answer. 3

1.4.2.13. *. Answer. −32

1.4.2.14. *. Answer. log(2)− 1

1.4.2.15. *. Answer. 14

1.4.2.16. *. Answer.1

2

1.4.2.17. *. Answer. 5

1.4.2.18. *. Answer. −6

1.4.2.19. Answer. −14

1.4.2.20. *. Answer. −13

1.4.2.21. *. Answer. 16

1.4.2.22. *. Answer. 1√3

1.4.2.23. *. Answer. 1

1.4.2.24. *. Answer. 12

1.4.2.25. Answer. 0

1.4.2.26. Answer. 12

595

Page 612: CLP-1 Differential Calculus

ANSWERS TO EXERCISES

1.4.2.27. *. Answer. 0

1.4.2.28. Answer. 5

1.4.2.29. Answer. −∞

1.4.2.30. Answer.√

2

3

1.4.2.31. Answer. DNE

1.4.2.32. Answer. ∞1.4.2.33. Answer. x5 − 32x+ 15

1.4.2.34. Answer. 0

1.4.2.35. *. Answer. 0

1.4.2.36. *. Answer. 2

1.4.2.37. Answer. 0

1.4.2.38. Answer. −32

9

1.4.2.39. Answer. DNE

1.4.2.40. Answer. DNE

1.4.2.41. Answer. −9

2

1.4.2.42. Answer. −4

1.4.2.43. *. Answer. a =7

2

1.4.2.44. Answer.

a limx→0

f(x) = 0

b limx→0

g(x) = DNE

c limx→0

f(x)g(x) = 2

d limx→0

f(x)

g(x)= 0

e limx→2

f(x) + g(x) =9

2

f limx→0

f(x) + 1

g(x+ 1)= 1

Exercises — Stage 3

596

Page 613: CLP-1 Differential Calculus

ANSWERS TO EXERCISES

1.4.2.45. Answer.

x

y

1

1

Pictures may vary somewhat; the important points are the values of the functionat integer values of x, and the vertical asymptotes.

1.4.2.46. Answer.

x

y

1

1

y =f(x)

g(x)

1.4.2.47. Answer. 10

1.4.2.48. Answer. 1.4.2.48.a DNE , DNE1.4.2.48.b 01.4.2.48.c No: it is only true when both lim

x→af(x) and lim

x→ag(x) exist.

1.4.2.49. Answer. 1.4.2.49.a limx→0−

f(x) = −3

1.4.2.49.b limx→0+

f(x) = 3

1.4.2.49.c limx→0

f(x) =DNE

597

Page 614: CLP-1 Differential Calculus

ANSWERS TO EXERCISES

1.4.2.50. Answer. 1.4.2.50.a limx→−4−

f(x) = 0

1.4.2.50.b limx→−4+

f(x) = 0

1.4.2.50.c limx→−4

f(x) = 0

1.5 · Limits at Infinity1.5.2 · Exercises

Exercises — Stage 11.5.2.1. Answer. There are many answers: any constant polynomial has thisproperty. One answer is f(x) = 1.

1.5.2.2. Answer. There are many answers: any odd-degree polynomial has thisproperty. One answer is f(x) = x.

Exercises — Stage 21.5.2.3. Answer. 0

1.5.2.4. Answer. ∞1.5.2.5. Answer. 0

1.5.2.6. Answer. DNE

1.5.2.7. Answer. −∞1.5.2.8. Answer.

√3

1.5.2.9. *. Answer. 3

1.5.2.10. *. Answer. −34

1.5.2.11. *. Answer. −1

2

1.5.2.12. *. Answer. 12

1.5.2.13. *. Answer. 53

1.5.2.14. *. Answer. 0

1.5.2.15. *. Answer. 47

1.5.2.16. Answer. 1

1.5.2.17. *. Answer. 0

1.5.2.18. Answer. −1

1.5.2.19. Answer. 1

1.5.2.20. *. Answer. −1

1.5.2.21. *. Answer. −32

598

Page 615: CLP-1 Differential Calculus

ANSWERS TO EXERCISES

1.5.2.22. *. Answer. −53

1.5.2.23. Answer. −∞

1.5.2.24. *. Answer.5

2

1.5.2.25. Answer. lima→0+

a2 − 1a

a− 1=∞

1.5.2.26. Answer. limx→3

2x+ 81

x−3+ 1

x2−9

= 0

Exercises — Stage 31.5.2.27. Answer. No such rational function exists.

1.5.2.28. Answer. This is the amount of the substance that will linger long-term. Since it’s nonzero, the substance would be something that would stay inyour body. Something like “tattoo ink” is a reasonable answer, while “penicillin”is not.

1.6 · Continuity1.6.4 · Exercises

Exercises — Stage 11.6.4.1. Answer. Many answers are possible; the tangent function behaves likethis.

1.6.4.2. Answer. At some time between my birth and now, I was exactly onemeter tall.

1.6.4.3. Answer. One example is f(x) =

0 when 0 ≤ x ≤ 1

2 when 1 < x ≤ 2. The IVT only

guarantees f(c) = 1 for some c in [0, 2] when f is continuous over [0, 2]. If f is notcontinuous, the IVT says nothing.

x

y

1.6.4.4. Answer. Yes

1.6.4.5. Answer. No

599

Page 616: CLP-1 Differential Calculus

ANSWERS TO EXERCISES

1.6.4.6. Answer. No

1.6.4.7. Answer. True.

1.6.4.8. Answer. True.

1.6.4.9. Answer. In general, false.

1.6.4.10. Answer. limx→0+

h(x) = 0

Exercises — Stage 21.6.4.11. Answer. k = 0

1.6.4.12. Answer. Since f is a polynomial, it is continuous over all real numbers.f(0) = 1 < 12345 and f(12345) = 123453 + 123452 + 12345 + 1 > 12345 (since allterms are positive). So by the IVT, f(c) = 12345 for some c between 0 and 12345.

1.6.4.13. *. Answer. (−∞,−1) ∪ (−1, 1) ∪ (1,+∞)

1.6.4.14. *. Answer. (−∞,−1) ∪ (1,+∞)

1.6.4.15. *. Answer. The function is continuous except at x = ±π,±3π,±5π, · · ·.1.6.4.16. *. Answer. x 6= nπ, where n is any integer

1.6.4.17. *. Answer. ±2

1.6.4.18. *. Answer. c = 1

1.6.4.19. *. Answer. −1, 4

1.6.4.20. *. Answer. c = 1, c = −1

Exercises — Stage 31.6.4.21. Answer. This isn’t the kind of equality that we can just solve; we’llneed a trick, and that trick is the IVT. The general idea is to show that sinx issomewhere bigger, and somewhere smaller, than x − 1. However, since the IVTcan only show us that a function is equal to a constant, we need to slightly adjustour language. Showing sinx = x − 1 is equivalent to showing sinx − x + 1 = 0,so let f(x) = sinx− x+ 1, and let’s show that it has a real root.First, we need to note that f(x) is continuous (otherwise we can’t use the IVT).Now, we need to find a value of x for which it is positive, and for which it’s neg-ative. By checking a few values, we find f(0) is positive, and f(100) is negative.So, by the IVT, there exists a value of x (between 0 and 100) for which f(x) = 0.Therefore, there exists a value of x for which sinx = x− 1.

1.6.4.22. *. Answer. We let f(x) = 3x − x2. Then f(x) is a continuous function,since both 3x and x2 are continuous for all real numbers.We want a value a such that f(a) > 0. We see that a = 0 works since

f(0) = 30 − 0 = 1 > 0.

600

Page 617: CLP-1 Differential Calculus

ANSWERS TO EXERCISES

We want a value b such that f(b) < 0. We see that b = −1 works since

f(−1) =1

3− 1 < 0.

So, because f(x) is continuous on (−∞,∞) and f(0) > 0 while f(−1) < 0, thenthe Intermediate Value Theorem guarantees the existence of a real number c ∈(−1, 0) such that f(c) = 0.

1.6.4.23. *. Answer. We let f(x) = 2 tan(x) − x − 1. Then f(x) is a continuousfunction on the interval (−π/2, π/2) since tan(x) = sin(x)/ cos(x) is continuouson this interval, while x+ 1 is a polynomial and therefore continuous for all realnumbers.We find a value a ∈ (−π/2, π/2) such that f(a) < 0. We observe immediately thata = 0 works since

f(0) = 2 tan(0)− 0− 1 = 0− 1 = −1 < 0.

We find a value b ∈ (−π/2, π/2) such that f(b) > 0. We see that b = π/4 workssince

f(π/4) = 2 tan(π/4)− π/4− 1 = 2− π/4− 1 = 1− π/4= (4− π)/4 > 0

because 3 < π < 4.So, because f(x) is continuous on [0, π/4] and f(0) < 0 while f(π/4) > 0, thenthe Intermediate Value Theorem guarantees the existence of a real number c ∈(0, π/4) such that f(c) = 0.

1.6.4.24. *. Answer. Let f(x) =√

cos(πx) − sin(2πx) − 1/2. This function iscontinuous provided cos(πx) ≥ 0. This is true for 0 ≤ x ≤ 1

2.

Now f takes positive values on [0, 1/2]:

f(0) =√

cos(0)− sin(0)− 1/2 =√

1− 1/2 = 1/2.

And f takes negative values on [0, 1/2]:

f(1/2) =√

cos(π/2)− sin(π)− 1/2 = 0− 0− 1/2 = −1/2

(Notice that f(1/3) = (√

2−√

3)/2− 1/2 also works)So, because f(x) is continuous on [0, 1/2) and f(0) > 0 while f(1/2) < 0, thenthe Intermediate Value Theorem guarantees the existence of a real number c ∈(0, 1/2) such that f(c) = 0.

1.6.4.25. *. Answer. We let f(x) =1

cos2(πx)− x− 3

2. Then f(x) is a continuous

function on the interval (−1/2, 1/2) since cosx is continuous everywhere and

601

Page 618: CLP-1 Differential Calculus

ANSWERS TO EXERCISES

non-zero on that interval.The function f takes negative values. For example, when x = 0:

f(0) =1

cos2(0)− 0− 3

2= 1− 3

2= −1

2< 0.

It also takes positive values, for instance when x = 1/4:

f(1/4) =1

(cosπ/4)2− 1

4− 3

2

=1

1/2− 1 + 6

4

= 2− 7/4 = 1/4 > 0.

By the IVT there is c, 0 < c < 1/3 such that f(c) = 0, in which case

1

(cos πc)2= c+

3

2.

1.6.4.26. Answer. [0, 1] is the easiest answer to find. Also acceptable are [−2,−1]and [14, 15].

1.6.4.27. Answer. 1.91

1.6.4.28. Answer.

• If f(a) = g(a), or f(b) = g(b), then we simply take c = a or c = b.

• Suppose f(a) 6= g(a) and f(b) 6= g(b). Then f(a) < g(a) and g(b) < f(b),so if we define h(x) = f(x) − g(x), then h(a) < 0 and h(b) > 0. Since his the difference of two functions that are continuous over [a, b], also h iscontinuous over [a, b]. So, by the Intermediate Value Theorem, there existssome c ∈ (a, b) with h(c) = 0; that is, f(c) = g(c).

2 · Derivatives2.1 · Revisiting Tangent Lines2.1.2 · Exercises

Exercises — Stage 12.1.2.1. Answer. IfQ is to the left of the y axis, the secant line has positive slope;if Q is to the right of the y axis, the secant line has negative slope.

2.1.2.2. Answer. 2.1.2.2.a closer2.1.2.2.b the tangent line has the larger slope

2.1.2.3. Answer. (a), (c), (e), (b), (f), (d)

Exercises — Stage 2

602

Page 619: CLP-1 Differential Calculus

ANSWERS TO EXERCISES

2.1.2.4. Answer. Something like 1.5. A reasonable answer would be between 1and 2.

2.1.2.5. Answer. There is only one tangent line to f(x) at P (shown in blue), butthere are infinitely many choices of Q and R (one possibility shown in red).

x

y

y = f(x)

P

Q

R

2.1.2.6. Answer.

x

y

y = f(x)

2.2 · Definition of the Derivative2.2.4 · Exercises

Exercises — Stage 12.2.4.1. Answer. (a), (d)

2.2.4.2. Answer. (e)

2.2.4.3. Answer. (b)

2.2.4.4. *. Answer. By definition, f(x) = x3 is differentiable at x = 0 if the limit

limh→0

f(h)− f(0)

h= lim

h→0

h3 − 0

h

exists.

603

Page 620: CLP-1 Differential Calculus

ANSWERS TO EXERCISES

2.2.4.5. Answer. x = −1 and x = 3

2.2.4.6. Answer. True. (Contrast to Question 2.2.4.7.)

2.2.4.7. Answer. In general, false. (Contrast to Question 2.2.4.6.)

2.2.4.8. Answer. metres per second

Exercises — Stage 22.2.4.9. Answer. y − 6 = 3(x− 1), or y = 3x+ 3

2.2.4.10. Answer.−1

x2

2.2.4.11. *. Answer. By definition

f ′(0) = limh→0

f(h)− f(0)

h= lim

h→0

h|h|h

= limh→0|h| = 0

In particular, the limit exists, so the derivative exists (and is equal to zero).

2.2.4.12. *. Answer.−2

(x+ 1)2

2.2.4.13. *. Answer.−2x

[x2 + 3]2

2.2.4.14. Answer. 1

2.2.4.15. *. Answer. f ′(x) = − 2

x3

2.2.4.16. *. Answer. a = 4, b = −4

2.2.4.17. *. Answer. f ′(x) =1

2√

1 + xwhen x > −1; f ′(x) does not exist when

x ≤ −1.

Exercises — Stage 32.2.4.18. Answer. v(t) = 4t3 − 2t

2.2.4.19. *. Answer. No, it does not.

2.2.4.20. *. Answer. No, it does not.

2.2.4.21. *. Answer. Yes, it is.

2.2.4.22. *. Answer. Yes, it is.

2.2.4.23. . Answer. Many answers are possible; here is one.

604

Page 621: CLP-1 Differential Calculus

ANSWERS TO EXERCISES

x

y

1

1

2.2.4.24. Answer.

p′(x) = limh→0

p(x+ h)− p(x)

h

= limh→0

f(x+ h) + g(x+ h)− f(x)− g(x)

h

= limh→0

f(x+ h)− f(x) + g(x+ h)− g(x)

h

= limh→0

[f(x+ h)− f(x)

h+g(x+ h)− g(x)

h

](∗) =

[limh→0

f(x+ h)− f(x)

h

]+

[limh→0

g(x+ h)− g(x)

h

]= f ′(x) + g′(x)

At step (∗), we use the limit law that limx→a

[F (x) +G(x)] = limx→a

F (x) + limx→a

G(x),

as long as limx→a

F (x) and limx→a

G(x) exist. Because the problem states that f ′(x) and

g′(x) exist, we know that limh→0

f(x+ h)− f(x)

hand lim

h→0

g(x+ h)− g(x)

hexist, so

our work is valid.

2.2.4.25. Answer. 2.2.4.25.a f ′(x) = 2 and g′(x) = 12.2.4.25.b p′(x) = 4x2.2.4.25.c no

2.2.4.26. *. Answer. y = 6x− 9 and y = −2x− 1

2.2.4.27. *. Answer. a > 1

605

Page 622: CLP-1 Differential Calculus

ANSWERS TO EXERCISES

2.3 · Interpretations of the Derivative2.3.3 · Exercises

Exercises — Stage 22.3.3.1. Answer. 2.3.3.1.a The average rate of change of the height of the waterover the single day starting at t = 0, measured in m

hr.

2.3.3.1.b The instantaneous rate of change of the height of the water at the timet = 0.

2.3.3.2. Answer. Profit per widget

2.3.3.3. Answer. T ′(d) measures how quickly the temperature is changing perunit change of depth, measured in degrees per metre. |T ′(d)| will probably belargest when d is near zero, unless there are hot springs or other underwater heatsources.

2.3.3.4. Answer. Calories per gram.

2.3.3.5. Answer. The acceleration of the object.

2.3.3.6. Answer. Degrees Celsius temperature change per joule of heat added.(This is closely related to heat capacity and to specific heat — there’s a nice ex-planation of this on Wikipedia.)

2.3.3.7. Answer. Number of bacteria added per degree. That is: the number ofextra bacteria (possibly negative) that will exist in the population by raising thetemperature by one degree.

Exercises — Stage 32.3.3.8. Answer. 360R′(t)

2.3.3.9. Answer. If P ′(t) is positive, your sample is below the ideal temperature,and if P ′(t) is negative, your sample is above the ideal temperature. If P ′(t) = 0,you don’t know whether the sample is exactly at the ideal temperature, or wayabove or below it with no living bacteria.

2.4 · Arithmetic of Derivatives - a Differentiation Toolbox2.4.2 · Exercises

Exercises — Stage 12.4.2.1. Answer. True

2.4.2.2. Answer. False, in general.

2.4.2.3. Answer. True

2.4.2.4. Answer. If you’re creative, you can find lots of ways to differentiate!

• Constant multiple: g′(x) = 3f ′(x).

606

Page 623: CLP-1 Differential Calculus

ANSWERS TO EXERCISES

• Product rule: g′(x) = ddx3f(x) + 3f ′(x) = 0f(x) + 3f ′(x) = 3f ′(x).

• Sum rule: g′(x) = ddxf(x) + f(x) + f(x) = f ′(x) + f ′(x) + f ′(x) = 3f ′(x).

• Quotient rule: g′(x) = ddx

f(x)

13

=

13f ′(x)−f(x)(0)

19

=13f ′(x)19

= 9(

13

)f ′(x) =

3f ′(x).

All rules give g′(x) = 3f ′(x).

Exercises — Stage 22.4.2.5. Answer. f ′(x) = 15x4 + 8

3x−1/3

2.4.2.6. Answer. −36x+ 24√x+ 20√

x− 45

2.4.2.7. *. Answer. y − 18

= 34·(x− 1

2

), or y = 3

4x− 1

4

2.4.2.8. *. Answer. 2.4.2.8.a 42.4.2.8.b left2.4.2.8.c decreasing

2.4.2.9. *. Answer.1

(x+ 1/2)2 , or4

(2x+ 1)2

2.4.2.10. Answer. −72

2.4.2.11. Answer. y − 12

= −18(x− 1), or y = −1

8x+ 5

8

Exercises — Stage 32.4.2.12. Answer. b′(t)− d′(t)2.4.2.13. *. Answer. (1, 3), (3, 27)

2.4.2.14. *. Answer.1

2√

100180

2.4.2.15. Answer. 20t+ 7 square metres per second.

2.4.2.16. Answer. 0

2.4.2.17. Answer. First expression, f(x) =g(x)

h(x):

f ′(x) =h(x)g′(x)− g(x)h′(x)

h2(x)

Second expresson, f(x) =g(x)

k(x)· k(x)

h(x):

f ′(x) =

(k(x)g′(x)− g(x)k′(x)

k2(x)

)(k(x)

h(x)

)

607

Page 624: CLP-1 Differential Calculus

ANSWERS TO EXERCISES

+

(g(x)

k(x)

)(h(x)k′(x)− k(x)h′(x)

h2(x)

)=k(x)g′(x)− g(x)k′(x)

k(x)h(x)+g(x)h(x)k′(x)− g(x)k(x)h′(x)

k(x)h2(x)

=h(x)k(x)g′(x)− h(x)g(x)k′(x)

k(x)h2(x)+g(x)h(x)k′(x)− g(x)k(x)h′(x)

k(x)h2(x)

=h(x)k(x)g′(x)− h(x)g(x)k′(x) + g(x)h(x)k′(x)− g(x)k(x)h′(x)

k(x)h2(x)

=h(x)k(x)g′(x)− g(x)k(x)h′(x)

k(x)h2(x)

=h(x)g′(x)− g(x)h′(x)

h2(x)

and this is exactly what we got from differentiating the first expression.

2.6 · Using the Arithmetic of Derivatives – Examples2.6.2 · Exercises

Exercises — Stage 12.6.2.1. Answer. In the quotient rule, there is a minus, not a plus. Also, 2(x +1) + 2x is not the same as 2(x+ 1).The correct version is:

f(x) =2x

x+ 1

f ′(x) =2(x+ 1)−2x

(x+ 1)2

=2

(x+ 1)2

2.6.2.2. Answer. False

Exercises — Stage 22.6.2.3. Answer. 4x(x2 + 2)(x2 + 3)

2.6.2.4. Answer. 12t3 + 15t2 + 1t2

2.6.2.5. Answer. x′(y) = 8y3 + 2y

2.6.2.6. Answer. T ′(x) =(x2 + 3)

(1

2√x

)− (√x+ 1)(2x)

(x2 + 3)2

2.6.2.7. *. Answer.21− 4x− 7x2

(x2 + 3)2

608

Page 625: CLP-1 Differential Calculus

ANSWERS TO EXERCISES

2.6.2.8. Answer. 7

2.6.2.9. Answer.3x4 + 30x3 − 2x− 5

(x2 + 5x)2

2.6.2.10. *. Answer.−3x2 + 12x+ 5

(2− x)2

2.6.2.11. *. Answer.−22x

(3x2 + 5)2

2.6.2.12. *. Answer.4x3 + 12x2 − 1

(x+ 2)2

2.6.2.13. *. Answer. The derivative of the function is

(1− x2) · 12√x−√x · (−2x)

(1− x2)2=

(1− x2)− 2x · (−2x)

2√x(1− x2)2

The derivative is undefined if either x < 0 or x = 0,±1 (since the square-root isundefined for x < 0 and the denominator is zero when x = 0, 1,−1. Putting thistogether — the derivative exists for x > 0, x 6= 1.

2.6.2.14. Answer.(

35x−45 + 5x

−23

)(3x2 + 8x− 5) + (3 5

√x+ 15 3

√x+ 8) (6x+ 8)

2.6.2.15. Answer. f ′(x) = (2x+5)(x−1/2+x−2/3)+(x2+5x+1)(−1

2x−3/2 − 2

3x−5/3

)2.6.2.16. Answer. x = −5 and x = 1

Exercises — Stage 3

2.6.2.17. *. Answer. y = x− 1

4

2.6.2.18. Answer.

609

Page 626: CLP-1 Differential Calculus

ANSWERS TO EXERCISES

y = 4x− 4 and y = −2x− 1

2.6.2.19. *. Answer. − sin(2015)

2.6.2.20. *. Answer. −√

3/2

2.6.2.21. *. Answer. −1

2.6.2.22. *. Answer. 2015 · 22014

2.7 · Derivatives of Exponential Functions2.7.3 · Exercises

Exercises — Stage 12.7.3.1. Answer. A-(a) and (d), B-(e), C-(c), D-(b)

2.7.3.2. Answer. (b), (d), (e)

2.7.3.3. Answer. False

2.7.3.4. Answer. increasing

Exercises — Stage 2

2.7.3.5. Answer.(x− 1)ex

2x2

2.7.3.6. Answer. 2e2x

2.7.3.7. Answer. ea+x

2.7.3.8. Answer. x > −1

2.7.3.9. Answer. −e−x

2.7.3.10. Answer. 2e2x

2.7.3.11. Answer. When t is in the interval (−2, 0).

Exercises — Stage 32.7.3.12. Answer. g′(x) = [f(x) + f ′(x)]ex

2.7.3.13. Answer. (b) and (d)

2.7.3.14. *. Answer. a = b =e

2

2.8 · Derivatives of Trigonometric Functions2.8.8 · Exercises

Exercises — Stage 1

610

Page 627: CLP-1 Differential Calculus

ANSWERS TO EXERCISES

2.8.8.1. Answer.

x

y

π−π

y = sinx

y = cosx

The graph f(x) = sinx has horizontal tangent lines precisely at those pointswhere cosx = 0.

2.8.8.2. Answer.

x

y

π−π

y = sinx

y = cosx

The graph f(x) = sinx has maximum slope at those points where cosx has amaximum. That is, where cosx = 1.

Exercises — Stage 22.8.8.3. Answer. f ′(x) = cos x− sinx+ sec2 x

2.8.8.4. Answer. x = π4

+ πn, for any integer n.

2.8.8.5. Answer. 0

2.8.8.6. Answer. f ′(x) = 2(cos2 x− sin2 x)

2.8.8.7. Answer. f ′(x) = ex(cotx− csc2 x)

2.8.8.8. Answer. f ′(x) =2 + 3 secx+ 2 sinx− 2 tanx secx+ 3 sinx tanx

(cosx+ tanx)2

2.8.8.9. Answer. f ′(x) =5 secx tanx− 5 secx− 1

ex

2.8.8.10. Answer. f ′(x) = (ex+cotx)(30x5+cscx cotx)+(ex−csc2 x)(5x6−cscx)

2.8.8.11. Answer. − sin(θ)

2.8.8.12. Answer. f ′(x) = − cosx− sinx

2.8.8.13. Answer.(

cos θ + sin θ

cos θ − sin θ

)2

+ 1

2.8.8.14. *. Answer. a = 0, b = 1.

2.8.8.15. *. Answer. y − π = 1 · (x− π/2)

611

Page 628: CLP-1 Differential Calculus

ANSWERS TO EXERCISES

Exercises — Stage 32.8.8.16. *. Answer. −

√3/2

2.8.8.17. Answer.

tan θ =sin θ

cos θ

So, using the quotient rule,

d

dθtan θ =

cos θ cos θ − sin θ(− sin θ)

cos2 θ=

cos2 θ + sin2 θ

cos2 θ

=

(1

cos θ

)2

= sec2 θ

2.8.8.18. *. Answer. a = −23, b = 2

2.8.8.19. *. Answer. All values of x except x = π2

+ nπ, for any integer n.

2.8.8.20. *. Answer. The function is differentiable whenever x2 +x−6 6= 0 sincethe derivative equals

10 cos(x) · (x2 + x− 6)− 10 sin(x) · (2x+ 1)

(x2 + x− 6)2,

which is well-defined unless x2+x−6 = 0. We solve x2+x−6 = (x−2)(x+3) = 0,and get x = 2 and x = −3. So, the function is differentiable for all real values xexcept for x = 2 and for x = −3.

2.8.8.21. *. Answer. The function is differentiable whenever sin(x) 6= 0 sincethe derivative equals

sin(x) · (2x+ 6)− cos(x) · (x2 + 6x+ 5)

(sinx)2,

which is well-defined unless sinx = 0. This happens when x is an integer mul-tiple of π. So, the function is differentiable for all real values x except x = nπ,where n is any integer.

2.8.8.22. *. Answer. y − 1 = 2 · (x− π/4)

2.8.8.23. *. Answer. y = 2x+ 2

2.8.8.24. Answer. x = 3π4

+ nπ for any integer n.

2.8.8.25. Answer. f ′(0) = 0

2.8.8.26. *. Answer. h′(x) =

cosx x > 0

− cosx x < 0It exists for all x 6= 0.

2.8.8.27. *. Answer. 2.8.8.27.iii

612

Page 629: CLP-1 Differential Calculus

ANSWERS TO EXERCISES

2.8.8.28. *. Answer. 2

2.9 · One More Tool – the Chain Rule2.9.4 · Exercises

Exercises — Stage 12.9.4.1. Answer. 2.9.4.1.a dK

dUis negative

2.9.4.1.b dUdO

is negative2.9.4.1.c dK

dOis positive

2.9.4.2. Answer. negative

Exercises — Stage 22.9.4.3. Answer. −5 sin(5x+ 3)

2.9.4.4. Answer. 10x(x2 + 2)4

2.9.4.5. Answer. 17(4k4 + 2k2 + 1)16 · (16k3 + 4k)

2.9.4.6. Answer. −2x(x2−1)

√x4−1

2.9.4.7. Answer. −ecos(x2) · sin(x2) · 2x2.9.4.8. *. Answer. −4

2.9.4.9. *. Answer. [cosx− x sinx]ex cos(x)

2.9.4.10. *. Answer. [2x− sinx]ex2+cos(x)

2.9.4.11. *. Answer. 3

2√x−1√x+2

3

2.9.4.12. *. Answer. f ′(x) = − 2

x3+

x√x2 − 1

is defined for x in (−∞, 1)∪ (1,∞).

2.9.4.13. *. Answer. f ′(x) =(1 + x2)(5 cos 5x)− (sin 5x)(2x)

(1 + x2)2

2.9.4.14. Answer. 2e2x+7 sec(e2x+7) tan(e2x+7)

2.9.4.15. Answer. y = 1

2.9.4.16. Answer. t = 23

and t = 4

2.9.4.17. Answer. 2e sec2(e)

2.9.4.18. *. Answer. y′ = 4e4x tanx+ e4x sec2 x

2.9.4.19. *. Answer.3

(1 + e3)2

2.9.4.20. *. Answer. 2 sin(x) · cos(x) · esin2(x)

613

Page 630: CLP-1 Differential Calculus

ANSWERS TO EXERCISES

2.9.4.21. *. Answer. cos (e5x) · e5x · 5

2.9.4.22. *. Answer. −ecos(x2) · sin(x2) · 2x

2.9.4.23. *. Answer. y′ = − sin(x2 +

√x2 + 1

)(2x+

x√x2 + 1

)2.9.4.24. *. Answer. y′ = 2x cos2 x− 2(1 + x2) sinx cosx

2.9.4.25. *. Answer. y′ =e3x(3x2 − 2x+ 3)

(1 + x2)2

2.9.4.26. *. Answer. −40

2.9.4.27. *. Answer. (1, 1) and (−1,−1).

2.9.4.28. Answer. Always

2.9.4.29. Answer. ex sec3(5x− 7)(1 + 15 tan(5x− 7))

2.9.4.30. *. Answer. e2x cos 4x+ 2xe2x cos 4x− 4xe2x sin 4x

Exercises — Stage 32.9.4.31. Answer. t =

π

4

2.9.4.32. *. Answer. Let f(x) = ex+x2 and g(x) = 1 + x. Then f(0) = g(0) = 1.f ′(x) = (1 + 2x)ex+x2 and g′(x) = 1. When x > 0,

f ′(x) = (1 + 2x)ex+x2 > 1 · ex+x2 = ex+x2 > e0+02 = 1 = g′(x).

Since f(0) = g(0), and f ′(x) > g′(x) for all x > 0, that means f and g start at thesame place, but f always grows faster. Therefore, f(x) > g(x) for all x > 0.

2.9.4.33. Answer. cos(2x) = cos2 x− sin2 x

2.9.4.34. Answer.

f ′(x) =1

3

(√x3 − 9 tanx

ecscx2

) 23

·

ecscx2(− 2x

√x3−9 tanx cos(x2)

sin2(x2)− 3x2 tanx

2√x3−9

−√x3−9 sec2 x

)(tan2 x)(x3 − 9)

2.9.4.35. Answer. 2.9.4.35.a

614

Page 631: CLP-1 Differential Calculus

ANSWERS TO EXERCISES

x

y

(1,0)t=π/2

(−1,0)t=3π/2

(0,1)t=0,π,2π

(− 1√

2, 12

)t=5π/4,7π/4

(1√2, 12

)t=π/4,3π/4

The particle traces the curve y = 1− x2 restricted to domain [−1, 1]. At t = 0, theparticle is at the top of the curve, (1, 0). Then it moves to the right, and goes backand forth along the curve, repeating its path every 2π units of time.2.9.4.35.b

√3

2.10 · The Natural Logarithm2.10.3 · Exercises

Exercises — Stage 12.10.3.1. Answer. Ten speakers: 13 dB. One hundred speakers: 23 dB.

2.10.3.2. Answer. 20 log 2 ≈ 14 years

2.10.3.3. Answer. (b)

Exercises — Stage 2

2.10.3.4. Answer. f ′(x) =1

x

2.10.3.5. Answer. f ′(x) =2

x

2.10.3.6. Answer. f ′(x) =2x+ 1

x2 + x

2.10.3.7. Answer. f ′(x) =1

x log 10

2.10.3.8. *. Answer. y′ =1− 3 log x

x4

2.10.3.9. Answer.d

dθlog(sec θ) = tan θ

615

Page 632: CLP-1 Differential Calculus

ANSWERS TO EXERCISES

2.10.3.10. Answer. f ′(x) =−ecos(log x) sin(log x)

x

2.10.3.11. *. Answer. y′ =2x+ 4x3

2√x4+1

x2 +√x4 + 1

2.10.3.12. *. Answer.tanx

2√− log(cosx)

2.10.3.13. *. Answer.√x2 + 4 + x

x√x2 + 4 + x2 + 4

=1√

x2 + 4

2.10.3.14. *. Answer. g′(x) =2xex

2√1 + x4 + 2x3

ex2√

1 + x4 + 1 + x4

2.10.3.15. *. Answer.4

3

2.10.3.16. Answer. f ′(x) =3x

x2 + 5− 2x3

x4 + 10

2.10.3.17. Answer.40

3

2.10.3.18. *. Answer. g′(x) = πx log π + πxπ−1

2.10.3.19. Answer. f ′(x) = xx(log x+ 1)

2.10.3.20. *. Answer. xx(log x+ 1) +1

x log 10

2.10.3.21. Answer. f ′(x) =1

4

(4

√(x4+12)(x4−x2+2)

x3

)(4x3

x4+12+

4x3 − 2x

x4−x2+2− 3

x

)2.10.3.22. Answer.

f ′(x) = (x+ 1)(x2 + 1)2(x3 + 1)3(x4 + 1)4(x5 + 1)5[1

x+ 1+

4x

x2 + 1+

9x2

x3 + 1+

16x3

x4 + 1+

25x4

x5 + 1

]

2.10.3.23. Answer.(

x2 + 2x+ 3

3x4 + 4x3 + 5

)(1

x2 + 2x+ 3− 6x2

3x4 + 4x3 + 5− 1

2(x+ 1)2

)2.10.3.24. *. Answer. f ′(x) = (cos x)sinx [(cosx) log(cosx)− sinx tanx]

2.10.3.25. *. Answer.d

dx(tanx)x = (tanx)x

(log(tan x) +

x

sinx cosx

)2.10.3.26. *. Answer. 2x(x2 + 1)x

2+1(1 + log(x2 + 1))

2.10.3.27. *. Answer. f ′(x) = (x2 + 1)sin(x) ·(cosx · log(x2 + 1) + 2x sinx

x2+1

)616

Page 633: CLP-1 Differential Calculus

ANSWERS TO EXERCISES

2.10.3.28. *. Answer. xcos3(x) ·(−3 cos2(x) sin(x) log(x) +

cos3(x)

x

)

2.10.3.29. *. Answer. (3 + sin(x))x2−3 ·

[2x log(3 + sin(x)) +

(x2 − 3) cos(x)

3 + sin(x)

]

Exercises — Stage 3

2.10.3.30. Answer.d

dx

[f(x)]g(x)

= [f(x)]g(x)

[g′(x) log(f(x)) +

g(x)f ′(x)

f(x)

]2.10.3.31. Answer. Let g(x) := log(f(x)). Notice g′(x) = f ′(x)

f(x).

In order to show that the two curves have horizontal tangent lines at the samevalues of x, we will show two things: first, that if f(x) has a horizontal tangentline at some value of x, then also g(x) has a horizontal tangent line at that valueof x. Second, we will show that if g(x) has a horizontal tangent line at some valueof x, then also f(x) has a horizontal tangent line at that value of x.Suppose f(x) has a horizontal tangent line where x = x0 for some point x0. Thismeans f ′(x0) = 0. Then g′(x0) = f ′(x0)

f(x0). Since f(x0) 6= 0, f

′(x0)f(x0)

= 0f(x0)

= 0, so g(x)

also has a horizontal tangent line when x = x0. This shows that whenever f hasa horizontal tangent line, g has one too.Now suppose g(x) has a horizontal tangent line where x = x0 for some point x0.This means g′(x0) = 0. Then g′(x0) = f ′(x0)

f(x0)= 0, so f ′(x0) exists and is equal to

zero. Therefore, f(x) also has a horizontal tangent line when x = x0. This showsthat whenever g has a horizontal tangent line, f has one too.

2.11 · Implicit Differentiation2.11.2 · Exercises

Exercises — Stage 12.11.2.1. Answer. (a) and (b)

2.11.2.2. Answer. At (0, 4) and (0,−4),dy

dxis 0; at (0, 0),

dy

dxdoes not exist.

2.11.2.3. Answer. (a) no

(b) nody

dx= −x

y. It is not possible to write

dy

dxas a function of x, because (as stated

in (b)) one value of x may give two values ofdy

dx. For instance, when x = π/4, at

the point(π

4,

1√2

)the circle has slope

dy

dx= −1, while at the point

4,−1√

2

)the

circle has slopedy

dx= 1.

Exercises — Stage 2

617

Page 634: CLP-1 Differential Calculus

ANSWERS TO EXERCISES

2.11.2.4. *. Answer.dy

dx= −e

x + y

ey + x

2.11.2.5. *. Answer.dy

dx=

y2 + 1

ey − 2xy

2.11.2.6. *. Answer. At (x, y) = (4, 1), y′ = − 1

π + 1. At (x, y) = (−4, 1), y′ =

1

π − 1.

2.11.2.7. *. Answer. −2x sin(x2 + y) + 3x2

4y3 + sin(x2 + y)

2.11.2.8. *. Answer. At (x, y) = (1, 0), y′ = −6, and at (x, y) = (−5, 0), y′ = 625

.

2.11.2.9. *. Answer. dydx

=cos(x+ y)− 2x

2y − cos(x+ y)

2.11.2.10. *. Answer. At (x, y) = (2, 0) we have y′ = −32, and at (x, y) = (−4, 0)

we have y′ = −34.

2.11.2.11. Answer.

(√3

2,−1

2√

3

),

(−√

3

2,

1

2√

3

)

2.11.2.12. *. Answer. −28

3

2.11.2.13. *. Answer.dy

dx= − 2xy2 + sin y

2x2y + x cos y

Exercises — Stage 32.11.2.14. *. Answer. At (x, y) = (2, 0), y′ = −2. At (x, y) = (−2, 0), y′ = 2.

2.11.2.15. Answer. x = 0, x = 1, x = −1

2.12 · Inverse Trigonometric Functions2.12.2 · Exercises

Exercises — Stage 12.12.2.1. Answer. (a) (−∞,∞)(b) all integer multiples of π(c) [−1, 1]

2.12.2.2. Answer. False

2.12.2.3. Answer.

618

Page 635: CLP-1 Differential Calculus

ANSWERS TO EXERCISES

x

y

y = f−1(x)

1

2.12.2.4. Answer.

• If |a| > 1, there is no point where the curve has horizontal tangent line.

• If |a| = 1, the curve has a horizontal tangent line where x = 2πn +aπ

2for

any integer n.

• If |a| < 1, the curve has a horizontal tangent line where x = 2πn+ arcsin(a)or x = (2n+ 1)π − arcsin(a) for any integer n.

2.12.2.5. Answer. Domain: x = ±1. Not differentiable anywhere.

Exercises — Stage 2

2.12.2.6. Answer. f ′(x) =1√

9− x2; domain of f is [−3, 3].

2.12.2.7. Answer. f ′(t) =− t2−1√

1−t2 − 2t arccos t

(t2 − 1)2, and the domain of f(t) is (−1, 1).

2.12.2.8. Answer. The domain of f(x) is all real numbers, and f ′(x) =−2x

(x2 + 2)√x4 + 4x2 + 3

.

2.12.2.9. Answer. f ′(x) =1

a2 + x2and the domain of f(x) is all real numbers.

2.12.2.10. Answer. f ′(x) = arcsin x, and the domain of f(x) is [−1, 1].

2.12.2.11. Answer. x = 0

2.12.2.12. Answer.d

dxarcsinx+ arccosx = 0

619

Page 636: CLP-1 Differential Calculus

ANSWERS TO EXERCISES

2.12.2.13. *. Answer. y′ =−1

x2

√1− 1

x2

2.12.2.14. *. Answer. y′ =−1

1 + x2

2.12.2.15. *. Answer. 2x arctanx+ 1

2.12.2.16. Answer. Let θ = arctanx. Then θ is the angle of a right triangle thatgives tan θ = x. In particular, the ratio of the opposite side to the adjacent side isx. So, we have a triangle that looks like this:

θ

x

1

√x2 + 1

where the length of the hypotenuse came from the Pythagorean Theorem. Now,

sin (arctanx) = sin θ =opphyp

=x√x2 + 1

From here, we differentiate using the quotient rule:

d

dx

x√x2 + 1

=

√x2 + 1− x 2x

2√x2+1

x2 + 1

=

(√x2 + 1− x2√

x2+1

x2 + 1

)·√x2 + 1√x2 + 1

=(x2 + 1)− x2

(x2 + 1)3/2

=1

(x2 + 1)3/2= (x2 + 1)−3/2

2.12.2.17. Answer. Let θ = arcsinx. Then θ is the angle of a right triangle thatgives sin θ = x. In particular, the ratio of the opposite side to the hypotenuse is x.So, we have a triangle that looks like this:

620

Page 637: CLP-1 Differential Calculus

ANSWERS TO EXERCISES

θ

x

√1− x2

1

where the length of the adjacent side came from the Pythagorean Theorem. Now,

cot (arcsinx) = cot θ =adjopp

=

√1− x2

x

From here, we differentiate using the quotient rule:

d

dx

√1− x2

x

=x −2x

2√

1−x2 −√

1− x2

x2

=−x2 − (1− x2)

x2√

1− x2

=−1

x2√

1− x2

2.12.2.18. *. Answer. (x, y) = ±(√

32, π

3

)2.12.2.19. Answer. x =

(2n+ 1)π

2for any integer n

Exercises — Stage 3

2.12.2.20. *. Answer. g′(y) =1

1− sin g(y)

2.12.2.21. *. Answer.1

2

2.12.2.22. *. Answer.1

e+ 1

2.12.2.23. Answer. f ′(x) = [sinx + 2]arcsecx

(log[sinx+ 2]

|x|√x2 − 1

+arcsecx · cosx

sinx+ 2

).

The domain of f(x) is |x| ≥ 1.

2.12.2.24. Answer. The function1√

x2 − 1exists only for those values of x with

x2 − 1 > 0: that is, the domain of1√

x2 − 1is |x| > 1. However, the domain of

arcsine is |x| ≤ 1. So, there is not one single value of xwhere arcsinx and1√

x2 − 1

621

Page 638: CLP-1 Differential Calculus

ANSWERS TO EXERCISES

are both defined.If the derivative of arcsin(x) were given by

1√x2 − 1

, then the derivative of

arcsin(x) would not exist anywhere, so we would probably just write “deriva-tive does not exist,” instead of making up a function with a mismatched domain.Also, the function f(x) = arcsin(x) is a smooth curve–its derivative exists at ev-ery point strictly inside its domain. (Remember not all curves are like this: forinstance, g(x) = |x| does not have a derivative at x = 0, but x = 0 is strictly inside

its domain.) So, it’s a pretty good bet that the derivative of arcsine is not1√

x2 − 1.

2.12.2.25. Answer.1

2

2.12.2.26. Answer. f−1(7) = −25

4

2.12.2.27. Answer. f(0) = −7

2.12.2.28. Answer. y′ =2x√

1− (x+ 2y)2 − 1

2− 2y√

1− (x+ 2y)2, or equivalently, y′ =

2x cos(x2 + y2)− 1

2− 2y cos(x2 + y2)

2.13 · The Mean Value Theorem2.13.5 · Exercises

Exercises — Stage 12.13.5.1. Answer. The caribou spent at least about 71 and a half hours travellingduring its migration (probably much more) in one year.

2.13.5.2. Answer. At some point during the day, the crane was travelling atexactly 10 kph.

2.13.5.3. Answer. One possible answer:

622

Page 639: CLP-1 Differential Calculus

ANSWERS TO EXERCISES

x

y

a bc

Another possible answer:

x

y

a bc

2.13.5.4. Answer. One possible answer: f(x) =

0 x 6= 10

10 x = 10

Another answer: f(x) =

10 x 6= 0

0 x = 0

Yet another answer: f(x) =

5 x 6= 0, 10

10 x = 10

0 x = 0

2.13.5.5. Answer. (a) No such function is possible: Rolle’s Theorem guarantees

623

Page 640: CLP-1 Differential Calculus

ANSWERS TO EXERCISES

f ′(c) = 0 for at least one point c ∈ (1, 2).For the other functions, examples are below, but many answers are possible.

x

y

1 2

(b)

x

y

1 2

(c)

x

y

1 2

(d)

2.13.5.6. Answer. The function f(x) is continuous over all real numbers, but itis only differentiable when x 6= 0. So, if we want to apply the MVT, our intervalmust consist of only positive numbers or only negative numbers: the interval(−4, 13) is not valid.It is possible to use the mean value theorem to prove what we want: if a = 1and b = 144, then f(x) is differentiable over the interval (1, 144) (since 0 is notcontained in that interval), and f(x) is continuous everywhere, so by the mean

value theorem there exists some point c where f ′(x) =

√|144| −

√|1|

144− 1=

11

143=

624

Page 641: CLP-1 Differential Calculus

ANSWERS TO EXERCISES

1

13.

That being said, an easier way to prove that a point exists is to simply find

it–without using the MVT. When x > 0, f(x) =√x, so f ′(x) =

1

2√x

. Then

f ′(

169

4

)=

1

13.

Exercises — Stage 22.13.5.7. *. Answer. We note that f(0) = f(2π) = 0. Then using the MeanValue Theorem (note that the function is differentiable for all real numbers), weconclude that there exists c in (0, 2π) such that

f ′(c) =f(2π)− f(0)

2π − 0= 0.

2.13.5.8. *. Answer. We note that f(0) = f(1) = 0. Then using the Mean ValueTheorem (note that the function is differentiable for all real numbers), we get thatthere exists c ∈ (0, 1) such that

f ′(c) =f(1)− f(0)

1− 0= 0.

2.13.5.9. *. Answer. We note that f(0) = f(2π) =√

3+π2. Then using the MeanValue Theorem (note that the function is differentiable for all real numbers since3 + sin x > 0), we get that there exists c ∈ (0, 2π) such that

f ′(c) =f(2π)− f(0)

2π − 0= 0.

2.13.5.10. *. Answer. We note that f(0) = 0 and f(π/4) = 0. Then using theMean Value Theorem (note that the function is differentiable for all real num-bers), we get that there exists c ∈ (0, π/4) such that

f ′(c) =f(π/4)− f(0)

π/4− 0= 0.

2.13.5.11. Answer. 1

2.13.5.12. Answer. 2

2.13.5.13. Answer. 1

2.13.5.14. Answer. 1

2.13.5.15. *. Answer. 2.13.5.15.a

f ′(x) = 15x4 − 30x2 + 15 = 15(x4 − 2x2 + 1

)= 15

(x2 − 1

)2 ≥ 0

The derivative is nonnegative everywhere. The only values of x for which f ′(x) =0 are 1 and −1, so f ′(x) > 0 for every x in (−1, 1).2.13.5.15.b If f(x) has two roots a and b in [−1, 1], then by Rolle’s Theorem, f ′(c) =

625

Page 642: CLP-1 Differential Calculus

ANSWERS TO EXERCISES

0 for some x strictly between a and b. But since a and b are in [−1, 1], and c isbetween a and b, that means c is in (−1, 1); however, we know for every c in(−1, 1), f ′(c) > 0, so this can’t happen. Therefore, f(x) does not have two roots aand b in [−1, 1]. This means f(x) has at most one root in [−1, 1].

2.13.5.16. *. Answer. log

(eT − 1

T

)2.13.5.17. Answer. See the solution for the argument that arcsecx = C−arccscxfor some constant C.The constant C =

π

2.

Exercises — Stage 32.13.5.18. *. Answer. Since e−f(x) is always positive (regardless of the value off(x)),

f ′(x) =1

1 + e−f(x)<

1

1 + 0= 1

for every x.Since f ′(x) exists for every x, we see that f is differentiable, so the Mean ValueTheorem applies. If f(100) is greater than or equal to 100, then by the Mean ValueTheorem, there would have to be some c between 0 and 100 such that

f ′(c) =f(100)− f(0)

100≥ 100

100= 1

Since f ′(x) ≤ 1 for every x, there is no value of c as described. Therefore, it is notpossible that f(100) ≥ 100. So, f(100) < 100.

2.13.5.19. Answer. Domain of f−1(x): (−∞,∞) Interval where f is one–to–one,and range of f−1(x): (−∞,∞)

2.13.5.20. Answer. One–to–one interval, and range of f−1:[−2π

3, 2π

3

]Domain of

f−1:[−(π3

+√

32

),(π3

+√

32

)]2.13.5.21. Answer. Define h(x) = f(x)− g(x), and notice h(a) = f(a)− g(a) < 0and h(b) = f(b) − g(b) > 0. Since h is the difference of two functions that arecontinuous over [a, b] and differentiable over (a, b), also h is continuous over [a, b]and differentiable over (a, b). So, by the Mean Value Theorem, there exists somec ∈ (a, b) with

h′(c) =h(b)− h(a)

b− aSince (a, b) is an interval, b > a, so the denominator of the above expression ispositive; since h(b) > 0 > h(a), also the numerator of the above expression ispositive. So, h′(c) > 0 for some c ∈ (a, b). Since h′(c) = f ′(c)− g′(c), we concludef ′(c) > g′(c) for some c ∈ (a, b).

2.13.5.22. Answer. 3

626

Page 643: CLP-1 Differential Calculus

ANSWERS TO EXERCISES

2.13.5.23. Answer. 2

2.14 · Higher Order Derivatives2.14.2 · Exercises

Exercises — Stage 12.14.2.1. Answer. ex

2.14.2.2. Answer. 2.14.2.2.ii, 2.14.2.2.iv

2.14.2.3. Answer.3

15!

2.14.2.4. Answer. The derivativedy

dxis

11

4only at the point (1, 3): it is not con-

stantly11

4, so it is wrong to differentiate the constant

11

4to find

d2y

dx2. Below is a

correct solution.

−28x+ 2y + 2xy′ + 2yy′ = 0

Plugging in x = 1, y = 3:

−28 + 6 + 2y′ + 6y′ = 0

y′ =11

4at the point (1, 3)

Differentiating the equation −28x+ 2y + 2xy′ + 2yy′ = 0:

−28 + 2y′ + 2y′ + 2xy′′ + 2y′y′ + 2yy′′ = 0

4y′ + 2(y′)2 + 2xy′′ + 2yy′′ = 28

At the point (1, 3), y′ =11

4. Plugging in:

4

(11

4

)+ 2

(11

4

)2

+ 2(1)y′′ + 2(3)y′′ = 28

y′′ =15

64

Exercises — Stage 2

2.14.2.5. Answer. f ′′(x) =1

x

2.14.2.6. Answer.d2

dx2arctanx =

−2x

(1 + x2)2

627

Page 644: CLP-1 Differential Calculus

ANSWERS TO EXERCISES

2.14.2.7. Answer.d2y

dx2=−1

y3

2.14.2.8. Answer. 0

2.14.2.9. Answer.d3

dx3log(5x2 − 12) =

100x(5x2 + 36)

(5x2 − 12)3

2.14.2.10. Answer. speeding up

2.14.2.11. Answer. slower

2.14.2.12. Answer. −4

2.14.2.13. Answer. (a) true(b) true(c) false

Exercises — Stage 32.14.2.14. Answer. (ii)

2.14.2.15. Answer. f (n) = 2x(log 2)n

2.14.2.16. Answer. n = 4

2.14.2.17. *. Answer.

• 2.14.2.17.a f ′(x) = (1 + 2x)ex+x2 f ′′(x) = (4x2 + 4x + 3)ex+x2 h′(x) = 1 + 3xh′′(x) = 3

• 2.14.2.17.b f(0) = h(0) = 1; f ′(0) = h′(0) = 1; f ′′(0) = h′′(0) = 3

• 2.14.2.17.c f and h “start at the same place”, since f(0) = h(0). Also f ′(0) =h′(0), and f ′′(x) = (4x2 + 4x + 3)ex+x2 > 3ex+x2 > 3 = h′′(x) when x > 0.Since f ′(0) = h′(0), and since f ′ grows faster than h′ for positive x, weconclude f ′(x) > h′(x) for all positive x. Now we can conclude that (sincef(0) = h(0) and f grows faster than h when x > 0) also f(x) > h(x) for allpositive x.

2.14.2.18. *. Answer.

a y′(1) =4

13

b

628

Page 645: CLP-1 Differential Calculus

ANSWERS TO EXERCISES

2.14.2.19. Answer.

• 2.14.2.19.a g′′(x) = [f(x) + 2f ′(x) + f ′′(x)]ex

• 2.14.2.19.b g′′′(x) = [f(x) + 3f ′(x) + 3f ′′(x) + f ′′′(x)]ex

• 2.14.2.19.c g(4)(x) = [f(x) + 4f ′(x) + 6f ′′(x) + 4f ′′′(x) + f (4)(x)]ex

2.14.2.20. Answer. m+ n

2.14.2.21. Answer. 2

2.14.2.22. *. Answer. 2.14.2.22.a In order to make f(x) a little more tractable,

let’s change the format. Since |x| =

x x ≥ 0

−x x < 0, then:

f(x) =

−x2 x < 0

x2 x ≥ 0.

Now, we turn to the definition of the derivative to figure out whether f ′(0) exists.

f ′(0) = limh→0

f(0 + h)− f(0)

h= lim

h→0

f(h)− 0

h= lim

h→0

f(h)

hif it exists.

Since f looks different to the left and right of 0, in order to evaluate this limit, welook at the corresponding one-sided limits. Note that when h approaches 0 fromthe right, h > 0 so f(h) = h2. By contrast, when h approaches 0 from the left,h < 0 so f(h) = −h2.

limh→0+

f(h)

h= lim

h→0+

h2

h= lim

h→0+h = 0

limh→0−

f(h)

h= lim

h→0−

−h2

h= lim

h→0−−h = 0

Since both one-sided limits exist and are equal to 0,

limh→0

f(0 + h)− f(0)

h= 0

and so f is differentiable at x = 0 and f ′(0) = 0.2.14.2.22.b From 2.14.2.22.a, f ′(0) = 0 and

f(x) =

−x2 x < 0

x2 x ≥ 0.

629

Page 646: CLP-1 Differential Calculus

ANSWERS TO EXERCISES

So,

f ′(x) =

−2x x < 0

2x x ≥ 0.

Then, we know the second derivative of f everywhere except at x = 0:

f ′′(x) =

−2 x < 0

?? x = 0

2 x > 0.

So, whenever x 6= 0, f ′′(x) exists. To investigate the differentiability of f ′(x) whenx = 0, again we turn to the definition of a derivative. If

limh→0

f ′(0 + h)− f ′(0)

h

exists, then f ′′(0) exists.

limh→0

f ′(0 + h)− f ′(0)

h= lim

h→0

f ′(h)− 0

h= lim

h→0

f ′(h)

h

Since f(h) behaves differently when h is greater than or less than zero, we lookat the one-sided limits.

limh→0+

f ′(h)

h= lim

h→0+

2h

h= 2

limh→0−

f ′(h)

h= lim

h→0−

−2h

h= −2

Since the one-sided limits do not agree,

limh→0

f ′(0 + h)− f ′(0)

h= DNE

So, f ′′(0) does not exist. Now we have a complete picture of f ′′(x):

f ′′(x) =

−2 x < 0

DNE x = 0

2 x > 0.

3 · Applications of derivatives3.1 · Velocity and Acceleration3.1.2 · Exercises

Exercises — Stage 13.1.2.1. Answer. False (but its velocity is zero)

630

Page 647: CLP-1 Differential Calculus

ANSWERS TO EXERCISES

3.1.2.2. Answer. It takes 10 seconds to accelerate from 2 ms

to 3 ms

, and 100seconds to accelerate from 3 m

sto 13 m

s.

3.1.2.3. Answer. In general, false.

3.1.2.4. Answer. True

Exercises — Stage 23.1.2.5. Answer. The pot is falling at 14 metres per second, just as it hits theground.

3.1.2.6. Answer. (a) 4.9x2 metres(b) 4.9x2 + x metres

3.1.2.7. Answer. 8− 4√

3 ≈ 1 sec

3.1.2.8. Answer. 7.2 sec

3.1.2.9. Answer. 72 000 kph per hour

3.1.2.10. Answer.100

7≈ 14 kph

3.1.2.11. Answer. about 1240 miles

3.1.2.12. Answer. 49 metres per second

3.1.2.13. Answer. About 416 metres

3.1.2.14. Answer. v0 =√

1960 ≈ 44 metres per second

3.1.2.15. Answer. ≈ 74.2 kph

Exercises — Stage 3

3.1.2.16. Answer. Time elapsed:1

4.9+

1

9.8≈ 0.3 seconds

3.1.2.17. Answer. The acceleration is given by 2tv0 log 2, where v0 is the velocityof the object at time t = 0.

3.2 · Related Rates3.2.2 · Exercises

Exercises — Stage 13.2.2.1. Answer. ii and iv

Exercises — Stage 2

3.2.2.2. *. Answer. −3

2

3.2.2.3. *. Answer. 6%

631

Page 648: CLP-1 Differential Calculus

ANSWERS TO EXERCISES

3.2.2.4. *. Answer. 3.2.2.4.a 03.2.2.4.b 100

F ′

F= 15%, or F ′ = 0.15F

3.2.2.5. *. Answer. −17

5units per second

3.2.2.6. *. Answer.4

5units per second

3.2.2.7. *. Answer. increasing at 7 mph

3.2.2.8. *. Answer. 8 cm per minute

3.2.2.9. *. Answer. −13

6metres per second

3.2.2.10. Answer. The height of the water is decreasing at3

16= 0.1875 cm

min.

3.2.2.11. Answer.1

29200metres per second (or about 1 centimetre every five

minutes)

3.2.2.12. Answer.

(2(

123572

)2+ 4

)(6175

3

)≈ 13.8 rad

hour≈ 0.0038 rad

sec

3.2.2.13. *. Answer. 3.2.2.13.a24

13≈ 1.85 km/min

3.2.2.13.b about .592 radians/min

3.2.2.14. Answer.55√

21π

42≈ 19 centimetres per hour.

3.2.2.15. *. Answer.dA

dt= −2π

cm2

s

3.2.2.16. Answer. 288 cubic units per unit time

3.2.2.17. Answer. 0 square centimetres per minute

3.2.2.18. Answer. −7π

12≈ −1.8

cm3

sec2

3.2.2.19. Answer. The flow is decreasing at a rate of√

7

1000

m3

sec2.

3.2.2.20. Answer.−15

49π≈ −0.097 cm per minute

Exercises — Stage 33.2.2.21. Answer.

adD

dt=

1

2√

2metres per hour

632

Page 649: CLP-1 Differential Calculus

ANSWERS TO EXERCISES

b The river is higher than 2 metres.

c The river’s flow has reversed direction. (This can happen near an ocean athigh tide.)

3.2.2.22. Answer. (a) 2 units per second

(b) Its y-coordinate is decreasing at1

2unit per second. The point is moving at

√5

2units per second.

3.2.2.23. Answer. (a) 10π = π[3(a+ b)−

√(a+ 3b)(3a+ b)

]or equivalently,

10 = 3(a+ b)−√

(a+ 3b)(3a+ b)(b) 20πab(c) The water is spilling out at about 375.4 cubic centimetres per second. The

exact amount is − 200π

9−√

35

(1− 2

(3√

35− 11

3√

35− 13

))cm3

sec.

3.2.2.24. Answer. B(10) = 0

3.3 · Exponential Growth and Decay — a First Look atDifferential Equations3.3.4 · Exercises· Exercises for § 3.3.1

Exercises — Stage 13.3.4.1. Answer. (a), (b)

3.3.4.2. Answer. (a), (d)

3.3.4.3. Answer. If C = 0, then there was none to start out with, and Q(t) = 0for all values of t.If C 6= 0, thenQ(t) will never be 0 (but as t gets bigger and bigger,Q(t) gets closerand closer to 0).

Exercises — Stage 2

3.3.4.4. *. Answer. A = 5, k =1

7· log (π/5)

3.3.4.5. *. Answer. y(t) = 2e−3(t−1), or equivalently, y(t) = 2e3e−3t

3.3.4.6. Answer. 5 · 2−100005730 ≈ 1.5 µg

3.3.4.7. Answer. Radium-226 has a half life of about 1600 years.

3.3.4.8. *. Answer.log 2

log 6= log6(2) years, which is about 139 days

633

Page 650: CLP-1 Differential Calculus

ANSWERS TO EXERCISES

3.3.4.9. Answer. 120 · log 10

log 2seconds, or about six and a half minutes.

Exercises — Stage 33.3.4.10. Answer. About 0.5% of the sample decays in a day. The exact amount

is[100

(1− 2−

1138

)]%.

3.3.4.11. Answer. After ten years, the sample contains between 6.2 and 6.8 µgof Uranium-232.

· Exercises for § 3.3.2

Exercises — Stage 13.3.4.1. Answer. (a), (c), (d)

3.3.4.2. Answer. The temperature of the room is -10 degrees, and the room iscolder than the object.

3.3.4.3. Answer. K is a negative number. It cannot be positive or zero.

3.3.4.4. Answer. If the object has a different initial temperature than its sur-roundings, then T (t) is never equal to A. (But as time goes on, it gets closer andcloser.)If the object starts out with the same temperature as its surrounding, then T (t) =A for all values of t.

Exercises — Stage 2

3.3.4.5. Answer.−10 log(750)

log(

215

) ≈ 32.9 seconds

3.3.4.6. Answer. 10log(10)

log(5)≈ 14.3 minutes

Exercises — Stage 33.3.4.7. *. Answer. If Newton adds his cream just before drinking, the coffeeends up cooler by 0.85 C.

3.3.4.8. *. Answer.

adT

dt=

1

5log

(4

5

)(T − 30)

b5 log(2/5)

log(4/5)≈ 20.53 min

634

Page 651: CLP-1 Differential Calculus

ANSWERS TO EXERCISES

3.3.4.9. Answer. positive

· Exercises for § 3.3.3

Exercises — Stage 13.3.4.1. Answer. If P (0) = 0, yes. If P (0) 6= 0, no: it does not take into accountexternal constraints on population growth.

Exercises — Stage 23.3.4.2. Answer. The Malthusian model predicts the herd will number 217 in-dividuals in 2020.

3.3.4.3. Answer.log(3)

log(2)≈ 1.6 hours

3.3.4.4. Answer. 1912 or 1913

3.3.4.5. Answer.106

54 − 1≈ 1603

Exercises — Stage 33.3.4.6. Answer. (a) At t = 0, there are 100 units of the radioactive isotope inthe sample. k is negative.(b) At t = 0, there are 100 individuals in the population. k is positive.(c) The ambient temperature is 0 degrees. k is negative.

· Further problems for § 3.33.3.4.1. *. Answer. f(2) = 2e2π

3.3.4.2. Answer. Solutions to the differential equation have the form

T (t) =

[T (0) +

9

7

]e7t − 9

7

for some constant T (0).

3.3.4.3. *. Answer.8 log(0.4)

log(0.8)≈ 32.85 days

3.3.4.4. Answer. 25 C

3.3.4.5. *. Answer.

a A(t) = 90,000·e0.05t−40,000. When the graduate is 65, they will have $625,015.05in the account.

b $49,437.96

635

Page 652: CLP-1 Differential Calculus

ANSWERS TO EXERCISES

3.3.4.6. *. Answer. 3.3.4.6.a A(t) = 150,000− 30,000 e0.06t

3.3.4.6.b after 26.8 yrs

3.3.4.7. *. Answer.9 log 2

log 3≈ 5.68 hr

3.3.4.8. *. Answer. (a) v(t) =[v0 + g

k

]e−kt − g

k

(b) limt→∞

v(t) = −gk

3.4 · Approximating Functions Near a Specified Point —Taylor Polynomials3.4.11 · Exercises· Exercises for § 3.4.1

Exercises — Stage 13.4.11.1. Answer. Since f(0) is closer to g(0) than it is to h(0), you would proba-bly want to estimate f(0) ≈ g(0) = 1 + 2 sin(1) if you had the means to efficientlyfigure out what sin(1) is, and if you were concerned with accuracy. If you hada calculator, you could use this estimation. Also, later in this chapter we willlearn methods of approximating sin(1) that do not require a calculator, but theydo require time.Without a calculator, or without a lot of time, using f(0) ≈ h(0) = 0.7 probablymakes the most sense. It isn’t as accurate as f(0) ≈ g(0), but you get an estimatevery quickly, without worrying about figuring out what sin(1) is.

Exercises — Stage 23.4.11.2. Answer. log(0.93) ≈ log(1) = 0

x

y

y = f(x)

y = 0

0.93

1

3.4.11.3. Answer. arcsin(0.1) ≈ 0

3.4.11.4. Answer.√

3 tan(1) ≈ 3

Exercises — Stage 33.4.11.5. Answer. 10.13 ≈ 103 = 1000

· Exercises for § 3.4.2

636

Page 653: CLP-1 Differential Calculus

ANSWERS TO EXERCISES

Exercises — Stage 13.4.11.1. Answer. (a) f(5) = 6(b) f ′(5) = 3(c) not enough information to know

3.4.11.2. Answer.

x

y

y = f(x)

2

The linear approximation is shown in red.

3.4.11.3. Answer. f(x) = 2x+ 5

Exercises — Stage 23.4.11.4. Answer. log(0.93) ≈ −0.07

x

y

y = f(x)

y = x− 1

1

3.4.11.5. Answer.√

5 ≈ 9

4

3.4.11.6. Answer. 5√

30 ≈ 79

40

Exercises — Stage 33.4.11.7. Answer. 10.13 ≈ 1030, 10.13 = 1030.301

3.4.11.8. Answer. There are many possible answers. One is f(x) = sin x, a = 0,and b = π.

3.4.11.9. Answer. a =√

3

637

Page 654: CLP-1 Differential Calculus

ANSWERS TO EXERCISES

· Exercises for § 3.4.3

Exercises — Stage 13.4.11.1. Answer. f(3) = 9, f ′(3) = 0, f ′′(3) = −2; there is not enough informa-tion to know f ′′′(3).

3.4.11.2. Answer. f(x) ≈ 2x+ 5

Exercises — Stage 23.4.11.3. Answer. log(0.93) ≈ −0.07245

3.4.11.4. Answer. cos

(1

15

)≈ 449

450

3.4.11.5. Answer. e2x ≈ 1 + 2x+ 2x2

3.4.11.6. Answer. One approximation: e43 ≈ 275

32

3.4.11.7. Answer. 3.4.11.7.a 263.4.11.7.b 16

3.4.11.7.c10

11

3.4.11.7.d75

64

3.4.11.8. Answer. For each of these, there are many solutions. We provide somebelow.

a 1 + 2 + 3 + 4 + 5 =5∑

n=1

n

b 2 + 4 + 6 + 8 =4∑

n=1

2n

c 3 + 5 + 7 + 9 + 11 =5∑

n=1

(2n+ 1)

d 9 + 16 + 25 + 36 + 49 =7∑

n=3

n2

e 9 + 4 + 16 + 5 + 25 + 6 + 36 + 7 + 49 + 8 =7∑

n=3

(n2 + n+ 1)

f 8 + 15 + 24 + 35 + 48 =7∑

n=3

(n2 − 1)

638

Page 655: CLP-1 Differential Calculus

ANSWERS TO EXERCISES

g 3− 6 + 9− 12 + 15− 18 =6∑

n=1

(−1)n+13n

Exercises — Stage 33.4.11.9. Answer. f(1) ≈ 2, f(1) = π

3.4.11.10. Answer. e ≈ 2.5

3.4.11.11. Answer.

• [3.4.11.11.a=3.4.11.11.d= 3.4.11.11.e], and

• [3.4.11.11.b=3.4.11.11.g], and

• [3.4.11.11.c=3.4.11.11.f]

· Exercises for § 3.4.4

Exercises — Stage 13.4.11.1. Answer. f ′′(1) = −4

3.4.11.2. Answer. f (10)(5) = 10!

Exercises — Stage 33.4.11.3. Answer. T3(x) = −x3 + x2 − x+ 1

3.4.11.4. Answer. T3(x) = −7 + 7(x− 1) + 9(x− 1)2 + 5(x− 1)3, or equivalently,T3(x) = 5x3 − 6x2 + 4x− 10

3.4.11.5. Answer. f (10)(5) =11 · 10!

6

3.4.11.6. Answer. a =√e

· Exercises for § 3.4.5

Exercises — Stage 13.4.11.1. Answer.

T16(x) = 1+x−1

2x2− 1

3!x3+

1

4!x4+

1

5!x5− 1

6!x6− 1

7!x7+

1

8!x8+

1

9!x9

− 1

10!x10− 1

11!x11+

1

12!x12+

1

13!x13− 1

14!x14− 1

15!x15

+1

16!x16

3.4.11.2. Answer. T100(t) = 4.9t2 − t+ 10

639

Page 656: CLP-1 Differential Calculus

ANSWERS TO EXERCISES

3.4.11.3. Answer. Tn(x) =n∑k=0

2(log 2)k

k!(x− 1)k

3.4.11.4. Answer.

T6(x) = 7 + 5(x− 1) +7

2(x− 1)2 +

1

3(x− 1)3 − 1

12(x− 1)4

+1

30(x− 1)5 − 1

60(x− 1)6

3.4.11.5. Answer. Tn(x) =n∑k=0

xk

Exercises — Stage 33.4.11.6. Answer. T3(x) = 1 + (x− 1) + (x− 1)2 + 1

2(x− 1)3

3.4.11.7. Answer. π = 6 arctan

(1√3

)≈ 82

45

√3 ≈ 3.156

3.4.11.8. Answer. T100(x) = −1 +100∑k=2

(−1)k

k(k − 1)(x− 1)k

3.4.11.9. Answer. T2n(x) =n∑k=0

[(−1)k

(2k)!√

2

(x− π

4

)2k

+(−1)k

(2k + 1)!√

2

(x− π

4

)2k+1]

3.4.11.10. Answer.1 +

1

2+

1

3!+

1

4!+ · · ·+ 1

157!≈ e

3.4.11.11. Answer. We estimate that the sum is close to − 1√2

.

· Exercises for § 3.4.6

Exercises — Stage 13.4.11.1. Answer.

640

Page 657: CLP-1 Differential Calculus

ANSWERS TO EXERCISES

x

y

y = f(x)

x x+ ∆x

f(x)

f (x+ ∆x)

∆x

∆y

3.4.11.2. Answer. Let f(x) be the number of problems finished after x minutesof work. The question tells us that 5∆y ≈ ∆x. So, if ∆x = 15, ∆y ≈ 3. That is, in15 minutes more, you will finish about 3 more problems.

Exercises — Stage 2

3.4.11.3. Answer. (a) ∆y ≈ 1

260≈ 0.003846

(b) ∆y ≈ 51

13520≈ 0.003772

3.4.11.4. Answer. (a) ∆y ≈ 1.1 metres per second(b) The increase from the first to the second jump is bigger.

· Exercises for § 3.4.7

Exercises — Stage 13.4.11.1. Answer. False.

3.4.11.2. Answer. Absolute error: 0.17; percentage error: 2.92%

3.4.11.3. Answer. The linear approximates estimates the error in f(x) to beabout 60, while the quadratic approximates estimates the error in f(x) to be about63.

Exercises — Stage 23.4.11.4. Answer. 1%

3.4.11.5. Answer. (a)9

(b) θ = 2 arcsin

(d

6

)(c)

9√36− 0.682

· 0.02 ≈ 0.03

641

Page 658: CLP-1 Differential Calculus

ANSWERS TO EXERCISES

3.4.11.6. Answer. We estimate the volume decreased by about 0.00078 cubicmetres, or about 780 cubic centimetres.

Exercises — Stage 33.4.11.7. Answer. Correct to within about 10.4 years (or about 53%)

· Exercises for § 3.4.8

Exercises — Stage 13.4.11.1. Answer. (a) False(b) True(c) True(d) True

3.4.11.2. Answer. Equation 3.4.33 gives us the bound |f(2) − T3(2)| < 6. Acalculator tells us actually |f(2)− T3(2)| ≈ 1.056.

3.4.11.3. Answer. |f(37)− T (37)| = 0

3.4.11.4. Answer. You do, you clever goose!

Exercises — Stage 2

3.4.11.5. Answer. |f(11.5)− T5(11.5)| < 9

7 · 26< 0.02

3.4.11.6. Answer. |f(0.1)− T2(0.1)| < 1

1125

3.4.11.7. Answer.∣∣∣∣f (−1

4

)− T5

(−1

4

)∣∣∣∣ < 1

6 · 46< 0.00004

3.4.11.8. Answer. Your answer may vary. One reasonable answer is

|f(30)− T3(30)| < 14

57 · 9 · 15< 0.000002.

Another reasonable answer is |f(30)− T3(30)| < 14

57 · 9 < 0.00002.

3.4.11.9. Answer. Equation 3.4.33 gives the bound |f(0.01) − Tn(0.01)| ≤1002

(100π− 1)2.

A more reasonable bound on the error is that it is less than 5.

3.4.11.10. Answer. Using Equation 3.4.33,∣∣∣∣f (1

2

)− T2

(1

2

)∣∣∣∣ < 1

10.

The actual error is ∣∣∣∣f (1

2

)− T2

(1

2

)∣∣∣∣ =π

6− 1

2

which is about 0.02.

642

Page 659: CLP-1 Differential Calculus

ANSWERS TO EXERCISES

Exercises — Stage 33.4.11.11. Answer. Any n greater than or equal to 3.

3.4.11.12. Answer. 7√

2200 ≈ 3 +13

7 · 36≈ 3.00255

3.4.11.13. Answer. If we’re going to use Equation 3.4.33, then we’ll probablybe taking a Taylor polynomial. Using Example 3.4.16, the 6th-degree Maclaurinpolynomial for sinx is

T6(x) = T5(x) = x− x3

3!+x5

5!

so let’s play with this a bit. Equation 3.4.33 tells us that the error will depend onthe seventh derivative of f(x), which is − cosx:

f(1)− T6(1) = f (7)(c)17

7!

sin(1)−(

1− 1

3!+

1

5!

)=− cos c

7!

sin(1)− 101

5!=− cos c

7!

sin(1) =4242− cos c

7!

for some c between 0 and 1. Since −1 ≤ cos c ≤ 1,

4242− 1

7!≤ sin(1) ≤ 4242 + 1

7!4241

7!≤ sin(1) ≤ 4243

7!4241

5040≤ sin(1) ≤ 4243

5040

Remark: there are lots of ways to play with this idea to get better estimates. Oneway is to take a higher-degree Maclaurin polynomial. Another is to note that,

since 0 < c < 1 <π

3, then

1

2< cos c < 1, so

4242− 1

7!< sin(1) <

4242− 12

7!4241

5040< sin(1) <

8483

10080<

4243

5040

If you got tighter bounds than asked for in the problem, congratulations!

3.4.11.14. Answer. (a) T4(x) =∑4

n=0xn

n!

(b) T4(1) = 6524

(c) See the solution.

· Further problems for § 3.4

643

Page 660: CLP-1 Differential Calculus

ANSWERS TO EXERCISES

Exercises — Stage 13.4.11.1. *. Answer. f ′(0) = 0 and f ′′(0) = 6.

3.4.11.2. *. Answer. 4

3.4.11.3. *. Answer. h′(2) =1

2, h′′(2) = 0

Exercises — Stage 23.4.11.4. *. Answer. (a) 1.92(b) 1.918

3.4.11.5. *. Answer. 101/3 ≈ 13

6; this approximation is too big.

3.4.11.6. *. Answer.√

2 ≈ 3

2

3.4.11.7. *. Answer. 3√

26 ≈ 80

27

3.4.11.8. *. Answer. (10.1)5 ≈ 105, 000

3.4.11.9. *. Answer. sin

(101π

100

)≈ − π

100

3.4.11.10. *. Answer. arctan(1.1) ≈(π

4+

1

20

)3.4.11.11. *. Answer.

8012

1000

3.4.11.12. *. Answer. (8.06)2/3 ≈ 402

100=

201

50

3.4.11.13. *. Answer. 1 + x+ 2x2 +14

3x3

3.4.11.14. *. Answer.

• By Equation 3.4.33, the absolute value of the error is∣∣∣∣f ′′′(c)3!· (2− 1)3

∣∣∣∣ =

∣∣∣∣ c

6(22− c2)

∣∣∣∣for some c ∈ (1, 2).

• When 1 ≤ c ≤ 2, we know that 18 ≤ 22 − c2 ≤ 21, and that numerator anddenominator are non-negative, so∣∣∣∣ c

6(22− c2)

∣∣∣∣ =c

6(22− c2)≤ 2

6(22− c2)≤ 2

6 · 18

=1

54≤ 1

50

644

Page 661: CLP-1 Differential Calculus

ANSWERS TO EXERCISES

as required.

• Alternatively, notice that c is an increasing function of c, while 22 − c2 is adecreasing function of c. Hence the fraction is an increasing function of cand takes its largest value at c = 2. Hence∣∣∣∣ c

6(22− c2)

∣∣∣∣ ≤ 2

6× 18=

1

54≤ 1

50.

3.4.11.15. *. Answer.

• By Equation 3.4.33, there is c ∈ (0, 0.5) such that the error is

R4 =f (4)(c)

4!(0.5− 0)4

=1

24 · 16· cos(c2)

3− c

• For any c we have | cos(c2)| ≤ 1, and for c < 0.5 we have 3− c > 2.5, so that∣∣∣∣cos(c2)

3− c

∣∣∣∣ ≤ 1

2.5.

• We conclude that

|R4| ≤1

2.5 · 24 · 16=

1

60 · 16<

1

60 · 10=

1

600<

1

500

3.4.11.16. *. Answer.

• By Equation 3.4.33, there is c ∈ (0, 1) such that the error is∣∣∣∣f ′′′(c)3!· (1− 0)3

∣∣∣∣ =

∣∣∣∣ e−c

6(8 + c2)

∣∣∣∣ .• When 0 < c < 1, we know that 1 > e−c > e−1 and 8 ≤ 8 + c2 < 9, so∣∣∣∣ e−c

6(8 + c2)

∣∣∣∣ =e−c

6(8 + c2)

<1

6|8 + c2|<

1

6× 8=

1

48<

1

40

as required.

3.4.11.17. *. Answer. 3.4.11.17.a 2.92593.4.11.17.b 2.92413.4.11.17.c 4

925−5/3

645

Page 662: CLP-1 Differential Calculus

ANSWERS TO EXERCISES

Exercises — Stage 33.4.11.18. Answer. T3(x) = 5x2 − 9

3.4.11.19. *. Answer. (a) 1.05(b) 1.0483

3.4.11.19.c

3.4.11.20. *. Answer. 3.4.11.20.a 0.93.4.11.20.b 0.8867

3.4.11.20.c

3.4.11.21. *. Answer. log 10.3 ≈ 2.33259 The error is between −0.00045 and−0.00042.

3.4.11.22. *. Answer. (a) L(x) = e+ ex(b) Q(x) = e+ ex+ ex2

(c) Since ex2 > 0 for all x > 0, L(x) < Q(x) for all x > 0.From the error formula, we know that

f(x) = f(0) + f ′(0)x+1

2f ′′(0)x2 +

1

3!f ′′′(c)x3

= Q(x) +1

6

(ec + 3e2c + e3c

)eec

x3

for some c between 0 and x. Since 16

(ec + 3e2c + e3c) eec is positive for any c, for

all x > 0, 16

(ec + 3e2c + e3c) eecx3 > 0, so Q(x) < f(x).

(d) 1.105 < e0.1 < 1.115

3.5 · Optimisation3.5.4 · Exercises· Exercises for § 3.5.1

646

Page 663: CLP-1 Differential Calculus

ANSWERS TO EXERCISES

Exercises — Stage 13.5.4.1. Answer.

x

y

y = f(x)

There is a critical point at x = 0. The x-value of the red dot is a singular point,and a local maximum occurs there.

3.5.4.2. Answer.

x

y

y = f(x)

a b

The x-coordinate corresponding to the blue dot (let’s call it a) is a critical point,and f(x) has a local and global minimum at x = a. The x-coordinate correspond-ing to the discontinuity (let’s call it b) is a singular point, but there is no a globalor local extremum at x = b.

3.5.4.3. Answer. One possible answer is shown below.

647

Page 664: CLP-1 Differential Calculus

ANSWERS TO EXERCISES

x

y

2

Exercises — Stage 23.5.4.4. Answer. The critical points are x = 5+

√15 and x = 5−

√15. These two

points are the only places where local extrema might exist. There are no singularpoints.

Exercises — Stage 33.5.4.5. Answer.

x

y

2

local max

x

y

2

neither

x

y

2

neither

x

y

2

local max

3.5.4.6. Answer. There are many possible answers. Every answer must havex = 2 as a singular point strictly inside the domain of f(x). Two possibilities areshown below.

648

Page 665: CLP-1 Differential Calculus

ANSWERS TO EXERCISES

x

y

2x

y

2

3.5.4.7. Answer. x = −7, x = −1, and x = 5

3.5.4.8. Answer. Every real number c is a critical point of f(x), and f(x) has alocal and global maximum and minimum at x = c. There are no singular points.

· Exercises for § 3.5.2

Exercises — Stage 13.5.4.1. Answer. Two examples are given below, but many are possible.

x

y

y = f(x)

x

y

y = f(x)

3.5.4.2. Answer. Two examples are given below, but many are possible.

649

Page 666: CLP-1 Differential Calculus

ANSWERS TO EXERCISES

x

y

y = ex

x

y

y = arctanx+ 2

3.5.4.3. Answer. One possible answer:

x

y

y = f(x)

−5 5

Exercises — Stage 2

650

Page 667: CLP-1 Differential Calculus

ANSWERS TO EXERCISES

3.5.4.4. Answer. The global maximum is 45 at x = 5 and the global minimumis −19 at x = −3.

3.5.4.5. Answer. The global maximum over the interval is 61 at x = −3, and theglobal minimum is 7 at x = 0.

· Exercises for § 3.5.3

Exercises — Stage 13.5.4.1. *. Answer. The global maximum is f(−1) = 6, the global minimum isf(−2) = −20.

3.5.4.2. *. Answer. Global maximum is f(2) = 12, global minimum is f(1) =−14.

3.5.4.3. *. Answer. Global maximum is f(4) = 30, global minimum is f(2) =−10.

3.5.4.4. *. Answer. Local max at (−2, 20), local min at (2,−12).

3.5.4.5. *. Answer. (−2, 33) max, and (2,−31) min

3.5.4.6. *. Answer. Q should be 4√

3 kilometres from A

3.5.4.7. *. Answer. 10× 30× 15

3.5.4.8. *. Answer. 2× 2× 6

3.5.4.9. *. Answer. X = Y =√

2

3.5.4.10. *. Answer. The largest possible perimeter is 2√

5R and the smallestpossible perimeter is 2R.

3.5.4.11. *. Answer.A3/2

3√

3.5.4.12. *. Answer.P 2

2(π + 4)

3.5.4.13. *. Answer. (a) x =

√A

3p, y =

√Ap

3, and z =

√Ap√

3(1 + p)(b) p = 1

(The dimensions of the resulting baking pan are x = y =

√A

3and z =

1

2

√A

3.)

Exercises — Stage 33.5.4.14. *. Answer. 3.5.4.14.a xx(1 + log x)

3.5.4.14.b x =1

e3.5.4.14.c local minimum

651

Page 668: CLP-1 Differential Calculus

ANSWERS TO EXERCISES

3.5.4.15. *. Answer. Maximum area: do not cut, make a circle and no square.

Minimum area: make a square out of a piece that is4

4 + πof the total length of

the wire.

3.6 · Sketching Graphs3.6.7 · Exercises· Exercises for § 3.6.1

Exercises — Stage 13.6.7.1. Answer. In general, false.

Exercises — Stage 23.6.7.2. Answer.f(x) = A(x)g(x) = C(x)h(x) = B(x)k(x) = D(x)

3.6.7.3. Answer. (a) p = e2

(b) b = −e2, 1− e2

3.6.7.4. Answer. vertical asymptote at x = 3; horizontal asymptotes

limx→±∞

f(x) =2

3

3.6.7.5. Answer. horizontal asymptote y = 0 as x→ −∞; no other asymptotes

· Exercises for § 3.6.2

Exercises — Stage 13.6.7.1. Answer.

A′(x) = l(x) B′(x) = p(x) C ′(x) = n(x)

D′(x) = o(x) E ′(x) = m(x)

Exercises — Stage 23.6.7.2. *. Answer. (−2,∞)

3.6.7.3. *. Answer. (1, 4) and [1, 4) are both acceptable answers

3.6.7.4. *. Answer. (−∞, 1)

· Exercises for § 3.6.3

Exercises — Stage 1

652

Page 669: CLP-1 Differential Calculus

ANSWERS TO EXERCISES

3.6.7.1. Answer.

x

y

concave up concave down concave up

concave down

3.6.7.2. Answer.

x

y

−5 5

3.6.7.3. Answer. In general, false.

Exercises — Stage 23.6.7.4. *. Answer. x = 1, y = 11

Exercises — Stage 33.6.7.5. *. Answer. Let

g(x) = f ′′(x) = x3 + 5x− 20.

653

Page 670: CLP-1 Differential Calculus

ANSWERS TO EXERCISES

Then g′(x) = 3x2 + 5, which is always positive. That means g(x) is strictly in-creasing for all x. So, g(x) can change signs once, from negative to positive, but itcan never change back to negative. An inflection point of f(x) occurs when g(x)changes signs. So, f(x) has at most one inflection point.Since g(x) is continuous, we can apply the Intermediate Value Theorem to it.Notice g(3) > 0 while g(0) < 0. By the IVT, g(x) = 0 for at least one x ∈ (0, 3).Since g(x) is strictly increasing, at the point where g(x) = 0, g(x) changes fromnegative to positive. So, the concavity of f(x) changes. Therefore, f(x) has atleast one inflection point.Now that we’ve shown that f(x) has at most one inflection point, and at leastone inflection point, we conclude it has exactly one inflection point.

3.6.7.6. *. Answer. 3.6.7.6.a Let

g(x) = f ′(x)

Then f ′′(x) is the derivative of g(x). Since f ′′(x) > 0 for all x, g(x) = f ′(x) isstrictly increasing for all x. In other words, if y > x then g(y) > g(x).Suppose g(x) = 0. Then for every y that is larger than x, g(y) > g(x), so g(y) 6= 0.Similarly, for every y that is smaller than x, g(y) < g(x), so g(y) 6= 0. Therefore,g(x) can only be zero for at most one value of x. Since g(x) = f ′(x), that meansf(x) can have at most one critical point.Suppose f ′(c) = 0. Since f ′(x) is a strictly increasing function, f ′(x) < 0 for allx < c and f ′(x) > 0 for all x > c.

c

x < c, sof ′(x) < f ′(c) = 0

x > c, sof ′(x) > f ′(c) = 0

Then f(x) is decreasing for x < c and increasing for x > c. So f(x) > f(c) for allx 6= c.

c

f(x) decreasing, sof(x) > f(c)

f(x) increasing, sof(x) > f(c)

y = f(x)

x < c x > c

654

Page 671: CLP-1 Differential Calculus

ANSWERS TO EXERCISES

Since f(x) > f(c) for all x 6= c, so c is an absolute minimum for f(x).3.6.7.6.b We know that the maximum over an interval occurs at an endpoint, acritical point, or a singular point.

• Since f ′(x) exists everywhere, there are no singular points.

• If the maximum were achieved at a critical point, that critical point wouldhave to provide both the absolute maximum and the absolute minimum(by part (a)). So, the function would have to be a constant and consequentlycould not have a nonzero second derivative. So the maximum is not at acritical point.

That leaves only the endpoints of the interval.

3.6.7.7. Answer. If x = 3 is an inflection point, then the concavity of f(x)changes at x = 3. That is, there is some interval strictly containing 3, with end-points a and b, such that

• f ′′(a) < 0 and f ′′(x) < 0 for every x between a and 3, and

• f ′′(b) > 0 and f ′′(x) > 0 for every x between b and 3.

Since f ′′(a) < 0 and f ′′(b) > 0, and since f ′′(x) is continuous, the IntermediateValue Theorem tells us that there exists some x strictly between a and b withf ′′(x) = 0. So, we know f ′′(x) = 0 somewhere between a and b. The question is,where exactly could that be?

• f ′′(x) < 0 (and hence f ′′(x) 6= 0) for all x between a and 3

• f ′′(x) > 0 (and hence f ′′(x) 6= 0) for all x between b and 3

• So, any number between a and b that is not 3 has f ′′(x) 6= 0.

So, x = 3 is the only possible place between a and b where f ′′(x) could be zero.Therefore, f ′′(3) = 0.

· Exercises for § 3.6.4

Exercises — Stage 13.6.7.1. Answer. even

3.6.7.2. Answer. odd, periodic

3.6.7.3. Answer.

655

Page 672: CLP-1 Differential Calculus

ANSWERS TO EXERCISES

x

y

3.6.7.4. Answer.

x

y

Exercises — Stage 2

656

Page 673: CLP-1 Differential Calculus

ANSWERS TO EXERCISES

3.6.7.5. Answer. A function is even if f(−x) = f(x).

f(−x) =(−x)4 − (−x)6

e(−x)2

=x4 − x6

ex2

= f(x)

So, f(x) is even.

3.6.7.6. Answer. For any real number x, we will show that f(x) = f(x+ 4π).

f(x+ 4π) = sin(x+ 4π) + cos

(x+ 4π

2

)= sin(x+ 4π) + cos

(x2

+ 2π)

= sin(x) + cos(x

2

)= f(x)

So, f(x) is periodic.

3.6.7.7. Answer. even

3.6.7.8. Answer. none

3.6.7.9. Answer. 1

Exercises — Stage 33.6.7.10. Answer. π

· Exercises for § 3.6.6

Exercises — Stage 13.6.7.1. *. Answer. 3.6.7.1.a (−∞, 3]3.6.7.1.b f(x) in increasing on (−∞, 2) and decreasing on (2, 3). There is a localmaximum at x = 2.3.6.7.1.c f(x) is always concave down and has no inflection points.3.6.7.1.d (3, 0)

657

Page 674: CLP-1 Differential Calculus

ANSWERS TO EXERCISES

3.6.7.1.e

x

y

(3, 0)

(2.2)

3.6.7.2. *. Answer. The open dot is the inflection point, and the closed dot isthe local and global maximum.

x

y(2, 3

8

)

2 3√

203√

2

3.6.7.3. *. Answer. The open dot marks the inflection point.

658

Page 675: CLP-1 Differential Calculus

ANSWERS TO EXERCISES

x

y

−1− 3√

4 3√

2

3.6.7.4. *. Answer.

x

y

−1 1−√

3√

3

3√

32

−3√

32

3.6.7.5. *. Answer. 3.6.7.5.a One branch of the function, the exponential func-tion ex, is continuous everywhere. So f(x) is continuous for x < 0. When x ≥ 0,

f(x) =x2 + 3

3(x+ 1), which is continuous whenever x 6= −1 (so it’s continuous for all

659

Page 676: CLP-1 Differential Calculus

ANSWERS TO EXERCISES

x > 0). So, f(x) is continuous for x > 0. To see that f(x) is continuous at x = 0,we see:

limx→0−

f(x) = limx→0−

ex = 1

limx→0+

f(x) = limx→0+

x2 + 3

3(x+ 1)= 1

So, limx→0

f(x) = 1 = f(0)

Hence f(x) is continuous at x = 0, so f(x) is continuous everywhere.3.6.7.5.b

• i. f(x) is increasing for x < 0 and x > 1, decreasing for 0 < x < 1, has alocal max at (0, 1), and has a local min at

(1, 2

3

).

• ii. f(x) is concave upwards for all x 6= 0.

• iii. The x–axis is a horizontal asymptote as x→ −∞.

3.6.7.5.c

x

y

(0, 1)(1, 2

3)

3.6.7.6. *. Answer.

3.6.7.7. *. Answer. 3.6.7.7.a

• Increasing: (−1, 1), decreasing: (−∞,−1) ∪ (1,∞)

• concave up: (−√

3, 0) ∪ (√

3,∞), concave down: (−∞,−√

3) ∪ (0,√

3)

660

Page 677: CLP-1 Differential Calculus

ANSWERS TO EXERCISES

• inflection points: x = ±√

3, 0

3.6.7.7.b The local and global minimum of f(x) is at (−1, −1√e), and the local and

global maximum of f(x) is at (1, 1√e).

3.6.7.7.c In the graph below, open dots are inflection points, and solid dots areextrema.

x

y

−1 1−√

3 √3

−1√e

1√e

3.6.7.8. Answer. Local maxima occur at x = 2π3

+ 2πn for all integers n, andlocal minima occur at x = −2π

3+ 2πn for all integers n. Inflection points occur at

every integer multiple of π.

661

Page 678: CLP-1 Differential Calculus

ANSWERS TO EXERCISES

x

y

2π3

−2π3

4π3

−4π3

8π3

−8π3

10π3

−10π3

14π3

−14π3

3.6.7.9. *. Answer. Below is the graph y = f(x) over the interval [−π, π]. Thesketch of the curve over a larger domain is simply a repetition of this figure.

662

Page 679: CLP-1 Differential Calculus

ANSWERS TO EXERCISES

x

y

π−π π2

−π2 −π

6−5π6

ab π − a−π − b

6

−3

−2

On the interval [0, π], the maximum value of f(x) is 6 and the minimum value is−2.

Let a = arcsin

(−1 +

√33

8

)≈ 0.635 ≈ 0.2π and b = arcsin

(−1−

√33

8

)≈

−1.003 ≈ −0.3π . The points −π − b , b , a , and π − a are inflection points.

3.6.7.10. Answer. The closed dot is the local minimum, and the open dots areinflection points at x = −1 and x = −2±

√1.5. The graph has horizontal asymp-

totes y = 0 as x goes to ±∞.

663

Page 680: CLP-1 Differential Calculus

ANSWERS TO EXERCISES

x

y

−1−2

−1

1

Exercises — Stage 33.6.7.11. *. Answer.

• 3.6.7.11.a decreasing for x < 0 and x > 2, increasing for 0 < x < 2, mini-mum at (0, 0), maximum at (2, 2).

• 3.6.7.11.b concave up for x < 2 −√

2 and x > 2 +√

2, concave down for2−√

2 < x < 2 +√

2, inflection points at x = 2±√

2.

• 3.6.7.11.c∞

3.6.7.11.dOpen dots indicate inflection points, and closed dots indicate local extrema.

x

y

22−√

2 2 +√

2

664

Page 681: CLP-1 Differential Calculus

ANSWERS TO EXERCISES

3.6.7.12. *. Answer. 3.6.7.12.a

x

y

y = f(x)

1

1

There are no inflection points or extrema, except the endpoint (0, 1).3.6.7.12.b

x

y

y = g(x)1

1

There are no inflection points or extrema, except the endpoint (1, 0).3.6.7.12.c The domain of g is (0, 1]. The range of g is [0,∞).3.6.7.12.d g′(1

2) = −2

665

Page 682: CLP-1 Differential Calculus

ANSWERS TO EXERCISES

3.6.7.13. *. Answer. (a)

x

y

−14√5

14√5

4

5 4√5

−4

5 4√5

Local extrema are shown; inflection point at the origin.(b) The number of distinct real roots of x5 − x+ k is:

• 1 when |k| > 4

5 4√

5

• 2 when |k| = 4

5 4√

5

• 3 when |k| < 4

5 4√

5

3.6.7.14. *. Answer. (a)

666

Page 683: CLP-1 Differential Calculus

ANSWERS TO EXERCISES

x

yy = sinhx

x

yy = coshx

1

(b) For any real x, define sinh−1(x) to be the unique solution of sinh(y) = x.For every x ∈ [1,∞), define cosh−1(x) to be the unique y ∈ [0,∞) that obeyscosh(y) = x.

x

y

y = sinh−1 x

x

y

y = cosh−1 x

(c)d

dxcosh−1(x) =

1√x2 − 1

3.7 · L’Hôpital’s Rule and Indeterminate Forms3.7.4 · Exercises

Exercises — Stage 13.7.4.1. Answer. There are many possible answers. Here is one: f(x) = 5x,g(x) = 2x.

3.7.4.2. Answer. There are many possible answers. Here is one: f(x) = x,g(x) = x2.

3.7.4.3. Answer. There are many possible answers. Here is one: f(x) = 1 + 1x,

g(x) = x log 5 (recall we use log to mean logarithm base e).

Exercises — Stage 2

667

Page 684: CLP-1 Differential Calculus

ANSWERS TO EXERCISES

3.7.4.4. *. Answer. − 2

π

3.7.4.5. *. Answer. −∞3.7.4.6. *. Answer. 0

3.7.4.7. *. Answer. 0

3.7.4.8. *. Answer. 3

3.7.4.9. Answer. 2

3.7.4.10. *. Answer. 0

3.7.4.11. *. Answer. 12

3.7.4.12. Answer. 0

3.7.4.13. Answer. 5

3.7.4.14. Answer. ∞3.7.4.15. *. Answer. 3

3.7.4.16. *. Answer. 32

3.7.4.17. *. Answer. 0

3.7.4.18. *. Answer. 13

3.7.4.19. Answer. 0

3.7.4.20. Answer. 1√e

3.7.4.21. Answer. 1

3.7.4.22. Answer. 1

3.7.4.23. *. Answer. c = 0

3.7.4.24. *. Answer.

limx→0

ek sin(x2) − (1 + 2x2)

x4=

−∞ k < 2

2 k = 2

∞ k > 2

Exercises — Stage 33.7.4.25. Answer.

• We want to find the limit as n goes to infinity of the percentage error,

limn→∞

100|S(n)− A(n)||S(n)| . Since A(n) is a nicer function than S(n), let’s sim-

668

Page 685: CLP-1 Differential Calculus

ANSWERS TO EXERCISES

plify: limn→∞

100|S(n)− A(n)||S(n)| = 100

∣∣∣∣1− limn→∞

A(n)

S(n)

∣∣∣∣.We figure out this limit the natural way:

100

∣∣∣∣1− limn→∞

A(n)

S(n)

∣∣∣∣ = 100

∣∣∣∣∣∣∣∣∣1− limn→∞

5n4

5n4 − 13n3 − 4n+ log(n)︸ ︷︷ ︸num→∞den→∞

∣∣∣∣∣∣∣∣∣= 100

∣∣∣∣1− limn→∞

20n3

20n3 − 39n2 − 4 + 1n

∣∣∣∣= 100

∣∣∣∣1− limn→∞

n3

n3· 20

20− 39n− 4

n3 + 1n4

∣∣∣∣= 100|1− 1| = 0

So, as n gets larger and larger, the relative error in the approximation getscloser and closer to 0.

• Now, let’s look at the absolute error.

limn→∞

|S(n)− A(n)| = limn→∞

| − 13n3 − 4n+ log n| =∞

So although the error gets small relative to the giant numbers we’re talkingabout, the absolute error grows without bound.

4 · Towards Integral Calculus4.1 · Introduction to Antiderivatives4.1.2 · Exercises

Exercises — Stage 14.1.2.1. Answer. F (x) = f(x) + C

4.1.2.2. Answer.C(x)

Exercises — Stage 24.1.2.3. Answer. F (x) = x3 + x5 + 5x2 − 9x+ C

4.1.2.4. Answer. F (x) =3

40x8 − 18

5x5 +

1

2x2 + C

4.1.2.5. Answer. F (x) = 3x43 +

45

17x1.7+ C

4.1.2.6. Answer. F (x) =2

7

√x+ C

669

Page 686: CLP-1 Differential Calculus

ANSWERS TO EXERCISES

4.1.2.7. Answer. F (x) =1

5e5x+11 + C

4.1.2.8. Answer. F (x) = −3

5cos(5x) +

7

13sin(13x) + C

4.1.2.9. Answer. F (x) = tan(x+ 1) + C

4.1.2.10. Answer. F (x) = log |x+ 2|+ C

4.1.2.11. Answer. F (x) =7√3

arcsin(x) + C

4.1.2.12. Answer. F (x) =1

5arctan(5x) + C

4.1.2.13. Answer. f(x) = x3 − 9

2x2 + 4x+

19

2

4.1.2.14. Answer. f(x) =1

2sin(2x) + π

4.1.2.15. Answer. f(x) = log |x|4.1.2.16. Answer. f(x) = arcsin x+ x− π − 1

4.1.2.17. Answer. It takes 12

log 7 hours (about 58 minutes) for the initial colonyto increase by 300 individuals.

4.1.2.18. Answer. At time t, the amount of money in your bank account is

75000et

50 + C dollars, for some constant C.

4.1.2.19. Answer.24

π+ 6 ≈ 13.6 kWh

Exercises — Stage 3

4.1.2.20. *. Answer. f ′(x) = g′(x) =1√

x− x2; f and g differ only by a constant.

4.1.2.21. Answer. F (x) = sin(2x) cos(3x) + C

4.1.2.22. Answer. F (x) =ex

x2 + 1+ C

4.1.2.23. Answer. F (x) = ex3

+ C

4.1.2.24. Answer. F (x) = −5

2cos(x2) + C

4.1.2.25. Answer. F (x) =1

2x2 + C.

4.1.2.26. Answer. F (x) = 7 arcsin

(x√3

)+ C

670

Page 687: CLP-1 Differential Calculus

ANSWERS TO EXERCISES

4.1.2.27. Answer. V (H) = 2π

(1

5H5 +

2

3H3 +H

)

671

Page 688: CLP-1 Differential Calculus

SOLUTIONS TO EXERCISES

Appendix F

1 · Limits1.1 · Drawing Tangents and a First Limit1.1.2 · Exercises

Exercises — Stage 11.1.2.1. Solution.

x

yy = f(x)

P

Q

(a)

(b)

(c)

The tangent line to y = f(x) at a point should go through the point, and be “inthe same direction” as f at that point. The secant line through P and Q is simplythe straight line passing through P and Q.

1.1.2.2. Solution.

a True: since y = 2x + 3 is the tangent line to y = f(x) at the point x = 2, thismeans the function and the tangent line have the same value at x = 2. Sof(2) = 2(2) + 3 = 7.

672

Page 689: CLP-1 Differential Calculus

SOLUTIONS TO EXERCISES

b In general, this is false. We are only guaranteed that the curve y = f(x)and its tangent line y = 2x + 3 agree at x = 2. The functions f(x) and2x + 3 may or may not take the same values when x 6= 2. For example, iff(x) = 2x + 3, then of course f(x) and 2x + 3 agree for all values of x. Butif f(x) = 2x+ 3 + (x− 2)2, then f(x) and 2x+ 3 agree only for x = 2.

1.1.2.3. Solution. Since the tangent line to the curve at point P passes throughpoint P , the curve and the tangent line touch at point P . So, they must intersectat least once. By drawing various examples, we can see that different curves maytouch their tangent lines exactly once, exactly twice, exactly three times, etc.

x

y y = f(x)

P

Blue tangent line touches curve only once, at P

x

yy = f(x)

P

Blue tangent line touches curve twice

x

y

y = f(x)

P

Blue tangent line touches curve many times

1.2 · Another Limit and Computing Velocity1.2.2 · Exercises

Exercises — Stage 11.2.2.1. Solution. Speed is nonnegative; velocity has a sign (positive or nega-tive) that indicates direction.

1.2.2.2. Solution. Yes–an object that is not moving has speed 0.

1.2.2.3. Solution. Since you started and ended in the same place, your differ-ence in position was 0, and your difference in time was 24 hours. So, your aver-

age velocity was0

24= 0 kph.

1.2.2.4. Solution. Objects accelerate as they fall — their speed gets bigger andbigger. So in the entire first second of falling, the object is at its fastest at theone-second mark. We have defined the average speed over a time interval to bedistance moved

time taken. We do not yet know how to compute the distanced travelled

by an object whose speed is not constant. But our intuition certainly says that thedistance travelled is no more than the maximum speed times the time taken, sothat the average speed is no larger that the maximum speed. In the next chapterwe will verify that intuition mathematically. See Example 2.13.7. So the averagespeed will be a smaller number then the speed at the one-second mark, which isthe maximum speed.

1.2.2.5. Solution. The slope of a curve is given bychange in vertical component

change in horizontal component. The change in the vertical component is

673

Page 690: CLP-1 Differential Calculus

SOLUTIONS TO EXERCISES

exactly s(b)− s(a), and the change in the horizontal component is exactly b− a.

1.2.2.6. Solution. The velocity is positive when the object is going in the in-creasing direction; it is going “up” on the graph when t is between 0 and 2, andwhen t is between 6 and 7. So, the velocity is positive when t is in (0, 2) ∪ (6, 7).

Exercises — Stage 21.2.2.7. Solution.

a Average velocity:

change in positionchange in time

=s(5)− s(3)

5− 3=

(3 · 52 + 5)− (3 · 32 + 5)

5− 3

= 24 units per second.

b From the notes, we know the velocity of an object at time a is

v(a) = limh→0

s(a+ h)− s(a)

h

So, in our case:

v(1) = limh→0

s(1 + h)− s(1)

h= lim

h→0

[3(1 + h)2 + 5]− [3(1)2 + 5]

h

= limh→0

6h+ 3h2

h= lim

h→06 + 3h = 6

So the velocity when t = 1 is 6 units per second.

1.2.2.8. Solution.

a Average velocity:

change in positionchange in time

=s(9)− s(1)

9− 1=

3− 1

9− 1

=1

4units per second.

b From the notes, we know the velocity of an object at time a is

v(a) = limh→0

s(a+ h)− s(a)

h

So, in our case:

v(1) = limh→0

s(1 + h)− s(1)

h= lim

h→0

√1 + h− 1

h

= limh→0

√1 + h− 1

h·(√

1 + h+ 1√1 + h+ 1

)

674

Page 691: CLP-1 Differential Calculus

SOLUTIONS TO EXERCISES

= limh→0

(1 + h)− 1

h(√

1 + h+ 1)

= limh→0

1√1 + h+ 1

=1

2units per second

c

v(9) = limh→0

s(9 + h)− s(9)

h= lim

h→0

√9 + h− 3

h

= limh→0

√9 + h− 3

h·(√

9 + h+ 3√9 + h+ 3

)= lim

h→0

(9 + h)− 9

h(√

9 + h+ 3)

= limh→0

1√9 + h+ 3

=1

6units per second

Remark: the average velocity is not the average of the two instantaneous veloci-ties.

1.3 · The Limit of a Function1.3.2 · Exercises

Exercises — Stage 11.3.2.1. Solution.

a limx→−2

f(x) = 1: as x gets very close to −2, y gets very close to 1.

b limx→0

f(x) = 0: as x gets very close to 0, y also gets very close to 0.

c limx→2

f(x) = 2: as x gets very close to 2, y gets very close to 2. We ignore thevalue of the function where x is exactly 2.

1.3.2.2. Solution. The limit does not exist. As x approaches 0 from the left, yapproaches -1; as x approaches 0 from the right, y approaches 1. This tells uslimx→0−

f(x) = −1 and limx→0+

f(x) = 1, but neither of these are what the question

asked. Since the limits from left and right do not agree, the limit does not exist.Put another way, there is no single number y approaches as x approaches 0, sothe limit lim

x→0f(x) does not exist.

1.3.2.3. Solution.

a limx→−1−

f(x) = 2: as x approaches −1 from the left, y approaches 2. It doesn’t

matter that the function isn’t defined at x = −1, and it doesn’t matter whathappens to the right of x = −1.

b limx→−1+

f(x) = −2: as x approaches −1 from the right, y approaches -2. It

675

Page 692: CLP-1 Differential Calculus

SOLUTIONS TO EXERCISES

doesn’t matter that the function isn’t defined at −1, and it doesn’t matterwhat happens to the left of −1.

c limx→−1

f(x) = DNE: since the limits from the left and right don’t agree, the

limit does not exist.

d limx→−2+

f(x) = 0: as x approaches −2 from the right, y approaches 0. It

doesn’t matter that the function isn’t defined at 2, or to the left of 2.

e limx→2−

f(x) = 0: as x approaches 2 from the left, y approaches 0. It doesn’t

matter that the function isn’t defined at 2, or to the right of 2.

1.3.2.4. Solution. Many answers are possible; here is one.

x

yy = f(x)

3

10

As x gets closer and closer to 3, y gets closer and closer to 10: this showslimx→3

f(x) = 10. Also, at 3 itself, the function takes the value 10; this shows

f(3) = 10.

1.3.2.5. Solution. Many answers are possible; here is one.

676

Page 693: CLP-1 Differential Calculus

SOLUTIONS TO EXERCISES

x

yy = f(x)

3

10

Note that, as x gets closer and closer to 3 except at 3 itself, y gets closer and closerto 10: this shows lim

x→3f(x) = 10. Then, when x = 3, the function has value 0: this

shows f(3) = 0.

1.3.2.6. Solution. In general, this is false. The limit as x goes to 3 does not takeinto account the value of the function at 3: f(3) can be anything.

1.3.2.7. Solution. False. The limit as x goes to 3 does not take into account thevalue of the function at 3: f(3) tells us nothing about lim

x→3f(x).

1.3.2.8. Solution. limx→−2−

f(x) = 16: in order for the limit limx→2

f(x) to exist and be

equal to 16, both one sided limits must exist and be equal to 16.

1.3.2.9. Solution. Not enough information to say. If limx→−2+

f(x) = 16, then

limx→−2

f(x) = 16. If limx→−2+

f(x) 6= 16, then limx→−2

f(x) does not exist.

Exercises — Stage 21.3.2.10. Solution. lim

t→0sin t = 0: as t approaches 0, sin t approaches 0 as well.

1.3.2.11. Solution. limx→0+

log x = −∞: as x approaches 0 from the right, log x is

negative and increasingly large, growing without bound.

1.3.2.12. Solution. limy→3

y2 = 9: as y gets closer and closer to 3, y2 gets closer and

closer to 32.

1.3.2.13. Solution. limx→0−

1

x= −∞: as x gets closer and closer to 0 from the left,

1

xbecomes a larger and larger negative number.

677

Page 694: CLP-1 Differential Calculus

SOLUTIONS TO EXERCISES

1.3.2.14. Solution. limx→0

1

x= DNE: as x gets closer and closer to 0 from the left,

1

xbecomes a larger and larger negative number; but as x gets closer and closer to 0

from the right,1

xbecomes a larger and larger positive number. So the limit from

the left is not the same as the limit from the right, and so limx→0

1

x= DNE. Contrast

this with Question 1.3.2.15.

1.3.2.15. Solution. limx→0

1

x2= ∞: as x gets closer and closer to 0 from the either

side,1

x2becomes a larger and larger positive number, growing without bound.

Contrast this with Question 1.3.2.14.

1.3.2.16. Solution. limx→3

1

10=

1

10: no matter what x is,

1

10is always

1

10. In partic-

ular, as x approaches 3,1

10stays put at

1

10.

1.3.2.17. Solution. When x is very close to 3, f(x) looks like the function x2. So:limx→3

f(x) = limx→3

x2 = 9

1.4 · Calculating Limits with Limit Laws1.4.2 · Exercises

Exercises — Stage 11.4.2.1. Solution. Zeroes cause a problem when they show up in the denom-inator, so we can only compute (1.4.2.1.a) and (1.4.2.1.d). (Both these limits arezero.) Be careful: there is no such rule as “zero divided by zero is one,” or “zerodivided by zero is zero.”

1.4.2.2. Solution. The statement limx→3

f(x)

g(x)= 10 tells us that, as x gets very close

to 3, f(x) is 10 times as large as g(x). We notice that if f(x) = 10g(x), thenf(x)

g(x)= 10, so lim

x→f(x)

g(x)= 10 wherever f and g exist. So it’s enough to find a

function g(x) that has limit 0 at 3. Such a function is (for example) g(x) = x − 3.So, we take f(x) = 10(x − 3) and g(x) = x − 3. It is easy now to check that

limx→3

f(x) = limx→3

g(x) = 0 and limx→3

f(x)

g(x)= lim

x→3

10(x− 3)

x− 3= lim

x→310 = 10.

1.4.2.3. Solution.

• As we saw in Question 1.4.2.2, x − 3 is a function with limit 0 at x = 3.So one way of thinking about this question is to try choosing f(x) so thatf(x)g(x)

= g(x) = x − 3 too, which leads us to the solution f(x) = (x − 3)2 andg(x) = x− 3. This is one of many, many possible answers.

• Another way of thinking about this problem is that f(x) should go to 0

678

Page 695: CLP-1 Differential Calculus

SOLUTIONS TO EXERCISES

“more strongly” than g(x) when x approaches 3. One way of a functiongoing to 0 really strongly is to make that function identically zero. So we

can set f(x) = 0 and g(x) = x− 3. Nowf(x)

g(x)is equal to 0 whenever x 6= 3,

and is undefined at x = 3. Since the limit as x goes to three does not take

into account the value of the function at 3, we have limx→3

f(x)

g(x)= 0.

There are many more possible answers.

1.4.2.4. Solution. One way to start this problem is to remember limx→0

1

x2= ∞.

(Using1

x2as opposed to

1

xis important, since lim

x→0

1

xdoes not exist.) Then by

“shifting” by three, we find limx→3

1

(x− 3)2= ∞. So it is enough to arrange that

f(x)

g(x)=

1

(x− 3)2. We can achieve this with f(x) = x− 3 and g(x) = (x− 3)3, and

maintain limx→3

f(x) = limx→3

g(x) = 0. Again, this is one of many possible solutions.

1.4.2.5. Solution. Any real number; positive infinity; negative infinity; does notexist.This is an important thing to remember: often, people see limits that look like0

0and think that the limit must be 1, or 0, or infinite. In fact, this limit could be

anything–it depends on the relationship between f and g.Questions 1.4.2.2 and 1.4.2.3 show us examples where the limit is 10 and 0; theycan easily be modified to make the limit any real number.Question 1.4.2.4 show us an example where the limit is∞; it can easily be modi-fied to make the limit −∞ or DNE.

Exercises — Stage 21.4.2.6. Solution. Since we’re not trying to divide by 0, or multiply by

infinity: limt→10

2(t− 10)2

t=

2 · 010

= 0

1.4.2.7. Solution. Since we’re not doing anything dodgy like putting 0 in the

denominator, limy→0

(y + 1)(y + 2)(y + 3)

cos y=

(0 + 1)(0 + 2)(0 + 3)

cos 0=

6

1= 6.

1.4.2.8. Solution. Since the limits of the numerator and denominator exist, and

since the limit of the denominator is nonzero: limx→3

(4x− 2

x+ 2

)4

=

(4(3)− 2

3 + 2

)4

=

16

1.4.2.9. *. Solution.

limt→−3

(1− tcos(t)

)=

limt→−3

(1− t)limt→−3

cos(t)= 4/ cos(−3) = 4/ cos(3)

679

Page 696: CLP-1 Differential Calculus

SOLUTIONS TO EXERCISES

1.4.2.10. *. Solution. If try naively then we get 0/0, so we expand and thensimplify:

(2 + h)2 − 4

2h=h2 + 4h+ 4− 4

2h=h

2+ 2

Hence the limit is limh→0

(h

2+ 2

)= 2.

1.4.2.11. *. Solution.

limt→−2

(t− 5

t+ 4

)=

limt→−2(t− 5)

limt→−2(t+ 4)= −7/2.

1.4.2.12. *. Solution.

limt→1

√5x3 + 4 =

√limt→1

(5x3 + 4

)=√

5 limt→1

(x3) + 4 =√

9 = 3.

1.4.2.13. *. Solution.

limt→−1

(t− 2

t+ 3

)=

limt→−1

(t− 2)

limt→−1

(t+ 3)= −3/2.

1.4.2.14. *. Solution. We simply plug in x = 1: limx→1

[log(1 + x)− x

x2

]= log(2)−1.

1.4.2.15. *. Solution. If we try naively then we get 0/0, so we simplify first:

x− 2

x2 − 4=

x− 2

(x− 2)(x+ 2)=

1

x+ 2

Hence the limit is limx→2

1

x+ 2= 1/4.

1.4.2.16. *. Solution. If we try to plug in x = 4, we find the denominator iszero. So to get a better idea of what’s happening, we factor the numerator anddenominator:

limx→4

x2 − 4x

x2 − 16= lim

x→4

x(x− 4)

(x+ 4)(x− 4)

= limx→4

x

x+ 4

=4

8=

1

2

1.4.2.17. *. Solution. If we try to plug in x = 2, we find the denominator is zero.So to get a better idea of what’s happening, we factor the numerator:

limx→2

x2 + x− 6

x− 2= lim

x→2

(x+ 3)(x− 2)

x− 2

680

Page 697: CLP-1 Differential Calculus

SOLUTIONS TO EXERCISES

= limx→2

(x+ 3) = 5

1.4.2.18. *. Solution. If we try naively then we get 0/0, so we simplify first:

x2 − 9

x+ 3=

(x− 3)(x+ 3)

(x+ 3)= x− 3

Hence the limit is limx→−3

(x− 3) = −6.

1.4.2.19. Solution. To calculate the limit of a polynomial, we simply evaluate

the polynomial: limt→2

1

2t4 − 3t3 + t =

1

2· 24 − 3 · 23 + 2 = −14

1.4.2.20. *. Solution.√x2 + 8− 3

x+ 1=

√x2 + 8− 3

x+ 1·√x2 + 8 + 3√x2 + 8 + 3

=(x2 + 8)− 32

(x+ 1)(√x2 + 8 + 3)

=x2 − 1

(x+ 1)(√x2 + 8 + 3)

=(x− 1)(x+ 1)

(x+ 1)(√x2 + 8 + 3)

=(x− 1)√x2 + 8 + 3

limx→−1

√x2 + 8− 3

x+ 1= lim

x→−1

(x− 1)√x2 + 8 + 3

=−2√9 + 3

= −2

6= −1

3.

1.4.2.21. *. Solution. If we try to do the limit naively we get 0/0. Hence wemust simplify.

√x+ 7−

√11− x

2x− 4=

√x+ 7−

√11− x

2x− 4·(√

x+ 7 +√

11− x√x+ 7 +

√11− x

)=

(x+ 7)− (11− x)

(2x− 4)(√x+ 7 +

√11− x)

=2x− 4

(2x− 4)(√x+ 7 +

√11− x)

=1√

x+ 7 +√

11− x

So, limx→2

√x+ 7−

√11− x

2x− 4= lim

x→2

1√x+ 7 +

√11− x

681

Page 698: CLP-1 Differential Calculus

SOLUTIONS TO EXERCISES

=1√

9 +√

9

=1

6

1.4.2.22. *. Solution. If we try to do the limit naively we get 0/0. Hence wemust simplify.

√x+ 2−

√4− x

x− 1=

√x+ 2−

√4− x

x− 1·√x+ 2 +

√4− x√

x+ 2 +√

4− x=

(x+ 2)− (4− x)

(x− 1)(√x+ 2 +

√4− x)

=2x− 2

(x− 1)(√x+ 2 +

√4− x)

=2√

x+ 2 +√

4− x

So the limit is

limx→1

√x+ 2−

√4− x

x− 1= lim

x→1

2√x+ 2 +

√4− x

=2√

3 +√

3

=1√3

1.4.2.23. *. Solution. If we try to do the limit naively we get 0/0. Hence wemust simplify.

√x− 2−

√4− x

x− 3=

√x− 2−

√4− x

x− 3·√x− 2 +

√4− x√

x− 2 +√

4− x=

(x− 2)− (4− x)

(x− 3)(√x− 2 +

√4− x)

=2x− 6

(x− 3)(√x− 2 +

√4− x)

=2√

x− 2 +√

4− x

So, limx→3

√x− 2−

√4− x

x− 3= lim

x→3

2√x− 2 +

√4− x

=2

1 + 1

= 1.

1.4.2.24. *. Solution. Here we get 0/0 if we try the naive approach. Hence we

682

Page 699: CLP-1 Differential Calculus

SOLUTIONS TO EXERCISES

must simplify.

3t− 3

2−√

5− t =3t− 3

2−√

5− t ×2 +√

5− t2 +√

5− t=(2 +√

5− t) 3t− 3

22 − (5− t)=(2 +√

5− t) 3t− 3

t− 1

=(2 +√

5− t) 3(t− 1)

t− 1

So there is a cancelation. Hence the limit is

limt→1

3t− 3

2−√

5− t = limt→1

(2 +√

5− t)· 3

= 12

1.4.2.25. Solution. This is a classic example of the Squeeze Theorem. It is tempt-

ing to try to use arithmetic of limits: limx→0−x2 = 0, and lim

x→0cos

(3

x

)= something,

and zero times something is 0. However, this is invalid reasoning, because we

can only use arithmetic of limits when those limits exist, and limx→0

cos

(3

x

)does

not exist. So, we need the Squeeze Theorem.Since −1 ≤ cos

(3x

)≤ 1, we can bound our function of interest from above and

below (being careful of the sign!):

−x2(1) ≤ −x2cos

(3

x

)≤ −x2(−1)

So our function of interest is between −x2 and x2. Since limx→0−x2 = lim

x→0x2 = 0, by

the Squeeze Theorem, also limx→0−x2 cos

(3

x

)= 0.

Advice about writing these up: whenever we use the Squeeze Theorem, we needto explicitly write that two things are true: that the function we’re interested isbounded above and below by two other functions, and that both of those func-tions have the same limit. Then we can conclude (and we need to write this downas well!) that our original function also shares that limit.

1.4.2.26. Solution. Recall that sine and cosine, no matter what (real-number)input we feed them, spit out numbers between −1 and 1. So we can bound ourhorrible numerator, rather than trying to deal with it directly.

x4(−1) + 5x2(−1) + 2 ≤ x4sin

(1

x

)+ 5x2cos

(1

x

)+ 2 ≤ x4(1) + 5x2(1) + 2

Further, notice that our bounded functions tend to the same value as x goes to 0:

limx→0

x4(−1) + 5x2(−1) + 2 = limx→0

x4 + 5x2 + 2 = 2.

683

Page 700: CLP-1 Differential Calculus

SOLUTIONS TO EXERCISES

So, by the Squeeze Theorem, also

limx→0

x4 sin

(1

x

)+ 5x2 cos

(1

x

)+ 2 = 2.

Now, we evaluate our original limit:

limx→0

x4 sin(

1x

)+ 5x2 cos

(1x

)+ 2

(x− 2)2=

2

(−2)2=

1

2.

1.4.2.27. *. Solution. limx→0

sin2

(1

x

)= DNE, so we think about using the

Squeeze Theorem. We’ll need to bound the expression x sin2

(1

x

), but the bound-

ing is a little delicate. For any non-zero value we plug in for x, sin2

(1

x

)is a

number in the interval [0, 1]. If a is a number in the interval [0, 1], then:

0 <xa < x when x is positivex <xa < 0 when x is negative

We’ll show you two ways to use this information to create bound that will allowyou to apply the Squeeze Theorem.

• Solution 1: We will evaluate separately the limit from the right and fromthe left.

When x > 0,

0 ≤ x sin2

(1

x

)≤ x

because 0 ≤ sin2

(1

x

)≤ 1. Since

limx→0+

0 = 0 and limx→0+

x = 0

then by the Squeeze Theorem, also

limx→0+

x sin2

(1

x

)= 0.

Similarly, When x < 0,

x ≤ x sin2

(1

x

)≤ 0

because 0 ≤ sin2

(1

x

)≤ 1. Since

limx→0−

x = 0 and limx→0−

0 = 0

684

Page 701: CLP-1 Differential Calculus

SOLUTIONS TO EXERCISES

then by the Squeeze Theorem, also

limx→0−

x sin2

(1

x

)= 0.

Since the one-sided limits are both equal to zero,

limx→0

x sin2

(1

x

)= 0.

Remark: this is a perfectly fine proof, but it seems to repeat itself. Since thecases x < 0 and x > 0 are so similar, we would like to take care of themtogether. This can be done as shown below.

• Solution 2: If x 6= 0, then 0 ≤ sin2

(1

x

)≤ 1, so

−|x| ≤ x sin2

(1

x

)≤ |x|.

Sincelimx→0−|x| = 0 and lim

x→0|x| = 0

then by the Squeeze Theorem, also

limx→0

x sin2

(1

x

)= 0.

1.4.2.28. Solution. When we plug w = 5 in to the numerator and denominator,we find that each becomes zero. Since we can’t divide by zero, we have to dig alittle deeper. When a polynomial has a root, that also means it has a factor: wecan factor (w − 5) out of the top. That lets us cancel:

limw→5

2w2 − 50

(w − 5)(w − 1)= lim

w→5

2(w − 5)(w + 5)

(w − 5)(w − 1)= lim

w→5

2(w + 5)

(w − 1).

Note that the function2w2 − 50

(w − 5)(w − 1)is NOT defined at w = 5, while the func-

tion2(w + 5)

(w − 1)IS defined at w = 5; so strictly speaking, these two functions are

not equal. However, for every value of w that is not 5, the functions are thesame, so their limits are equal. Furthermore, the limit of the second functionis quite easy to calculate, since we’ve eliminated the zero in the denominator:

limw→5

2(w + 5)

(w − 1)=

2(5 + 5)

5− 1= 5.

So limw→5

2w2 − 50

(w − 5)(w − 1)= lim

w→5

2(w + 5)

(w − 1)= 5.

685

Page 702: CLP-1 Differential Calculus

SOLUTIONS TO EXERCISES

1.4.2.29. Solution. When we plug in r = −5 to the denominator, we find thatit becomes 0, so we need to dig deeper. The numerator is not zero, so cancellingis out. Notice that the denominator is factorable: r2 + 10r + 25 = (r + 5)2. Asr approaches −5 from either side, the denominator gets very close to zero, butstays positive. The numerator gets very close to−5. So, as r gets closer to−5, wehave something close to −5 divided by a very small, positive number. Since thedenominator is small, the fraction will have a large magnitude; since the numer-ator is negative and the denominator is positive, the fraction will be negative. So,limr→−5

r

r2 + 10r + 25= −∞

1.4.2.30. Solution. First, we find limx→−1

x3 + x2 + x+ 1

3x+ 3. When we plug in x = −1

to the top and the bottom, both become zero. In a polynomial, where there is aroot, there is a factor, so this tells us we can factor out (x + 1) from both the topand the bottom. It’s pretty easy to see how to do this in the bottom. For the top, ifyou’re having a hard time, one factoring method (of many) to try is long divisionof polynomials; another is to factor out (x + 1) from the first two terms and thelast two terms. (Long division is covered in Appendix A.14 of CLP-II IntegralCalculus.)

limx→−1

x3 + x2 + x+ 1

3x+ 3= lim

x→−1

x2(x+ 1) + (x+ 1)

3x+ 3= lim

x→−1

(x+ 1)(x2 + 1)

3(x+ 1)

= limx→−1

x2 + 1

3=

(−1)2 + 1

3=

2

3.

One thing to note here is that the functionx3 + x2 + x+ 1

3x+ 3is not defined at x = −1

(because we can’t divide by zero). So we replaced it with the functionx2 + 1

3,

which IS defined at x = −1. These functions only differ at x = −1; they are thesame at every other point. That is why we can use the second function to findthe limit of the first function.Now we’re ready to find the actual limit asked in the problem:

limx→−1

√x3 + x2 + x+ 1

3x+ 3=

√2

3.

1.4.2.31. Solution. When we plug x = 0 into the denominator, we get 0, whichmeans we need to look harder. The numerator is not zero, so we won’t be able tocancel our problems away. Let’s factor to make things clearer.

x2 + 2x+ 1

3x5 − 5x3=

(x+ 1)2

x3(3x2 − 5)

As x gets close to 0, the numerator is close to 1; the term (3x2−5) is negative; andthe sign of x3 depends on the direction we’re approaching 0 from. Since we’re

686

Page 703: CLP-1 Differential Calculus

SOLUTIONS TO EXERCISES

dividing a numerator that is very close to 1 by something that’s getting veryclose to 0, the magnitude of the fraction is getting bigger and bigger withoutbound. Since the sign of the fraction flips depending on whether we are usingnumbers slightly bigger than 0, or slightly smaller than 0, that means the one-

sided limits are ∞ and −∞, respectively. (In particular, limx→0−

x2 + 2x+ 1

3x5 − 5x3= ∞

and limx→0+

x2 + 2x+ 1

3x5 − 5x3= −∞.) Since the one-sided limits don’t agree, the limit

does not exist.

1.4.2.32. Solution. As usual, we first try plugging in t = 7, but the denominatoris 0, so we need to think harder. The top and bottom are both squares, so let’s go

ahead and factor:t2x2 + 2tx+ 1

t2 − 14t+ 49=

(tx+ 1)2

(t− 7)2. Since x is positive, the numerator

is nonzero. Also, the numerator is positive near t = 7. So, we have somethingpositive and nonzero on the top, and we divide it by the bottom, which is positiveand getting closer and closer to zero. The quotient is always positive near t = 7,

and it is growing in magnitude without bound, so limt→7

t2x2 + 2tx+ 1

t2 − 14t+ 49=∞.

Remark: there is an important reason we specified that x must be a positive con-stant. Suppose x were −1

7(which is negative and so was not allowed in the

question posed). In this case, we would have

limt→7

t2x2 + 2tx+ 1

t2 − 14t+ 49= lim

t→7

(tx+ 1)2

(t− 7)2

= limt→7

(−t/7 + 1)2

(t− 7)2

= limt→7

(−1/7)2(t− 7)2

(t− 7)2

= limt→7

(−1/7)2

=1

496=∞

1.4.2.33. Solution. The function whose limit we are taking does not depend ond. Since x is a constant, x5−32x+15 is also a constant–it’s just some number, thatdoesn’t change, regardless of what d does. So lim

d→0x5 − 32x+ 15 = x5 − 32x+ 15.

1.4.2.34. Solution. There’s a lot going on inside that sine function... and wedon’t have to care about any of it. No matter what horrible thing we put inside asine function, the sine function will spit out a number between −1 and 1. So thatmeans we can bound our horrible function like this:

(x− 1)2 · (−1) ≤ (x− 1)2sin

[(x2 − 3x+ 2

x2 − 2x+ 1

)2

+ 15

]≤ (x− 1)2 · (1)

687

Page 704: CLP-1 Differential Calculus

SOLUTIONS TO EXERCISES

Since limx→1

(x− 1)2 · (−1) = limx→1

(x− 1)2 · (1) = 0, the Squeeze Theorem tells us that

limx→1

(x− 1)2 sin

[(x2 − 3x+ 2

x2 − 2x+ 1

)2

+ 15

]= 0

as well.

1.4.2.35. *. Solution. Since −1 ≤ sinx ≤ 1 for all values of x,

−1 ≤ sin(x−100

)≤ 1

(−1)x1/101 ≤ x1/101sin(x−100

)≤ (1)x1/101 when x > 0, and

(1)x1/101 ≤ x1/101sin(x−100

)≤ (−1)x1/101 when x < 0. Also,

limx→0

x1/101 = limx→0−x1/101 = 0 So, by the Squeeze Theorem,

limx→0−

x1/101 sin(x−100

)= 0 = lim

x→0+x1/101 sin

(x−100

)and so

0 = limx→0

x1/101 sin(x−100

).

Remark: there is a technical point here. When x is a positive number, −x isnegative, so (−1)x < x. But, when x is negative, −x is positive, so (−1)x > x.This is why we take the one-sided limits of our function, and apply the SqueezeTheorem to them separately. It is not true to say that, for instance, (−1)x1/101 ≤x1/101 sin

(x−100

)when x is near zero, because this does not hold when x is less

than zero.

1.4.2.36. *. Solution.

limx→2

x2 − 4

x2 − 2x= lim

x→2

(x− 2)(x+ 2)

x(x− 2)= lim

x→2

x+ 2

x= 2

1.4.2.37. Solution. When we plug in x = 5 to the top and the bottom, both

limits exist, and the bottom is nonzero. So limx→5

(x− 5)2

x+ 5=

0

10= 0.

1.4.2.38. Solution. Since we can’t plug in t = 12, we’ll simplify. One way to start

is to add the fractions in the numerator. We’ll need a common demoninator, suchas 3t2(t2 − 1).

limt→ 1

2

13t2

+ 1t2−1

2t− 1= lim

t→ 12

t2−13t2(t2−1)

+ 3t2

3t2(t2−1)

2t− 1

= limt→ 1

2

4t2−13t2(t2−1)

2t− 1

= limt→ 1

2

4t2 − 1

3t2(t2 − 1)(2t− 1)

= limt→ 1

2

(2t+ 1)(2t− 1)

3t2(t2 − 1)(2t− 1)

688

Page 705: CLP-1 Differential Calculus

SOLUTIONS TO EXERCISES

= limt→ 1

2

2t+ 1

3t2(t2 − 1)

Since we cancelled out the term that was causing the numerator and denomina-tor to be zero when t = 1

2, now t = 1

2is in the domain of our function, so we

simply plug it in:

=1 + 1

34

(14− 1)

=2

34

(−3

4

)= −32

9

1.4.2.39. Solution. We recall that

|x| =

x , x ≥ 0

−x , x < 0

So,

|x|x

=

xx

, x > 0−xx

, x < 0

=

1 , x > 0

−1 , x < 0

Therefore,

3 +|x|x

=

4 , x > 0

2 , x < 0

Since our function gives a value of 4 when x is to the right of zero, and a value of

2 when x is to the left of zero, limx→0

(3 +|x|x

)does not exist.

To further clarify the situation, the graph of y = f(x) is sketched below:

689

Page 706: CLP-1 Differential Calculus

SOLUTIONS TO EXERCISES

x

y

4

2

1.4.2.40. Solution. If we factor out 3 from the numerator, our function becomes

3|d+ 4|d+ 4

. We recall that

|X| =

X , X ≥ 0

−X , X < 0

So, with X = d+ 4,

3|d+ 4|d+ 4

=

3d+4d+4

, d+ 4 > 0

3−(d+4)d+4

, d+ 4 < 0

=

3 , d > −4

−3 , d < −4

Since our function gives a value of 3 when d > −4, and a value of −3 when

d < −4, limd→−4

|3d+ 12|d+ 4

does not exist.

To further clarify the situation, the graph of y = f(x) is sketched below:

690

Page 707: CLP-1 Differential Calculus

SOLUTIONS TO EXERCISES

x

y

−3

−4

1.4.2.41. Solution. Note that x = 0 is in the domain of our function, and nothing“weird” is happening there: we aren’t dividing by zero, or taking the square rootof a negative number, or joining two pieces of a piecewise-defined function. So,

as x gets extremely close to zero,5x− 9

|x|+ 2is getting extremely close to

0− 9

0 + 2=−9

2.

That is, limx→0

5x− 9

|x|+ 2= −9

2.

1.4.2.42. Solution. Since we aren’t dividing by zero, and all these limits exist:

limx→−1

xf(x) + 3

2f(x) + 1=

(−1)(−1) + 3

2(−1) + 1= −4.

1.4.2.43. *. Solution. As x→ −2, the denominator goes to 0, and the numeratorgoes to −2a + 7. For the ratio to have a limit, the numerator must also converge

to 0, so we need a =7

2. Then,

limx→−2

x2 + ax+ 3

x2 + x− 2= lim

x→−2

x2 + 72x+ 3

(x+ 2)(x− 1)

= limx→−2

(x+ 2)(x+ 32)

(x+ 2)(x− 1)

= limx→−2

x+ 32

x− 1

=1

6

so the limit exists when a =7

2.

691

Page 708: CLP-1 Differential Calculus

SOLUTIONS TO EXERCISES

1.4.2.44. Solution.

a limx→0

f(x) = 0: as x approaches 0, so does 2x.

b limx→0

g(x) = DNE: the left and right limits do not agree, so the limit does not

exist. In particular: limx→0−

g(x) = −∞ and limx→0−

g(x) =∞.

c limx→0

f(x)g(x) = limx→0

2x · 1

x= lim

x→02 = 2. Remark: although the limit of g(x)

does not exist here, the limit of f(x)g(x) does.

d limx→0

f(x)

g(x)= lim

x→0

2x1x

= limx→0

2x2 = 0

e limx→2

f(x) + g(x) = limx→2

2x+1

x= 4 +

1

2=

9

2

f limx→0

f(x) + 1

g(x+ 1)= lim

x→0

2x+ 11

x+1

=1

1= 1

Exercises — Stage 31.4.2.45. Solution. We can begin by plotting the points that are easy to read offthe diagram.

x f(x) 1f(x)

−3 −3 −13

−2 0 UND

−1 3 13

0 3 13

1 32

23

2 0 UND

3 1 1

Note that 1f(x)

is undefined when f(x) = 0. So 1f(x)

is undefined at x = −2 andx = 2. We shall look more closely at the behaviour of 1

f(x)for x near ±2 shortly.

Plotting the above points, we get the following picture:

692

Page 709: CLP-1 Differential Calculus

SOLUTIONS TO EXERCISES

x

y

1

1

Since f(x) is constant when x is between -1 and 0, then also 1f(x)

is constant be-tween -1 and 0, so we update our picture:

x

y

1

1

The big question that remains is the behaviour of 1f(x)

when x is near -2 and 2.We can answer this question with limits. As x approaches −2 from the left, f(x)gets closer to zero, and is negative. So 1

f(x)will be negative, and will increase in

magnitude without bound; that is, limx→−2−

1

f(x)= −∞. Similarly, as x approaches

−2 from the right, f(x) gets closer to zero, and is positive. So 1f(x)

will be positive,

693

Page 710: CLP-1 Differential Calculus

SOLUTIONS TO EXERCISES

and will increase in magnitude without bound; that is, limx→−2−

1

f(x)=∞. We add

this behaviour to our graph:

x

y

1

1

Now, we consider the behaviour at x = 2. Since f(x) gets closer and closer to 0

AND is positive as x approaches 2, we conclude limx→2

1

f(x)= ∞. Adding to our

picture:

x

y

1

1

Now the only remaining blank space is between x = 0 and x = 1. Since f(x) isa smooth curve that stays away from 0, we can draw some kind of smooth curve

694

Page 711: CLP-1 Differential Calculus

SOLUTIONS TO EXERCISES

here, and call it good enough. (Later on we’ll go into more details about drawinggraphs. The purpose of this exercise was to utilize what we’ve learned aboutlimits.)

x

y

1

1

1.4.2.46. Solution. We can start by examining points.

x f(x) g(x)f(x)

g(x)−3 −3 −1.5 2

−2 0 0 UND−1 3 1.5 2

−0 3 1.5 2

1 1.5 .75 2

2 0 0 UND3 1 .5 2

We cannot divide by zero, sof(x)

g(x)is not defined when x = ±2. But for every

other value of x that we plotted, f(x) is twice as large as g(x),f(x)

g(x)= 2. With this

in mind, we see that the graph of f(x) is exactly the graph of 2g(x).This gives us the graph below.

695

Page 712: CLP-1 Differential Calculus

SOLUTIONS TO EXERCISES

x

y

1

1

y =f(x)

g(x)

Remark: f(2) = g(2) = 0, sof(2)

g(2)does not exist, but lim

x→2

f(x)

g(x)= 2. Although we

are trying to “divide by zero” at x = ±2, it would be a mistake here to interpretthis as a vertical asymptote.

1.4.2.47. Solution. Velocity of white ball when t = 1 is limh→0

s(1 + h)− s(1)

h, so

the given information tells us limh→0

s(1 + h)− s(1)

h= 5. Then the velocity of the

red ball when t = 1 is limh→0

2s(1 + h)− 2s(1)

h= lim

h→02 · s(1 + h)− s(1)

h= 2 · 5 = 10.

1.4.2.48. Solution. 1.4.2.48.a Neither limit exists. When x gets close to 0, theselimits go to positive infinity from one side, and negative infinity from the other.

1.4.2.48.b limx→0

[f(x) + g(x)] = limx→0

[1

x− 1

x

]= lim

x→00 = 0.

1.4.2.48.c No: this is an example of a time when the two individual functionshave limits that don’t exist, but the limit of their sum does exist. This “sum rule”is only true when both lim

x→af(x) and lim

x→ag(x) exist.

1.4.2.49. Solution. 1.4.2.49.a When we evaluate the limit from the left, we onlyconsider values of x that are less than zero. For these values of x, our function isx2 − 3. So, lim

x→0−f(x) = lim

x→0−(x2 − 3) = −3.

1.4.2.49.b When we evaluate the limit from the right, we only consider values ofx that are greater than zero. For these values of x, our function is x2 + 3. So,limx→0+

f(x) = limx→0+

(x2 + 3) = 3.

1.4.2.49.c Since the limits from the left and right do not agree, limx→0

f(x) =DNE.

696

Page 713: CLP-1 Differential Calculus

SOLUTIONS TO EXERCISES

x

y

3

−3

1.4.2.50. Solution. 1.4.2.50.a When we evaluate limx→−4−

f(x), we only consider

values of x that are less than −4. For these values, f(x) = x3 + 8x2 + 16x. So,

limx→4−

f(x) = limx→−4−

(x3 + 8x2 + 16x) = (−4)3 + 8(−4)2 + 16(−4) = 0

Note that, because x3 + 8x2 + 16x is a polynomial, we can evaluate the limit bydirectly substituting in x = −4.1.4.2.50.b When we evaluate lim

x→−4+f(x), we only consider values of x that are

greater than −4. For these values,

f(x) =x2 + 8x+ 16

x2 + 30x− 4

So

limx→−4+

f(x) = limx→−4+

x2 + 8x+ 16

x2 + 30x− 4

This is a rational function, and x = −4 is in its domain (we aren’t doing anythingsuspect, like dividing by 0), so again we can directly substitute x = −4 to evaluate

697

Page 714: CLP-1 Differential Calculus

SOLUTIONS TO EXERCISES

the limit:

limx→−4+

f(x) =(−4)2 + 8(−4) + 16

(−4)2 + 30(−4)− 4=

0

−108= 0

1.4.2.50.c Since limx→−4−

f(x) = limx→−4+

f(x) = 0, we conclude that limx→−4

f(x) = 0

1.5 · Limits at Infinity1.5.2 · Exercises

Exercises — Stage 11.5.2.1. Solution. Any polynomial of degree one or higher will go to∞ or −∞as x goes to∞. So, we need a polynomial of degree 0–that is, f(x) is a constant.One possible answer is f(x) = 1.

1.5.2.2. Solution. This will be the case for any polynomial of odd degree. Forinstance, f(x) = x.Many answers are possible: also f(x) = x15−32x2 +9 satisfies lim

x→∞f(x) =∞ and

limx→−∞

f(x) = −∞.

Exercises — Stage 2

1.5.2.3. Solution. limx→∞

2−x = limx→∞

1

2x= 0

1.5.2.4. Solution. As x gets larger and larger, 2x grows without bound. (For in-teger values of x, you can imagine multiplying 2 by itself more and more times.)So, lim

x→∞2x =∞.

1.5.2.5. Solution. Write X = −x. As x becomes more and more negative, Xbecomes more and more positive. From Question 1.5.2.4, we know that 2X growswithout bound as X gets larger and larger. Since 2x = 2−(−x) = 2−X = 1

2X, as we

let x become a huge negative number, we are in effect dividing by a huge positivenumber; hence lim

x→−∞2x = 0.

A more formulaic way to describe the above is this: limx→−∞

2x = limX→∞

2−X =

limX→∞

12X

= 0.

1.5.2.6. Solution. There is no single number that cosx approaches as x becomesmore and more strongly negative: as x grows in the negative direction, the func-tion oscillates between −1 and +1, never settling close to one particular number.So, this limit does not exist.

1.5.2.7. Solution. The highest-order term in this polynomial is −3x5, so thisdominates the function’s behaviour as x goes to infinity. More formally:

limx→∞

(x− 3x5 + 100x2

)= lim

x→∞−3x5

(1− 1

3x4− 100

3x3

)

698

Page 715: CLP-1 Differential Calculus

SOLUTIONS TO EXERCISES

= limx→∞−3x5 = −∞

because

limx→∞

(1− 1

3x4− 100

3x3

)= 1− 0− 0 = 1.

1.5.2.8. Solution. Our standard trick is to factor out the highest power of x inthe denominator: x4. We just have to be a little careful with the square root. Sincewe are taking the limit as x goes to positive infinity, we have positive x-values,so√x2 = x and

√x8 = x4.

limx→∞

√3x8 + 7x4 + 10

x4 − 2x2 + 1= lim

x→∞

√x8(3 + 7

x4) + 10

x4(1− 2x2

+ 1x4

)

= limx→∞

√x8

√3 + 7

x4+ 10

x4(1− 2x2

+ 1x4

)

= limx→∞

x4√

3 + 7x4

+ 10

x4(1− 2x2

+ 1x4

)

= limx→∞

x4(√

3 + 7x4

+ 10x4

)x4(1− 2

x2+ 1

x4)

= limx→∞

√3 + 7

x4+ 10

x4

1− 2x2

+ 1x4

=

√3 + 0 + 0

1− 0 + 0=√

3

1.5.2.9. *. Solution. We have two terms, each getting extremely large. It’s un-clear at first what happens when we subtract them. To get this equation intoanother form, we multiply and divide by the conjugate,

√x2 + 5x+

√x2 − x.

limx→∞

[√x2 + 5x−

√x2 − x

]= lim

x→∞

[(√x2 + 5x−

√x2 − x)(

√x2 + 5x+

√x2 − x)√

x2 + 5x+√x2 − x

]

= limx→∞

(x2 + 5x)− (x2 − x)√x2 + 5x+

√x2 − x

= limx→∞

6x√x2 + 5x+

√x2 − x

Now we divide the numerator and denominator by x. In the case of the denomi-nator, since x > 0, x =

√x2.

= limx→∞

6(x)√x2

√1 + 5

x+√x2

√1− 1

x

699

Page 716: CLP-1 Differential Calculus

SOLUTIONS TO EXERCISES

= limx→∞

6(x)

(x)√

1 + 5x

+ (x)√

1− 1x

= limx→∞

6√1 + 5

x+√

1− 1x

=6√

1 + 0 +√

1− 0= 3

1.5.2.10. *. Solution. Note that for large negative x, the first term in the denom-inator

√4x2 + x ≈

√4x2 = |2x| = −2x not +2x. A good way to avoid incorrectly

computing√x2 when x is negative is to define y = −x and express everything in

terms of y. That’s what we’ll do.

limx→−∞

3x√4x2 + x− 2x

= limy→+∞

−3y√4y2 − y + 2y

= limy→+∞

−3y

y√

4− 1y

+ 2y

= limy→+∞

−3√4− 1

y+ 2

=−3√

4− 0 + 2since 1/y → 0 as y → +∞

= −3

4

1.5.2.11. *. Solution. The highest power of x in the denominator is x2, so wedivide the numerator and denominator by x2:

limx→−∞

1− x− x2

2x2 − 7= lim

x→−∞1/x2 − 1/x− 1

2− 7/x2

=0− 0− 1

2− 0= −1

2

1.5.2.12. *. Solution.

limx→∞

(√x2 + x− x

)= lim

x→∞

(√x2 + x− x

)(√x2 + x+ x

)√x2 + x+ x

= limx→∞

(x2 + x)− x2

√x2 + x+ x

= limx→∞

x√x2 + x+ x

= limx→∞

1√1 + 1

x+ 1

=1

2

1.5.2.13. *. Solution. We have, after dividing both numerator and denominator

700

Page 717: CLP-1 Differential Calculus

SOLUTIONS TO EXERCISES

by x2 (which is the highest power of the denominator) that

5x2 − 3x+ 1

3x2 + x+ 7=

5− 3x

+ 1x2

3 + 1x

+ 7x2

.

Since 1/x→ 0 and also 1/x2 → 0 as x→ +∞, we conclude that

limx→+∞

5x2 − 3x+ 1

3x2 + x+ 7=

5

3.

1.5.2.14. *. Solution. We have, after dividing both numerator and denominatorby x (which is the highest power of the denominator) that

√4x+ 2

3x+ 4=

√4x

+ 2x2

3 + 4x

.

Since 1/x→ 0 and also 1/x2 → 0 as x→ +∞, we conclude that

limx→+∞

√4x+ 2

3x+ 4=

0

3= 0.

1.5.2.15. *. Solution. The dominant terms in the numerator and denominatorhave order x3. Taking out that common factor we get

4x3 + x

7x3 + x2 − 2=

4 + 1x2

7 + 1x− 2

x3

.

Since 1/xa → 0 as x→ +∞ (for a > 0), we conclude that

limx→+∞

4x3 + x

7x3 + x2 − 2=

4

7.

1.5.2.16. Solution.

• Solution 1

We want to factor out x, the highest power in the denominator. Since ourlimit only sees negative values of x, we must remember that 4

√x4 = |x| =

−x, although 3√x3 = x.

limx→−∞

3√x2 + x− 4

√x4 + 5

x+ 1= lim

x→−∞

3

√x3( 1

x+ 1

x2)− 4

√x4(1 + 5

x4)

x(1 + 1x)

= limx→−∞

3√x3 3

√1x

+ 1x2− 4√x4 4

√1 + 5

x4

x(1 + 1x)

= limx→−∞

x 3

√1x

+ 1x2− (−x) 4

√1 + 5

x4

x(1 + 1x)

701

Page 718: CLP-1 Differential Calculus

SOLUTIONS TO EXERCISES

= limx→−∞

3

√1x

+ 1x2

+ 4

√1 + 5

x4

1 + 1x

=3√

0 + 0 + 4√

1 + 0

1 + 0= 1

• Solution 2

Alternately, we can use the transformation limx→−∞

f(x) = limx→∞

f(−x). Then

we only look at positive values of x, so roots behave nicely: 4√x4 = |x| = x.

limx→−∞

3√x2 + x− 4

√x4 + 5

x+ 1= lim

x→∞

3√

(−x)2 − x− 4√

(−x)4 + 5

−x+ 1

= limx→∞

3√x2 − x− 4

√x4 + 5

−x+ 1

= limx→∞

3√x3 3

√1x− 1

x2− 4√x4 4

√1 + 5

x4

x(−1 + 1x)

= limx→∞

x 3

√1x− 1

x2− x 4

√1 + 5

x4

x(−1 + 1x)

= limx→∞

3

√1x− 1

x2− 4

√1 + 5

x4

−1 + 1x

=3√

0− 0− 4√

1 + 0

−1 + 0=−1

−1= 1

1.5.2.17. *. Solution. We have, after dividing both numerator and denominatorby x3 (which is the highest power of the denominator) that:

limx→∞

5x2 + 10

3x3 + 2x2 + x= lim

x→∞

5x

+ 10x3

3 + 2x

+ 1x2

=0

3= 0.

1.5.2.18. Solution. Since we only consider negative values of x,√x2 = |x| = −x.

limx→−∞

x+ 1√x2

= limx→−∞

x+ 1

−x= lim

x→−∞x

−x +1

−x= lim

x→−∞−1− 1

x

= −1

1.5.2.19. Solution. Since we only consider positive values of x,√x2 = |x| = x.

limx→∞

x+ 1√x2

= limx→∞

x+ 1

x

702

Page 719: CLP-1 Differential Calculus

SOLUTIONS TO EXERCISES

= limx→∞

1 +1

x= 1 + 0 = 1

1.5.2.20. *. Solution. When x < 0, |x| = −x and so limx→∞

sin

2· |x|x

)+

1

x=

sin(−π/2) = −1.

1.5.2.21. *. Solution. We divide both the numerator and the denominator bythe highest power of x in the denominator, which is x. Since x < 0, we have√x2 = |x| = −x, so that

√x2 + 5

x= −

√x2 + 5

x2= −

√1 +

5

x2.

Since 1/x→ 0 and also 1/x2 → 0 as x→ −∞, we conclude that

limx→−∞

3x+ 5√x2 + 5− x

= limx→−∞

3 + 5x

−√

1 + 5x2− 1

=3

−1− 1= −3

2.

1.5.2.22. *. Solution. We divide both the numerator and the denominator bythe highest power of x in the denominator, which is x. Since x < 0, we have√x2 = |x| = −x, so that

√4x2 + 15

x=

√4x2 + 15

−√x2

= −√

4x2 + 15

x2= −

√4 +

15

x2.

Since 1/x→ 0 and also 1/x2 → 0 as x→ −∞, we conclude that

limx→−∞

5x+ 7√4x2 + 15− x

= limx→−∞

5 + 7x

−√

4 + 15x2− 1

=5

−2− 1= −5

3.

1.5.2.23. Solution.

limx→−∞

3x7 + x5 − 15

4x2 + 32x= lim

x→−∞

x2(3x5 + x3 − 15x2

)

x2(4 + 32x

)

= limx→−∞

3x5 + x3 − 15x2

4 + 32x

= limx→+∞

3(−x)5 + (−x)3 − 15(−x)2

4 + 32−x

= limx→+∞

−3x5 − x3 − 15x2

4− 32x

= −∞

1.5.2.24. *. Solution. We multiply and divide the expression by its conjugate,

703

Page 720: CLP-1 Differential Calculus

SOLUTIONS TO EXERCISES(√n2 + 5n+ n

).

limn→∞

(√n2 + 5n− n

)= lim

n→∞

(√n2 + 5n− n

)(√n2 + 5n+ n√n2 + 5n+ n

)

= limn→∞

(n2 + 5n)− n2

√n2 + 5n+ n

= limn→∞

5n√n2 + 5n+ n

= limn→∞

5 · n√n2

√1 + 5

n+ n

Since n > 0, we can simplify√n2 = n.

= limn→∞

5 · nn√

1 + 5n

+ n

= limn→∞

5√1 + 5

n+ 1

=5√

1 + 0 + 1=

5

2

1.5.2.25. Solution.

• Solution 1:

When a approaches 0 from the right, the numerator approaches negative

infinity, and the denominator approaches −1. So, lima→0+

a2 − 1a

a− 1=∞.

More precisely, using Theorem 1.5.9:

lima→0+

1

a= +∞

Also, lima→0+

a2 = 0

So, using Theorem 1.5.9,

lima→0+

a2 − 1

a= −∞

Furthermore, lima→0+

a− 1 = −1

So, using our theorem, lima→0+

a2 − 1a

a− 1=∞

• Solution 2:

Since a = 0 is not in the domain of our function, a reasonable impulse is to

704

Page 721: CLP-1 Differential Calculus

SOLUTIONS TO EXERCISES

simplify.

a2 − 1a

a− 1

(aa

)=

a3 − 1

a(a− 1)=

(a− 1)(a2 + a+ 1)

a(a− 1)

So,

lima→0+

a2 − 1a

a− 1= lim

a→0+

(a− 1)(a2 + a+ 1)

a(a− 1)

= lima→0+

a2 + a+ 1

a

= lima→0+

a+ 1 +1

a=∞

1.5.2.26. Solution. Since x = 3 is not in the domain of the function, we simplify,hoping we can cancel a problematic term.

limx→3

2x+ 81

x−3+ 1

x2−9

= limx→3

2x+ 8x+3x2−9

+ 1x2−9

= limx→3

2x+ 8x+4x2−9

= limx→3

(2x+ 8)(x2 − 9)

x+ 4= 0

Exercises — Stage 31.5.2.27. Solution. First, we need a rational function whose limit at infinity is areal number. This means that the degree of the bottom is greater than or equalto the degree of the top. There are two cases: the denominator has higher degreethan the numerator, or the denominator has the same degree as the numerator.If the denominator has higher degree than the numerator, then lim

x→∞f(x) =

limx→−∞

f(x) = 0, so the limits are equal–not what we’re looking for.

If the denominator has the same degree as the numerator, then the limit as x goesto ±∞ is the ratio of the leading terms: again, the limits are equal. So no rationalfunction exists as described.

1.5.2.28. Solution. The amount of the substance that will linger long-term issome positive number–the substance will stick around. One example of a sub-stance that does this is the ink in a tattoo. (If the injection was of medicine, prob-ably it will be metabolized, and lim

t→∞c(t) = 0.)

Remark: it actually doesn’t make much sense to let t go to infinity: after a fewmillion hours, you won’t even have a body, so what is c(t) measuring? Oftenwhen we use formulas in the real world, there is an understanding that the areonly good for some fixed range. We often use the limit as t goes to infinity as astand-in for the function’s long-term behaviour.

1.6 · Continuity

705

Page 722: CLP-1 Differential Calculus

SOLUTIONS TO EXERCISES

1.6.4 · Exercises

Exercises — Stage 11.6.4.1. Solution. Many answers are possible; the tangent function behaves likethis.

1.6.4.2. Solution. If we let f be my height, the time of my birth is a, and now isb, then we know that f(a) ≤ 1 ≤ f(b). It is reasonable to assume that my heightis a continuous function. So by the IVT, there is some value c between a and bwhere f(c) = 1. That is, there is some time (we called it c) between my birth andtoday when I was exactly one meter tall.Notice the IVT does not say precisely what day I was one meter tall; it onlyguarantees that such a day occurred between my birth and today.

1.6.4.3. Solution. One example is f(x) =

0 when 0 ≤ x ≤ 1

2 when 1 < x ≤ 2. The IVT only

guarantees f(c) = 1 for some c in [0, 2] when f is continuous over [0, 2]. If f is notcontinuous, the IVT says nothing.

x

y

1.6.4.4. Solution. Yes. This is a straightforward application of IVT.

1.6.4.5. Solution. No. IVT says that f(x) = 0 for some x between 10 and 20, butit doesn’t have to be exactly half way between.

1.6.4.6. Solution. No. IVT says nothing about functions that are not guar-anteed to be continuous at the outset. It’s quite easy to construct a functionthat is as described, but not continuous. For example, the function picturedbelow, whose equation format is somewhat less enlightening than its graph:

f(x) =

−26

5x+ 65, 10 ≤ x < 15

−265x+ 91, 15 ≤ x ≤ 20

.

706

Page 723: CLP-1 Differential Calculus

SOLUTIONS TO EXERCISES

x

y

10 15 20

13

−13

1.6.4.7. Solution. True. Since f(t) is continuous at t = 5, that means limt→5

f(t) =

f(5). For that to be true, f(5) must exist — that is, 5 is in the domain of f(x).

1.6.4.8. Solution. True. Using the definition of continuity, limt→5

f(t) = f(5) = 17.

1.6.4.9. Solution. In general, false. If f(t) is continuous at t = 5, then f(5) = 17;if f(t) is discontinuous at t = 5, then f(5) either does not exist, or is a numberother than 17.

An example of a function with limt→5

f(t) = 17 6= f(5) is f(t) =

17 , t 6= 5

0 , t = 5,

shown below.

x

y

5

1.6.4.10. Solution. Since f(x) and g(x) are continuous at zero, and since g2(x)+1must be nonzero, then h(x) is continuous at 0 as well. According to the definitionof continuity, then lim

x→0h(x) exists and is equal to h(0) = 0f(0)

g2(0)+1= 0.

Since the limit limx→0

h(x) exists and is equal to zero, also the one-sided limit

limx→0+

h(x) exists and is equal to zero.

Exercises — Stage 2

707

Page 724: CLP-1 Differential Calculus

SOLUTIONS TO EXERCISES

1.6.4.11. Solution. Using the definition of continuity, we need k = limx→0

f(x).

Since the limit is blind to what actually happens to f(x) at x = 0, this is equivalent

to k = limx→0

x sin

(1

x

). So if we find the limit, we solve the problem.

As x gets small, sin(

1x

)goes a little crazy (see example 1.3.5), so let’s get rid of it

by using the Squeeze Theorem. We can bound the function above and below, butwe should be a little careful about whether we’re going from the left or the right.The reason we need to worry about direction is illustrated with the followingobservation:If a ≤ b and x > 0, then xa ≤ xb. (For example, plug in x = 1, a = 2, b = 3.) Butif a ≤ b and x < 0, then xa ≥ xb. (For example, plug in x = −1, a = 2, b = 3.) Sofirst, let’s find lim

x→0−sin(

1x

). When x < 0,

1 ≥ sin

(1

x

)≥ −1

so: x(1) ≤ x sin

(1

x

)≤ x(−1)

and limx→0−

x = limx→0−

−x = 0, so by the Squeeze Theorem, also limx→0−

x sin

(1

x

)= 0.

Now, let’s find limx→0+

x sin

(1

x

). When x > 0,

−1 ≤ sin

(1

x

)≤ 1

so: x(−1) ≤ xsin

(1

x

)≤ x(1)

and limx→0+

x = limx→0+

−x = 0, so by the Squeeze Theorem, also limx→0+

x sin

(1

x

)= 0.

Since the limits from the left and right agree, we conclude limx→0

x sin

(1

x

)= 0, so

when k = 0, the function is continuous at x = 0.

1.6.4.12. Solution. Since f is a polynomial, it is continuous over all real num-bers. f(0) = 1 < 12345 and f(12345) = 123453 + 123452 + 12345 + 1 > 12345(since all terms are positive). So by the IVT, f(c) = 12345 for some c between 0and 12345.

1.6.4.13. *. Solution. f(x) is a rational function and so is continuous exceptwhen its denominator is zero. That is, except when x = 1 and x = −1.

1.6.4.14. *. Solution. The function is continuous when x2−1 > 0, i.e. (x−1)(x+1) > 0, which yields the intervals (−∞,−1) ∪ (1,+∞).

1.6.4.15. *. Solution. The function 1/√x is continuous on (0,+∞) and the func-

708

Page 725: CLP-1 Differential Calculus

SOLUTIONS TO EXERCISES

tion cos(x) + 1 is continuous everywhere.So 1/

√cos(x) + 1 is continuous except when cosx = −1. This happens when

x is an odd multiple of π. Hence the function is continuous except at x =±π,±3π,±5π, · · ·.1.6.4.16. *. Solution. The function is continuous when sin(x) 6= 0. That is, whenx is not an integer multiple of π.

1.6.4.17. *. Solution. The function is continuous for x 6= c since each of thosetwo branches are polynomials. So, the only question is whether the function iscontinuous at x = c; for this we need

limx→c−

f(x) = f(c) = limx→c+

f(x).

We computelimx→c−

f(x) = limx→c−

8− cx = 8− c2;

f(c) = 8− c · c = 8− c2 and

limx→c+

f(x) = limx→c+

x2 = c2.

So, we need 8− c2 = c2, which yields c2 = 4, i.e. c = −2 or c = 2.

1.6.4.18. *. Solution. The function is continuous for x 6= 0 since x2 +c and cos cxare continuous everywhere. It remains to check continuity at x = 0. To do thiswe must check that the following three are equal.

limx→0+

f(x) = limx→0+

x2 + c = c

f(0) = c

limx→0−

f(x) = limx→0−

cos cx = cos 0 = 1

Hence when c = 1 we have the limits agree.

1.6.4.19. *. Solution. The function is continuous for x 6= c since each of thosetwo branches are defined by polynomials. Thus, the only question is whether thefunction is continuous at x = c. Furthermore,

limx→c−

f(x) = c2 − 4

andlimx→c+

f(x) = f(c) = 3c .

For continunity we need both limits and the value to agree, so f is continuous ifand only if c2 − 4 = 3c, that is if and only if

c2 − 3c− 4 = 0 .

Factoring this as (c− 4)(c+ 1) = 0 yields c = −1 or c = +4.

709

Page 726: CLP-1 Differential Calculus

SOLUTIONS TO EXERCISES

1.6.4.20. *. Solution. The function is continuous for x 6= c since each of thosetwo branches are polynomials. So, the only question is whether the function iscontinuous at x = c; for this we need

limx→2c−

f(x) = f(2c) = limx→2c+

f(x).

We computelim

x→2c−f(x) = lim

x→2c−6− cx = 6− 2c2;

f(2c) = 6− c · 2c = 6− 2c2 and

limx→2c+

f(x) = limx→2c+

x2 = 4c2.

So, we need 6− 2c2 = 4c2, which yields c2 = 1, i.e. c = −1 or c = 1.

Exercises — Stage 31.6.4.21. Solution. This isn’t the kind of equality that we can just solve; we’llneed a trick, and that trick is the IVT. The general idea is to show that sinx issomewhere bigger, and somewhere smaller, than x − 1. However, since the IVTcan only show us that a function is equal to a constant, we need to slightly adjustour language. Showing sinx = x − 1 is equivalent to showing sinx − x + 1 = 0,so let f(x) = sinx− x+ 1, and let’s show that it has a real root.First, we need to note that f(x) is continuous (otherwise we can’t use the IVT).Now, we need to find a value of x for which it is positive, and for which it’s neg-ative. By checking a few values, we find f(0) is positive, and f(100) is negative.So, by the IVT, there exists a value of x (between 0 and 100) for which f(x) = 0.Therefore, there exists a value of x for which sinx = x− 1.

1.6.4.22. *. Solution. We let f(x) = 3x − x2. Then f(x) is a continuous function,since both 3x and x2 are continuous for all real numbers.We find a value a such that f(a) > 0. We observe immediately that a = 0 workssince

f(0) = 30 − 0 = 1 > 0.

We find a value b such that f(b) < 0. We see that b = −1 works since

f(−1) =1

3− 1 < 0.

So, because f(x) is continuous on (−∞,∞) and f(0) > 0 while f(−1) < 0, thenthe Intermediate Value Theorem guarantees the existence of a real number c inthe interval (−1, 0) such that f(c) = 0.

1.6.4.23. *. Solution. We let f(x) = 2 tan(x) − x − 1. Then f(x) is a continuousfunction on the interval (−π/2, π/2) since tan(x) = sin(x)/ cos(x) is continuouson this interval, while x+ 1 is a polynomial and therefore continuous for all realnumbers.We find a value a ∈ (−π/2, π/2) such that f(a) < 0. We observe immediately that

710

Page 727: CLP-1 Differential Calculus

SOLUTIONS TO EXERCISES

a = 0 works since

f(0) = 2 tan(0)− 0− 1 = 0− 1 = −1 < 0.

We find a value b ∈ (−π/2, π/2) such that f(b) > 0. We see that b = π/4 workssince

f(π/4) = 2 tan(π/4)− π/4− 1 = 2− π/4− 1 = 1− π/4= (4− π)/4 > 0

because 3 < π < 4.So, because f(x) is continuous on [0, π/4] and f(0) < 0 while f(π/4) > 0, thenthe Intermediate Value Theorem guarantees the existence of a real number c ∈(0, π/4) such that f(c) = 0.

1.6.4.24. *. Solution. Let f(x) =√

cos(πx) − sin(2πx) − 1/2. This function iscontinuous provided cos(πx) ≥ 0. This is true for 0 ≤ x ≤ 1

2.

Now f takes positive values on [0, 1/2]:

f(0) =√

cos(0)− sin(0)− 1/2 =√

1− 1/2 = 1/2.

And f takes negative values on [0, 1/2]:

f(1/2) =√

cos(π/2)− sin(π)− 1/2 = 0− 0− 1/2 = −1/2

(Notice that f(1/3) = (√

2−√

3)/2− 1/2 also works)So, because f(x) is continuous on [0, 1/2) and f(0) > 0 while f(1/2) < 0, thenthe Intermediate Value Theorem guarantees the existence of a real number c ∈(0, 1/2) such that f(c) = 0.

1.6.4.25. *. Solution. We let f(x) =1

cos2(πx)− x− 3

2. Then f(x) is a continuous

function on the interval (−1/2, 1/2) since cosx is continuous everywhere andnon-zero on that interval.The function f takes negative values. For example, when x = 0:

f(0) =1

cos2(0)− 0− 3

2= 1− 3

2= −1

2< 0.

It also takes positive values, for instance when x = 1/4:

f(1/4) =1

(cos π/4)2− 1

4− 3

2

=1

1/2− 1 + 6

4

= 2− 7/4 = 1/4 > 0.

By the IVT there is c, 0 < c < 1/3 such that f(c) = 0, in which case1

(cos πc)2= c+

3

2.

711

Page 728: CLP-1 Differential Calculus

SOLUTIONS TO EXERCISES

1.6.4.26. Solution. f(x) is a polynomial, so it’s continuous everywhere. If wecan find values a and b so that f(a) > 0 and f(b) < 0, then by the IVT, there willexist some c in (a, b) where f(c) = 0; that is, there is a root in the interval [a, b].Let’s start plugging in easy values of x.f(0) = 15, and f(1) = 1 − 15 + 9 − 18 + 15 = −8. Since 0 is between f(0) andf(1), and since f is continuous, by IVT there must be some x in [0, 1] for whichf(x) = 0: that is, there is some root in [0, 1].That was the easiest interval to find. If you keep playing around, you find twomore. f(−1) = 26 (positive) and f(−2) = −1001 (negative), so there is a root in[−2,−1].The arithmetic is nasty, but there is also a root in [14, 15].This is an important trick. For high-degree polynomials, it is often impossibleto get the exact values of the roots. Using the IVT, we can at least give a rangewhere a root must be.

1.6.4.27. Solution. Let f(x) = x3. Since f is a polynomial, it is continuouseverywhere. If f(a) < 7 < f(b), then 3

√7 is somewhere between a and b. If we

can find a and b that satisfy these inequalities, and are very close together, thatwill give us a good approximation for 3

√7.

• Let’s start with integers. 13 < 7 < 23, so 3√

7 is in the interval (1, 2).

• Let’s narrow this down, say by testing f(1.5). (1.5)3 = 3.375 < 7, so 3√

7 isin the interval (1.5, 2).

• Let’s narrow further, say by testing f(1.75). (1.75)3 ≈ 5.34 < 7, so 3√

7 is inthe interval (1.75, 2).

• Testing various points, we find f(1.9) < 7 < f(2), so 3√

7 is between 1.9 and2.

• By testing more, we find f(1.91) < 7 < f(1.92), so 3√

7 is in (1.91, 1.92).

• In order to get an approximation for 3√

7 that is rounded to two decimalplaces, we have to know whether 3

√7 is greater or less than 1.915; indeed

f(1.915) ≈ 7.02 > 7, so 3√

7 < 1.915; then rounded to two decimal places,3√

7 ≈ 1.91.

If this seems like the obvious way to approximate 3√

7, that’s good. The IVT is aformal statement of a very intuitive principle.

1.6.4.28. Solution.

• If f(a) = g(a), or f(b) = g(b), then we simply take c = a or c = b.

• Suppose f(a) 6= g(a) and f(b) 6= g(b). Then f(a) < g(a) and g(b) < f(b),so if we define h(x) = f(x) − g(x), then h(a) < 0 and h(b) > 0. Since his the difference of two functions that are continuous over [a, b], also h iscontinuous over [a, b]. So, by the Intermediate Value Theorem, there exists

712

Page 729: CLP-1 Differential Calculus

SOLUTIONS TO EXERCISES

some c ∈ (a, b) with h(c) = 0; that is, f(c) = g(c).

2 · Derivatives2.1 · Revisiting Tangent Lines2.1.2 · Exercises

Exercises — Stage 12.1.2.1. Solution. If Q is to the left of the y axis, the line through Q and P isincreasing, so the secant line has positive slope. If Q is to the right of the y axis,the line through Q and P is decreasing, so the secant line has negative slope.

2.1.2.2. Solution. 2.1.2.2.a By drawing a few pictures, it’s easy to see that slidingQ closer to P , the slope of the secant line increases.2.1.2.2.b Since the slope of the secant line increases the closer Q gets to P , thatmeans the tangent line (which is the limit as Q approaches P ) has a larger slopethan the secant line between Q and P (using the location where Q is right now).Alternately, by simply sketching the tangent line at P , we can see that has asteeper slope than the secant line between P and Q.

2.1.2.3. Solution. The slope of the secant line will bef(2)− f(−2)

2− (−2)=

f(2)− f(−2)

4, in every part. So, if two lines have the same slope, that means

their differences f(2)− f(−2) will be the same.The graphs in (a),(c), and (e) all have f(2)− f(−2) = 1, so they all have the samesecant line slope. The graphs in (b) and (f) both have f(2)− f(−2) = −1, so theyboth have the same secant line slope. The graph in (d) has f(2)− f(−2) = 0, andit is the only graph with this property, so it does not share its secant line slopewith any of the other graphs.

Exercises — Stage 22.1.2.4. Solution. A good approximation from the graph is f(5) = 0.5. We wantto find a secant line whose endpoints are both very close to x = 5, but that alsogive us clear y-values. It looks like f(5.25) ≈ 1, and f(4.75) ≈ 1

8. The secant line

from x = 5 to x = 5.25 has approximate slopef(5.25)− f(5)

5.25− 5≈ 1− .5

.25= 2. The

secant line from x = 5 to x = 4.75 has approximate slope0.5− 1

8

5− 4.75=

3

2.

The graph increases more and more quickly (gets steeper and steeper), so itmakes sense that the secant line to the left of x = 5 has a smaller slope thanthe secant line to the right of x = 5. Also, if you’re taking secant lines that haveendpoints farther out from x = 5, you’ll notice that the slopes of the secant lineschange quite dramatically. You have to be very, very close to x = 5 to get anykind of accuracy.If we split the difference, we might approximate the slope of the secant line to be

713

Page 730: CLP-1 Differential Calculus

SOLUTIONS TO EXERCISES

the average of 32

and 2, which is 74.

Another way to try to figure out the tangent line is by carefully drawing it in witha ruler. This is shown here in blue:

x

y

1 5

1

It’s much easier to take the slope of a line than a curve, and this one looks like ithas slope about 1.5. However, we drew this with a computer: by hand it’s muchharder to draw an accurate tangent line. (That’s why we need calculus!)The actual slope of the tangent line to the function at x = 5 is about 1.484. This isextremely hard to figure out just from the graph–by hand, a guess between 1.25and 1.75 would be very accurate.

2.1.2.5. Solution. There is only one tangent line to f(x) at P (shown in blue),but there are infinitely many choices of Q and R (one possibility shown in red).One easy way to sketch the secant line on paper is to draw any line parallel tothe tangent line, and choose two intercepts with y = f(x).

x

y

y = f(x)

P

Q

R

2.1.2.6. Solution. Any place the graph looks flat (if you imagine zooming in) iswhere the tangent line has slope 0. This occurs three times.

714

Page 731: CLP-1 Differential Calculus

SOLUTIONS TO EXERCISES

x

y

y = f(x)

Notice that two of the indicated points are at a low point and a high point, re-spectively. Later, we’ll use these places where the tangent line has slope zero tofind where a graph achieves its biggest and smallest values.

2.2 · Definition of the Derivative2.2.4 · Exercises

Exercises — Stage 12.2.4.1. Solution. The function shown is a line, so it has a constant slope–(a) .Since the function is always increasing, f ′ is always positive, so also (d) holds.Remark: it does not matter that the function itself is sometimes negative; theslope is always positive because the function is always increasing. Also, sincethe slope is constant, f ′ is neither increasing nor decreasing: it is the function thatis increasing, not its derivative.

2.2.4.2. Solution. The function is always decreasing, so f ′ is always negative,option (e). However, the function alternates between being more and less steep,so f ′ alternates between increasing and decreasing several times, and no otheroptions hold.Remark: f is always positive, but (d) does not hold!

2.2.4.3. Solution. At the left end of the graph, f is decreasing rapidly, so f ′ isa strongly negative number. Then as we move towards x = 0, f decreases lessrapidly, so f ′ is a less strongly negative number. As we pass 0, f increases, so f ′ isa positive number. As we move to the right, f increases more and more rapidly,so f ′ is an increasing positive number. This description tells us that f ′ increasesfor the entire range shown. So (b) holds, but not (a) or (c). Since f ′ is negative tothe left of the y axis, and positive to the right of it, also (d) and (e) do not hold.

2.2.4.4. *. Solution. By definition, f(x) = x3 is differentiable at x = 0 if the limit

limh→0

f(h)− f(0)

h= lim

h→0

h3 − 0

h

exists.

715

Page 732: CLP-1 Differential Calculus

SOLUTIONS TO EXERCISES

2.2.4.5. Solution. f ′(−1) does not exist, because to the left of x = −1 the slopeis a pretty big positive number (looks like around +1) and to the right the slopeis −1/4. Since the derivative involves a limit, that limit needs to match the limitfrom the left and the limit from the right. The sharp angle made by the graphat x = −1 indicates that the left and right limits do not match, so the derivativedoes not exist.f ′(3) also does not exist. One way to see this is to notice that the function isdiscontinuous here. More viscerally, note that f(3) = 1, so as we take secant lineswith one endpoint (3, 1), and the other endpoint just to the right of x = 3, we getslopes that are more and more strongly negative, as shown in the picture below.If we take the limit of the slopes of these secant lines as x goes to 3 from the right,we get −∞. (This certainly doesn’t match the slope from the left, which is −1

4.)

x

y

1

1

At x = −3, there is some kind of “change” in the graph; however, it is a smoothcurve, so the derivative exists here.

2.2.4.6. Solution. True. The definition of the derivative tells us that

f ′(a) = limh→0

f(a+ h)− f(a)

h,

if it exists. We know from our work with limits that if both one-sided limitslimh→0−

f(a+ h)− f(a)

hand lim

h→0+

f(a+ h)− f(a)

hexist and are equal to each other,

then limh→0

f(a+ h)− f(a)

hexists and has the same value as the one-sided limits.

So, since the one-sided limits exist and are equal to one, we conclude f ′(a) existsand is equal to one.

2.2.4.7. Solution. In general, this is false. The key problem that can arise is thatf(x) might not be continuous at x = 1. One example is the function

f(x) =

x x < 0

x− 1 x ≥ 0

where f ′(x) = 1 whenever x 6= 0 (so in particular, limx→0−

f ′(x) = limx→0+

f ′(x) = 1)

but f ′(0) does not exist.There are two ways to see that f ′(0) does not exist. One is to notice that f is notcontinuous at x = 0.

716

Page 733: CLP-1 Differential Calculus

SOLUTIONS TO EXERCISES

x

y

y = f(x)

Another way to see that f ′(0) does not exist is to use the definition of the deriva-tive. Remember, in order for a limit to exist, both one-sided limits must exist.Let’s consider the limit from the left. If h → 0−, then h < 0, so f(h) is equal to h(not h− 1).

limh→0−

f(0 + h)− f(0)

h= lim

h→0−

(h)− (−1)

h

= limh→0−

h+ 1

h

= limh→0−

1 +1

h

= −∞

In particular, this limit does not exist. Since the one-sided limit does not exist,

limh→0

f(0 + h)− f(0)

h= DNE

and so f ′(0) does not exist.

2.2.4.8. Solution. Using the definition of the derivative,

s′(t) = limh→0

s(t+ h)− s(t)h

The units of the numerator are meters, and the units of the denominator are sec-onds (since the denominator comes from the change in the input of the function).So, the units of s′(t) are metres per second.Remark: we learned that the derivative of a position function gives velocity. Inthis example, the position is given in metres, and the velocity is measured inmetres per second.

717

Page 734: CLP-1 Differential Calculus

SOLUTIONS TO EXERCISES

Exercises — Stage 22.2.4.9. Solution. We can use point-slope form to get the equation of the line,if we have a point and its slope. The point is given: (1, 6). The slope is thederivative:

y′(1) = limh→0

y(1 + h)− y(1)

h

= limh→0

[(1 + h)3 + 5]− [13 + 5]

h

= limh→0

[1 + 3h+ 3h2 + h3 + 5]− [1 + 5]

h

= limh→0

3h+ 3h2 + h3

h= lim

h→03 + 3h+ h2

= 3

So our slope is 3, which gives a line of equation y − 6 = 3(x− 1).

2.2.4.10. Solution. We set up the definition of the derivative.

f ′(x) = limh→0

f(x+ h)− f(x)

h

= limh→0

1x+h− 1

x

h

= limh→0

xx(x+h)

− x+hx(x+h)

h

= limh→0

x−(x+h)x(x+h)

h

= limh→0

−hx(x+h)

h

= limh→0

−1

x(x+ h)

=−1

x2

2.2.4.11. *. Solution. By definition

f ′(0) = limh→0

f(h)− f(0)

h= lim

h→0

h|h|h

= limh→0|h| = 0

In particular, the limit exists, so the derivative exists (and is equal to zero).

2.2.4.12. *. Solution. We set up the definition of the derivative.

f ′(x) = limh→0

f(x+ h)− f(x)

h= lim

h→0

1

h

( 2

x+ h+ 1− 2

x+ 1

)

718

Page 735: CLP-1 Differential Calculus

SOLUTIONS TO EXERCISES

= limh→0

2

h

(x+ 1)− (x+ h+ 1)

(x+ h+ 1)(x+ 1)

= limh→0

2

h

−h(x+ h+ 1)(x+ 1)

= limh→0

−2

(x+ h+ 1)(x+ 1)

=−2

(x+ 1)2

2.2.4.13. *. Solution.

f ′(x) = limh→0

f(x+ h)− f(x)

h= lim

h→0

1

h

( 1

(x+ h)2 + 3− 1

x2 + 3

)= lim

h→0

1

h

x2 − (x+ h)2

[(x+ h)2 + 3][x2 + 3]

= limh→0

1

h

−2xh− h2

[(x+ h)2 + 3][x2 + 3]

= limh→0

−2x− h[(x+ h)2 + 3][x2 + 3]

=−2x

[x2 + 3]2

2.2.4.14. Solution. The slope of the tangent line is the derivative. We set this upusing the same definition of the derivative that we always do. This limit is hardto take for general x, but easy when x = 0.

f ′(0) = limh→0

f(0 + h)− f(0)

h

= limh→0

h log10(2h+ 10)− 0

h= lim

h→0log10(2h+ 10) = log10(10) = 1

So, the slope of the tangent line is 1.

2.2.4.15. *. Solution.

f ′(x) = limh→0

f(x+ h)− f(x)

h= lim

h→0

1(x+h)2

− 1x2

h= lim

h→0

x2 − (x+ h)2

(x+ h)2x2h

= limh→0

−2xh− h2

(x+ h)2x2h= lim

h→0

−2x− h(x+ h)2x2

=−2x

x4

= − 2

x3

2.2.4.16. *. Solution. When x is not equal to 2, then the function isdifferentiable– the only place we have to worry about is when x is exactly 2.In order for f to be differentiable at x = 2, it must also be continuous at x = 2.

719

Page 736: CLP-1 Differential Calculus

SOLUTIONS TO EXERCISES

This forces x2∣∣x=2

=[ax+ b

]x=2

or

2a+ b = 4.

In order for a limit to exist, the left- and right-hand limits must exist and be equalto each other. Since a derivative is a limit, in order for f to be differentiable atx = 2, the left hand derivative of ax + b at x = 2 must be the same as the righthand derivative of x2 at x = 2. Since ax+b is a line, its derivative is a everywhere.We’ve already seen the derivative of x2 is 2x, so we need

a = 2x∣∣x=2

= 4.

So, the values of a and b that makes f differentiable everywhere are a = 4 andb = −4.

2.2.4.17. *. Solution. We plug in f(x) to the definition of a derivative. To eval-uate the limit, we multiply and divide by the conjugate of the numerator, thensimplify.

f ′(x) = limh→0

f(x+ h)− f(x)

h= lim

h→0

√1 + x+ h−

√1 + x

h

= limh→0

√1 + x+ h−

√1 + x

h

(√1 + x+ h+

√1 + x√

1 + x+ h+√

1 + x

)= lim

h→0

(1 + x+ h)− (1 + x)

h(√

1 + x+ h+√

1 + x)

= limh→0

h

h(√

1 + x+ h+√

1 + x)

= limh→0

1√1 + x+ h+

√1 + x

=1√

1 + x+ 0 +√

1 + x=

1

2√

1 + x

The domain of the function is [−1,∞). In particular, f(x) is defined when x = −1.However, f ′(x) is not defined when x = −1, so f ′(x) only exists over (−1,∞).Remark: lim

x→−1+f ′(x) = ∞, so the tangent line to f(x) at the point x = −1 has a

vertical slope.

Exercises — Stage 32.2.4.18. Solution. From Section 1.2, we see that the velocity is exactly thederivative.

v(t) = limh→0

s(t+ h)− s(t)h

= limh→0

(t+ h)4 − (t+ h)2 − t4 + t2

h

720

Page 737: CLP-1 Differential Calculus

SOLUTIONS TO EXERCISES

= limh→0

(t4 + 4t3h+ 6t2h2 + 4th3 + h4)− (t2 + 2th+ h2)− t4 + t2

h

= limh→0

4t3h+ 6t2h2 + 4th3 + h4 − 2th− h2

h= lim

h→04t3 + 6t2h+ 4th2 + h3 − 2t− h

= 4t3 − 2t

So, the velocity is given by v(t) = 4t3 − 2t.

2.2.4.19. *. Solution. The function is differentiable at x = 0 if the followinglimit:

limx→0

f(x)− f(0)

x− 0= lim

x→0

f(x)− 0

x= lim

x→0

f(x)

x

exists (note that we used the fact that f(0) = 0 as per the definition of the firstbranch which includes the point x = 0). We start by computing the left limit. Forthis computation, recall that if x < 0 then

√x2 = |x| = −x.

limx→0−

f(x)

x= lim

x→0−

√x2 + x4

x= lim

x→0−

√x2√

1 + x2

x

= limx→0

−x√

1 + x2

x= −1

Now, from the right:limx→0+

x cosx

x= lim

x→0+cosx = 1.

Since the limit from the left does not equal the limit from the right, the derivativedoes not exist at x = 0.

2.2.4.20. *. Solution. The function is differentiable at x = 0 if the followinglimit:

limx→0

f(x)− f(0)

x− 0= lim

x→0

f(x)− 0

x= lim

x→0

f(x)

x

exists (note that we used the fact that f(0) = 0 as per the definition of the firstbranch which includes the point x = 0).We start by computing the left limit.

limx→0−

f(x)

x= lim

x→0−

x cosx

x= lim

x→0−cosx = 1.

Now, from the right:

limx→0+

√1 + x− 1

x= lim

x→0+

√1 + x− 1

x·√

1 + x+ 1√1 + x+ 1

= limx→0+

1 + x− 1

x(√

1 + x+ 1)= lim

x→0+

1√1 + x+ 1

=1

2

Since the limit from the left does not equal the limit from the right, the derivativedoes not exist at x = 0.

721

Page 738: CLP-1 Differential Calculus

SOLUTIONS TO EXERCISES

2.2.4.21. *. Solution. The function is differentiable at x = 0 if the followinglimit:

limx→0

f(x)− f(0)

x− 0= lim

x→0

f(x)− 0

x= lim

x→0

f(x)

x

exists (note that we used the fact that f(0) = 0 as per the definition of the firstbranch which includes the point x = 0). We compute left and right limits; so

limx→0−

f(x)

x= lim

x→0−

x3 − 7x2

x= lim

x→0−x2 − 7x = 0

and

limx→0+

x3 cos(

1x

)x

= limx→0+

x2 · cos

(1

x

).

This last limit equals 0 by the Squeeze Theorem since

−1 ≤ cos

(1

x

)≤ 1

and so,

−x2 ≤ x2 · cos

(1

x

)≤ x2,

where in these inequalities we used the fact that x2 ≥ 0. Finally, sincelimx→0+ −x2 = limx→0+ x

2 = 0, the Squeeze Theorem yields that alsolimx→0+ x

2 cos(

1x

)= 0, as claimed.

Since the left and right limits match (they’re both equal to 0), we conclude thatindeed f(x) is differentiable at x = 0 (and its derivative at x = 0 is actually equalto 0).

2.2.4.22. *. Solution. The function is differentiable at x = 1 if the followinglimit:

limx→1

f(x)− f(1)

x− 1= lim

x→1

f(x)− 0

x− 1= lim

x→1

f(x)

x− 1

exists (note that we used the fact that f(1) = 0 as per the definition of the firstbranch which includes the point x = 0). We compute left and right limits; so

limx→1−

f(x)

x− 1= lim

x→1−

4x2 − 8x+ 4

x− 1= lim

x→1−

4(x− 1)2

x− 1

= limx→1−

4(x− 1) = 0

and

limx→1+

(x− 1)2 sin(

1x−1

)x− 1

= limx→1+

(x− 1) · sin(

1

x− 1

).

722

Page 739: CLP-1 Differential Calculus

SOLUTIONS TO EXERCISES

This last limit equals 0 by the Squeeze Theorem since

−1 ≤ sin

(1

x− 1

)≤ 1

and so,

−(x− 1) ≤ (x− 1) · sin(

1

x− 1

)≤ x− 1,

where in these inequalities we used the fact that x → 1+ yields positive valuesfor x− 1. Finally, since limx→1+ −x+ 1 = limx→1+ x− 1 = 0, the Squeeze Theoremyields that also limx→1+(x− 1) sin

(1

x−1

)= 0, as claimed.

Since the left and right limits match (they’re both equal to 0), we conclude thatindeed f(x) is differentiable at x = 1 (and its derivative at x = 1 is actually equalto 0).

2.2.4.23. . Solution. Many answers are possible; here is one.

x

y

1

1

The key is to realize that the few points you’re given suggest a pattern, but don’tguarantee it. You only know nine points; anything can happen in between.

2.2.4.24. Solution.

p′(x) = limh→0

p(x+ h)− p(x)

h

= limh→0

f(x+ h) + g(x+ h)− f(x)− g(x)

h

= limh→0

f(x+ h)− f(x) + g(x+ h)− g(x)

h

723

Page 740: CLP-1 Differential Calculus

SOLUTIONS TO EXERCISES

= limh→0

[f(x+ h)− f(x)

h+g(x+ h)− g(x)

h

](∗) =

[limh→0

f(x+ h)− f(x)

h

]+

[limh→0

g(x+ h)− g(x)

h

]= f ′(x) + g′(x)

At step (∗), we use the limit law that limx→a

[F (x) +G(x)] = limx→a

F (x) + limx→a

G(x),

as long as limx→a

F (x) and limx→a

G(x) exist. Because the problem states that f ′(x) and

g′(x) exist, we know that limh→0

f(x+ h)− f(x)

hand lim

h→0

g(x+ h)− g(x)

hexist, so

our work is valid.

2.2.4.25. Solution. 2.2.4.25.a Since y = f(x) = 2x and y = g(x) = x are straightlines, we don’t need the definition of the derivative (although you can use it ifyou like). f ′(x) = 2 and g′(x) = 1.2.2.4.25.b p(x) = 2x2, so p(x) is not a line: we use the definition of a derivative tofind p′(x).

p′(x) = limh→0

p(x+ h)− p(x)

h

= limh→0

2(x+ h)2 − 2x2

h

= limh→0

2x2 + 4xh+ 2h2 − 2x2

h

= limh→0

4xh+ 2h2

h= lim

h→04x+ 2h

= 4x

2.2.4.25.c No, p′(x) = 4x 6= 2 · 1 = f ′(x) · g′(x). In general, the derivative of aproduct is not the same as the derivative of the functions being multiplied.

2.2.4.26. *. Solution. We know that y′ = 2x. So, if we choose a point (α, α2) onthe curve y = x2, then the tangent line to the curve at that point has slope 2α.That is, the tangent line has equation

(y − α2) = 2α(x− α)

simplified, y = (2α)x− α2

So, if (1,−3) is on the tangent line, then

−3 = (2α)(1)− α2

⇐⇒ 0 = α2 − 2α− 3

⇐⇒ 0 = (α− 3)(α + 1)

⇐⇒ α = 3, or α = −1.

724

Page 741: CLP-1 Differential Calculus

SOLUTIONS TO EXERCISES

So, the tangent lines y = (2α)x− α2 are

y = 6x− 9 and y = −2x− 1.

2.2.4.27. *. Solution. Using the definition of the derivative, f is differentiableat 0 if and only if

limh→0

f(h)− f(0)

h

exists. In particular, this means f is differentiable at 0 if and only if both one-sided limits exist and are equal to each other.

When h < 0, f(h) = 0, so

limh→0−

f(h)− f(0)

h= lim

h→0−

0− 0

h= 0

So, f is differentiable at x = 0 if and only if

limh→0+

f(h)− f(0)

h= 0.

To evaluate the limit above, we note f(0) = 0 and, when h > 0, f(h) = ha sin(

1h

),

so

limh→0+

f(h)− f(0)

h= lim

h→0+

ha sin(

1h

)h

= limh→0+

ha−1 sin

(1

h

)We will spend the rest of this solution evaluating the limit above for differentvalues of a, to find when it is equal to zero and when it is not. Let’s consider thedifferent values that could be taken by ha−1.

• If a = 1, then a− 1 = 0, so ha−1 = h0 = 1 for all values of h. Then

limh→0+

ha−1 sin

(1

h

)= lim

h→0+sin

(1

h

)= DNE

(Recall that the function sin(

1x

)oscillates faster and faster as x goes to 0. We

first saw this behaviour in Example 1.3.5.)

• If a < 1, then a − 1 < 0, so limh→0+

ha−1 = ∞. (Since we have a negative

exponent, we are in effect dividing by a smaller and smaller positive number.

For example, if a = 12, then lim

h→0+ha−1 = lim

h→0+h−

12 = lim

h→0+

1√h

= ∞.) Since

sin(

1x

)goes back and forth between one and negative one,

limh→0+

ha−1 sin

(1

x

)= DNE

725

Page 742: CLP-1 Differential Calculus

SOLUTIONS TO EXERCISES

since as h goes to 0, the function oscillates between positive and negativenumbers of ever-increasing magnitude.

• If a > 1, then a− 1 > 0, so limh→0+

ha−1 = 0. Although sin(

1x

)oscillates wildly

near x = 0, it is bounded by −1 and 1. So,

(−1)ha−1 ≤ ha−1 sin

(1

h

)≤ ha−1

Since both limh→0+

(−1)ha−1 = 0 and limh→0+

ha−1 = 0, by the Squeeze Theorem,

limh→0+

ha−1 sin

(1

x

)= 0

as well.

In the above cases, we learned limh→0+

f(h)− f(0)

h= lim

h→0+ha−1 sin

(1

x

)= 0 when

a > 1, and

limh→0+

f(h)− f(0)

h= lim

h→0+ha−1 sin

(1

x

)6= 0 when a ≤ 1.

So, f is differentiable at x = 0 if and only if a > 1.

2.3 · Interpretations of the Derivative2.3.3 · Exercises

Exercises — Stage 2

2.3.3.1. Solution. 2.3.3.1.a The slope of the secant line ish(24)− h(0)

24− 0

m

hr; this

is the change in height over the first day divided by the number of hours in thefirst day. So, it is the average rate of change of the height over the first day,measured in meters per hour.2.3.3.1.b Consider 2.3.3.1.a. The secant line gives the average rate of change of theheight of the dam; as we let the second point of the secant line get closer andcloser to (0, h(0)), its slope approximates the instantaneous rate of change of theheight of the water. So the slope of the tangent line is the instantaneous rate ofchange of the height of the water at the time t = 0, measured in m

hr.

2.3.3.2. Solution. p′(t) = limh→0

p(t+ h)− p(t)h

≈ p(t+ 1)− p(t)1

= p(t + 1) − p(t),

or the difference in profit caused by the sale of the (t+ 1)st widget. So, p′(t) is theprofit from the (t+ 1)st widget, so p′(t) is the profit per widget.

2.3.3.3. Solution. How quickly the temperature is changing per unit change ofdepth, measured in degrees per metre. In an ordinary body of water, the temper-ature near the surface (d = 0) is pretty variable, depending on the sun, but deepdown it is more stable (unless there are heat sources). So, one might reasonably

726

Page 743: CLP-1 Differential Calculus

SOLUTIONS TO EXERCISES

expect that |T ′(d)| is larger when d is near 0.

2.3.3.4. Solution.

C ′(w) = limh→0

C(w + h)− C(w)

h≈ C(w + 1)− C(w)

1= C(w + 1)− C(w)

which is the number of calories in C(w + 1) grams minus the number of caloriesin C(w) grams. This is the number of calories per gram.

2.3.3.5. Solution. The rate of change of velocity is acceleration. (If your ve-locity is increasing, you’re accelerating; if your velocity is decreasing, you havenegative acceleration.)

2.3.3.6. Solution. The rate of change in this case will be the relationship be-

tween the heat added and the temperature change. limh→0

T (j + h)− T (j)

h≈

T (j + 1)− T (j)

1= T (j + 1) − T (j), or the change in temperature after the ap-

plication of one joule. (This is closely related to heat capacity and to specific heat— there’s a nice explanation of this on Wikipedia.)

2.3.3.7. Solution. As usual, it is instructive to think about the definition of thederivative:

P ′(T ) = limh→0

P (T + h) = P (T )

h≈ P (T + 1)− P (t)

1= P (T + 1)− P (T )

This is the difference in population between two hypothetical populations, raisedone degree in temperature apart. So, it is the number of extra individuals thatexist in the hotter experiment (with the understanding that this number couldbe negative, as one would expect in conditions that are hotter than the bacteriaprefer). So P ′(T ) is the number of bacteria added to the colony per degree.

Exercises — Stage 32.3.3.8. Solution. R′(t) is the rate at which the wheel is rotating measured inrotations per second. To convert to degrees, we multiply by 360: 360R′(t) .

2.3.3.9. Solution. If P ′(t) is positive, your sample is below the ideal temper-ature, because adding heat increases the population. If P ′(t) is negative, yoursample is above the ideal temperature, because adding heat decreases the popu-lation. If P ′(t) = 0, then adding a little bit of heat doesn’t change the population,but it’s unclear why this is. Perhaps your sample is deeply frozen, and addingheat doesn’t change the fact that your population is 0. Perhaps your sample isboiling, and again, changing the heat a little will keep the population constant at“none.” But also, at the ideal temperature, you would expect P ′(t) = 0. This is

727

Page 744: CLP-1 Differential Calculus

SOLUTIONS TO EXERCISES

best seen by noting in the curve below, the tangent line is horizontal at the peak.

x

y

frozen, P ′(t) = 0 boiled, P ′(t) = 0ideal, P ′(t) = 0

too co

ld, P′ (t)

>0 too hot, P ′(t) <

0

2.4 · Arithmetic of Derivatives - a Differentiation Toolbox2.4.2 · Exercises

Exercises — Stage 12.4.2.1. Solution. True: this is exactly what the Sum Rule states.

2.4.2.2. Solution. False, in general. The product rule tells us ddxf(x)g(x) =

f ′(x)g(x) + f(x)g′(x). An easy example of why we can’t do it the other way is totake f(x) = g(x) = x. Then the equation becomes d

dxx2 = (1)(1), which is false.

2.4.2.3. Solution. True: the quotient rule tells us

d

dx

f(x

g(x)

=g(x)f ′(x)− f(x)g′(x)

g2(x)=g(x)f ′(x)

g2(x)− f(x)g′(x)

g2(x)

=f ′(x)

g(x)− f(x)g′(x)

g2(x)

2.4.2.4. Solution. If you’re creative, you can find lots of ways to differentiate!

• Constant multiple: g′(x) = 3f ′(x).

• Product rule:

g′(x) =d

dx3f(x) + 3f ′(x) = 0f(x) + 3f ′(x) = 3f ′(x)

• Sum rule:

g′(x) =d

dxf(x) + f(x) + f(x) = f ′(x) + f ′(x) + f ′(x)

= 3f ′(x)

728

Page 745: CLP-1 Differential Calculus

SOLUTIONS TO EXERCISES

• Quotient rule:

g′(x) =d

dx

f(x)

13

=

13f ′(x)− f(x)(0)

19

=13f ′(x)

19

= 9

(1

3

)f ′(x) = 3f ′(x)

All rules give g′(x) = 3f ′(x).

Exercises — Stage 22.4.2.5. Solution. f ′(x) = 5 · 3x4 + 2

3· 4x2/3−1 = 15x4 + 8

3x−1/3

2.4.2.6. Solution. We have already seen ddx√x = 1

2√x, but if you forget the

formula, it’s easy to find using the power rule: ddx√x = d

dx

x1/2

= 1

2x−1/2 =

12√x.

Now:

f ′(x) = (2)(8√x− 9x) + (2x+ 5)

(8

2√x− 9

)= 16

√x− 18x+ (2x+ 5)

(4√x− 9

)= −36x+ 24

√x+

20√x− 45

2.4.2.7. *. Solution. We compute the derivative of x3 as being 3x2, which eval-uated at x = 1

2yields 3

4. Since we also compute

(12

)3= 1

8, then the equation of the

tangent line is

y − 1

8=

3

4·(x− 1

2

).

2.4.2.8. *. Solution. Let f(t) = t3 − 4t2 + 1. Then

f ′(t) = 3t2 − 8t f ′(2) = 3× 4− 8× 2 = −4

f ′′(t) = 6t− 8 f ′′(2) = 6× 2− 8 = 4

Hence at t = 2, 2.4.2.8.a the particle has speed of magnitude 4, and 2.4.2.8.b ismoving towards the left. At t = 2, f ′′(2) > 0, so f ′ is increasing, i.e. becomingless negative. Since f ′ is getting closer to zero, 2.4.2.8.c the magnitude of thespeed is decreasing.

2.4.2.9. *. Solution. We can use the quotient rule here.

d

dx

2x− 1

2x+ 1

=

(2x+ 1)(2)− (2x− 1)(2)

(2x+ 1)2=

4

(2x+ 1)2

729

Page 746: CLP-1 Differential Calculus

SOLUTIONS TO EXERCISES

=1

(x+ 1/2)2

2.4.2.10. Solution. First, we find the y′ for general x. Using the corollary toTheorem 2.4.3 and the quotient rule:

y′ = 2

(3x+ 1

3x− 2

)· d

dx

3x+ 1

3x− 2

= 2

(3x+ 1

3x− 2

)((3x− 2)(3)− (3x+ 1)(3)

(3x− 2)2

)= 2

(3x+ 1

3x− 2

)( −9

(3x− 2)2

)=−18(3x+ 1)

(3x− 2)3

So, plugging in x = 1:

y′(1) =−18(3 + 1)

(3− 2)3= −72

2.4.2.11. Solution. We need f ′(1), so first we must find f ′(x). Since f(x) is thereciprocal of

√x+ 1, we can use the Corollary 2.4.6:

f ′(x) =− d

dx√x+ 1

(√x+ 1)2

=− 1

2√x

(√x+ 1)2

=−1

2√x(√x+ 1)2

,

so f ′(1) =−1

2√

1(√

1 + 1)2= −1

8.

Now, using the point(1, 1

2

)and the slope −1

8, our tangent line has equation y− 1

2=

−18(x− 1).

Exercises — Stage 32.4.2.12. Solution. Population growth is rate of change of population. Popu-lation in year 2000 + t is given by P (t) = P0 + b(t) − d(t), where P0 is the ini-tial population of the town. Then P ′(t) is the expression we’re looking for, andP ′(t) = b′(t)− d′(t).It is interesting to note that the initial population does not obviously show up inthis calculation. It would probably affect b(t) and d(t), but if we know these wedo not need to know P0 to answer our question.

2.4.2.13. *. Solution. The slope of y = 3x2 at x = a is 6a. The tangent line toy = 3x2 at x = a, y = 3a2 is y − 3a2 = 6a(x− a). This tangent line passes through(2, 9) if

9− 3a2 = 6a(2− a)

3a2 − 12a+ 9 = 0

730

Page 747: CLP-1 Differential Calculus

SOLUTIONS TO EXERCISES

a2 − 4a+ 3 = 0

(a− 3)(a− 1) = 0

=⇒ a = 1, 3

The points are (1, 3), (3, 27).

2.4.2.14. *. Solution. This limit represents the derivative computed at x =

100180 of the function f(x) =√x. Since the derivative of f(x) is

1

2√x

, then

its value at x = 100180 is exactly1

2√

100180.

2.4.2.15. Solution. Let w(t) and l(t) be the width and length of the rectangle.Given in the problem is that w′(t) = 2 and l′(t) = 5. Since both functions haveconstant slopes, both must be lines. Their slopes are given, and their interceptsare w(0) = l(0) = 1. So, w(t) = 2t+ 1 and l(t) = 5t+ 1.The area of the rectangle is A(t) = w(t) · l(t), so using the product rule, the rate atwhich the area is increasing is A′(t) = w′(t)l(t) +w(t)l′(t) = 2(5t+ 1) + 5(2t+ 1) =20t+ 7 square metres per second.

2.4.2.16. Solution. Using the product rule, f ′(x) = (2x)g(x) +x2g′(x), so f ′(0) =0 · g(x) + 0 · g′(x) = 0. (Since g is differentiable, g′ exists.)

2.4.2.17. Solution. First expression, f(x) =g(x)

h(x):

f ′(x) =h(x)g′(x)− g(x)h′(x)

h2(x)

Second expresson, f(x) =g(x)

k(x)· k(x)

h(x):

f ′(x) =

(k(x)g′(x)− g(x)k′(x)

k2(x)

)(k(x)

h(x)

)+

(g(x)

k(x)

)(h(x)k′(x)− k(x)h′(x)

h2(x)

)=k(x)g′(x)− g(x)k′(x)

k(x)h(x)+g(x)h(x)k′(x)− g(x)k(x)h′(x)

k(x)h2(x)

=h(x)k(x)g′(x)− h(x)g(x)k′(x)

k(x)h2(x)+g(x)h(x)k′(x)− g(x)k(x)h′(x)

k(x)h2(x)

=h(x)k(x)g′(x)− h(x)g(x)k′(x) + g(x)h(x)k′(x)− g(x)k(x)h′(x)

k(x)h2(x)

=h(x)k(x)g′(x)− g(x)k(x)h′(x)

k(x)h2(x)

=h(x)g′(x)− g(x)h′(x)

h2(x)

and this is exactly what we got from differentiating the first expression.

731

Page 748: CLP-1 Differential Calculus

SOLUTIONS TO EXERCISES

2.6 · Using the Arithmetic of Derivatives – Examples2.6.2 · Exercises

Exercises — Stage 12.6.2.1. Solution. In the quotient rule, there is a minus, not a plus. Also, 2(x +1) + 2x is not the same as 2(x+ 1).The correct version is:

f(x) =2x

x+ 1

f ′(x) =2(x+ 1)−2x

(x+ 1)2

=2

(x+ 1)2

2.6.2.2. Solution. False: Lemma 2.6.9 tells us that, for a constant n,d

dxxn =

nxn−1. Note that the base x is the variable and the exponent n is a constant. Inthe equation given in the question, the base 2 is a constant, and the exponent x isthe variable: this is the opposite of the situation where Lemma 2.6.9 applies.We do not yet know how to differentiate 2x. We’ll learn about it in Section 2.7

Exercises — Stage 22.6.2.3. Solution. f(x) = 2

3x6 + 5x4 + 12x2 + 9 is a polynomial:

f ′(x) = 4x5 + 20x3 + 24x

= 4x(x4 + 5x2 + 6)

= 4x((x2)2 + 5(x2) + 6)

= 4x(x2 + 2)(x2 + 3)

2.6.2.4. Solution. We can rewrite slightly to make every term into a power of t:

s(t) = 3t4 + 5t3 − t−1

s′(t) = 4 · 3t3 + 3 · 5t2 − (−1) · t−2

= 12t3 + 15t2 +1

t2

2.6.2.5. Solution. We could use the product rule here, but it’s easier to simplifyfirst. Don’t be confused by the role reversal of x and y: x is the name of thefunction, and y is the variable.

x(y) =

(2y +

1

y

)· y3

= 2y4 + y2

x′(y) = 8y3 + 2y

732

Page 749: CLP-1 Differential Calculus

SOLUTIONS TO EXERCISES

2.6.2.6. Solution. We’ve already seen that ddx√x = 1

2√x, but if you forget this

formula it is easy to figure out:√x = x1/2, so d

dx√x = 1

2x−1/2 = 1

2√x.

Using the quotient rule:

T (x) =

√x+ 1

x2 + 3

T ′(x) =(x2 + 3)

(1

2√x

)− (√x+ 1)(2x)

(x2 + 3)2

2.6.2.7. *. Solution. We use quotient rule:

(x2 + 3) · 7− 2x · (7x+ 2)

(x2 + 3)2=

21− 4x− 7x2

(x2 + 3)2

2.6.2.8. Solution. Instead of multiplying to get our usual form of this polyno-mial, we can use the quotient rule. If f1(x) = 3x3 + 4x2 +x+ 1 and f2(x) = 2x+ 5,then f ′1(x) = 9x2 + 8x+ 1 and f ′2(x) = 2. Then

f ′(0) = f ′1(0)f2(0) + f1(0)f ′2(0)

= (1)(5) + (1)(2) = 7

2.6.2.9. Solution. Using the quotient rule,

f ′(x) =(x2 + 5x)(9x2)− (3x3 + 1)(2x+ 5)

(x2 + 5x)2=

3x4 + 30x3 − 2x− 5

(x2 + 5x)2

2.6.2.10. *. Solution. We use quotient rule:

(2− x)(6x)− (3x2 + 5)(−1)

(2− x)2=−3x2 + 12x+ 5

(x− 2)2

2.6.2.11. *. Solution. We use quotient rule:

(3x2 + 5)(−2x)− (2− x2)(6x)

(3x2 + 5)2=

−22x

(3x2 + 5)2

2.6.2.12. *. Solution. We use quotient rule:

6x2 · (x+ 2)− (2x3 + 1) · 1(x+ 2)2

=4x3 + 12x2 − 1

(x+ 2)2

2.6.2.13. *. Solution. The derivative of the function is

(1− x2) · 12√x−√x · (−2x)

(1− x2)2=

(1− x2)− 2x · (−2x)

2√x(1− x2)2

The derivative is undefined if either x < 0 or x = 0,±1 (since the square-root isundefined for x < 0 and the denominator is zero when x = 0, 1,−1. Putting thistogether — the derivative exists for x > 0, x 6= 1.

733

Page 750: CLP-1 Differential Calculus

SOLUTIONS TO EXERCISES

2.6.2.14. Solution. Using the product rule seems faster than expanding.

f ′(x) =d

dx

3 5√x+ 15 3

√x+ 8

(3x2 + 8x− 5

)+(3 5√x+ 15 3

√x+ 8

)d

dx

3x2 + 8x− 5

=

d

dx

3x

15 + 15x

13 + 8

(3x2 + 8x− 5

)+(3 5√x+ 15 3

√x+ 8

)d

dx

3x2 + 8x− 5

=

(3

5x−45 +5x

−23

)(3x2+8x−5

)+(3 5√x+15 3

√x+ 8

)(6x+8)

2.6.2.15. Solution. To avoid the quotient rule, we can divide through the de-nominator:

f(x) =(x2 + 5x+ 1)(

√x+ 3√x)

x= (x2 + 5x+ 1)

(√x+ 3√x)

x= (x2 + 5x+ 1)(x−1/2 + x−2/3)

Now, product rule:

f ′(x) = (2x+5)(x−1/2+x−2/3) + (x2+5x+1)

(−1

2x−3/2− 2

3x−5/3

)(If you simplified differently, or used the quotient rule, you probably came upwith a different-looking answer. There is only one derivative, though, so all cor-rect answers will look the same after sufficient algebraic manipulation.)

2.6.2.16. Solution. The question asks us to find where f ′(x) = 0 and f(x) exists.We can use the formula for the derivative of a reciprocal, Corollary 2.4.6:

f ′(x) =− d

dx

15x5 + x4 − 5

3x3(

15x5 + x4 − 5

3x3)2

=− (x4 + 4x3 − 5x2)(

15x5 + x4 − 5

3x3)2

=−x2 (x2 + 4x− 5)(

15x5 + x4 − 5

3x3)2

=−x2(x+ 5)(x− 1)(

15x5 + x4 − 5

3x3)2

So our candidates for x-values where f ′(x) = 0 are x = 0, x = −5, and x = 1.However, we need to check that f exists at these places: f(0) is undefined (andf ′(0) doesn’t exist). So f ′(x) = 0 only when x = −5 and x = 1.

734

Page 751: CLP-1 Differential Calculus

SOLUTIONS TO EXERCISES

Exercises — Stage 32.6.2.17. *. Solution. Denote by m the slope of the common tangent, by (x1, y1)the point of tangency with y = x2, and by (x2, y2) the point of tangency withy = x2 − 2x+ 2. Then we must have

y1 = x21 y2 = x2

2 − 2x2 + 2 m = 2x1 = 2x2 − 2 =y2 − y1

x2 − x1

From the “m” equations we get x1 = m2

, x2 = m2

+ 1 and

m =y2 − y1

x2 − x1

= y2 − y1

= x22 − 2x2 + 2− x2

1

= (x2 − x1)(x2 + x1)− 2(x2 − 1)

=(m

2+ 1− m

2

)(m2

+ 1 +m

2

)− 2

(m2

+ 1− 1)

= (1)(m+ 1)− 2m

2= 1

So m = 1 and

x1 =1

2, y1 =

1

4, x2 =

3

2, y2 =

9

4− 3 + 2 =

5

4

An equation of the common tangent is y = x− 14.

2.6.2.18. Solution. The line y = mx+ b is tangent to y = x2 at x = α if

2α = m and α2 = mα + b ⇐⇒ m = 2α and b = −α2

The same line y = mx+ b is tangent to y = −x2 + 2x− 5 at x = β if

−2β + 2 = m and − β2 + 2β − 5 = mβ + b

⇐⇒ m = 2− 2β and b = −β2 + 2β − 5− (2− 2β)β = β2 − 5

For the line to be simultaneously tangent to the two parabolas we need

m = 2α = 2− 2β and b = −α2 = β2 − 5

Substituting α = 1−β into−α2 = β2−5 gives−(1−β)2 = β2−5 or 2β2−2β−4 = 0or β = −1, 2. The corresponding values of the other parameters are α = 2,−1,m = 4,−2 and b = −4,−1. The two lines are y = 4x− 4 and y = −2x− 1.

735

Page 752: CLP-1 Differential Calculus

SOLUTIONS TO EXERCISES

2.6.2.19. *. Solution. This limit represents the derivative computed at x = 2015of the function f(x) = cos(x). To see this, simply use the definition of the deriva-tive at a = 2015 with f(x) = cos x:

d

dxf(x)

∣∣∣∣a

= limx→a

f(x)− f(a)

x− ad

dxcosx

∣∣∣∣2015

= limx→2015

cos(x)− cos(2015)

x− 2015

Since the derivative of f(x) is− sin(x), its value at x = 2015 is exactly− sin(2015).

2.6.2.20. *. Solution. This limit represents the derivative computed at x = π/3of the function f(x) = cosx. To see this, simply use the definition of the deriva-tive at a = π/3 with f(x) = cos x:

d

dxf(x)

∣∣∣∣a

= limx→a

f(x)− f(a)

x− ad

dxcosx

∣∣∣∣π/3

= limx→π/3

cos(x)− cos(π/3)

x− π/3

= limx→π/3

cos(x)− 1/2

x− π/3

Since the derivative of f(x) is − sinx, then its value at x = π/3 is exactly− sin(π/3) = −

√3/2.

2.6.2.21. *. Solution. This limit represents the derivative computed at x = π ofthe function f(x) = sin(x). To see this, simply use the definition of the derivativeat a = π with f(x) = sin x:

d

dxf(x)

∣∣∣∣a

= limx→a

f(x)− f(a)

x− a

736

Page 753: CLP-1 Differential Calculus

SOLUTIONS TO EXERCISES

d

dxsinx

∣∣∣∣π

= limx→π

sin(x)− sin(π)

x− π

= limx→π

sin(x)

x− πSince the derivative of f(x) is cos(x), then its value at x = π is exactly cos(π) = −1.

2.6.2.22. *. Solution. This limit represents the derivative computed at x = 2 ofthe function f(x) = x2015. To see this, simply use the definition of the derivativeat a = 2 with f(x) = x2015:

d

dxf(x)

∣∣∣∣a

= limx→a

f(x)− f(a)

x− ad

dxx2015

∣∣∣∣2

= limx→2

x2015 − 22015

x− 2

Since the derivative of f(x) is 2015 · x2014, then its value at x = 2 is exactly 2015 ·22014.

2.7 · Derivatives of Exponential Functions2.7.3 · Exercises

Exercises — Stage 12.7.3.1. Solution. Since 1x = 1 for any x, we see that (b) is just the constantfunction y = 1, so D matches to (b).Since 2−x = 1

2x=(

12

)x, functions (a) and (d) are the same. This is the onlyfunction out of the lot that grows as x → −∞ and shrinks as x → ∞, so Amatches to (a) and (d).This leaves B and C to match to (c) and (e). Since 3 > 2, when x > 0, 3x > 2x. So,(e) matches to the function that grows more quickly to the right of the x-axis: Bmatches to (e), and C matches to (c).

2.7.3.2. Solution. First, let’s consider the behaviour of exponential functions ax

based on whether a is greater or less than 1. As we know, limx→∞

ax =

∞ a > 1

0 a < 1

and limx→−∞

ax =

0 a > 1

∞ a < 1. Our function has lim

x→∞f(x) = ∞ and lim

x→−∞f(x) =

0, so we conclude a > 1: thus (d) and also (b) hold. (We could have also seen that(b) holds because ax is defined for all real numbers.)It remains to decide whether a is greater or less than e. (If a were equal to e, thenf ′(x) would be the same as f(x).) We saw in the text that d

dxax = C(a)ax for

the function C(a) = limh→0

ah − 1

h. We know that C(e) = 1. (Actually, we chose e to

be the number that has this property.) From our graph, we see that f ′(x) < f(x),

so C(a) < 1 = C(e). In other words, limh→0

ah − 1

h< lim

h→0

eh − 1

h; so, a < e. Thus (e)

holds.

737

Page 754: CLP-1 Differential Calculus

SOLUTIONS TO EXERCISES

2.7.3.3. Solution. The power rule tells us that ddxxn = nxn−1. In this equation,

the variable is the base, and the exponent is a constant. In the function ex, it’sreversed: the variable is the exponent, and the base it a constant. So, the powerrule does not apply.

2.7.3.4. Solution. P (t) is an increasing function over its domain, so the popula-tion is increasing.There are a few ways to see that P (t) is increasing.What we really care about is whether e0.2t is increasing or decreasing, since anincreasing function multiplied by 100 is still an increasing function, and a de-creasing function multiplied by 100 is still a decreasing function. Since f(t) = et

is an increasing function, we can use what we know about graphing functions tosee that f(0.2t) = e0.2t is also increasing.

Exercises — Stage 22.7.3.5. Solution. Using the quotient rule,

f ′(x) =2xex − 2ex

4x2=ex(2x− 2)

4x2=

(x− 1)ex

2x2

2.7.3.6. Solution.

f ′(x) =d

dxe2x =

d

dx(ex)2 = 2

d

dxexex = 2exex = 2(ex)2 = 2e2x

2.7.3.7. Solution.

ea+x = eaex

Since ea is just a constant,

d

dxeaex = ea

d

dxex = eaex = ea+x

So, f ′(x) = f(x) = ea+x.

2.7.3.8. Solution. If the derivative is positive, the function is increasing, solet’s start by finding the derivative. We use the product rule (although Ques-tion 2.7.3.12 gives a shortcut).

f ′(x) = 1 · ex + xex = (1 + x)ex

Since ex is always positive, f ′(x) > 0 when 1 + x > 0. So, f(x) is increasing whenx > −1.

2.7.3.9. Solution.

e−x =1

ex

738

Page 755: CLP-1 Differential Calculus

SOLUTIONS TO EXERCISES

Using the rule for differentiating the reciprocal:

d

dxe−x =

−ex(ex)2

=−1

ex= −e−x

2.7.3.10. Solution. Using the product rule,

f ′(x) = (ex)(ex − 1) + (ex + 1)(ex) = ex(ex − 1 + ex + 1) = 2(ex)2

= 2e2x

Alternate solution: using Question 2.7.3.6:

f(x) = e2x − 1 =⇒ f ′(x) = 2e2x.

2.7.3.11. Solution. The question asks when s′(t) is negative. So, we start bydifferentiating. Using the product rule:

s′(t) = et(t2 + 2t)

= et · t(t+ 2)

et is always positive, so s′(t) is negative when t and 2 + t have opposite signs.This occurs when −2 < t < 0.

Exercises — Stage 32.7.3.12. Solution. Using the product rule, g′(x) = f ′(x)ex + f(x)ex = [f(x) +f ′(x)]ex

2.7.3.13. Solution. We simplify the functions to get a better idea of what’s goingon.(a): y = e3 log x + 1 =

(elog x

)3+ 1 = x3 + 1. This is not a line.

(b): 2y + 5 = e3+log x = e3elog x = e3x. Since e3 is a constant, 2y + 5 = e3x is a line.(c): There isn’t a fancy simplification here–this isn’t a line. If that isn’t a satis-factory answer, we can check: a line is a function with a constant slope. For ourfunction, y′ = d

dxe2x + 4 = d

dxe2x = d

dx(ex)2 = 2exex = 2e2x. Since the

derivative isn’t constant, the function isn’t a line.(d): y = elog x3e + log 2 = 3ex+ log 2. Since e3 and log 2 are constants, this is a line.

2.7.3.14. *. Solution. When we say a function is differentiable without specify-ing a range, we mean that it is differentiable over its domain. The function f(x)is differentiable when x 6= 1 for any values of a and b; it is up to us to figure outwhich constants make it differentiable when x = 1.In order to be differentiable, a function must be continuous. The definition ofcontinuity tells us that, for f to be continuous at x = 1, we need lim

x→1f(x) = f(1).

From the definition of f , we see f(1) = a+ b = limx→1−

f(x), so we need limx→1+

f(x) =

a+ b. Since limx→1+

f(x) = e1 = e, we specifically need

e = a+ b.

739

Page 756: CLP-1 Differential Calculus

SOLUTIONS TO EXERCISES

Now, let’s consider differentiability of f at x = 1. We need the following limit toexist:

limh→0

f(1 + h)− f(1)

h

In particular, we need the one-sided limits to exist and be equal:

limh→0−

f(1 + h)− f(1)

h= lim

h→0+

f(1 + h)− f(1)

h

If h < 0, then 1 + h < 1, so f(1 + h) = a(1 + h)2 + b. If h > 0, then 1 + h > 1, sof(1 + h) = e1+h. With this in mind, we begin to evaluate the one-sided limits:

limh→0−

f(1 + h)− f(1)

h= lim

h→0−

[a(1 + h)2 + b]− [a+ b]

h

= limh→0−

ah2 + 2ah

h= 2a

limh→0+

f(1 + h)− f(1)

h= lim

h→0+

e1+h − (a+ b)

h

Since we take a+ b to be equal to e (to ensure continuity):

= limh→0+

e1+h − e1

h

=d

dxex

∣∣∣∣x=1

= e1 = e

So, we also need2a = e

Therefore, the values of a and b that make f differentiable are a = b =e

2.

2.8 · Derivatives of Trigonometric Functions2.8.8 · Exercises

Exercises — Stage 12.8.8.1. Solution.

x

y

π−π

y = sinx

y = cosx

The graph f(x) = sinx has horizontal tangent lines precisely at those pointswhere cosx = 0. This must be true, since d

dxsinx = cosx: where the derivative

740

Page 757: CLP-1 Differential Calculus

SOLUTIONS TO EXERCISES

of sine is zero, cosine itself is zero.

2.8.8.2. Solution.

x

y

π−π

y = sinx

y = cosx

The graph f(x) = sinx has maximum slope at those points where cosx has amaximum. This makes sense, because f ′(x) = cosx: the maximum values of theslope of sine correspond to the maximum values of cosine.

Exercises — Stage 22.8.8.3. Solution. You should memorize the derivatives of sine, cosine, andtangent. f ′(x) = cos x− sinx+ sec2 x

2.8.8.4. Solution. f ′(x) = cos x− sinx, so f ′(x) = 0 precisely when sinx = cosx.This happens at π/4, but it also happens at 5π/4. By looking at the unit circle, itis clear that sinx = cosx whenever x = π

4+ πn for some integer n.

x

y

sin(π/4) = cos(π/4)

sin(5π/4) = cos(5π/4)

2.8.8.5. Solution.

• Solution 1: f(x) = sin2 x+ cos2 x = 1, so f ′(x) = ddx1 = 0.

• Solution 2: Using the formula for the derivative of a squared function,

f ′(x) = 2 sinx cosx+ 2 cosx(− sinx)

= 2 sin x cosx− 2 sinx cosx = 0.

2.8.8.6. Solution. It is true that 2 sinx cosx = sin(2x), but we don’t know thederivative of sin(2x). So, we use the product rule:

f ′(x) = 2 cosx cosx+ 2 sinx(− sinx) = 2(cos2 x− sin2 x).

741

Page 758: CLP-1 Differential Calculus

SOLUTIONS TO EXERCISES

2.8.8.7. Solution.

• Solution 1: using the product rule,

f ′(x) = ex cotx+ ex(− csc2 x) = ex(cotx− csc2 x).

• Solution 2: using the formula from Question 2.7.3.12, Section 2.7,

f ′(x) = ex(cotx− csc2 x).

2.8.8.8. Solution. We use the quotient rule.

f ′(x)

=(cosx+tan x)(2 cosx+3 sec2 x)−(2 sinx+3 tanx)(− sinx+sec2 x)

(cosx+ tanx)2

=2 cos2 x+ 3 cosx sec2 x+ 2 cosx tanx+ 3 tanx sec2 x

(cosx+ tanx)2

+2 sin2 x− 2 sinx sec2 x+ 3 sinx tanx− 3 tanx sec2 x

(cosx+ tanx)2

=2 + 3 secx+ 2 sinx− 2 tanx secx+ 3 sinx tanx

(cosx+ tanx)2

2.8.8.9. Solution. We use the quotient rule.

f ′(x) =ex(5 secx tanx)− (5 secx+ 1)ex

(ex)2

=5 secx tanx− 5 secx− 1

ex

2.8.8.10. Solution. We use the product rule:

f ′(x) = (ex + cotx)(30x5 + cscx cotx) + (ex − csc2 x)(5x6 − cscx)

2.8.8.11. Solution. We don’t know how to differentiate this function as it is writ-ten, but an identity helps us. Since sin

(π2− θ)

= cos θ, we see f ′(θ) = ddθcos θ =

− sin(θ).

2.8.8.12. Solution. We know the derivative of sinx, but not of sin(−x). So were-write f(x) using identities:

f(x) = sin(−x) + cos(−x)

= − sinx+ cosx

f ′(x) = − cosx− sinx

2.8.8.13. Solution. We apply the quotient rule.

s′(θ) =(cos θ − sin θ)(− sin θ + cos θ)− (cos θ + sin θ)(− sin θ − cos θ)

(cos θ − sin θ)2

742

Page 759: CLP-1 Differential Calculus

SOLUTIONS TO EXERCISES

=(cos θ − sin θ)2 + (cos θ + sin θ)2

(cos θ − sin θ)2

= 1 +

(cos θ + sin θ

cos θ − sin θ

)2

2.8.8.14. *. Solution. In order for f to be differentiable at x = 0, it must also becontinuous at x = 0. This forces

limx→0−

f(x) = limx→0+

f(x) = f(0)

or limx→0−

cos(x) = limx→0+

(ax+ b) = 1

or b = 1.In order for f to be differentiable at x = 0, we need the limit

limh→0

f(0 + h)− f(0)

h

to exist. This is the case if and only if the two one-sided limits

limh→0−

f(0 + h)− f(0)

h= lim

h→0−

cos(h)− cos(0)

h

and

limh→0+

f(0 + h)− f(0)

h= lim

h→0+

(ah+ b)− cos(0)

h= a since b = 1

exist and are equal. Because cos(x) is differentiable at x = 0 we have

limh→0−

cos(h)− cos(0)

h=

d

dxcos(x)

∣∣∣∣x=0

= − sin(x)∣∣∣x=0

= 0

So, we need a = 0 and b = 1.

2.8.8.15. *. Solution. We compute the derivative of cos(x) + 2x as being− sin(x) + 2, which evaluated at x = π

2yields −1 + 2 = 1. Since we also com-

pute cos(π/2) + 2(π/2) = 0 + π, then the equation of the tangent line is

y − π = 1 · (x− π/2).

Exercises — Stage 32.8.8.16. *. Solution. This limit represents the derivative computed at x = π/3of the function f(x) = cosx. Since the derivative of f(x) is − sinx, then its valueat x = π/3 is exactly −

√3/2.

2.8.8.17. Solution.

tan θ =sin θ

cos θ

743

Page 760: CLP-1 Differential Calculus

SOLUTIONS TO EXERCISES

So, using the quotient rule,

d

dθtan θ =

cos θ cos θ − sin θ(− sin θ)

cos2 θ=

cos2 θ + sin2 θ

cos2 θ

=

(1

cos θ

)2

= sec2 θ

2.8.8.18. *. Solution. In order for the function f(x) to be continuous at x = 0,

the left half formula ax+b and the right half formula6 cosx

2 + sin x+ cosxmust match

up at x = 0. This forces

a× 0 + b =6 cos 0

2 + sin 0 + cos 0=

6

3=⇒ b = 2

In order for the derivative f ′(x) to exist at x = 0, the limit limh→0

f(h)− f(0)

hmust

exist. In particular, the limits limh→0−

f(h)− f(0)

hand lim

h→0+

f(h)− f(0)

hmust exist

and be equal to each other.When h→ 0−, this means h < 0, so f(h) = ah+ b = ah+ 2. So:

limh→0−

f(h)− f(0)

h= lim

h→0−

(ah+ 2)− 2

h=

d

dxax+ 2

∣∣∣∣x=0

= a.

Similarly, when h→ 0+, then h > 0, so f(h) =6 cosh

1 + sinh+ coshand

limh→0+

f(h)− f(0)

h=

d

dx

6 cosx

2 + sin x+ cosx

∣∣∣∣x=0

=−6 sinx(2 + sin x+ cosx)− 6 cosx(cosx− sinx)

(2 + sin x+ cosx)2

∣∣∣∣x=0

.

Since the limits from the left and right must be equal, this forces

a =−6 sin 0(2 + sin 0 + cos 0)− 6 cos 0(cos 0− sin 0)

(2 + sin 0 + cos 0)2=

−6

(2 + 1)2

=⇒ a = −2

3

2.8.8.19. *. Solution. In order for f ′(x) to exist, f(x) has to exist. We alreadyknow that tanx does not exist whenever x = π

2+ nπ for any integer n. If we

look a little deeper, since tanx = sinxcosx

, the points where tangent does not existcorrespond exactly to the points where cosine is zero.From its graph, tangent looks like a smooth curve over its domain, so we mightguess that everywhere tangent is defined, its derivative is defined. We can checkthis: f ′(x) = sec2 x =

(1

cosx

)2. Indeed, wherever cosx is nonzero, f ′ exists.So, f ′(x) exists for all values of x except when x = π

2+ nπ for some integer n.

744

Page 761: CLP-1 Differential Calculus

SOLUTIONS TO EXERCISES

2.8.8.20. *. Solution. The function is differentiable whenever x2 + x − 6 6= 0since the derivative equals

10 cos(x) · (x2 + x− 6)− 10 sin(x) · (2x+ 1)

(x2 + x− 6)2,

which is well-defined unless x2+x−6 = 0. We solve x2+x−6 = (x−2)(x+3) = 0,and get x = 2 and x = −3. So, the function is differentiable for all real values xexcept for x = 2 and for x = −3.

2.8.8.21. *. Solution. The function is differentiable whenever sin(x) 6= 0 sincethe derivative equals

sin(x) · (2x+ 6)− cos(x) · (x2 + 6x+ 5)

(sinx)2,

which is well-defined unless sinx = 0. This happens when x is an integer mul-tiple of π. So, the function is differentiable for all real values x except x = nπ,where n is any integer.

2.8.8.22. *. Solution. We compute the derivative of tan(x) as being sec2(x),which evaluated at x = π

4yields 2. Since we also compute tan(π/4) = 1, then the

equation of the tangent line is

y − 1 = 2 · (x− π/4).

2.8.8.23. *. Solution. We compute the derivative y′ = cos(x)− sin(x)+ex, whichevaluated at x = 0 yields 1−0+1 = 2. Since we also compute y(0) = 0+1+1 = 2,the equation of the tangent line is

y − 2 = 2(x− 0)

ie y = 2x+ 2.

2.8.8.24. Solution. We are asked to solve f ′(x) = 0. That is, ex[sinx+ cosx] = 0.Since ex is always positive, that means we need to find all points where sinx +cosx = 0. That is, we need to find all values of x where sinx = − cosx. Lookingat the unit circle, we see this happens whenever x = 3π

4+ nπ for any integer n.

745

Page 762: CLP-1 Differential Calculus

SOLUTIONS TO EXERCISES

x

y

sin(−π/4) = − cos(−π/4)

sin(3π/4) = − cos(3π/4)

2.8.8.25. Solution. First, we note that our function is continuous, because

limx→0

f(x) = limx→0

sinx

x= 1 = f(0)

This is a handy thing to check: if the function were discontinuous at x = 0, thenwe would automatically know that it was not differentiable there.Now, on to the derivative. We can use the limit definition:

f ′(0) = limh→0

f(0 + h)− f(0)

hif it exists

= limh→0

f(h)− 1

h

= limh→0

sinhh− 1

h

As h approaches 0, both the numerator and the denominator approach 0. So, toevaluate the limit, we need to do more work. The key insight we can use is aresult that was shown in the text while evaluating the derivative of sine. When his close to 0, cosh ≤ sinh

h≤ 1. We use this to bound our limit, and then apply the

squeeze theorem.

cosh ≤sinh

h≤1

So, cosh− 1 ≤sinh

h− 1 ≤1− 1 = 0

and 1h

(sinhh− 1)

is between 0 and cosh−1h

. We can evaluate the limit of cosh−1h

bynoticing its similarity to the definition of the derivative.

limh→0

cosh− 1

h= lim

h→0

cos(0 + h)− cos(0)

h

=d

dxcosx|x=0 = 0

So, by the Squeeze Theorem, f ′(0) = limh→0

sinhh− 1

h= 0.

746

Page 763: CLP-1 Differential Calculus

SOLUTIONS TO EXERCISES

2.8.8.26. *. Solution. As usual, when dealing with the absolute value function,we can make things a little clearer by splitting it up into two pieces.

|x| =

x x ≥ 0

−x x < 0

So,

sin |x| =

sinx x ≥ 0

sin(−x) x < 0=

sinx x ≥ 0

− sinx x < 0

where we used the identity sin(−x) = − sinx. From here, it’s easy to see h′(x)when x is anything other than zero.

d

dxsin |x| =

cosx x > 0

?? x = 0

− cosx x < 0

To decide whether h(x) is differentiable at x = 0, we use the definition of thederivative. One word of explanation: usually in the definition of the derivative,h is the tiny “change in x” that is going to zero. Since h is the name of ourfunction, we need another letter to stand for the tiny change in x, the size ofwhich is tending to zero. We chose t.

limt→0

h(t+ 0)− h(0)

t= lim

t→0

sin |t|t

We consider the behaviour of this function to the left and right of t = 0:

sin |t|t

=

sin tt

t ≥ 0sin(−t)

tt < 0

=

sin tt

t ≥ 0

− sin tt

t < 0

Since we’re evaluating the limit as t goes to zero, we need the fact that limt→0

sin t

t=

1. We saw this in Section 2.7, but also we know enough now to evaluate it anotherway. Using the definition of the derivative:

limt→0

sin t

t= lim

t→0

sin(t+ 0)− sin(0)

t=

d

dxsinx

∣∣∣∣t=0

= cos 0 = 1

At any rate, since we know limt→0

sin t

t= 1, then:

limt→0+

h(t+ 0)− h(0)

t= lim

t→0+

sin t

t= 1

limt→0−

h(t+ 0)− h(0)

t= lim

t→0−

− sin t

t= −1

So, since the one-sided limits disagree,

limt→0

h(t+ 0)− h(0)

t= DNE

so h(x) is not differentiable at x = 0. Therefore,

h′(x) =

cosx x > 0

− cosx x < 0

747

Page 764: CLP-1 Differential Calculus

SOLUTIONS TO EXERCISES

2.8.8.27. *. Solution. Statement 2.8.8.27.i is false, since f(0) = 0. Statement2.8.8.27.iv cannot hold, since a function that is differentiable is also continuous.

Since limx→0+

sinx

x= 1 (we saw this in Section 2.8 ),

limx→0+

f(x) = limx→0+

sinx√x

= limx→0+

√x

sinx

x

= 0 · 1 = 0

So f is continuous at x = 0, and so Statement 2.8.8.27.ii does not hold. Now let’sconsider f ′(x)

limx→0+

f(x)− f(0)

x= lim

x→0+

sinx√x− 0

x

= limx→0+

1√x

sinx

x= +∞

Therefore, using the definition of the derivative,

f ′(0) = limx→0

f(x)− f(0)

xif it exists, but

limx→0

f(x)− f(0)

x= DNE

since one of the one-sided limits does not exist. So f is continuous but not differ-entiable at x = 0. The correct statement is 2.8.8.27.iii.

2.8.8.28. *. Solution. Recall that limx→0

sinx

x= 1. In order to take advantage of this

knowledge, we divide the numerator and denominator by x5 (because 5 is thepower of sine in the denominator, and a denominator that goes to zero generallymakes a limit harder).

limx→0

sinx27 + 2x5ex99

sin5 x= lim

x→0

sinx27

x5+ 2ex

99(sinx

x

)5

Now the denominator goes to 1, which is nice, but we need to take care of the

fractionsinx27

x5in the numerator. This fraction isn’t very familiar, but we know

that, as x goes to zero, x27 also goes to zero, so thatsinx27

x27goes to 1. Conse-

748

Page 765: CLP-1 Differential Calculus

SOLUTIONS TO EXERCISES

quently,

limx→0

sinx27 + 2x5ex99

sin5 x= lim

x→0

x22 sinx27

x27+ 2ex

99(sinx

x

)5 =0× 1 + 2× e0

15= 2

2.9 · One More Tool – the Chain Rule2.9.4 · Exercises

Exercises — Stage 12.9.4.1. Solution. 2.9.4.1.a More urchins means less kelp, and fewer urchinsmeans more kelp. This means kelp and urchins are negatively correlated, sodKdU

< 0.If you aren’t sure why that is, we give a more detailed explanation here, usingthe definition of the derivative. When h is a positive number, U + h is greaterthan U , so K(U + h) is less than U , hence K(U + h)−K(U) < 0. Therefore:

limh→0+

K(U + h)−K(U)

h=

negativepositive

< 0.

Similarly, when h is negative, U + h is less than U , so K(U + h)−K(U) > 0, and

limh→0−

K(U + h)−K(U)

h=

positivenegative

< 0.

Therefore:dK

dU= lim

h→0

K(U + h)−K(U)

h< 0.

2.9.4.1.b More otters means fewer urchins, and fewer otters means more urchins.So, otters and urchins are negatively correlated: dU

dO< 0.

2.9.4.1.c Using the chain rule, dKdO

= dKdU· dU

dO. Parts 2.9.4.1.a and 2.9.4.1.b tell us

both these derivatives are negative, so their product is positive: dKdO

> 0.We can also see that dK

dO> 0 by thinking about the relationships as described.

When the otter population increases, the urchin population decreases, so the kelppopulation increases. That means when the otter population increases, the kelppopulation also increases, so kelp and otters are positively correlated. The chainrule is a formal version of this kind of reasoning.

2.9.4.2. Solution.dA

dE=

dA

dB· dB

dC· dC

dD· dD

dE< 0

since we multiply three positive quantities and one negative.

Exercises — Stage 2

749

Page 766: CLP-1 Differential Calculus

SOLUTIONS TO EXERCISES

2.9.4.3. Solution. Applying the chain rule:

d

dxcos(5x+ 3) = − sin(5x+ 3) · d

dx5x+ 3

= − sin(5x+ 3) · 5

2.9.4.4. Solution. Using the chain rule,

f ′(x) =d

dx

(x2 + 2

)5

= 5(x2 + 2

)4 · d

dxx2 + 2

= 5(x2 + 2)4 · 2x= 10x(x2 + 2)4

2.9.4.5. Solution. Using the chain rule,

T ′(k) =d

dk

(4k4 + 2k2 + 1

)17

= 17(4k4 + 2k2 + 1)16 · d

dk4k4 + 2k2 + 1

= 17(4k4 + 2k2 + 1)16 · (16k3 + 4k)

2.9.4.6. Solution. Using the chain rule:

d

dx

√x2 + 1

x2 − 1

=

1

2√

x2+1x2−1

· d

dx

x2 + 1

x2 − 1

=1

2

√x2 − 1

x2 + 1· d

dx

x2 + 1

x2 − 1

And now, the quotient rule:

=1

2

√x2 − 1

x2 + 1·(

(x2 − 1)(2x)− (x2 + 1)2x

(x2 − 1)2

)=

1

2

√x2 − 1

x2 + 1·( −4x

(x2 − 1)2

)=

√x2 − 1

x2 + 1·( −2x

(x2 − 1)2

)=

−2x

(x2 − 1)√x4 − 1

2.9.4.7. Solution. If we let g(x) = ex and h(x) = cos(x2), then f(x) = g(h(x)), sof ′(x) = g′(h(x)) · h′(x).

f ′(x) = ecos(x2) · d

dxcos(x2)

750

Page 767: CLP-1 Differential Calculus

SOLUTIONS TO EXERCISES

In order to evaluate ddxcos(x2), we’ll need the chain rule again.

= ecos(x2) · [− sin(x2)] · d

dxx2

= −ecos(x2) · sin(x2) · 2x

2.9.4.8. *. Solution. We use the chain rule, followed by the quotient rule:

f ′(x) = g′(

x

h(x)

)· d

dx

x

h(x)

= g′

(x

h(x)

)· h(x)− xh′(x)

h(x)2

When x = 2:

f ′(2) = g′(

2

h(2)

)h(2)− 2h′(2)

h(2)2

= 42− 2× 3

22= −4

2.9.4.9. *. Solution. Using the chain rule, followed by the product rule:

d

dx

ex cos(x)

= ex cosx d

dxx cosx

= [cosx− x sinx]ex cos(x)

2.9.4.10. *. Solution. Using the chain rule:

d

dx

ex

2+cos(x)

= ex2+cosx d

dx

x2 + cosx

= [2x− sinx]ex

2+cos(x)

2.9.4.11. *. Solution. Using the chain rule, followed by the quotient rule:

d

dx

√x− 1

x+ 2

=

1

2

√x− 1

x+ 2

d

dx

x− 1

x+ 2

=

√x+ 2

2√x− 1

· (x+ 2)− (x− 1)

(x+ 2)2

=3

2√x− 1

√x+ 2

3

2.9.4.12. *. Solution. First, we manipulate our function to make it easier todifferentiate:

f(x) = x−2 + (x2 − 1)1/2

751

Page 768: CLP-1 Differential Calculus

SOLUTIONS TO EXERCISES

Now, we can use the power rule to differentiate1

x2. This will be easier than

differentiating1

x2using quotient rule, but if you prefer, quotient rule will also

work.

f ′(x) = −2x−3 +1

2(x2 − 1)−1/2 · d

dxx2 − 1

= −2x−3 +1

2(x2 − 1)−1/2(2x)

=−2

x3+

x√x2 − 1

The function f(x) is only defined when x 6= 0 and when x2−1 ≥ 0. That is, whenx is in (−∞,−1] ∪ [1,∞). We have an added restriction on the domain of f ′(x):x2 − 1 must not be zero. So, the domain of f ′(x) is (−∞,−1) ∪ (1,∞).

2.9.4.13. *. Solution. We use the quotient rule, noting thatd

dxsin 5x =

5 cos 5x:

f ′(x) =(1 + x2)(5 cos 5x)− (sin 5x)(2x)

(1 + x2)2

2.9.4.14. Solution. If we let g(x) = secx and h(x) = e2x+7, then f(x) = g(h(x)),so by the chain rule, f ′(x) = g′(h(x)) · h′(x). Since g′(x) = sec x tanx:

f ′(x) = g′(h(x)) · h′(x)

= sec(h(x)) tan(h(x)) · h′(x)

= sec(e2x+7) tan(e2x+7) · d

dx

e2x+7

Here, we need the chain rule again:

= sec(e2x+7) tan(e2x+7) ·[e2x+7 · d

dx2x+ 7

]= sec(e2x+7) tan(e2x+7) ·

[e2x+7 · 2

]= 2e2x+7 sec(e2x+7) tan(e2x+7)

2.9.4.15. Solution. It is possible to start in on this problem with the productrule and then the chain rule, but it’s easier if we simplify first. Since tan2 x+ 1 =sec2 x = 1

cos2 x, we see

f(x) =cos2 x

cos2 x= 1

for all values of x for which cosx is nonzero. That is, f(x) = 1 for every x thatis not an integer multiple of π/2 (and f(x) is not defined when x is an integermultiple of π/2). Therefore, f ′(x) = 0 for every x on which f exists, and inparticular f ′(π/4) = 0. Also, f(π/4) = 1, so the tangent line to f at x = π/4 is the

752

Page 769: CLP-1 Differential Calculus

SOLUTIONS TO EXERCISES

line with slope 0, passing through the point (π/4, 1):

y = 1

2.9.4.16. Solution. Velocity is the derivative of position with respect to time. So,the velocity of the particle is given by s′(t). We need to find s′(t), and determinewhen it is zero.To differentiate, we us the chain rule.

s′(t) = et3−7t2+8t · d

dtt3 − 7t2 + 8t

= et3−7t2+8t · (3t2 − 14t+ 8)

To determine where this function is zero, we factor:

= et3−7t2+8t · (3t− 2)(t− 4)

So, the velocity is zero when et3−7t2+8t = 0, when 3t− 2 = 0, and when t− 4 = 0.Since et3−7t2+8t is never zero, this tells us that the velocity is zero precisely whent = 2

3or t = 4.

2.9.4.17. Solution. The slope of the tangent line is the derivative. If we letf(x) = tan x and g(x) = ex

2 , then f(g(x)) = tan(ex2), so y′ = f ′(g(x)) · g′(x):

y′ = sec2(ex2

) · d

dxex2

We find ourselves once more in need of the chain rule:

= sec2(ex2

) · ex2 d

dxx2

= sec2(ex2

) · ex2 · 2x

Finally, we evaluate this derivative at the point x = 1:

y′(1) = sec2(e) · e · 2= 2e sec2 e

2.9.4.18. *. Solution. Using the Product rule,

y′ =d

dxe4x tanx+ e4x sec2 x

and the chain rule:

= e4x · d

dx4x · tanx+ e4x sec2 x

= 4e4x tanx+ e4x sec2 x

753

Page 770: CLP-1 Differential Calculus

SOLUTIONS TO EXERCISES

2.9.4.19. *. Solution. Using the quotient rule,

f ′(x) =(3x2)(1 + e3x)− (x3) · d

dx1 + e3x

(1 + e3x)2

Now, the chain rule:

=(3x2)(1 + e3x)− (x3)(3e3x)

(1 + e3x)2

So, when x = 1:

f ′(1) =3(1 + e3)− 3e3

(1 + e3)2 =3

(1 + e3)2

2.9.4.20. *. Solution. This requires us to apply the chain rule twice.

d

dx

esin2(x)

= esin2(x) · d

dx

sin2(x)

= esin2(x)(2 sin(x)) · d

dxsin(x)

= esin2(x)(2 sin(x)) · cos(x)

2.9.4.21. *. Solution. This requires us to apply the chain rule twice.

d

dx

sin(e5x)

= cos

(e5x)· d

dx

e5x

= cos(e5x)(e5x) · d

dx5x

= cos(e5x)(e5x) · 5

2.9.4.22. *. Solution. We’ll use the chain rule twice.

d

dx

ecos(x2)

= ecos(x2) · d

dxcos(x2)

= ecos(x2) · (− sin(x2)) · d

dxx2

= −ecos(x2) · sin(x2) · 2x

2.9.4.23. *. Solution. We start with the chain rule:

y′ = − sin(x2 +

√x2 + 1

)· d

dx

x2 +

√x2 + 1

= − sin

(x2 +

√x2 + 1

)·(

2x+d

dx

√x2 + 1

)and find ourselves in need of chain rule a second time:

= − sin(x2 +

√x2 + 1

)·(

2x+1

2√x2 + 1

· d

dx

x2 + 1

)

754

Page 771: CLP-1 Differential Calculus

SOLUTIONS TO EXERCISES

= − sin(x2 +

√x2 + 1

)·(

2x+2x

2√x2 + 1

)2.9.4.24. *. Solution.

y = (1 + x2) cos2 x

Using the product rule,

y′ = (2x) cos2 x+ (1 + x2)d

dxcos2 x

Here, we’ll need to use the chain rule. Remember cos2 x = [cosx]2.

= 2x cos2 x+ (1 + x2)2cosx · d

dxcosx

= 2x cos2 x+ (1 + x2)2 cosx · (− sinx)

= 2x cos2 x− 2(1 + x2) sinx cosx

2.9.4.25. *. Solution. We use the quotient rule, noting by the chain rule thatd

dxe3x = 3e3x:

y′ =(1 + x2) · 3e3x − e3x(2x)

(1 + x2)2

=e3x(3x2 − 2x+ 3)

(1 + x2)2

2.9.4.26. *. Solution. By the chain rule,

d

dx

h(x2)

= h′(x2) · 2x

Using the product rules and the result above,

g′(x) = 3x2h(x2) + x3h′(x2)2x

Plugging in x = 2:

g′(2) = 3(22)h(22) + 23h′(22)2× 2

= 12h(4) + 32h′(4) = 12× 2− 32× 2

= −40

2.9.4.27. *. Solution. Let f(x) = xe−(x2−1)/2 = xe(1−x2)/2. Then, using the prod-uct rule,

f ′(x) = e(1−x2)/2 + x · d

dx

e(1−x2)/2

755

Page 772: CLP-1 Differential Calculus

SOLUTIONS TO EXERCISES

Here, we need the chain rule:

= e(1−x2)/2 + x · e(1−x2)/2 d

dx

1

2(1− x2)

= e(1−x2)/2 + x · e(1−x2)/2 · (−x)

= (1− x2)e(1−x2)/2

There is no power of e that is equal to zero; so if the product above is zero, it mustbe that 1 − x2 = 0. This happens for x = ±1. On the curve, when x = 1, y = 1,and when x = −1, y = −1. So the points are (1, 1) and (−1,−1).

2.9.4.28. Solution. The question asks when s′(t) is negative. So, we start bydifferentiating. Using the chain rule:

s′(t) = cos

(1

t

)· d

dt

1

t

= cos

(1

t

)· −1

t2

When t ≥ 1, 1t

is between 0 and 1. Since cos θ is positive for 0 ≤ θ < π/2, andπ/2 > 1, we see that cos

(1t

)is positive for the entire domain of s(t). Also, −1

t2is

negative for the entire domain of the function. We conclude that s′(t) is negativefor the entire domain of s(t), so the particle is always moving in the negativedirection.

2.9.4.29. Solution. We present two solutions: one where we dive right in anduse the quotient rule, and another where we simplify first and use the productrule.

• Solution 1: We begin with the quotient rule:

f ′(x) =cos3(5x− 7) d

dxex − ex d

dxcos3(5x− 7)

cos6(5x− 7)

=cos3(5x− 7)ex − ex d

dxcos3(5x− 7)

cos6(5x− 7)

Now, we use the chain rule. Since cos3(5x−7) = [cos(5x−7)]3, our “outside”function is g(x) = x3, and our “inside” function is h(x) = cos(5x− 1).

=cos3(5x− 7)ex − ex · 3cos2(5x− 7) · d

dxcos(5x− 7)

cos6(5x− 7)

We need the chain rule again!

=cos3(5x−7)ex−ex · 3cos2(5x−7) · [− sin(5x−7) · d

dx5x−7]

cos6(5x− 7)

=cos3(5x− 7)ex − ex · 3cos2(5x− 7) · [− sin(5x− 7) · 5]

cos6(5x− 7)

756

Page 773: CLP-1 Differential Calculus

SOLUTIONS TO EXERCISES

We finish by simplifying:

=ex cos2(5x− 7) (cos(5x− 7) + 15 sin(5x− 7))

cos6(5x− 7)

= excos(5x− 7) + 15 sin(5x− 7)

cos4(5x− 7)

= ex(sec3(5x− 7) + 15 tan(5x− 7) sec3(5x− 7))

= ex sec3(5x− 7)(1 + 15 tan(5x− 7))

• Solution 2: We simplify to avoid the quotient rule:

f(x) =ex

cos3(5x− 7)

= ex sec3(5x− 7)

Now we use the product rule to differentiate:

f ′(x) = ex sec3(5x− 7) + exd

dxsec3(5x− 7)

Here, we’ll need the chain rule. Since sec3(5x − 7) = [sec(5x − 7)]3, our“outside” function is g(x) = x3 and our “inside” function is h(x) = sec(5x−7), so that g(h(x)) = [sec(5x− 7)]3 = sec3(5x− 7).

= ex sec3(5x− 7) + ex · 3 sec2(5x− 7) · d

dxsec(5x− 7)

We need the chain rule again! Recall ddxsecx = secx tanx.

= ex sec3(5x− 7)

+ ex · 3 sec2(5x−7) · sec(5x−7) tan(5x−7) · d

dx5x−7

= ex sec3(5x−7)

+ ex · 3 sec2(5x−7) · sec(5x−7) tan(5x−7) · 5

We finish by simplifying:

= ex sec3(5x− 7)(1 + 15 tan(5x− 7))

2.9.4.30. *. Solution.

• Solution 1: In Example 2.6.6, we generalized the product rule to three fac-tors:

d

dxf(x)g(x)h(x) = f ′(x)g(x)h(x) + f(x)g′(x)h(x)

+ f(x)g(x)h′(x)

757

Page 774: CLP-1 Differential Calculus

SOLUTIONS TO EXERCISES

Using this rule:

d

dx

(x)(e2x)

(cos 4x)

=d

dxx · e2x cos 4x+x · d

dx

e2x· cos 4x+xe2x · d

dxcos 4x

= e2x cos 4x+ x(2e2x

)cos 4x+ xe2x(−4 sin 4x)

= e2x cos 4x+ 2xe2x cos 4x− 4xe2x sin 4x

• Solution 2: We can use the product rule twice. In the first step, we split thefunction xe2x cos 4x into the product of two functions.

d

dx

(xe2x

)(cos 4x)

=

d

dx

xe2x

· cos 4x+ xe2x · d

dxcos 4x

=

(d

dxx · e2x + x · d

dx

e2x)· cos 4x+ xe2x · d

dxcos 4x

=(e2x + x

(2e2x

))· cos 4x+ xe2x(−4 sin 4x)

= e2x cos 4x+ 2xe2x cos 4x− 4xe2x sin 4x

Exercises — Stage 32.9.4.31. Solution. At time t, the particle is at the point

(x(t), y(t)

), with x(t) =

cos t and y(t) = sin t. Over time, the particle traces out a curve; let’s call that curvey = f(x). Then y(t) = f

(x(t)

), so the slope of the curve at the point

(x(t), y(t)

)is

f ′(x(t)

). You are to determine the values of t for which f ′

(x(t)

)= −1.

By the chain rule

y′(t) = f ′(x(t)

)· x′(t)

Substituting in x(t) = cos t and y(t) = sin t gives

cos t = f ′(x(t)

)·(− sin t

)so that

f ′(x(t)

)= −cos t

sin t

is −1 precisely when sin t = cos t. This happens whenever t = π4.

Remark: the path traced by the particle is a semicircle. You can think about thepoint on the unit circle with angle t, or you can notice that x2+y2 = sin2 t+cos2 t =1.

758

Page 775: CLP-1 Differential Calculus

SOLUTIONS TO EXERCISES

2.9.4.32. *. Solution. Let f(x) = ex+x2 and g(x) = 1 + x. Then f(0) = g(0) = 1.f ′(x) = (1 + 2x)ex+x2 and g′(x) = 1. When x > 0,

f ′(x) = (1 + 2x)ex+x2 > 1 · ex+x2 = ex+x2 > e0+02 = 1 = g′(x).

Since f(0) = g(0), and f ′(x) > g′(x) for all x > 0, that means f and g start at thesame place, but f always grows faster. Therefore, f(x) > g(x) for all x > 0.

2.9.4.33. Solution. Since sin 2x and 2 sinx cosx are the same function, they havethe same derivative.

sin 2x = 2 sin x cosx

⇒ d

dxsin 2x =

d

dx2 sinx cosx

2 cos 2x = 2[cos2 x− sin2 x]

cos 2x = cos2 x− sin2 x

We conclude cos 2x = cos2 x− sin2 x, which is another common trig identity.Remark: if we differentiate both sides of this equation, we get the original iden-tity back.

2.9.4.34. Solution.

f(x) =3

√ecscx2

√x3 − 9 tanx

=

(ecscx2

√x3 − 9 tanx

) 13

To begin the differentiation, we can choose our “outside” function to be g(x) =

x13 , and our “inside” function to be h(x) =

ecscx2

√x3 − 9 tanx

. Then f(x) = g(h(x)),

so f ′(x) = g′(h(x)) · h′(x) = 13(h(x))−

23h′(x):

f ′(x) =1

3

(ecscx2

√x3 − 9 tanx

)−23

· d

dx

ecscx2

√x3 − 9 tanx

=1

3

(√x3 − 9 tanx

ecscx2

) 23

· d

dx

ecscx2

√x3 − 9 tanx

This leads us to use the quotient rule:

=1

3

(√x3 − 9 tanx

ecscx2

) 23

√x3 − 9 tanx ddx

ecscx2

− ecscx2 d

dx

√x3 − 9 tanx

(tan2 x)(x3 − 9)

759

Page 776: CLP-1 Differential Calculus

SOLUTIONS TO EXERCISES

Let’s figure out those two derivatives on their own, then plug them in. Using thechain rule twice:

d

dx

ecscx2

= ecscx2 d

dx

cscx2

= ecscx2 · (− csc(x2) cot(x2)) · d

dxx2

= −2xecscx2 cos(x2)

sin2(x2)

For the other derivative, we start with the product rule, then chain:

d

dx

√x3 − 9 tanx

=

d

dx

√x3 − 9

· tanx+

√x3 − 9 sec2 x

=1

2√x3−9

d

dx

x3−9

· tanx+

√x3−9 sec2 x

=3x2 tanx

2√x3 − 9

+√x3 − 9 sec2 x

Now, we plug these into our equation for f ′(x):

f ′(x) =1

3

(√x3 − 9 tanx

ecscx2

) 23

√x3 − 9 tanx ddx

ecscx2

− ecscx2 d

dx

√x3 − 9 tanx

(tan2 x)(x3 − 9)

=

1

3

(√x3 − 9 tanx

ecscx2

) 23

·

ecscx2(− 2x

√x3−9 tanx cos(x2)

sin2(x2)− 3x2 tanx

2√x3−9

−√x3−9 sec2 x

)(tan2 x)(x3 − 9)

2.9.4.35. Solution. 2.9.4.35.a The table below gives us a number of points onour graph, and the times they occur.

760

Page 777: CLP-1 Differential Calculus

SOLUTIONS TO EXERCISES

t (sin t, cos2 t)

0 (0, 1)

π/4 ( 1√2, 1

2)

π/2 (1, 0)

3π/4 ( 1√2, 1

2)

π (0, 1)

5π/4 (− 1√2, 1

2)

3π/2 (−1, 0)

7π/4 (− 1√2, 1

2)

2π (0, 1)

These points will repeat with a period of 2π. With this information, we have apretty good idea of the particle’s motion:

x

y

(1,0)t=π/2

(−1,0)t=3π/2

(0,1)t=0,π,2π

(− 1√

2, 12

)t=5π/4,7π/4

(1√2, 12

)t=π/4,3π/4

The particle traces out an arc, pointing down. It starts at t = 0 at the top part ofthe graph at (1, 0), then is moves to the right until it hits (1, 0) at time t = π/2.From there it reverses direction and moves along the curve to the left, hitting thetop at time t = π and reaching (−1, 0) at time t = 3π/2. Then it returns to the topat t = 2π and starts again.So, it starts at the top of the curve, then moves back for forth along the length ofthe curve. If goes right first, and repeats its cycle every 2π units of time.2.9.4.35.b Let y = f(x) be the curve the particle traces in the xy-plane. Since x

is a function of t, y(t) = f(x(t)). What we want to find isdf

dxwhen t =

(10π

3

).

Sincedf

dxis a function of x, we note that when t =

(10π

3

), x = sin

(10π

3

)=

sin

(4π

3

)= −√

3

2. So, the quantity we want to find (the slope of the tangent line

to the curve y = f(x) traced by the particle at the time t =

(10π

3

)is given by

761

Page 778: CLP-1 Differential Calculus

SOLUTIONS TO EXERCISES

df

dx

(−√

3

2

).

Using the chain rule:

y(t) = f(x(t))

dy

dt=

d

dtf(x(t)) =

df

dx· dx

dt

so,df

dx=

dy

dt÷ dx

dt

Using y(t) = cos2 t and x(t) = sin t:df

dx= (−2 cos t sin t)÷ (cos t) = −2 sin t = −2x

So, when t =10π

3and x = −

√3

2,

df

dx

(−√

3

2

)= −2 · −

√3

2=√

3.

Remark: The standard way to write this problem is to omit the notation f(x),and let the variable y stand for two functions. When t is the variable, y(t) =cos2 t gives the y-coordinate of the particle at time t. When x is the variable, y(x)gives the y-coordinate of the particle given its position along the x-axis. Thisis an abuse of notation, because if we write y(1), it is not clear whether we arereferring to the y-coordinate of the particle when t = 1 (in this case, y = cos2(1) ≈0.3), or the y-coordinate of the particle when x = 1 (in this case, looking at ourtable of values, y = 0). Although this notation is not strictly “correct,” it is verycommonly used. So, you might see a solution that looks like this:

The slope of the curve isdy

dx. To find

dy

dx, we use the chain rule:

dy

dt=

dy

dx· dx

dtd

dt

cos2 t

=

dy

dx· d

dtsin t

−2 cos t sin t =dy

dx· cos t

dy

dx= −2 sin t

So, when t =10π

3,

dy

dx= −2 sin

(10π

3

)= −2

(−√

3

2

)=√

3.

In this case, it is up to the reader to understand when y is used as a functionof t, and when it is used as a function of x. This notation (using y to be twofunctions, y(t) and y(x)) is actually the accepted standard, so you should be ableto understand it.

2.10 · The Natural Logarithm2.10.3 · Exercises

762

Page 779: CLP-1 Differential Calculus

SOLUTIONS TO EXERCISES

Exercises — Stage 12.10.3.1. Solution. We are given that one speaker produces 3dB. So if P is thepower of one speaker,

3 = V (P ) = 10 log10

(P

S

).

So, for ten speakers:

V (10P ) = 10 log10

(10P

S

)= 10 log10

(P

S

)+ 10 log10 (10)

= 3 + 10(1) = 13dB

and for one hundred speakers:

V (100P ) = 10 log10

(100P

S

)= 10 log10

(P

S

)+ 10 log10 (100)

= 3 + 10(2) = 23dB

2.10.3.2. Solution. The investment doubles when it hits $2000. So, we find thevalue of t that gives A(t) = 2000:

2000 = A(t)

2000 = 1000et/20

2 = et/20

log 2 =t

2020 log 2 = t

2.10.3.3. Solution. From our logarithm rules, we know that when y is positive,log(y2) = 2 log y. However, the expression cosx does not always take on positivevalues, so (a) is not correct. (For instance, when x = π, log(cos2 x) = log(cos2 π) =log ((−1)2) = log(1) = 0, while 2 log(cos π) = 2 log(−1), which does not exist.)Because cos2 x is never negative, we notice that cos2 x = | cosx|2. When cosxis nonzero, | cosx| is positive, so our logarithm rules tell us log (| cosx|2) =2 log | cosx|. When cosx is exactly zero, then both log(cos2 x) and 2 log | cosx| donot exist. So, log(cos2 x) = 2 log | cosx|.

Exercises — Stage 22.10.3.4. Solution.

• Solution 1: Using the chain rule,d

dxlog(10x) =

1

10x· 10 =

1

x.

• Solution 2: Simplifying,d

dxlog(10x) =

d

dxlog(10) + log x = 0 +

1

x=

1

x.

763

Page 780: CLP-1 Differential Calculus

SOLUTIONS TO EXERCISES

2.10.3.5. Solution.

• Solution 1: Using the chain rule,d

dx

log(x2)

=

1

x2· 2x =

2

x.

• Solution 2: Simplifying,d

dx

log(x2)

=

d

dx2 log(x) =

2

x.

2.10.3.6. Solution. Don’t be fooled by a common mistake: log(x2 + x) is not the

same as log(x2) + log x. We differentiate using the chain rule:d

dx

log(x2 + x)

=

1

x2 + x· (2x+ 1) =

2x+ 1

x2 + x.

2.10.3.7. Solution. We know the derivative of the natural logarithm (base e), sowe use the base-change formula:

f(x) = log10 x =log x

log 10

Since log 10 is a constant:

f ′(x) =1

x log 10.

2.10.3.8. *. Solution.

• Solution 1: Using the quotient rule,

y′ =x3 1

x− (log x) · 3x2

x6=x2 − 3x2 log x

x6=

1− 3 log x

x4.

• Solution 2: Using the product rule with y = log x · x−3,

y′ =1

xx−3 + log x · (−3)x−4 = x−4(1− 3 log x)

2.10.3.9. Solution. Using the chain rule,

d

dθlog(sec θ) =

1

sec θ· (sec θ · tan θ)

= tan θ

Remark: the domain of the function log(sec θ) is those values of θ for which sec θis positive: so, the intervals

((2n− 1

2

)π,(2n+ 1

2

)π)

where n is any integer. Cer-tainly the tangent function has a larger domain than this, but outside the domainof log(sec θ), tan θ is not the derivative of log(sec θ).

2.10.3.10. Solution. Let’s start in with the chain rule.

f ′(x) = ecos(log x) · d

dxcos (log x)

764

Page 781: CLP-1 Differential Calculus

SOLUTIONS TO EXERCISES

We’ll need the chain rule again:

= ecos(log x)(− sin(log x)) · d

dxlog x

= ecos(log x)(− sin(log x)) · 1

x

=−ecos(log x) sin(log x)

x

Remark: Although we have a logarithm in the exponent, we can’t cancel. Theexpression ecos(log x) is not the same as the expression xcosx, or cosx.

2.10.3.11. *. Solution.

y = log(x2 +√x4 + 1)

So, we’ll need the chain rule:

y′ =d

dx

x2 +

√x4 + 1

x2 +

√x4 + 1

=2x+ d

dx

√x4 + 1

x2 +

√x4 + 1

We need the chain rule again:

=2x+

ddxx4+12√x4+1

x2 +√x4 + 1

=2x+ 4x3

2√x4+1

x2 +√x4 + 1

.

2.10.3.12. *. Solution. This requires us to apply the chain rule twice.

d

dx

√− log(cos x)

=

1

2√− log(cosx)

· d

dx− log (cosx)

= − 1

2√− log(cos x)

· 1

cosx

d

dxcosx

= − 1

2√− log(cos x)

· 1

cosx· (− sinx)

=tanx

2√− log(cosx)

Remark: it looks strange to see a negative sign in the argument of a square root.Since the cosine function always gives values that are at most 1, log(cos x) is al-ways negative or zero over its domain. So,

√log(cosx) is only defined for the

points where cosx = 1 (and so log(cos x) = 0–this isn’t a very interesting func-tion! In contrast, − log(cos x) is always positive or zero over its domain – andtherefore we can always take its square root.

765

Page 782: CLP-1 Differential Calculus

SOLUTIONS TO EXERCISES

2.10.3.13. *. Solution. Under the chain rule, ddx

log f(x) = 1f(x)

f ′(x). So

d

dx

log(x+√x2 + 4

)=

1

x+√x2 + 4

· d

dx

x+√x2 + 4

=

1

x+√x2 + 4

·(

1 +2x

2√x2 + 4

)=

1

x+√x2 + 4

·(

2√x2 + 4 + 2x

2√x2 + 4

)=

1√x2 + 4

2.10.3.14. *. Solution. Using the chain rule,

g′(x) =d

dxex2 +

√1 + x4

ex2 +√

1 + x4

=2xex

2+ 4x3

2√

1+x4

ex2 +√

1 + x4

(√1 + x4

√1 + x4

)

=2xex

2√1 + x4 + 2x3

ex2√

1 + x4 + 1 + x4

2.10.3.15. *. Solution. Using logarithm rules makes this an easier problem:

g(x) = log(2x− 1)− log(2x+ 1)

So, g′(x) =2

2x− 1− 2

2x+ 1

and g′(1) =2

1− 2

3=

4

3

2.10.3.16. Solution. We begin by simplifying:

f(x) = log

√(x2 + 5)3

x4 + 10

= log

(((x2 + 5)3

x4 + 10

)1/2)

=1

2log

((x2 + 5)3

x4 + 10

)=

1

2

[log((x2 + 5)3

)− log(x4 + 10)

]=

1

2

[3 log

((x2 + 5)

)− log(x4 + 10)

]Now, we differentiate using the chain rule:

f ′(x) =1

2

[3

2x

x2 + 5− 4x3

x4 + 10

]

766

Page 783: CLP-1 Differential Calculus

SOLUTIONS TO EXERCISES

=3x

x2 + 5− 2x3

x4 + 10

Remark: it is a common mistake to write log(x2 + 4) as log(x2) + log(4). Theseexpressions are not equivalent!

2.10.3.17. Solution. We use the chain rule twice, followed by the product rule:

f ′(x) =1

g(xh(x))· d

dxg(xh(x))

=1

g(xh(x))· g′(xh(x)) · d

dxxh(x)

=1

g(xh(x)

) · g′(xh(x))·[h(x) + xh′(x)

]In particular, when x = 2:

f ′(2) =1

g(2h(2)

) · g′(2h(2))·[h(2) + 2h′(2)

]=g′(4)

g(4)

[2 + 2× 3

]=

5

3

[2 + 2× 3

]=

40

3

2.10.3.18. *. Solution. In the text, we saw thatd

dxax = ax log a for any con-

stant a. So,d

dxπx = πx log π.

By the power rule,d

dxxπ = πxπ−1.

Therefore, g′(x) = πx log π + πxπ−1.Remark: we had to use two different rules for the two different terms in g(x).Although the functions πx and xπ look superficially the same, they behave differ-ently, as do their derivatives. A function of the form (constant)x is an exponentialfunction and not eligible for the power rule, while a function of the form xconstant

is exactly the class of function the power rule applies to.

2.10.3.19. Solution. We have the power rule to tell us the derivative of func-tions of the form xn, where n is a constant. However, here our exponent is not aconstant. Similarly, in this section we learned the derivative of functions of theform ax, where a is a constant, but again, our base is not a constant! Although

the resultd

dxax = ax log a is not what we need, the method used to differentiate ax

will tell us the derivative of xx.We’ll set g(x) = log(xx), because now we can use logarithm rules to simplify:

g(x) = log(f(x)) = x log x

767

Page 784: CLP-1 Differential Calculus

SOLUTIONS TO EXERCISES

Now, we can use the product rule to differentiate the right side, and the chainrule to differentiate log(f(x)):

g′(x) =f ′(x)

f(x)= log x+ x

1

x= log x+ 1

Finally, we solve for f ′(x):

f ′(x) = f(x)(log x+ 1) = xx(log x+ 1)

2.10.3.20. *. Solution. In Question 2.10.3.19, we sawd

dxxx = xx(log x + 1).

Using the base-change formula, log10(x) =log x

log 10. Since log10 is a constant,

f ′(x) =d

dx

xx +

log x

log 10

= xx(log x+ 1) +

1

x log 10

2.10.3.21. Solution. Rather than set in with a terrible chain rule problem, we’lluse logarithmic differentiation. Instead of differentiating f(x), we differentiate anew function log(f(x)), after simplifying.

log(f(x)) = log4

√(x4 + 12)(x4 − x2 + 2)

x3

=1

4log

((x4 + 12)(x4 − x2 + 2)

x3

)=

1

4

(log(x4 + 12) + log(x4 − x2 + 2)− 3 log x

)Now that we’ve simplified, we can efficiently differentiate both sides. It is impor-tant to remember that we aren’t differentiating f(x) directly–we’re differentiatinglog(f(x)).

f ′(x)

f(x)=

1

4

(4x3

x4 + 12+

4x3 − 2x

x4 − x2 + 2− 3

x

)Our final step is to solve for f ′(x):

f ′(x) = f(x)1

4

(4x3

x4 + 12+

4x3 − 2x

x4 − x2 + 2− 3

x

)=

1

4

(4

√(x4 + 12)(x4 − x2 + 2)

x3

)(4x3

x4 + 12+

4x3 − 2x

x4 − x2 + 2− 3

x

)It was possible to differentiate this function without logarithms, but the loga-rithms make it more efficient.

768

Page 785: CLP-1 Differential Calculus

SOLUTIONS TO EXERCISES

2.10.3.22. Solution. It’s possible to do this using the product rule a number oftimes, but it’s easier to use logarithmic differentiation. Set

g(x) = log(f(x)) = log[(x+ 1)(x2 + 1)2(x3 + 1)3(x4 + 1)4(x5 + 1)5

]Now we can use logarithm rules to change g(x) into a form that is friendlier todifferentiate:

= log(x+ 1) + log(x2 + 1)2 + log(x3 + 1)3 + log(x4 + 1)4

+ log(x5 + 1)5

= log(x+ 1) + 2 log(x2 + 1) + 3 log(x3 + 1) + 4 log(x4 + 1)

+ 5 log(x5 + 1)

Now, we differentiate g(x) using the chain rule:

g′(x) =f ′(x)

f(x)=

1

x+ 1+

4x

x2 + 1+

9x2

x3 + 1+

16x3

x4 + 1+

25x4

x5 + 1

Finally, we solve for f ′(x):

f ′(x) = f(x)

[1

x+ 1+

4x

x2 + 1+

9x2

x3 + 1+

16x3

x4 + 1+

25x4

x5 + 1

]= (x+ 1)(x2 + 1)2(x3 + 1)3(x4 + 1)4(x5 + 1)5

·[

1

x+ 1+

4x

x2 + 1+

9x2

x3 + 1+

16x3

x4 + 1+

25x4

x5 + 1

]2.10.3.23. Solution. We could do this with quotient and product rules, but itwould be pretty painful. Insteady, let’s use a logarithm.

f(x) =

(5x2 + 10x+ 15

3x4 + 4x3 + 5

)(1

10(x+ 1)

)=

(x2 + 2x+ 3

3x4 + 4x3 + 5

)(1

2(x+ 1)

)log(f(x)) = log

[(x2 + 2x+ 3

3x4 + 4x3 + 5

)(1

2(x+ 1)

)]= log

(x2 + 2x+ 3

3x4 + 4x3 + 5

)+ log

(1

2(x+ 1)

)= log

(x2+2x+3

)− log

(3x4+4x3+5

)− log(x+1)− log(2)

Now we have a function that we can differentiate more cleanly than our originalfunction.

d

dxlog(f(x)) =

d

dx

log(x2 + 2x+ 3

)− log

(3x4 + 4x3 + 5

)− log (x+ 1)− log (2)

769

Page 786: CLP-1 Differential Calculus

SOLUTIONS TO EXERCISES

f ′(x)

f(x)=

2x+ 2

x2 + 2x+ 3− 12x3 + 12x2

3x4 + 4x3 + 5− 1

x+ 1

=2(x+ 1)

x2 + 2x+ 3− 12x2(x+ 1)

3x4 + 4x3 + 5− 1

x+ 1

Finally, we solve for f(x):

f ′(x) = f(x)

(2(x+ 1)

x2 + 2x+ 3− 12x2(x+ 1)

3x4 + 4x3 + 5− 1

x+ 1

)=

(x2 + 2x+ 3

3x4+4x3+5

)(1

2(x+1)

)(2(x+ 1)

x2+2x+3− 12x2(x+ 1)

3x4+4x3+5− 1

x+1

)=

(x2 + 2x+ 3

3x4 + 4x3 + 5

)(1

x2 + 2x+ 3− 6x2

3x4 + 4x3 + 5− 1

2(x+ 1)2

)2.10.3.24. *. Solution. Since f(x) has the form of a function raised to a func-tional power, we will use logarithmic differentiation.

log(f(x)) = log((cosx)sinx

)= sinx · log(cos x)

Logarithm rules allowed us to simplify. Now, we differentiate both sides of thisequation:

f ′(x)

f(x)= (cosx) log(cosx) + sin x · − sinx

cosx

= (cosx) log(cosx)− sinx tanx

Finally, we solve for f ′(x):

f ′(x) = f(x) [(cosx) log(cosx)− sinx tanx]

= (cosx)sinx [(cosx) log(cosx)− sinx tanx]

Remark: negative numbers behave in a complicated manner when they are thebase of an exponential expression. For example, the expression (−1)x is definedwhen x is the reciprocal of an odd number (like x = 1

5or x = 1

7), but not when

x is the reciprocal of an even number (like x = 12). Since the domain of f(x) was

restricted to (0, π2), cosx is always positive, and we avoid these complications.

2.10.3.25. *. Solution. Since f(x) has the form of a function raised to a func-tional power, we will use logarithmic differentiation. We take the logarithm ofthe function, and make use of logarithm rules:

log ((tanx)x) = x log(tan x)

Now, we can differentiate:d

dx(tanx)x(tanx)x

= log(tanx) + x · sec2 x

tanx

= log(tanx) +x

sinx cosx

770

Page 787: CLP-1 Differential Calculus

SOLUTIONS TO EXERCISES

Finally, we solve for the derivative we want,d

dx(tanx)x:

d

dx(tanx)x = (tanx)x

(log(tan x) +

x

sinx cosx

)Remark: the restricted domain (0, π/2) ensures that tanx is a positive number,so we avoid the problems that arise by raising a negative number to a variety ofpowers.

2.10.3.26. *. Solution. We use logarithmic differentiation.

log f(x) = log(x2 + 1) · (x2 + 1)

We differentiate both sides to obtain:

f ′(x)

f(x)=

d

dx

log(x2 + 1) · (x2 + 1)

=

2x

x2 + 1(x2 + 1) + 2x log(x2 + 1)

= 2x(1 + log(x2 + 1))

Now, we solve for f ′(x):

f ′(x) = f(x) · 2x(1 + log(x2 + 1))

= (x2 + 1)x2+1 · 2x(1 + log(x2 + 1))

2.10.3.27. *. Solution. We use logarithmic differentiation: we modify our func-tion to consider

log f(x) = log(x2 + 1) · sinx

We differentiate using the product and chain rules:

f ′(x)

f(x)=

d

dx

log(x2 + 1) · sinx

= cosx · log(x2 + 1) +

2x sinx

x2 + 1

Finally, we solve for f ′(x)

f ′(x) = f(x) ·(

cosx · log(x2 + 1) +2x sinx

x2 + 1

)= (x2 + 1)sin(x) ·

(cosx · log(x2 + 1) +

2x sinx

x2 + 1

)2.10.3.28. *. Solution. We use logarithmic differentiation; so we modify ourfunction to consider

log f(x) = log(x) · cos3(x)

771

Page 788: CLP-1 Differential Calculus

SOLUTIONS TO EXERCISES

Differentiating, we find:

f ′(x)

f(x)=

d

dx

log(x) · cos3(x)

= 3 cos2(x) · (− sin(x)) · log(x) +

cos3(x)

x

Finally, we solve for f ′(x):

f ′(x) = f(x) ·(−3 cos2(x) sin(x) log(x) +

cos3(x)

x

)= xcos3(x) ·

(−3 cos2(x) sin(x) log(x) +

cos3(x)

x

)Remark: negative numbers behave in a complicated manner when they are thebase of an exponential expression. For example, the expression (−1)x is definedwhen x is the reciprocal of an odd number (like x = 1

5or x = 1

7), but not when

x is the reciprocal of an even number (like x = 12). Since the domain of f(x) was

restricted so that x is always positive, we avoid these complications.

2.10.3.29. *. Solution. We use logarithmic differentiation. So, we modify ourfunction and consider

log f(x) = (x2 − 3) · log(3 + sin(x)) .

We differentiate:

f ′(x)

f(x)=

d

dx

(x2 − 3) · log(3 + sin(x))

= 2x log(3 + sin(x)) + (x2 − 3)

cos(x)

3 + sin(x)

Finally, we solve for f ′(x):

f ′(x) = f(x) ·[2x log(3 + sin(x)) +

(x2 − 3) cos(x)

3 + sin(x)

]= (3 + sin(x))x

2−3 ·[2x log(3 + sin(x)) +

(x2 − 3) cos(x)

3 + sin(x)

]

Exercises — Stage 32.10.3.30. Solution. We will use logarithmic differentiation. First, we take thelogarithm of our function, so we can use logarithm rules.

log([f(x)]g(x)

)= g(x) log(f(x))

772

Page 789: CLP-1 Differential Calculus

SOLUTIONS TO EXERCISES

Now, we differentiate. On the left side we use the chain rule, and on the rightside we use product and chain rules.

d

dx

log([f(x)]g(x)

)=

d

dxg(x) log(f(x))

ddx[f(x)]g(x)[f(x)]g(x)

= g′(x) log(f(x)) + g(x) · f′(x)

f(x)

Finally, we solve for the derivative of our original function.

d

dx[f(x)]g(x) = [f(x)]g(x)

(g′(x) log(f(x)) + g(x) · f

′(x)

f(x)

)Remark: in this section, we have differentiated problems of this type severaltimes–for example, Questions 2.10.3.24through 2.10.3.29.

2.10.3.31. Solution. Let g(x) := log(f(x)). Notice g′(x) = f ′(x)f(x)

.In order to show that the two curves have horizontal tangent lines at the samevalues of x, we will show two things: first, that if f(x) has a horizontal tangentline at some value of x, then also g(x) has a horizontal tangent line at that valueof x. Second, we will show that if g(x) has a horizontal tangent line at some valueof x, then also f(x) has a horizontal tangent line at that value of x.Suppose f(x) has a horizontal tangent line where x = x0 for some point x0. Thismeans f ′(x0) = 0. Then g′(x0) = f ′(x0)

f(x0). Since f(x0) 6= 0, f

′(x0)f(x0)

= 0f(x0)

= 0, so g(x)

also has a horizontal tangent line when x = x0. This shows that whenever f hasa horizontal tangent line, g has one too.Now suppose g(x) has a horizontal tangent line where x = x0 for some point x0.This means g′(x0) = 0. Then g′(x0) = f ′(x0)

f(x0)= 0, so f ′(x0) exists and is equal to

zero. Therefore, f(x) also has a horizontal tangent line when x = x0. This showsthat whenever g has a horizontal tangent line, f has one too.Remark: if we were not told that f(x) gives only positive numbers, it would notnecessarily be true that f(x) and log(f(x)) have horizontal tangent lines at thesame values of x. If f(x) had a horizontal tangent line at an x-value where f(x)were negative, then log(f(x)) would not exist there, let alone have a horizontaltangent line.

2.11 · Implicit Differentiation2.11.2 · Exercises

Exercises — Stage 12.11.2.1. Solution. We use the power rule (a) and the chain rule (b): the powerrule tells us to “bring down the 2”, and the chain rule tells us to multiply by y′.There is no need for the quotient rule here, as there are no quotients. Exponen-tial functions have the form (constant)function, but our function has the form

773

Page 790: CLP-1 Differential Calculus

SOLUTIONS TO EXERCISES

(function)constant, so we did not use (d).

2.11.2.2. Solution. At (0, 4) and (0,−4), the curve looks to be horizontal, if youzoom in: a tangent line here would have derivative zero. At the origin, the curve

looks like its tangent line is vertical, sody

dxdoes not exist.

x

y

2.11.2.3. Solution. (a) No. A function must pass the vertical line test: one inputcannot result in two (or more) outputs. Since one value of x sometimes corre-sponds to two values of y (for example, when x = π/4, y is ±1/

√2), there is no

function f(x) so that y = f(x) captures every point on the circle.Remark: y = ±

√1− x2 does capture every point on the unit circle. However,

since one input x sometimes results in two outputs y, this expression is not afunction.(b) No, for the same reasons as (a). If f ′(x) is a function, then it can give at mostone slope corresponding to one value of x. Since one value of x can correspondto two points on the circle with different slopes, f ′(x) cannot give the slope ofevery point on the circle. For example, fix any 0 < a < 1. There are two pointson the circle with x-coordinate equal to a. At the upper one, the slope is strictlynegative. At the lower one, the slope is strictly positive.(c) We differentiate:

2x+ 2ydy

dx= 0

and solve fordy

dx

dy

dx= −x

y

774

Page 791: CLP-1 Differential Calculus

SOLUTIONS TO EXERCISES

But there is a y in the right-hand side of this equation, and it’s not clear how toget it out. Our answer in (b) tells us that, actually, we can’t get it out, if we wantthe right-hand side to be a function of x. The derivative cannot be expressed as afunction of x, because one value of x corresponds to multiple points on the circle.Remark: since y = ±

√1− x2, we could try writing

dy

dx= −x

y= ± x√

1− x2

but this is not a function of x. Again, in a function, one input leads to at most oneoutput, but here one value of x will usually lead to two values of dy

dx.

Exercises — Stage 22.11.2.4. *. Solution. Remember that y is a function of x. We begin with implicitdifferentiation.

xy + ex + ey = 1

y + xdy

dx+ ex + ey

dy

dx= 0

Now, we solve fordy

dx.

xdy

dx+ ey

dy

dx= −(ex + y)

(x+ ey)dy

dx= −(ex + y)

dy

dx= −e

x + y

ey + x

2.11.2.5. *. Solution. Differentiate both sides of the equation with respect to x:

eydy

dx= x · 2ydy

dx+ y2 + 1

Now, get the derivative on one side and solve

eydy

dx− 2xy

dy

dx= y2 + 1

dy

dx(ey − 2xy) = y2 + 1

dy

dx=

y2 + 1

ey − 2xy

2.11.2.6. *. Solution.

• First we find the x-coordinates where y = 1.

x2 tan(π

4

)+ 2x log(1) = 16

775

Page 792: CLP-1 Differential Calculus

SOLUTIONS TO EXERCISES

x2 · 1 + 2x · 0 = 16

x2 = 16

So x = ±4.

• Now we use implicit differentiation to get y′ in terms of x, y:

x2 tan(πy/4) + 2x log(y) = 16

2x tan(πy/4) + x2π

4sec2(πy/4) · y′ + 2 log(y) +

2x

y· y′ = 0 .

• Now set y = 1 and use tan(π/4) = 1 , sec(π/4) =√

2 to get

2x tan(π/4) + x2π

4sec2(π/4)y′ + 2 log(1) + 2x · y′ = 0

2x+π

2x2y′ + 2xy′ = 0

y′ = − 2x

x2π/2 + 2x= − 4

πx+ 4

• So at (x, y) = (4, 1) we have y′ = − 4

4π + 4= − 1

π + 1

• and at (x, y) = (−4, 1) we have y′ =1

π − 1

2.11.2.7. *. Solution. Differentiate the equation and solve:

3x2 + 4y3 dy

dx= − sin(x2 + y) ·

(2x+

dy

dx

)dy

dx= −2x sin(x2 + y) + 3x2

4y3 + sin(x2 + y)

2.11.2.8. *. Solution.

• First we find the x-coordinates where y = 0.

x2e0 + 4x cos(0) = 5

x2 + 4x− 5 = 0

(x+ 5)(x− 1) = 0

So x = 1,−5.

• Now we use implicit differentiation to get y′ in terms of x, y. Differentiateboth sides of

x2ey + 4x cos(y) = 5

to get

x2 · ey · y′ + 2xey + 4x(− sin(y)) · y′ + 4 cos(y) = 0

776

Page 793: CLP-1 Differential Calculus

SOLUTIONS TO EXERCISES

• Now set y = 0 to get

x2 · e0 · y′ + 2xe0 + 4x(− sin(0)) · y′ + 4 cos(0) = 0

x2y′ + 2x+ 4 = 0

y′ = −4 + 2x

x2.

• So at (x, y) = (1, 0) we have y′ = −6,

• and at (x, y) = (−5, 0) we have y′ = 625

.

2.11.2.9. *. Solution. Differentiate the equation and solve:

2x+ 2ydy

dx= cos(x+ y) ·

(1 +

dy

dx

)dy

dx=

cos(x+ y)− 2x

2y − cos(x+ y)

2.11.2.10. *. Solution.

• First we find the x-coordinates where y = 0.

x2 cos(0) + 2xe0 = 8

x2 + 2x− 8 = 0

(x+ 4)(x− 2) = 0

So x = 2,−4.

• Now we use implicit differentiation to get y′ in terms of x, y. Differentiateboth sides of

x2 cos(y) + 2xey = 8

to get

x2 · (− sin y) · y′ + 2x cos y + 2xey · y′ + 2ey = 0

• Now set y = 0 to get

x2 · (− sin 0) · y′ + 2x cos 0 + 2xe0 · y′ + 2e0 = 0

0 + 2x+ 2xy′ + 2 = 0

y′ = −2 + 2x

2x

= −1 + x

x

• So at (x, y) = (2, 0) we have y′ = −32,

• and at (x, y) = (−4, 0) we have y′ = −34.

777

Page 794: CLP-1 Differential Calculus

SOLUTIONS TO EXERCISES

2.11.2.11. Solution. The question asks at which points on the ellipsedy

dx= 1.

So, we begin by differentiating, implicitly:

2x+ 6ydy

dx= 0

We could solve fordy

dxat this point, but it’s not necessary. We want to know when

dy

dxis equal to one:

2x+ 6y(1) = 0

x = −3y

That is,dy

dx= 1 at those points along the ellipse where x = −3y. We plug this

into the equation of the ellipse to find the coordinates of these points.

(−3y)2 + 3y2 = 1

12y2 = 1

y = ± 1√12

= ± 1

2√

3

So, the points along the ellipse where the tangent line is parallel to the line y = x

occur when y =1

2√

3and x = −3y, and when y =

−1

2√

3and x = −3y. That is, the

points

(−√

3

2,

1

2√

3

)and

(√3

2,−1

2√

3

).

2.11.2.12. *. Solution. First, we differentiate implicitly with respect to x.

√xy = x2y − 2

1

2√xy· d

dxxy = (2x)y + x2 dy

dx

y + xdydx

2√xy

= 2xy + x2 dy

dx

Now, we plug in x = 1, y = 4, and solve fordy

dx:

4 + dydx

4= 8 +

dy

dxdy

dx= −28

3

2.11.2.13. *. Solution. Implicitly differentiating x2y(x)2 + x sin(y(x)) = 4 with

778

Page 795: CLP-1 Differential Calculus

SOLUTIONS TO EXERCISES

respect to x gives

2xy2 + 2x2yy′ + sin y + xy′ cos y = 0

Then we gather the terms containing y′ on one side, so we can solve for y′:

2x2yy′ + xy′ cos y = −2xy2 − sin y

y′(2x2y + x cos y) = −2xy2 − sin y

y′ = − 2xy2 + sin y

2x2y + x cos y

Exercises — Stage 32.11.2.14. *. Solution.

• First we find the x-ordinates where y = 0.

x2 + (1)e0 = 5

x2 + 1 = 5

x2 = 4

So x = 2,−2.

• Now we use implicit differentiation to get y′ in terms of x, y:

2x+ (y + 1)eydy

dx+ ey

dy

dx= 0

• Now set y = 0 to get

2x+ (0 + 1)e0 dy

dx+ e0 dy

dx= 0

2x+dy

dx+

dy

dx= 0

2x = −2dy

dx

x = −dy

dx

• So at (x, y) = (2, 0) we have y′ = −2,

• and at (x, y) = (−2, 0) we have y′ = 2.

2.11.2.15. Solution. The slope of the tangent line is, of course, given by thederivative, so let’s start by finding dy

dxof both shapes.

779

Page 796: CLP-1 Differential Calculus

SOLUTIONS TO EXERCISES

For the circle, we differentiate implicitly

2x+ 2ydy

dx= 0

and solve fordy

dx

dy

dx= −x

y

For the ellipse, we also differentiate implicitly:

2x+ 6ydy

dx= 0

and solve fordy

dx

dy

dx= − x

3y

What we want is a value of x where both derivatives are equal. However, theymight have different values of y, so let’s let y1 be the y-values associated with xon the circle, and let y2 be the y-values associated with x on the ellipse. That is,x2 + y2

1 = 1 and x2 + 3y22 = 1. For the slopes at (x, y1) on the circle and (x, y2) on

the ellipse to be equal, we need:

− xy1

= − x

3y2

x

(1

y1

− 1

3y2

)= 0

So x = 0 or y1 = 3y2. Let’s think about which x-values will have a y-coordinateof the circle be three times as large as a y-coordinate of the ellipse. If y1 = 3y2,(x, y1) is on the circle, and (x, y2) is on the ellipse, then x2 + y2

1 = x2 + (3y2)2 = 1and x2 + 3y2

2 = 1. In this case:

x2 + 9y22 = x2 + 3y2

2

9y22 = 3y2

2

y2 = 0

x = ±1

We need to be a tiny bit careful here: when y = 0, y′ is not defined for eithercurve. For both curves, when y = 0, the tangent lines are vertical (and so haveno real-valued slope!). Two vertical lines are indeed parallel.So, for x = 0 and for x = ±1, the two curves have parallel tangent lines.

780

Page 797: CLP-1 Differential Calculus

SOLUTIONS TO EXERCISES

x

y

x2 + y2 = 1

x2 + 3y2 = 1

2.12 · Inverse Trigonometric Functions2.12.2 · Exercises

Exercises — Stage 12.12.2.1. Solution. (a) We can plug any number into the cosine function, and itwill return a number in [−1, 1]. The domain of arcsinx is [−1, 1], so any numberwe plug into cosine will give us a valid number to plug into arcsine. So, thedomain of f(x) is all real numbers.(b) We can plug any number into the cosine function, and it will return a numberin [−1, 1]. The domain of arccscx is (−∞,−1] ∪ [1,∞), so in order to have a validnumber to plug into arccosecant, we need cosx = ±1. That is, the domain of g(x)is all values x = nπ for some integer n.(c) The domain of arccosine is [−1, 1]. The domain of sine is all real numbers, sono matter what number arccosine spits out, we can safely plug it into sine. So,the domain of h(x) is [−1, 1].

2.12.2.2. Solution. False: cos t = 1 for infinitely many values of t; arccosinegives only the single value t = 0 for which cos t = 1 and 0 ≤ t ≤ π. The particledoes not start moving until t = 10, so t = 0 is not in the domain of the functiondescribing its motion.The particle will have height 1 at time 2πn, for any integer n ≥ 2.

2.12.2.3. Solution. First, we restrict the domain of f to force it to be one–to–one.There are many intervals we could choose over which f is one–to–one, but thequestion asks us to contain x = 0 and be as large as possible; this leaves us withthe following restricted function:

781

Page 798: CLP-1 Differential Calculus

SOLUTIONS TO EXERCISES

x

y

1

The inverse of a function swaps the role of the input and output; so if the graphof y = f(x) contains the point (a, b), then the graph of Y = f−1(X) contains thepoint (b, a). That is, the graph of Y = f−1(X) is the graph of y = f(x) withthe x-coordinates and y-coordinates swapped. (So, since y = f(x) crosses they-axis at y = 1, then Y = f−1(X) crosses the X-axis at X = 1.) This swapping isequivalent to reflecting the curve y = f(x) over the line y = x.

x

y

1

1

y = f(x)

y = f−1(x)

y = x

Remark: while you’re getting accustomed to inverse functions, it is sometimesclearer to consider y = f(x) and Y = f−1(X): using slightly different notations

782

Page 799: CLP-1 Differential Calculus

SOLUTIONS TO EXERCISES

for x (the input of f , hence the output of f−1) and X (the input of f−1, whichcomes from the output of f ). However, the convention is to use x for the inputsof both functions, and y as the outputs of both functions, as is written on thegraph above.

2.12.2.4. Solution. The tangent line is horizontal when 0 = y′ = a − sinx. Thatis, when a = sinx.

• If |a| > 1, then there is no value of x for which a = sinx, so the curve has nohorizontal tangent lines.

• If |a| = 1, then there are infinitely many solutions to a = sinx, but only onesolution in the interval [−π, π]: x = arcsin(a) = arcsin(±1) = ±π

2. Then the

values of x for which a = sinx are x = 2πn+ aπ2

for any integer n.

• If |a| < 1, then there are infinitely many solutions to a = sinx. The solutionin the interval

(−π

2, π

2

)is given by x = arcsin(a). The other solution in the

interval (−π, π) is given by x = π − arcsin(a), as shown in the unit circlesbelow.

x

y

a

arcsin(a)

783

Page 800: CLP-1 Differential Calculus

SOLUTIONS TO EXERCISES

x

y

a

arcsin(a)

π−

arcs

in(a

)So, the values of x for which x = sin a are x = 2πn + arcsin(a) and x =2πn+ π − arcsin(a) for any integer n.

Remark: when a = 1, then

2πn+ arcsin(a) = 2πn+π

2= 2πn+ π −

(π2

)= 2πn+ π − arcsin(a).

Similarly, when a = −1,

2πn+ arcsin(a) = 2πn− π

2= 2π(n− 1) + π −

(−π

2

)= 2π(n− 1) + π − arcsin(a)

So, if we try to use the descriptions in the third bullet point to describe pointswhere the tangent line is horizontal when |a| = 1, we get the correct points buteach point is listed twice. This is why we separated the case |a| = 1 from the case|a| < 1.

2.12.2.5. Solution. The function arcsinx is only defined for |x| ≤ 1, and thefunction arccscx is only defined for |x| ≥ 1, so f(x) has domain |x| = 1. That is,x = ±1.In order for f(x) to be differentiable at a point, it must exist in an open intervalaround that point. (See Definition 2.2.1.) Since our function does not exist overany open interval, f(x) is not differentiable anywhere.So, actually, f(x) is a pretty boring function, which we can entirely describe as:f(−1) = −π and f(1) = π.

Exercises — Stage 2

784

Page 801: CLP-1 Differential Calculus

SOLUTIONS TO EXERCISES

2.12.2.6. Solution. Using the chain rule,

d

dx

arcsin

(x3

)=

1√1−

(x3

)2· 1

3

=1

3√

1− x2

9

=1√

9− x2

Since the domain of arcsine is [−1, 1], and we are plugging inx

3to arcsine, the

values of x that we can plug in are those that satisfy −1 ≤ x

3≤ 1, or −3 ≤ x ≤ 3.

So the domain of f is [−3, 3].

2.12.2.7. Solution. Using the quotient rule,

d

dt

arccos t

t2 − 1

=

(t2 − 1)(−1√1−t2

)− (arccos t)(2t)

(t2 − 1)2

The domain of arccosine is [−1, 1], and since t2 − 1 is in the denominator, thedomain of f requires t2 − 1 6= 0, that is, t 6= ±1. So the domain of f(t) is (−1, 1).

2.12.2.8. Solution. The domain of arcsecx is |x| ≥ 1: that is, we can plug intoarcsecant only values with absolute value greater than or equal to one. Since−x2 − 2 ≤ −2, every real value of x gives us an acceptable value to plug intoarcsecant. So, the domain of f(x) is all real numbers.

To differentiate, we use the chain rule. Rememberd

dxarcsecx =

1

|x|√x2 − 1

.

d

dx

arcsec(−x2 − 2)

=

1

| − x2 − 2|√

(−x2 − 2)2 − 1· (−2x)

=−2x

(x2 + 2)√x4 + 4x+ 3

.

2.12.2.9. Solution. We use the chain rule, remembering that a is a constant.

d

dx

1

aarctan

(xa

)=

1

a· 1

1 +(xa

)2 ·1

a

=1

a2 + x2

The domain of arctangent is all real numbers, so the domain of f(x) is also allreal numbers.

785

Page 802: CLP-1 Differential Calculus

SOLUTIONS TO EXERCISES

2.12.2.10. Solution. We differentiate using the product and chain rules.

d

dx

x arcsinx+

√1− x2

= arcsinx+

x√1− x2

+−2x

2√

1− x2

= arcsinx

The domain of arcsinx is [−1, 1], and the domain of√

1− x2 is all values of x sothat 1− x2 ≥ 0, so x in [−1, 1]. Therefore, the domain of f(x) is [−1, 1].

2.12.2.11. Solution. We differentiate using the chain rule:

d

dxarctan(x2) =

2x

1 + x4

This is zero exactly when x = 0.

2.12.2.12. Solution. Using formulas you should memorize from this section,

d

dxarcsinx+ arccosx =

1√1− x2

+−1√

1− x2= 0

Remark: the only functions with derivative equal to zero everywhere are con-stant functions, so arcsinx + arccosx should be a constant. Since sin θ =cos(π2− θ), we can set

sin θ = x cos(π

2− θ)

= x

where x and θ are the same in both expressions, and −π2≤ θ ≤ π

2. Then

arcsinx = θ arccosx =π

2− θ

We note here that arcsine is the inverse of the sine function restricted to[−π

2, π

2

].

So, since we restricted θ to this domain, sin θ = x really does imply arcsinx = θ.(For an example of why this matters, note sin(2π) = 0, but arcsin(0) = 0 6= 2π.)Similarly, arccosine is the inverse of the cosine function restricted to [0, π]. Since−π

2≤ θ ≤ π

2, then 0 ≤ (π

2− θ) ≤ π, so cos

(π2− θ)

= x really does imply arccosx =π2− θ.

So,arcsinx+ arccosx = θ +

π

2− θ =

π

2

which means the derivative we were calculating was actually justd

dx

π2

= 0.

2.12.2.13. *. Solution. Using the chain rule,

y′ =− 1x2√

1−(

1x

)2=

−1

x2

√1− 1

x2

.

786

Page 803: CLP-1 Differential Calculus

SOLUTIONS TO EXERCISES

2.12.2.14. *. Solution. Using the chain rule,

y′ =− 1x2

1 +(

1x

)2 =−1

x2 + 1.

2.12.2.15. *. Solution. Using the product rule:

d

dx

(1 + x2) arctanx

= 2x arctanx+ (1 + x2)

1

1 + x2

= 2x arctanx+ 1

2.12.2.16. Solution. Let θ = arctanx. Then θ is the angle of a right triangle thatgives tan θ = x. In particular, the ratio of the opposite side to the adjacent side isx. So, we have a triangle that looks like this:

θ

x

1

√x2 + 1

where the length of the hypotenuse came from the Pythagorean Theorem. Now,

sin (arctanx) = sin θ =opphyp

=x√x2 + 1

From here, we differentiate using the quotient rule:

d

dx

x√x2 + 1

=

√x2 + 1− x 2x

2√x2+1

x2 + 1

=

(√x2 + 1− x2√

x2+1

x2 + 1

)·√x2 + 1√x2 + 1

=(x2 + 1)− x2

(x2 + 1)3/2

=1

(x2 + 1)3/2= (x2 + 1)−3/2

Remark: another strategy is to differentiate first, using the chain rule, then draw

a triangle to simplify the resulting expressiond

dxsin (arctanx) =

cos(arctanx)

1 + x2.

2.12.2.17. Solution. Let θ = arcsinx. Then θ is the angle of a right triangle thatgives sin θ = x. In particular, the ratio of the opposite side to the hypotenuse is x.So, we have a triangle that looks like this:

787

Page 804: CLP-1 Differential Calculus

SOLUTIONS TO EXERCISES

θ

x

√1− x2

1

where the length of the adjacent side came from the Pythagorean Theorem. Now,

cot (arcsinx) = cot θ =adjopp

=

√1− x2

x

From here, we differentiate using the quotient rule:

d

dx

√1− x2

x

=x −2x

2√

1−x2 −√

1− x2

x2

=−x2 − (1− x2)

x2√

1− x2

=−1

x2√

1− x2

Remark: another strategy is to differentiate first, using the chain rule, then

draw a triangle to simplify the resulting expressiond

dxcot (arcsinx) =

− csc2(arcsinx)√1− x2

.

2.12.2.18. *. Solution. The line y = 2x+9 has slope 2, so we must find all valuesof x between −1 and 1 (arcsinx is only defined for these values of x) for whichd

dxarcsinx = 2. Evaluating the derivative:

y = arcsinx

2 = y′ =1√

1− x2

4 =1

1− x2

1

4= 1− x2

x2 =3

4

x = ±√

3

2

(x, y) = ±(√3

2,π

3

)788

Page 805: CLP-1 Differential Calculus

SOLUTIONS TO EXERCISES

2.12.2.19. Solution. We differentiate using the chain rule:

d

dxarctan(cscx) =

1

1 + csc2 x· d

dxcscx

=− cscx cotx

1 + csc2 x

=− 1

sinx· cosx

sinx

1 +(

1sinx

)2

=− cosx

sin2 x+ 1

So if f ′(x) = 0, then cosx = 0, and this happens when x =(2n+ 1)π

2for any

integer n. We should check that these points are in the domain of f . Arctangentis defined for all real numbers, so we only need to check the domain of cosecant;

when x =(2n+ 1)π

2, then sinx = ±1 6= 0, so cscx =

1

sinxexists.

Exercises — Stage 32.12.2.20. *. Solution. Since g(y) = f−1(y),

f(g(y)) = f(f−1(y)

)= y

Now, we can differentiate with respect to y using the chain rule.

d

dyf(g(y)) =

d

dyy

f ′(g(y)) · g′(y) = 1

g′(y) =1

f ′(g(y))=

1

1− sin g(y)

2.12.2.21. *. Solution. Write g(y) = f−1(y). Then g(f(x)) = x, so differentiatingboth sides (using the chain rule), we see

g′(f(x)) · f ′(x) = 1

What we want is g′(π−1), so we need to figure out which value of x gives f(x) =π − 1. A little trial and error leads us to x = π

2.

g′(π − 1) · f ′(π

2

)= 1

Since f ′(x) = 2− cos(x), f ′(π2

)= 2− 0 = 2:

g′(π − 1) · 2 = 1

g′(π − 1) =1

2

789

Page 806: CLP-1 Differential Calculus

SOLUTIONS TO EXERCISES

2.12.2.22. *. Solution. Write g(y) = f−1(y). Then g(f(x)) = x, so differentiatingboth sides (using the chain rule), we see

g′(f(x))f ′(x) = 1

What we want is g′(e+ 1), so we need to figure out which value of x gives f(x) =e+ 1. A little trial and error leads us to x = 1.

g′(f(1))f ′(1) = 1

g′(e+ 1) · f ′(1) = 1

g′(e+ 1) =1

f ′(1)

It remains only to note that f ′(x) = ex + 1, so f ′(1) = e+ 1

g′(e+ 1) =1

e+ 1

2.12.2.23. Solution. We use logarithmic differentiation, our standard method ofdifferentiating an expression of the form (function)function.

f(x) = [sin x+ 2]arcsecx

log(f(x)) = arcsec x · log[sinx+ 2]

f ′(x)

f(x)=

1

|x|√x2 − 1

log[sinx+ 2] + arcsec x · cosx

sinx+ 2

f ′(x) = [sin x+ 2]arcsecx

(log[sinx+ 2]

|x|√x2 − 1

+arcsecx · cosx

sinx+ 2

)The domain of arcsecx is |x| ≥ 1. For any x, sinx + 2 is positive, and a positivenumber can be raised to any power. (Recall negative numbers cannot be raisedto any power–for example, (−1)1/2 =

√−1 is not a real number.) So, the domain

of f(x) is |x| ≥ 1.

2.12.2.24. Solution. The function1√

x2 − 1exists only for those values of x with

x2 − 1 > 0: that is, the domain of1√

x2 − 1is |x| > 1. However, the domain of

arcsine is |x| ≤ 1. So, there is not one single value of xwhere arcsinx and1√

x2 − 1are both defined.If the derivative of arcsin(x) were given by

1√x2 − 1

, then the derivative of

arcsin(x) would not exist anywhere, so we would probably just write “deriva-tive does not exist,” instead of making up a function with a mismatched domain.Also, the function f(x) = arcsin(x) is a smooth curve–its derivative exists at ev-ery point strictly inside its domain. (Remember not all curves are like this: forinstance, g(x) = |x| does not have a derivative at x = 0, but x = 0 is strictly inside

790

Page 807: CLP-1 Differential Calculus

SOLUTIONS TO EXERCISES

its domain.) So, it’s a pretty good bet that the derivative of arcsine is not1√

x2 − 1.

2.12.2.25. Solution. This limit represents the derivative computed at x = 1of the function f(x) = arctanx. To see this, simply use the definition of thederivative at a = 1:

d

dxf(x)

∣∣∣∣a

= limx→a

f(x)− f(a)

x− ad

dxarctanx

∣∣∣∣1

= limx→1

arctanx− arctan 1

x− 1

= limx→1

arctanx− π4

x− 1

= limx→1

((x− 1)−1

(arctanx− π

4

)).

Since the derivative of f(x) is1

1 + x2, its value at x = 1 is exactly

1

2.

2.12.2.26. Solution. First, let’s interpret the given information: when the input

of our function is 2x + 1 for some x, then its output is5x− 9

3x+ 7, for that same x.

We’re asked to evaluate f−1(7), which is the number y with the property thatf(y) = 7. If the output of our function is 7, that means

7 =5x− 9

3x+ 7

and so

7(3x+ 7) = 5x− 9

x = −29

8

So, when x = −29

8, our equation f(2x+ 1) =

5x− 9

3x+ 7becomes:

f

(2 · −29

8+ 1

)=

5 · −298− 9

3 · −298

+ 7

Or, equivalently:

f

(−25

4

)= 7

Therefore, f−1(7) = −25

4.

2.12.2.27. Solution. If f−1(y) = 0, that means f(0) = y. So, we want to find outwhat we plug into f−1 to get 0. Since we only know f−1 in terms of a variable x,

791

Page 808: CLP-1 Differential Calculus

SOLUTIONS TO EXERCISES

let’s figure out what x gives us an output of 0:

2x+ 3

x+ 1= 0

2x+ 3 = 0

x = −3

2

Now, the equation f−1(4x− 1) =2x+ 3

x+ 1with x =

−3

2tells us:

f−1

(4 · −3

2− 1

)=

2 · −32

+ 3−32

+ 1

Or, equivalently:

f−1(−7) = 0

Therefore, f(0) = −7.

2.12.2.28. Solution.

• Solution 1: We begin by differentiating implicitly. Following the usual con-vention, we use y′ to mean y′(x). We start with

arcsin(x+ 2y) = x2 + y2

Using the chain rule,

1 + 2y′√1− (x+ 2y)2

= 2x+ 2yy′

1√1− (x+ 2y)2

+2y′√

1− (x+ 2y)2= 2x+ 2yy′

2y′√1− (x+ 2y)2

− 2yy′ = 2x− 1√1− (x+ 2y)2

y′(

2√1− (x+ 2y)2

− 2y

)= 2x− 1√

1− (x+ 2y)2

Finally, solving for y′ gives

y′ =2x− 1√

1−(x+2y)2

2√1−(x+2y)2

− 2y

(√1− (x+ 2y)2√1− (x+ 2y)2

)

y′ =2x√

1− (x+ 2y)2 − 1

2− 2y√

1− (x+ 2y)2

• Solution 2: We begin by taking the sine of both sides of the equation.

arcsin(x+ 2y) = x2 + y2

792

Page 809: CLP-1 Differential Calculus

SOLUTIONS TO EXERCISES

x+ 2y = sin(x2 + y2)

Now, we differentiate implicitly.

1 + 2y′ = cos(x2 + y2) · (2x+ 2yy′)

1 + 2y′ = 2x cos(x2 + y2) + 2yy′ cos(x2 + y2)

2y′ − 2yy′ cos(x2 + y2) = 2x cos(x2 + y2)− 1

y′(2− 2y cos(x2 + y2)

)= 2x cos(x2 + y2)− 1

y′ =2x cos(x2 + y2)− 1

2− 2y cos(x2 + y2)

• We used two different methods, and got two answers that look pretty dif-ferent. However, the answers ought to be equivalent. To see this, we re-member that for all values of x and y that we care about (those pairs (x, y)in the domain of our curve), the equality

arcsin(x+ 2y) = x2 + y2

holds. Drawing a triangle:

x2 + y

2

x+ 2y1

√1− (x+ 2y)2

where the adjacent side (in red) come from the Pythagorean Theorem.Then, cos(x2 + y2) =

√1− (x+ 2y)2, so using our second solution:

y′ =2x cos(x2 + y2)− 1

2− 2y cos(x2 + y2)

=2x√

1− (x+ 2y)2 − 1

2− 2y√

1− (x+ 2y)2

which is exactly the answer from our first solution.

793

Page 810: CLP-1 Differential Calculus

SOLUTIONS TO EXERCISES

2.13 · The Mean Value Theorem2.13.5 · Exercises

Exercises — Stage 12.13.5.1. Solution. We know the top speed of the caribou, so we can use this togive the minimum possible number of hours the caribou spent travelling during

its migration. If the caribou travels at 70kph, it will take (5000km)

(1hr

70km

)≈

71.4hrs to travel 5000 kilometres. Probably the caribou wasn’t sprinting the wholetime, so probably it took it longer than that, but we can only say for sure that thecaribou spent at least about 71.4 hours migrating.

2.13.5.2. Solution. If f(x) is the position of the crane at time x, measuredin hours, then (if we let x = 0 be the beginning of the day) we know thatf(24) − f(0) = 240. Since f(x) is the position of the bird, f(x) is contin-uous and differentiable. So, the MVT says there is a c in (0, 24) such that

f ′(x) =f(24)− f(0)

24− 0=

240

24= 10. That is, at some point c during the day, the

speed of the crane was exactly 10 kph.

2.13.5.3. Solution. The MVT guarantees there is some point c strictly betweena and b where the tangent line to f(x) at x = c has the same slope as the secantline of f(x) from x = a to x = b. So, let’s start by drawing in the secant line.

x

y

a b

What we’re looking for is a point on the curve where the tangent line is parallelto this secant line. In fact, there are two.

794

Page 811: CLP-1 Differential Calculus

SOLUTIONS TO EXERCISES

x

y

a b

So, either of the two values c1 and c2 marked below can serve as the point guar-anteed by the MVT:

x

y

a bc1 c2

2.13.5.4. Solution. Since f(x) is differentiable for all x ∈ (0, 10), then f(x) isalso continuous for all x ∈ (0, 10). If f(x) were continuous on the closed interval

[0, 10], then the MVT would guarantee f ′(x) =f(10)− f(0)

10− 0= 1 for some c ∈

(0, 10); however, this is not the case. So, it must be that f(x) is continuous for allx ∈ (0, 10), but not for all x ∈ [0, 10].Since f ′(c) = 0 for c ∈ (0, 10), that means f is constant on that interval. So, f(x)is a function like this:

795

Page 812: CLP-1 Differential Calculus

SOLUTIONS TO EXERCISES

x

y

where the height of the constant function can be anything.

So, one possible answer is f(x) =

0 x 6= 10

10 x = 10.

2.13.5.5. Solution. (a) No such function is possible: Rolle’s Theorem guaranteesf ′(c) = 0 for at least one point c ∈ (1, 2).For the other functions, examples are below, but many answers are possible.

x

y

1 2

(b)

x

y

1 2

(c)

796

Page 813: CLP-1 Differential Calculus

SOLUTIONS TO EXERCISES

x

y

1 2

(d)

2.13.5.6. Solution. The function f(x) is continuous over all real numbers, but itis only differentiable when x 6= 0. So, if we want to apply the MVT, our intervalmust consist of only positive numbers or only negative numbers: the interval(−4, 13) is not valid.It is possible to use the mean value theorem to prove what we want: if a = 1and b = 144, then f(x) is differentiable over the interval (1, 144) (since 0 is notcontained in that interval), and f(x) is continuous everywhere, so by the mean

value theorem there exists some point c where f ′(x) =

√|144| −

√|1|

144− 1=

11

143=

1

13.

That being said, an easier way to prove that a point exists is to simply find

it–without using the MVT. When x > 0, f(x) =√x, so f ′(x) =

1

2√x

. Then

f ′(

169

4

)=

1

13.

Exercises — Stage 22.13.5.7. *. Solution. We note that f(0) = f(2π) = 0. Then using the MeanValue Theorem (note that the function is differentiable for all real numbers), weconclude that there exists c in (0, 2π) such that

f ′(c) =f(2π)− f(0)

2π − 0= 0.

2.13.5.8. *. Solution. We note that f(0) = f(1) = 0. Then using the Mean ValueTheorem (note that the function is differentiable for all real numbers), we get thatthere exists c ∈ (0, 1) such that

f ′(c) =f(1)− f(0)

1− 0= 0.

2.13.5.9. *. Solution. We note that f(0) = f(2π) =√

3 + π2. Then using theMean Value Theorem (note that the function is differentiable for all real numbers

797

Page 814: CLP-1 Differential Calculus

SOLUTIONS TO EXERCISES

since 3 + sin x > 0), we get that there exists c ∈ (0, 2π) such that

f ′(c) =f(2π)− f(0)

2π − 0= 0.

2.13.5.10. *. Solution. We note that f(0) = 0 and f(π/4) = 0. Then using theMean Value Theorem (note that the function is differentiable for all real num-bers), we get that there exists c ∈ (0, π/4) such that

f ′(c) =f(π/4)− f(0)

π/4− 0= 0.

2.13.5.11. Solution. By inspection, we see x = 0 is a root of f(x). The questionnow is whether there could possibly be other roots. Since f(x) is differentiableover all real numbers, if there is another root a, then by Rolle’s Theorem, f ′(c) = 0for some c strictly between 0 and a. However, f ′(x) = 3− cosx is never zero. So,there is no second root: f(x) has precisely one root.

2.13.5.12. Solution. The function f(x) is continuous and differentiable over allreal numbers. If a and b are distinct roots of f(x), then f ′(c) = 0 for some c strictlybetween a and b (Rolle’s Theorem). So, let’s think about f ′(x).

f ′(x) = (4x+ 1)3 + 1

This is simple enough that we can find its zero explicitly:

(4x+ 1)3 + 1 = 0 ⇔ (4x+ 1)3 = −1 ⇔ 4x+ 1 = −1

⇔ 4x = −2

Hence f ′(c) = 0 only when c =−1

2. That the derivative only has a single zero

is very useful. It means (via Rolle’s theorem) that if f(x) has distinct roots a

and b with a < b, then we must have a <−1

2< b. This also means that f(x)

cannot have 3 distinct roots a, b and (say) q with a < b < q, because then Rolle’stheorem would imply that f ′(x) would have two zeros — one between a and band another between b and q.We’ve learned that f(x) has at most two roots, and we’ve learned somethingabout where those roots can exist, if there are indeed two of them. But that meansf(x) could have 0, 1, or 2 roots.It’s not easy to find a root of f(x) by inspection. But we can get a good enoughpicture of the graph of y = f(x) to tell exactly how many roots there are, just byexploiting the following properties of f(x) and f ′(x).

• As x tends to ±∞, f(x) tends to +∞.

• The derivative f ′(x) = (4x+1)3 +1 is negative for x < −12

and is positive forx > −1

2. That is, f(x) is decreasing for x < −1

2and increasing for x > −1

2.

• f(−1

2

)= 1

16− 1

2< 0.

798

Page 815: CLP-1 Differential Calculus

SOLUTIONS TO EXERCISES

This means the function must look something like the graph below:

x

y

except we are unsure of the locations of the x-intercepts. At present we onlyknow that one is to the left of x = 1/2 and one is to the right.Note what happens to f(x) as x increases from strongly negative values tostrongly positive values.

• When x is large and negative, f(x) > 0.

• As x increases, f(x) decreases continuously until x = −12, where f(x) < 0.

In particular, since f(−1) = 6516> 0 and f

(− 1

2

)< 0 and f(x) is continuous,

the intermediate value theorem guarantees that f(x) takes the value zerofor some x between−1

2and−1. More descriptively put, as x increases from

hugely negative numbers to −12, f(x) passes through zero exactly once.

• As x increases beyond −12, f(x) increases continuously, starting negative

and becoming very large and positive when x becomes large and positive.In particular, since f(0) = 1

16> 0 and f

(− 1

2

)< 0 and f(x) is continuous,

the intermediate value theorem guarantees that f(x) takes the value zerofor some x between −1

2and 0. So, as x increases from −1

2to near +∞, f(x)

again passes through zero exactly once.

So f(x) must have exactly two roots, one with x < −12

and one with x > −12.

2.13.5.13. Solution.

• We can see by inspection that f(0) = 0, so there is at least one root.

• To hunt for other roots, notice than when f(x) = 0, x3 = sin (x5), and sincethe sine function only takes on values in [−1, 1], this means −1 ≤ x3 ≤ 1.That is, all roots of f(x) are in the interval [−1, 1]. (An equivalent way tosee this is to notice that for all x > 1, f(x) > 0, and for all x < 1, f(x) < 0.)

• If there is a root a 6= 0, then by Rolle’s Theorem (since f(x) is continuousand differentiable for all real numbers) f ′(c) = 0 for some c strictly between0 and a. In particular, since we already know any roots a will be between−1 and 1, if f(x) has two roots then f ′(c) = 0 for some c ∈ (−1, 0) ∪ (0, 1).

•f ′(x) = 3x2 + 5x4 cos

(x5)

= x2(3 + 5x2 cos

(x5))

799

Page 816: CLP-1 Differential Calculus

SOLUTIONS TO EXERCISES

So, if f ′(x) = 0, then x = 0 or (3 + 5x2 cos (x5)) = 0. If x ∈ (−1, 0) ∪ (0, 1),then |x| < 1 and so |x5| < 1 < π

2, so cos (x5) > 0. Then 3 + 5x2 cos (x5) > 3,

so f ′(x) 6= 0. That is, there is no c ∈ (−1, 0)∪ (0, 1) with f ′(c) = 0. Therefore,following our last bullet point, f(x) has only one root.

Note here that f ′(x) has many zeroes–infinitely many, in fact. However, x = 0 isthe only root of f ′(x) in the interval (−1, 1).

2.13.5.14. Solution. We are to find the number of positive solutions to theequation ex = 4 cos(2x). The figure below contains the graphs of y = ex andy = 4 cos(2x). The solutions to ex = 4 cos(2x) are precisely the x’s where y = ex

and y = 4 cos(2x) cross.

It sure looks like there is exactly one crossing with x ≥ 0 and that one crossing issomewhere between x = 0 and x = 1. Indeed since

[ex − 4 cos(2x)

]x=0

= −3 < 0

and[ex − 4 cos(2x)

]x=1

> e > 0 and f(x) = ex − 4 cos(2x) is continuous, theintermediate value theorem guarantees that there is at least one root with 0 <x < 1.We still have to show that there is no second root — even if our graphs are notaccurate.Recall that the range of the cosine function is [−1, 1]. If ex = 4 cos(2x), then ex ≤ 4,so x ≤ log(4) ≈ 1.39. So, we only need to search for roots of f(x) on the interval(0, 1.4): we are guaranteed there are no roots elsewhere. Over this interval, 2x ∈(0, 2.8), so sin(2x) > 0, and thus f ′(x) = ex + 8 sin(2x) > 0. Since f ′(x) has noroots in (0, 1.4), we conclude by Rolle’s Theorem that f(x) has at most one rootin (0, 1.4) (and so at most one positive root total). Since we’ve already foundthat a root of f(x) exists in (0, 1), we conclude ex = 4 cos(2x) has precisely onepositive-valued solution.

2.13.5.15. *. Solution. 2.13.5.15.a

f ′(x) = 15x4 − 30x2 + 15 = 15(x4 − 2x2 + 1

)= 15

(x2 − 1

)2 ≥ 0

The derivative is nonnegative everywhere. The only values of x for which f ′(x) =

800

Page 817: CLP-1 Differential Calculus

SOLUTIONS TO EXERCISES

0 are 1 and −1, so f ′(x) > 0 for every x in (−1, 1).2.13.5.15.b If f(x) has two roots a and b in [−1, 1], then by Rolle’s Theorem, f ′(c) =0 for some x strictly between a and b. But since a and b are in [−1, 1], and c isbetween a and b, that means c is in (−1, 1); however, we know for every c in(−1, 1), f ′(c) > 0, so this can’t happen. Therefore, f(x) does not have two roots aand b in [−1, 1]. This means f(x) has at most one root in [−1, 1].

2.13.5.16. *. Solution. Write f(x) = ex. Since f(x) is continuous and differen-tiable, the Mean Value Theorem asserts that there exists some c between 0 and Tsuch that

f ′(c) =f(T )− f(0)

T − 0

The problem asks us to find this value of c. Solving:

ec =eT − e0

T

ec =eT − 1

T

c = log

(eT − 1

T

)2.13.5.17. Solution. The domains of arcsecx and C−arccscx are the same: |x| ≥1. Define f(x) = arcsecx + arccscx, and note the domain of f(x) is also |x| ≥ 1.Using Theorem 2.12.8,

f ′(x) =d

dxarcsecx+

d

dxarccsc =

1

|x|√x2 − 1

+−1

|x|√x2 − 1

= 0.

By Corollary 2.13.12, this means that f(x) is constant on any interval in |x| ≥ 1.So f(x) is a constant, call it C+, on x ≥ 1, and f(x) is also a constant, call it C−, onx ≤ −1.In order to find C+, we find f(1), because we know angles for which the secantand cosecant are x = 1.

cos(0) = 1 =⇒ sec(0) = 11

= 1 =⇒ arcsec(1) = 0

sin(π2

)= 1 =⇒ csc

(π2

)= 1

1= 1 =⇒ arccsc(1) = π

2

SoC+ = arcsec(1) + arccsc(1) = π

2

In order to find C−, we find f(−1), because we know angles for which the secantand cosecant are x = −1.

cos(π) = −1 =⇒ sec(π) = 1−1

= −1 =⇒ arcsec(−1) = π

sin(− π

2

)= −1 =⇒ csc

(− π

2

)= 1−1

= −1 =⇒ arccsc(−1) = −π2

SoC− = arcsec(−1) + arccsc(−1) = π

2

801

Page 818: CLP-1 Differential Calculus

SOLUTIONS TO EXERCISES

This shows that f(x) = arcsec x + arccscx = π2

for all |x| ≥ 1 and arcsecx =π2− arccscx for all |x| ≥ 1.

Exercises — Stage 32.13.5.18. *. Solution. Since e−f(x) is always positive (regardless of the value off(x)),

f ′(x) =1

1 + e−f(x)<

1

1 + 0= 1

for every x.Since f ′(x) exists for every x, we see that f is differentiable, so the Mean ValueTheorem applies. If f(100) is greater than or equal to 100, then by the Mean ValueTheorem, there would have to be some c between 0 and 100 such that

f ′(c) =f(100)− f(0)

100≥ 100

100= 1

Since f ′(x) < 1 for every x, there is no value of c as described. Therefore, it is notpossible that f(100) ≥ 100. So, f(100) < 100.

2.13.5.19. Solution. If 2x + sin x is one–to–one over an interval, it never takesthe same value for two distinct numbers in that interval. By Rolle’s Theorem,if f(a) = f(b) for distinct a and b, then f ′(c) = 0 for some c between a and b.However, f ′(x) = 2+cosx, which is never zero. In fact, f ′(x) ≥ 1 for all x, so f(x)is strictly increasing over its entire domain. Therefore, our function f never takesthe same value twice, so it is one–to–one over all the real numbers, (−∞,∞).When we define the inverse function f−1(x), the domain of f is the range of f−1,and vice-versa. In general, we might have to restrict the domain of f (and hencethe range of f−1) to an interval where f is one–to–one, but in our case, this isn’tnecessary. So, the range of f−1 is (−∞,∞) and the domain of f−1 is the range off : (−∞,∞).

2.13.5.20. Solution. If f(x) =x

2+ sinx is one–to–one over an interval, it never

takes the same value for two distinct numbers in that interval. By Rolle’s Theo-rem, if f(a) = f(b) for distinct a and b, then f ′(c) = 0 for some c between a and b.Since f ′(x) = 1

2+ cosx, f ′(x) = 0 when x = 2nπ ± 2π

3for some integer n. So, in

particular, if a and b are distinct numbers in the interval[−2π

3, 2π

3

], then for every

c strictly between a and b, f ′(c) 6= 0, so by Rolle’s Theorem f(a) 6= f(b). Thereforef(x) is one–to–one on the interval

[−2π

3, 2π

3

].

We should also show that the interval[−2π

3, 2π

3

]cannot be extended to a larger

interval over which f(x) is still one–to–one. Consider the derivative f ′(x) =12

+ cosx. For all −2π3< x < 2π

3, we have cosx > −1

2(sketch the graph of cosx

yourself) so that f ′(x) > 0 and f(x) is increasing. But at x = 2π3

, f ′(x) = 0, andthen for x a bit bigger than 2π

3we have cosx < −1

2so that f ′(x) < 0 and f(x) is

decreasing. So the graph “reverses direction”, and f(x) repeats values. (See thegraph of y = f(x) below.) The same is true for x a little smaller than −2π

3.

802

Page 819: CLP-1 Differential Calculus

SOLUTIONS TO EXERCISES

x

y

y = f(x) =x

2+ sinx

2π3

−2π3

When we define the inverse function f−1(x), first we restrict f to[−2π

3, 2π

3

]. Then

the range of f−1 is also[−2π

3, 2π

3

]. The domain of f−1 is the range of f over this

interval, so[−(π3

+√

32

),(π3

+√

32

)].

2.13.5.21. Solution. Define h(x) = f(x)−g(x), and notice h(a) = f(a)−g(a) < 0and h(b) = f(b) − g(b) > 0. Since h is the difference of two functions that arecontinuous over [a, b] and differentiable over (a, b), also h is continuous over [a, b]and differentiable over (a, b). So, by the Mean Value Theorem, there exists somec ∈ (a, b) with

h′(c) =h(b)− h(a)

b− aSince (a, b) is an interval, b > a, so the denominator of the above expression ispositive; since h(b) > 0 > h(a), also the numerator of the above expression ispositive. So, h′(c) > 0 for some c ∈ (a, b). Since h′(c) = f ′(c)− g′(c), we concludef ′(c) > g′(c) for some c ∈ (a, b).

2.13.5.22. Solution. Since f(x) is differentiable over all real numbers, it is alsocontinuous over all real numbers. We claim that f(x) cannot have four or moredistinct roots. For every two distinct roots a < b, Rolle’s Theorem tells us thereis a c ∈ (a, b) such that f ′(c) = 0: that is, c is a root of f ′. Since f ′ has only twodistinct roots, f can have at most three distinct roots.

803

Page 820: CLP-1 Differential Calculus

SOLUTIONS TO EXERCISES

a1 a2 a3

roots of f(x)

c1 c2

roots of f ′(x)

2.13.5.23. Solution. We are asked to find the number of solutions to the equa-tion x2 + 5x+ 1 = − sinx. The figure below contains the graphs of y = x2 + 5x+ 1and y = − sinx. The solutions to x2 + 5x+ 1 = − sinx are precisely the x’s wherey = x2 + 5x+ 1 and y = − sinx cross.

x

y

y = x2 + 5x+ 1

y = 1

y = −1y = − sinx

From the figure, it sure looks like there are two crossings. Since the function− sinx has range [−1, 1], if the two functions cross, then also−1 ≤ x2 +5x+1 ≤ 1.This portion of the quadratic function is highlighted in blue in the figure. The xcoordinates of the end points of the blue arcs are found by solving x2 + 5x+ 1 =

±1, i.e. x = 0, −5, and (using the quadratic equation) x = −5±√

172

.We are now in a position to exploit the intuition that we have built using the

804

Page 821: CLP-1 Differential Calculus

SOLUTIONS TO EXERCISES

above figure to write a concise argument showing that f(x) has exactly two roots.Remember that, in general, if we want to show that a function has n roots, weneed to show that there exist n distinct roots somewhere, and that there do notexist n+ 1 distinct roots. This argument is given below, in blue text.

• f(x) is continuous over all real numbers

• f(x) = 0 only when − sinx = x2 + 5x + 1, which only happens when |x2 +

5x + 1| ≤ 1. Thus, f(x) only has roots in the intervals[−5, −5−

√17

2

]and[

−5+√

172

, 0].

• f(−5) = sin(−5) + 1 > 0, and f(−5−

√17

2

)= sin

(−5−

√17

2

)− 1 < 0. So, by the

IVT, f(c) = 0 for some c ∈(−5, −5−

√17

2

).

• f(0) = 1 > 0, and f(−5+

√17

2

)= sin

(−5+

√17

2

)− 1 < 0. So, by the IVT,

f(c) = 0 for some c ∈(−5+

√17

2, 0)

.

• f ′(x) = cos x+ 2x+ 5. If f ′(x) = 0, then 2x+ 5 = − cosx, so |2x+ 5| ≤ 1. So,the only interval that can contain roots of f ′(x) is [−3,−2].

• Suppose f(x) has more than two roots. Then it has two roots in the interval[−5, −5−

√17

2

]OR it has two roots in the interval

[−5+

√17

2, 0]. Since f(x) is

differentiable for all real numbers, Rolle’s Theorem tells us that f ′(x) has aroot in

(−5, −5−

√17

2

)or in

(−5+

√17

2, 0)

. However, since all roots of f ′(x) are

in the interval [−3,−2], and this interval shares no points with(−5, −5−

√17

2

)or(−5+

√17

2, 0)

, this cannot be the case. Therefore f(x) does not have morethan two roots.

−5 −5−√

172

−5+√

172

0

roots of f(x)

−3 −2

roots of f ′(x)

• Since f(x) has at least two roots, and not more than two roots, f(x) hasexactly two roots.

805

Page 822: CLP-1 Differential Calculus

SOLUTIONS TO EXERCISES

2.14 · Higher Order Derivatives2.14.2 · Exercises

Exercises — Stage 12.14.2.1. Solution. The derivative of ex is ex: taking derivatives leaves the func-tion unchanged, even if we do it 180 times. So f (180) = ex.

2.14.2.2. Solution. Since f ′(x) > 0 over (a, b), we know from Corollary 2.13.12that f(x) is increasing over (a, b), so 2.14.2.2.ii holds. Since f ′′(x) > 0, and f ′′(x)is the derivative of f ′(x), by the same reasoning we see that f ′(x) is increasing.Since f ′(x) is the rate at which f(x) is increasing, that means that the rate at whichf ′(x) is increasing is itself increasing: this is, 2.14.2.2.iv holds (and not 2.14.2.2.iii).There is no reason to think 2.14.2.2.i or 2.14.2.2.v holds, but to be thorough wewill give an example showing that they do not need to be true. If f(x) = x2 − 10and (a, b) = (0, 1), then f ′(x) = 2x > 0 over (0, 1), and f ′′(x) = 2 > 0 everywhere,but f(x) < 0 for all x ∈ (0, 1), so 2.14.2.2.i does not hold. Also, f ′′′(x) = 0everywhere, so 2.14.2.2.v does not hold either.

2.14.2.3. Solution. Every time we differentiate f(x), the constant out front getsmultiplied by an ever-decreasing constant, while the power decreases by one. As

in Example 2.14.2,d15

dx15ax15 = a · 15!. So, if a · 15! = 3, then a =

3

15!.

2.14.2.4. Solution. The derivativedy

dxis

11

4only at the point (1, 3): it is not

constantly11

4, so it is wrong to differentiate the constant

11

4to find

d2y

dx2. Below is

a correct solution.

−28x+ 2y + 2xy′ + 2yy′ = 0

Plugging in x = 1, y = 3:

−28 + 6 + 2y′ + 6y′ = 0

y′ =11

4at the point (1, 3)

Differentiating the equation −28x+ 2y + 2xy′ + 2yy′ = 0 :

−28 + 2y′ + 2y′ + 2xy′′ + 2y′y′ + 2yy′′ = 0

4y′ + 2(y′)2 + 2xy′′ + 2yy′′ = 28

At the point (1, 3), y′ =11

4. Plugging in:

4

(11

4

)+ 2

(11

4

)2

+ 2(1)y′′ + 2(3)y′′ = 28

y′′ =15

64

806

Page 823: CLP-1 Differential Calculus

SOLUTIONS TO EXERCISES

Exercises — Stage 22.14.2.5. Solution.

f(x) = x log x− x

f ′(x) = log x+ x · 1

x− 1

= log x

f ′′(x) =1

x

2.14.2.6. Solution.

d

dxarctanx =

1

1 + x2

d

dx

1

1 + x2

=

d

dx

(1 + x2)−1

= (−1)(1 + x2)−2(2x)

=−2x

(1 + x2)2

2.14.2.7. Solution. We use implicit differentiation, twice.

2x+ 2yy′ = 0

2 + (2y)y′′ + (2y′)y′ = 0

y′′ = −(y′)2 + 1

y

So, we need an expression for y′. We use the equation 2x + 2yy′ = 0 to concludey′ = −x

y:

y′′ = −

(−xy

)2

+ 1

y

= −x2

y2+ 1

y

= −x2 + y2

y3

= − 1

y3

2.14.2.8. Solution. The question asks for s′′(1). We start our differentiation us-ing the quotient rule:

s′(t) =et(t2 + 1)− et(2t)

(t2 + 1)2

807

Page 824: CLP-1 Differential Calculus

SOLUTIONS TO EXERCISES

=et(t2 − 2t+ 1)

(t2 + 1)2

Using the quotient rule again,

s′′(t) =(t2 + 1)2 d

dtet(t2 − 2t+ 1) − et(t2 − 2t+ 1) d

dt(t2 + 1)2

(t2 + 1)4

=(t2+1)2 · [et(2t−2) + et(t2−2+1)]− et(t2−2t+1) · 2(t2+1)(2t)

(t2+1)4

=et(t2 + 1)2(t2 − 1)− 4tet(t− 1)2(t2 + 1)

(t2 + 1)4

so that

s′′(1) = 0

2.14.2.9. Solution. We differentiate using the chain rule.

d

dxlog(5x2 − 12) =

10x

5x2 − 12

Using the quotient rule:

d2

dx2log(5x2 − 12) =

d

dx

10x

5x2 − 12

=

(5x2 − 12)(10)− 10x(10x)

(5x2 − 12)2

=−10(5x2 + 12)

(5x2 − 12)2

Using the quotient rule one last time:

d3

dx3log(5x2 − 12) =

d

dx

−10(5x2 + 12)

(5x2 − 12)2

=

(5x2 − 12)2(−10)(10x) + 10(5x2 + 12)(2)(5x2 − 12)(10x)

(5x2 − 12)4

=(5x2 − 12)(−100x) + (200x)(5x2 + 12)

(5x2 − 12)3

=100x(−5x2 + 12 + 10x2 + 24)

(5x2 − 12)3

=100x(5x2 + 36)

(5x2 − 12)3

2.14.2.10. Solution. The velocity of the particle is given by h′(t) = sin t. Note0 < 1 < π, so h′(1) > 0–the particle is rising (moving in the positive direction, inthis case “up”). The acceleration of the particle is h′′(t) = cos t. Since 0 < 1 < π

2,

808

Page 825: CLP-1 Differential Calculus

SOLUTIONS TO EXERCISES

h′′(t) > 0, so h′(t) is increasing: the particle is moving up, and it’s doing so at anincreasing rate. So, the particle is speeding up.

2.14.2.11. Solution. For this problem, remember that velocity has a sign indi-cating direction, while speed does not.The velocity of the particle is given by h′(t) = 3t2 − 2t − 5. At t = 1, the velocityof the particle is−4, so the particle is moving downwards with a speed of 4 unitsper second. The acceleration of the particle is h′′(t) = 6t − 2, so when t = 1, theacceleration is (positive) 4 units per second per second. That means the velocity(currently−4 units per second) is becoming a bigger number–since the velocity isnegative, a bigger number is closer to zero, so the speed of the particle is gettingsmaller. (For instance, a velocity of−3 represents a slower motion than a velocityof −4.) So, the particle is slowing down at t = 1.

2.14.2.12. Solution.

x2 + x+ y = sin(xy)

We differentiate implicitly. For ease of notation, we write y′ fordy

dx.

2x+ 1 + y′ = cos(xy)(y + xy′)

We’re interested in y′′, so we implicitly differentiate again.

2 + y′′ = − sin(xy)(y + xy′)2 + cos(xy)(2y′ + xy′′)

We want to know what y′′ is when x = y = 0. Plugging these in yields thefollowing:

2 + y′′ = 2y′

So, we need to know what y′ is when x = y = 0. We can get this from the equation2x + 1 + y′ = cos(xy)(y + xy′), which becomes 1 + y′ = 0 when x = y = 0. So, atthe origin, y′ = −1, and

2 + y′′ = 2(−1)

y′′ = −4

Remark: a common mistake is to stop at the equation 2x+1+y′ = cos(xy)(y+xy′),

plug in x = y = 0, find y′ = −1, and decide y′′ =d

dx−1 = 0. This is due to a

slight sloppiness in the usual notation. When we wrote y′ = 1, what we meant is

that at the point (0, 0),dy

dx= −1. More properly written:

dy

dx

∣∣∣∣x=0,y=0

= −1. This is

not the same as saying y′ = 1 everywhere (in which case, indeed, y′′ would be 0everywhere).

809

Page 826: CLP-1 Differential Calculus

SOLUTIONS TO EXERCISES

2.14.2.13. Solution. For (a) and (b), notice the following:

d

dxsinx = cosx

d

dxcosx = − sinx

d

dx− sinx = − cosx

d

dx− cosx = sinx

d

dxsinx = cosx

The fourth derivative is sinx is sinx, and the fourth derivative of cosx is cosx, so(a) and (b) are true.

d

dxtanx = sec2 x

d

dxsec2 x = 2 sec x(secx tanx) = 2 sec2 x tanx

d

dx2 sec2 x tanx = (4 sec x · secx tanx) tanx+ 2 sec2 x sec2 x

= 4 sec2 x tan2 x+ 2 sec4 x

and

d

dx4 sec2 x tan2 x+ 2 sec4 x

= (8 sec x · secx tanx) tan2 x+ 4 sec2 x(2 tanx · sec2 x)

+ 8 sec3 x · secx tanx

= 8 sec2 x tan3 x+ 16 sec4 x tanx

So,d4

dx4tanx = 8 sec2 x tan3 x+16 sec4 x tanx. It certainly seems like this is not the

same as tanx, but remember that sometimes trig identities can fool you: tan2 x+1 = sec2 x, and so on. So, to be absolutely sure that these are not equal, we needto find a value of x so that the output of one is not the same as the output of theother. When x = π

4:

8 sec2 x tan3 x+ 16 sec4 x tanx = 8(√

2)2

(1)3 + 16(√

2)4

(1)

= 80 6= 1 = tanx

So, (c) is false.

Exercises — Stage 3

810

Page 827: CLP-1 Differential Calculus

SOLUTIONS TO EXERCISES

2.14.2.14. Solution. Since f ′(x) < 0, we need a decreasing function. This onlyapplies to (ii), (iii), and (v). Since f ′′(x) > 0, that means f ′(x) is increasing, so theslope of the function must be increasing. In (v), the slope is constant, so f ′′(x) = 0–therefore, it’s not (v). In (iii), the slope is decreasing, because near a the curve isquite flat (f ′(x) near zero) but near b the curve is very steeply decreasing (f ′(x)is a large negative number), so (iii) has a negative second derivative. By contrast,in (ii), the line starts out as steeply decreasing (f ′(x) is a strongly negative num-ber) and becomes flatter and flatter (f ′(x) nears 0), so f ′(x) is increasing–in otherwords, f ′′(x) > 0. So, (ii) is the only curve that has f ′(x) < 0 and f ′′(x) > 0.

2.14.2.15. Solution. We differentiate a few time to find the pattern.

d

dx2x = 2x log 2

d2

dx22x = 2x log 2 · log 2 = 2x(log 2)2

d3

dx32x = 2x(log 2)2 · log 2 = 2x(log 2)3

Every time we differentiate, we multiply the original function by another factorof log 2. So, the nth derivative is given by:

dn

dxn2x = 2x(log 2)n

2.14.2.16. Solution. We differentiate using the power rule.

df

dx= 3ax2 + 2bx+ c

d2f

dx2= 6ax+ 2b

d3f

dx3= 6a

d4f

dx4= 0

In the above work, remember that a, b, c, and d are all constants. Since they are

nonzero constants,d3f

dx3= 6a 6= 0. So, the fourth derivative is the first derivative

to be identically zero: n = 4.

2.14.2.17. *. Solution. 2.14.2.17.a Using the chain rule for f(x):

f ′(x) = (1 + 2x)ex+x2

f ′′(x) = (1 + 2x)(1 + 2x)ex+x2 + (2)ex+x2 = (4x2 + 4x+ 3)ex+x2

h′(x) = 1 + 3x

h′′(x) = 3

811

Page 828: CLP-1 Differential Calculus

SOLUTIONS TO EXERCISES

• 2.14.2.17.b f(0) = h(0) = 1; f ′(0) = h′(0) = 1; f ′′(0) = h′′(0) = 3

• 2.14.2.17.c f and h “start at the same place”, since f(0) = h(0). If it wereclear that f ′(x) were greater than h′(x) for x > 0, then we would know thatf grows faster than h, so we could conclude that f(x) > h(x), as desired.Unfortunately, it is not obvious whether (1+2x)ex+x2 is always greater than1 + 3x for positive x. So, we look to the second derivative. f ′(0) = h′(0),and f ′′(x) = (4x2 + 4x + 3)ex+x2 > 3ex+x2 > 3 = h′′(x) when x > 0. Sincef ′(0) = h′(0), and since f ′ grows faster than h′ for positive x, we concludef ′(x) > h′(x) for all positive x. Now we can conclude that (since f(0) = h(0)and f grows faster than h when x > 0) also f(x) > h(x) for all positive x.

2.14.2.18. *. Solution.

a We differentiate implicitly.

x3y(x) + y(x)3 = 10x

3x2y(x) + x3y′(x) + 3y(x)2y′(x) = 10

Subbing in x = 1 and y(1) = 2 gives

(3)(1)(2) + (1)y′(1) + (3)(4)y′(1) = 10

13y′(1) = 4

y′(1) =4

13

b From part 2.14.2.18.a, the slope of the curve at x = 1, y = 2 is4

13, so

the curve is increasing, but fairly slowly. The angle of the tangent line istan−1

(413

)≈ 17. We are also told that y′′(1) < 0. So the slope of the curve is

decreasing as x passes through 1. That is, the line is more steeply increas-ing to the left of x = 1, and its slope is decreasing (getting less steep, thenpossibly the slope even becomes negative) as we move past x = 1.

2.14.2.19. Solution. 2.14.2.19.a Using the product rule,

g′′(x) = [f ′(x) + f ′′(x)]ex + [f(x) + f ′(x)]ex

= [f(x) + 2f ′(x) + f ′′(x)]ex

812

Page 829: CLP-1 Differential Calculus

SOLUTIONS TO EXERCISES

2.14.2.19.b Using the product rule and our answer from 2.14.2.19.a,

g′′′(x) = [f ′(x) + 2f ′′(x) + f ′′′(x)]ex + [f(x) + 2f ′(x) + f ′′(x)]ex

= [f(x) + 3f ′(x) + 3f ′′(x) + f ′′′(x)]ex

2.14.2.19.c We notice that the coefficients of the derivatives of f correspond to theentries in the rows of Pascal’s Triangle.

11 1

1 121 13 3

1 14 46

Pascal’s Triangle

• In the first derivative of g, the coefficients of f and f ′ correspond to theentries in the second row of Pascal’s Triangle.

• In the second derivative of g, the coefficients of f , f ′, and f ′′ correspond tothe entries in the third row of Pascal’s Triangle.

• In the third derivative of g, the coefficients of f , f ′, f ′′, and f ′′′ correspondto the entries in the fourth row of Pascal’s Triangle.

• We guess that, in the fourth derivative of g, the coefficients of f , f ′, f ′′, f ′′′,and f (4) will correspond to the entries in the fifth row of Pascal’s Triangle.

That is, we guess

g(4)(x) = [f(x) + 4f ′(x) + 6f ′′(x) + 4f ′′′(x) + f (4)(x)]ex

This is verified by differentiating our answer from 2.14.2.19.a using the productrule:

g′′′(x) = [f(x) + 3f ′(x) + 3f ′′(x) + f ′′′(x)]ex

g(4)(x) = [f ′(x) + 3f ′′(x) + 3f ′′′(x) + f (4)(x)]ex

+ [f(x) + 3f ′(x) + 3f ′′(x) + f ′′′(x)]ex

= [f(x) + 4f ′(x) + 6f ′′(x) + 4f ′′′(x) + f (4)(x)]ex.

2.14.2.20. Solution. Since f(x) is differentiable over all real numbers, it is alsocontinuous over all real numbers. Similarly, f ′(x) is differentiable over all realnumbers, so it is also continuous over all real numbers, and so on for the first nderivatives of f(x).Rolle’s Theorem tells us that if a and b are distinct roots of a function g, then

813

Page 830: CLP-1 Differential Calculus

SOLUTIONS TO EXERCISES

g′(x) = 0 for some c in (a, b). That is, g′ has a root strictly between a and b.Expanding this idea, if g has m + 1 distinct roots, then g′ must have at least mdistinct roots, as in the sketch below.

a1 a2 a3 am+1

roots of g(x)

c1 c2 cm

roots of g′(x)

So, if f (n)(x) has only m roots, then f (n−1)(x) has at most m + 1 roots. Similarly,since f (n−1)(x) has at most m+ 1 roots, f (n−2)(x) has at most m+ 2 roots. Contin-uing in this way, we see f(x) = f (n−n)(x) has at most m+ n distinct roots.

2.14.2.21. Solution.

• Let’s begin by noticing that the domain of f(x) is (−1,∞).

• By inspection, f(0) = 0, so f(x) has at least one root.

• If x ∈ (−1, 0), then (x+1) is positive, log(x+1) is negative, sin(x) is negative,and−x2 is negative. Therefore, if x < 0 is in the domain of f , then f(x) < 0.So, f(x) has no negative roots. We focus our attention on the case x > 0.

• f ′(x) = 1−2x+log(x+1)+cos x. We would like to know how many positiveroots f ′(x) has, but it isn’t obvious. So, let’s differentiate again.

• f ′′(x) = −2 + 1x+1− sinx. When x > 0, 1

x+1< 1, so f ′′(x) < −1− sin(x) ≤ 0,

so f ′′(x) has no positive roots. Since f ′(x) is continuous and differentiableover (0,∞), and since f ′′(x) 6= 0 for all x ∈ (0,∞), by Rolle’s Theorem, f ′(x)has at most one root in [0,∞).

• Since f(x) is continuous and differentiable over [0,∞), and f ′(x) has atmost one root in (0,∞), by Rolle’s Theorem f(x) has at most two dis-tinct roots in [0,∞). (Otherwise, f(a) = f(b) = f(c) = 0 for some values0 ≤ a < b < c, so f ′(d) = f ′(e) = 0 for some d ∈ (a, b) and some e ∈ (b, c),but since f ′(x) has at most one root, this is impossible.)

• We know f(0) = 0, so the remaining question is whether or not f(x) hasa second root (which would have to be positive). As usual, we can show

814

Page 831: CLP-1 Differential Calculus

SOLUTIONS TO EXERCISES

another root exists using the intermediate value theorem. We see that forlarge values of x, f(x) is negative, for example:

f(4) = 5 log 5 + sin(4)− (4)2 < 5 log(e2) + 1− 16 = 11− 16 < 0

For positive values of x closer to zero, we hope to find a positive valueof f(x). However, it’s quite difficult to get a number c that obviouslygives f(c) > 0. It suffices to observe that f(0) = 0 and f ′(0) = 2 > 0.From the definition of the derivative, we can conclude f(x) > 0 for somex > 0. (If it is not true that f(x) > 0 for some x > 0, then f(x) ≤ 0 forall x > 0. The definition of the derivative tells us that [since f ′(0) exists]

f ′(0) = limh→0+

f(h)− f(0)

h= lim

h→0+

f(h)

h; the denominator is positive, so if

the numerator were always less than or equal to zero, the limit would beless than or equal to zero as well. However, the derivative is positive, sof(x) > 0 for some x > 0.) Therefore, f(x) has a second root, so f(x) hasprecisely two roots.

2.14.2.22. *. Solution. 2.14.2.22.a In order to make f(x) a little more tractable,

let’s change the format. Since |x| =

x x ≥ 0

−x x < 0, then:

f(x) =

−x2 x < 0

x2 x ≥ 0.

Now, we turn to the definition of the derivative to figure out whether f ′(0) exists.

f ′(0) = limh→0

f(0 + h)− f(0)

h= lim

h→0

f(h)− 0

h= lim

h→0

f(h)

hif it exists.

Since f looks different to the left and right of 0, in order to evaluate this limit, welook at the corresponding one-sided limits. Note that when h approaches 0 fromthe right, h > 0 so f(h) = h2. By contrast, when h approaches 0 from the left,h < 0 so f(h) = −h2.

limh→0+

f(h)

h= lim

h→0+

h2

h= lim

h→0+h = 0

limh→0−

f(h)

h= lim

h→0−

−h2

h= lim

h→0−−h = 0

Since both one-sided limits exist and are equal to 0,

limh→0

f(0 + h)− f(0)

h= 0

and so f is differentiable at x = 0 and f ′(0) = 0.2.14.2.22.b From 2.14.2.22.a, f ′(0) = 0 and

f(x) =

−x2 x < 0

x2 x ≥ 0.

815

Page 832: CLP-1 Differential Calculus

SOLUTIONS TO EXERCISES

So,

f ′(x) =

−2x x < 0

2x x ≥ 0.

Then, we know the second derivative of f everywhere except at x = 0:

f ′′(x) =

−2 x < 0

?? x = 0

2 x > 0.

So, whenever x 6= 0, f ′′(x) exists. To investigate the differentiability of f ′(x) whenx = 0, again we turn to the definition of a derivative. If

limh→0

f ′(0 + h)− f ′(0)

h

exists, then f ′′(0) exists.

limh→0

f ′(0 + h)− f ′(0)

h= lim

h→0

f ′(h)− 0

h= lim

h→0

f ′(h)

h

Since f(h) behaves differently when h is greater than or less than zero, we lookat the one-sided limits.

limh→0+

f ′(h)

h= lim

h→0+

2h

h= 2

limh→0−

f ′(h)

h= lim

h→0−

−2h

h= −2

Since the one-sided limits do not agree,

limh→0

f ′(0 + h)− f ′(0)

h= DNE

So, f ′′(0) does not exist. Now we have a complete picture of f ′′(x):

f ′′(x) =

−2 x < 0

DNE x = 0

2 x > 0.

3 · Applications of derivatives3.1 · Velocity and Acceleration3.1.2 · Exercises

Exercises — Stage 13.1.2.1. Solution. False. The acceleration of the ball is given by h′′(t) = −9.8.This is constant throughout its trajectory (and is due to gravity).Remark: the velocity of the ball at t = 2 is zero, since h′(2) = −9.8(2) + 19.6 = 0,

816

Page 833: CLP-1 Differential Calculus

SOLUTIONS TO EXERCISES

but the velocity is only zero for an instant. Since the velocity is changing, theacceleration is nonzero.

3.1.2.2. Solution. The acceleration is constant, which means the rate of changeof the velocity is constant. So, since it took 10 seconds for the velocity to increaseby 1 metre per second (from 1 m

sto 2 m

s), then it always takes 10 seconds for the

velocity to increase by 1 metre per second.So, it takes 10 seconds to accelerate from 2 m

sto 3 m

s. To accelerate from 3 m

sto

13 ms

(that is, to change its velocity by 10 metres per second), it takes 10×10 = 100seconds.

3.1.2.3. Solution. Let v(a) = s′(a) be the velocity of the particle. If s′′(a) >0, then v′(a) > 0 — so the velocity of the particle is increasing. However, thatdoes not mean that its speed (the absolute value of velocity) is increasing as well.For example, if a velocity is increasing from −4 kph to −3 kph, the speed isdecreasing from 4 kph to 3 kph. So, the statement is false in general.Contrast this to Question 3.1.2.4.

3.1.2.4. Solution. Since s′(a) > 0, |s′(a)| = s′(a): that is, the speed and velocityof the particle are the same. (This means the particle is moving in the positivedirection.) If s′′(a) > 0, then the velocity (and hence speed) of the particle isincreasing. So, the statement is true.

Exercises — Stage 23.1.2.5. Solution. From Example 3.1.2, we know that an object falling from reston the Earth is subject to the acceleration due to gravity, 9.8 m

s2. So, if h(t) is the

height of the flower pot t seconds after it rolls out the window, then h′′(t) = −9.8.(We make the acceleration negative, since the measure “height” has “up” as thepositive direction, while gravity pulls the pot in the negative direction, “down.”)Then h′(t) is a function whose derivative is the constant −9.8 and with h′(0) = 0(since the object fell, instead of being thrown up or down), so h′(t) = −9.8t.What we want to know is h′(t) at the time t when the pot hits the ground. Wedon’t know yet exactly what time that happens, so we go a little farther and findan expression for h(t). The function h(t) has derivative −9.8t and h(0) = 10, so(again following the ideas in Example 3.1.2)

h(t) =−9.8

2t2 + 10

Now, we can find the time when the pot hits the ground: it is the time whenh(t) = 0 (and t > 0).

0 =−9.8

2t2 + 10

9.8

2t2 = 10

t2 =20

9.8

817

Page 834: CLP-1 Differential Calculus

SOLUTIONS TO EXERCISES

t = +

√20

9.8≈ 1.4sec

The velocity of the pot at this time is

h′(√

20

9.8

)= −9.8

(√20

9.8

)= −√

20 · 9.8 = −14m

s

So, the pot is falling at 14 metres per second, just as it hits the ground.

3.1.2.6. Solution. (a)

• Let s(t) be the distance the stone has fallen t seconds after dropping it. Sincethe acceleration due to gravity is 9.8 m

s2, s′′(t) = 9.8. (We don’t make this

negative, because s(t) measures how far the stone has fallen, which meansthe positive direction in our coordinate system is “down”, which is exactlythe way gravity is pulling.)

• Then s′(t) has a constant derivative of 9.8, so s′(t) = 9.8t + c for some con-stant c. Notice s′(0) = c, so c is the velocity of the stone at the very instantyou dropped it, which is zero. Therefore, s′(t) = 9.8t.

• So, s(t) is a function with derivative 9.8t. It’s not too hard to figure outby guessing and checking that s(t) = 9.8

2t2 + d for some constant d. No-

tice s(0) = d, so d is the distance the rock has travelled at the instant youdropped it, which is zero. So, s(t) = 9.8

2t2 = 4.9t2.

Remark: this is exactly the formula found in Example 3.1.2. You may, ingeneral, use that formula without proof, but you need to know where itcomes from and be able to apply it in other circumstances where it mightbe slightly different–like part (b) below.

• The rock falls for x seconds, so the distance fallen is

4.9x2

Remark: this is a decent (if imperfect) way to figure out how deep a wellis, or how tall a cliff is, when you’re out and about. Drop a rock, square thetime, multiply by 5.

(b) We’ll go through a similar process as before.Again, let s(t) be the distance the rock has fallen t seconds after it is let go. Thens′′(t) = 9.8, so s′(t) = 9.8t + c. In this case, since the initial speed of the rock is 1metre per second, 1 = s′(0) = c, so s′(t) = 9.8t+ 1.Then, s(t) is a function whose derivative is 9.8t+1, so s(t) = 9.8

2t2 + t+d for some

constant d. Since 0 = s(0) = d, we see s(t) = 4.9t2 + t.So, if the rock falls for x seconds, the distance fallen is

4.9x2 + x

Remark: This means there is an error of x metres in your estimation of the depthof the well.

818

Page 835: CLP-1 Differential Calculus

SOLUTIONS TO EXERCISES

3.1.2.7. Solution. Let s(t) be the distance your keys have travelled since theyleft your hand. The rate at which they are travelling, s′(t), is decreasing by 0.25metres per second. That is, s′′(t) = −0.25. Therefore, s′(t) = −0.25t + c for someconstant c. Since c = s′(0) = 2, we see

s′(t) = 2− 0.25t

Then s(t) has 2− 0.25t as its derivative, so s(t) = 2t− 18t2 + d for some constant d.

At time t = 0, the keys have not yet gone anywhere, so 0 = s(0) = d. Therefore,

s(t) = 2t− 1

8t2

The keys reach your friend when s(t) = 2 and t > 0. That is:

2 = 2t− 1

8t2

0 =1

8t2 − 2t+ 2

t = 8± 4√

3

We need to figure out which of these values of t is really the time when the keysreach your friend. The keys travel this way from t = 0 to the time they reachyour friend. (Then s(t) no longer describes their motion.) So, we need to find thefirst value of t that is positive with s(t) = 2. Since 8 − 4

√3 > 0, this is the first

time s(t) = 2 and t > 0. So, the keys take

8− 4√

3 ≈ 1 second

to reach your friend.

3.1.2.8. Solution. We proceed with the technique of Example 3.1.3 in mind.Let v(t) be the velocity (in kph) of the car at time t, where t is measured in hoursand t = 0 is the instant the brakes are applied. Then v(0) = 100 and v′(t) =−50000. Since v′(t) is constant, v(t) is a line with slope −50000 and intercept(0, 100), so

v(t) = 100− 50000t

The car comes to a complete stop when v(t) = 0, which occurs at t = 10050000

= 1500

hours. This is a confusing measure, so we convert it to seconds:(1

500hrs

)(3600 sec

1hr

)= 7.2 sec

3.1.2.9. Solution. Suppose the deceleration provided by the brakes is d kmhr2

.Then if v(t) is the velocity of the car, v(t) = 120 − dt (at t = 0, the velocity is120, and it decreases by d kph per hour). The car stops when 0 = v(t), so t = 120

d

hours.Let s(t) be the distance the car has travelled t hours after applying the brakes.

819

Page 836: CLP-1 Differential Calculus

SOLUTIONS TO EXERCISES

Then s′(t) = v(t), so s(t) = 120t− d2t2 + c for some constant c. Since 0 = s(0) = c,

s(t) = 120t− d

2t2

The car needs to stop in 100 metres, which is 110

kilometres. We already foundthat the stopping time is t = 120

d. So:

1

10= s

(120

d

)1

10= 120

(120

d

)− d

2

(120

d

)2

Multiplying both sides by d:

d

10= 1202 − 1202

2d = 5 · 1202

So, the brakes need to apply 72 000 kph per hour of deceleration.

3.1.2.10. Solution. Since your deceleration is constant, your speed decreasessmoothly from 100 kph to 0 kph. So, one second before your stop, you only have17

of our speed left: you’re going 1007

kph.A less direct way to solve this problem is to note that v(t) = 100−dt is the velocityof car t hours after braking, if d is its deceleration. Since it stops in 7 seconds (or

73600

hours), 0 = v(

73600

)= 100− 7

3600d, so d = 360000

7. Then

v

(6

3600

)= 100−

(360000

7

)(6

3600

)= 100− 6

7· 100 =

100

7kph

3.1.2.11. Solution. If the acceleration was constant, then it was

17500 mph8.560

hr≈ 123500

miles

hr2

So, the velocity t hours from liftoff is

v(t) = 123500t

Therefore, the position of the shuttle t hours from liftoff (taking s(0) = 0 to be itsinitial position) is

s(t) =123500

2t2 = 61750t2

So, after 8.560

hours, the shuttle has travelled

s

(8.5

60

)= (61750)

(8.5

60

)2

≈ 1240 miles

or a little less than 2000 kilometres.

820

Page 837: CLP-1 Differential Calculus

SOLUTIONS TO EXERCISES

3.1.2.12. Solution. We know that the acceleration of the ball will be constant. Ifthe height of the ball is given by h(t) while it is in the air, h′′(t) = −9.8. (The neg-ative indicates that the velocity is decreasing: the ball starts at its largest velocity,moving in the positive direction, then the velocity decreases to zero and then toa negative number as the ball falls.) As in Example 3.1.2, we need a function h(t)with h′′(t) = −9.8. Since this is a constant, h′(t) is a line with slope −9.8, so it hasthe form

h′(t) = −9.8t+ a

for some constant a. Notice when t = 0, h′(0) = a, so in fact a is the initial velocityof the ball–the quantity we want to solve for.Again, as in Example 3.1.2, we need a function h(t) with h′(t) = −9.8t + a. Sucha function must have the form

h(t) = −4.9t2 + at+ b

for some constant b. You can find this by guessing and checking, or simply re-member it from the text. (In Section 4.1, you’ll learn more about figuring outwhich functions have a particular derivative.) Notice when t = 0, h(0) = b, so bis the initial height of the baseball, which is 0.So, h(t) = −4.9t2 + at = t(−4.9t + a). The baseball is at height zero when it ispitched (t = 0) and when it hits the ground (which we want to be t = 10). So,we want (−4.9)(10) + a = 0. That is, a = 49. So, the initial pitch should be at 49metres per second.Incidentally, this is on par with the fastest pitch in baseball, as recordedby Guiness World Records: http://www.guinnessworldrecords.com/

world-records/fastest-baseball-pitch-(male)

3.1.2.13. Solution. The acceleration of a falling object due to gravity is 9.8 me-tres per second squared. So, the object’s velocity t seconds after being droppedis

v(t) = 9.8tm

s

We want v(t) to be the speed of the peregrine’s dive, so we should convert thatto metres per second:

325km

hr·(

1000 m

1 km

)(1 hr

3600 sec

)=

1625

18

m

s

The stone will reach this velocity when

9.8t =1625

18⇒ t =

1625

18(9.8)

What is left to figure out is how far the stone will fall in this time. The positionof the stone s(t) has derivative 9.8t, so

s(t) = 4.9t2

821

Page 838: CLP-1 Differential Calculus

SOLUTIONS TO EXERCISES

if we take s(0) = 0. So, if the stone falls for 162518(9.8)

seconds, in that time it travels

s

(1625

18(9.8)

)= 4.9

(1625

18(9.8)

)2

≈ 416 m

So, you would have to drop a stone from about 416 metres for it to fall as fast asthe falcon.

3.1.2.14. Solution. Since gravity alone brings your cannon ball down, its accel-eration is a constant −9.8 m

s2. So, v(t) = v0 − 9.8t and thus its height is given by

s(t) = v0t− 4.9t2 (if we set s(0) = 0).We want to know what value of v0 makes the maximum height 100 metres. Themaximum height is reached when v(t) = 0, which is at time t = v0

9.8. So, we solve:

100 = s( v0

9.8

)100 = v0

( v0

9.8

)− 4.9

( v0

9.8

)2

100 =

(1

9.8− 4.9

9.82

)v2

0

100 =1

2 · 9.8v20

v20 = 1960

v0 =√

1960 ≈ 44m

s

where we choose the positive square root because v0 must be positive for thecannon ball to get off the ground.

3.1.2.15. Solution. The derivative of acceleration is constant, so the accelera-tion a(t) has the form mt + b. We know a(0) = −50000 and a

(3

3600

)= −60000

(where we note that 3 seconds is 33600

hours). So, the slope of a(t) is −60000+500003

3600

=

−12000000, which leads us to

a(t) = −50000− (12000000)t

where t is measured in hours.Since v′(t) = a(t) = −50000− (12000000)t, we see

v(t) =−12000000

2t2 − 50000t+ c = −6000000t2 − 50000t+ c

for some constant c. Since 120 = v(0) = c:

v(t) = −6000000t2 − 50000t+ 120

Then after three seconds of braking,

v

(3

3600

)= −6000000

(3

3600

)2

− 50000

(3

3600

)+ 120

822

Page 839: CLP-1 Differential Calculus

SOLUTIONS TO EXERCISES

= −25

6− 125

3+ 120

≈ 74.2 kph

Remark: When acceleration is constant, the position function is a quadratic func-tion, but we don’t want you to get the idea that position functions are alwaysquadratic functions–in the example you just did, it was the velocity function thatwas quadratic. Position, velocity, and acceleration functions don’t have to bepolynomial at all–it’s only in this section, where we’re dealing with the simplestcases, that they seem that way.

Exercises — Stage 33.1.2.16. Solution. Different forces are acting on you (1) after you jump butbefore you land on the trampoline, and (2) while you are falling into the trampo-line. In both instances, the acceleration is constant, so both height functions arequadratic, of the form a

2t2 + vt + h, where a is the acceleration, v is the velocity

when t = 0, and h is the initial height.

• Let’s consider (1) first, the time during your jump before your feet touchthe trampoline. Let t = 0 be the moment you jump, and let the rim of thetrampoline be height 0. Then, since your initial velocity was (positive) 1meter per second, your height is given by

h1(t) =−9.8

2t2 + t = t

(−9.8

2t+ 1

)Notice that, because your acceleration is working against your positive ve-locity, it has a negative sign.

• We’ll need to know your velocity when your feet first touch the trampolineon your fall. The time your feet first first touch the trampoline after yourjump is precisely when h1(t) = 0 and t > 0. That is, when t = 2

9.8. Now,

since h′(t) = −9.8t+1, h′(

29.8

)= −9.8

(2

9.8

)+1 = −1. So, you are descending

at a rate of 1 metre per second at the instant your feet touch the trampoline.

Remark: it is not only coincidence that this was your initial speed. Thinkabout the symmetries of parabolas, and conservation of energy.

• Now we need to think about your height as the trampoline is slowing yourfall. One thing to remember about our general equation a

2t2 + vt + h is

that v is the velocity when t = 0. But, you don’t hit the trampoline att = 0, you hit it at t = 2

9.8. In order to keep things simple, let’s use a

different time scale for this second part of your journey. Let’s let h2(T ) beyour height at time T , from the moment your feet touch the trampoline skin(T = 0) to the bottom of your fall. Now, we can use the fact that your initialvelocity is −1 metres per second (negative, since your height is decreasing)

823

Page 840: CLP-1 Differential Calculus

SOLUTIONS TO EXERCISES

and your acceleration is 4.9 metres per second per second (positive, sinceyour velocity is increasing from a negative number to zero):

h2(T ) =4.9

2T 2 − T

where still the height of the rim of the trampoline is taken to be zero.

Remark: if it seems very confusing that your free-falling acceleration is neg-ative, while your acceleration in the trampoline is positive, remember thatgravity is pushing you down, but the trampoline is pushing you up.

• How long were you falling in the trampoline? The equation h2(T ) tells youyour height only as long as the trampoline is slowing your fall. You reachthe bottom of your fall when your velocity is zero.

h′2(T ) = 4.9T − 1

so you reach the bottom of your fall at T = 14.9

. Be careful: this is 14.9

secondsafter you entered the trampoline, not after the peak of your fall, or after youjumped.

• The last piece of the puzzle is how long it took you to fall from the peak ofyour jump to the surface of the trampoline. We know the equation of yourmotion during that time: h1(t) = −9.8

2t2 + t. You reached the peak when

your velocity was zero:

h′1(t) = −9.8t+ 1 = 0 ⇒ t =1

9.8

So, you fell from your peak at t = 19.8

and reached the level of the trampolinerim at t = 2

1.98, which means the fall took 1

9.8seconds.

Remark: by the symmetry mentioned early, the time it took to fall from thepeak of your jump to the surface of the trampoline is the same as one-halfthe time from the moment you jumped off the rim to the moment you’reback on the surface of the trampoline.

• So, your time falling from the peak of your jump to its bottom was 19.8

+ 14.9≈

0.3 seconds.

3.1.2.17. Solution. Let v(t) be the velocity of the object. From the given infor-mation:

• v(0) is some value, call it v0,

• v(1) = 2v0 (since the speed doubled in the first second),

• v(2) = 2(2)v0 (since the speed doubled in the second second),

• v(3) = 2(2)(2)v0, and so on.

824

Page 841: CLP-1 Differential Calculus

SOLUTIONS TO EXERCISES

So, for general t:v(t) = 2tv(0)

To find its acceleration, we simply differentiate. Recalld

dx2x = 2x log 2, where

log denotes logarithm base e.

a(t) = 2tv0 log 2

Remark: we can also write a(t) = v(t) log 2. The acceleration doubles every sec-ond as well.

3.2 · Related Rates3.2.2 · Exercises

Exercises — Stage 13.2.2.1. Solution. We have an equation relating P and Q:

P = Q3

We differentiate implicitly with respect to a third variable, t:

dP

dt= 3Q2 · dQ

dt

If we know two of the three quantitiesdP

dt,Q, and

dQ

dt, then we can find the third.

Therefore, ii is a question we can solve. If we know P , then we also know Q (it’sjust the cube root of P ), so also we can solve iv. However, if we know neither

P nor Q, then we can’t finddP

dtbased only off

dQ

dt, and we can’t find

dQ

dtbased

only offdP

dt. So we can’t solve i or iii.

Exercises — Stage 23.2.2.2. *. Solution. Suppose that at time t, the point is at

(x(t), y(t)

). Then

x(t)2 + y(t)2 = 1 so that 2x(t)x′(t) + 2y(t)y′(t) = 0. We are told that at some timet0, x(t0) = 2/

√5, y(t0) = 1/

√5 and y′(t0) = 3. Then

2x(t0)x′(t0) + 2y(t0)y′(t0) = 0 ⇒

2

(2√5

)x′(t) + 2

(1√5

)(3) = 0 ⇒

x′(t0) = −3

2

3.2.2.3. *. Solution. The instantaneous percentage rate of change for R is

100R′

R= 100

(PQ)′

PQR=PQ

825

Page 842: CLP-1 Differential Calculus

SOLUTIONS TO EXERCISES

= 100P ′Q+ PQ′

PQproduct rule

= 100

[P ′

P+Q′

Q

]simplify

= 100[0.08− 0.02] = 6%

3.2.2.4. *. Solution. 3.2.2.4.a By the quotient rule, F ′ =P ′Q− PQ′

Q2. At the

moment in question, F ′ =5× 5− 25× 1

52= 0.

3.2.2.4.b We are told that, at the second moment in time, P ′ = 0.1P and Q′ =−0.05Q(or equivalently 100P

P= 10 and 100Q

Q= −5

). Substituting in these values:

F ′ =P ′Q− PQ′

Q2

=0.1PQ− P (−0.05Q)

Q2

=0.15PQ

Q2

= 0.15P

Q

= 0.15F

=⇒ F ′ = 0.15F

or 100F′

F= 15%. That is, the instantaneous percentage rate of change of F is 15%.

3.2.2.5. *. Solution.

• The distance z(t) between the particles at any moment in time is

z2(t) = x(t)2 + y(t)2,

where x(t) is the position on the x-axis of the particle A at time t (measuredin seconds) and y(t) is the position on the y-axis of the particle B at the sametime t.

• We differentiate the above equation with respect to t and get

2z · z′ = 2x · x′ + 2y · y′,

• We are told that x′ = −2 and y′ = −3. (The values are negative because xand y are decreasing.) It will take 3 seconds for particle A to reach x = 4,and in this time particle B will reach y = 3.

• At this point z =√x2 + y2 =

√32 + 42 = 5.

826

Page 843: CLP-1 Differential Calculus

SOLUTIONS TO EXERCISES

• Hence

10z′ = 8 · (−2) + 6 · (−3) = −34

z′ = −34

10= −17

5units per second.

3.2.2.6. *. Solution.

• We compute the distance z(t) between the two particles after t seconds as

z2(t) = 32 + (yA(t)− yB(t))2,

where yA(t) and yB(t) are the y-coordinates of particles A and B after tseconds, and the horizontal distance between the two particles is always 3units.

• We are told the distance between the particles is 5 units, this happens when

(yA − yB)2 = 52 − 32 = 16

yA − yB = 4

That is, when the difference in y-coordinates is 4. This happens when t = 4.

• We differentiate the distance equation (from the first bullet point) with re-spect to t and get

2z · z′ = 2(yA′ − yB ′)(yA − yB),

• We know that (yA− yB) = 4, and we are told that z = 5, y′A = 3, and y′B = 2.Hence

10z′(4) = 2× 1× 4 = 8

• Therefore

z′(4) =8

10=

4

5units per second.

3.2.2.7. *. Solution.

y(t)

x(t)

A

BH

z(t)

15 mph

20 mph

827

Page 844: CLP-1 Differential Calculus

SOLUTIONS TO EXERCISES

As in the above figure, let x(t) be the distance between H (Hawaii) and ship B,and y(t) be the distance between H and ship A, and z(t) be the distance betweenships A and B, all at time t. Then

x(t)2 + y(t)2 = z(t)2

Differentiating with respect to t,

2x(t)x′(t) + 2y(t)y′(t) = 2z(t)z′(t)

x(t)x′(t) + y(t)y′(t) = z(t)z′(t)

At the specified time, x(t) is decreasing, so x′(t) is negative, and y(t) is increasing,so y′(t) is positive.

(300)(−15) + (400)(20) =√

3002 + 4002z′(t)

500z′(t) = 3500

z′(t) = 7 mph

3.2.2.8. *. Solution.

• We compute the distance d(t) between the two snails after t minutes as

d2(t) = 302 + (y1(t)− y2(t))2,

where y1(t) is the altitude of the first snail, and y2(t) the altitude of thesecond snail after t minutes.

• We differentiate the above equation with respect to t and get

2d · d′ = 2(y1′ − y2

′)(y1 − y2)

d · d′ = (y1′ − y2

′)(y1 − y2)

• We are told that y1′ = 25 and y2

′ = 15. It will take 4 minutes for the firstsnail to reach y1 = 100, and in this time the second snail will reach y2 = 60.

• At this point d2 = 302 + (100− 60)2 = 900 + 1600 = 2500, hence d = 50.

• Therefore

50d′ = (25− 15)× (100− 60)

d′ =400

50= 8 cm per minute.

3.2.2.9. *. Solution.

• If we write z(t) for the length of the ladder at time t and y(t) for the heightof the top end of the ladder at time t we have

z(t)2 = 52 + y(t)2.

828

Page 845: CLP-1 Differential Calculus

SOLUTIONS TO EXERCISES

• We differentiate the above equation with respect to t and get

2z · z′ = 2y · y′,

• We are told that z′(t) = −2, so z(3.5) = 20− 3.5 · 2 = 13.

• At this point y =√z2 − 52 =

√169− 25 =

√144 = 12.

• Hence

2 · 13 · (−2) = 2 · 12y′

y′ = −2 · 13

12= −13

6meters per second.

3.2.2.10. Solution. What we’re given isdV

dt(where V is volume of water in the

trough, and t is time), and what we are asked for isdh

dt(where h is the height of

the water). So, we need an equation relating V and h. First, let’s get everythingin the same units: centimetres.

60 cm

100 cm

50 cm

200 cm

hw

We can calculate the volume of water in the trough by multiplying the area ofits trapezoidal cross section by 200 cm. A trapezoid with height h and bases b1

and b2 has area h(b1+b2

2

). (To see why this is so, draw the trapezoid as a rectangle

flanked by two triangles.) So, using w as the width of the top of the water (as inthe diagram above), the area of the cross section of the water in the trough is

A = h

(60 + w

2

)and therefore the volume of water in the trough is

V = 100h(60 + w) cm3.

We need a formula for w in terms of h. If we draw lines straight up from thebottom corners of the trapezoid, we break it into rectangles and triangles.

829

Page 846: CLP-1 Differential Calculus

SOLUTIONS TO EXERCISES

60

50

h

a

20

Using similar triangles,a

h=

20

50, so a =

2

5h. Then

w = 60 + 2a

= 60 + 2

(2

5h

)= 60 +

4

5h

so

V = 100h(60 + w)

= 100h(120 +4

5h)

= 80h2 + 12000h

This is the equation we need, relating V and h. Differentiating implicitly withrespect to t:

dV

dt= 2 · 80h · dh

dt+ 12000

dh

dt

= (160h+ 12000)dh

dt

We are given that h = 25 anddV

dt= 3 litres per minute. Converting to cubic

centimetres,dV

dt= −3000 cubic centimetres per minute. So:

−3000 = (160 · 25 + 12000)dh

dtdh

dt= − 3

16= −.1875

cm

min

So, the water level is dropping at3

16centimetres per minute.

3.2.2.11. Solution. If V is the volume of the water in the tank, and t is time,

then we are givendV

dt. What we want to know is

dh

dt, where h is the height of the

water in the tank. A reasonable plan is to find an equation relating V and h, and

830

Page 847: CLP-1 Differential Calculus

SOLUTIONS TO EXERCISES

differentiate it implicitly with respect to t.Let’s be a little careful about units. The volume of water in the tank is

(area of cross section of water)×(length of tank)

If we measure these values in metres (area in square metres, length in metres),then the volume is going to be in cubic metres. So, when we differentiate withrespect to time, our units will be cubic metres per second. The water is flowingin at one litre per second, or 1000 cubic centimetres per second. So, we eitherhave to measure our areas and distances in centimetres, or convert litres to cubicmetres. We’ll do the latter, but both are fine.If we imagine one cubic metre as a cube, with each side of length 1 metre, then it’seasy to see the volume inside is (100)3 = 106 cubic centimetres: it’s the volume ofa cube with each side of length 100 cm. Since a litre is 103 cubic centimetres, and a

cubic metre is 106 cubic centimetres, one litre is 10−3 cubic metres. So,dV

dt=

1

103

cubic metres per second.Let h be the height of the water (in metres). We can figure out the area of thecross section by breaking it into three pieces: a triangle on the left, a rectangle inthe middle, and a trapezoid on the right.

h

a b1.25 m

3 m3 m

1 m

• The triangle on the left has height h metres. Let its base be a metres. Itforms a similar triangle with the triangle whose height is 1.25 metres andwidth is 1 metre, so:

a

h=

1

1.25

a =4

5h

So, the area of the triangle on the left is

1

2ah =

2

5h2

831

Page 848: CLP-1 Differential Calculus

SOLUTIONS TO EXERCISES

• The rectangle in the middle has length 3 metres and height h metres, so itsarea is 3h square metres.

• The trapezoid on the right is a portion of a triangle with base 3 metres andheight 1.25 metres. So, its area is(

1

2(3)(1.25)

)︸ ︷︷ ︸

area of big triangle

−(

1

2(b)(1.25− h)

)︸ ︷︷ ︸

area of little triangle

The little triangle (of base b and height 1.25− h) is formed by the air on theright side of the tank. It is a similar triangle to the triangle of base 3 andheight 1.25, so

b

1.25− h =3

1.25

b =3

1.25(1.25− h)

So, the area of the trapezoid on the right is

1

2(3)(1.25)− 1

2

(3

1.25

)(1.25− h) (1.25− h)

= 3h− 6

5h2

So, the area A of the cross section of the water is

A =2

5h2︸︷︷︸

triangle

+ 3h︸︷︷︸rectangle

+ 3h− 6

5h2︸ ︷︷ ︸

trapezoid

= 6h− 4

5h2

So, the volume of water is

V = 5

(6h− 4

5h2

)= 30h− 4h2

Differentiating with respect to time, t:

dV

dt= 30

dh

dt− 8h

dh

dt

When h =1

10metre, and

dV

dt=

1

103cubic metres per second,

1

103= 30

dh

dt− 8

(1

10

)dh

dt

dh

dt=

1

29200metres per second

This is about 1 centimetre every five minutes. You might want a bigger hose.

3.2.2.12. Solution. Let θ be the angle of your head, where θ = 0 means you arelooking straight ahead, and θ =

π

2means you are looking straight up. We are

832

Page 849: CLP-1 Differential Calculus

SOLUTIONS TO EXERCISES

interested indθ

dt, but we only have information about h. So, a reasonable plan is

to find an equation relating h and θ, and differentiate with respect to time.

θ

The right triangle formed by you, the rocket, and the rocket’s original positionhas adjacent side (to θ) length 2km, and opposite side (to θ) length h(t) kilome-tres, so

tan θ =h

2

Differentiating with respect to t:

sec2 θ · dθ

dt=

1

2

dh

dtdθ

dt=

1

2cos2 θ · dh

dt

We know tan θ = h2. We draw a right triangle with angle θ (filling in the sides

using SOH CAH TOA and the Pythagorean theorem) to figure out cos θ:

θ2

h√ h

2 +4

Using the triangle, cos θ =2√

h2 + 4, so

dt=

1

2

(2√

h2 + 4

)2

· dh

dt

=

(2

h2 + 4

)dh

dt

833

Page 850: CLP-1 Differential Calculus

SOLUTIONS TO EXERCISES

So, the quantities we need to know one minute after liftoff (that is, when t =1

60)

are h(

1

60

)and

dh

dt

(1

60

). Recall h(t) = 61750t2.

h

(1

60

)=

61750

3600=

1235

72

dh

dt= 2(61750)t

dh

dt

(1

60

)=

2(61750)

60=

6175

3

Returning to the equationdθ

dt=

(2

h2 + 4

)dh

dt:

dt

(1

60

)=

(2(

123572

)2+ 4

)(6175

3

)≈ 13.8

rad

hour≈ 0.0038

rad

sec

3.2.2.13. *. Solution. 3.2.2.13.a Let x(t) be the distance of the train along thetrack at time t, measured from the point on the track nearest the camera. Let z(t)be the distance from the camera to the train at time t.

Then x′(t) = 2 and at the time in question, z(t) = 1.3 km and x(t) =√1.32 − 0.52 = 1.2 km. So

z(t)2 = x(t)2 + 0.52

2z(t)z′(t) = 2x(t)x′(t)

2× 1.3z′(t) = 2× 1.2× 2

z′(t)=2× 1.2

1.3≈ 1.85 km/min

3.2.2.13.b Let θ(t) be the angle shown at time t. Then

sin (θ(t)) =x(t)

z(t)

Differentiating with respect to t:

θ′(t) cos (θ(t)) =x′(t)z(t)− x(t)z′(t)

z(t)2

θ′(t) =x′(t)z(t)− x(t)z′(t)

z(t)2 cos (θ(t))

834

Page 851: CLP-1 Differential Calculus

SOLUTIONS TO EXERCISES

From our diagram, we see cos (θ(t)) =0.5

z(t), so:

= 2x′(t)z(t)− x(t)z′(t)

z(t)

Substituting in x′(t) = 2, z(t) = 1.3, x(t) = 1.2, and z′(t) =2× 1.2

1.3:

θ′(t) = 22× 1.3− 1.2× 2×1.2

1.3

1.3≈ .592 radians/min

3.2.2.14. Solution. Let θ be the angle between the two hands.

5cm

10cm

The Law of Cosines (Appendix B.4.1) tells us that

D2 = 52 + 102 − 2 · 5 · 10 · cos θ

D2 = 125− 100 cos θ

Differentiating with respect to time t,

2DdD

dt= 100 sin θ · dθ

dt

Our tasks now are to find D, θ anddθ

dtwhen the time is 4:00. At 4:00, the minute

hand is straight up, and the hour hand is4

12=

1

3of the way around the clock, so

θ =1

3(2π) =

3at 4:00. Then D2 = 125− 100 cos

(2π3

)= 125− 100

(−1

2

)= 175, so

D =√

175 = 5√

7 at 4:00.

To calculatedθ

dt, remember that both hands are moving. The hour hand makes a

full rotation every 12 hours, so its rotational speed is2π

12=π

6radians per hour.

The hour hand is being chased by the minute hand. The minute hand makes

a full rotation every hour, so its rotational speed is2π

1= 2π radians per hour.

Therefore, the angle θ between the two hands is changing at a rate of

dt= −

(2π − π

6

)=−11π

6

rad

hr.

835

Page 852: CLP-1 Differential Calculus

SOLUTIONS TO EXERCISES

Now, we plug in D, θ, anddθ

dtto find

dD

dt:

2DdD

dt= 100 sin θ · dθ

dt

2(

5√

7) dD

dt= 100 sin

(2π

3

)(−11π

6

)10√

7dD

dt= 100

(√3

2

)(−11π

6

)= −275π√

3

dD

dt=−55√

21π

42

cm

hr

So D is decreasing at55√

21π

42≈ 19 centimetres per hour.

3.2.2.15. *. Solution. The area at time t is the area of the outer circle minus thearea of the inner circle:

A(t) = π(R(t)2 − r(t)2

)So, A′(t) = 2π

(R(t)R′(t)− r(t)r′(t)

)Plugging in the given data,

A′ = 2π(3 · 2− 1 · 7

)= −2π

So the area is shrinking at a rate of 2πcm2

s.

3.2.2.16. Solution. The volume between the spheres, while the little one is in-side the big one, is

V =4

3πR3 − 4

3πr3

Differentiating implicitly with respect to t:

dV

dt= 4πR2 dR

dt− 4πr2 dr

dt

We differentiate R = 10+2t and r = 6t to finddR

dt= 2 and

dr

dt= 6. When R = 2r,

10 + 2t = 2(6t), so t = 1. When t = 1, R = 12 and r = 6. So:

dV

dt= 4π

(122)

(2)− 4π(62)

(6) = 288π

So the volume between the two spheres is increasing at 288 cubic units per unittime.Remark: when the radius of the inner sphere increases, we are “subtracting”more area. Since the radius of the inner sphere grows faster than the radius ofthe outer sphere, we might expect the area between the spheres to be decreasing.

836

Page 853: CLP-1 Differential Calculus

SOLUTIONS TO EXERCISES

Although the radius of the outer sphere grows more slowly, a small increase inthe radius of the outer sphere results in a larger change in volume than the sameincrease in the radius of the inner sphere. So, a result showing that the volumebetween the spheres is increasing is not unreasonable.

3.2.2.17. Solution. We know something about the rate of change of the heighth of the triangle, and we want to know something about the rate of change of itsarea, A. A reasonable plan is to find an equation relating A and h, and differen-tiate implicitly with respect to t. The area of a triangle with height h and base bis

A =1

2bh

Note, bwill change with time as well as h. So, differentiating with respect to time,t:

dA

dt=

1

2

(db

dt· h+ b · dh

dt

)We are given dh

dtand h, but those b’s are a mystery. We need to relate them to h.

We can do this by breaking our triangle into two right triangles and using thePythagorean Theorem:

150 cm 200 cmh

√1502 − h2

√2002 − h2

So, the base of the triangle is

b =√

1502 − h2 +√

2002 − h2

Differentiating with respect to t:

db

dt=

−2hdhdt

2√

1502 − h2+

−2hdhdt

2√

2002 − h2

=−hdh

dt√1502 − h2

+−hdh

dt√2002 − h2

Usingdh

dt= −3 centimetres per minute:

db

dt=

3h√1502 − h2

+3h√

2002 − h2

837

Page 854: CLP-1 Differential Calculus

SOLUTIONS TO EXERCISES

When h = 120,√

1502 − h2 = 90 and√

2002 − h2 = 160. So, at this moment intime:

b = 90 + 160 = 250

db

dt=

3(120)

90+

3(120)

160= 4 +

9

4=

25

4

We return to our equation relating the derivatives of A, b, and h.

dA

dt=

1

2

(db

dt· h+ b · dh

dt

)

When h = 120 cm, b = 250,dh

dt= −3, and

db

dt=

25

4:

dA

dt=

1

2

(25

4(120) + 250(−3)

)= 0

Remark: What does it mean thatdA

dt

∣∣∣∣h=120

= 0? Certainly, as the height changes,

the area changes as well. As the height sinks to 120 cm, the area is increasing,but after it sinks past 120 cm, the area is decreasing. So, at the instant when theheight is exactly 120 cm, the area is neither increasing nor decreasing: it is at alocal maximum. You’ll learn more about this kind of problem in Section 3.5.

3.2.2.18. Solution. Let S be the flow of salt (in cubic centimetres per second).

We want to knowdS

dt: how fast the flow is changing at time t. We are given an

equation for S:

S =1

5A

where A is the uncovered area of the cut-out. So,

dS

dt=

1

5

dA

dt

If we can finddA

dt, then we can find

dS

dt. We are given information about how

quickly the door is rotating. If we let θ be the angle made by the leading edge of

the door and the far edge of the cut-out (shown below), thendθ

dt= −π

6radians

per second. (Since the door is covering more and more of the cut-out, θ is getting

smaller, sodθ

dtis negative.)

838

Page 855: CLP-1 Differential Calculus

SOLUTIONS TO EXERCISES

edge

ofcu

t-ou

t

edge of door

θ

Since we knowdθ

dt, and we want to know

dA

dt(in order to get

dS

dt), it is reason-

able to look for an equation relating A and θ, and differentiate it implicitly with

respect to t to get an equation relatingdA

dtand

dt.

The area of an annulus with outer radius 6 cm and inner radius 1 cm is π · 62−π ·12 = 35π square centimetres. A sector of that same annulus with angle θ has area(θ

)(35π), since θ

2πis the ratio of the sector to the entire annulus. (For example,

if θ = π, then the sector is half of the entire annulus, so its area is (1/2)35π.)So, when 0 ≤ θ ≤ π

2, the area of the cutout that is open is

A =θ

2π(35π) =

35

This is the formula we wanted, relating A and θ. Differentiating with respect tot,

dA

dt=

35

2

dt=

35

2

(−π

6

)= −35π

12

SincedS

dt=

1

5

dA

dt,

dS

dt= −1

5

35π

12= −7π

12≈ −1.8

cm3

sec2

Remark: the change in flow of salt is constant while the door covers more andmore of the cut-out, so we never used the fact that precisely half of the cut-outwas open. We also never used the radius of the lid, which is immaterial to theflow of salt.

839

Page 856: CLP-1 Differential Calculus

SOLUTIONS TO EXERCISES

3.2.2.19. Solution. Let F be the flow of water through the pipe, so F =1

5A. We

want to knowdF

dt, so differentiating implicitly with respect to t, we find

dF

dt=

1

5

dA

dt.

If we can finddA

dt, then we can find

dF

dt. We know something about the shape of

the uncovered area of the pipe; a reasonable plan is to find an equation relatingthe height of the door with the uncovered area of the pipe. Let h be the distancefrom the top of the pipe to the bottom of the door, measured in metres.

doorh

1 1− h

√2h− h2

θ

Since the radius of the pipe is 1 metre, the orange line has length 1 − h metres,and the blue line has length 1 metre. Using the Pythagorean Theorem, the greenline has length

√12 − (1− h)2 =

√2h− h2 metres.

The uncovered area of the pipe can be broken up into a triangle (of height 1− hand base 2

√2h− h2) and a sector of a circle (with angle 2π − 2θ). The area of the

triangle is(1− h)︸ ︷︷ ︸height

√2h− h2︸ ︷︷ ︸12base

.

840

Page 857: CLP-1 Differential Calculus

SOLUTIONS TO EXERCISES

The area of the sector is (2π − 2θ

)︸ ︷︷ ︸

fractionof circle

(π · 12)︸ ︷︷ ︸area

of circle

= π − θ.

Remember: what we want is to finddA

dt, and what we know is

dh

dt= 0.01 me-

tres per second. If we find θ in terms of h, we find A in terms of h, and thendifferentiate with respect to t.Since θ is an angle in a right triangle with hypotenuse 1 and adjacent side length1−h, cos θ = 1−h

1= 1−h. We want to conclude that θ = arccos(1−h), but let’s be a

little careful: remember that the range of the arccosine function is angles in [0, π].We must be confident that 0 ≤ θ ≤ π in order to conclude θ = arccos(1 − h)–butclearly, θ is in this range. (Remark: we could also have said sin θ =

√2h−h2

1, and

so θ = arcsin(√

2h− h2). This would require −π

2≤ θ ≤ π

2, which is true when

h < 1, but false for h > 1. Since our problem asks about h = 0.25, we could alsouse arcsine.)Now, we know the area of the open pipe in terms of h.

A = (area of triangle) + (area of sector)

= (1− h)√

2h− h2 + (π − θ)= (1− h)

√2h− h2 + π − arccos (1− h)

We want to differentiate with respect to t. Using the chain rule:

dA

dt=

dA

dh· dh

dt

dA

dt=

((1− h)

2− 2h

2√

2h− h2+ (−1)

√2h− h2 +

−1√1− (1− h)2

)dh

dt

=

((1− h)2

√2h− h2

−√

2h− h2 − 1√2h− h2

)dh

dt

=

((1− h)2 − 1√

2h− h2−√

2h− h2

)dh

dt

=

(−(2h− h2)√2h− h2

−√

2h− h2

)dh

dt

=(−√

2h− h2 −√

2h− h2) dh

dt

= −2√

2h− h2dh

dt

We note here that the negative sign makes sense: as the door lowers, h increases

and A decreases, sodh

dtand

dA

dtshould have opposite signs.

841

Page 858: CLP-1 Differential Calculus

SOLUTIONS TO EXERCISES

When h =1

4metres, and

dh

dt=

1

100metres per second:

dA

dt= −2

√2

4− 1

42

(1

100

)= −√

7

200

cm2

s

SincedF

dt=

1

5

dA

dt:

dF

dt= −

√7

1000

m3

sec2

That is, the flow is decreasing at a rate of√

7

1000

m3

sec2.

3.2.2.20. Solution. We are given the rate of change of the volume of liquid,and are asked for the rate of change of the height of the liquid. So, we need anequation relating volume and height.The volume V of a cone with height h and radius r is 1

3πr2h. Since we know

dV

dt, and want to know

dh

dt, we need to find a way to deal with the unwanted

variable r. We can find r in terms of h by using similar triangles. Viewed fromthe side, the conical glass is an equilateral triangle, as is the water in it. Using thePythagorean Theorem, the cone has height 5

√3.

h

r10

5

5√

3

Using similar triangles,r

h=

5

5√

3, so r =

h√3

. (Remark: we could also use the fact

that the water forms a cone that looks like an equilateral triangle when viewed

from the side to conclude r =h√3

.)

Now, we can write the volume of water in the cone in terms of h, and no othervariables.

V =1

3πr2h

=1

(h√3

)2

h

842

Page 859: CLP-1 Differential Calculus

SOLUTIONS TO EXERCISES

9h3

Differentiating with respect to t:

dV

dt=π

3h2 dh

dt

When h = 7 cm anddV

dt= −5 mL per minute,

−5 =π

3(49)

dh

dtdh

dt=−15

49π≈ −0.097 cm per minute

Exercises — Stage 33.2.2.21. Solution. As is so often the case, we use a right triangle in this problemto relate the quantities.

2

θ

D

sin θ =D

2D = 2 sin θ

Using the chain rule, we differentiate both sides with respect to time, t.

dD

dt= 2 cos θ · dθ

dt

So, ifdθ

dt= 0.25 radians per hour and θ =

π

4radians, then

(a)dD

dt= 2 cos

(π4

)· 0.25 = 2

(1√2

)1

4=

1

2√

2metres per hour.

Setting aside part (b) for a moment, let’s think about (c). Ifdθ

dtand

dD

dthave

different signs, then becausedD

dt= 2 cos θ · dθ

dt, that means cos θ < 0. We have to

have a nonnegative depth, so D > 0 and D = 2 sin θ implies sin θ > 0. If sin θ ≥ 0and cos θ < 0, then θ ∈ (π/2, π]. On the diagram, that looks like this:

843

Page 860: CLP-1 Differential Calculus

SOLUTIONS TO EXERCISES

2θD

That is: the water has reversed direction. This happens, for instance, when a riverempties into the ocean and the tide is high. Skookumchuck Narrows provincialpark, in the Sunshine Coast, has reversing rapids.Now, let’s return to (b). If the rope is only 2 metres long, and the river riseshigher than 2 metres, then our equation D = 2 sin θ doesn’t work any more: thebuoy might be stationary underwater while the water rises or falls (but stays ator above 2 metres deep).

3.2.2.22. Solution. (a) When the point is at (0,−2), its y-coordinate is not chang-ing, because it is moving along a horizontal line. So, the rate at which the particle

moves is simplydx

dt. Let θ be the angle an observer would be looking at, in order

to watch the point. Since we knowdθ

dt, a reasonable plan is to find an equation

relating θ and x, and then differentiate implicitly with respect to t. To do this,let’s return to our diagram.

x

y

x

θ2

When the point is a little to the right of (0,−2), then we can make a triangle with

the origin, as shown. If we let θ be the indicated angle, thendθ

dt= 1 radian per

second. (It is given that the observer is turning one radian per second, so this ishow fast θ is increasing.) From the right triangle in the diagram, we see

tan θ =x

2

844

Page 861: CLP-1 Differential Calculus

SOLUTIONS TO EXERCISES

Now, we have to take care of a subtle point. The diagram we drew only makessense for the point when it is at a position a little to the right of (0,−2). So, rightnow, we’ve only made a set-up that will find the derivative from the right. But,with a little more thought, we see that even when x is negative (that is, when thepoint is a little to the left of (0,−2)), our equation holds if we are careful abouthow we define θ. Let θ be the angle between the line connecting the point andthe origin, and the y-axis, where θ is negative when the point is to the left of they-axis.

x

y

|x|

2|θ|

Since x and θ are both negative when the point is to the left of the y-axis,

tan |θ| = 2

|x|tan(−θ) =

−x2

So, since tan(−θ) = − tan(θ):

tan θ =x

2

So, we’ve shown that the relationship tan θ =x

2holds when our point is at

(x,−2), regardless of the sign of x.

Moving on, since we are givendθ

dtand asked for

dx

dt, we differentiate with respect

to t:

sec2 θ · dθ

dt=

1

2· dx

dt

When the point is at (0,−2), since the observer is turning at one radian per sec-

ond, alsodθ

dt= 1. Also, looking at the diagram, θ = 0. Plugging in these values:

sec2 (0) · (1) =1

2· dx

dt

845

Page 862: CLP-1 Differential Calculus

SOLUTIONS TO EXERCISES

1 =1

2· dx

dtdx

dt= 2

So, the particle is moving at 2 units per second.(b) When the point is at (0, 2), it is moving along a line with slope −1

2and y-

intercept 2. So, it is on the line

y = 2− 1

2x

That is, at time t, if the point is at (x(t), y(t)), then x(t) and y(t) satisfy y(t) =2− 1

2x(t). Implicitly differentiating with respect to t:

dy

dt= −1

2· dx

dt

So, whendx

dt= 1,

dy

dt= −1

2. That is, its y-coordinate is decreasing at

1

2unit per

second.For the question “How fast is the point moving?”, remember that the velocity ofan object can be found by differentiating (with respect to time) the equation thatgives the position of the object. The complicating factors in this case are that (1)the position of our object is not given as a function of time, and (2) the positionof our object is given in two dimensions (an x coordinate and a y coordinate), notone.Remark: the solution below is actually pretty complicated. It is within your abil-ities to figure it out, but later on in your mathematical career you will learn aneasier way, using vectors. For now, take this as a relatively tough exercise, and amotivation to keep learning: your intuition that there must be an easier way is wellfounded!The point is moving along a straight line. So, to take care of complication (2), wecan give its position as a point on the line. We can take the line as a sort of axis.We’ll need to choose a point on the axis to be the “origin”: (2, 1) is a convenientpoint. Let D be the point’s (signed) distance along the “axis” from (2, 1). Whenthe point is a distance of one unit to the left of (2, 1), we’ll have D = −1, andwhen the point is a distance of one unit to the right of (2, 1), we’ll have D = 1.

Then D changes with respect to time, anddD

dtis the velocity of the point. Since

we knowdx

dtand

dy

dt, a reasonable plan is to find an equation relating x, y, andD,

and differentiate implicitly with respect to t. (This implicit differentiation takescare of complication (1).) Using the Pythagorean Theorem:

D2 = (x− 2)2 + (y − 1)2

Differentiating with respect to t:

2D · dD

dt= 2(x− 2) · dx

dt+ 2(y − 1) · dy

dt

846

Page 863: CLP-1 Differential Calculus

SOLUTIONS TO EXERCISES

We plug in x = 0, y = 2, dxdt

= 1, dydt

= −12, and D = −

√(0− 2)2 + (2− 1)2 = −

√5

(negative because the point is to the left of (2, 1)):

−2√

5 · dD

dt= 2(−2)(1) + 2(1)

(−1

2

)dD

dt=

√5

2units per second

3.2.2.23. Solution. (a) Since the perimeter of the bottle is unchanged (youaren’t stretching the plastic), it is always the same as the perimeter before it wassmooshed, which is the circumference of a circle of radius 5, or 2π(5) = 10π. So,using our approximation for the perimeter of an ellipse,

10π = π[3(a+ b)−

√(a+ 3b)(3a+ b)

]10 = 3(a+ b)−

√(a+ 3b)(3a+ b)

(b) The area of the base of the bottle is πab (see Section A.10), and its height is 20cm, so the volume of the bottle is

V = 20πab

(c) As you smoosh the bottle, its volume decreases, so the water spills out. (Ifit turns out that the volume is increasing, then no water is spilling out–but lifeexperience suggests, and our calculations verify, that this is not the case.) The

water will spill out at a rate of−dV

dtcubic centimetres per second, where V is the

volume inside the bottle. We know something about a andda

dt, so a reasonable

plan is to differentiate the equation from (b) (relating V and a) with respect to t.Using the product rule, we differentiate the equation in (b) implicitly with respectto t and get

dV

dt= 20π

(da

dtb+ a

db

dt

)So, we need to find the values of a, b,

da

dt, and

db

dtat the moment when a = 2b.

The equation from (a) tells us 10 = 3(a+ b)−√

(a+ 3b)(3a+ b). So, when a = 2b,

10 = 3(2b+ b)−√

(2b+ 3b)(6b+ b)

10 = 9b−√

(5b)(7b) = b(

9−√

35)

b=10

9−√

35

where we use the fact that b is a positive number, so√b2 = |b| = b.

847

Page 864: CLP-1 Differential Calculus

SOLUTIONS TO EXERCISES

Since a = 2b,

a=20

9−√

35

Now we know a and b at the moment when a = 2b. We still need to knowda

dt

anddb

dtat that moment. Since a = 5 + t, always

da

dt= 1. The equation from

(a) relates a and b, so differentiating both sides with respect to t will give us an

equation relatingda

dtand

db

dt. When differentiating the portion with a square root,

be careful not to forget the chain rule.

0 = 3

(da

dt+

db

dt

)−(

dadt

+ 3dbdt

)(3a+ b) + (a+ 3b)

(3da

dt+ db

dt

)2√

(a+ 3b)(3a+ b)

Sinceda

dt= 1:

0 = 3

(1 +

db

dt

)−(1 + 3db

dt

)(3a+ b) + (a+ 3b)

(3 + db

dt

)2√

(a+ 3b)(3a+ b)

At this point, we could plug in the values we know for a and b at the momentwhen a = 2b. However, the algebra goes a little smoother if we start by pluggingin a = 2b:

0 = 3

(1 +

db

dt

)−(1 + 3db

dt

)(7b) + (5b)

(3 + db

dt

)2√

(5b)(7b)

0 = 3

(1 +

db

dt

)− b

(7 + 21db

dt+ 15 + 5db

dt

)2b√

35

0 = 3

(1 +

db

dt

)− 22 + 26db

dt

2√

35

0 = 3 + 3db

dt− 11√

35− 13√

35

db

dt

−3 +11√35

=

(3− 13√

35

)db

dt

db

dt=−3 + 11√

35

3− 13√35

=−3√

35 + 11

3√

35− 13

Now, we can calculatedV

dtat the moment when a = 2b. We already found

dV

dt= 20π

(da

dtb+ a

db

dt

)So, plugging in the values of a, b,

da

dt, and

db

dtat the moment when a = 2b:

dV

dt= 20π

((1)

(10

9−√

35

)+

(20

9−√

35

)(−3√

35 + 11

3√

35− 13

))

848

Page 865: CLP-1 Differential Calculus

SOLUTIONS TO EXERCISES

=200π

9−√

35

(1− 2

(3√

35− 11

3√

35− 13

))

≈ −375.4cm3

sec

So the water is spilling out of the cup at about 375.4 cubic centimetres per second.Remark: the algebra in this problem got a little nasty, but the method behind itssolution is no more difficult than most of the problems in this section. One ofthe reasons why calculus is so widely taught in universities is to give you lots ofpractice with problem-solving: taking a big problem, breaking it into pieces youcan manage, solving the pieces, and getting a solution.A problem like this can sometimes derail people. Breaking it up into pieces isn’tso hard, but when you actually do those pieces, you can get confused and forgetwhy you are doing the calculations you’re doing. If you find yourself in this sit-uation, look back a few steps to remind yourself why you started the calculationyou just did. It can also be helpful to write notes, like “We are trying to find dV

dt.

We already know that dVdt

= .... We still need to find a, b, dadt

and dbdt

.”

3.2.2.24. Solution. Since A = 0, the equation relating the variables tells us:

0 = log(C2 +D2 + 1

)1 = C2 +D2 + 1

0 = C2 +D2

0 = C = D

This will probably be useful information. Since we’re also given the value of aderivative, let’s differentiate the equation relating the variables implicitly with

respect to t. For ease of notation, we will writedA

dt= A′, etc.

A′B + AB′ =2CC ′ + 2DD′

C2 +D2 + 1

At t = 10, A = C = D = 0:

A′B + 0 =0 + 0

0 + 0 + 1

A′B = 0

at t = 10, A′ = 2 units per second:

2B = 0

B = 0.

3.3 · Exponential Growth and Decay — a First Look atDifferential Equations

849

Page 866: CLP-1 Differential Calculus

SOLUTIONS TO EXERCISES

3.3.4 · Exercises· Exercises for § 3.3.1

Exercises — Stage 13.3.4.1. Solution. In the beginning of this section, the text says “A differentialequation is an equation for an unknown function that involves the derivative ofthe unknown function.” Our unknown function is y, so a differential equation is

an equation that relates y anddy

dx. This applies to (a) and (b), but not (c), (d), or

(e).

Note thatdx

dx= 1: this is the derivative of x with respect to x.

3.3.4.2. Solution. Theorem 3.3.2 tells us that a function is a solution to the dif-ferential equation

dQ

dt= kQ(t) if and only if the function has the formQ(t) = Cekt

for some constant C. In our case, we want Q(t) = 5dQ

dt, so

dQ

dt=

1

5Q(t). So, the

theorem tells us that the solutions are the functions of the formQ(t) = Cet/5. Thisapplies to (a) (with C = 0) and (d) (with C = 1), but none of the other functions.We don’t actually need a theorem to answer this question, though: we can justtest every option.

adQ

dt= 0, so Q(t) = 0 = 5 · 0 = 5

dQ

dt, so (a) is a solution.

bdQ

dt= 5et = Q(t), so Q(t) =

dQ

dt6= 5

dQ

dt, so (b) is not a solution.

cdQ

dt= 5e5t = 5Q(t), so Q(t) =

1

5

dQ

dt6= 5

dQ

dt, so (c) is not a solution.

ddQ

dt=

1

5et/5 =

1

5Q(t), so Q(t) = 5

dQ

dt, so (d) is a solution.

edQ

dt=

1

5et/5 =

1

5(Q(t)− 1), so Q(t) = 5

dQ

dt+ 1, so (e) is not a solution.

3.3.4.3. Solution. What we’re asked to find is when

Q(t) = 0

That is,

Ce−kt = 0

If C = 0, then this is the case for all t. There was no isotope to begin with, andthere will continue not being any undecayed isotope forever.If C > 0, then since e−kt > 0, also Q(t) > 0: so Q(t) is never 0 for any value of t.(But as t gets bigger and bigger, Q(t) gets closer and closer to 0.)Remark: The last result is somewhat disturbing: surely at some point the lastatom has decayed. The differential equation we use is a model that assumes Q

850

Page 867: CLP-1 Differential Calculus

SOLUTIONS TO EXERCISES

runs continuously. This is a good approximation only when there is a very largenumber of atoms. In practice, that is almost always the case.

Exercises — Stage 23.3.4.4. *. Solution. The two pieces of information give us

f(0) = A = 5 f(7) = Ae7k = π

Thus we know that A = 5 and so π = f(7) = 5e7k. Hence

e7k =π

57k = log(π/5)

k =1

7· log(π/5).

where we use log to mean natural logarithm, loge.

3.3.4.5. *. Solution. In Theorem 3.3.2, we saw that if y is a function of t, anddy

dt= −ky, then y = Ce−kt for some constant C.

Our equation y satisfiesdy

dt= −3y, so the theorem tells us y = Ce−3t for some

constant C.We are also told that y(1) = 2. So, 2 = Ce−3×1 tells us C = 2e3. Then:

y = 2e3 · e−3t = 2e−3(t−1).

3.3.4.6. Solution. The amount of Carbon-14 in the sample t years after the ani-mal died will be

Q(t) = 5e−kt

for some constant k (where 5 is the amount of Carbon-14 in the sample at timet = 0). So, the answer we’re looking for is Q(10000). We need to replace k withan actual number to evaluate Q(10000), and the key to doing this is the half-life.The text tells us that the half-life of Carbon-14 is 5730 years, so we know:

Q(5730) =5

2

5e−k·5730 =5

2(e−k)5730

=1

2

e−k =5730

√1

2= 2−

15730

So:

Q(t) = 5(e−k)t

= 5 · 2−t

5730

851

Page 868: CLP-1 Differential Calculus

SOLUTIONS TO EXERCISES

Now, we can evaluate:

Q(10000) = 5 · 2−100005730 ≈ 1.5 µg

Remark: after 2(5730) = 11, 460 years, the sample will have been sitting for twohalf-lives, so its remaining Carbon-14 will be a quarter of its original amount, or1.25 µg. It makes sense that at 10,000 years, the sample will contain slightly moreCarbon-14 than at 11,460 years. Indeed, 1.5 is slightly larger than 1.25, so ouranswer seems plausible.It’s a good habit to look for ways to quickly check whether your answer seemsplausible, since a small algebra error can easily turn into a big error in your solu-tion.

3.3.4.7. Solution. Let 100 years ago be the time t = 0. Then ifQ(t) is the amountof Radium-226 in the sample, Q(0) = 1, and

Q(t) = e−kt

for some positive constant k. When t = 100, the amount of Radium-226 left is0.9576 grams, so

0.9576 = Q(100) = e−k·100 =(e−k)100

e−k = 0.95761

100

This tells us

Q(t) = 0.9576t

100

So, if half the original amount of Radium-226 is left,

1

2= 0.9576

t100

log

(1

2

)= log

(0.9576

t100

)− log 2 =

t

100log(0.9576)

t = −100log 2

log 0.9576≈ 1600

So, the half life of Radium-226 is about 1600 years.

3.3.4.8. *. Solution. Let Q(t) denote the mass at time t. ThendQ

dtis the rate at

which the mass is changing. Since the rate the mass is decreasing is proportional

to the mass remaining, we knowdQ

dt= −kQ(t), where k is a positive constant.

(Remark: since Q is decreasing,dQ

dtis negative. Since we cannot have a negative

852

Page 869: CLP-1 Differential Calculus

SOLUTIONS TO EXERCISES

mass, if we choose k to be positive, then k and Q are both positive–this is whywe added the negative sign.)The information given in the question is:

Q(0) = 6dQ

dt= −kQ(t) Q(1) = 1

for some constant k > 0. By Theorem 3.3.2, we know

Q(t) = Ce−kt

for some constant C. Since Q(0) = Ce0 = C, the given information tells us 6 = C.(This is the initial mass of our sample.) So, Q(t) = 6e−kt. To get the full pictureof the behaviour of Q, we should find k. We do this using the given informationQ(1) = 1:

1 = Q(1) = 6e−k(1)

6−1 =1

6= e−k

So, all together,

Q(t) = 6(e−k)t

= 6 ·(6−1)t

= 61−t

The question asks us to determine the time th which obeys Q(th) =6

2= 3. Now

that we know the equation for Q(t), we simply solve:

Q(t) = 61−t

3 = Q(th) = 61−th

log 3 = log(61−th) = (1− th) log 6

log 3

log 6= 1− th

th = 1− log 3

log 6=

log 6− log 3

log 6=

log 2

log 6

The half-life of Polonium-210 islog 2

log 6years, or about 141 days.

Remark: The actual half-life of Polonium-210 is closer to 138 days. The numbersin the question are made to work out nicely, at the expense of some accuracy.

3.3.4.9. Solution. The amount of Radium-221 in a sample will be

Q(t) = Ce−kt

where C is the amount in the sample at time t = 0, and k is some positive con-stant. We know the half-life of the isotope, so we can find e−k:

C

2= Q(30) = Ce−k·30

853

Page 870: CLP-1 Differential Calculus

SOLUTIONS TO EXERCISES

1

2=(e−k)30

2−130 = e−k

So,

Q(t) = C(e−k)t

= C · 2−t

30

When only 0.01% of the original sample is left, Q(t) = 0.0001C:

0.0001C = Q(t) = C · 2−t

30

0.0001 = 2−t

30

log(0.0001) = log

(2−

t30

)log(10−4

)= − t

30log 2

−4 log 10 = − t

30log 2

t = 120 · log 10

log 2≈ 398.6

It takes about 398.6 seconds (that is, roughly 6 and a half minutes) for all but0.01% of the sample to decay.Remark: we can do another reality check here. The half-life is 30 seconds. 6and a half minutes represents 13 half-lives. So, the sample is halved 13 times:(

12

)13 ≈ 0.00012 = 0.012%. So these 13 half-lives should reduce the sample toabout 0.01% of its original amount, as desired.

Exercises — Stage 33.3.4.10. Solution. We know that the amount of Polonium-210 in a sample aftert days is given by

Q(t) = Ce−kt

where C is the original amount of the sample, and k is some positive constant.The question asks us what percentage of the sample decays in a day. Since tis measured in days, the amount that decays in a day is Q(t) − Q(t + 1). The

percentage of Q(t) that this represents is 100Q(t)−Q(t+ 1)

Q(t). (For example, if

there were two grams at time t, and one gram at time t + 1, then 1002− 1

1= 50:

50% of the sample decayed in a day.)In order to simplify, we should figure out a better expression for Q(t). As usual,we make use of the half-life.

Q(138) =C

2

854

Page 871: CLP-1 Differential Calculus

SOLUTIONS TO EXERCISES

Ce−k·138 =C

2(e−k)138

=1

2= 2−1

e−k = 2−1

138

Now, we have a better formula for Q(t):

Q(t) = C(e−k)t

Q(t) = C · 2−t

138

Finally, we can evaluate what percentage of the sample decays in a day.

100Q(t)−Q(t+ 1)

Q(t)= 100

C · 2−t

138 − C · 2−t+1138

C · 2−t

138

( 1C1C

)

= 1002−

t138 − 2−

t+1138

2−t

138

= 100

(2−

t138 − 2−

t+1138

)2

t138

= 100

(1− 2−

1138

)≈ 0.5

About 0.5% of the sample decays in a day.Remark: when we say that half a percent of the sample decays in a day, wedon’t mean half a percent of the original sample. If a day starts out with, say, 1microgram, then what decays in the next 24 hours is about half a percent of that1 microgram, regardless of what the “original” sample (at some time t = 0) held.In particular, since the sample is getting smaller and smaller, that half of a percentthat decays every day represents fewer and fewer actual atoms decaying. That’swhy we can’t say that half of the sample (50%) will decay after about 100 days,even though 0.5% decays every day and 100× 0.5 = 50.

3.3.4.11. Solution. The amount of Uranium-232 in the sample of ore at time twill be

Q(t) = Q(0)e−kt

where 6.9 ≤ Q(0) ≤ 7.5. We don’t exactly know Q(0), and we don’t exactlyknow the half-life, so we also won’t exactly know Q(10): we can only say that isit between two numbers. Our strategy is to find the highest and lowest possiblevalues of Q(10), given the information in the problem. In order for the mostpossible Uranium-232 to be in the sample after 10 years, we should start with themost and have the longest half-life (since this represents the slowest decay). So,we take Q(0) = 7.5 and Q(70) = 1

2(7.5).

Q(t) = 7.5e−kt

855

Page 872: CLP-1 Differential Calculus

SOLUTIONS TO EXERCISES

1

2(7.5) = Q(70) = 7.5e−k(70)

1

2=(e−k)70

2−170 = e−k

So, in this secenario,

Q(t) = 7.5 · 2−t

70

After ten years,

Q(10) = 7.5 · 2−1070 ≈ 6.79

So after ten years, the sample contains at most 6.8 µg. Now, let’s think about theleast possible amount of Uranium-232 that could be left after 10 years. We shouldstart with as little as possible, so take Q(0) = 6.9, and the sample should decayquickly, so take the half-life to be 68.8 years.

Q(t) = 6.9e−kt

1

26.9 = Q(68.8) = 6.9e−k(68.8)

1

2=(e−k)68.8

2−1

68.8 = e−k

In this scenario,

Q(t) = 6.9 · 2−t

68.8

After ten years,

Q(10) = 6.9 · 2−10

68.8 ≈ 6.24

So after ten years, the sample contains at least 6.2 µg.After ten years, the sample contains between 6.2 and 6.8 µg of Uranium-232.

· Exercises for § 3.3.2

Exercises — Stage 13.3.4.1. Solution. Using Corollary 3.3.8 (with A = 20 and K = 5), solutions tothe differential equation all have the form

T (t) = [T (0)− 20]e5t + 20

for some constant T (0). This fits (a) (with T (0) = 20), (c) (with T (0) = 21), and

856

Page 873: CLP-1 Differential Calculus

SOLUTIONS TO EXERCISES

(d) (with T (0) = 40), but not (b) (since the constant has the wrong sign).Instead of using the corollary, we can also just check each function for ourselves.

adT

dt= 0 = 5 · 0 = 5[T (t) − 20], so (a) gives a solution to the differential

equation.

bdT

dt= 5[20e5t] = 5[T + 20] 6= 5[T − 20], so (b) does not give a solution to the

differential equation.

cdT

dt= 5[e5t] = 5[T − 20], so (c) gives a solution to the differential equation.

ddT

dt= 5[20e5t] = 5[T−20], so (d) gives a solution to the differential equation.

3.3.4.2. Solution. From Newton’s Law of Cooling and Corollary 3.3.8, the tem-perature of the object will be

T (t) = [T (0)− A]eKt + A

where A is the ambient temperature (the temperature of the room), T (0) is theinitial temperature of the copper, and K is some constant. So, the ambienttemperature–the temperature of the room– is −10 degrees. Since the coefficientof the exponential part of the function is positive, the temperature of the object ishigher than the temperature of the room.

3.3.4.3. Solution. As t grows very large, T (t) approaches A. That is:

limt→∞

T (t) = A

limt→∞

[T (0)− A]eKt + A = A

limt→∞

[T (0)− A]eKt = 0

Since the object is warmer than the room, T (0)− A is a nonzero constant. So,

limt→∞

eKt = 0

This tells us that K is a negative number. So, K must be negative–not zero, andnot positive.Remark: in our work, we used the fact that the object and the room have differenttemperatures (but it didn’t matter which one was hotter). If not, then T (0) = A,and T (t) = A: that is, the temperature of the object is constant. In this case, ourusual form for the temperature of the object looks like this:

T (t) = 0eKt + A

Keeping the exponential piece in there is overkill: the temperature isn’t chang-ing, the function is simply constant. If the object and the room have the sametemperature, K could be any real number since we’re multiplying eKt by zero.Remark: contrast this to Question 3.3.4.9.

857

Page 874: CLP-1 Differential Calculus

SOLUTIONS TO EXERCISES

3.3.4.4. Solution. We want to know when

[T (0)− A]ekt + A = A

That is, when

[T (0)− A]ekt = 0

Since ekt > 0 for all values of k and t, this happens exactly when

T (0)− A = 0

So: if the initial temperature of the object is not the same as the ambient temper-ature, then according to this model, it never will be! (However, as t gets largerand larger, T (t) gets closer and closer to A–it just never exactly reaches there.)If the initial temperature of the object starts out the same as the ambient temper-ature, then T (t) = A for all values of t.

Exercises — Stage 23.3.4.5. Solution. From Newton’s Law of Cooling and Corollary 3.3.8, we knowthe temperature of the copper will be

T (t) = [T (0)− A]eKt + A

where A is the ambient temperature (100), T (0) is the temperature of the copperat time 0 (let’s make this the instant it was dumped in the water, so T (0) = 25),and K is some constant. That is:

T (t) = [25− 100]eKt + 100

= −75eKt + 100

The information given tells us that T (10) = 90, so

90 = −75e10K + 100

75(eK)10

= 10(eK)10

=2

15

eK =

(2

15

) 110

This lets us describe T (t) without any unknown constants.

T (t) = −75(eK)t

+ 100

= −75

(2

15

) t10

+ 100

858

Page 875: CLP-1 Differential Calculus

SOLUTIONS TO EXERCISES

The question asks what value of t gives T (t) = 99.9.

99.9 = −75

(2

15

) t10

+ 100

75

(2

15

) t10

= 0.1(2

15

) t10

=1

750

log

( 2

15

) t10

= log

(1

750

)t

10log

(2

15

)= − log(750)

t =−10 log(750)

log(

215

) ≈ 32.9

It takes about 32.9 seconds.

3.3.4.6. Solution. The temperature of the stone t minutes after taking it fromthe bonfire is

T (t) = [T (0)− A]eKt + A

= [500− 0]eKt + 0

= 500eKt

for some constant K. We are given that T (10) = 100.

100 = T (10) = 500e10K

e10K =1

5

eK = 5−110

This gives us the more complete picture for the temperature of the stone.

T (t) = 500(eK)t

= 500 · 5−t

10

If T (t) = 50 :

50 = T (t) = 500 · 5−t

10

1

10= 10−1 = 5−

t10

10 = 5t

10

log(10) =t

10log(5)

859

Page 876: CLP-1 Differential Calculus

SOLUTIONS TO EXERCISES

t = 10log(10)

log(5)≈ 14.3

So the stone has been out of the fire for about 14.3 minutes.

Exercises — Stage 33.3.4.7. *. Solution.

• First scenario: At time 0, Newton mixes 9 parts coffee at temperature 95 Cwith 1 part cream at temperature 5 C. The resulting mixture has tempera-ture

9× 95 + 1× 5

9 + 1= 86

The mixture cools according to Newton’s Law of Cooling, with initial tem-perature 86 and ambient temperature 22:

T (t) = [86− 22]e−kt + 22

T (t) = 64e−kt + 22

After 10 minutes,

54 = T (10) = 22 + 64e−10k

e−10k =54− 22

64=

1

2

We could compute k from this, but we don’t need it.

• Second scenario: At time 0, Newton gets hot coffee at temperature 95 C. Itcools according to Newton’s Law of Cooling

T (t) = [T (0)− 22]e−kt + 22

In this second scenario, T (0) = 95, so

T (t) = [95− 22]e−kt + 22 = 73e−kt + 22

The value of k is the same as in the first scenario, so after 10 minutes

T (10) = 22 + 73e−10k = 22 + 731

2= 58.5

This cooled coffee is mixed with cold cream to yield a mixture of tempera-ture

9× 58.5 + 1× 5

9 + 1= 53.15

Under the second (add cream just before drinking) scenario, the coffee ends upcooler by 0.85 C .

860

Page 877: CLP-1 Differential Calculus

SOLUTIONS TO EXERCISES

3.3.4.8. *. Solution.

a By Newton’s law of cooling, the rate of change of temperature is propor-tional to the difference between T (t) and the ambient temperature, whichin this case is 30. Thus

dT

dt= k[T (t)− 30]

for some constant of proportionality k. To answer part (a), all we have todo is find k.

Under Newton’s Law of Cooling, the temperature at time t will be given by

T (t) = [T (0)− A]ekt + A

= [5− 30]ekt + 30

= −25ekt + 30

We are told T (5) = 10:

10 = −25e5k + 30

25e5k = 20

e5k =4

55k = log(4/5)

k = 15

log(4/5)

So, the differential equation is

dT

dt(t) =

1

5log(4/5)[T (t)− 30]

b Since T (t) = 30− 25ekt, the temperature of the tea is 20 when

30− 25ekt = 20

ekt =10

25

kt = log

(10

25

)t =

1

klog

2

5

=5 log(2/5)

log(4/5)

≈ 20.53 min

3.3.4.9. Solution. As time goes on, temperatures that follow Newton’s Law ofCooling get closer and closer to the ambient temperature. So, lim

t→∞T (t) exists. In

particular, limt→∞

0.8kt exists.

861

Page 878: CLP-1 Differential Calculus

SOLUTIONS TO EXERCISES

• If k < 0, then limt→∞

0.8kt =∞, since 0.8 < 1. So, k ≥ 0.

• If k = 0, then T (t) = 16 for all values of t. But, in the statement of thequestion, the object is changing temperature. So, k > 0.

Therefore, k is positive.Remark: contrast this to Question 3.3.4.3. The reason we get a different answeris that our base in this question (0.8) is less than one, while the base in Ques-tion 3.3.4.3 (e) is greater than one.Although the given equation T (t) does not exactly look like the Newton’s Lawequations we’re used to, it is equivalent. Remembering elog(0.8) = 0.8:

T (t) = 0.8kt + 15

=(elog 0.8

)kt+ 15

= e(k log 0.8)t + 15

= [16− 15]e(k log 0.8)t + 15

= [16− 15]eKt + 15

with K = k log 0.8. This is now the more familiar form for Newton’s Law ofCooling (with A = 15 and T (0) = 16).Since 0.8 < 1, log(0.8) is negative, so k and K have opposite signs.

· Exercises for § 3.3.3

Exercises — Stage 13.3.4.1. Solution. Since b is a positive constant, lim

t→∞ebt =∞. Therefore:

limt→∞

P (t) = limt→∞

P (0)ebt =

0 if P (0) = 0

∞ if P (0) > 0

If P (0) = 0, then the model simply says “a population that starts with no in-dividuals continues to have no individuals indefinitely,” which certainly makessense. If P (0) 6= 0, then (since we can’t have a negative population) P (0) > 0,and the model says “a population that starts out with some individuals will endup with any gigantically huge number you can think of, given enough time.”This one doesn’t make so much sense. Populations only grow to a certain finiteamount, due to scarcity of resources and such. In the derivation of the Malthu-sian model, we assume a constant net birth rate–that the birth and death rates(per individual) don’t depend on the population, which is not a reasonable as-sumption long-term.

Exercises — Stage 23.3.4.2. Solution. The assumption that the animals grow according to theMalthusian model tells us that their population t years after 2015 is given by

862

Page 879: CLP-1 Differential Calculus

SOLUTIONS TO EXERCISES

P (t) = 121ebt for some constant b, since 121 = P (0), the population 0 years after2015. The information about 2016 tells us

136 = P (1) = 121eb

136

121= eb

This gives us a better idea of P (t):

P (t) = 121ebt = 121

(136

121

)t2020 is 5 years after 2015, so in 2020 (assuming the population keeps growingaccording to the Malthusian model) the population will be

P (5) = 121

(136

121

)5

≈ 217

In 2020, the Malthusian model predicts the herd will number 217 individuals.

3.3.4.3. Solution. Since the initial population of bacteria is 1000 individuals, theMalthusian model says that the population of bacteria t hours after being placedin the dish will be P (t) = 1000ebt for some constant b. Since P (1) = 2000,

2000 = P (1) = 1000eb

2 = eb

So, the population at time t is

P (t) = 1000 · 2t

We want to know at what time the population triples, to 3,000 individuals.

3000 = 1000 · 2t3 = 2t

log(3) = log(2t)

= t log(2)

t =log(3)

log(2)≈ 1.6

The population triples in about 1.6 hours. Note that the tripling time dependsonly on the constant b. In particular, it does not depend on the initial conditionP (0).

3.3.4.4. Solution. According to the Malthusian Model, if the ship wrecked atyear t = 0 and 2 rats washed up on the island, then t years after the wreck, thepopulation of rats will be

P (t) = 2ebt

863

Page 880: CLP-1 Differential Calculus

SOLUTIONS TO EXERCISES

for some constant b. We want to get rid of this extraneous variable b, so we usethe given information. If 1928 is a years after the wreck:

1000 = P (a) = 2eba

1500 = P (a+ 1) = 2eb(a+1) = 2ebaeb

So,

(1000)(eb)

=(2eba

) (eb)

= 1500

Which tells us

eb =1500

1000= 1.5

Now, our model is complete:

P (t) = 2(eb)t

= 2 · 1.5t

Since P (a) = 1000, we can find a:

1000 = P (a) = 2 · 1.5a500 = 1.5a

log(500) = log (1.5a) = a log(1.5)

a =log(500)

log(1.5)≈ 15.3

So, the year 1928 was about 15.3 years after the shipwreck. Since we aren’t givena month when the rats reached exactly 1000 in number, that puts the shipwreckat the year 1912 or 1913.

3.3.4.5. Solution. The Malthusian model suggests that, if we start with P (0)cochineals, their population after 3 months will be

P (t) = P (0)ebt

for some constant b. The constant b is the net birthrate per population memberper unit time. Assuming that the net birthrate for a larger population will be thesame as the test population, we can use the data from the test to find eb. Let Q(t)be the number of individuals in the test population at time t.

Q(t) = Q(0)ebt = 200ebt

1000 = Q(3) = 200e3b

5 = e3b

eb = 51/3

Now that we have an idea of the birthrate, we predict

P (t) = P (0)(eb)t

= P (0) · 5t3

864

Page 881: CLP-1 Differential Calculus

SOLUTIONS TO EXERCISES

We want P (12) = 106 + P (0).

106 + P (0) = P (12) = P (0) · 5123 = P (0) · 54

106 = P (0) · 54 − P (0) = P (0)[54 − 1

]P (0) =

106

54 − 1≈ 1603

The farmer should use an initial population of (at least) about 1603 individuals.Remark: if we hadn’t specified that we need to save P (0) individuals to start nextyear’s population, the number of individual cochineals we would want to startwith to get a million in a year would be 1600–almost the same!

Exercises — Stage 33.3.4.6. Solution.

• [(a)] Since f(t) gives the amount of the radioactive isotope in the sample attime t, the amount of the radioactive isotope in the sample when t = 0 isf(0) = 100e0 = 100 units. Since the sample is decaying, f(t) is decreasing,so f ′(t) is negative. Differentiating, f ′(t) = k(100ekt). Since 100ekt is positiveand f ′(t) is negative, k is negative.

• [(b)] Since f(t) gives the size of the population at time t, the number of in-dividuals in the population when t = 0 is f(0) = 100e0 = 100. Since thepopulation is growing, f(t) is increasing, so f ′(t) is positive. Differentiat-ing, f ′(t) = k(100ekt). Since 100ekt is positive and f ′(t) is positive, k is alsopositive.

• [(c)] Newton’s Law of Cooling give the temperature of an object at time tas f(t) = [f(0)− A]ekt + A, where A is the ambient temperature surround-ing the object. In our case, the ambient temperature is 0 degrees. In anobject whose temperature is being modelled by Newton’s Law of Cooling,it doesn’t matter whether the object is heating or cooling, k is negative. Wesaw this in Question 3.3.4.3 of Section 3.3.2, but it bears repeating. Since f(t)approaches the ambient temperature (in this case, 0) as t goes to infinty:

limt→∞

100ekt = 0

so k is negative.

· Further problems for § 3.3

3.3.4.1. *. Solution. The first piece of information given tells usdf

dx= πf(x). Then

by Theorem 3.3.2,

f(x) = Ceπx

865

Page 882: CLP-1 Differential Calculus

SOLUTIONS TO EXERCISES

for some constant C. The second piece of given information tells us f(0) = 2. Usingthis, we can solve for C:

2 = f(0) = Ce0 = C

Now, we know f(x) entirely:

f(x) = 2eπt

So, we can evaluate f(2)

f(2) = 2e2π

3.3.4.2. Solution. To use Corollary 3.3.8, we re-write the differential equation as

dT

dt= 7

[T −

(−9

7

)].

Now, A = −9

7and K = 7, so we see that the solutions to the differential equation

have the form

T (t) =

[T (0) +

9

7

]e7t − 9

7

for some constant T (0).

We can check that this is reasonable: if

T (t) =

[T (0) +

9

7

]e7t − 9

7

then

dT

dt= 7

[T (0) +

9

7

]e7t

= 7

[T +

9

7

]= 7T + 9.

3.3.4.3. *. Solution. LetQ(t) denote the amount of radioactive material after t days.Then Q(t) = Q(0)ekt. We are told

Q(8) = 0.8Q(0)

So,

Q(0)e8k = 0.8Q(0)

e8k = 0.8

866

Page 883: CLP-1 Differential Calculus

SOLUTIONS TO EXERCISES

ek = 0.818

If Q(0) = 100, the time t at which Q(t) = 40 is determined by

40 = Q(t) = Q(0)ekt = 100ekt = 100

(0.8

18

)t= 100 · 0.8

t8

Solving for t:

40

100= 0.8

t8

log (0.4) = log

(0.8

t8

)=t

8log(0.8)

t =8 log(0.4)

log(0.8)≈ 32.85 days

100 grams will decay to 40 grams in about 32.85 days.

3.3.4.4. Solution. Let t = 0 be the time the boiling water is left in the room, and letT (t) be the temperature of the water t minutes later, so T (0) = 100. Using Newton’sLaw of Cooling, we model the temperature of the water at time t as

T (t) = [100− A]eKt + A

where A is the room temperature, and K is some constant. We are told that T (15) =85 and T (30) = 73, so:

85 = T (15) = [100− A]e15K + A

73 = T (30) = [100− A]e30K + A

Rearranging both equations:

85− A100− A = e15K

73− A100− A = e30K =

(e15K

)2

Using these equations:(85− A100− A

)2

=(e15K

)2= e30K =

73− A100− A

(85− A)2

100− A = 73− A

(85− A)2 = (73− A)(100− A)

852 − 170A+ A2 = 7300− 173A+ A2

173A− 170A = 7300− 852

3A = 75

A = 25

The room temperature is 25 C.

867

Page 884: CLP-1 Differential Calculus

SOLUTIONS TO EXERCISES

3.3.4.5. *. Solution.

a The amount of money at time t obeys

dA

dt= 0.05A+ 2,000 = 0.05[A− (−40,000)]

Using Corollary 3.3.8

A(t) = [A(0) + 40,000]e0.05t − 40,000

= 90,000 · e0.05t − 40,000

where t = 0 corresponds to the year when the graduate is 25.

When the graduate is 65 years old, t = 40, so

A(40) = 90,000 e0.05×40 − 40, 000 ≈ 625, 015.05

b When the graduate stops depositing money and instead starts withdrawingmoney at a rate W , the equation for A becomes

dA

dt= 0.05A−W = 0.05[A(t)− 20W ]

Using Corollary 3.3.8 and assuming that the interest rate remains 5%

A(t) = [A(0)− 20W ]e0.05t + 20W

= [625, 015.05− 20W ]e0.05t + 20W

Note that, for part (b), we only care about what happens when the graduatestarts withdrawing money. We take t = 0 to correspond to the year when thegraduate is 65–so we’re using a different t from part (a). Then from part (a),A(0) = 625, 025.05.

We want the account to be depleted when the graduate is 85. So, we want

0 = A(20)

0 = 20W + e0.05×20(625, 015.05− 20W )

0 = 20W + e(625, 015.05− 20W )

20(e− 1)W = 625, 015.05e

W =625, 015.05e

20(e− 1)≈ $49, 437.96

868

Page 885: CLP-1 Differential Calculus

SOLUTIONS TO EXERCISES

3.3.4.6. *. Solution. 3.3.4.6.a The amount of money at time t obeys

dA

dt= 0.06A− 9,000 = 0.06[A− 150,000]

Using Corollary 3.3.8

A(t) = [A(0)− 150,000]e0.06t + 150,000

= [120,000− 150,000]e0.06t + 150,000

= −30,000e0.06t + 150,000

3.3.4.6.b The money runs out when A(t) = 0.

A(t) = 0

150,000− 30,000 e0.06t = 0

30,000 e0.06t = 150,000

e0.06t = 5

0.06t = log 5

t =log 5

0.06≈ 26.8 yrs

The money runs out in about 26.8 years.Remark: without earning any interest, the money would have run out in about 13.3years.

3.3.4.7. *. Solution. Let Q(t) denote the number of bacteria at time t. We are toldthat Q′(t) = kQ(t) for some constant of proportionality k. Consequently, Q(t) =Q(0)ekt (Corollary 3.3.2). We are also told

Q(9) = 3Q(0)

So, Q(0)e9k = 3Q(0)

e9k = 3

ek = 319

The doubling time t obeys:

Q(t) = 2Q(0)

So, Q(0)ekt = 2Q(0)

ekt = 2

3t9 = 2

t

9log 3 = log 2

t = 9log 2

log 3≈ 5.68 hr

869

Page 886: CLP-1 Differential Calculus

SOLUTIONS TO EXERCISES

3.3.4.8. *. Solution. (a) We want our differential equation to have the format of theequation in Corollary 3.3.8:

dv

dt(t) = −g − kv(t)

= −k(v(t) +

g

k

)= −k

(v(t)−

(−gk

))So, we can use the corollary, with K = −k, T = v, and A = −g

k.

v(t) =(v(0)−

(−gk

))e−kt − g

k

=(v0 +

g

k

)e−kt − g

k

(b)

limt→∞

v(t) = limt→∞

[(v0 +

g

k

)e−kt − g

k

]=(v0 +

g

k

)(limt→∞

e−kt)− g

k

Since k is positive:

=(v0 +

g

k

)(0)− g

k

= −gk

Remark: This means, as the object falls, instead of accelerating without bound, itapproaches some maximum speed. The velocity is negative because the object ismoving in the negative direction–downwards.

3.4 · Approximating Functions Near a Specified Point —Taylor Polynomials3.4.11 · Exercises· Exercises for § 3.4.1

Exercises — Stage 13.4.11.1. Solution. Since f(0) is closer to g(0) than it is to h(0), you would prob-ably want to estimate f(0) ≈ g(0) = 1+2 sin(1) if you had the means to efficientlyfigure out what sin(1) is, and if you were concerned with accuracy. If you hada calculator, you could use this estimation. Also, later in this chapter we willlearn methods of approximating sin(1) that do not require a calculator, but theydo require time.Without a calculator, or without a lot of time, using f(0) ≈ h(0) = 0.7 probably

870

Page 887: CLP-1 Differential Calculus

SOLUTIONS TO EXERCISES

makes the most sense. It isn’t as accurate as f(0) ≈ g(0), but you get an estimatevery quickly, without worrying about figuring out what sin(1) is.Remark: when you’re approximating something in real life, there probably won’tbe an obvious “correct” way to do it. There’s usually a trade-off between accu-racy and ease.

Exercises — Stage 23.4.11.2. Solution. 0.93 is pretty close to 1, and we know log(1) = 0, so weestimate log(0.93) ≈ log(1) = 0.

x

y

y = f(x)

y = 0

0.93

1

3.4.11.3. Solution. We don’t know arcsin(0.1), but 0.1 is reasonably close to 0,and arcsin(0) = 0. So, we estimate arcsin(0.1) ≈ 0.

3.4.11.4. Solution. We don’t know tan(1), but we do know tan(π

3

)=√

3. Sinceπ

3≈ 1.047 is pretty close to 1, we estimate

√3 tan(1) ≈

√3 tan

(π3

)=(√

3)2

= 3.

Exercises — Stage 33.4.11.5. Solution. Since 10.1 is pretty close to 10, we estimate 10.13 ≈ 103 =1000.Remark: these kinds of approximations are very useful when you are doing com-putations. It’s easy to make a mistake in your work, and having in mind that10.13 should be about a thousand is a good way to check that whatever answeryou have makes sense.

· Exercises for § 3.4.2

Exercises — Stage 13.4.11.1. Solution. The linear approximation is L(x) = 3x − 9. Since we’reapproximating at x = 5, f(5) = L(5), and f ′(5) = L′(5). However, there is noguarantee that f(x) and L(x) have the same value when x 6= 5. So:(a) f(5) = L(5) = 6(b) f ′(5) = L′(5) = 3(c) there is not enough information to find f(0).

871

Page 888: CLP-1 Differential Calculus

SOLUTIONS TO EXERCISES

3.4.11.2. Solution. The linear approximation is a line, passing through (2, f(2)),with slope f ′(2). That is, the linear approximation to f(x) about x = 2 is thetangent line to f(x) at x = 2. It is shown below in red.

x

y

y = f(x)

2

3.4.11.3. Solution. For any constant a, f(a) = (2a + 5), and f ′(a) = 2, so ourapproximation gives us

f(x) ≈ (2a+ 5) + 2(x− a) = 2x+ 5

Since f(x) itself is a linear function, the linear approximation is actually just f(x)itself. As a consequence, the linear approximation is perfectly accurate for allvalues of x.

Exercises — Stage 23.4.11.4. Solution. We have no idea what f(0.93) is, but 0.93 is pretty close to1, and we definitely know f(1). The linear approximation of f(x) about x = 1 isgiven by

f(x) ≈ f(1) + f ′(1)(x− 1)

So, we calculate:

f(1) = log(1) = 0

f ′(x) =1

x

f ′(1) =1

1= 1

Therefore,

f(x) ≈ 0 + 1(x− 1) = x− 1

When x = 0.93:

f(0.93) ≈ 0.93− 1 = −0.07

872

Page 889: CLP-1 Differential Calculus

SOLUTIONS TO EXERCISES

x

y

y = f(x)

y = x− 1

1

3.4.11.5. Solution. We approximate the function f(x) =√x about x = 4, since

4 is a perfect square and it is close to 5.

f(4) =√

4 = 2

f ′(x) =1

2√x

⇒ f ′(4) =1

2√

4=

1

4

f(x) ≈ f(4) + f ′(4)(x− 4) = 2 +1

4(x− 4)

f(5) ≈ 2 +1

4(5− 4) =

9

4

We estimate√

5 ≈ 9

4.

Remark:(

9

4

)2

=81

16, which is pretty close to

80

16= 5. Our approximation seems

pretty good.

3.4.11.6. Solution. We approximate the function f(x) = 5√x. We need to centre

the approximation about some value x = a such that we know f(a) and f ′(a),and a is not too far from 30.

f(x) = 5√x = x

15

f ′(x) =1

5x−

45 =

1

5 5√x

4

a needs to be a number whose fifth root we know. Since 5√

32 = 2, and 32 isreasonably close to 30, a = 32 is a great choice.

f(32) =5√

32 = 2

f ′(32) =1

5 · 24=

1

80

The linear approximation of f(x) about x = 32 is

f(x) ≈ 2 +1

80(x− 32)

When x = 30:

f(30) ≈ 2 +1

80(30− 32) = 2− 1

40=

79

40

873

Page 890: CLP-1 Differential Calculus

SOLUTIONS TO EXERCISES

We estimate 5√

30 ≈ 79

40.

Remark:79

40= 1.975, while 5

√30 ≈ 1.97435. This is a decent estimation.

Exercises — Stage 33.4.11.7. Solution. If f(x) = x3, then f(10.1) = 10.13, which is the value wewant to estimate. Let’s take the linear approximation of f(x) about x = 10:

f(10) = 103 = 1000

f ′(x) = 3x2

f ′(10) = 3(102) = 300

f(a) ≈ f(10) + f ′(10)(x− 10)

= 1000 + 300(x− 10)

f(10.1) ≈ 1000 + 300(10.1− 10) = 1030

We estimate 10.13 ≈ 1030. If we calculate 10.13 exactly (which is certainly possibleto do by hand), we get 1030.301.Remark: in the previous subsection, we used a constant approximation to esti-mate 10.13 ≈ 1000. That approximation was easy to do in your head, in a matterof seconds. The linear approximation is more accurate, but not much faster thansimply calculating 10.13.

3.4.11.8. Solution. There are many possible answers. One is:

f(x) = sinx a = 0 b = π

We know that f(π) = 0 and f(0) = 0. Using a constant approximation of f(x)about x = 0, our estimation is f(π) ≈ f(0) = 0, which is exactly the correct value.However, is we make a linear approximation of f(x) about x = 0, we get

f(π) ≈ f(0) + f ′(0)(π − 0) = sin(0) + cos(0)π = π

which is not exactly the correct value.

874

Page 891: CLP-1 Differential Calculus

SOLUTIONS TO EXERCISES

x

y

π y = f(x)

const

linear

Remark: in reality, we wouldn’t estimate sin(π), because we know its value ex-actly. The purpose of this problem is to demonstrate that fancier approximationsare not always more accurate. At the of this section, we’ll talk about how to boundthe error of your estimations, to make sure that you are finding something suffi-ciently accurate.

3.4.11.9. Solution. The linear approximation L(x) of f(x) about x = a is chosenso that L(a) = f(a) and L′(a) = f ′(a). So,

L′(a) = f ′(a) =1

1 + a2

1

4=

1

1 + a2

a = ±√

3

We’ve narrowed down a to√

3 or −√

3. Recall the linear approximation of f(x)about x = a is f(a)+f ′(a)(x−a), so its constant term is f(a)−af ′(a) = arctan(a)−a

1 + a2. We compute this for a =

√3 and a = −

√3.

a =√

3 : arctan (a)− a

1 + a2= arctan

(√3)−

√3

1 +(√

3)2

3−√

3

4=

4π −√

27

12

a = −√

3 : arctan (a)− a

1 + a2= arctan

(−√

3)− −

√3

1 +(−√

3)2

= −π3

+

√3

4=−4π +

√27

12

875

Page 892: CLP-1 Differential Calculus

SOLUTIONS TO EXERCISES

So, when a =√

3,

L(x) =1

4x+

4π −√

27

12

and this does not hold when a = −√

3. We conclude a =√

3.

· Exercises for § 3.4.3

Exercises — Stage 13.4.11.1. Solution. If Q(x) is the quadratic approximation of f about 3, thenQ(3) = f(3), Q′(3) = f ′(3), and Q′′(3) = f ′′(3). There is no guarantee that f(x)and Q(x) share the same third derivative, though, so we do not have enoughinformation to know f ′′′(3).

f(3) = −32 + 6(3) = 9

f ′(3) =d

dx

−x2 + 6x

∣∣∣∣x=3

= −2x+ 6|x=3 = 0

f ′′(3) =d2

dx2

−x2 + 6x

∣∣∣∣x=3

=d

dx−2x+ 6

∣∣∣∣x=3

= −2

3.4.11.2. Solution. The quadratic approximation of f(x) about x = a is

f(x) ≈ f(a) + f ′(a)(x− a) +1

2f ′′(a)(x− a)2

We subsitute f(a) = 2a+ 5, f ′(a) = 2, and f ′′(a) = 0:

f(x) ≈ (2a+ 5) + 2(x− a) = 2x+ 5

So, our approximation is f(x) ≈ 2x+ 5.Remark: Our approximation is exact for every value of x. This will always hap-pen with a quadratic approximation of a function that is quadratic, linear, orconstant.

Exercises — Stage 23.4.11.3. Solution. We approximate the function f(x) = log x about the pointx = 1. We choose 1 because it is close to 0.93, and we can evaluate f(x) and itsfirst two derivatives at x = 1.

f(1) = 0

f ′(x) =1

x⇒ f ′(1) = 1

f ′′(x) =−1

x2⇒ f ′′(1) = −1

876

Page 893: CLP-1 Differential Calculus

SOLUTIONS TO EXERCISES

So,

f(x) ≈ f(1) + f ′(1)(x− 1) +1

2f ′′(1)(x− 1)2

= 0 + (x− 1)− 1

2(x− 1)2

When x = 0.93:

f(0.93) ≈ (0.93− 1)− 1

2(0.93− 1)2 = −0.07− 1

2(0.0049)

= −0.07245

We estimate log(0.93) ≈ −0.07245.Remark: a calculator approximates log(0.93) ≈ −0.07257. We’re pretty close.

3.4.11.4. Solution. We approximate the function f(x) = cosx. We can easilyevaluate cosx and sinx (sinx will appear in the first derivative) at x = 0, and 0 is

quite close to1

15, so we centre our approximation about x = 0.

f(0) = 1

f ′(x) = − sinx

f ′(0) = − sin(0) = 0

f ′′(x) = − cosx

f ′′(0) = − cos(0) = −1

Using the quadratic approximation f(x) ≈ f(0) + f ′(0)(x− 0) + 12f ′′(0)(x− 0)2:

f(x) ≈ 1− 1

2x2

f

(1

15

)≈ 1− 1

2 · 152=

449

450

We approximate cos

(1

15

)≈ 449

450.

Remark:449

450= 0.9977, while a calculator gives cos

(115

)≈ 0.9977786. Our ap-

proximation has an error of about 0.000001.

3.4.11.5. Solution. The quadratic approximation of a function f(x) about x = ais

f(x) ≈ f(a) + f ′(a)(x− a) +1

2f ′′(a)(x− a)2

We compute derivatives.

f(0) = e0 = 1

f ′(x) = 2e2x

877

Page 894: CLP-1 Differential Calculus

SOLUTIONS TO EXERCISES

f ′(0) = 2e0 = 2

f ′′(x) = 4e2x

f ′′(0) = 4e0 = 4

Substituting:

f(x) ≈ 1 + 2(x− 0) +4

2(x− 0)2

f(x) ≈ 1 + 2x+ 2x2

3.4.11.6. Solution. There are a few functions we could choose to approximate.For example:

• f(x) = x4/3. In this case, we would probably choose to approximate f(x)about x = 8 (since 8 is a cube, 84/3 = 24 = 16 is something we can evaluate)or x = 1.

• f(x) = 5x. We can easily figure out f(x) when x is a whole number, sowe would want to centre our approximation around some whole numberx = a, but then f ′(a) = 5a log(5) gives us a problem: without a calculator,it’s hard to know what log(5) is.

• Since 54/3 = 5 3√

5, we can use f(x) = 5 3√x. As in the first bullet, we would

centre about x = 8, or x = 1.

There isn’t much difference between the first and third bullets. We’ll go withf(x) = 5 3

√x, centred about x = 8.

f(x) = 5x13 ⇒ f(8) = 5 · 2 = 10

f ′(x) =5

3x−

23 ⇒ f ′(8) =

5

3

(2−2)

=5

12

f ′′(x) =5

3

(−2

3

)x−

53 = −10

9x−

53 ⇒ f ′′(8) = −10

9

(2−5)

= − 5

144

Using the quadratic approximation f(x) ≈ f(a) + f ′(a)(x− a) + 12f ′′(a)(x− a)2:

f(x) ≈ 10 +5

12(x− 8)− 5

288(x− 8)2

f(5) ≈ 10 +5

12(−3)− 5

288(9) =

275

32

We estimate 54/3 ≈ 275

32

Remark:275

32= 8.59375, and a calculator gives 54/3 ≈ 8.5499. Although 5 and 8

are somewhat far apart, our estimate is only off by about 0.04.

3.4.11.7. Solution.

• [3.4.11.7.a] For every value of n, the term being added is simply the constant

878

Page 895: CLP-1 Differential Calculus

SOLUTIONS TO EXERCISES

1. So,30∑n=5

1 = 1+1+· · ·+1. The trick is figuring out how many 1s are added.

Our index n takes on all integers from 5 to 30, including 5 and 30, which is

26 numbers. So,30∑n=5

= 26.

If you’re having a hard time seeing why the sum is 26, instead of25, think of it this way: there are thirty numbers in the collection1, 2, 3, 4, 5, 6, . . . , 29, 30. If we remove the first four, we get 30 − 4 = 26numbers in the collection 5, 6, . . . , 30.

• [3.4.11.7.b]

3∑n=1

[2(n+ 3)− n2

]= 2(1 + 3)− 12︸ ︷︷ ︸

n=1

+ 2(2 + 3)− 22︸ ︷︷ ︸n=2

+ 2(3 + 3)− 32︸ ︷︷ ︸n=3

= 8− 1 + 10− 4 + 12− 9 = 16

• [3.4.11.7.c]

10∑n=1

[1

n− 1

n+ 1

]=

1

1− 1

1 + 1︸ ︷︷ ︸n=1

+1

2− 1

2 + 1︸ ︷︷ ︸n=2

+1

3− 1

3 + 1︸ ︷︷ ︸n=3

+1

4− 1

4 + 1︸ ︷︷ ︸n=4

+1

5− 1

5 + 1︸ ︷︷ ︸n=5

+1

6− 1

6 + 1︸ ︷︷ ︸n=6

+1

7− 1

7 + 1︸ ︷︷ ︸n=7

+1

8− 1

8 + 1︸ ︷︷ ︸n=8

+1

9− 1

9 + 1︸ ︷︷ ︸n=9

+1

10− 1

10 + 1︸ ︷︷ ︸n=10

Most of these cancel!

=1

1−1

2+

1

2︸ ︷︷ ︸0

−1

3+

1

3︸ ︷︷ ︸0

−1

4+

1

4︸ ︷︷ ︸0

−1

5+

1

5︸ ︷︷ ︸0

−1

6+

1

6︸ ︷︷ ︸0

−1

7+

1

7︸ ︷︷ ︸0

−1

8+

1

8︸ ︷︷ ︸0

−1

9+

1

9︸ ︷︷ ︸0

− 1

10+

1

10︸ ︷︷ ︸0

− 1

11

= 1− 1

11=

10

11

879

Page 896: CLP-1 Differential Calculus

SOLUTIONS TO EXERCISES

• [3.4.11.7.d]

4∑n=1

5 · 2n4n+1

= 54∑

n=1

2n

4 · 4n =5

4

4∑n=1

2n

4n=

5

4

4∑n=1

1

2n

=5

4

1

2︸︷︷︸n=1

+1

4︸︷︷︸n=2

+1

8︸︷︷︸n=3

+1

16︸︷︷︸n=4

=75

64

3.4.11.8. Solution. For each of these, there are many solutions. We providesome below.

a 1 + 2 + 3 + 4 + 5 =5∑

n=1

n

b 2 + 4 + 6 + 8 =4∑

n=1

2n

c 3 + 5 + 7 + 9 + 11 =5∑

n=1

(2n+ 1)

d 9 + 16 + 25 + 36 + 49 =7∑

n=3

n2

e 9 + 4 + 16 + 5 + 25 + 6 + 36 + 7 + 49 + 8 =7∑

n=3

(n2 + n+ 1)

f 8 + 15 + 24 + 35 + 48 =7∑

n=3

(n2 − 1)

g 3− 6 + 9− 12 + 15− 18 =6∑

n=1

(−1)n+13n

Remark: if we had written (−1)n instead of (−1)n+1, with everything elsethe same, the signs would have been reversed.

Exercises — Stage 33.4.11.9. Solution. Let’s start by taking the first two derivative of f(x).

f(x) = 2 arcsinx ⇒ f(0) = 2(0) = 0

f ′(x) =2√

1− x2⇒ f ′(0) =

2

1= 2

f ′′(x) =d

dx

2(1− x2)−

12

880

Page 897: CLP-1 Differential Calculus

SOLUTIONS TO EXERCISES

= 2(−1

2

)(1− x2)−

32 (−2x) (chain rule)

=2x(√

1− x2)3 ⇒ f ′′(0) = 0

Now, we can find the quadratic approximation about x = 0.

f(x) ≈ f(0) + f ′(0)x+1

2f ′′(0)x2

= 2x

f(1) ≈ 2

Our quadratic approximation gives 2 arcsin(1) ≈ 2. However, 2 arcsin(1) is ex-actly equal to 2

(π2

)= π. We’ve just made the rather unfortunate approximation

π ≈ 2.

3.4.11.10. Solution. From the text, the quadratic approximation of ex about x =0 is

ex ≈ 1 + x+1

2x2

So,

e = e1 ≈ 1 + 1 +1

2= 2.5

We estimate e ≈ 2.5.Remark: actually, e ≈ 2.718.

3.4.11.11. Solution.

• First, we’ll show that 3.4.11.11.a,3.4.11.11.d, 3.4.11.11.e are equivalent:

3.4.11.11.d = 210∑n=1

n = 2(1 + 2 + · · ·+ 10)

= 2(1) + 2(2) + · · ·+ 2(10) =10∑n=1

2n

= 3.4.11.11.a

So 3.4.11.11.a and 3.4.11.11.d are equivalent.

3.4.11.11.e = 211∑n=2

(n− 1) = 2(1 + 2 + · · ·+ 10) = 3.4.11.11.d

So 3.4.11.11.e and 3.4.11.11.d are equivalent.

• Second, we’ll show that 3.4.11.11.b and 3.4.11.11.g are equivalent.

3.4.11.11.g =1

4

10∑n=1

(4n+1

2n

)=

1

4

10∑n=1

(4 · 4n

2n

)

881

Page 898: CLP-1 Differential Calculus

SOLUTIONS TO EXERCISES

=4

4

10∑n=1

(4n

2n

)=

10∑n=1

(4

2

)n=

10∑n=1

2n = 3.4.11.11.b

• Third, we’ll show that 3.4.11.11.c and 3.4.11.11.f are equivalent.

3.4.11.11.f =14∑n=5

(n− 4)2 = 12 + 22 + · · ·+ 102 =10∑n=1

n2

= 3.4.11.11.c

• Now, we have three groups, where each group consists of equivalent ex-pressions. To be quite thorough, we should show that no two of thesegroups contain expressions that are secretly equivalent. They would behard to evaluate, but we can bound them and show that no two expres-sions in two separate groups could possibly be equivalent. Notice that

10∑n=1

2n = 21 + 22 + · · ·+ 210 > 210 = 1024

10∑n=1

n2 <10∑n=1

102 = 10(100) = 1000

10∑n=1

n2 = 12 + 22 + · · · 82 + 92 + 102 > 82 + 92 + 102 = 245

10∑n=1

2n <10∑n=1

20 = 200

The expressions in the blue group add to less than 200, but the expressionsin the green group add to more than 245, and the expressions in the redgroup add to more than 1024, so the blue groups expressions can’t possiblysimplify to the same number as the red and green group expressions.

The expressions in the green group add to less than 1000. Since the expres-sions in the red group add to more than 1024, the expressions in the greenand red groups can’t possibly simplify to the same numbers.

We group our expressions in to collections of equivalent expressions as follows:

• [3.4.11.11.a=3.4.11.11.d=3.4.11.11.e] and

• [3.4.11.11.b=3.4.11.11.g], and

• [3.4.11.11.c=3.4.11.11.f].

882

Page 899: CLP-1 Differential Calculus

SOLUTIONS TO EXERCISES

· Exercises for § 3.4.4

Exercises — Stage 13.4.11.1. Solution. Since T3(x) is the third-degree Taylor polynomial for f(x)about x = 1:

• T3(1) = f(1)

• T ′3(1) = f ′(1)

• T ′′3 (1) = f ′′(1)

• T ′′′3 (1) = f ′′′(1)

In particular, f ′′(1) = T ′′3 (1).

T ′3(x) = 3x2 − 10x+ 9

T ′′3 (x) = 6x− 10

T ′′3 (1) = 6− 10 = −4

So, f ′′(1) = −4.

3.4.11.2. Solution. In Question 3.4.11.1, we differentiated the Taylor polynomialto find its derivative. We don’t really want to differentiate this ten times, though,so let’s look for another way. Unlike Question 3.4.11.1, our Taylor polynomialis given to us in a form very similar to its definition. The nth degree Taylorpolynomial for f(x) about x = 5 is

Tn(x) =n∑k=0

f (k)(5)

k!(x− 5)k

So,

n∑k=0

f (k)(5)

k!(x− 5)k =

n∑k=0

2k + 1

3k − 9(x− 5)k

For any k from 0 to n,

f (k)(5)

k!=

2k + 1

3k − 9

In particular, when k = 10,

f (10)(5)

10!=

20 + 1

30− 9= 1

f (10)(5) = 10!

883

Page 900: CLP-1 Differential Calculus

SOLUTIONS TO EXERCISES

Exercises — Stage 33.4.11.3. Solution. The fourth-degree Maclaurin polynomial for f(x) is

T4(x) = f(0) + f ′(0)x+1

2f ′′(0)x2 +

1

3!f ′′′(0)x3 +

1

4!f (4)(0)x4

while the third-degree Maclaurin polynomial for f(x) is

T3(x) = f(0) + f ′(0)x+1

2f ′′(0)x2 +

1

3!f ′′′(0)x3

So, we simply “chop off” the part of T4(x) that includes x4:

T3(x) = −x3 + x2 − x+ 1

3.4.11.4. Solution. We saw this kind of problem in Question 3.4.11.3. Thefourth-degree Taylor polynomial for f(x) about x = 1 is

T4(x) = f(1) + f ′(1)(x− 1) +1

2f ′′(1)(x− 1)2 +

1

3!f ′′′(1)(x− 1)3

+1

4!f (4)(1)(x− 1)4

while the third-degree Maclaurin polynomial for f(x) about x = 1 is

T3(x) = f(1) + f ′(1)(x− 1) +1

2f ′′(1)(x− 1)2 +

1

3!f ′′′(1)(x− 1)3

So, we “chopped off” the term of degree 4 to get T3(x). However, our polynomialis not in this form. It’s not clear, right away, what the term f (4)(x − 1)4 is in ourgiven T4(x). So, we will use a different method from Question 3.4.11.3.One option is to do some fancy factoring to get T4(x) into the more standard formof a Taylor polynomial. Another option (which we will use) is to recover f(1),f ′(1), f ′′(1), and f ′′′(1) from T4(x).

Recall that T4(x) and f(x) have the same values at x = 1 (although maybe notanywhere else!), and they also have the same first, second, third, and fourthderivatives at x = 1 (but again, maybe not anywhere else, and maybe their fifthderivatives don’t agree). This tells us the following:

T4(x) = x4 + x3 − 9 ⇒ f(1) = T4(1) = −7

T ′4(x) = 4x3 + 3x2 ⇒ f ′(1) = T ′4(1) = 7

T ′′4 (x) = 12x2 + 6x ⇒ f ′′(1) = T ′′4 (1) = 18

T ′′′4 (x) = 24x+ 6 ⇒ f ′′′(1) = T ′′′4 (1) = 30

Now, we can write the third-degree Taylor polynomial for f(x) about x = 1:

T3(x) = −7 + 7(x− 1) +1

2(18)(x− 1)2 +

1

3!(30)(x− 1)3

884

Page 901: CLP-1 Differential Calculus

SOLUTIONS TO EXERCISES

= −7 + 7(x− 1) + 9(x− 1)2 + 5(x− 1)3

Remark: expanding the expression above, we get the equivalent polynomialT3(x) = 5x3 − 6x2 + 4x − 10. From this, it is clear that we can’t just “chop off”the term with x4 to change T4(x) into T3(x) when the Taylor polynomial is notcentred about x = 0.

3.4.11.5. Solution. The nth degree Taylor polynomial for f(x) about x = 5 is

Tn(x) =n∑k=0

1

k!f (k)(5)(x− 5)k

We expand this somewhat:

Tn(x) = f(5) + f ′(x− 5) + · · ·+ 1

10!f (10)(5)(x− 5)10 + · · ·

+1

n!f (n)(5)(x− 1)n

So, the coefficient of (x − 5)10 is1

10!f (10)(5). Expanding the given form of the

Taylor polynomial:

Tn(x) =

n/2∑k=0

2k + 1

3k − 9(x− 5)2k

=1

−9︸︷︷︸k=0

+3

−6(x− 5)2︸ ︷︷ ︸k=1

+ · · ·+ 11

6(x− 5)10︸ ︷︷ ︸k=5

+ · · ·

+n+ 1

(3/2)n− 9(x− 5)n︸ ︷︷ ︸

k=n/2

Equating the coefficients of (x− 5)10 in the two expressions:

1

10!f (10)(5) =

11

6

f (10)(5) =11 · 10!

6

3.4.11.6. Solution. Since T3(x) is the third-degree Taylor polynomial for f(x)about x = a, we know the following things to be true:

• f(a) = T3(a)

• f ′(a) = T ′3(a)

• f ′′(a) = T ′′3 (a)

• f ′′′(a) = T ′′′3 (a)

885

Page 902: CLP-1 Differential Calculus

SOLUTIONS TO EXERCISES

But, some of these don’t look super useful. For instance, if we try to use the firstbullet, we get this equation:

a3

[2 log a− 11

3

]= −2

3

√e3 + 3ea− 6

√ea2 + a3

Solving this would be terrible. Instead, let’s think about how the equations lookwhen we move further down the list. Since T3(x) is a cubic equation, T ′′′3 (x) is aconstant (and so T ′′′3 (a) does not depend on a). That sounds like it’s probably thesimplest option. Let’s start differentiating. We’ll need to know both f ′′′(a) andT ′′′3 (a).

f(x) = x3

[2 log x− 11

3

]f ′(x) = x3

[2

x

]+ 3x2

[2 log x− 11

3

]= 6x2 log x− 9x2

f ′′(x) = 6x2 · 1

x+ 12x log x− 18x = 12x log x− 12x

f ′′′(x) = 12x · 1

x+ 12 log x− 12 = 12 log x

f ′′′(a)= 12 log a

Now, let’s move to the Taylor polynomial. Remember that e is a constant.

T3(x) = −2

3

√e3 + 3ex− 6

√ex2 + x3

T ′3(x) = 3e− 12√ex+ 3x2

T ′′3 (x) = −12√e+ 6x

T ′′′3 (x) = 6

T ′′′3 (a)= 6

The final bullet point gives us the equation:

f ′′′(a) = T ′′′3 (a)

12 log a = 6

log a =1

2

a = e12

So, a =√e.

· Exercises for § 3.4.5

Exercises — Stage 1

886

Page 903: CLP-1 Differential Calculus

SOLUTIONS TO EXERCISES

3.4.11.1. Solution. If we were to find the 16th degree Maclaurin polynomialfor a generic function, we might expect to have to differentiate 16 times (ugh).But, we know that the derivatives of sines and cosines repeat themselves. So, it’senough to figure out the pattern:

f(x) = sin x+ cosx f(0) = 1

f ′(x) = cos x− sinx f ′(0) = 1

f ′′(x) = − sinx− cosx f ′′(0) = −1

f ′′′(x) = − cosx+ sinx f ′′′(0) = −1

f (4) = sinx+ cosx f (4)(0) = 1

Since f (4)(x) = f(x), our derivatives repeat from here. They follow the pattern:+1, +1, −1, −1.

T16(x) = 1+x−1

2x2− 1

3!x3+

1

4!x4+

1

5!x5− 1

6!x6− 1

7!x7+

1

8!x8+

1

9!x9

− 1

10!x10− 1

11!x11+

1

12!x12+

1

13!x13− 1

14!x14− 1

15!x15

+1

16!x16

3.4.11.2. Solution. A Taylor polynomial gives a polynomial approximation fora function s(t). Since s(t) is itself a polynomial, any nth-degree Taylor polyno-mial, with n greater than or equal to the degree of s(t), will simply give s(t). So,in our case, T100(t) = s(t) = 4.9t2 − t+ 10.If that isn’t satisfying, you can go through the normal method of calculatingT100(t).

s(t) = 4.9t2 − t+ 10 s(5) = 4.9(25)− 5 + 10 = 4.9(25) + 5

s′(t) = 9.8t− 1 s′(5) = 9.8(5)− 1

s′′(t) = 9.8 s′′(5) = 9.8

The rest of the derivatives of s(t) are identically zero, so they are (in particular)zero when t = 5. Therefore,

T100(t) = 4.9(25) + 5 + (9.8(5)− 1)(t− 5) +1

29.8(t− 5)2

= 4.9(25) + 5 + (9.8(5)− 1)(t− 5) + 4.9(t2 − 10t+ 25)

= [4.9(25) + 5 + (9.8(5)− 1)(−5) + 4.9(25)]

+ [9.8(5)− 1− 4.9(10)]t+ 4.9t2

= 10− t+ 4.9t2 = s(t)

3.4.11.3. Solution. Let’s start by differentiating f(x) and looking for a pattern.Remember that log 2 = loge 2 is a constant number.

f(x) = 2x

f ′(x) = 2x log 2

887

Page 904: CLP-1 Differential Calculus

SOLUTIONS TO EXERCISES

f ′′(x) = 2x (log 2)2

f (3)(x) = 2x (log 2)3

f (4)(x) = 2x (log 2)4

f (5)(x) = 2x (log 2)5

So, in general,

f (k)(x) = 2x (log 2)k

We notice that this formula works even when k = 0 and k = 1. When x = 1,

f (k)(1) = 2 (log 2)k

The nth degree Taylor polynomial of f(x) about x = 1 is

Tn(x) =n∑k=0

f (k)(1)

k!(x− 1)k

=n∑k=0

2(log 2)k

k!(x− 1)k

3.4.11.4. Solution. We need to know the first six derivatives of f(x) at x = 1.Let’s get started.

f(x) = x2 log x+ 2x2 + 5 f(1)= 7

f ′(x) = (x2)1

x+ (2x) log x+ 4x

= 2x log x+ 5x f ′(1)= 5

f ′′(x) = (2x)1

x+ (2) log x+ 5

= 2 log x+ 7 f ′′(1)= 7

f ′′′(x) = 2x−1 f ′′′(1)= 2

f (4) = −2x−2 f (4)(1)= −2

f (5) = 4x−3 f (5)(1)= 4

f (6) = −12x−4 f (6)(1)= −12

Now, we can plug in.

T6(x) = f(1) + f ′(1)(x− 1) +1

2f ′′(1)(x− 1)2 +

1

3!f ′′′(1)(x− 1)3

+1

4!f (4)(1)(x− 1)4 +

1

5!f (5)(1)(x− 1)5 +

1

6!f (6)(1)(x− 1)6

= 7 + 5(x− 1) +1

2(7)(x− 1)2 +

1

3!(2)(x− 1)3

+1

4!(−2)(x− 1)4 +

1

5!(4)(x− 1)5 +

1

6!(−12)(x− 1)6

888

Page 905: CLP-1 Differential Calculus

SOLUTIONS TO EXERCISES

= 7 + 5(x− 1) +7

2(x− 1)2 +

1

3(x− 1)3 − 1

12(x− 1)4

+1

30(x− 1)5 − 1

60(x− 1)6

3.4.11.5. Solution. We’ll start by differentiating and looking for a pattern.

f(x) =1

1− x = (1− x)−1

Using the chain rule,

f ′(x) = (−1)(1− x)−2(−1) = (1− x)−2

f ′′(x) = (−2)(1− x)−3(−1) = 2(1− x)−3

f (3)(x) = (−3)(2)(1− x)−4(−1) = 2(3)(1− x)−4

f (4)(x) = (−4)(2)(3)(1− x)−5(−1) = 2(3)(4)(1− x)−5

f (5)(x) = (−5)(2)(3)(4)(1− x)−6(−1) = 2(3)(4)(5)(1− x)−6

Recognizing the pattern,

f (k)(x) = k!(1− x)−(k+1)

f (k)(0) = k!(1)−(k+1) = k!

The nth degree Maclaurin polynomial for f(x) is

Tn(x) =n∑k=0

f (k)(0)

k!xk

=n∑k=0

k!

k!xk

=n∑k=0

xk

Exercises — Stage 33.4.11.6. Solution. We’ll need to know the first three derivatives of xx at x = 1.This is a good review of logarithmic differentiation, covered in Section 2.10.

f(x) = xx

=⇒ f(1)= 1

log(f(x)) = log (xx) = x log x

d

dxlog(f(x)) =

d

dxx log x

f ′(x)

f(x)= x · 1

x+ log x = 1 + log x

889

Page 906: CLP-1 Differential Calculus

SOLUTIONS TO EXERCISES

So

f ′(x) = xx [1 + log x]

=⇒ f ′(1)= 1

f ′′(x) =d

dxxx [1 + log x] + xx

d

dx1 + log x

= (xx [1 + log x]) [1 + log x] + xx · 1

x

= xx(

(1 + log x)2 +1

x

)=⇒ f ′′(1)= 2

f ′′′(x) =d

dxxx

((1 + log x)2 +

1

x

)+ xx

d

dx

(1 + log x)2 +

1

x

= xx [1 + log x]

((1 + log x)2 +

1

x

)+ xx

[2

x(1 + log x)− 1

x2

]= xx

((1 + log x)3 +

3

x(1 + log x)− 1

x2

)=⇒ f ′′′(1)= 3

Now, we can plug in:

T3(x) = f(1) + f ′(1)(x− 1) +1

2f ′′(1)(x− 1)2 +

1

3!f ′′′(1)(x− 1)3

= 1 + 1(x− 1) +1

2(2)(x− 1)2 +

1

6(3)(x− 1)3

= 1 + (x− 1) + (x− 1)2 +1

2(x− 1)3

3.4.11.7. Solution. We note that 6 arctan

(1√3

)= 6

(π6

)= π. We will

find the 5th-degree Maclaurin polynomial T5(x) for f(x) = 6 arctanx. Then

π = f

(1√3

)≈ T5

(1√3

). Let’s begin by finding the first five derivatives of

f(x) = 6 arctanx.

f(x) = 6 arctanx

=⇒ f(0) = 0

f ′(x) = 6

(1

1 + x2

)=⇒ f ′(0) = 6

f ′′(x) = 6

(0− 2x

(1 + x2)2

)= −12

(x

(1 + x2)2

)=⇒ f ′′(0) = 0

890

Page 907: CLP-1 Differential Calculus

SOLUTIONS TO EXERCISES

f ′′′(x) = −12

((1 + x2)2 − x · 2(1 + x2)(2x)

(1 + x2)4

)= −12

((1 + x2)− 4x2

(1 + x2)3

)= −12

(1− 3x2

(1 + x2)3

)=⇒ f ′′′(0) = −12

f (4)(x) = −12

((1 + x2)3(−6x)− (1− 3x2) · 3(1 + x2)2(2x)

(1 + x2)6

)= −12

(−6x(1 + x2)− 6x(1− 3x2)

(1 + x2)4

)= 144

(x− x3

(1 + x2)4

)=⇒ f (4)(0) = 0

f (5)(x) = 144

((1 + x2)4(1− 3x2)− (x− x3) · 4(1 + x2)3(2x)

(1 + x2)8

)= 144

((1 + x2)(1− 3x2)− 8x(x− x3)

(1 + x2)5

)= 144

5x4 − 10x2 + 1

(1 + x2)5

=⇒ f (5)(0) = 144

We now use these values to compute the 5th-degree Maclaurin polynomial forf(x).

T5(x) = f(0) + f ′(0)x+1

2f ′′(0)x2 +

1

3!f ′′′(0)x3 +

1

4!f (4)(0)x4

+1

5!f (5)(0)x5

= 6x− 12

6x3 +

144

120x5

= 6x− 2x3 +6

5x5

Now, if we want to approximate f(

1√3

)= 6 arctan

(1√3

)= π:

π = f

(1√3

)≈ T5

(1√3

)=

6√3− 2√

33 +

6

5√

35

= 2√

3

(1− 1

3 · 3 +1

5 · 9

)≈ 3.156

Remark: There are numerous methods for computing π to any desired degreeof accuracy. Many of them use the Maclaurin expansion of arctanx. In 1706John Machin computed π to 100 decimal digits by using the Maclaurin expansiontogether with π = 16 arctan 1

5− 4 arctan 1

239.

891

Page 908: CLP-1 Differential Calculus

SOLUTIONS TO EXERCISES

3.4.11.8. Solution. Let’s start by differentiating, and looking for a pattern.

f(x) = x(log x− 1) f(1) = −1

f ′(x) = x

(1

x

)+ log x− 1 = log x f ′(1) = 0

f ′′(x) =1

x= x−1 f ′′(1) = 1

f (3)(x) = (−1)x−2 f (3)(1) = −1

f (4)(x) = (−2)(−1)x−3 = 2!x−3 f (4)(1) = 2!

f (5)(x) = (−3)(−2)(−1)x−4 = −3!x−4 f (4)(1) = −3!

f (6)(x) = (−4)(−3)(−2)(−1)x−5 = 4!x−5 f (4)(1) = 4!

f (7)(x) = (−5)(−4)(−3)(−2)(−1)x−6 = −5!x−6 f (7)(1) = −5!

f (8)(x) = (−6)(−5)(−4)(−3)(−2)(−1)x−7 = 6!x−7 f (8)(1) = 6!

When k ≥ 2, making use of the fact that 0! = 1 and (−1)k−2 = (−1)k:

f (k)(x) = (−1)k−2(k − 2)!x−(k−1) f (k)(1) = (−1)k(k − 2)!

Now we use the standard form of a Taylor polynomial. Since the first two termsdon’t fit the pattern, we add those outside of the sigma.

T100(x) =100∑k=0

f (k)(1)

k!(x− 1)k

= f(1) + f ′(1)(x− 1) +100∑k=2

f (k)(1)

k!(x− 1)k

= −1 + 0(x− 1) +100∑k=2

(−1)k(k − 2)!

k!(x− 1)k

= −1 +100∑k=2

(−1)k

k(k − 1)(x− 1)k

3.4.11.9. Solution. Let’s start by taking some derivatives. Of course, since we’redifferentiating sine, the derivatives will repeat every four iterations.

f(x) = sinx f(π

4

)=

1√2

f ′(x) = cos x f ′(π

4

)=

1√2

f ′′(x) = − sinx f ′′(π

4

)= − 1√

2

f ′′′(x) = − cosx f ′′′(π

4

)= − 1√

2

892

Page 909: CLP-1 Differential Calculus

SOLUTIONS TO EXERCISES

So, the pattern of derivatives is1√2

,1√2

, − 1√2

, − 1√2

,1√2

,1√2

, − 1√2

, − 1√2

, etc.

This is a little tricky to write in sigma notation. We can deal with the “doubles”by pairing consecutive even and odd terms, as shown below.

T2n(x) =2n∑k=0

f (k)(π4

)k!

(x− π

4

)k

=n∑k=0

f (2k)(π4

)(2k)!

(x− π

4

)2k

︸ ︷︷ ︸even term

+f (2k+1)

(π4

)(2k + 1)!

(x− π

4

)2k+1

︸ ︷︷ ︸odd term

=

n∑k=0

[(−1)k

(2k)!√

2

(x− π

4

)2k

+(−1)k

(2k + 1)!√

2

(x− π

4

)2k+1]

To verify that the first sum really is the same as the second sum, write out thefirst few terms of each explicitly.

3.4.11.10. Solution. From Example 3.4.12in the text, we see that the nth Maclaurin polynomial for f(x) = ex is

Tn(x) =n∑k=0

1

k!xk

If n = 157 and x = 1,

T157(1) =157∑k=0

1

k!= 1 +

1

2+

1

3!+

1

4!+ · · ·+ 1

157!

Although we wouldn’t expect T157(1) to be exactly equal to e1, it’s probably prettyclose since T157(x) is the Maclaurin polynomial for ex, and 0 is pretty close to 1.So, we estimate

1 +1

2+

1

3!+

1

4!+ · · ·+ 1

157!≈ e

3.4.11.11. Solution. While you’re working with sums, it’s easy to mistake aconstant for a function. The sum given in this question is some number: π is aconstant, and k is an index– if you wrote out all 100 terms of this sum, therewould be no letter k. So, the sum given is indeed a number, but we don’t wantto have to add 100 terms to get a good idea of what number it is.From Example 3.4.14in the text, we see that the (2n)th-degree Maclaurin polynomial for f(x) = cosxis

T2n(x) =n∑k=0

(−1)k

(2k)!· x2k

893

Page 910: CLP-1 Differential Calculus

SOLUTIONS TO EXERCISES

If n = 100 and x =5π

4, this equation becomes

T200

(5π

4

)=

100∑k=0

(−1)k

(2k)!·(

4

)2k

So, the sum corresponds to the 200th Maclaurin polynomial for f(x) = cos xevaluated at x = 5π

4. We should be careful to understand that T200(x) is not equal

to f(x), in general. However, when x is reasonably close to 0, these two functionsare approximations of one another. So, we estimate

100∑k=0

(−1)k

2k!

(5π

4

)2k

= T200

(5π

4

)≈ cos

(5π

4

)= − 1√

2

· Exercises for § 3.4.6

Exercises — Stage 13.4.11.1. Solution.

x

y

y = f(x)

x x+ ∆x

f(x)

f (x+ ∆x)

∆x

∆y

3.4.11.2. Solution. Let f(x) be the number of problems finished after x minutesof work. The question tells us that 5∆y ≈ ∆x. So, if ∆x = 15, ∆y ≈ 3. That is, in15 minutes more, you will finish about 3 more problems.Remark: the math behind this problem is intended to be easy! Looking at sym-bols like ∆y and f (x+ ∆x) can be confusing, but the basic idea is pretty simple.

Exercises — Stage 23.4.11.3. Solution. First, let’s find the first and second derivatives of f whenx = 5.

f ′(x) =1

1 + x2f ′(5) =

1

26

f ′′(x) =−2x

(1 + x2)2 f ′′(5) =−10

262

894

Page 911: CLP-1 Differential Calculus

SOLUTIONS TO EXERCISES

The linear approximation for ∆y when ∆x =1

10is

∆y ≈ f ′(5)∆x =1

26· 1

10=

1

260≈ 0.003846

The quadratic approximation for ∆y when ∆x =1

10is

∆y ≈ f ′(5)∆x+1

2f ′′(5) (∆x)2 =

1

26· 1

10+

1

2· −10

262· 1

102

=51

13520≈ 0.003772

Remark: ∆y = arctan(5.1)− arctan(5) ≈ 0.003774.

3.4.11.4. Solution. (a) When x is near 4, the linear approximation of ∆y is

∆y ≈ s′(4)∆x

From the problem, ∆x = 5− 4 = 1. Differentiating,

s′(t) =√

19.6 · 1

2√x

=

√4.9

x, so

s′(4) =

√4.9

4

The linear approximation gives us

∆y ≈√

4.9

4(1) ≈ 1.1

So moving from 4 metres to 5 metres increases the speed with which you hit thewater by about 1.1 metres per second.(b) Again, we’ll use the linear approximation

∆y ≈ s′(a)∆x

=

√4.9

a·∆x

The difference in height between the first two jumps and between the last twojumps is the same, ∆x. The initial height of the first jump is smaller than theinitial height of the second jump. So, the value corresponding to a is smaller forthe first jump than for the second. Therefore, ∆y is larger between the first twojumps than it is between the last two jumps. So, the increase in speed from thefirst jump to the second is larger than the increase in speed from the second jumpto the third.To put that more symbolically, the change in terminal speed between the first two

895

Page 912: CLP-1 Differential Calculus

SOLUTIONS TO EXERCISES

jumps is

∆y ≈√

4.9

x·∆x

while the change in terminal speed between the next two jumps is

∆y ≈√

4.9

x+ ∆x·∆x

Since√

4.9

x· ∆x >

√4.9

x+ ∆x· ∆x (when x and ∆x are positive), the change in

terminal speed is greater between the first two jumps than between the next twojumps.

· Exercises for § 3.4.7

Exercises — Stage 13.4.11.1. Solution. False. The linear approximation is an approximation. It tellsus

∆y ≈ f ′(x0)∆x = f ′(2)(1) = 25

However, from our definition of ∆y,

∆y = f(x0 + ∆x)− f(x0) = f(2 + 1)− f(2) = 58− 26 = 32

Remark: this is to emphasize that the calculations in this subsection are estima-tions of error bounds, rather than actual error bounds. All we can say is that weestimate the error will be no more than some number–we don’t guarantee it.In the next subsection, we will introduce an error bound that is guaranteed to beaccurate. It is usually harder to calculate than the estimations in this section.

3.4.11.2. Solution. When an exact value Q0 is measured as Q0 + ∆Q, Defini-tion 3.4.25gives us the absolute error as |∆Q|, and the percentage error as 100

|∆Q|Q0

.

In our situation, Q0 = 5.83 and Q0 +∆Q = 6, so ∆Q = 0.17. So, the absolute erroris 0.17, and the percentage error is

1000.17

5.83≈ 2.92%

3.4.11.3. Solution. Since f ′(x) = 6x, when x = 10, f ′(10) = 60. If ∆y = f(11) −f(10), and ∆x = 11− 10, then the linear approximation tells us

∆y ≈ 60∆x = 60

So, the linear approximates estimates the error in f(x) to be about 60.Since f ′′(x) = 6, the quadratic approximation (using f ′(10) = 60, f ′′(10) = 6, and

896

Page 913: CLP-1 Differential Calculus

SOLUTIONS TO EXERCISES

∆x = 1) tells us

∆y ≈ f ′(10)∆x+1

2f ′′(1) (∆x)2 = 60 · 1 +

1

2(6)(1)2 = 63

So, the quadratic approximates estimates the error in f(x) to be about 63. (In-deed, the exact value of f(11) − f(10) is 63. It is not a fluke that our estimatederror, using a quadratic approximation, is exactly the same as our actual error. Itis a consequence of the fact that f(x) is a quadratic function.)

Exercises — Stage 23.4.11.4. Solution. Let A be the area of a pen of radius r. Then

A(r) = πr2

Differentiating with respect to r,

A′(r) = 2πr

The exact area desired is A0. Let the corresponding exact radius desired be r0.

Using the linear approximation formula, where ∆A is the change in A corre-sponding to a change in r of ∆r,

∆A ≈ A′(r0)∆r = 2πr0∆r

∆r ≈ ∆A

2πr0

What we’re interested in is the percent error r can have. The percent error is:

100∆r

r0

≈ 100∆A

2πr0 · r0

= 100∆A

2(πr20)

= 100∆A

2 · A0

=

(100

∆A

A0

)1

2

≤ (2)1

2= 1

(To get the last line, we used the given information that the percent error in the

area, 100∆A

A0

, must be less than 2%.)

We conclude the error in r cannot be more than 1%.

897

Page 914: CLP-1 Differential Calculus

SOLUTIONS TO EXERCISES

3.4.11.5. Solution. (a) The area removed represents a proportion ofθ

2πof the

entire circle, whose area is π(32) = 9π. So, the area of the sector removed is

θ

2π· 9π =

9

(b) To find θ from d, we cut our triangle (with angle θ opposite side of length d)into two equivalent right triangles, as shown below.

3 3 d

2θ2

Using the information that the radius of the circle (also the hypotenuse of theright triangle) is 3,

sin

2

)=

d2

3=d

6

Since the question tells us the sector is no more than a quarter of the circle, we

know 0 ≤ θ ≤ π

2, so 0 ≤ θ

2≤ π

4. This puts

θ

2well within the range of arcsine.

θ = 2 arcsin

(d

6

)(c) First, let’s get an expression for the area of the sector in terms of d.

A =9

2θ =

9

2

(2 arcsin

(d

6

))= 9 arcsin

(d

6

)Differentiating,

A′(d) =9√

1−(d6

)2· 1

6

=9√

36− d2

Let ∆A is the error in A corresponding to an error of ∆d in d. Since we measuredd to be 0.7 instead of 0.68, in the linear approximation we take ∆d = 0.02 andd0 = 0.68.

∆A ≈ A′(d0) ·∆d

898

Page 915: CLP-1 Differential Calculus

SOLUTIONS TO EXERCISES

= A′ (0.68) · 0.02

=9√

36− 0.682· 0.02

≈ 0.03

So, the error in A is about 0.03.

3.4.11.6. Solution. Suppose we have an equation V (h) that gives the volume ofwater in the tank as a function of its height.Let h0 = 0.5 metres, ∆h = −0.05, and ∆V = V (h0 + ∆h) − V (h0) = V (0.45) −V (0.5). Then, by the linear approximation,

∆V ≈ V ′(0.5) ·∆h = −0.05V ′(0.5)

In order to solve the problem, we will find an equation V (h) giving the volumeof water in terms of the height, then find V ′(0.5), and finally approximate thatthe change in the volume of the water is ∆V ≈ −0.05V ′(0.5).The water in the tank forms a cone. The volume of a cone of height h and radiusr is

V =1

3πr2h

We need to get rid of the variable r. We can do this using similar triangles. Thediagram below shows the side view of the tank and the water.

1

2r2

h

The side view of the tank forms a triangle that is similar to the triangle formedby the side view of the water, so

1

2=

2r

h

r =h

4

Using this, we find our equation for the volume of the water in terms of h.

V (h) =1

3πr2h =

1

3

(h

4

)2

h =h3

48

899

Page 916: CLP-1 Differential Calculus

SOLUTIONS TO EXERCISES

Differentiating,

V ′(h) =h2

16

V ′(0.5) =0.25

16=

1

64

Finally, using the approximation ∆V ≈ −0.05V ′(0.5),

∆V ≈ −0.05

64= − 1

1280≈ −0.00078 m3

We estimate the volume decreased by about 0.00078 cubic metres, or about 780cubic centimetres.

Exercises — Stage 33.4.11.7. Solution. Let q be the measured amount of the isotope remaining after3 years. Let h(q) be the half-life of the isotope that we calculate using q. Wemeasured q = 0.9, but we want to know what the change in h is if q moves by0.05. So, let ∆q = ±0.05, and let ∆h be the corresponding change in h. The linearapproximation tells us

∆h ≈ h′(0.9)∆q

So,

|∆h| ≈ |h′(0.9)| |∆q| = h′(0.9) · 0.05

This suggests a plan for solving the problem. We will find the equation h(q)giving the calculated half-life of the isotope based on the measurement q. Then,we will find h′(0.9). Finally, the equation |∆h| = h′(0.9) · 0.05 will tell us thechange in h that corresponds with a change of 0.05 in our measurement.Let us find the half-life of the isotope, if after three years q µg is remaining. Theamount of the isotope present after t years is given by

Q(t) = Q(0)e−kt

for some constant k. Let’s take t = 0 to be the time when precisely one µg waspresent. Then

Q(t) = e−kt

After three years, q is the amount of the isotope remaining, so

q = e−k·3

q13 = e−k

Q(t) =(e−k)t

= qt3

900

Page 917: CLP-1 Differential Calculus

SOLUTIONS TO EXERCISES

The half-life is the value of t for which Q(t) = 12Q(0) = 1

2.

1

2= Q(t) = q

t3

log

(1

2

)= log

(qt3

)− log 2 =

t

3log q

t =−3 log 2

log q

So, we calculate the half-life to be−3 log 2

log q. This gives us our first goal: a function

h(q) that tells us the calculated half-life of the element.

h(q) =−3 log 2

log q

Following our plan, we find h′(0.9).

h′(q) =d

dq

−3 log 2

log q

= −3 log 2 · d

dq

(log q)−1

= −3 log 2 · (−1) (log q)−2 · 1

q

=3 log 2

q log2 q

h′(0.9) =3 log 2

0.9 log2(0.9)≈ 208

Finally, as outlined in our plan,

|∆h| = h′(0.9) · 0.05

=3 log 2

18 log2(0.9)≈ 10.4

If our measurement changes by ±0.05 µg, then we estimate our calculated half-life changes by about ±10.4 years. Since our measurement is accurate to within0.05 µg, that means we estimate our calculated half-life to be accurate to withinabout 10.4 years.

Remark: since h(0.9) =−3 log 2

log 0.9≈ 19.7, an absolute error of 10.4 years corre-

sponds to a percentage error of 10010.4

19.7≈ 53%. The question did not specify

absolute or percentage error. Since both make sense, you can use either one.

901

Page 918: CLP-1 Differential Calculus

SOLUTIONS TO EXERCISES

· Exercises for § 3.4.8

Exercises — Stage 13.4.11.1. Solution. From the given information,

|R(10)| = |f(10)− F (10)| = | − 3− 5| = | − 8| = 8

So, (a) is false (since 8 is not less than or equal to 7), while (b), (c), and (d) aretrue.Remark: R(x) is the error in our approximation. As mentioned in the text, wealmost never knowR exactly, but we can give a bound. We don’t need the tightestbound–just a reasonable one that is easy to calculate. If we were dealing withreal functions and approximations, we might not know that |R(10)| = 8, but ifwe knew it was at most 9, that would be a pretty decent approximation.Often in this section, we will make simplifying assumptions to get a bound thatis easy to calculate. But, don’t go overboard! It is a true statement to say thatour absolute error is at most 100, but this statement would probably not be veryhelpful as a bound.

3.4.11.2. Solution. Equation 3.4.33 tells us that, when Tn(x) is the nth degreeTaylor polynomial for a function f(x) about x = a, then

|f(x)− Tn(x)| =∣∣∣∣f (n+1)(c)

(n+ 1)!(x− a)n+1

∣∣∣∣for some c strictly between x and a. In our case, n = 3, a = 0, x = 2, andf (4)(c) = ec, so

|f(2)− T3(2)| =∣∣∣∣f (4)(c)

4!(2− 0)4

∣∣∣∣=

24

4!ec =

2

3ec

Since c is strictly between 0 and 2, ec < e2:

≤ 2

3e2

but this isn’t a number we really know. Indeed: e2 is the very number we’retrying to approximate. So, we use the estimation e < 3:

<2

3· 32 = 6

We conclude that the error |f(2)− T3(2)| is less than 6.Now we’ll get a more exact idea of the error using a calculator. (Calculatorswill also only give approximations of numbers like e, but they are generally verygood approximations.)

|f(2)− T3(2)| =∣∣∣∣e2 −

(1 + 2 +

1

2· 22 +

1

3!· 23

)∣∣∣∣902

Page 919: CLP-1 Differential Calculus

SOLUTIONS TO EXERCISES

=

∣∣∣∣e2 −(

1 + 2 + 2 +4

3

)∣∣∣∣=

∣∣∣∣e2 − 19

3

∣∣∣∣ ≈ 1.056

So, our actual answer was only off by about 1.Remark: 1 < 6, so this does not in any way contradict our bound |f(2)− T3(2)| <6.

3.4.11.3. Solution. Whenever you approximate a polynomial with a Taylorpolynomial of greater or equal degree, your Taylor polynomial is exactly the sameas the function you are approximating. So, the error is zero.

3.4.11.4. Solution. The constant approximation gives

sin(33) ≈ sin(0) = 0

while the linear approximation gives

f(x) ≈ f(0) + f ′(0)x

sin(x) ≈ sin(0) + cos(0)x

= x

sin(33) ≈ 33

Since−1 ≤ sin(33) ≤ 1, the constant approximation is better. (But both are a littlesilly.)

x

y

T0(x)

T1(x)

903

Page 920: CLP-1 Differential Calculus

SOLUTIONS TO EXERCISES

Exercises — Stage 23.4.11.5. Solution. Equation 3.4.33 tells us that, when Tn(x) is the nth degreeTaylor polynomial for a function f(x) about x = a, then

|f(x)− Tn(x)| =∣∣∣∣f (n+1)(c)

(n+ 1)!(x− a)n+1

∣∣∣∣for some c strictly between x and a. In our case, n = 5, a = 11, x = 11.5, and

f (6)(c) =6!(2c− 5)

c+ 3.

|f(11.5)− T5(11.5)| =∣∣∣∣ 1

6!

(6!(2c− 5)

c+ 3

)(11.5− 11)6

∣∣∣∣=

∣∣∣∣2c− 5

c+ 3

∣∣∣∣ · 1

26

for some c in (11, 11.5). We don’t know exactly which c this is true for, but sincewe know that c lies in (11, 11.5), we can provide bounds.

• 2c− 5 < 2(11.5)− 5 = 18

• c+ 3 > 11 + 3 = 14

• Therefore,∣∣∣∣2c− 5

c+ 3

∣∣∣∣ =2c− 5

c+ 3<

18

14=

9

7when c ∈ (11, 11.5).

With this bound, we see

|f(11.5)− T5(11.5)| =∣∣∣∣2c− 5

c+ 3

∣∣∣∣ · 1

26

<

(9

7

)(1

26

)≈ 0.0201

Our error is less than 0.02.

3.4.11.6. Solution. Equation 3.4.33 tells us that, when Tn(x) is the nth degreeTaylor polynomial for a function f(x) about x = a, then

|f(x)− Tn(x)| =∣∣∣∣f (n+1)(c)

(n+ 1)!(x− a)n+1

∣∣∣∣for some c strictly between x and a. In our case, n = 2, a = 0, and x = 0.1, so

|f(0.1)− T2(0.1)| =∣∣∣∣f (3)(c)

3!(0.1− 0)3

∣∣∣∣=|f ′′′(c)|6000

for some c in (0, 0.1).

904

Page 921: CLP-1 Differential Calculus

SOLUTIONS TO EXERCISES

We will find f ′′′(x), and use it to give an upper bound for

|f(0.1)− T2(0.1)| = |f′′′(c)|

6000

when c is in (0, 0.1).

f(x) = tan x

f ′(x) = sec2 x

f ′′(x) = 2 secx · secx tanx

= 2 sec2 x tanx

f ′′′(x) =(2 sec2 x

)sec2 x+ (4 secx · secx tanx) tanx

= 2 sec4 x+ 4 sec2 x tan2 x

When 0 < c <1

10, also 0 < c <

π

6, so:

• tan c < tan(π

6

)=

1√3

• cos c > cos(π

6

)=

√3

2

• sec c <2√3

With these bounds in mind for secant and tangent, we return to the expressionwe found for our error.

|f(0.1)− T2(0.1)| = |f′′′(c)|

6000=|2 sec4 x+ 4 sec2 x tan2 x|

6000

<2(

2√3

)4

+ 4(

2√3

)2 (1√3

)2

6000

=1

1125

The error is less than1

1125.

3.4.11.7. Solution. Equation 3.4.33 tells us that, when Tn(x) is the nth degreeTaylor polynomial for a function f(x) about x = a, then

|f(x)− Tn(x)| =∣∣∣∣f (n+1)(c)

(n+ 1)!(x− a)n+1

∣∣∣∣for some c strictly between x and a. In our case, n = 5, a = 0, and x = −1

4, so∣∣∣∣f (−1

4

)− T5

(−1

4

)∣∣∣∣ =

∣∣∣∣∣f (6)(c)

6!

(−1

4− 0

)6∣∣∣∣∣

905

Page 922: CLP-1 Differential Calculus

SOLUTIONS TO EXERCISES

=

∣∣f (6)(c)∣∣

6! · 46

for some c in(−1

4, 0). We’ll need to know the sixth derivative of f(x).

f(x) = log(1− x)

f ′(x) = −(1− x)−1

f ′′(x) = −(1− x)−2

f ′′′(x) = −2(1− x)−3

f (4)(x) = −3!(1− x)−4

f (5)(x) = −4!(1− x)−5

f (6)(x) = −5!(1− x)−6

Plugging in∣∣f (6)(c)

∣∣ =5!

(1− c)6:

∣∣∣∣f (−1

4

)− T5

(−1

4

)∣∣∣∣ =5!

6! · 46 · (1− c)6=

1

6 · 46 · (1− c)6

for some c in(−1

4, 0).

We’re interested in an upper bound for the error: we want to know the worstcase scenario, so we can say that the error is no worse than that. We need to know

what the biggest possible value of1

6 · 46 · (1− c)6is, given −1

4< c < 0. That

means we want to know the biggest possible value of1

(1− c)6. This corresponds

to the smallest possible value of (1−c)6, which in turn corresponds to the smallestabsolute value of 1− c.

• Since −1

4≤ c ≤ 0, the smallest absolute value of 1 − c occurs when c = 0.

In other words, |1− c| ≤ 1.

• That means the smallest possible value of (1− c)6 is 16 = 1.

• Then the largest possible value of1

(1− c)6is 1.

• Then the largest possible value of1

6 · 46· 1

(1− c)6is

1

6 · 46≈ 0.0000407.

Finally, we conclude∣∣∣∣f (−1

4

)− T5

(−1

4

)∣∣∣∣ =1

6 · 46 · (1− c)6<

1

6 · 46< 0.00004

3.4.11.8. Solution. Equation 3.4.33 tells us that, when Tn(x) is the nth degree

906

Page 923: CLP-1 Differential Calculus

SOLUTIONS TO EXERCISES

Taylor polynomial for a function f(x) about x = a, then

|f(x)− Tn(x)| =∣∣∣∣f (n+1)(c)

(n+ 1)!(x− a)n+1

∣∣∣∣for some c strictly between x and a. In our case, n = 3, a = 30, and x = 32, so

|f(30)− T3(30)| =∣∣∣∣f (4)(c)

4!(30− 32)4

∣∣∣∣=

2

3

∣∣f (4)(c)∣∣

for some c in (30, 32).We will now find f (4)(x). Then we can give an upper bound on |f(30)−T3(30)| =2

3

∣∣f (4)(c)∣∣when c ∈ (30, 32).

f(x) = x15

f ′(x) =1

5x−

45

f ′′(x) = − 4

52x−

95

f ′′′(x) =4 · 953

x−145

f (4)(x) = −4 · 9 · 14

54x−

195

Using this,

|f(30)− T3(30)| = 2

3

∣∣f (4)(c)∣∣

=2

3

∣∣∣∣−4 · 9 · 14

54c−

195

∣∣∣∣=

336

54 · c195

Since 30 < c < 32,

<336

54 · 30195

=336

54 · 303 · 3045

=14

57 · 9 · 3045

This isn’t a number we know. We’re trying to find the error in our estimation of5√

30, but 5√

30 shows up in our error. From here, we have to be a little creative toget a bound that actually makes sense to us. There are different ways to go about

it. You could simply use 3045 > 1. We will be a little more careful, and use the

following estimation:

14

57 · 9 · 3045

=14 · 30

15

57 · 9 · 30

907

Page 924: CLP-1 Differential Calculus

SOLUTIONS TO EXERCISES

<14 · 32

15

57 · 9 · 30

<14 · 2

57 · 9 · 30

<14

57 · 9 · 15< 0.000002

We conclude |f(30)− T3(30)| < 0.000002.

3.4.11.9. Solution. Equation 3.4.33 tells us that, when Tn(x) is the nth degreeTaylor polynomial for a function f(x) about x = a, then

|f(x)− Tn(x)| =∣∣∣∣f (n+1)(c)

(n+ 1)!(x− a)n+1

∣∣∣∣for some c strictly between x and a. In our case, n = 1, a =

1

π, and x = 0.01, so

|f(0.01)− T1(0.01)| =∣∣∣∣∣f ′′(c)2

(0.01− 1

π

)2∣∣∣∣∣

=1

2

(100− π

100π

)2

· |f ′′(c)|

for some c in(

1100, 1π

).

Let’s find f ′′(x).

f(x) = sin

(1

x

)f ′(x) = cos

(1

x

)· −1

x2=− cos

(1x

)x2

f ′′(x) =x2 sin

(1x

)(−x−2) + cos

(1x

)(2x)

x4

=2x cos

(1x

)− sin

(1x

)x4

Now we can plug in a better expression for f ′′(c):

|f(0.01)− T1(0.01)| = 1

2

(100− π

200π

)2

· |f ′′(c)|

=1

2

(100− π

100π

)2

·∣∣2c cos

(1c

)− sin

(1c

)∣∣c4

for some c in(

1100, 1π

).

What we want to do now is find an upper bound on this expression containing

c,1

2

(100− π

100π

)2

·∣∣2c cos

(1c

)− sin

(1c

)∣∣c4

.

908

Page 925: CLP-1 Differential Calculus

SOLUTIONS TO EXERCISES

• Since c ≥ 1

100, it follows that c4 ≥ 1

1004, so

1

c4≤ 1004.

• For any value of x, | cosx| and | sinx| are at most 1. Since |c| < 1, also|c cos

(1c

)| < | cos

(1c

)| ≤ 1. So,

∣∣2c cos(

1c

)− sin

(1c

)∣∣ < 3

• Therefore,

|f(0.01)− Tn(0.01)|

=1

2

(100− π

100π

)2

· 1

c4·∣∣∣∣2c cos

(1

c

)− sin

(1

c

)∣∣∣∣<

1

2

(100− π

100π

)2

· 1004 · 3

=3 · 1002

2

(100

π− 1

)2

Equation 3.4.33 gives the bound |f(0.01)− T1(0.01)| ≤ 3·1002

2

(100π− 1)2.

The bound above works out to approximately fourteen million. One way to un-derstand why the bound is so high is that sin

(1x

)moves about crazily when x is

near zero–it moves up and down incredibly fast, so a straight line isn’t going toapproximate it very well at all.That being said, because sin

(1x

)is still “sine of something”, we know −1 ≤

f (0.01) ≤ 1. To get a better bound on the error, let’s find T1(x).

f(x) = sin

(1

x

)f

(1

π

)= sin(π) = 0

f ′(x) =− cos

(1x

)x2

f ′(

1

π

)= −π2 cos(π) = π2

T1(x) = f

(1

π

)+ f ′

(1

π

)(x− 1

π

)= 0 + π2

(x− 1

π

)= π2x− π

T1 (0.01) =π2

100− π

Now that we know T1(0.01), and we know −1 ≤ f(0.01) ≤ 1, we can give thebound

|f(0.01)− T1(0.01)| ≤ |f(0.01)|+ |T1(0.01)|

≤ 1 +

∣∣∣∣ π2

100− π

∣∣∣∣= 1 + π

∣∣∣1− π

100

∣∣∣909

Page 926: CLP-1 Differential Calculus

SOLUTIONS TO EXERCISES

< 1 + π

< 1 + 4 = 5

A more reasonable bound on the error is that it is less than 5.Still more reasonably, we would not use T1(x) to evaluate sin(100) approximately.We would write sin(100) = sin(100 − 32π) and approximate the right hand side,which is roughly sin(−π/6).

3.4.11.10. Solution. Equation 3.4.33 tells us that, when Tn(x) is the nth degreeTaylor polynomial for a function f(x) about x = a, then

|f(x)− Tn(x)| =∣∣∣∣f (n+1)(c)

(n+ 1)!(x− a)n+1

∣∣∣∣for some c strictly between x and a. In our case, n = 2, a = 0, and x =

1

2, so

∣∣∣∣f (1

2

)− T2

(1

2

)∣∣∣∣ =

∣∣∣∣∣f (3)(c)

3!

(1

2− 0

)3∣∣∣∣∣

=

∣∣f (3)(c)∣∣

3! · 23

for some c in(0, 1

2

). The next task that suggests itself is finding f (3)(x).

f(x) = arcsin x

f ′(x) =1√

1− x2= (1− x2)−

12

f ′′(x) = −1

2(1− x2)−

32 (−2x)

= x(1− x2)−32

f ′′′(x) = x

(−3

2

)(1− x2)−

52 (−2x) + (1− x2)−

32

= 3x2(1− x2)−52 + (1− x2)−

52

+1

= (1− x2)−52(3x2 + (1− x2)

)= (1− x2)−

52(2x2 + 1

)Since

∣∣f (12

)− T2

(12

)∣∣ =

∣∣f (3)(c)∣∣

3! · 23for some c in

(0, 1

2

),

∣∣∣∣f (1

2

)− T2

(1

2

)∣∣∣∣ =

∣∣∣∣∣ 1 + 2c2(√1− c2

)5

∣∣∣∣∣3! · 23

=1 + 2c2

48(√

1− c2)5

910

Page 927: CLP-1 Differential Calculus

SOLUTIONS TO EXERCISES

for some c in(0, 1

2

).

We want to know what is the worst case scenario-what’s the biggest this expres-

sion can be. So, now we find an upper bound on1 + 2c2

48(√

1− c2)5 when 0 ≤ c ≤ 1

2.

Remember that our bound doesn’t have to be exact, but it should be relativelyeasy to calculate.

• When 0 ≤ c ≤ 1

2, the biggest 1 + 2c2 can be is 1 + 2

(1

2

)2

=3

2.

So, the numerator of1 + 2c2

48(√

1− c2)5 is at most

3

2.

• The smallest 1− c2 can be is 1−(

1

2

)2

=3

4.

• So, the smallest(√

1− c2)5

can be is

(√3

4

)5

=

(√3

2

)5

.

• Then smallest possible value for the denominator of1 + 2c2

48(√

1− c2)5 is

48

(√3

2

)5

• Then

1 + 2c2

48(√

1− c2)5 ≤

3

2

48

(√3

2

)5

=1√

35 =

1

9√

3

<1

10

Let’s put together these pieces. We found that∣∣∣∣f (1

2

)− T2

(1

2

)∣∣∣∣ =1 + 2c2

48(√

1− c2)5

for some c in(0, 1

2

). We also found that

1 + 2c2

48(√

1− c2)5 <

1

10

911

Page 928: CLP-1 Differential Calculus

SOLUTIONS TO EXERCISES

when c is in(0, 1

2

). We conclude∣∣∣∣f (1

2

)− T2

(1

2

)∣∣∣∣ < 1

10.

For the second part of the question, we need to find f(

12

)and T2

(12

). Finding

f(

12

)is not difficult.

f(x) = arcsin x

f

(1

2

)= arcsin

(1

2

)=π

6

In order to find T2

(12

), we need to find T2(x).

T2(x) = f(0) + f ′(0)x+1

2f ′′(0)x2

Conveniently, we’ve already found the first few derivatives of f(x).

T2(x) = arcsin(0) +

(1√

1− 02

)x+

1

2

(0(√

1− 02)3

)x2

= 0 + x+ 0

= x

T2

(1

2

)=

1

2

So, the actual error is∣∣∣∣f (1

2

)− T2

(1

2

)∣∣∣∣ =

∣∣∣∣π6 − 1

2

∣∣∣∣ =π

6− 1

2

A calculator tells us that this is about 0.02.

Exercises — Stage 3

3.4.11.11. Solution. Our error will have the formf (n+1)(c)

(n+ 1)!(x − 1)n+1 for some

constant c, so let’s find an equation for f (n)(x). This has been done before in thetext, but we’ll do it again here: we’ll take several derivatives, then notice thepattern.

f(x) = log x

f ′(x) = x−1

f ′′(x) = −x−2

f ′′′(x) = 2! x−3

f (4)(x) = −3!x−4

912

Page 929: CLP-1 Differential Calculus

SOLUTIONS TO EXERCISES

f (5)(x) = 4! x−5

So, when n ≥ 1,

f (n)(x) = (−1)n−1(n− 1)! · x−n

Now that we know the derivative of f(x), we have a better idea what the errorin our approximation looks like.

|f(1.1)− Tn(1.1)| =∣∣∣∣f (n+1)(c)

(n+ 1)!(1.1− 1)n+1

∣∣∣∣=∣∣f (n+1)(c)

∣∣ 0.1n+1

(n+ 1)!

=

∣∣∣∣ n!

cn+1

∣∣∣∣ 1

10n+1(n+ 1)!

=1

|c|n+1 · 10n+1 · (n+ 1)

for some c in (1, 1.1)

<1

(n+ 1)10n+1 · 1n+1

=1

(n+ 1)10n+1

What we’ve shown so far is

|f(1.1)− Tn(1.1)| < 1

(n+ 1)10n+1

If we can show that1

(n+ 1)10n+1≤ 10−4, then we’ll be able to conclude

|f(1.1)− Tn(1.1)| < 1

(n+ 1)10n+1≤ 10−4

That is, our error is less than 10−4.

So, our goal for the problem is to find a value of n that makes1

(n+ 1)10n+1≤

10−4. Certainly, n = 3 is such a number. Therefore, any n greater than or equal to3 is an acceptable value.

3.4.11.12. Solution. We will approximate f(x) = x17 using a Taylor polynomial.

Since 37 = 2187, we will use x = 2187 as our centre.We need to figure out which degree Taylor polynomial will result in a small-

913

Page 930: CLP-1 Differential Calculus

SOLUTIONS TO EXERCISES

enough error.If we use the nth Taylor polynomial, our error will be

|f(2200)− Tn(2200)| =∣∣∣∣f (n+1)(c)

(n+ 1)!(2200− 2187)n+1

∣∣∣∣=∣∣f (n+1)(c)

∣∣ · 13n+1

(n+ 1)!

for some c in (2187, 2200). In order for this to be less than 0.001, we need∣∣f (n+1)(c)∣∣ · 13n+1

(n+ 1)!< 0.001∣∣f (n+1)(c)∣∣ < (n+ 1)!

1000 · 13n+1

It’s a tricky thing to figure out which n makes this true. Let’s make a table. Wewon’t show all the work of filling it in, but the work is standard.

n(n+ 1)!

1000 · 13n+1

∣∣f (n+1)(c)∣∣ (n+ 1)!

1000 · 13n+1

01

1000 · 13|f ′(c)| = 1

7c6/7<

1

7 · 36

1

1000 · 131

12

1000 · 132|f ′′(c)| = 6

72 · c137

<6

72 · 313

(2)!

1000 · 132

Since72 · 313

3= 26, 040, 609 > 169, 000 = 1000 · 132

we have∣∣f (n+1)(c)

∣∣ < (n+1)!1000·13n+1 when n = 1.

That is: if we use the first-degree Taylor polynomial, then for some c between2187 and 2200,

|f(2200)− T1(2200)| = |f ′′(c)| · 132

2!

=6

72 · c137

· 132

2

<6

72 · 313· 132

2

=3 · 132

72 · 313≈ 0.0000065

So, actually, the linear Taylor polynomial (or any higher-degree Taylor poly-nomial) will result in an approximation that is much more accurate than re-quired. (We don’t know, however, that the constant approximation will be ac-curate enough–so we’d better stick with n ≥ 1.)

914

Page 931: CLP-1 Differential Calculus

SOLUTIONS TO EXERCISES

Now that we know we can take the first-degree Taylor polynomial, let’s compute

T1(x). Recall we are taking the Taylor polynomial for f(x) = x17 about x = 2187.

f(2187) = 218717 = 3

f ′(x) =1

7x−

67

f ′(2187) =1

7 7√

21876 =

1

7 · 36

T1(x) = f(2187) + f ′(2187)(x− 2187)

= 3 +x− 2187

7 · 36

T1(2200) = 3 +2200− 2187

7 · 36

= 3 +13

7 · 36

≈ 3.00255

We conclude 7√

2200 ≈ 3.00255.

3.4.11.13. Solution. If we’re going to use Equation 3.4.33, then we’ll probablybe taking a Taylor polynomial. Using Example 3.4.16, the 6th-degree Maclaurinpolynomial for sinx is

T6(x) = T5(x) = x− x3

3!+x5

5!

so let’s play with this a bit. Equation 3.4.33 tells us that the error will depend onthe seventh derivative of f(x), which is − cosx:

f(1)− T6(1) = f (7)(c)17

7!

sin(1)−(

1− 1

3!+

1

5!

)=− cos c

7!

sin(1)− 101

5!=− cos c

7!

sin(1) =4242− cos c

7!

for some c between 0 and 1. Since −1 ≤ cos c ≤ 1,

4242− 1

7!≤ sin(1) ≤ 4242 + 1

7!4241

7!≤ sin(1) ≤ 4243

7!4241

5040≤ sin(1) ≤ 4243

5040

Remark: there are lots of ways to play with this idea to get better estimates. Oneway is to take a higher-degree Maclaurin polynomial. Another is to note that,

915

Page 932: CLP-1 Differential Calculus

SOLUTIONS TO EXERCISES

since 0 < c < 1 <π

3, then

1

2< cos c < 1, so

4242− 1

7!< sin(1) <

4242− 12

7!4241

5040< sin(1) <

8483

10080<

4243

5040

If you got tighter bounds than asked for in the problem, congratulations!

3.4.11.14. Solution. 3.4.11.14.a For every whole number n, the nth derivative ofex is ex. So:

T4(x) =4∑

n=0

e0

n!xn =

4∑n=0

xn

n!

3.4.11.14.b

T4(1) =4∑

n=0

1n

n!=

4∑n=0

1

n!

=1

0!+

1

1!+

1

2!+

1

3!+

1

4!

=1

1+

1

1+

1

2+

1

6+

1

24

=65

24

3.4.11.14.c Using Equation 3.4.33,

e1 − T4(1) =1

5!ec for some strictly between 0 and 1. So,

e− 65

24=

ec

120

e =65

24+

ec

120

Since ex is a strictly increasing function, and 0 < c < 1, we conclude e0 < ec < e1:

65

24+

1

120< e <

65

24+

e

120

Simplifying the left inequality, we see

326

120< e

From the right inequality, we see

e <65

24+

e

120

e− e

120<

65

24

916

Page 933: CLP-1 Differential Calculus

SOLUTIONS TO EXERCISES

e · 119

120<

65

24

e <65

24· 120

119=

325

119

So, we conclude

326

120< e <

325

119,

as desired.Remark:

326

120≈ 2.717, and

325

119≈ 2.731.

· Further problems for § 3.4

Exercises — Stage 13.4.11.1. *. Solution. The third Maclaurin polynomial for f(x) is

f(0) + f ′(0)x+f ′′(0)

2· x2 +

f ′′′(0)

6· x3 = 4 + 3x2 +

1

2x3.

The coefficient of x is f ′(0) on one side, and 0 on the other, so f ′(0) = 0.

The coefficient of x2 is1

2f ′′(0) on one side, and 3 on the other, so f ′′(0) = 6.

3.4.11.2. *. Solution. The third Maclaurin polynomial for h(x) is

h(0) + h′(0)x+h′′(0)

2· x2 +

h(3)(0)

3!· x3 = 1 + 4x− 1

3x2 +

2

3x3

The coefficient of x3 is1

3!h(3)(0) on one side, and

2

3on the other. Thus

h(3)(0)

6=

2

3,

so h(3)(0) = 6 · 2

3= 4.

3.4.11.3. *. Solution. The third-degree Taylor polynomial for h(x) about x = 2is

h(2) + h′(2)(x− 2) +h′′(2)

2· (x− 2)2 +

h′′′(2)

6· (x− 2)3

The coefficient of (x− 2) is h′(2) in the definition, and1

2in the given function, so

h′(2) =1

2.

The coefficient of (x− 2)2 is1

2h′′(2) in the definition, and 0 in the given function,

so h′(2) = 0.

Exercises — Stage 2

917

Page 934: CLP-1 Differential Calculus

SOLUTIONS TO EXERCISES

3.4.11.4. *. Solution.

a For x near 3,

f(x) ≈ f(3) + f ′(3)(x− 3) = 2 + 4(x− 3)

In particular

f(2.98) ≈ 2 + 4(2.98− 3) = 2− 0.08 = 1.92

b For x near 3,

f(x) ≈ f(3) + f ′(3)(x− 3) +1

2f ′′(3)(x− 3)2

= 2 + 4(x− 3)− 1

210(x− 3)2

In particular

f(2.98) ≈ 2 + 4(2.98− 3)− 5(2.98− 3)2

= 2− 0.08− 0.002 = 1.918

3.4.11.5. *. Solution. Let’s name g(x) = x1/3. Then g′(x) =1

3x−2/3 and g′′(x) =

−2

9x−5/3. In particular, g(8) = 2, g′(8) =

1

12, and g′′(x) < 0 for all x > 0.

The tangent line approximation to 101/3 is

g(10) ≈ g(8) + g′(8)(10− 8)

= 2 +1

12(2) =

13

6

Using the error formula:

g(10) = g(8) + g′(8)(10− 8) +1

2g′′(c)(10− 8)2

for some 8 < c < 10. Since g′′(c) = − 2

9c5/3, g′′(c) is negative, so g(10) is

13

6plus

some negative quantity. So, the tangent line approximation is too big.

3.4.11.6. *. Solution. We use the function f(x) =√x and point a = 1 as the

centre of our approximation, since we can easily calculate

f(a) = f(1) =√

1 = 1.

We compute f ′(x) =1

2√x

, so

f ′(1) =1

2√

1=

1

2.

918

Page 935: CLP-1 Differential Calculus

SOLUTIONS TO EXERCISES

So, a linear approximation of√

2 = f(2) is√

2 ≈ T1(2) = f(1) + f ′(1) · (2− 1)

= 1 +1

2=

3

2.

3.4.11.7. *. Solution. We use the function f(x) = x1/3 and point a = 27 as thecentre of our approximation since we can easily compute

f(27) = 3

We compute f ′(x) =1

3x−2/3, so

f ′(27) =1

3· (27)−2/3 =

1

27

So, the linear approximation of 261/3 = f(26) is

261/3 ≈ T1 (26) = f(27) + f ′(27) · (26− 27)

= 3− 1

27=

80

27

3.4.11.8. *. Solution. We use the function f(x) = x5 and point a = 10 as thecentre of our approximation since we know that f(a) = f(10) = 105.Since f ′(x) = 5x4 we have f ′(10) = 50, 000.So, a linear approximation of 10.15 is

T1(10.1) = f(10) + f ′(10) · (10.1− 10)

= 100, 000 + 50, 000 · 0.1 = 105, 000 .

3.4.11.9. *. Solution. We use the function f(x) = sin(x) and point a = π as thecentre of our approximation since we know that

sin(a) = f(π) = sin π = 0.

and π is reasonably close to101π

100. We compute f ′(x) = cos(x), so

f ′(π) = cos(π) = −1.

So, the linear approximation of sin(

101π100

)is

f

(101π

100

)≈ T1

(101π

100

)= f(π) + f ′(π) ·

(101π

100− π

)= 0 + (−1) · π

100= − π

100.

919

Page 936: CLP-1 Differential Calculus

SOLUTIONS TO EXERCISES

3.4.11.10. *. Solution. Set f(x) = arctan(x). Then f ′(x) =1

1 + x2, so f ′(1) =

1

2and

f(1.1) ≈ f(1) + f ′(1)(1.1− 1) =π

4+

1

20

3.4.11.11. *. Solution. Set f(x) = (2 + x)3, so we are approximating f(0.001).The obvious choice of a is a = 0.Then f ′(x) = 3(2 + x)2, so

(2.001)3 = f(0.001) ≈ f(0) + f ′(0)(0.001− 0) = 8 +12

1000=

8012

1000

Remark: if we had chosen f(x) = x3 and a = 2, the result would have beenexactly the same.

3.4.11.12. *. Solution. We set f(x) = (8 + x)2/3, and choose a = 0 as our centre.

Then f ′(x) =2

3(8 + x)−1/3, so that

(8.06)2/3 = f(0.06) ≈ f(0) + f ′(0) · 0.06

= 82/3 +2

38−1/3 · 0.06

=3√

82

+2

3 3√

8· 0.06

= 22 +2

3 · 2 · 0.06

= 4 +1

3· 0.06

= 4.02 =402

100

3.4.11.13. *. Solution. We begin by finding the derivatives of f at x = 0.

f(x) = (1− 3x)−1/3 f(0) = 1

f ′(x) = (−3)−1

3(1− 3x)−4/3 = (1− 3x)−4/3 f ′(0) = 1

f ′′(x) = (−3)−4

3(1− 3x)−7/3 = 4(1− 3x)−7/3 f ′′(0) = 4

f (3)(x) = (−3)(4)−7

3(1− 3x)−10/3 = 28(1− 3x)−10/3 f (3)(0) = 28

Plugging these into the definition of a Taylor Polynomial, we find that the third–order Taylor polynomial for f around x = 0 is

T3(x) = 1 + x+4

2!x2 +

28

3!x3

= 1 + x+ 2x2 +14

3x3

920

Page 937: CLP-1 Differential Calculus

SOLUTIONS TO EXERCISES

3.4.11.14. *. Solution.

• By Equation 3.4.33, the absolute value of the error is∣∣∣∣f ′′′(c)3!· (2− 1)3

∣∣∣∣ =

∣∣∣∣ c

6(22− c2)

∣∣∣∣for some c ∈ (1, 2).

• When 1 ≤ c ≤ 2, we know that 18 ≤ 22 − c2 ≤ 21, and that numerator anddenominator are non-negative, so∣∣∣∣ c

6(22− c2)

∣∣∣∣ =c

6(22− c2)≤ 2

6(22− c2)≤ 2

6 · 18

=1

54≤ 1

50

as required.

• Alternatively, notice that c is an increasing function of c, while 22 − c2 is adecreasing function of c. Hence the fraction is an increasing function of cand takes its largest value at c = 2. Hence∣∣∣∣ c

6(22− c2)

∣∣∣∣ ≤ 2

6× 18=

1

54≤ 1

50.

3.4.11.15. *. Solution.

• By Equation 3.4.33, there is c ∈ (0, 0.5) such that the error is

R4 =f (4)(c)

4!(0.5− 0)4

=1

24 · 16· cos(c2)

3− c

• For any c we have | cos(c2)| ≤ 1, and for c < 0.5 we have 3− c > 2.5, so that∣∣∣∣cos(c2)

3− c

∣∣∣∣ ≤ 1

2.5.

• We conclude that

|R4| ≤1

2.5 · 24 · 16=

1

60 · 16<

1

60 · 10=

1

600<

1

500

3.4.11.16. *. Solution.

• By Equation 3.4.33, there is c ∈ (0, 1) such that the error is∣∣∣∣f ′′′(c)3!· (1− 0)3

∣∣∣∣ =

∣∣∣∣ e−c

6(8 + c2)

∣∣∣∣ .921

Page 938: CLP-1 Differential Calculus

SOLUTIONS TO EXERCISES

• When 0 < c < 1, we know that 1 > e−c > e−1 and 8 ≤ 8 + c2 < 9, so∣∣∣∣ e−c

6(8 + c2)

∣∣∣∣ =e−c

6(8 + c2)

<1

6|8 + c2|<

1

6× 8=

1

48<

1

40

as required.

• Alternatively, notice that e−c is a decreasing function of c, while for 0 < c8 + c2 is an increasing function of c. Hence the fraction is a decreasingfunction of c and takes its largest value at c = 0. Hence∣∣∣∣ ec

6(8 + c2)

∣∣∣∣ ≤ 1

6× 8=

1

48<

1

40.

3.4.11.17. *. Solution. 3.4.11.17.a, 3.4.11.17.b:Let f(x) = x1/3 and x0 = 27. Then

f(x) = x1/3 f ′(x) =1

3x−2/3 f ′′(x) = −2

9x−5/3

f(27) = 271/3 = 3 f ′(27) =1

3· 1

32=

1

27f ′′(27) = −2

9· 1

35

= − 2

2187

so that, with x = 25,

52/3 = f(25) ≈ f(27) + f ′(27)(25− 27) = 3− 2

27≈ 2.9259 (linear approx)

52/3 = f(25) ≈ f(27) + f ′(27)(25− 27) +1

2f ′′(27)(25− 27)2

= 3− 2

27− 1

2· 2 · 4

2187≈ 2.9241 (quadratic app)

3.4.11.17.c To obtain an error estimate for the linear approximation, we use that

52/3 = f(25) = f(27) + f ′(27)(25− 27) +1

2f ′′(z)(25− 27)2

for some z between 25 and 27. The error is exactly∣∣∣∣12f ′′(z)(25− 27)2

∣∣∣∣ =

∣∣∣∣12(−2

9x−5/3

)(−2)2

∣∣∣∣ =4

9z−5/3.

For z between 25 and 27, z−5/3 is between 25−5/3 and 27−5/3. The biggest this can

be is 25−5/3, so the maximum possible error is 4

925−5/3.

To get a better idea of what this number is, we note 2.93 < 25, so 4925−5/3 <

492.9−5 = 0.0022.

922

Page 939: CLP-1 Differential Calculus

SOLUTIONS TO EXERCISES

Exercises — Stage 33.4.11.18. Solution. The fourth-degree Maclaurin polynomial for f(x) is

T4(x) = f(0) + f ′(0)x+1

2f ′′(0)x2 +

1

3!f ′′′(0)x3 +

1

4!f (4)(0)x4

while the third-degree Maclaurin polynomial for f(x) is

T3(x) = f(0) + f ′(0)x+1

2f ′′(0)x2 +

1

3!f ′′′(0)x3

So, we simply “chop off” the part of T4(x) that includes x4. Since that’s already0, in this case T3(x) = T4(x).

T3(x) = 5x2 − 9

3.4.11.19. *. Solution. y is a function of x that obeys

y(x)4 + xy(x) = x2 − 1

By implicit differentiation (and then subbing in x = 2, y(2) = 1)

4y(x)3y′(x) + y(x) + xy′(x) = 2x

4y′(2) + 1 + 2y′(2) = 4

y′(2) =1

2

Differentiating with respect to x a second time and then subbing in x = 2, y(2) =1, y′(2) = 1

2:

12y(x)2y′(x)2 + 4y(x)3y′′(x) + y′(x) + y′(x) + xy′′(x) = 2

12× 1× 1

4+ 4y′′(2) +

1

2+

1

2+ 2y′′(2) = 2

6y′′(2) = −2

y′′(2) = −1

3

The tangent line approximation to y(x) at x = 2 is

y(x) ≈ y(2) + y′(2)(x− 2) = 1 +1

2(x− 2)

In particular,

y(2.1) ≈ y(2) + y′(2)(2.1− 2) = 1 +1

2(.1) = 1.05

The quadratic approximation to y(x) at x = 2 is

y(x) ≈ y(2) + y′(2)(x− 2) +1

2y′′(2)(x− 2)2

923

Page 940: CLP-1 Differential Calculus

SOLUTIONS TO EXERCISES

= 1 +1

2(x− 2)− 1

6(x− 2)2

In particular,

y(2.1) ≈ y(2) + y′(2)(2.1− 2) +1

2y′′(2)(2.1− 2)2

= 1 +1

2(.1)− 1

6(.1)2 = 1.0483

At x = 2, y = 1 and y′ = 12. So the tangent line passes through (2, 1) and has

slope 12. At x = 2, y′′ = −1

3, so the graph y = f(x) (locally!) looks like a parabola

pointing down near x = 2. This gives the graph fragment below.Alternatively, we could observe that, near x = 2, y(x) will be quite close to itsquadratic approximation, 1 + 1

2(x− 2)− 1

6(x− 2)2.

3.4.11.20. *. Solution. 3.4.11.20.a y is a function of x that obeys

1 = x4 + y(x) + xy(x)4

By implicit differentiation (and then subbing in x = −1, y(−1) = 1)

0 = 4x3 + y′(x) + y(x)4 + 4xy(x)3y′(x)

0 = −4 + y′(−1) + 1− 4y′(−1)

−1 = y′(−1)

Differentiating with respect to x a second time and then subbing in x = −1,y(−1) = 1, and y′(−1) = −1:

0 = 12x2 + y′′(x) + 4y(x)3y′(x) + 4y(x)3y′(x) + 12xy(x)2y′(x)2

+ 4xy(x)3y′′(x)

0 = 12 + y′′(−1)− 4− 4− 12− 4y′′(−1)

−8 = 3y′′(−1)

y′′(−1) = −8

3

The tangent line approximation to y(x) at x = −1 is

y(x) ≈ y(−1) + y′(−1)(x+ 1) = 1− (x+ 1) = −x

924

Page 941: CLP-1 Differential Calculus

SOLUTIONS TO EXERCISES

In particular,

y(−0.9) ≈ 0.9

3.4.11.20.b The quadratic approximation to y(x) at x = −1 is

y(x) ≈ y(−1) + y′(−1)(x+ 1) +1

2y′′(−1)(x+ 1)2

= 1− (x+ 1)− 4

3(x+ 1)2

In particular,

y(−0.9) ≈ 1− (.1)− 4

3(.1)2 ≈ 0.8867

3.4.11.20.c At x = −1, the slope of the curve is y′(−1) = −1. Its tangent line isfalling at 45. At x = −1, y′′(−1) = −8

3, so the slope of the curve is decreasing

as x passes through −1. Zoomed in very close, the curve looks like a parabolaopening downwards. This gives the figure

3.4.11.21. *. Solution. Let f(x) = log x and x0 = 10. Then

f(x) = log x f ′(x) =1

xf ′′(x) = − 1

x2

f(10) = log 10 ≈ 2.30259 f ′(10) =1

10f ′′(10) = − 1

100

so that, with x = 10.3,

log 10.3 = f(10.3) ≈ f(10) + f ′(10)(10.3− 10) = 2.30259 +0.3

10= 2.33259

The error in this approximation (excluding the error in the given data log 10 ≈2.30259) is

1

2f ′′(z)(10.3 − 10)2 for some z between 10 and 10.3. Because f ′′(z) =

− 1

z2increases as z increases, it must be between − 1

102and − 1

10.32. This forces

1

2f ′′(z)(10.3−10)2 to be between−1

2· 1

102(0.3)2 = −0.00045 and−1

2· 1

10.32(0.3)2 <

−0.00042.

925

Page 942: CLP-1 Differential Calculus

SOLUTIONS TO EXERCISES

3.4.11.22. *. Solution. We begin by finding the values of the derivatives of f atx = 0. We can use the chain rule, or the formula we found in Question 2.14.2.19,Section 2.14.

f(x) = eex

f(0) = e

f ′(x) = exeex

f ′(0) = e

f ′′(x) =(ex + e2x

)eex

f ′′(0) = 2e

f ′′′(x) =(ex + 3e2x + e3x

)eex

(a) L(x) = f(0) + f ′(0)(x− 0) = e+ ex

(b) Q(x) = f(0) + f ′(0)(x− 0) +1

2f ′′(0)(x− 0)2 = e+ ex+ ex2

(c) Since ex2 > 0 for all x > 0, L(x) < Q(x) for all x > 0.From the error formula, we know that

f(x) = f(0) + f ′(0)x+1

2f ′′(0)x2 +

1

3!f ′′′(c)x3

= Q(x) +1

6

(ec + 3e2c + e3c

)eec

x3

for some c between 0 and x. Since 16

(ec + 3e2c + e3c) eec is positive for any c, for

all x > 0, 16

(ec + 3e2c + e3c) eecx3 > 0, so Q(x) < f(x).

(d) Write g(x) = ex = 1 + x+1

2!ecx2, for some c between 0 and x. For x = 0.1 we

have 0 < c < 0.1 and 1 < ec < e0.1 < e < 3. So

e0.1=f(0.1)=1 + 0.1 +1

2ec(0.1)2>1 + 0.1 +

1

2(1)(0.1)2=1.105

e0.1=f(0.1)=1 + 0.1 +1

2ec(0.1)2<1 + 0.1 +

1

2(3)(0.1)2=1.115

That is, 1.105 < e0.1 < 1.115.

3.5 · Optimisation3.5.4 · Exercises· Exercises for § 3.5.1

Exercises — Stage 13.5.4.1. Solution.

926

Page 943: CLP-1 Differential Calculus

SOLUTIONS TO EXERCISES

x

y

y = f(x)

a

When x = 0, the curve y = f(x) appears to have a flat tangent line, so the x = 0 isa critical point. However, it is not a local extremum: it is not true that f(0) ≥ f(x)for all x near 0, and it is not true that f(0) ≤ f(x) for all x near 0.To the right of the x-axis, there is a spike where the derivative of f(x) does notexist. The x-value corresponding to this spike (call it a) is a singular point, andf(x) has a local maximum at x = a.

3.5.4.2. Solution.

x

y

y = f(x)

a b

The x-coordinate corresponding to the blue dot (let’s call it a) is a critical point,because the tangent line to f(x) at x = a is horizontal. There is no lower pointnearby, and actually no lower point on the whole interval shown, so f(x) hasboth a local minimum and a global minimum at x = a.If a function is not continuous at a point, then it is not differentiable at that point.So, the x-coordinate corresponding to the discontinuity (let’s call it b) is a sin-

927

Page 944: CLP-1 Differential Calculus

SOLUTIONS TO EXERCISES

gular point. Values of f(x) immediately to the right of b are lower, and valuesimmediately to the left of b are higher, so f(x) has no local (or global) extremumat x = b.If we wanted to know the derivative of the function at the left endpoint, wewould take the derivative from the right. This looks to be some negative realnumber: so it exists (therefore, the left endpoint is not a singular point) and isnot zero (therefore, the left endpoint is not a critical point). Endpoints can becritical and singular points, but this one happens to be neither.Similarly, at the right endpoint of the interval f ′(x) is some negative real number,so this point is neither critical nor singular.Remark: f(x) has a global maximum at the left-most endpoint of the interval.However, we do not call this a local maximum. In order for f(x) to have a localmaximum at x = c, c must be strictly inside the domain of the function. In short:endpoints can’t be local extrema.

3.5.4.3. Solution. One possible answer is shown below.

x

y

2

For every x in the red interval shown below, f(2) ≥ f(x), so f(2) is a local max-imum. However, the point marked with a blue dot shows that f(x) > f(2) forsome x, so f(2) is not a global maximum.

928

Page 945: CLP-1 Differential Calculus

SOLUTIONS TO EXERCISES

x

y

2

Exercises — Stage 23.5.4.4. Solution. Critical points are those values of x in the domain of f(x) forwhich f ′(x) = 0, and singular points are those values of x in the domain of f(x)for which f(x) is not differentiable. So, we ought to find f ′(x). Using the quotientrule,

f ′(x) =(x− 5)(2x)− (x2 − 10)

(x− 5)2

=x2 − 10x+ 10

(x− 5)2

The only place where this function doesn’t exist is x = 5. Since 5 is not in thedomain of f(x),

(b) f(x) has no singular points.

To find the critical points, we find where f ′(x) = 0. That is:

0 = x2 − 10x+ 10

Using the quadratic formula,

x =10±

√102 − 4(1)(10)

2

= 5±√

15

(a) So the critical points of f(x) are x = 5 +√

15 and x = 5−√

15.(c) Theorem 3.5.4 tells us that local extrema of f(x) can only occur at criticalpoints and singular points. So, the possible points where extrema of f(x) mayexist are x = 5 +

√15 and x = 5−

√15.

929

Page 946: CLP-1 Differential Calculus

SOLUTIONS TO EXERCISES

Exercises — Stage 33.5.4.5. Solution.

x

y

2

local max

x

y

2

neither

x

y

2

neither

x

y

2

local max

For the first curve, the function’s value at x = 2 (that is, the y-value of the soliddot) is higher than anything around it. So, it’s a local maximum.For the second curve, the function’s value at x = 2 (that is, the y-value of the soliddot) is higher than everything to the left, but lower than values immediately tothe right. (On the graph reproduced below, f(x) is higher than everything in thered section, and lower than everything in the blue section.) So, it is neither a localmax nor a local min.

x

y

2

Similarly, for the third curve, f(2) is lower than the values to the right of it, andhigher than values to the left of it, so it is neither a local minimum nor a localmaximum.In the final curve, f(2) (remember–this is the y-value of the solid dot) is higherthan everything immediately to the left or right of it (for instance, over the inter-val marked in red below), so it is a local maximum.

x

y

2

3.5.4.6. Solution. By Theorem 3.5.4, if x = 2 not a critical point, then it must bea singular point. That is, f(x) is not differentiable at x = 2. Also, remember that

930

Page 947: CLP-1 Differential Calculus

SOLUTIONS TO EXERCISES

endpoints are not local extrema. Two possibilities are shown below, but there areinfinitely many possible answers.

x

y

2x

y

2

3.5.4.7. Solution. Critical points are those values of x for which f ′(x) = 0, andsingular points are those values of x for which f(x) is not differentiable. So, weought to find f ′(x). Since f(x) has an absolute value sign, let’s re-write it in aversion that is friendlier to differentiation. Remember that |X| = X when X ≥ 0,and |X| = −X when X < 0.

f(x) =√|(x− 5)(x+ 7)|

=

√(x− 5)(x+ 7) if (x− 5)(x+ 7) ≥ 0√−(x− 5)(x+ 7) if (x− 5)(x+ 7) < 0

The product (x − 5)(x + 7) is positive when (x − 5) and (x + 7) have the samesign, and negative when they have opposite signs, so

f(x) =

√(x− 5)(x+ 7) if x ∈ (−∞,−7] ∪ [5,∞)√−(x− 5)(x+ 7) if x ∈ (−7, 5)

Now, when x 6= −7, 5, we can differentiate, using the chain rule.

f ′(x) =

ddx(x−5)(x+7)

2√

(x−5)(x+7)if x ∈ (−∞,−7) ∪ (5,∞)

ddx−(x−5)(x+7)

2√−(x−5)(x+7)

if x ∈ (−7, 5)

?? if x = −7, x = 5

=

2x+2

2√

(x−5)(x+7)if x ∈ (−∞,−7) ∪ (5,∞)

−2x−2

2√−(x−5)(x+7)

if x ∈ (−7, 5)

?? if x = −7, x = 5

We are tempted to say that the derivative doesn’t exist when x = −7 and x = 5,but be careful– we don’t actually know that yet. The formulas we have for thef ′(x) are only good when x is not −7 or 5.

The middle formula−2x− 2

2√−(x− 5)(x+ 7)

tells us x = −1 is a critical point: when

x = −1, f ′(x) is given by the middle line, and it is 0. Note that x = −1 also makes

931

Page 948: CLP-1 Differential Calculus

SOLUTIONS TO EXERCISES

the top formula 0, but f ′(−1) is not given by the top formula, so that doesn’tmatter.What we’ve concluded so far is that x = −1 is a critical point of f(x), and f(x)has no other critical points or singular points when x 6= −7, 5. It remains to figureout what’s going on at−7 and 5. One way to do this is to use the definition of thederivative to figure out what f ′(−7) and f ′(5) are, if they exist. This is somewhatlaborious. Let’s look for a better way.

• First, let’s notice that f(x) is defined for all values of x, thanks to that handyabsolute value sign.

• Next, notice f(x) ≥ 0 for all x, since square roots never give a negativevalue.

• Then if there is some value of x that gives f(x) = 0, that x gives a globalminimum, and therefore a local minimum.

• f(x) = 0 exactly when (x− 5)(x+ 7) = 0, which occurs at x = −7 and x = 5

• Therefore, f(x) has global and local minima at x = −7 and x = 5

• So, x = −7 and x = 5 are critical points or singular points by Theorem 3.5.4.

So, all together:x = −1 is a critical point, and x = −7 and x = 5 are critical points or singularpints (but we don’t know which).Remark: if you would like a review of how to use the definition of the derivative,below we show that f(x) is not differentiable at x = −7. (In fact, x = −7 andx = 5 are both singular points.)

f ′(−7) = limh→0

f(−7 + h)− f(−7)

h

= limh→0

√|(−13 + h)(h)| −

√|0|

h

= limh→0

√|(−13 + h)(h)|

h

Let’s first consider the case h > 0.

limh→0+

√|(−13 + h)(h)|

h= lim

h→0+

√(13− h)(h)

h

= limh→0+

√13h− h2

√h2

= limh→0+

√13h− h2

h2

= limh→0+

√13

h− 1

932

Page 949: CLP-1 Differential Calculus

SOLUTIONS TO EXERCISES

=∞

Since one side of the limit doesn’t exist,

limh→0

f(−7 + h)− (−7)

h= DNE

so f ′(x) is not differentiable at x = −7. Therefore, x = −7 is a singular point.

3.5.4.8. Solution. For any real number c, c is in the domain of f(x) and f ′(c)exists and is equal to zero. So, following Definition 3.5.6, every real number is acritical point of f(x), and f(x) has no singular points.For every number c, let a = c − 1 and b = c + 1, so a < c < b. Then f(x) isdefined for every x in the interval [a, b], and f(x) = f(c) for every a ≤ x ≤ b. Thatmeans f(x) ≤ f(c) and f(x) ≥ f(c). So, comparing with Definition 3.5.3, we seethat f(x) has a global and local maximum AND minimum at every real numberx = c.

· Exercises for § 3.5.2

Exercises — Stage 13.5.4.1. Solution. Two examples are given below, but many are possible.

x

y

y = −x2

x

y

y = −√|x|

If f(x) = −x2 or f(x) = −√|x|, then f(x) has a global maximum at x = 0. Since

f(x) keeps getting more and more strongly negative as x gets farther and fartherfrom 0, f(x) has no global minimum.

3.5.4.2. Solution. Two examples are given below, but many are possible.

933

Page 950: CLP-1 Differential Calculus

SOLUTIONS TO EXERCISES

x

y

y = ex

If f(x) = ex, then f(x) > 0 for all x. As we move left along the x-axis, f(x)gets smaller and smaller, approaching 0 but never reaching it. Since f(x) getssmaller and smaller as we move left, there is no global minimum. Likewise, f(x)increases more and more as we move right, so there is no maximum.

x

y

y = arctanx+ 2

If f(x) = arctan(x) + 2, then f(x) >(−π

2

)+ 2 > 0 for all x.

As we move left along the x-axis, f(x) gets smaller and smaller, approaching(−π

2+ 2)

but never reaching it. Since f(x) gets smaller and smaller as we moveleft, there is no global minimum.Likewise, as we move right along the x-axis, f(x) gets bigger and bigger, ap-proaching

(π2

+ 2)

but never reaching it. Since f(x) gets bigger and bigger as wemove right, there is no global maximum.

3.5.4.3. Solution. Since f(5) is a global minimum, f(5) ≤ f(x) for all x, and soin particular f(5) ≤ f(−5).Similarly, f(−5) ≤ f(x) for all x, so in particular f(−5) ≤ f(5).Since f(−5) ≤ f(5) AND f(5) ≤ f(−5), it must be true that f(−5) = f(5).A sketch of one such graph is below.

934

Page 951: CLP-1 Differential Calculus

SOLUTIONS TO EXERCISES

x

y

y = f(x)

−5 5

Exercises — Stage 23.5.4.4. Solution. Global extrema will occur at critical or singular points in theinterval (−5, 5) or at the endpoints x = 5, x = −5.f ′(x) = 2x + 6. Since this is defined for all real numbers, there are no singularpoints. The only time f ′(x) = 0 is when x = −3. This is inside the interval [−5, 5].So, our points to check are x = −3, x = −5, and x = 5.

c −3 −5 5

type critical point endpoint endpointf(c) −19 −15 45

The global maximum is 45 at x = 5 and the global minimum is −19 at x = −3.

3.5.4.5. Solution. Global extrema will occur at the endpoints of the interval,x = −4 and x = 0, or at singular or critical points inside the interval. Since f(x)is a polynomial, it is differentiable everywhere, so there are no singular points.To find the critical points, we set the derivative equal to zero.

f ′(x) = 2x2 − 4x− 30

0 = 2x2 − 4x− 30 = (2x− 10)(x+ 3)

x = 5, −3

The only critical point inside the interval is x = −3.

c −3 −4 0

type critical point endpoint endpointf(c) 61 157

3= 52 + 1

37

The global maximum over the interval is 61 at x = −3, and the global minimumis 7 at x = 0.

935

Page 952: CLP-1 Differential Calculus

SOLUTIONS TO EXERCISES

· Exercises for § 3.5.3

Exercises — Stage 13.5.4.1. *. Solution. We compute f ′(x) = 5x4− 5, which means that f(x) has nosingular points (i.e., it is differentiable for all values of x), but it has two criticalpoints:

0 = 5x4 − 5

0 = x4 − 1 = (x2 + 1)(x2 − 1)

0 = x2 − 1

x = ±1

Note, however, that 1 is not in the interval [−2, 0].The global maximum and the global minimum for f(x) on the interval [−2, 0]will occur at x = −2, x = 0, or x = −1.

c −2 0 −1

type endpoint endpoint critical pointf(c) −20 2 6

So, the global maximum is f(−1) = 6 while the global minimum is f(−2) = −20.

3.5.4.2. *. Solution. We compute f ′(x) = 5x4 − 5, which means that f(x) has nosingular points (i.e., it is differentiable for all values of x), but it has two criticalpoints:

0 = 5x4 − 5

0 = x4 − 1 = (x2 + 1)(x2 − 1)

0 = x2 − 1

x = ±1

Note, however, that −1 is not in the interval [0, 2].The global maximum and the global minimum for f(x) on the interval [0, 2] willoccur at x = 2, x = 0, or x = 1.

c 2 0 1

type endpoint endpoint critical pointf(c) 12 −10 −14

So, the global maximum is f(2) = 12 while the global minimum is f(1) = −14.

3.5.4.3. *. Solution. We compute f ′(x) = 6x2 − 12x = 6x(x − 2), which meansthat f(x) has no singular points (i.e., it is differentiable for all values of x), butit has the two critical points: x = 0 and x = 2. Note, however, 0 is not in theinterval [1, 4].

936

Page 953: CLP-1 Differential Calculus

SOLUTIONS TO EXERCISES

c 1 4 2

type endpoint endpoint critical pointf(c) −6 30 −10

So, the global maximum is f(4) = 30 while the global minimum is f(2) = −10.

3.5.4.4. *. Solution. Since h(x) is a polynomial, it has no singular points. Wecompute its critical points:

h′(x) = 3x2 − 12

0 = 3x2 − 12

x = ±2

Notice as x → ∞, h(x) → ∞, and as x → −∞ h(x) → −∞. So Theorem 3.5.17doesn’t exactly apply. Instead, let’s consider the signs of h′(x).

x (−∞,−2) (−2, 2) (2,∞)

h′(x) > 0 < 0 > 0

h(x) increasing decreasing increasing

So, h(x) increases until x = −2, then decreases. That means h(x) has a localmaximum at x = −2. The function decreases from −2 until 2, after which isincreases, so h(x) has a local minimum at x = 2. We compute f(−2) = 20 andf(2) = −12.

3.5.4.5. *. Solution. Since h(x) is a polynomial, it has no singular points. Wecompute its critical points:

h′(x) = 6x2 − 24

0 = 6x2 − 24

x = ±2

Notice as x → ∞, h(x) → ∞, and as x → −∞ h(x) → −∞. So Theorem 3.5.17doesn’t exactly apply. Instead, let’s consider the signs of h′(x).

x (−∞,−2) (−2, 2) (2,∞)

h′(x) > 0 < 0 > 0

h(x) increasing decreasing increasing

So, h(x) increases until x = −2, then decreases. That means h(x) has a localmaximum at x = −2. The function decreases from −2 until 2, after which isincreases, so h(x) has a local minimum at x = 2.We compute f(−2) = 33 and f(2) = −31.

937

Page 954: CLP-1 Differential Calculus

SOLUTIONS TO EXERCISES

3.5.4.6. *. Solution. Suppose that Q is a distance of x from A. Then it is adistance of 18− x from B.

A Q B

P

12 km

x 18− x

Using the Pythagorean Theorem, the distance from P to Q is√

122 + x2 kilome-tres, and the buggy travels 15 kph over this off-road stretch. The travel time from

P to Q is√

122 + x2

15hours.

The distance from Q to B is 18−x kilometres, and the dune buggy travels 30 kph

along this road. The travel time from Q to B is18− x

30hours. So, the total travel

time is

f(x) =

√122 + x2

15+

18− x30

.

We wish to minimize this for 0 ≤ x ≤ 18. We will test all singular points, criticalpoints, and endpoints to find which yields the smallest value of f(x). Since thereare no singular points, we begin by locating the critical points.

0 = f ′(x) =1

15· 1

2(144 + x2)−1/2(2x)− 1

301

15· x√

144 + x2=

1

30x√

144 + x2=

1

2

x2

144 + x2=

1

4

4x2 = 144 + x2

x =12√

3= 4√

3

So the minimum travel times must be one of f(0), f(18), and f(4√

3).

f(0) =12

15+

18

30= 1.4

f(18) =

√122 + 182

15≈ 1.44

f(

4√

3)

=

√144 + 144/3

15+

18− 12/√

3

30≈ 1.29

So Q should be 4√

3 km from A.

938

Page 955: CLP-1 Differential Calculus

SOLUTIONS TO EXERCISES

3.5.4.7. *. Solution. Let `, w and h denote the length, width and height of the

box respectively. We are told that `wh = 4500 and that ` = 3w. Hence h =4500

`w=

4500

3w2=

1500

w2. The surface area of the box is

A = 2`w + 2`h+ 2wh = 2

(3w2 + 3w

1500

w2+ w

1500

w2

)= 2

(3w2 +

6000

w

)= 6

(w2 +

2000

w

)

w

h

`

wh

`w

`h

As w tends to zero or to infinity, the surface area approaches infinity. By Theo-

rem 3.5.17 the minimum surface area must occur at a critical point of w2 +2000

w.

0 =d

dw

w2 +

2000

w

= 2w − 2000

w2

2w =2000

w2

w3 = 1000

w = 10

Therefore,

` = 3w = 30

h =1500

w2= 15.

The dimensions of the box with minimum surface area are 10× 30× 15.

3.5.4.8. *. Solution. Let the length of the sides of the square base be b metresand let the height be h metres. The area of the base is b2, the area of the top is b2

939

Page 956: CLP-1 Differential Calculus

SOLUTIONS TO EXERCISES

and the area of each of the remaining four sides is bh so the total cost is

5(b2)︸︷︷︸cost of base

+ 1(b2 + 4bh)︸ ︷︷ ︸cost of 5 sides

= 6b2 + 4bh = 72

Solving for h

h =72− 6b2

4b

=6

4

(12− b2

b

)=

3

2

(12− b2

b

)The volume is

V = b2h = b2 · 3

2

(12− b2

b

)= 18b− 3

2b3.

This is the function we want to maximize. Since volume is never negative, theendpoints of the functions are the values of b that make the volume 0. So, themaximum volume will not occur at an endpoint, it will occur at a critical point.The only critical point is b = 2:

0 =d

db

18b− 3

2b3

= 18− 9

2b2

b2 = 4

b = 2, h =3

2

(12− 4

2

)= 6

The desired dimensions are 2× 2× 6.

3.5.4.9. *. Solution. It suffices to consider X and Y such that the line XY istangent to the circle. Otherwise we could reduce the area of the triangle by, forexample, holding X fixed and reducing Y . So let X and Y be the x– and y–

intercepts of the line tangent to the circle at (cos θ, sin θ). Then1

X= cos θ and

1

Y= cos

(π2− θ)

= sin θ. The area of the triangle is

1

2XY =

1

2 cos θ sin θ=

1

sin(2θ)

940

Page 957: CLP-1 Differential Calculus

SOLUTIONS TO EXERCISES

This is a minimum when sin(2θ) is a maximum. That is when 2θ =π

2. Hence

X =1

cos(π/4)and Y =

1

sin(π/4). That is, X = Y =

√2.

3.5.4.10. *. Solution. For ease of notation, we place the semicircle on a Cartesianplane with diameter along the x-axis and centre at the origin.

x

y

x

R

If x is the point where the rectangle touches the diameter to the right of the y-axis, then 2x is the width of the rectangle. The origin and the two right cornersof the rectangle form a right triangle with hypotenuse R, so by the PythagoreanTheorem, the upper right hand corner of the rectangle is at

(x,√R2 − x2

). The

perimeter of the rectangle is given by the function:

P (x) = 4x+ 2√R2 − x2

So, this is what we optimize. The endpoints of the domain for this function arex = 0 and x = R. To find the critical points, we differentiate:

P ′(x) = 4− 2x√R2 − x2

P ′(x) = 0 ⇐⇒ 4 =2x√

R2 − x2

x = 2√R2 − x2

x2 = 4(R2 − x2)

5x2 = 4R2

x =2√5R

941

Page 958: CLP-1 Differential Calculus

SOLUTIONS TO EXERCISES

Note that since our perimeter formula was defined to work only for x in [0, R],

we neglect the negative square root, − 2√5R.

Now, we find the size of the perimeter at the critical point and the endpoints:

c 0 R 2√5R

type endpoint endpoint critical pointP (c) 2R 4R 2

√5R

So, the largest possible perimeter is 2√

5R and the smallest possible perimeter is2R.Remark: as a check on the correctness of our formula for P (x), when x = 0 therectangle degenerates to the line segment from (0, 0) to (0, R). The perimeter ofthis “width zero rectangle” is 2R, agreeing with P (0). Similarly, when x = R therectangle degenerates to the line segment from (R, 0) to (−R, 0). The perimeterof this “width zero rectangle” is 4R, agreeing with P (R).

3.5.4.11. *. Solution. Let the cylinder have radius r and height h. If we imaginepopping off the ends, they are two circular disks, each with surface area πr2.Then we imagine unrolling the remaining tube. It has height h, and its otherdimension is given by the circumference of the disks, which is 2πr. Then the areaof the “unrolled tube” is 2πrh.

h

r

r

h(2πr)h

πr2

So, the surface area is 2πr2 + 2πrh. Since the area is given as A, we can solve forh:

A = 2πr2 + 2πrh

2πrh = A− 2πr2

h =A− 2πr2

2πr.

942

Page 959: CLP-1 Differential Calculus

SOLUTIONS TO EXERCISES

Then we can write the volume as a function of the variable r and the constant A:

V (r) = πr2h

= πr2

(A− 2πr2

2πr

)=

1

2

(Ar − 2πr3

)This is the function we want to maximize. Let’s find its critical points.

V ′(r) =1

2

(A− 6πr2

)V ′(r) = 0 ⇐⇒ A = 6πr2 ⇐⇒ r =

√A

since negative values of r don’t make sense. At this critical point,

V

(√A

)=

1

2

A(√ A

)− 2π

(√A

)3

=1

2

[A3/2

√6π− 2πA3/2

6π√

]=

1

2

[A3/2

√6π− A3/2

3√

]=

A3/2

3√

6π.

We should also check the volume of the cylinder at the endpoints of the function.Since r ≥ 0, one endpoint is r = 0. Since h ≥ 0, and r grows as h shrinks, theother endpoint is whatever value of r causes h to be 0. We could find this valueof r, but it’s not strictly necessary: when r = 0, the volume of the cylinder iszero, and when h = 0, the volume of the cylinder is still zero. So, the maximumvolume does not occur at the endpoints.Therefore, the maximum volume is achieved at the critical point, where

Vmax =A3/2

3√

6π.

Remark: as a check,A has unitsm2 and, because of theA3/2, our answer has unitsm3, which are the correct units for a volume.

3.5.4.12. *. Solution. Denote by r the radius of the semicircle, and let h be theheight of the recangle.

943

Page 960: CLP-1 Differential Calculus

SOLUTIONS TO EXERCISES

r

h

πr

2r

h h

Since the perimeter is required to be P , the height, h, of the rectangle must obey

P = πr + 2r + 2h

h =1

2(P − πr − 2r)

So the area is

A(r) =1

2πr2 + 2rh

=1

2πr2 + r(P − πr − 2r)

= rP − 1

2(π + 4)r2

Finding all critical points:

0 = A′(r) = P − (π + 4)r

r =P

π + 4

Now we want to know what radius yields the maximum area. We notice that

A′(r) > 0 for r <P

π + 4and A′(r) < 0 for r >

P

π + 4. So, A(r) is increasing until

the critical point, then decreasing after it. That means the global maximum occurs

at the critical point, r =P

π + 4. The maximum area is

rP − 1

2(π + 4)r2 =

P 2

π + 4− 1

2(π + 4)

P 2

(π + 4)2

=P 2

2(π + 4)

944

Page 961: CLP-1 Differential Calculus

SOLUTIONS TO EXERCISES

Remark: another way to see that the global maximum occurs at the critical pointis to compare the area at the critical point to the areas at the endpoints of the func-

tion. The smallest value of r is 0, while the biggest isP

π + 2(when the shape is

simply a half-circle). Comparing A(0), A(

P

π + 2

), and A

(P

π + 4

)is somewhat

laborious, but certainly possible.

3.5.4.13. *. Solution.

y = pxx

z

a The surface area of the pan is

xy + 2xz + 2yz = px2 + 2xz + 2pxz

= px2 + 2(1 + p)xz

and the volume of the pan is xyz = px2z. Assuming that all A cm2 is used,we have the constraint

px2 + 2(1 + p)xz = A or z =A− px2

2(1 + p)x

So

V (x) = xyz = x(px)

(A− px2

2(1 + p)x

)=

p

2(1 + p)x(A− px2)

Using the product rule,

V ′(x) =p

2(1 + p)

[x(−2px) + (A− px2)

]=

p

2(1 + p)

[A− 3px2

]The derivative V ′(x) is 0 when x =

√A

3p. The derivative is positive (i.e.

V (x) is increasing) for x <

√A

3pand is negative (i.e. V (x) is decreasing)

for x >√A

3p. So the pan of maximum volume has dimensions x =

√A

3p,

y = p

√A

3p=

√Ap

3and z =

2A/3

2(1 + p)√A/(3p)

=

√Ap√

3(1 + p).

945

Page 962: CLP-1 Differential Calculus

SOLUTIONS TO EXERCISES

b The volume of the pan from part (a) is

V (p) =

(√A

3p

)(p

√A

3p

) √Ap√

3(1 + p)=

(A

3

)3/2 √p1 + p

Since

d

dp

√p

1 + p

=

12(1 + p)/

√p−√p

(1 + p)2=

√p

(1

p− 1

)2(1 + p)2

the volume is increasing with p for p < 1 and decreasing with p for p > 1.So the maximum volume is achieved for p = 1 (a square base).

Exercises — Stage 33.5.4.14. *. Solution. 3.5.4.14.a We use logarithmic differentiation.

f(x) = xx

log f(x) = log (xx) = x log x

d

dxlog f(x) =

d

dxx log x

f ′(x)

f(x)= x

(1

x

)+ log x = 1 + log x

f ′(x) = f(x) (1 + log x) = xx(1 + log x)

3.5.4.14.b Since x > 0, xx > 0. Therefore,

f ′(x) = 0 ⇐⇒ 1 + log x = 0 ⇐⇒ log x = −1 ⇐⇒ x =1

e

3.5.4.14.c Since x > 0, xx > 0. So, the sign of f ′(x) is the same as the sign of1 + log x.

For x <1

e, log x < −1 and f ′(x) < 0. That is, f(x) decreases as x increases, when

x <1

e. For x >

1

e, log x > −1 and f ′(x) > 0. That is, f(x) increases as x increases,

when x >1

e. Hence f(x) is a local minimum at x =

1

e.

3.5.4.15. *. Solution. Call the length of the wire L units and suppose that it iscut ` units from one end. Make the square from the piece of length `, and makethe circle from the remaining piece of length L− `.

The square has perimeter `, so its side length is `/4 and its area is(`

4

)2

. The

circle has circumference L− `, so its radius isL− `

2πand its area is π

(L− `

)2

=

946

Page 963: CLP-1 Differential Calculus

SOLUTIONS TO EXERCISES

(L− `)2

4π.

The area enclosed by the shapes, when the square is made from a length of size`, is

A(`) =`2

16+

(L− `)2

We want to find the global max and min for this function, given the constraint0 ≤ ` ≤ L, so we find its derivative:

A′(`) =`

8− L− `

2π=π + 4

8π`− L

Now, we find the critical point.

A′(`) = 0

π + 4

8π` =

L

` =4L

π + 4

` 0 L..

.4Lπ+4

type endpoint endpoint critical pointA(`)

..

.L2

4πL2

16A(

4Lπ+4

)It seems obnoxious to evaluate A

(4Lπ+4

), and the problem doesn’t ask for it–but

we still have to figure out whether it is a global max or min.When ` < 4L

π+4, A′(`) < 0, and when ` > 4L

π+4, A′(`) > 0. So, A(`) is decreasing

until ` = 4Lπ+4

, then increasing. That means our critical point ` = 4Lπ+4

is a localminimum.

So, the minimum occurs at the only critical point, which is ` =

(4

4 + π

)L. This

corresponds to`

L=

4

4 + π: the proportion of the wire that is cut is

4

4 + π.

The maximum has to be either at ` = 0 or at ` = L. As A(0) =L2

4π> A(L) =

L2

16,

the maximum has ` = 0 (that is, no square).

3.6 · Sketching Graphs3.6.7 · Exercises· Exercises for § 3.6.1

Exercises — Stage 1

947

Page 964: CLP-1 Differential Calculus

SOLUTIONS TO EXERCISES

3.6.7.1. Solution. In general, this is false. For example, the function f(x) =x2 − 9

x2 − 9has no vertical asymptotes, because it is equal to 1 in every point in its

domain (and is undefined when x = ±3).However, it is certainly possible that f(x) has a vertical asymptote at x = −3. For

example, f(x) =1

x2 − 9has a vertical asymptote at x = −3. More generally, if

g(x) is continuous and g(−3) 6= 0, then f(x) has a vertical asympotote at x = −3.

Exercises — Stage 23.6.7.2. Solution. Since x2 + 1 and x2 + 4 are always positive, f(x) and h(x) aredefined over all real numbers. So, f(x) and h(x) correspond to A(x) and B(x).Which is which? A(0) = 1 = f(0) while B(0) = 2 = h(0), so A(x) = f(x) andB(x) = h(x).That leaves g(x) and k(x) matching to C(x) and D(x). The domain of g(x) is all xsuch that x2 − 1 ≥ 0. That is, |x| ≥ 1, like C(x). The domain of k(x) is all x suchthat x2 − 4 ≥ 0. That is, |x| ≥ 2, like D(x). So, C(x) = g(x) and D(x) = k(x).

3.6.7.3. Solution. (a) Since f(0) = 2, we solve

2 =

√log2(0 + p)

=

√log2 p

= |log p|log p = ±2

p = e±2

p = e2 or p =1

e2

We know that p is e2 or1

e2, but we have to decide between the two. In both cases,

f(0) = 2. Let’s consider the domain of f(x). Since log2(x + p) is never negative,the square root does not restrict our domain. However, we can only take thelogarithm of positive numbers. Therefore, the domain is

x such that x+ p > 0

x such that x > −p

If p =1

e2, then the domain of f(x) is

(− 1

e2,∞)

. In particular, since − 1

e2> −1,

the domain of f(x) does not include x = −1. However, it is clear from the graphthat f(−1) exists. So, p = e2.(b) Now, we need to figure out what b is. Notice that b is the end of the domainof f(x), which we already found to be (−p,∞). So, b = −p = −e2.(As a quick check, if we take e ≈ 2.7, then−e2 = −7.29, and this looks about right

948

Page 965: CLP-1 Differential Calculus

SOLUTIONS TO EXERCISES

on the graph.)(c) The x-intercept is the value of x for which f(x) = 0:

0 =

√log2(x+ p)

0 = log(x+ p)

1 = x+ p

x = 1− p = 1− e2

The x-intercept is 1− e2.(As another quick check, the x-intercept we found is a distance of 1 from thevertical asymptote, and this looks about right on the graph.)

3.6.7.4. Solution. Vertical asymptotes occur where the function blows up. Inrational functions, this can only happen when the denominator goes to 0. In ourcase, the denominator is 0 when x = 3, and in this case the numerator is 147. Thatmeans that as x gets closer and closer to 3, the numerator gets closer and closerto 147 while the denominator gets closer and closer to 0, so |f(x)| grows withoutbound. That is, there is a vertical asymptote at x = 3.The horizontal asymptotes are found by taking the limits as x goes to infinity andnegative infinity. In our case, they are the same, so we condense our work.

limx→±∞

x(2x+ 1)(x− 7)

3x3 − 81= lim

x→±∞2x3 + ax2 + bx+ c

3x3 − 81

where a, b, ad c are some constants. Remember, for rational functions, youcan figure out the end behaviour by looking only at the terms with the high-est degree–the others won’t matter, so we don’t bother finding them. From here,we divide the numerator and denominator by the highest power of x in the de-nominator, x3.

= limx→±∞

2x3 + ax2 + bx+ c

3x3 − 81

( 1x3

1x3

)= lim

x→±∞

2 + ax

+ bx2

+ cx3

3− 81x3

=2 + 0 + 0 + 0

3− 0=

2

3

So there is a horizontal asymptote of y =2

3both as x→∞ and as x→ −∞.

3.6.7.5. Solution. Since f(x) is continuous over all real numbers, it has no ver-tical asymptote.To find the horizontal asymptotes, we evaluate lim

x→±∞f(x).

limx→∞

103x−7 = limX→∞

10X︸ ︷︷ ︸let X=3x−7

=∞

949

Page 966: CLP-1 Differential Calculus

SOLUTIONS TO EXERCISES

So, there’s no horizontal asymptote as x→∞.

limx→−∞

103x−7 = limX→−∞

10X︸ ︷︷ ︸let X=3x−7

= limX′→∞

10−X′︸ ︷︷ ︸

let X′=−X

= limX′→∞

1

10X′

= 0

That is, y = 0 is a horizontal asymptote as x→ −∞.

· Exercises for § 3.6.2

Exercises — Stage 13.6.7.1. Solution. Functions A(x) and B(x) share something in common thatsets them apart from the others: they have a horizontal tangent line only once. Inparticular, A′(−2) 6= 0 and B′(2) 6= 0. The only listed functions that do not havetwo distinct roots are l(x) and p(x). Since l(−2) 6= 0 and p(2) 6= 0, we conclude

A′(x) = l(x) B′(x) = p(x)

Function C(x) is never decreasing. Its tangent line is horizontal when x = ±2,but the curve never decreases, so C ′(x) ≥ 0 for all x and C ′(2) = C ′(−2) = 0. Theonly function that matches this is n(x) = (x − 2)2(x + 2)2. Since its linear termshave even powers, it is never negative, and its roots are precisely x = ±2.

C ′(x) = n(x)

For the functions D(x) and E(x) we consider their behaviour near x = 0. D(x) isdecreasing near x = 0, so D′(0) < 0, which matches with o(0) < 0. Contrastingly,E(x) is increasing near zero, so E ′(0) > 0, which matches with m(0) > 0.

D′(x) = o(x) E ′(x) = m(x)

Exercises — Stage 23.6.7.2. *. Solution. The domain of f(x) is all real numbers except −3 (becausewhen x = −3 the denominator is zero). For x 6= −3, we differentiate using thequotient rule:

f ′(x) =ex(x+ 3)− ex(1)

(x+ 3)2=

ex

(x+ 3)2(x+ 2) .

Since ex and (x + 3)2 are positive for every x in the domain of f(x), the sign off ′(x) is the same as the sign of x+2. We conclude that f(x) is increasing for everyx in its domain with x+ 2 > 0. That is, over the interval (−2,∞).

950

Page 967: CLP-1 Differential Calculus

SOLUTIONS TO EXERCISES

3.6.7.3. *. Solution. Since we can’t take the square root of a negative number,f(x) is only defined when x ≥ 1. Furthermore, since we can’t have a zero as adenominator, x = −2 is not in the domain–but as long as x ≥ 1, also x 6= −2. So,the domain of the function is [1,∞).In order to find where is f(x) increasing, we find where is f ′(x) positive.

f ′(x) =

2x+42√x−1− 2√x− 1

(2x+ 4)2=

(x+ 2)− 2(x− 1)√x− 1(2x+ 4)2

=−x+ 4√

x− 1(2x+ 4)2

The denominator is never negative, so f(x) is increasing when the numerator off ′(x) is positive, i.e. when 4 − x > 0, or x < 4. Recalling that the domain ofdefinition for f(x) is [1,+∞), we conclude that f(x) is increasing on the interval(1, 4).Remark: [1, 4) is also an acceptable answer, but [1, 4] is not. At x = 4 the functionhas a local maximum: f(x) is increasing before x = 4 and decreasing after.

3.6.7.4. *. Solution. The domain of arctangent is all real numbers. The domainof the logarithm function is all positive numbers, and 1 + x2 is positive for all x.So, the domain of f(x) is all real numbers.In order to find where is f(x) increasing, we find where is f ′(x) positive.

f ′(x) =2

1 + x2− 2x

1 + x2=

2− 2x

1 + x2

Since the denominator is always positive, f(x) is increasing when when 2− 2x >0.We conclude that f(x) is increasing on the interval (−∞, 1).

· Exercises for § 3.6.3

Exercises — Stage 13.6.7.1. Solution.

951

Page 968: CLP-1 Differential Calculus

SOLUTIONS TO EXERCISES

x

y

concave up concave down concave up

concave down

In the graph above, the concave-up sections are marked in red. These are wherethe graph has an increasing derivative; equivalently, where the graph lies aboveits tangent lines; more descriptively, where it curves like a smiley face.Concave-down sections are marked in blue. These are where the graph has adecreasing derivative; equivalently, where the graph lies below its tangent lines;more descriptively, where it curves like a frowney face.

3.6.7.2. Solution. The most basic shape of the graph is given by the last twobullet points:

x

y

−5 5

The curve is concave down over the interval (−5, 5), so let’s give it a frowney-facecurvature there.

952

Page 969: CLP-1 Differential Calculus

SOLUTIONS TO EXERCISES

x

y

−5 5

Finally, when x > 5 or x < −5, our curve should be concave up, so let’s giveit smiley-face curvature there, without changing its basic increasing/decreasingshape.

x

y

−5 5

This finishes our sketch.

3.6.7.3. Solution. An inflection point is where the concavity of a functionchanges. It is possible that x = 3 is an inflection point, but it is also possiblethat is not. So, the statement is false, in general.For example, let f(x) = (x − 3)4. Since f(x) is a polynomial, all its derivativesexist and are continuous. f ′′(x) = 12(x − 3)2, so f(3) = 0. However, since f ′′(x)is something squared, it is never negative, so f(x) is never concave down. Sincef(x) is never concave down, it never changes concavity, so it has no inflectionpoints.Remark: finding inflection points is somewhat reminiscent of finding local ex-trema. To find local extrema, we first find all critical and singular points, sincelocal extrema can only occur there. Then, we have to figure out which criticaland singular points are actually local extrema. Similarly, if you want to findinflection points, start by finding where f ′′(x) is zero or non-existant, because in-flection points can only occur there (see Question 3.6.7.7). Then, you still have to

953

Page 970: CLP-1 Differential Calculus

SOLUTIONS TO EXERCISES

check whether those points are actually inflection points.

Exercises — Stage 23.6.7.4. *. Solution. Inflection points occur where f ′′(x) changes sign. Sincef(x) is a polynomial, its first and second derivatives exist everywhere, and arethemselves polynomials. In particular,

f(x) = 3x5 − 5x4 + 13x

f ′(x) = 15x4 − 20x3 + 13

f ′′(x) = 60x3 − 60x2 = 60x2(x− 1)

The second derivative is negative for x < 1 and positive for x > 1. Thus theconcavity changes between concave up and concave down at x = 1, y = 11.This is the only inflection point. It is true that f ′′(0) = 0, but for values of x botha little larger than and a little smaller than 0, f ′′(x) < 0, so the concavity does notchange at x = 0.

Exercises — Stage 33.6.7.5. *. Solution. In order to show that f(x) has exactly one inflection point,we will show that is has at least one, and no more than one.Let

g(x) = f ′′(x) = x3 + 5x− 20.

Then g′(x) = 3x2 + 5, which is always positive. That means g(x) is strictly in-creasing for all x. So, g(x) can change signs once, from negative to positive, but itcan never change back to negative. An inflection point of f(x) occurs when g(x)changes signs. So, f(x) has at most one inflection point. (At this point, we don’tknow that f(x) has any inflection points: maybe g(x) is always positive.)Since g(x) is continuous, we can apply the Intermediate Value Theorem to it.Notice g(3) > 0 while g(0) < 0. By the IVT, g(x) = 0 for at least one x ∈ (0, 3).Since g(x) is strictly increasing, at the point where g(x) = 0, g(x) changes fromnegative to positive. So, the concavity of f(x) changes. Therefore, f(x) has atleast one inflection point.Now that we’ve shown that f(x) has at most one inflection point, and at leastone inflection point, we conclude it has exactly one inflection point.

3.6.7.6. *. Solution. 3.6.7.6.a Let

g(x) = f ′(x)

Then f ′′(x) is the derivative of g(x). Since f ′′(x) > 0 for all x, g(x) = f ′(x) isstrictly increasing for all x. In other words, if y > x then g(y) > g(x).Suppose g(x) = 0. Then for every y that is larger than x, g(y) > g(x), so g(y) 6= 0.Similarly, for every y that is smaller than x, g(y) < g(x), so g(y) 6= 0. Therefore,g(x) can only be zero for at most one value of x. Since g(x) = f ′(x), that meansf(x) can have at most one critical point.Suppose f ′(c) = 0. Since f ′(x) is a strictly increasing function, f ′(x) < 0 for all

954

Page 971: CLP-1 Differential Calculus

SOLUTIONS TO EXERCISES

x < c and f ′(x) > 0 for all x > c.

c

x < c, sof ′(x) < f ′(c) = 0

x > c, sof ′(x) > f ′(c) = 0

Then f(x) is decreasing for x < c and increasing for x > c. So f(x) > f(c) for allx < c and f(x) > f(c) for all x > c.

c

f(x) decreasing, sof(x) > f(c)

f(x) increasing, sof(x) > f(c)

y = f(x)

x < c x > c

We have concluded that f(x) > f(c) for all x 6= c, so c is an absolute minimumfor f(x).3.6.7.6.b We know that the maximum over an interval occurs at an endpoint, at acritical point, or at a singular point.

• Since f ′(x) exists everywhere, there are no singular points.

• If the maximum were achieved at a critical point, that critical point wouldhave to provide both the absolute maximum and the absolute minimum(by part a). So, the function would have to be a constant and consequentlycould not have a nonzero second derivative. So the maximum is not at acritical point.

That leaves only the endpoints of the interval.

3.6.7.7. Solution. If x = 3 is an inflection point, then the concavity of f(x)changes at x = 3. That is, there is some interval strictly containing 3, with end-points a and b, such that

• f ′′(a) < 0 and f ′′(x) < 0 for every x between a and 3, and

• f ′′(b) > 0 and f ′′(x) > 0 for every x between b and 3.

Remark: we are leaving unknown whether a < 3 < b or b < 3 < a. Since we don’tknow whether f(x) changes from concave up to concave down, or from concavedown to concave up, by remaining vague we cover both cases.

955

Page 972: CLP-1 Differential Calculus

SOLUTIONS TO EXERCISES

3a b

concave down concave up

OR

3b a

concave up concave down

Since f ′′(a) < 0 and f ′′(b) > 0, and since f ′′(x) is continuous, the IntermediateValue Theorem tells us that there exists some x strictly between a and b withf ′′(x) = 0. So, we know f ′′(x) = 0 somewhere between a and b. The question is,where exactly could that be?

• f ′′(x) < 0 (and hence f ′′(x) 6= 0) for all x between a and 3

• f ′′(x) > 0 (and hence f ′′(x) 6= 0) for all x between b and 3

• So, any number between a and b that is not 3 has f ′′(x) 6= 0.

So, x = 3 is the only possible place between a and b where f ′′(x) could be zero.Therefore, f ′′(3) = 0.Remark: this is why, in general, we set f ′′(x) = 0 to find inflection points. (Theycan also occur where f ′′(x) does not exist.)

· Exercises for § 3.6.4

Exercises — Stage 13.6.7.1. Solution. This function is symmetric across the y-axis, so it is even.

3.6.7.2. Solution. The function is not even, because it is not mirrored across they-axis.Assuming it continues as shown, the function is periodic, because the unit shownbelow is repeated:

956

Page 973: CLP-1 Differential Calculus

SOLUTIONS TO EXERCISES

x

y

y = f(x)

Additionally, f(x) is odd. In a function with odd symmetry, if we mirror theright-hand portion of the curve (the portion to the right of the y-axis) across boththe y-axis and the x-axis, it lines up with the left-hand portion of the curve.

x

y

y = f(x)

reflected across y-axis

957

Page 974: CLP-1 Differential Calculus

SOLUTIONS TO EXERCISES

x

y

y = f(x)

reflected across both axes

Since reflecting the right-hand portion of the graph across the y-axis, then thex-axis, gives us f(x), we conclude f(x) is odd.

3.6.7.3. Solution. Since the function is even, we simply reflect the portionshown across the y-axis to complete the sketch.

x

y

3.6.7.4. Solution. Since the function is odd, to complete the sketch, we reflectthe portion shown across the y-axis (shown dashed), then the x-axis (shown inred).

958

Page 975: CLP-1 Differential Calculus

SOLUTIONS TO EXERCISES

x

y

Exercises — Stage 23.6.7.5. Solution. A function is even if f(−x) = f(x).

f(−x) =(−x)4 − (−x)6

e(−x)2

=x4 − x6

ex2

= f(x)

So, f(x) is even.

3.6.7.6. Solution. For any real number x, we will show that f(x) = f(x+ 4π).

f(x+ 4π) = sin(x+ 4π) + cos

(x+ 4π

2

)= sin(x+ 4π) + cos

(x2

+ 2π)

= sin(x) + cos(x

2

)= f(x)

So, f(x) is periodic.

3.6.7.7. Solution. f(x) is not periodic. (You don’t really have to justify this, butif you wanted to, you could say something like this. Notice f(0) = 1. Wheneverx > 10, f(x) > 1. Then the value of f(0) is not repeated indefinitely, so f(x) is not

959

Page 976: CLP-1 Differential Calculus

SOLUTIONS TO EXERCISES

periodic.)To decide whether f(x) is even, odd, or neither, simplify f(−x):

f(−x) = (−x)4 + 5(−x)2 + cos((−x)3

)= x4 + 5x3 + cos(−x)

= x4 + 5x3 + cos(x)

= f(x)

Since f(−x) = f(x), our function is even.

3.6.7.8. Solution. It should be clear that f(x) is not periodic. (If you wanted tojustify this, you could note that f(x) = 0 has exactly two solutions, x = 0, −5.Since the value of f(0) is repeated only twice, and not indefinitely, f(x) is notperiodic.)To decide whether f(x) is odd, even, or neither, we simplify f(−x).

f(−x) = (−x)5 + 5(−x)4

= −x5 + 5x4

We see that f(−x) is not equal to f(x) or to −f(x). For instance, when x = 1:

• f(−x) = f(−1) = 4,

• f(x) = f(1) = 6, and

• −f(x) = −f(1) = −6.

Since f(−x) is not equal to f(x) or to −f(x), f(x) is neither even nor odd.

3.6.7.9. Solution. Recall the period of g(X) = tanX is π.

tan(X + π) = tan(X) for any X in the domain of tanX

Replacing X with πx:

tan(πx+ π) = tan(πx) for any x in the domain of tan(πx)

tan(π(x+ 1)) = tan(πx) for any x in the domain of tan(πx)

f(x+ 1) = f(x) for any x in the domain of tan(πx)

The period of f(x) is 1.

Exercises — Stage 33.6.7.10. Solution. Let’s consider g(x) = tan(3x) and h(x) = sin(4x) separately.Recall that π is the period of tangent.

tanX = tan(X + π) for every X in the domain of tanX

960

Page 977: CLP-1 Differential Calculus

SOLUTIONS TO EXERCISES

Replacing X with 3x:

tan(3x) = tan(3x+ π) for every x in the domain of tan 3x

tan(3x) = tan(

3(x+

π

3

))for every x in the domain of tan 3x

g(x) = g(x+

π

3

)for every x in the domain of tan 3x

So, the period of g(x) = tan(3x) isπ

3.

Similarly, 2π is the period of sine.

sin(X) = sin(X + 2π) for every X in the domain of sin(X)

Replacing X with 4x:

sin(4x) = sin(4x+ 2π) for every x in the domain of sin(4x)

sin(4x) = sin(

4(x+

π

2

))for every x in the domain of sin(4x)

h(x) = h(x+

π

2

)for every x in the domain of sin(4x)

So, the period of h(x) = sin(4x) isπ

2.

All together, f(x) = g(x) + h(x) will repeat when both g(x) and h(x) repeat. Theleast common integer multiple of

π

3and

π

2is π. Since g(x) repeats every

π

3units,

and h(x) repeats everyπ

2units, they will not both repeat until we move π units.

So, the period of f(x) is π.

· Exercises for § 3.6.6

Exercises — Stage 13.6.7.1. *. Solution. 3.6.7.1.a Since we must have 3 − x ≥ 0, this tells us x ≤ 3.So, the domain is (−∞, 3].3.6.7.1.b

f ′(x) =√

3− x− x

2√

3− x = 32− x

2√

3− xFor every x in the domain of f ′(x), the denominator is positive, so the sign off ′(x) depends only on the numerator.

x (−∞, 2) 2 (2, 3) 3

f ′(x) positive 0 negative DNEf(x) increasing maximum decreasing endpoint

So, f is increasing for x < 2, has a local (in fact global) maximum at x = 2, and isdecreasing for 2 < x < 3.Remark: this shows us the basic skeleton of the graph. It consists of a single

961

Page 978: CLP-1 Differential Calculus

SOLUTIONS TO EXERCISES

hump.

x

2 3

3.6.7.1.c When x < 3,

f ′′(x) =1

4(3x− 12)(3− x)−3/2 < 0

The domain of f ′′(x) is (−∞,−3), and over its domain it is always negative (thefactor (3x − 12) is negative for all x < 4 and the factor (3 − x)−3/2 is positive forall x < 3). So, f(x) has no inflection points and is concave down everywhere.3.6.7.1.d We already found

f ′(x) = 32− x

2√

3− x.

This is undefined at x = 3. Indeed,

limx→3−

32− x

2√

3− x = −∞,

so f(x) has a vertical tangent line at (3, 0).3.6.7.1.e To sketch the curve y = f(x), we already know its intervals of increaseand decrease, and its concavity. We also note its intercepts are (0, 0) and (3, 0).

x

y

(3, 0)

(2.2)

increasing decreasing

concave down

3.6.7.2. *. Solution.

962

Page 979: CLP-1 Differential Calculus

SOLUTIONS TO EXERCISES

• Asymptotes:

limx→±∞

f(x) = limx→±∞

1

x− 2

x4= 0

So y = 0 is a horizontal asymptote both at x =∞ and x = −∞.

limx→0

f(x) = limx→0

x3 − 2

x4= −∞

So there is a vertical asymptote at x = 0, where the function plunges down-wards from both the right and the left.

• Intervals of increase and decrease:

f ′(x) = − 1

x2+

8

x5=

8− x3

x5

The only place where f ′(x) is zero only at x = 2. So f(x) has a horizontaltangent at x = 2, y = 3

8. This is a critical point.

The derivative is undefined at x = 0, as is the function.

x (−∞, 0) 0 (0, 2) 2 (2,∞)

f ′(x) negative DNE positive 0 negativef(x) decreasing vertical asymptote increasing local max decreasing

Since the function changes from increasing to decreasing at x = 2, the onlylocal maximum is at x = 2.

At this point, we get a rough sketch of f(x).

963

Page 980: CLP-1 Differential Calculus

SOLUTIONS TO EXERCISES

x

y

2

horizontal asymptotes y = 0

decreasing decreasingincreasing

(2, 3

8

)

• Concavity:

f ′′(x) =2

x3− 40

x6=

2x3 − 40

x6

The second derivative of f(x) is positive for x > 3√

20 and negative forx < 3

√20. So the curve is concave up for x > 3

√20 and concave down for

x < 3√

20. There is an inflection point at x = 3√

20 ≈ 2.7, y = 18204/3

≈ 0.3.

• Intercepts:

Since f(x) is not defined at x = 0, there is no y-intercept. The only x-intercept is x = 3

√2 ≈ 1.3.

• Sketch:

We can add concavity to our skeleton sketched above, and label our inter-cept and inflection point (the open dot).

964

Page 981: CLP-1 Differential Calculus

SOLUTIONS TO EXERCISES

x

y(2, 3

8

)

2 3√

203√

2

decreasing decreasingincreasing

concave down concave up

3.6.7.3. *. Solution.

• Asymptotes:

When x = −1, the denominator 1 + x3 of f(x) is zero while the numeratoris 1, so x = −1 is a vertical asymptote. More precisely,

limx→−1−

f(x) = −∞ limx→−1+

f(x) =∞

There are no horizontal asymptotes, because

limx→∞

x4

1 + x3=∞ lim

x→−∞x4

1 + x3= −∞

• Intervals of increase and decrease:

We note that f ′(x) is defined for all x 6= −1, so f ′(x) is defined for all x inthe domain of f(x). Therefore, f(x) has no singular points.

To find critical points, we set

f ′(x) = 0

4x3 + x6 = 0

x3(4 + x3) = 0

x3 = 0 or 4 + x3 = 0

x = 0 or x = − 3√

4 ≈ −1.6

At these critical points, f(0) = 0 and f(− 3√

4) = 4 3√4−3

< 0. The denominatorof f ′(x) is never negative, so the sign of f ′(x) is the same as the sign of itsnumerator, x3(4 + x3).

965

Page 982: CLP-1 Differential Calculus

SOLUTIONS TO EXERCISES

x (−∞,− 3√

4) − 3√

4 (− 3√

4,−1) −1 (−1, 0) 0 (0,∞)

f ′(x) positive 0 negative DNE negative 0 positivef(x) increasing l. max decreasing VA decreasing l. min increasing

Now, we have enough information to make a skeleton of our graph.

x

y

−1− 3√

4

increasing decr decr increasing

• Concavity:

The second derivative is undefined when x = −1. It is zero when 12x2 −6x5 = 6x2(2 − x3) = 0. That is, at x = 3

√2 ≈ 1.3 and x = 0. Notice that the

sign of f ′′(x) does not change at x = 0, so x = 0 is not an inflection point.

x (−∞,−1) −1 (−1, 0) 0 (0, 3√

2) 3√

2 ( 3√

2,∞)

f ′′(x) negative DNE positive 0 positive 0 negativef(x) concave down VA concave up concave up IP concave down

Now we can refine our skeleton by adding concavity.

966

Page 983: CLP-1 Differential Calculus

SOLUTIONS TO EXERCISES

x

y

−1− 3√

4 3√

2

increasing decr decr increasing

concave down concave up concave down

3.6.7.4. *. Solution.

• Asymptotes:

limx→−∞

x3

1− x2=∞ lim

x→∞x3

1− x2= −∞

So, f(x) has no horizontal asymptotes.

On the other hand f(x) blows up at both x = 1 and x = −1, so there arevertical asymptotes at x = 1 and x = −1. More precisely,

limx→−1−

x3

1− x2=∞ lim

x→−1+

x3

1− x2= −∞

limx→1−

x3

1− x2=∞ lim

x→1+

x3

1− x2= −∞

• Symmetry:

f(x) is an odd function, because

f(−x) =(−x)3

1− (−x)2=−x3

1− x2= −f(x)

967

Page 984: CLP-1 Differential Calculus

SOLUTIONS TO EXERCISES

• Intercepts:

The only intercept of f(x) is the origin. In particular, that means that outof the three intervals where it is continuous, namely (−∞,−1), (−1, 1) and(1,∞), in two of them f(x) is always positive or always negative.

When x < −1: 1− x2 < 0 and x3 < 0, so f(x) > 0.

When x > 1: 1− x2 < 0 and x3 > 0, so f(x) < 0.

When −1 < x < 0, 1− x2 > 0 and x3 < 0 so f(x) < 0.

When 0 < x < 1, 1− x2 > 0 and x3 > 0 so f(x) > 0.

• Intervals of increase and decrease:

f ′(x) =3x2 − x4

(1− x2)2=x2(3− x2)

(1− x2)2

There are no singular points (but remember the domain of f(x) does notinclude x = ±1 ). The critical points are:

f ′(x) = 0

x2 = 0 or 3− x2 = 0

x = 0 or x = ±√

3 ≈ ±1.7

The values of f at its critical points are f(0) = 0, f(√

3) = −3√

3

2≈ −2.6

and f(−√

3) =3√

3

2≈ 2.6.

Notice the sign of f ′(x) is the same as the sign of 3− x2.

x (−∞,−√

3) −√

3 (−√

3,−1) −1

f ′(x) negative 0 positive DNEf(x) decreasing local min increasing VA

x (−1, 0) 0 (0,√

3)√

3 (√

3,∞)

f ′(x) positive 0 positive 0 negativef(x) increasing increasing local max decreasing

Now we have enough information to sketch a skeleton of f(x).

968

Page 985: CLP-1 Differential Calculus

SOLUTIONS TO EXERCISES

x

y

−1 1−√

3√

3

3√

32

−3√

32

decreasing incr increasing incr decreasing

• Concavity:

f ′′(x) =2x(3 + x2)

(1− x2)3

The second derivative is zero when x = 0, and is undefined when x = ±1.

x (−∞,−1) (−1, 0) 0 (0, 1) (1,∞)

f ′′(x) positive negative 0 positive negativef(x) concave up concave down inflection point concave up concave down

Now, we can refine our skeleton.

969

Page 986: CLP-1 Differential Calculus

SOLUTIONS TO EXERCISES

x

y

−1 1−√

3√

3

3√

32

−3√

32

decreasing incr increasing incr decreasing

concave up cc down concave up concave down

3.6.7.5. *. Solution. 3.6.7.5.a One branch of the function, the exponential func-tion ex, is continuous everywhere. So f(x) is continuous for x < 0. When x ≥ 0,

f(x) =x2 + 3

3(x+ 1), which is continuous whenever x 6= 1 (so it’s continuous for all

x > 0). So, f(x) is continuous for x > 0. To see that f(x) is continuous at x = 0,we see:

limx→0−

f(x) = limx→0−

ex = 1

limx→0+

f(x) = limx→0+

x2 + 3

3(x+ 1)= 1

So, limx→0

f(x) = 1 = f(0)

Hence f(x) is continuous at x = 0, so f(x) is continuous everywhere.3.6.7.5.b We differentiate the function twice. Notice

d

dx

x2 + 3

3(x+ 1)

=

3(x+ 1)(2x)− (x2 + 3)(3)

9(x+ 1)2

=x2 + 2x− 3

3(x+ 1)2

=(x− 1)(x+ 3)

3(x+ 1)2where x 6= −1

970

Page 987: CLP-1 Differential Calculus

SOLUTIONS TO EXERCISES

Then limx→0+

f ′(x) =(0− 1)(0 + 3)

3(0 + 1)2= −1 6= 1 = e0 = lim

x→0−f ′(x)

so f ′(x) =

ex x < 0

DNE x = 0(x−1)(x+3)

3(x+1)2x > 0

Differentiating again,

d2

dx2

x2 + 3

3(x+ 1)

=

d

dx

x2 + 2x− 3

3(x+ 1)2

=

3(x+ 1)2(2x+ 2)− (x2 + 2x− 3)(6)(x+ 1)

9(x+ 1)4 (÷3(x+ 1)

÷3(x+ 1)

)=

(x+ 1)(2x+ 2)− 2(x2 + 2x− 3)

3(x+ 1)3

=8

3(x+ 1)3where x 6= −1

so f ′′(x) =

ex x < 0

DNE x = 08

3(x+1)3x > 0

i. The only singular point is x = 0, and the only critical point is x = 1. (Whenyou’re reading off the expression for f ′(x), remember that the bottom line onlyapplies when x > 0.)

x (−∞, 0) 0 (0, 1) 1 (1,∞)

f ′(x) positive DNE negative 0 positivef(x) increasing local max decreasing local min increasing

The coordinates of the local maximum are (0, 1) and the coordinates of the localminimum are

(1, 2

3

).

ii. When x 6= 0, f ′′(x) is always positive, so f(x) is concave up.iii.

limx→∞

f(x) = limx→∞

x2 + 3

3x+ 3

= limx→∞

x+ 3x

1 + 3x

=∞

So, there is no horizontal asymptote to the right.

limx→−∞

f(x) = limx→−∞

ex = 0

971

Page 988: CLP-1 Differential Calculus

SOLUTIONS TO EXERCISES

So, y = 0 is a horizontal asymptote to the left.Since f(x) is continuous everywhere, there are no vertical asymptotes.3.6.7.5.c

x

y

(0, 1)(1, 2

3)

increasing decr increasing

concave up concave up

3.6.7.6. *. Solution.

• Asymptotes: In the problem statement, we are told:

limx→±∞

1 + 2x

ex2= 0

So, y = 0 is a horizontal asymptote both at x =∞ and at x = −∞.

Since f(x) is continuous, it has no vertical asymptotes.

• Intervals of increase and decrease:

The critical points are the zeroes of 1 − x − 2x2 = (1 − 2x)(1 + x). That is,x = 1

2, −1.

x (−∞,−1) −1 (−1, 12) 1

2(1

2,∞)

f ′(x) negative 0 positive 0 negativef(x) decreasing local min increasing local max decreasing

At these critical points, f(

12

)= 2e−1/4 > 0 and f(−1) = −e−1 < 0.

From here, we can sketch a skeleton of the graph.

972

Page 989: CLP-1 Differential Calculus

SOLUTIONS TO EXERCISES

x

y

−1 12

decreasing increasing decr increasing

• Concavity:

We are told that we don’t have to actually solve for the inflection points.We just need to know enough to get a basic idea. So, we’ll turn the skeletonof the graph into smooth curve.

Inflection points are points where the convexity changes from up to downor vice versa. It looks like our graph is convex down for x from−∞ to about−1.8, convex up from about x = −1.8 to about x = −0.1, convex downfrom about x = −0.1 to about x = 1.4 and convex up from about x = 1.4 toinfinity. So there are three inflection points at roughly x = −1.8, −0.1, 1.4.

3.6.7.7. *. Solution. 3.6.7.7.a We need to know the first and second derivativeof f(x). Using the product and chain rules, f ′(x) = e−x

2/2(1− x2). Given to us isf ′′(x) = (x3−3x)e−x

2/2. (These derivatives are also easy to find using the formuladeveloped in Question 2.14.2.19, Section 2.14.)Since e−x2/2 is always positive, the sign of f ′(x) is the same as the sign of 1 − x2.f(x) has no singular points and its only critical points are x = ±1. At these critical

points, f(−1) = − 1√e

and f(1) =1√e

.

973

Page 990: CLP-1 Differential Calculus

SOLUTIONS TO EXERCISES

x (−∞,−1) −1 (−1, 1) 1 (1,∞)

f ′(x) negative 0 positive 0 negativef(x) decreasing local min increasing local max decreasing

This, together with the observations that f(x) < 0 for x < 0, f(0) = 0 and f(x) >0 for x > 0 (in fact f is an odd function), is enough to sketch a skeleton of ourgraph.

x

y

−1 1− 1√

e

1√e

We can factor f ′′(x) = (x3 − 3x)e−x2/2 = x(x +

√3)(x −

√3)e−x

2/2. Since e−x2/2 isalways positive, the sign of f ′′(x) is the same as the sign of x(x+

√3)(x−

√3).

x (−∞,−√

3) −√

3 (−√

3, 0) 0 (0,√

3)√

3 (√

3,∞)

f ′′(x) negative 0 positive 0 negative 0 positivef(x) concave down IP concave up IP concave down IP concave up

3.6.7.7.b We’ve already seen that f(x) has a local min at x = −1 and a local maxat x = 1.As x tends to negative infinity, f(x) tends to 0, and f(x) is decreasing on(−∞,−1). Then f(x) is between 0 and f(−1) = −1√

eon (−∞,−1). Then f(x)

is increasing on (−1, 1) from f(−1) = −1√e

to f(1) = 1√e. Finally, for x > 1, f(x) is

decreasing from f(1) = 1√e

and tending to 0. So when x > 1, f(x) is between 1√e

and 0.So, over its entire domain, f(x) is between −1√

eand 1√

e, and it only achieves those

values at x = −1 and x = 1, respectively. Therefore, the local and global min off(x) is at (−1, −1√

e), and the local and global max of f(x) is at (1, 1√

e).

3.6.7.7.c In the graph below, open dots are inflection points, and solid dots areextrema.

974

Page 991: CLP-1 Differential Calculus

SOLUTIONS TO EXERCISES

x

y

−1 1−√

3 √3

−1√e

1√e

3.6.7.8. Solution.

• Symmetry:

f(−x) = −x+ 2 sin(−x) = −x− 2 sinx = −f(x)

So, f(x) is an odd function. If we can sketch y = f(x) for nonnegative x, wecan use symmetry to complete the curve for all x.

• Asymptotes:

Since f(x) is continuous, it has no vertical asymptotes. It also has no hori-zontal asymptotes, since

limx→−∞

f(x) = −∞ limx→∞

f(x) =∞

• Intervals of increase and decrease:

Since f(x) is differentiable everywhere, there are no singular points.

f ′(x) = 1 + 2 cos x

So, the critical points of f(x) occur when

cosx = −1

2

x = 2πn± 2π

3for any integer n

For instance, f(x) has critical points at x =2π

3, x =

3, x =

3, and

x =10π

3.

From the unit circle, or the graph of y = 1 + 2 cosx, we see:

975

Page 992: CLP-1 Differential Calculus

SOLUTIONS TO EXERCISES

x(−2π

3, 2π

3

)2π3

(2π3, 4π

3

)4π3

(4π3, 8π

3

)8π3

(8π3, 10π

3

)f ′(x) positive 0 negative 0 positive 0 negativef(x) increasing l. max decreasing l. min increasing l. max decreasing

We have enough information to sketch a skeleton of the curve y = f(x). Weuse the pattern above for the graph to the right of the y-axis, and use oddsymmetry for the graph to the left of the y-axis.

x

y

2π3

4π3

8π3

10π3

14π3

−8π3

−10π3

−14π3

• Concavity:f ′′(x) = −2 sinx

So, f ′′(x) exists everywhere, and is zero for x = π + πn for every integer n.

x (0, π) π (π, 2π) 2π (2π, 3π) 3π (3π, 4π)

f ′′(x) negative 0 positive 0 negative 0 positivef(x) concave down IP concave up IP concave down IP concave up

976

Page 993: CLP-1 Differential Calculus

SOLUTIONS TO EXERCISES

Using these values, and the odd symmetry of f(x), we can refine our skele-ton. The closed dots are local extrema, and the open dots are inflectionpoints occurring at every integer multiple of π.

x

y

2π3

4π3

8π3

10π3

14π3

−2π3

−4π3

−8π3

−10π3

−14π3

3.6.7.9. *. Solution. We first compute the derivatives f ′(x) and f ′′(x).

f ′(x) = 4 cosx+ 4 sin 2x = 4 cos x+ 8 sinx cosx

= 4 cos x(1 + 2 sinx)

f ′′(x) = −4 sinx+ 8 cos 2x = −4 sinx+ 8− 16 sin2 x

= −4(4 sin2 x+ sinx− 2)

The graph has the following features.

• Symmetry: f(x) is periodic of period 2π. We’ll consider only −π ≤ x ≤ π.(Any interval of length 2π will do.)

• y-intercept: f(0) = −2

• Intervals of increase and decrease: f ′(x) = 0 when cosx = 0, i.e. x = ±π2

,

and when sinx = −1

2, i.e. x = −π

6,−5π

6.

977

Page 994: CLP-1 Differential Calculus

SOLUTIONS TO EXERCISES

x (−π,−5π6

)(−5π

6,−π

2

) (−π

2,−π

6

) (−π

6, π

2

) (π2, π)

f ′(x) negative positive negative positive negativef(x) decreasing increasing decreasing increasing decreasing

This tells us local maxima occur at x = ±π2

and local minima occur at x =

−5π

6and x = −π

6.

Here is a table giving the value of f at each of its critical points.

x −5

6π −π

2

π

2

5

sin(x) −12

−1 −12

1

cos(2x) 12

−1 12

−1

f(x) −3 −2 −3 6

From here, we can graph a skeleton of of f(x):

x

y

π−π π2

−π2 −π

6−5π6

6

−3

−2

• Concavity: To find the points where f ′′(x) = 0, set y = sinx, so f ′′(x) =

978

Page 995: CLP-1 Differential Calculus

SOLUTIONS TO EXERCISES

−4(4y2 + y − 2). Then we really need to solve

4y2 + y − 2 = 0 which gives us

y =−1±

√33

8

These two y-values map to the following two x-values, which we’ll name aand b for convenience:

a = arcsin

(−1 +

√33

8

)≈ 0.635

b = arcsin

(−1−

√33

8

)≈ −1.003

However, these are not the only values of x in [−π, π] with sinx = −1±√

338

.The analysis above misses the others because the arcsine function only re-turns numbers in the range

[−π

2, π

2

]. The graph below shows that there

should be other values of x with sinx = −1±√

338

, and hence f ′′(x) = 0.

x

y

y = sinx

−1+√

338

−1−√

338

b a

We can recover the other solutions in [−π, π] by recalling that

sin(x) = sin(π − x)

(see CLP appendix A7). So, if we choose x = arcsin(−1+

√33

8

)≈ 0.635 to

make sin(x) = −1+√

338

so that f ′′(x) = 0, then setting

x = π − a = π − arcsin

(−1 +

√33

8

)≈ 2.507

will also give us sin(x) = −1+√

338

and f ′′(x) = 0. Similarly, setting

x = π − b = π − arcsin

(−1−

√33

8

)≈ 4.145

979

Page 996: CLP-1 Differential Calculus

SOLUTIONS TO EXERCISES

would give us f ′′(x) = 0. However, this value is outside [−π, π]. To findanother solution inside [−π, π] we use the identity

sin(x) = sin(−π − x)

(which we can obtain from the identity we used above and the fact thatsin(θ) = sin(θ ± 2π) for any angle θ). Using this, we can show that

x = −π − b = −π − arcsin

(−1−

√33

8

)≈ −2.139

also gives f ′′(x) = 0.

So, all together, f ′′(x) = 0 when x = −π − b, x = b, x = a, and x = π − a.

Now, we should compute the sign of f ′′(x) while x is between −π and π.Recall that, if y = sinx, then f ′′(x) = −4(4y2 + y − 2). So, in terms ofy, f ′′ is a parabola pointing down, with intercepts y = −1±

√33

8. Then f ′′

is positive when y is in the interval(−1−

√33

8, −1+

√33

8

), and f ′′ is negative

otherwise. From the graph of sine, we see that y is between −1−√

338

and−1+

√33

8precisely on the intervals (−π,−π − b), (b, a), and (π − a, π).

Therefore, f(x) is concave up on the intervals (−π,−π − b), (b, a), and (π −a, π), and f(x) is concave down on the intervals (−π − b, b) and (a, π − a).So, the inflection points of f occur at x = −π−b, x = b, x = a, and x = π−a.

x

y

π−π π2

−π2 −π

6−5π6

ab π − a−π − b

6

−3

−2

980

Page 997: CLP-1 Differential Calculus

SOLUTIONS TO EXERCISES

To find the maximum and minimum values of f(x) on [0, π], we compare thevalues of f(x) at its critical points in this interval (only x = π

2) with the values of

f(x) at its endpoints x = 0, x = π.Since f(0) = f(π) = −2, the minimum value of f on [0, π] is −2, achieved atx = 0, π and the maximum value of f on [0, π] is 6, achieved at x =

π

2.

3.6.7.10. Solution. Let f(x) = 3

√x+ 1

x2.

• Asymptotes: Since limx→0

f(x) = ∞, f(x) has a vertical asymptote at x = 0

where the curve reaches steeply upward from both the left and the right.

limx→±∞

f(x) = 0, so y = 0 is a horizontal asymptote for x→ ±∞.

• Intercepts: f(−1) = 0.

• Intervals of increase and decrease:

f ′(x) =−(x+ 2)

3x5/3(x+ 1)2/3

There is a singular point at x = −1 and a critical point at x = −2, in additionto a discontinuity at x = 0. Note that (x+ 1)2/3 =

(3√x+ 1

)2, which is nevernegative. Note also that lim

x→−1f ′(x) = ∞, so f(x) has a vertical tangent line

at x = −1.

x (−∞,−2) −2 (−2,−1) −1 (−1, 0) 0 (0,∞)

f ′(x) negative 0 positive DNE positive DNE negativef(x) decreasing l. min increasing vertical increasing VA decreasing

This gives us enough information to sketch a skeleton of the curve.

x

y

−2 −1

• Concavity:

f ′′(x) =4x2 + 16x+ 10

9x8/3(x+ 1)5/3

981

Page 998: CLP-1 Differential Calculus

SOLUTIONS TO EXERCISES

We still have a discontinuity at x = 0, and f ′′(x) does not exist at x = −1.The second derivative is zero when 4x2 + 16x+ 10 = 0. Using the quadraticformula, we find this occurs when x = −2 ±

√1.5 ≈ −0.8,−3.2. Note

x8/3 = ( 3√x)

8 is never negative.

x(−∞,−2−

√1.5)−2−

√1.5 (−2−

√1.5,−1) −1

f ′′(x) negative 0 positive DNEf(x) concave down IP concave up IP

x (−1,−2 +√

1.5) −2 +√

1.5 (−2 +√

1.5, 0) (0,∞)

f ′′(x) negative 0 positive positivef(x) concave down IP concave up concave up

Now, we can refine our skeleton. The closed dot is the local minimum, andthe open dots are inflection points.

x

y

−1−2

−1

1

Exercises — Stage 33.6.7.11. *. Solution. The parts of the question are just scaffolding to lead youthrough sketching the curve. Their answers are given explicitly, in an organizedmanner, in the “answers” section. In this solution, they are scattered throughout.

• Asymptotes:

Since the function has a derivative at every real number, the function iscontinuous for every real number, so it has no vertical asymptotes. In theproblem statement, you are told lim

x→∞f(x) = 0, so y = 0 is a horizontal

asymptote as x goes to infinity. It remains to evaluate limx→−∞

f(x). Let’s

982

Page 999: CLP-1 Differential Calculus

SOLUTIONS TO EXERCISES

consider the limit of f ′(x) instead. Recall K is a positive constant.

limx→−∞

e−x = limx→∞

ex =∞

limx→−∞

K(2x− x2) = −∞

So,

limx→−∞

K(2x− x2)e−x = −∞

That is, as x becomes a hugely negative number, f ′(x) also becomes ahugely negative number. As we move left along the x-axis, f(x) is decreas-ing with a steeper and steeper slope, as in the sketch below. That meanslim

x→−∞f(x) =∞.

x

y

Sketch: various tangent lines to f(x),their slopes getting more strongly negative

as x gets more strongly negative.

• Intervals of increase and decrease:

We are given f ′(x) (although we don’t know f(x)):

f ′(x) = Kx(2− x)e−x

The critical points of f(x) are x = 0 and x = 2, and there are no singularpoints. Recall e−x is always positive, and K is a positive constant.

983

Page 1000: CLP-1 Differential Calculus

SOLUTIONS TO EXERCISES

x (−∞, 0) 0 (0, 2) 2 (2,∞)

f ′(x) negative 0 positive 0 negativef(x) decreasing local min increasing local max decreasing

So, f(0) = 0 is a local minimum, and f(2) = 2 is a local maximum.

Looking ahead to part 3.6.7.11.d, we have a skeleton of the curve.

x

y

decreasing increasingdecreasing

2

• Concavity:

Since we’re given f ′(x), we can find f ′′(x).

f ′′(x) = K(2− 2x− 2x+ x2)e−x

= K(2− 4x+ x2)e−x

= K(x− 2−

√2)(x− 2 +

√2)e−x

where the last line can be found using the quadratic equation. So, f ′′(x) = 0for x = 2±

√2, and f ′′(x) exists everywhere.

x (−∞, 2−√

2) 2−√

2 (2−√

2, 2 +√

2) 2 +√

2 (2 +√

2,∞)

f ′(x) positive 0 negative 0 positivef(x) concave up IP concave down IP concave up

Now, we can add concavity to our sketch.

984

Page 1001: CLP-1 Differential Calculus

SOLUTIONS TO EXERCISES

x

y

22−√

2 2 +√

2

decr increasing decreasing

cc up concave down concave up

3.6.7.12. *. Solution. 3.6.7.12.a You should be familiar with the graph of y = ex.You can construct the graph of y = e−x just by reflecting the graph of y = ex

across the y–axis. To see why this is the case, imagine swapping each value of xits negative: for example, swapping the point at x = −1 with the point at x = 1,etc. Alternatively, you can graph y = f(x) = e−x, x ≥ 0, using the methods ofthis section: at x = 0, y = f(0) = 1; as x increases, y = f(x) = e−x decreases, withno local extrema; and as x→ +∞, y = f(x)→ 0.There are no inflection points or extrema, except the endpoint (0, 1).

985

Page 1002: CLP-1 Differential Calculus

SOLUTIONS TO EXERCISES

x

y

y = f(x)

1

1

3.6.7.12.bRecall that, to graph the inverse of a function, we reflect the original functionacross the line y = x.You should be familiar with the graph of y = ex. You can construct the graph ofy = e−x just by reflecting the graph of y = ex across the y–axis. To see why this isthe case, imagine swapping each value of x its negative: for example, swappingthe point at x = −1 with the point at x = 1, etc.Alternatively, you can graph y = f(x) = e−x, x ≥ 0, using the methods of thissection: at x = 0, y = f(0) = 1; as x increases, y = f(x) = e−x decreases, with nolocal extrema; and as x→ +∞, y = f(x)→ 0.To see why this is true, consider the following. Since g(x) is the inverse of f(x),then, for any pair of numbers x and y, we have

f(x) = y if and only g(y) = x

That is, g is the function that swaps the input and output of f . Now the point(x, y)lies on the graph of f if and only if y = f(x). Similarly, the point (X, Y ) lieson the graph of g if and only if Y = g(X). Setting Y = x and X = y we see thatthe point (y, x) lies on the graph of g if and only if x = g(y), which in turn is thecase if and only if y = f(x). So

(y, x) is on the graph of g if and only if (x, y) is on the graph of f .

To get from the point(x, y) to the point (y, x) we have to exchange x ↔ y, whichwe can do by reflecting over the line y = x. Thus we can construct the graph of gby reflecting the curve y = f(x) over the line y = x.

986

Page 1003: CLP-1 Differential Calculus

SOLUTIONS TO EXERCISES

x

y

y = f(x)

y = x

y = g(x)

1

1

3.6.7.12.c The domain of g is the range of f , which is (0, 1]. The range of g is thedomain of f , which is [0,∞).3.6.7.12.d Since g and f are inverses,

g(f(x)) = x

Using the chain rule,

g′(f(x)) · f ′(x) = 1

Since f ′(x) = −e−x = −f(x):

g′(f(x)) · f(x) = −1

We plug in f(x) = 12.

g′(

1

2

)· 1

2= −1

g′(

1

2

)= −2

3.6.7.13. *. Solution. (a) First, we differentiate.

f(x) = x5 − x f ′(x) = 5x4 − 1 f ′′(x) = 20x3

The function and its first derivative tells us the following:

• limx→∞

f(x) =∞, limx→−∞

f(x) = −∞

987

Page 1004: CLP-1 Differential Calculus

SOLUTIONS TO EXERCISES

• f ′(x) > 0 (i.e. f is increasing) for |x| > 14√

5

• f ′(x) = 0 (i.e. f has critical points) for x = ± 14√

5≈ ±0.67

• f ′(x) < 0 (i.e. f is decreasing) for |x| < 14√

5

• f

(± 1

4√

5

)= ∓ 4

5 4√

5≈ ∓0.53

This gives us a first idea of the shape of the graph.

x

y

−14√5

14√5

4

5 4√5

−4

5 4√5

We refine this skeleton using information from the second derivative.

• f ′′(x) > 0 (i.e. f is concave up) for x > 0,

• f ′′(x) = 0 (i.e. f has an inflection point) for x = 0, and

• f ′′(x) < 0 (i.e. f is concave down) for x < 0

Thus

• f has no asymptotes

• f has a local maximum at x = − 14√

5and a local minimum at x =

14√

5

• f has an inflection point at x = 0

• f is concave down for x < 0 and concave up for x > 0

988

Page 1005: CLP-1 Differential Calculus

SOLUTIONS TO EXERCISES

x

y

−14√5

14√5

4

5 4√5

−4

5 4√5

(b) The function x5−x+k has a root at x = x0 if and only if x5−x = −k at x = x0.So the number of distinct real roots of x5−x+ k is the number of times the curvey = x5−x crosses the horizontal line y = −k. The local maximum of x5−x (when

x = − 14√

5) is

4

5 4√

5, and the local minimum of x5−x (when x =

14√

5) is − 4

5 4√

5. So,

looking at the graph of x5−x above, we see that the number of distinct real rootsof x5 − x+ k is

• 1 when |k| > 4

5 4√

5

• 2 when |k| = 4

5 4√

5

• 3 when |k| < 4

5 4√

5

3.6.7.14. *. Solution. (a) You might not be familiar with hyperbolic sine andcosine, but you don’t need to be. We can graph them using the same methods asthe other curves in this section. The derivatives are given to us:

d

dxsinhx = coshx =

ex + e−x

2d

dxcoshx = sinhx =

ex − e−x2(

d

dx

)2

sinhx = sinhx =ex − e−x

2

989

Page 1006: CLP-1 Differential Calculus

SOLUTIONS TO EXERCISES(d

dx

)2

coshx = coshx =ex + e−x

2

Observe that:

• sinh(x) has a derivative that is always positive, so sinh(x) is always increas-ing. The second derivative of sinh(x) is negative to the left of x = 0 andpositive to the right of x = 0, so sinh(x) is concave down to the left of they-axis and concave up to its right, with an inflection point at x = 0.

• cosh(x) has a derivative that is positive when x > 0 and negative whenx < 0. The second derivative of cosh(x) is always positive, so it is alwaysconcave up.

• cosh(0) = 1 and sinh(0) = 0.

• limx→∞

sinhx = limx→∞

coshx = limx→∞

ex

2=∞, since lim

x→∞e−x = 0

limx→−∞

sinhx = limx→−∞

(ex

2− e−x

2

)= lim

x→∞

(e−x

2− ex

2

)= −∞

and

limx→−∞

coshx = limx→−∞

(ex

2+e−x

2

)= lim

x→∞

(e−x

2+ex

2

)=∞

• cosh(x) is even, since

cosh(−x) =e−x + e−(−x)

2=e−x + ex

2= cosh(x)

and sinh(x) is odd, since

sinh(−x) =e−x − e−(−x)

2=e−x − ex

2=− (ex − e−x)

2= − sinh(x)

x

yy = sinhx

x

yy = coshx

1

(b)

990

Page 1007: CLP-1 Differential Calculus

SOLUTIONS TO EXERCISES

• As y runs over (−∞,∞) the function sinh(y) takes every real value exactlyonce. So, for each x ∈ (−∞,∞), define sinh−1(x) to be the unique solutionof sinh(y) = x.

• As y runs over [0,∞) the function cosh(y) takes every real value in [1,∞)exactly once. In particular, the smallest value of cosh(y) is cosh(0) = 1. So,for each x ∈ [1,∞), define cosh−1(x) to be the unique y ∈ [0,∞) that obeyscosh(y) = x.

To graph the inverse of a (one-to-one) function, we reflect the original functionover the line y = x. Using this method to graph y = sinh−1(x) is straightforward.To graph y = cosh(x), we need to be careful of the domains: we are restrictingcosh(x) to values of x in [0,∞). The graphs are

x

y

y = sinhx

y = sinh−1 x

x

y

y = sinh−1 x

x

yy = coshx

y = cosh−1 x

x

y

y = cosh−1 x

(c) Let y(x) = cosh−1(x). Then, using the definition of cosh−1,

cosh y(x) = x

We differentiate with respect to x using the chain rule.

d

dxcosh y(x) =

d

dxx

y′(x) sinh y(x) = 1

We solve for y′(x).

y′(x) =1

sinh y(x)

991

Page 1008: CLP-1 Differential Calculus

SOLUTIONS TO EXERCISES

We want to have our answer in terms of x, not y. We know that cosh y = x, so ifwe can convert hyperbolic sine into hyperbolic cosine, we can get rid of y. Ourtool for this is the identity, given in the question statement, cosh2(x)− sinh2(x) =1. This tells us sinh2(y) = 1− cosh2(y). Now we have to decide whether sinh(y) isthe positive or negative square root of 1− cosh2(y) in our context. Looking at thegraph of y(x) = cosh−1(x), we see y′(x) > 0. So we use the positive square root:

y′(x) =1√

cosh2 y(x)− 1=

1√x2 − 1

Remark:d

dxarccos(x) =

−1√1− x2

, so again the hyperbolic trigonometric func-

tion has properties similar to (but not exactly the same as) its trigonometric coun-terpart.

3.7 · L’Hôpital’s Rule and Indeterminate Forms3.7.4 · Exercises

Exercises — Stage 13.7.4.1. Solution. There are many possible answers. Consider these: f(x) = 5x,

g(x) = 2x. Then limx→∞

f(x) = limx→∞

g(x) = ∞, and limx→∞

f(x)

g(x)= lim

x→∞5x

2x= lim

x→∞5

2=

5

2= 2.5.

3.7.4.2. Solution. There are many possible answers. Consider these: f(x) = x,

g(x) = x2. Then limx→∞

f(x) = limx→∞

g(x) = ∞, and limx→∞

f(x)

g(x)= lim

x→∞x

x2= lim

x→∞1

x=

0.

3.7.4.3. Solution. From Example 3.7.20, we know that limx→0

(1 + x)ax = ea, so

limx→0

(1 + x)log 5x = elog 5 = 5. However, this is the limit as x goes to 0, which is

not what we were asked. So, we modify the functions by replacing x with 1x. If

x→ 0+, then 1x→∞.

Taking f(x) = 1 + 1x

and g(x) = x log 5, we see:

• 3.7.4.3.i limx→∞

f(x) = limx→∞

(1 +

1

x

)= 1

• 3.7.4.3.ii limx→∞

g(x) = limx→∞

x log 5 =∞

• 3.7.4.3.iii Let us name1

x= X . Then as x→∞, X → 0+, so:

limx→∞

[f(x)]g(x) = limx→∞

[1 +

1

x

]x log 5

= limx→∞

[1 +

1

x

] log 51x

992

Page 1009: CLP-1 Differential Calculus

SOLUTIONS TO EXERCISES

= limX→0+

[1 +X]log 5X = elog 5 = 5,

where in the penultimate step, we used the result of Example 3.7.20.

Exercises — Stage 23.7.4.4. *. Solution. If we plug in x = 1 to the numerator and the denominator,we find they are both zero. So, we have an indeterminate form appropriate forL’Hôpital’s Rule.

limx→1

x3 − ex−1

sin(πx)︸ ︷︷ ︸num→0den→0

= limx→1

3x2 − ex−1

π cos(πx)= − 2

π

3.7.4.5. *. Solution. Be careful– this is not an indeterminate form!As x → 0+, the numerator log x → −∞. That is, the numerator is becomingan increasingly huge, negative number. As x → 0+, the denominator x → 0+,which only serves to make the total fraction even larger, and still negative. So,

limx→0+

log x

x= −∞.

Remark: if we had tried to use l’Hôpital’s Rule here, we would have come upwith the wrong answer. If we differentiate the numerator and the denomina-

tor, the fraction becomes1x

1= 1

x, and lim

x→0+

1

x= ∞. The reason we cannot apply

l’Hôpital’s Rule is that we do not have an indeterminate form, like both numera-tor and denominator going to infinity, or both numerator and denominator goingto zero.

3.7.4.6. *. Solution. We rearrange the expression to a more natural form:

limx→∞

(log x)2e−x = limx→∞

(log x)2

ex︸ ︷︷ ︸num→∞den→∞

Both the numerator and denominator go to infinity as x goes to infinity. So, wecan apply l’Hôpital’s Rule. In fact, we end up applying it twice.

limx→∞

(log x)2e−x = limx→∞

2 log x

xex︸ ︷︷ ︸num→∞den→∞

= limx→∞

2/x

xex + ex

The numerator gets smaller and smaller while the denominator gets larger andlarger, so:

limx→∞

(log x)2e−x = 0

993

Page 1010: CLP-1 Differential Calculus

SOLUTIONS TO EXERCISES

3.7.4.7. *. Solution.

limx→∞

x2e−x = limx→∞

x2

ex︸︷︷︸num→∞den→∞

= limx→∞

2x

ex︸︷︷︸num→∞den→∞

= limx→∞

2

ex︸︷︷︸num→∞den→∞

= 0

3.7.4.8. *. Solution.

limx→0

x− x cosx

x− sinx︸ ︷︷ ︸num→0den→0

= limx→0

1− cosx+ x sinx

1− cosx︸ ︷︷ ︸num→0den→0

= limx→0

sinx+ sinx+ x cosx

sinx︸ ︷︷ ︸num→0den→0

= limx→0

2 cosx+ cosx− x sinx

cosx= 3

3.7.4.9. Solution. If we plug in x = 0 to the numerator and denominator, bothare zero, so this is a candidate for l’Hôpital’s Rule. However, an easier way toevaluate the limit is to factor x2 from the numerator and denominator, and cancel.

limx→0

√x6 + 4x4

x2 cosx= lim

x→0

√x4√x2 + 4

x2 cosx

= limx→0

x2√x2 + 4

x2 cosx

= limx→0

√x2 + 4

cosx

=

√02 + 4

cos(0)= 2

3.7.4.10. *. Solution.

limx→∞

(log x)2

x︸ ︷︷ ︸num→∞den→∞

= limx→∞

2(log x) 1x

1= 2 lim

x→∞log x

x︸ ︷︷ ︸num→∞den→∞

= 2 limx→∞

1x

1= 0

3.7.4.11. *. Solution.

limx→0

1− cosx

sin2 x︸ ︷︷ ︸num→0den→0

= limx→0

sinx

2 sinx cosx= lim

x→0

1

2 cosx=

1

2

3.7.4.12. Solution. If we plug in x = 0, the numerator is zero, and the denomi-

nator is sec 0 =1

cos 0=

1

1= 1. So the limit is

0

1= 0.

Be careful: you cannot use l’Hôpital’s Rule here, because the fraction does notgive an indeterminate form. If you try to differentiate the numerator and the

994

Page 1011: CLP-1 Differential Calculus

SOLUTIONS TO EXERCISES

denominator, you get an expression whose limit does not exist:

limx→0

1

secx tanx= lim

x→0cosx · cosx

sinx= DNE.

3.7.4.13. Solution. If we plug x = 0 into the denominator, we get 1. However,the numerator is an indeterminate form: tan 0 = 0, while lim

x→0+cscx = ∞ and

limx→0−

cscx = −∞. If we use cscx = 1sinx

, our expression becomes

limx→0

tanx · (x2 + 5)

sinx · ex

Since plugging in x = 0 makes both the numerator and the denominator equal tozero, this is a candidate for l’Hôspital’s Rule. However, a much easier way is tosimplify the trig first.

limx→0

tanx · (x2 + 5)

sinx · ex = limx→0

sinx · (x2 + 5)

cosx · sinx · ex

= limx→0

x2 + 5

cosx · ex

=02 + 5

cos(0) · e0= 5

3.7.4.14. Solution. limx→∞

√2x2 + 1−

√x2 + x has the indeterminate form∞−∞.

To get a better idea of what’s going on, let’s rationalize.

limx→∞

√2x2 + 1−

√x2 + x

= limx→∞

(√2x2 + 1−

√x2 + x

)(√2x2 + 1 +√x2 + x√

2x2 + 1 +√x2 + x

)

= limx→∞

(2x2 + 1)− (x2 + x)√2x2 + 1 +

√x2 + x

= limx→∞

x2 − x+ 1√2x2 + 1 +

√x2 + x

Here, we have the indeterminate form ∞∞ , so l’Hôpital’s Rule applies. However,

if we try to use it here, we quickly get a huge mess. Instead, remember how wedealt with these kinds of limits in the past: factor out the highest power of thedenominator, which is x.

= limx→∞

x(x− 1 + 1

x

)√x2(2 + 1

x2) +

√x2(1 + 1

x)

= limx→∞

x(x− 1 + 1

x

)x(√

2 + 1x2

+√

1 + 1x

)

995

Page 1012: CLP-1 Differential Calculus

SOLUTIONS TO EXERCISES

= limx→∞

x− 1 + 1x√

2 + 1x2

+√

1 + 1x︸ ︷︷ ︸

num→∞den→

√2+1

=∞

3.7.4.15. *. Solution. If we plug in x = 0, both numerator and denominatorbecome zero. So, we have exactly one of the indeterminate forms that l’Hôpital’sRule applies to.

limx→0

sin(x3 + 3x2)

sin2 x︸ ︷︷ ︸num→0den→0

= limx→0

(3x2 + 6x) cos(x3 + 3x2)

2 sinx cosx

If we plug in x = 0, still we find that both the numerator and the denominator goto zero. We could jump in with another iteration of l’Hôpital’s Rule. However,the derivatives would be a little messy, so we use limit laws and break up thefraction into the product of two fractions. If both limits exist:

limx→0

(3x2 + 6x) cos(x3 + 3x2)

2 sinx cosx

=

(limx→0

x2 + 2x

sinx

)·(

limx→0

3 cos(x3 + 3x2)

2 cosx

)We can evaluate the right-hand limit by simply plugging in x = 0:

limx→0

(3x2 + 6x) cos(x3 + 3x2)

2 sinx cosx=

3

2limx→0

x2 + 2x

sinx︸ ︷︷ ︸num→0den→0

=3

2limx→0

2x+ 2

cosx

=3

2

(2

1

)= 3

3.7.4.16. *. Solution.

limx→1

log(x3)

x2 − 1= lim

x→1

3 log(x)

x2 − 1︸ ︷︷ ︸num→0den→0

= limx→1

3/x

2x=

3

2

3.7.4.17. *. Solution.

• Solution 1.

limx→0

e−1/x2

x4= lim

x→0

1x4

e1/x2︸ ︷︷ ︸num→∞den→∞

= limx→0

−4x5

−2x3e1/x2

= limx→0

2x2

e1/x2︸ ︷︷ ︸num→∞den→∞

996

Page 1013: CLP-1 Differential Calculus

SOLUTIONS TO EXERCISES

= limx→0

−4x3

−2x3e1/x2

= limx→0

2

e1/x2= 0

since, as x→ 0, the exponent 1x2→∞ so that e1/x2 →∞ and e−1/x2 → 0.

• Solution 2.

limx→0

e−1/x2

x4= lim

t= 1x2→∞

e−t

t−2= lim

t→∞t2

et︸︷︷︸num→∞den→∞

= limt→∞

2t

et︸︷︷︸num→∞den→∞

= limt→∞

2

et= 0

3.7.4.18. *. Solution.

limx→0

xex

tan(3x)︸ ︷︷ ︸num→0den→0

= limx→0

ex + xex

3 sec2(3x)=

1

3

3.7.4.19. Solution. limx→0

sin2 x = 0, and limx→0

1

x2= ∞, so we have the form 0∞.

(Note that sin2 x is positive, so our root is defined.) This is not an indeterminateform: lim

x→0

x2√

sin2 x = 0.

3.7.4.20. Solution. limx→0

cosx = 1 and limx→0

1

x2= ∞, so lim

x→0(cosx)

1x2 has the inde-

terminate form 1∞. We want to use l’Hôpital, but we need to get our functioninto a fractional indeterminate form. So, we’ll use a logarithm.

y : = (cos x)1x2

log y = log(

(cosx)1x2

)=

1

x2log(cosx) =

log cosx

x2

limx→0

log y = limx→0

log cosx

x2︸ ︷︷ ︸num→0den→0

= limx→0

− sinxcosx

2x= lim

x→0

− tanx

2x︸ ︷︷ ︸num→0den→0

= limx→0

− sec2 x

2= lim

x→0

−1

2 cos2 x= −1

2

Therefore, limx→0

y = limx→0

elog y = e−1/2 =1√e

3.7.4.21. Solution.

• Solution 1

y : = ex log x = (ex)log x

limx→0+

y = limx→0+

(ex)log x

This has the form 1−∞ = 11∞

, and 1∞ is an indeterminate form. We want touse l’Hôpital, but we need to get a different indeterminate form. So, we’ll

997

Page 1014: CLP-1 Differential Calculus

SOLUTIONS TO EXERCISES

use logarithms.

limx→0+

log y = limx→0+

log((ex)log x

)= lim

x→0+log x log (ex)

= limx→0+

(log x) · x

This has the indeterminate form 0·∞, so we need one last adjustment beforewe can use l’Hôpital’s Rule.

limx→0+

(log x) · x = limx→0+

log x1x︸ ︷︷ ︸

num→−∞den→∞

= limx→0+

1x−1x2

= limx→0+

−x = 0

Now, we can figure out what happens to our original function, y:

limx→0+

y = limx→0+

elog y = e0 = 1

• Solution 2

y : = ex log x =(elog x

)x= xx

limx→0+

y = limx→0+

xx

We have the indeterminate form 00. We want to use l’Hôpital, but we needa different indeterminate form. So, we’ll use logarithms.

limx→0+

log y = limx→0+

log(xx) = limx→0+

x log x

Now we have the indeterminate form 0 ·∞, so we need one last adjustmentbefore we can use l’Hôpital’s Rule.

limx→0+

y = limx→0+

log x1x︸ ︷︷ ︸

num→0den→−∞

= limx→0+

1x−1x2

= limx→0+

−x = 0

Now, we can figure out what happens to our original function, y:

limx→0+

y = limx→0+

elog y = e0 = 1

3.7.4.22. Solution. First, note that the function exists near 0: x2 is positive, solog(x2) exists; near 0, log x2 is negative, so − log(x2) is positive, so [− log(x2)]

x

exists even when x is negative.Since lim

x→0− log(x2) = ∞ and lim

x→0x = 0, we have the indeterminate form∞0. We

need l’Hôpital, but we need to manipulate our function into an appropriate form.We do this using logarithms.

y : =[− log(x2)

]x998

Page 1015: CLP-1 Differential Calculus

SOLUTIONS TO EXERCISES

log y = log([− log(x2)

]x)= x︸︷︷︸→0

· log

− log(x2)︸ ︷︷ ︸→∞

︸ ︷︷ ︸

→∞

=log (− log(x2))

1x

limx→0

log y = limx→0

log (− log(x2))1x︸ ︷︷ ︸

num→∞den→±∞

= limx→0

− 2x

− log(x2)

−1x2

= limx→0

−2x

log(x2)︸ ︷︷ ︸num→0

den→−∞

= 0

Now, we’re ready to figure out our original limit.

limx→0

y = limx→0

elog y = e0 = 1

3.7.4.23. *. Solution. Both the numerator and denominator converge to 0 asx→ 0. So, by l’Hôpital,

limx→0

1 + cx− cosx

ex2 − 1︸ ︷︷ ︸num→0den→0

= limx→0

c+ sinx

2xex2

The new denominator still converges to 0 as x → 0. For the limit to exist, thesame must be true for the new numerator. This tells us that if c 6= 0, the limitdoes not exist. We should check whether the limit exists when c = 0. Usingl’Hôpital:

limx→0

sinx

2xex2︸ ︷︷ ︸num→0den→0

= limx→0

cosx

ex2(4x2 + 2)=

1

1(0 + 2)=

1

2.

So, the limit exists when c = 0.

3.7.4.24. *. Solution. The first thing we notice is, regardless of k, when we plugin x = 0 both numerator and denominator become zero. Let’s use this fact, andapply l’Hôpital’s Rule.

limx→0

ek sin(x2) − (1 + 2x2)

x4︸ ︷︷ ︸num→0den→0

= limx→0

2kx cos(x2)ek sin(x2) − 4x

4x3

= limx→0

2k cos(x2)ek sin(x2) − 4

4x2

When we plug in x = 0, the denominator becomes 0, and the numerator becomes2k − 4. So, we’ll need some cases, because the behaviour of the limit depends on

999

Page 1016: CLP-1 Differential Calculus

SOLUTIONS TO EXERCISES

k.For k = 2:

limx→0

2k cos(x2)ek sin(x2) − 4

4x2= lim

x→0

4 cos(x2)e2 sin(x2) − 4

4x2︸ ︷︷ ︸num→0den→0

= limx→0

−8x sin(x2)e2 sin(x2) + 16x cos2(x2)e2 sin(x2)

8x

= limx→0

[− sin(x2)e2 sin(x2) + 2 cos2(x2)e2 sin(x2)

]= 2

For k > 2, the numerator goes to 2k − 4, which is a positive constant, while thedenominator goes to 0 from the right, so:

limx→0

2k cos(x2)ek sin(x2) − 4

4x2=∞

For k < 2, the numerator goes to 2k − 4, which is a negative constant, while thedenominator goes to 0 from the right, so:

limx→0

2k cos(x2)ek sin(x2) − 4

4x2= −∞

Exercises — Stage 33.7.4.25. Solution.

• We want to find the limit as n goes to infinity of the percentage error,

limn→∞

100|S(n)− A(n)||S(n)| . Since A(n) is a nicer function than S(n), let’s sim-

plify: limn→∞

100|S(n)− A(n)||S(n)| = 100

∣∣∣∣1− limn→∞

A(n)

S(n)

∣∣∣∣.We figure out this limit the natural way:

100

∣∣∣∣1− limn→∞

A(n)

S(n)

∣∣∣∣ = 100

∣∣∣∣∣∣∣∣∣1− limn→∞

5n4

5n4 − 13n3 − 4n+ log(n)︸ ︷︷ ︸num→∞den→∞

∣∣∣∣∣∣∣∣∣= 100

∣∣∣∣1− limn→∞

20n3

20n3 − 39n2 − 4 + 1n

∣∣∣∣= 100

∣∣∣∣1− limn→∞

n3

n3· 20

20− 39n− 4

n3 + 1n4

∣∣∣∣= 100|1− 1| = 0

So, as n gets larger and larger, the relative error in the approximation getscloser and closer to 0.

1000

Page 1017: CLP-1 Differential Calculus

SOLUTIONS TO EXERCISES

• Now, let’s look at the absolute error.

limn→∞

|S(n)− A(n)| = limn→∞

| − 13n3 − 4n+ log n| =∞

So although the error gets small relative to the giant numbers we’re talkingabout, the absolute error grows without bound.

4 · Towards Integral Calculus4.1 · Introduction to Antiderivatives4.1.2 · Exercises

Exercises — Stage 14.1.2.1. Solution. An antiderivative of f ′(x) is a function whose derivative isf ′(x). Our original function f(x) has this property, so f(x) is an antiderivativeof f ′(x), but it’s not the most general. We can add a constant to f(x) withoutaffecting its derivative. The most general antiderivative of f ′(x) is f(x) + C,where C is any constant.

4.1.2.2. Solution. Notice f(x) is nonnegative for an interval covering the leftpart of the graph, and negative on the right part of the graph. That means itsantiderivative is increasing for the left interval, then decreasing for the right in-terval. This applies to A(x) and C(x), but not B(x).There are only three points where A(x) has a horizontal tangent line: at its globalmaximum and the endpoints of the interval shown. By contrast, C(x) has a hor-izontal tangent line in four places: at its global maximum, at its inflection point,and at the endpoints of the interval shown. Since f(x) = 0 four times (and theseline up with the horizontal portions of C(x)) we conclude C(x) is the antideriva-tive of f(x).

Exercises — Stage 24.1.2.3. Solution. For any constant n 6= −1, an antiderivative of xn is 1

n+1xn+1.

F ′(x) = 3x2 + 5x4 + 10x− 9

F (x) = 3

(1

3

)x3 + 5

(1

5

)x5 + 10

(1

2

)x2 − 9

(1

1

)x1 + C

= x3 + x5 + 5x2 − 9x+ C

Remark: we can always check by differentiating:

F ′(x) =d

dx

x3 + x5 + 5x2 − 9x+ C

= 3x2 + 5x4 + 10x− 9

= f(x)

so F (x) is indeed an antiderivative of f(x).

1001

Page 1018: CLP-1 Differential Calculus

SOLUTIONS TO EXERCISES

4.1.2.4. Solution.

F ′(x) =3

5x7 − 18x4 + x

F (x) =

(3

5

)(1

8

)x8 − 18

(1

5

)x5 +

1

2x2 + C

=3

40x8 − 18

5x5 +

1

2x2 + C

4.1.2.5. Solution. For any constant n 6= 1, an antiderivative of xn is 1n+1

xn+1.The constant n does not have to be an integer.

F ′(x) = 4 3√x− 9

2x2.7

= 4x13 − 9

2x−2.7

F (x) = 4

(1

13

+ 1

)x

(13

+1)−(

9

2

)(1

−2.7 + 1

)x(−2.7+1) + C

= 4

(3

4

)x

43 −

(9

2

)(10

−17

)x−1.7 + C

= 3x43 +

45

17x1.7+ C

4.1.2.6. Solution.

• Solution 1: We can re-write f(x) to make it a power of x.

F ′(x) =1

7x−

12

F (x) =

(1

7

)(1

−12

+ 1

)x

(−1

2+1

)+ C

=

(1

7

)(2)x

12 + C

=2

7

√x+ C

• Solution 2: We notice that1

7√x

looks a lot like1

2√x

, which is the derivative

of√x. So:

d

dx

√x

=1

2√x

d

dx

2

7

√x

=

(2

7

)1

2√x

= f(x)

So, an antiderivative of f(x) is2

7

√x. Then the most general antiderivative

is F (x) =2

7

√x+ C.

1002

Page 1019: CLP-1 Differential Calculus

SOLUTIONS TO EXERCISES

4.1.2.7. Solution. We recalld

dxex = ex. That is, ex is its own antiderivative. So,

a first guess for the antiderivative of f(x) might be itself.

d

dx

e5x+11

= 5e5x+11

This isn’t exactly right, so we modify it by multiplying by a constant.

d

dx

1

5e5x+11

= e5x+11

This tells us that1

5e5x+11 is an antiderivative of e5x+11. Therefore, the most general

antiderivative of e5x+11 is F (x) =1

5e5x+11 + C.

4.1.2.8. Solution. We know the derivatives of sine and cosine. We’ll work fromthere to build a function whose derivative is f(x). We’ll start by finding an an-tiderivative of 7 cos(13x).

d

dxsinx = cosx

d

dxsin(13x) = 13 cos(13x)

d

dx

1

13sin(13x)

=

13

13cos(13x) = cos(13x)

d

dx

7

13sin(13x)

= 7 cos(13x)

So, an antiderivative of 7 cos(13x) is7

13sin(13x).

d

dxcosx = − sinx

d

dx− cosx = sinx

d

dx− cos(5x) = 5 sin(5x)

d

dx

−3

5cos(5x)

=

(3

5

)5 sin(5x) = 3 sin(5x)

So, an antiderivative of 3 sin(5x) is −3

5cos(5x).

The most general antiderivative of 3 sin(5x) + 7 cos(13x) is

F (x) = −3

5cos(5x) +

7

13sin(13x) + C.

1003

Page 1020: CLP-1 Differential Calculus

SOLUTIONS TO EXERCISES

4.1.2.9. Solution. We know the derivative of tanx is sec2 x. Modifying thisslightly, we see (using the chain rule)

d

dxtan(x+ 1) = sec2(x+ 1) · d

dxx+ 1

= sec2(x+ 1)

So, tan(x + 1) is an antiderivative of sec2(x + 1). Therefore, the most generalantiderivative of sec2(x+ 1) is F (x) = tan(x+ 1) + C.

4.1.2.10. Solution. We note that f(x) looks similar to1

x.

d

dxlog |x| =

1

xd

dxlog |x+ 2| =

1

x+ 2

The most general antiderivative of f(x) is F (x) = log |x+ 2|+ C.

4.1.2.11. Solution. Our function f(x) bears some resemblance to the derivative

of arcsine,1√

1− x2:

f(x) =7√

3− 3x2=

7√3

(1√

1− x2

)d

dx

7√3

arcsin(x)

=

7√3

(1√

1− x2

)= f(x)

So, the most general antiderivative of f(x) is F (x) =7√3

arcsin(x) + C.

4.1.2.12. Solution. We notice that f(x) looks similar to the derivative of the

arctangent function,1

1 + x2.

f(x) =1

1 + 25x2=

1

1 + (5x)2

This gives us a first guess for our antiderivative: perhaps arctan(5x) will work.We test it by differentiating, making sure we don’t forget the chain rule.

d

dxarctan(5x) =

1

1 + (5x)2 · 5

We’re close to f(x), but we’ve multiplied by 5. That’s easy to take care of: we candivide our guess by 5.

d

dx

1

5arctan(5x)

=

1

5

(1

1 + (5x)2

)· 5

1004

Page 1021: CLP-1 Differential Calculus

SOLUTIONS TO EXERCISES

=1

1 + 25x2= f(x)

So, the most general antiderivative of f(x) is F (x) =1

5arctan(5x) + C.

4.1.2.13. Solution. First, let’s find the antiderivative of f ′(x). It’s a polynomial,so we can use the observation from the text that an antiderivative of xn, for anyconstant n 6= 1, is 1

n+1xn+1. Remember that the most general antiderivative will

have an added constant.

f(x) =3

3x3 − 9

2x2 + 4x+ C

= x3 − 9

2x2 + 4x+ C

Use the fact that f(1) = 10 to solve for C.

10 = 1− 9

2+ 4 + C

C =19

2

All together,

f(x) = x3 − 9

2x2 + 4x+

19

2.

4.1.2.14. Solution. First, let’s find the antiderivative of f ′(x). We know that oneantiderivative of cos(x) is sinx. We might guess that an antiderivative of cos(2x)is sin(2x). Check by differentiating:

d

dxsin(2x) = 2 cos(2x)

This is close to f ′(x), but we need to divide by 2.

d

dx

1

2sin(2x)

= cos(2x)

So, f(x) =1

2sin(2x) + C for some constant C. We can find C using the given

information f(π) = π.

π = f(π) =1

2sin(2π) + C

π = C

Therefore, f(x) =1

2sin(2x) + π.

1005

Page 1022: CLP-1 Differential Calculus

SOLUTIONS TO EXERCISES

4.1.2.15. Solution. Looking at the table in the notes, we see the antiderivative

of1

xis f(x) = log |x|+ C. The given information f(−1) = 0 lets us find C:

0 = f(−1) = log | − 1|+ C

0 = log(1) + C

0 = C

So, f(x) = log |x|.Remark: it is true that log x is an antiderivative of

1

x, since the derivative of log x

is1

x. However, log x is only defined for positive values of x. Since the given

information tells you that f(x) is defined when x = −1, you need to use a more

general antiderivative of1

x: log |x|+ C.

4.1.2.16. Solution. An antiderivative of 1 is x, and an antiderivative of1√

1− x2

is arcsin(x). So, f(x) = arcsin x+ x+ C. The given information lets us find C.

−π2

= f(1) = arcsin(1) + 1 + C

−π2

2+ 1 + C

C = −π − 1

So, f(x) = arcsin x+ x− π − 1.

4.1.2.17. Solution. If P (t) is the population at time t, then the information givenin the problem is P ′(t) = 100e2t. Antidifferentiating, we see P (t) = 50e2t + C,where C is some constant. We want to know for what value of t we get P (t) =P (0) + 300.

P (t) = P (0) + 300

50e2t + C = 50e0 + C + 300

50e2t = 350

e2t = 7

2t = log(7)

t =1

2log(7)

It takes 12

log 7 hours (about 58 minutes) for the initial colony to increase by 300individuals.

4.1.2.18. Solution. If A(t) is the amount of money in your account at time t,then the given information is

A′(t) = 1500et

50

1006

Page 1023: CLP-1 Differential Calculus

SOLUTIONS TO EXERCISES

Antidifferentiating,

A(t) = 75000et

50 + C

for some constant C.That is, at time t, the amount of money in your bank account is 75000e

t50 + C

dollars, for some constant C.

4.1.2.19. Solution. Let P (t) be the amount of power your house has used sincetime t = 0. If t is measured in hours, and P (t) in kWh (kilowatt-hours), thenP ′(t) is the rate at which your house is consuming power, in kW. So, the giveninformation is that

P ′(t) = 0.5 sin( π

24t)

+ 0.25

Antidifferentiating,

P (t) = −12

πcos( π

24t)

+ 0.25t+ C

Since P (0) is the amount of energy consumed after 0 hours, P (0) = 0, so

0 = P (0) = −12

π+ C

C =12

π

P (t) =12

π

[1− cos

( π24t)]

+ 0.25t

After 24 hours, your energy consumed is

P (24) =12

π[1− cos (π)] + 0.25(24)

=24

π+ 6 ≈ 13.6 kWh

Exercises — Stage 34.1.2.20. *. Solution. We differentiate f(x) and g(x) using the chain rule. Recalld

dxarcsin(x) =

1√1− x2

.

f ′(x) =2√

1−√x2· d

dx

√x

=2√

1− x ·1

2√x

=1√

x− x2

1007

Page 1024: CLP-1 Differential Calculus

SOLUTIONS TO EXERCISES

g′(x) =1√

1− (2x− 1)2· d

dx2x− 1

=2√

1− 4x2 + 4x− 1

=1√

x− x2

So, the derivative of f(x)− g(x) is 0, which implies that f(x) and g(x) differ by aconstant–perhaps a surprising result!Remark: we don’t need calculus to show that f(x) and g(x) only differ by a con-stant. Define θ = sin−1√x, so that sin θ =

√x and f(x) = 2θ , and then

sin[f(x)− π

2

]= − cos f(x) = − cos 2θ = −[1− 2 sin2 θ]

= 2 sin2 θ − 1 = 2x− 1

Our goal is to take the arcsine of the first and last expressions and conclude f(x)−π

2= arcsin(2x− 1) = g(x). However, before we can say arcsin

(sin[f(x)− π

2

])=

f(x) − π

2, we have to check that −π

2≤ f(x) − π

2≤ π

2. That is, we need to show

that 0 ≤ f(x) ≤ π.As 0 ≤ θ ≤ π

2(since sin−1 always takes values between −π

2and π

2and since√

x ≥ 0), we have that 0 ≤ f(x) = 2θ ≤ π. So,

f(x)− π

2= arcsin

(sin[f(x)− π

2

])= arcsin(2x+ 1) = g(x).

4.1.2.21. Solution. The derivative of sin(2x) (which occurs in the second term)is 2 cos(2x) (which occurs in the first term). Similarly, the derivative of cos(3x)(which occurs in the first term) is −3 sin(3x) (which occurs in the second term).So, f(x) seems to have come from the product rule.

d

dxsin(2x) cos(3x) = 2 cos(2x) cos(3x)− 3 sin(2x) sin(3x)

Then the antiderivative of f(x) is F (x) = sin(2x) cos(3x) + C.

4.1.2.22. Solution. The function f(x) looks like perhaps it came from the quo-tient rule. Recall

d

dx

u(x)

v(x)

=v(x)u′(x)− u(x)v′(x)

(v(x))2

Then, since the denominator of f(x) is (x2 + 1)2, we might guess v(x) = (x2 + 1).That leaves u(x) = ex.

d

dx

ex

x2 + 1

=

(x2 + 1)ex − ex(2x)

(x2 + 1)2= f(x)

1008

Page 1025: CLP-1 Differential Calculus

SOLUTIONS TO EXERCISES

So, the antiderivative of f(x) is F (x) =ex

x2 + 1+ C.

4.1.2.23. Solution. We know that ex is its own antiderivative. The derivative ofeg(x), for some function g(x), is g′(x)eg(x). Since 3x2 is the derivative of x3, f(x) fitsthis pattern. We guess the antiderivative of f(x) is F (x) = ex

3+ C.

We check by differentiating.

d

dx

ex

3

+ C

= 3x2ex3

= f(x)

Indeed, the antiderivative of f(x) is F (x) = ex3

+ C.

4.1.2.24. Solution. We know the antiderivative of sinx is − cosx. Since sin(x2)appears in our function, let’s investigate the derivative of − cos(x2).

d

dx

− cos

(x2)

= sin(x2)· 2x

This differs from f(x) only by a constant multiple.

d

dx

−5

2cos(x2)

=5

2sin(x2)· 2x = 5x sin(x2) = f(x)

So, the antiderivative of f(x) is F (x) = −5

2cos (x2) + C

Remark: as in Question 4.1.2.23, our function f(x) involved some function g(x)as well as g′(x). This pattern is the basis of an important method of antidifferen-tiation, called the Substitution Rule.

4.1.2.25. Solution. For any x in the domain of log(x), elog x = x. So, f(x) = x for

every x in its domain. Then its antiderivative is F (x) =1

2x2 + C.

4.1.2.26. Solution. As in Question 4.1.2.11, we notice that our function is similar

to1√

1− x2, but in this case it doesn’t factor quite as nicely.

f(x) =7√

3− x2=

7√3(1− x2

3)

=7√3

1√1− x2

3

1009

Page 1026: CLP-1 Differential Calculus

SOLUTIONS TO EXERCISES

What we really want under that square root, instead ofx2

3, is simply x2. We can

get close: we can get something squared.

f(x) =7√3

1√1−

(x√3

)2

Now, the thing that’s squared isn’t x, it’s

x√3

. This gives us a first guess for an an-

tiderivative: perhaps F (x) =7√3

arcsin

(x√3

)will work. Let’s try it! Remember

to use the chain rule when you differentiate.

d

dx

7√3

arcsin

(x√3

)=

7√3

1√1−

(x√3

)2

· 1√3

We’re very close! We’re only off by a constant, and those are easy to fix. We’redividing by

√3 when we differentiate, so let’s multiply our function by

√3.

d

dx

7 arcsin

(x√3

)= 7

1√1−

(x√3

)2

· 1√3

=7

√3

√1−

(x√3

)2

=7√

3− x2= f(x)

Therefore, the most general antiderivative of f(x) is F (x) = 7 arcsin

(x√3

)+ C.

4.1.2.27. Solution. Following Example 4.1.7, let V (H) be the volume of the solidformed by rotating the segment of the parabola from x = −H to x = H . Our

plan is to evaluate V ′(H) = limh→H

V (H)− V (h)

H − h and then antidifferentiate V ′(H)

to find V (H). Since we don’t know V (H) − V (h) (yet) we first find upper andlower bounds on it when h < H .For a constant h < H , V (H) − V (h) is the volume of the solid inside the largerobject (with length 2H) and outside the smaller object (with length 2h, shownbelow in blue). There are two regions inside the larger object and outside thesmaller.

1010

Page 1027: CLP-1 Differential Calculus

SOLUTIONS TO EXERCISES

y

x

H

h

One of the regions inside the larger object and outside the smaller (blue) objectlooks like this:

y = x2 + 1

H − h

x

h H

Remember we formed this solid by rotating the curve y = x2 +1 about the x-axis.So, the cross-section of this solid is a circle, and the radius of the circle when weare x units from the origin is x2+1. So, the largest radius of the little shape shownabove, which occurs at the right end, is H2 +1, and the smallest radius (at the leftend) is h2 + 1.The volume of the shape shown above is less than a cylinder of radiusH2 +1 andheight H − h, and it is more than the volume of a cylinder of radius h2 + 1 andheight H−h. So, the volume of the shape shown above is between (H−h)π(h2 +

1011

Page 1028: CLP-1 Differential Calculus

SOLUTIONS TO EXERCISES

1)2 and (H − h)π(H2 + 1)2 cubic units.Recall that the volume inside the object of length 2H and outside the object oflength 2h consists of two copies of the shape shown above. Therefore:

2(H − h)π(h2 + 1)2 < V (H)− V (h) < 2(H − h)π(H2 + 1)2

Now that we have upper and lower bounds for V (H)−V (h), we can find V ′(H).

2(H − h)π(h2 + 1)2

H − h <V (H)− V (h)

H − h <2(H − h)π(H2 + 1)2

H − h2π(h2 + 1)2<

V (H)− V (h)

H − h <2π(H2 + 1)2

limh→H

2π(h2 + 1)2≤ limh→H

V (H)− V (h)

H − h ≤ limh→H

2π(H2 + 1)2

Since the limits on both ends are simply 2π(H2 + 1)2, by the Squeeze Theorem,

V ′(H) = limh→H

V (H)− V (h)

H − h = 2π(H2 + 1)2

Now that we know V ′(H), we antidifferentiate to find V (H).

V ′(H) = 2π(H4 + 2H2 + 1)

V (H) = 2π

(1

5H5 +

2

3H3 +H

)+ C

When H = 0, there is no solid, so V (0) = 0. Therefore,

V (H) = 2π

(1

5H5 +

2

3H3 +H

).

1012